You are on page 1of 486

The Most Accepted

Crash Course Programme

in

Physics

ARIHANT PRAKASHAN (Series), MEERUT


ARIHANT PRAKASHAN (Series), MEERUT
All Rights Reserved

© Publisher
No part of this publication may be re-produced, stored in a retrieval system or by any
means, electronic, mechanical, photocopying, recording, scanning, web or otherwise
without the written permission of the publisher. Arihant has obtained all the information
in this book from the sources believed to be reliable and true. However, Arihant or its
editors or authors or illustrators don’t take any responsibility for the absolute accuracy of
any information published and the damage or loss suffered thereupon.
All disputes subject to Meerut (UP) jurisdiction only.

Administrative & Production Offices


Regd. Office
‘Ramchhaya’ 4577/15, Agarwal Road, Darya Ganj, New Delhi -110002
Tele: 011- 47630600, 43518550

Head Office
Kalindi, TP Nagar, Meerut (UP) - 250002
Tel: 0121-7156203, 7156204

Sales & Support Offices


Agra, Ahmedabad, Bengaluru, Bareilly, Chennai, Delhi, Guwahati,
Hyderabad, Jaipur, Jhansi, Kolkata, Lucknow, Nagpur & Pune.

ISBN 978-93-25795-51-8
PO No : TXT-XX-XXXXXXX-X-XX
Published by Arihant Publications (India) Ltd.
For further information about the books published by Arihant, log on to
www.arihantbooks.com or e-mail at info@arihantbooks.com
Follow us on
PREFACE
Every year lakhs of students dreaming to get that venerable prefix Dr. before their names go through
intense rigors and grinds of pre-medical tests. In order to be eligible for admission to MBBS/BDS courses
for a particular academic year, it shall be necessary for a candidate to obtain minimum marks in the
concerned Pre-medical test held for the same academic year besides obtaining minimum prescribed marks
at the qualifying examination. The number of students appearing for medical entrance exams has
increased manifold over the years.

The pattern of the questions being asked in these exams has changed dramatically and the difficultly level
has also increased considerably. One needs to be highly motivated to get success in these entrance
examinations. You cannot expect to attain the level of devotion that such entrances require unless you’re
motivated from the inside. To succeed in these exams, and to actualise your dream, you are required to
prepare strategically and study in a focused manner. Clearity of concepts and expertise in problem solving
techniques are essential.

The book 40 Days Physics for NEET serves the above cited purpose in perfect manner. At whatever level of
preparation you are before the exam, this book gives you an accelerated way to master the whole NEET
Physics syllabus. It has been conceived keeping in mind the latest trend of questions and the level of
different types of students. The whole syllabus of Physics has been divided into day-wise learning modules
with clear groundings into concepts and sufficient practice with solved and unsolved questions on that
day. After every few days you get a Unit Test based upon the topics covered before that day. On last three
days you get three full-length Mock Tests, making you ready to face the test. It is not necessary that you
start working with this book in 40 days just before the exam. This will only keep you in good frame of mind
and relaxed, vital for success at this level

Salient Features
Ÿ Concepts discussed clearly and directly without being superfluous. Only the required material for
NEET being described comprehensively to keep the students focussed.
Ÿ Exercises for each day give you the collection of only the Best Questions of the concept, giving
you the perfect practice in less time.
Ÿ Each day has two Exercises; Foundation Questions Exercise having Topically Arranged Questions
& Progressive Question Exercise having higher Difficulty Level Questions.
Ÿ All types of Objective Questions included in Daily Exercises (Single Option Correct, Assertion &
Reason, etc).
Ÿ Along with Daywise Exercises, there above also the Unit Tests & Full Length Mock Tests.
Ÿ At the end of book, there are NEET Solved Paper 2021.
We are sure that 40 Days Physics for NEET will give you a fast way to prepare for Physics without any other
support or guidance.

Publisher
CONTENTS
Preparing NEET 2022 Physics in 40 Days !
Day 1. Physical World and Measurement 1-9
Day 2. Kinematics 10-21
Day 3. Scalar and Vector 22-30
Day 4. Laws of Motion 31-44
Day 5. Circular Motion 45-52
Day 6. Work, Energy and Power 53-65
Day 7. System of Particle and Rigid Body 66-74
Day 8. Rotational Motion 75-83
Day 9. Gravitation 84-96
Day 10. Unit Test 1 97-103

Day 11. Properties of Matter 104-115


Day 12. Transfer of Heat 116-128
Day 13. Behaviour of Perfect Gas and Kinetic Theory 129-137
Day 14. Thermodynamics 138-150
Day 15. Unit Test 2 151-155

Day 16. Oscillations 156-170


Day 17. Waves 171-185
Day 18. Unit Test 3 186-192

Day 19. Electrostatics 193-214


Day 20. Current Electricity 215-232
Day 21. Unit Test 4 233-241
Day 22. Magnetic Effects of Current 242-254
Day 23. Magnetism 255-265
Day 24. Electromagnetic Induction 266-275
Day 25. Alternating Current 276-286
Day 26. Electromagnetic Waves 287-293
Day 27. Unit Test 5 294-298

Day 28. Ray Optics 299-314


Day 29. Wave Optics 315-324
Day 30. Unit Test 6 325-330

Day 31. Matter Waves 331-337


Day 32. Photoelectric Effect 338-346
Day 33. Atoms and Nuclei 347-356
Day 34. Radioactivity 357-365
Day 35. Unit Test 7 366-371

Day 36. Electronic Devices 372-386


Day 37. Unit Test 8 387-393

Day 38. Mock Test 1 394-402


Day 39. Mock Test 2 403-411
Day 40. Mock Test 3 412-419

NEET Solved Papers 2019 (National & Odisha) 1-26


NEET Solved Paper 2020 1-8
NEET Solved Paper 2021 1-16
SYLLABUS
CLASS 11th
UNIT I Physical World and Measurement
Physics : Scope and excitement, nature of physical laws; Physics, technology and society.
Need for measurement: Units of measurement, systems of units, SI units, fundamental and derived
units. Length, mass and time measurements, accuracy and precision of measuring instruments, errors
in measurement, significant figures.
Dimensions of physical quantities, dimensional analysis and its applications.

UNIT II Kinematics
Frame of reference, Motion in a straight line, Position-time graph, speed and velocity. Uniform and
non-uniform motion, average speed and instantaneous velocity. Uniformly accelerated motion,
velocity-time and position-time graphs, for uniformly accelerated motion (graphical treatment).
Elementary concepts of differentiation and integration for describing motion. Scalar and vector
quantities: Position and displacement vectors, general vectors, general vectors and notation, equality
of vectors, multiplication of vectors by a real number, addition and subtraction of vectors. Relative
velocity.
Unit vectors. Resolution of a vector in a plane-rectangular components.
Scalar and Vector products of Vectors. Motion in a plane. Cases of uniform velocity and uniform
acceleration- projectile motion. Uniform circular motion.

UNIT III Laws of Motion


Intuitive concept of force. Inertia, Newton's first law of motion; momentum and Newton's second law
of motion, impulse, Newton's third law of motion. Law of conservation of linear momentum and its
applications.
Equilibrium of concurrent forces. Static and Kinetic friction, laws of friction, rolling friction, lubrication.
Dynamics of uniform circular motion. Centripetal force, examples of circular motion (vehicle on level
circular road, vehicle on banked road).

UNIT IV Work, Energy and Power


Work done by a constant force and variable force, kinetic energy, work-energy theorem, power. Notion
of potential energy, potential energy of a spring, conservative forces, conservation of mechanical
energy (kinetic and potential energies), non-conservative forces, motion in a vertical circle, elastic and
inelastic collisions in one and two dimensions.

UNIT V Motion of System of Particles and Rigid Body


Centre of mass of a two-particle system, momentum conservation and centre of mass motion. Centre
of mass of a rigid body, centre of mass of uniform rod.
Moment of a force, torque, angular momentum, conservation of angular momentum with some
examples.
Equilibrium of rigid bodies, rigid body rotation and equation of rotational motion, comparison of linear
and rotational motions, moment of inertia, radius of gyration. Values of MI for simple geometrical
objects (no derivation). Statement of parallel and perpendicular axes theorems and their applications.

UNIT VI Gravitation
Kepler's laws of planetary motion. The universal law of gravitation. Acceleration due to gravity and its
variation with altitude and depth.
Gravitational potential energy, gravitational potential. Escape velocity, orbital velocity of a satellite.
Geostationary satellites.

UNIT VII Properties of Bulk Matter


Elastic behavior, Stress-strain relationship. Hooke's law, Young's modulus, bulk modulus, shear,
modulus of rigidity, poisson's ratio; elastic energy.
Viscosity, Stokes' law, terminal velocity, Reynold's number, streamline and turbulent flow. Critical
velocity, Bernoulli's theorem and its applications.
Surface energy and surface tension, angle of contact, excess of pressure, application of surface tension
ideas to drops, bubbles and capillary rise.
Heat, temperature, thermal expansion, thermal expansion of solids, liquids and gases. Anomalous
expansion. Specific heat capacity, CP, CV -calorimetry; change of state – latent heat.
Heat transfer- conduction and thermal conductivity, convection and radiation. Qualitative ideas of
Black Body Radiation, Wein's displacement law and Green House effect.
Newton's law of cooling and Stefan's law.

UNIT VIII Thermodynamics


Thermal equilibrium and definition of temperature (zeroth law of Thermodynamics). Heat, work and
internal energy.
First law of thermodynamics: Isothermal and adiabatic processes.
Second law of the thermodynamics: Reversible and irreversible processes. Heat engines and
refrigerators.

UNIT IX Behaviour of Perfect Gas and Kinetic Theory


Equation of state of a perfect gas, work done on compressing a gas.
Kinetic theory of gases: Assumptions, concept of pressure. Kinetic energy and temperature, degrees of
freedom, law of equipartition of energy (statement only) and application to specific heat capacities of
gases, concept of mean free path.

UNIT X Oscillations and Waves


Periodic motion-period, frequency, displacement as a function of time. Periodic functions. Simple
harmonic motion(SHM) and its equation, phase, oscillations of a spring-restoring force and force
constant, energy in SHM–kinetic and potential energies, simple pendulum-derivation of expression for
its time period, free, forced and damped oscillations (qualitative ideas only), resonance.
Wave motion. Longitudinal and transverse waves, speed of wave motion. Displacement relation for a
progressive wave. Principle of superposition of waves, reflection of waves, standing waves in strings
and organ pipes, fundamental mode and harmonics. Beats, Doppler effect.

CLASS 12th
UNIT I Electrostatics
Electric charges and their conservation. Coulomb's law-force between two point charges, forces
between multiple charges, superposition principle and continuous charge distribution.
Electric field, electric field due to a point charge, electric field lines, electric dipole, electric field due to
a dipole, torque on a dipole in a uniform electric field.
Electric flux, statement of Gauss's theorem and its applications to find field due to infinitely long
straight wire, uniformly charged infinite plane sheet and uniformly charged thin spherical shell (field
inside and outside).
Electric potential, potential difference, electric potential due to a point charge, a dipole and system of
charges, equipotential surfaces, electrical potential energy of a system of two point charges and of
electric diploes in an electrostatic field.
Conductors and insulators, free charges and bound charges inside a conductor, Dielectrics and electric
polarization, capacitors and capacitance, combination of capacitors in series and in parallel,
capacitance of a parallel plate capacitor with and without dielectric medium between the plates,
energy stored in a capacitor, Van de Graaff generator.

UNIT II Current Electricity


Electric current, flow of electric charges in a metallic conductor, drift velocity and mobility and their
relation with electric current, Ohm's law, electrical resistance, V-I characteristics (linear and non-linear),
electrical energy and power, electrical resistivity and conductivity.
Carbon resistors, colour code for carbon resistors, series and parallel combinations of resistors,
temperature dependence of resistance.
Internal resistance of a cell, potential difference and emf of a cell, combination of cells in series and in
parallel.
Kirchhoff's laws and simple applications. Wheatstone bridge, metre bridge.
Potentiometer-principle and applications to measure potential difference, and for comparing emf of
two cells, measurement of internal resistance of a cell.

UNIT III Magnetic Effects of Current and Magnetism


Concept of magnetic field, Oersted's experiment. Biot-Savart’s law and its application to current
carrying circular loop.
Ampere's law and its applications to infinitely long straight wire, straight and toroidal solenoids. Force
on a moving charge in uniform magnetic and electric fields. Cyclotron.
Force on a current-carrying conductor in a uniform magnetic field. Force between two parallel current-
carrying conductors-definition of ampere. Torque experienced by a current loop in a magnetic field,
moving coil galvanometer-its current sensitivity and conversion to ammeter and voltmeter.
Current loop as a magnetic dipole and its magnetic dipole moment. Magnetic dipole moment of a
revolving electron. Magnetic field intensity due to a magnetic dipole (bar magnet) along its axis and
perpendicular to its axis. Torque on a magnetic dipole (bar magnet) in a uniform magnetic field, bar
magnet as an equivalent solenoid, magnetic field lines, Earth's magnetic field and magnetic elements.
Para-, dia-and ferro-magnetic substances with examples.
Electromagnetic and factors affecting their strengths. Permanent magnets.

UNIT IV Electromagnetic Induction and Alternating Currents


Electromagnetic induction: Faraday's law, induced emf and current, Lenz's Law, Eddy currents. Self and
mutual inductance.
Alternating currents, peak and rms value of alternating current/ voltage, reactance and impedance, LC
oscillations (qualitative treatment only), LCR series circuit, resonance, power in AC circuits, wattles
current.
AC generator and transformer.

UNIT V Electromagnetic Waves


Need for displacement current.
Electromagnetic waves and their characteristics (qualitative ideas only). Transverse nature of
electromagnetic waves.
Electromagnetic spectrum (radiowaves, microwaves, infrared, visible, ultraviolet, X-rays, gamma rays)
including elementary facts about their uses.

UNIT VI Optics
Reflection of light, spherical mirrors, mirror formula. Refraction of light, total internal reflection and its
applications optical fibres, refraction at spherical surfaces, lenses, thin lens formula, lens-maker's
formula. Magnification, power of a lens, combination of thin lenses in contact combination of a lens
and a mirror. Refraction and dispersion of light through a prism.
Scattering of light- blue colour of the sky and reddish appearance of the sun at sunrise
and sunset.
Optical instruments Human eye, image formation and accommodation, correction of eye defects
(myopia and hypermetropia) using lenses.
Microscopes and astronomical telescopes (reflecting and refracting) and their magnifying powers.
Wave optics: Wavefront and Huygens' principle, reflection and refraction of plane wave at a plane
surface using wavefronts.
Proof of laws of reflection and refraction using Huygens' principle.
Interference, Young's double hole experiment and expression for fringe width, coherent sources and
sustained interference of light.
Diffraction due to a single slit, width of central maximum.
Resolving power of microscopes and astronomical telescopes. Polarisation, plane polarised light,
Brewster's law, uses of plane polarised light and Polaroids.
UNIT VII Dual Nature of Matter and Radiation
Photoelectric effect, Hertz and Lenard's observations, Einstein's photoelectric equation- particle
nature of light.
Matter waves- wave nature of particles, de-Broglie relation. Davisson-Germer experiment
(experimental details should be omitted, only conclusion should be explained).

UNIT VIII Atoms and Nuclei


Alpha- particle scattering experiments, Rutherford's model of atom, Bohr model, energy levels,
hydrogen spectrum. Composition and size of nucleus, atomic masses, isotopes, isobars, isotones.
Radioactivity- a, b and g particles/ rays and their properties decay law. Mass-energy relation, mass
defect, binding energy per nucleon and its variation with mass number, nuclear fission and fusion.

UNIT IX Electronic Devices


Energy bands in solids (qualitative ideas only), conductors, insulators and semiconductors,
semiconductor diode- I-V characteristics in forward and reverse bias, diode as a rectifier, I-V
characteristics of LED, photodiode, solar cell and Zener diode, Zener diode as a voltage regulator.
Junction transistor, transistor action, characteristics of a transistor, transistor as an amplifier
(common emitter configuration) and oscillator. Logic gates (OR, AND, NOT, NAND and NOR).
Transistor as a switch.
EXAM BITES

This Pdf Is
Downloaded From
www.exambites.in

Visit www.exambites.in for


More Premium Stuffs,Latest
Books,Test Papers,Lectures etc.
jeeneetadda
jeeneetadda_official
jeeneetadda

VISIT NOW !!
DAY ONE

Physical
World and
Measurement
Learning & Revision for the Day
u Physics u Accuracy and Precision u Dimensions of Physical
u Units u Errors in Measurement Quantities
u Significant Figures

Physics
Physics is the study of matter and its motion, as well as space and time using concepts
such as energy, force, mass and charge. It is an experimental science, creating theories
that are tested against observation.

Scope and Excitement


Scope of Physics is very vast, as it deals with a wide variety of disciplines such as
mechanics, heat, light, etc.
It also deals with very large magnitude of astronomical phenomenon as well as very
small magnitude involving electrons, protons, etc.

Nature of Physical Laws PREP


Physics is the study of nature and natural phenomena. All observations and
experiments in physics lead to certain facts. These facts can be explained on the basis of
MIRROR
Your Personal Preparation Indicator
certain laws.
u No. of Questions in Exercises (x)—

Physics, Technology and Society u

u
No. of Questions Attempted (y)—
No. of Correct Questions (z)—
Connection between physics, technology and society can be seen in many examples like (Without referring Explanations)
working of heat engines gave rise to thermodynamics. Wireless communication
technology arose from basic laws of electricity and magnetism. Lately discovery of u Accuracy Level (z / y × 100)—
silicon chip triggered the computer revolution. u Prep Level (z / x × 100)—

Units In order to expect good rank in NEET, your


Accuracy Level should be above 85 & Prep
Level should be above 75.
Measurement of any physical quantity involves comparison with a certain basic, widely
accepted reference standard called unit.
02 40 DAYS ~ NEET PHYSICS DAY ONE

Fundamental and Derived Units Amount of


substance
mole
(mol)
The mole is the amount of substance of a
system, which contains as many
Fundamental units are the units which can neither be elementary entities as there are atoms in
derived from one another, nor they can be further resolved 0.012 kg of carbon-12.
into more simpler units. Luminous candela The candela is the luminous intensity in a
These are the units of fundamental quantity. However, (cd) given direction of a source emitting
derived units are the units of measurement of all physical intensity
monochromatic radiation of frequency
quantities which can be obtained from fundamental units. 540 × 1012 Hz and having a radiant
1
intensity of W sr −1 in that direction.
System of Units 683

A complete set of these units, both fundamental and derived Supplementary Units
Supplementary
unit is known as the system of units.
Quantity Name and
The common systems are given below: Definition
Symbol
1. CGS System (Centimetre, Gram, Second) are often used Plane angle radian It is the angle subtended at the
in scientific work. This system measures, Length in (rad) centre by an arc of a circle having
centimetre (cm), Mass in gram (g), Time in second (s). a length equal to the radius of the
2. FPS System (Foot, Pound, Second) It is also called the circle.
British Unit System. This unit measures, Length in foot Solid angle steradian It is the solid angle which is having
(foot), Mass in pound (pound), Time in second (s). (sr) its vertex at the centre of the sphere,
it cuts-off an area of the surface of
3. MKS System In this system also length, mass and time
sphere equal to that of a square with
have been taken as fundamental quantities and
the length of each side equal to the
corresponding fundamental units are metre, kilogram radius of the sphere.
and second.
4. International System (SI) of Units It is an extended NOTE • Angle subtended by a closed curve at an inside points
version of the MKS (Metre, Kilogram, Second) system. It is 2π rad.
has seven base units and two supplementary units. • Solid angle subtended by a closed surface at an inside
Seven base quantities and two supplementary point is 4 π steradian.
quantities, their units along with definitions are
tabulated below.
Basic Units
Significant Figures
Base In the measured value of a physical quantity, the digits about
Name and
Quantity Definintion
Symbol the correction of which we are sure, plus the last digits which
Length metre The metre is the length of path travelled is doubtful, are called the significant figures.
(m) by light in vacuum during a time interval Larger the number of significant figures obtained in a
of 1/299, 792, 458 part of a second. measurement, greater is the accuracy of the measurement.
Mass kilogra It is the mass of the international prototype
m (kg) of the kilogram (a platinum iridium alloy
cylinder) kept at International Bureau of Accuracy and Precision
Weights and Measures, at Sevres (France). The accuracy of a measurement is a measure of how close the
Time second The second is the duration of 9, 192, 631, measured value is to the true value of the quantity. However,
(s) 770 periods of the radiation corresponding precision tells us to what resolution or limit, the quantity is
to the transition between the two hyperfine measured by a measuring instrument.
levels of the ground state of cesium-133
atom. Least Count
Electric Ampere The ampere is that constant current, The least count of a measuring instrument is the least value,
current (A) which if maintained in two straight,
that can be measured using the instrument. It is denoted as
parallel conductors of infinite length
LC.
placed 1 m apart in vacuum would
(i) Least count of vernier callipers
produce a force equal to 2 × 10−7 Nm −1
Value of 1 main scale division
on either conductor. LC =
Total number of vernier scale division
Thermodyn- Kelvin 1
The kelvin is th fraction of the
amic (K) 273.16 (ii) Least count of screw gauge
temperature Value of 1 pitch scale reading
thermodynamic temperature of the triple LC =
point of water. Total number of head scale division
DAY ONE PHYSICAL WORLD AND MEASUREMENT 03

Errors in Measurement Uses of Dimensions


The difference in the true value (mean value) and measured (i) To check the correctness of a given physical equation.
value of a quantity is called error of measurement. Different (ii) Derivation of formula.
types of error are given below:
(i) Absolute error, (iii) Dimensional formula is useful to convert the value of a
a + a2 + a3 + ... an 1 i = n physical quantity from one system to the other. Physical
amean = a0 = 1 = ∑ ai quantity is expressed as a product of numerical value
n n i= 1
and unit. In any system of measurement, this product
∆a1 = mean value − observed value remains constant.
∆a1 = a0 − a1 Let dimensional formula of a given physical quantity be
∆a2 = a0 − a2 [Ma Lb T c ].If in a system having base units [M1L1T1] the
:. :. :. numerical value of given quantity be n1 and numerical
∆an = a0 − an value n2 in another unit system having the base units
(ii) Mean absolute error, [ M2 , L2 , T2 ], then Q = n1u1 = n2u2
[| ∆a1| + | ∆a2| + | ∆a3| + .... + | ∆an|] n1[M 1a L b1 T c1] = n2 [M2b L 2b T 2c ]
∆amean =
n a b c
n M  L  T 
n2 = n1  1   1   1 
∑| ∆ai|  M2   L2   T2 
i= 1
=
n Dimensions of Important Physical Quantities
∆amean
(iii) Relative or fractional error = Physical Quantity SI Unit
Dimensional
amean Formula
(v) Percentage error, Power Watt (W) [ML2 T −3 ]
∆amean Pressure, stress, coefficient of Pascal (Pa) [ML−1 T −2 ]
δ a = Relative error × 100 % = × 100%
amean elasticity (ρ, σ, η) or Nm −2
Frequency, angular frequency Hz or s −1 [T −1 ]
Combination of Errors Angular momentum kg m2 s −1 [ML2 T −1]
(i) If X = A + B, then (∆X ) = ± (∆A + ∆B) Torque Nm [ML2 T −2 ]
 ∆X   ∆A ∆B ∆C  Gravitational constant (G) N m2 kg −2 [M −1 L3 T −2 ]
(ii) If X = ABC, then   =± + +
 X  max  A B C 
Moment of inertia kg m2 [ ML2 ]
 ∆X   ∆A ∆B ∆C 
(iii) If X = A B C , then 
k l
 = ± k
n
+l +n Acceleration, acceleration due ms −2 [LT −2 ]
 X   A B C  to gravity
Force, thrust, tension, weight Newton (N) [MLT −2 ]
Dimensions of Physical Quantities Linear momentum, impulse kg ms −1 or Ns [MLT −1 ]
The dimensions of a physical quantity are the powers to Work, energy, KE, PE, thermal Joule (J) [ML2 T −2 ]
which the fundamental (base) quantities are raised, to energy, internal energy, etc.
represent that quantity. Surface area, area of m2 [L2 ]
To make it clear, consider the physical quantity force. cross-section
‘Force = mass × acceleration = mass × length × (time)−2 ’ Electric conductivity Sm −1 [M −1 L−3 T3 A2 ]
Thus, the dimension of force are 1 in mass [M]
1 in length [L] and −2 in time [ T−2 ], that is [MLT−2 ]. Young’s modulus, Pa [ML−1 T −2 ]
Bulk modulus
NOTE • Dimensions of a physical quantity do not depend on its Compressibility m2 N −1 [M −1 LT2 ]
magnitude or the units in which it is measured.
Magnetic Flux Wb [ML2 T −2 A −1 ]

Principle of Homogeneity of Magnetic Flux density (σ ) Wb / m2 [MT −2 A −1 ]


Wm −2 [MT −3 ]
Dimensions and Applications Intensity of a wave

According to this principle, a correct dimensional equation Photon flux density m −2 s −1 [L−2 T −1 ]
must be homogeneous, i.e. dimensions of all the terms in a Luminous energy Lm s [ML2 T −2 ]
physical expression must be same.
Luminance Lux [MT −3 ]
LHS = RHS
04 40 DAYS ~ NEET PHYSICS DAY ONE

Dimensional Dimensional
Physical Quantity SI Unit Physical Quantity SI Unit
Formula Formula
Specific heat capacity Jkg −1 K −1 [L2 T −2 K −1 ] Magnetisation Am −1 [L−1 A]

Latent heat of vaporisation Jkg −1 [L2 T −2 ] Magnetic induction T [MT −2 A −1 ]

Coefficient of Thermal Wm −1 K −1 [MLT −3 K −1 ]


Planck’s constant J-s [ML2 T −1 ]
conductivity Radioactive decay constant Bq [T −1 ]
−1 2 −3 −1
Electric voltage JC [ML T A ] Binding energy MeV [ML2 T −2 ]

DAY PRACTICE SESSION 1

FOUNDATION QUESTIONS EXERCISE


1 The numerical value of a given quantity is 10 The density of a cube is measured by measuring its
(a) independent of unit mass and length of its sides. If the maximum error in the
(b) directly proportional to unit measurement of mass and length are 4% and 3%,
(c) inversely proportional to unit respectively, the maximum error in the measurement of
(d) directly proportional to the square root of the unit density will be
2 Unit of reduction factor is (a) 7% (b) 9% (c) 12% (d) 13%
(a) ampere (b) ohm (c) tesla (d) weber 11 In an experiment four quantities a, b, c and d are
3 Lumen is the unit of measured with percentage error 1%, 2%, 3% and 4%
(a) illuminating power (b) luminous flux respectively. Quantity P is calculated as follows
(c) luminous intensity (d) None of these a3 b 2
P= %. Error in P is
4 Which one of the following is not a unit of Young’s cd j
NEET 2013
modulus? (a) 14% (b) 10%
(a) Nm −1 (b) Nm −2 (c) 7% (d) 4%
(c) Dyne cm−2 (d) Mega pascal 12 If force (F ), length (L) and time (T ) be considered
5 Young’s modulus of the material of a wire is fundamental units, then the units of mass will be
18 × 1011 dyne cm −2 . Its value in SI is (a) [FLT−2 ] (b) [FL−2 T−1 ]
(a)18 × 10 Nm15 −2
(b) 18 × 10 Nm 10 −2
(c) [FL−1T2 ] (d) [F 2LT−2 ]
(c)18 × 109 Nm−2 (d) 18 × 1012 Nm−2
13 Which of the following quantities has units but not
6 Which of the following measurement is most precise? dimensions?
(a) 5.00 mm 1(b) 5.00 cm (a) Displacement (b) Angle
(c) 5.00 m (d) 5.00 km (c) Couple (d) Speed
7 The respective number of significant figures for the 14 Pascal-second has the dimensions of
numbers 23.023, 0.0003 and 2.1 × 10−3 are (a)force (b) energy
(a) 5, 1, 2 (b) 5, 1, 5 (c) 5, 5, 2 (d) 4, 4, 2 (c)pressure (d) coefficient of viscosity
8 A student measured the diameter of a small steel ball 15 The physical quantity which does not have the same
using a screw gauge of least count 0.001 cm. The main dimensions as the other three is
scale reading is 5 mm and zero of circular scale division (a) spring constant
coincides with 25 divisions above the reference level. If (b) surface tension
screw gauge has a zero error of −0.004 cm, the correct (c) surface energy
diameter of the ball is j
NEET 2018 (d) acceleration due to gravity
(a) 0.053 cm (b) 0.525 cm (c) 0.521 cm (d) 0.529 cm
16 Pressure gradient has the same dimensions as that of
9 If the error in the measurement of radius of a sphere is (a)velocity gradient
2%, then the error in the determination of volume of the (b) potential gradient
sphere will be (c)energy gradient
(a) 4% (b) 6% (c) 8% (d) 2% (d) None of these
DAY ONE PHYSICAL WORLD AND MEASUREMENT 05

− αz
17 Dimensions of the resistance in an electrical circuit in α kθ
terms of dimension of mass M, length L ,time T and 26 In the relation p = e , p is pressure, z is distance, k
β
current I, are
is Boltzmann constant and θ is temperature. The
(a) [ML2 T−3 A −1 ] (b) [ML2 T−2 ]
dimensional formula of β will be
(c) [ML2 T−1A −1 ] (d) [ML2 T−3 A −2 ] (a) [M0L2 T0 ] (b) [ML2 T ]

1 (c) [ML0 T−1] (d) [M0L2 T−1 ]
18 The dimensions of (µ 0ε 0 ) 2 are j CBSE-AIPMT 2011
−1 −1 −1/ 2
27 If momentum (p ), area (A) and time (T ) are taken to be
(a) [L T] (b) [LT ] (c) [L T1/ 2
] (d) [L1/ 2 T−1/ 2 ]
fundamental quantities, the energy has the dimensional
19 If L is the inductance, C capacitance and R resistance formula
the ratios L /R and R-C have the same dimensions as (a) [p A −1 T1] (b) [p 2 A1 T1] (c) [p1 A −1/ 2 T1] (d) [p1 A1/ 2 T−1]
those of
28 The refractive index of a material is given by the equation
(a) frequency (b) time (c) energy (d) length B
n=A+ , where A and B are constants. The
20 The ratio of the dimensions of Planck’s constant and that λ2
of the moment of inertia is the dimensions of dimensional formula for B is
(a) frequency (b) velocity (a) [M0L2 T] (b) [M0L−2 T 0 ]
(b) angular momentum (d) time
(c) [M0L2 T−2 ] (d) [M0L2 T 0 ]
21 If energy (E ), velocity (v) and time (T ) are chosen as the 29 If the dimensions of a physical quantity are given by
fundamental quantities, the dimensional formula of
surface tension will be j CBSE AIPMT 2015 [MaLb Tc ], then the physical quantity will be
j CBSE AIPMT 2009
(a) [Ev –1T−2 ] (b) [Ev –2 T−2 ]
–2 –1 −3 –2 −1 (a) pressure if a = 1, b = −1, c = −2
(c) [E v T ] (d) [Ev T ]
(b) velocity if a = 1, b = 0, c = −1
22 If force (F ) , velocity (v ) and time (T ) are taken as (c) acceleration if a = 1, b = 1, c = −2
fundamental units, then the dimensions of mass are (d) force if a = 0, b = −1, c = −2
j
CBSE AIPMT 2014
30 If F = 6πηar b v c ,where, F = viscous force,
(a) [FvT −1 ] (b) [FvT −2 ] (c) [Fv –1T−1 ] (d) [Fv –1T]
η = coefficient of viscosity, r = radius of spherical body,
1 v = terminal velocity of the body
23 The dimensions of ε 0 E 2 , where ε 0 is permittivity of free
2
space and E is electric field, are j
CBSE AIPMT 2010 The values of a, b and c are
2
(a) [ML T ] −2 −1 −2
(b) [ML T ] (c) [ML T ] 2 −1
(d) [MLT ]−1 (a)a = 1, b = 2, c = 1 (b) a = 1, b = 1, c = 1
(c)a = 2, b = 1, c = 1 (d) a = 2, b = 1, c = 2
γp
24 Velocity of sound in a gas is given by v = . 31 If dimensions of critical velocity vc of a liquid flowing
ρ
through a tube are expressed as [ ηx ρ y r z ], where η, ρ and
Dimensional formula for γ is
r are the coefficient of viscocity of liquid, density of liquid
(a) [MLT] (b) [M0L0 T 0 ] (c) [M0LT 0 ] (d) [ML0 T 0 ]
and radius of the tube respectively, then the value of x , y
25 In the equation y = a sin (ωt + kx ), the dimensional and z are given by j
CBSE AIPMT 2015
formula of ω is
(a) 1, − 1, − 1 (b) −1, − 1, 1
(a) [M0L0 T−1 ] (b) [M0LT−1 ]
(c) −1, − 1, − 1 (d) 1, 1, 1
(c) [ML0 T0 ] (d) [M0L−1T0 ]

DAY PRACTICE SESSION 2

PROGRESSIVE QUESTIONS EXERCISE


1 1 Wb/ m 2 is equal to . 3 SI unit of permittivity is
(a) 104 G (b) 102 G (a) C 2 m 2 N 2 (b) C 2m–2N–1
(c) 10−2 G (d) 10−4 G (c) C 2 m 2 N −1 (d) C −1m2N2
2 The magnetic moment has dimensions of 4 If h is Planck’s constant and λ is wavelength, h/λ has
(a) [L A] (b) [L A]2 dimensions of
(c) [LT−1A] (d) [L2 T−1A] (a) momentum (b) energy (c) mass (d) velocity
06 40 DAYS ~ NEET PHYSICS DAY ONE

5 The length and breadth of a rectangular sheet are type f = Cm x k y , where C is a dimensionless constant.
16.2 cm and 10.1 cm, respectively. The area of the sheet The values of x and y are
in appropriate significant figures and error is 1 1 1 1
(a) x = ,y = (b) x = − ,y = −
(a)(164 ± 3) cm2 (b) (163 . 62 ± 2 . 6) cm2 2 2 2 2
(c)(163 . 6 ± 2 . 6) cm2 (d) (163 . 62 ± 3) cm2 1 1 1 1
(c) x = , y = − (d) x = − , y =
2 2 2 2
6 Which of the following pairs of physical quantities does
not have same dimensional formulae?
12 A student measured the length of a rod and wrote it as
3.50 cm.Which instrument did he use to measure it?
(a) Work and torque
(b) Angular momentum and planck’s constant (a) A meter scale
(c) Tension and surface tension (b) A Vernier calliper, where the 10 divisions in Vernier
(d) Impulse and linear momentum scale matches with 9 divisions in main scale and main
scale has 10 divisions in 1 cm
7. If E , M , L and G denote energy, mass, angular
(c) A screw gauge having 100 divisions in the circular scale
momentum and gravitational constant respectively, then
and pitch as 1 mm
the quantity (E 2L2/M 5G 2 ) has the dimensions of
(d) A screw gauge having 50 divisions in the circular scale
(a) angle (b) length
and pitch as 1 mm
(c) mass (d) None of these
13 Resistance of a given wire is obtained by measuring the
8 van der Waals’, equation of state is
 a  current flowing in it and the voltage difference applied
 p + 2  (V − b ) = nRT . The dimensions of a and b are
 V  across it. If the percentage errors in the measurement of
the current and the voltage difference are 3% each, then
(a)[ML3 T2 ], [ML3 T 0 ] (b) [ML5 T–2 ], [M0L3 T 0 ]
error in the value of resistance of the wire is
(c)[M2LT2 ], [ML3 T2 ] (d) [ML2 T], [ML2 T2 ]
(a) 6% (b) zero (c) 1% (d) 3%
9 According to Newton, the viscous force acting between
∆v 14 A physical quantity of the dimensions of length that can
liquid layers of area A and velocity gradient is given e2
∆z be formed out of c, G and is [c is velocity of light, G
dv 4πε 0
by F = − ηA , where η is constant called coefficient of
dz is universal constant of gravitation and e is charge]
viscosity. The dimensional formula of η is j
NEET 2017
−2 −2 0 0 0 1/ 2 1/ 2
(a) [ML T ] (b) [M L T ] 1  e  2  e 
2
(c) [ML2 T−2 ] (d) [ML−1T−1 ] (a) G  (b) c 2 G 
c 2  4 πε0   4 πε0 
10 A physical quantity is given by X = [MaLb T c ]. The 1/ 2
1  e2  1 e2
percentage error in measurements of M, L and T are (c)   (d) G
α , β and γ. Then, the maximum % error in the quantity X c 2  G 4 πε0  c 4 πε0
is 15. The period of oscillation of a simple pendulum is
(a) aα + bβ + c γ (b) aα + bβ − c γ
T = 2π L / g . Measured value of L is 20.0 cm known to
a b c
(c) + + (d) None of these 1mm accuracy and time for 100 oscillations of the
α β γ
pendulum is found to be 90 s using a wrist watch of
11 The frequency of vibration f of a mass m suspended from resolution.The accuracy in the determination of g is
a spring of spring constant k is given by a relation of the (a) 2% (b) 3% (c) 1% (d) 5%

ANSWERS
SESSION 1 1 (c) 2 (a) 3 (b) 4 (a) 5 (b) 6 (a) 7 (a) 8 (d) 9 (b) 10 (d)
11 (a) 12 (c) 13 (b) 14 (d) 15 (d) 16 (d) 17 (d) 18 (b) 19 (b) 20 (a)
21 (b) 22 (d) 23 (b) 24 (b) 25 (a) 26 (a) 27 (d) 28 (d) 29 (a) 30 (b)
31 (a)

SESSION 2 1 (a) 2 (b) 3 (b) 4 (a) 5 (a) 6 (c) 7 (d) 8 (b) 9 (d) 10 (a)
11 (d) 12 (b) 13 (a) 14 (a) 15 (b)
DAY ONE PHYSICAL WORLD AND MEASUREMENT 07

Hints and Explanations


SESSION 1 Here, zero error = −0.004 cm From the formula of coefficient of
Final reading obtained from the screw viscosity, we have
1 In general, n [u] = constant F
gauge is given as η=
where, n = numerical value, A( ∆ v / ∆ z )
u = unit of physical quantity = MSR + VSR × LC − zero error
1 Final reading from the screw gauge ∆v
n∝ where, F is force, A is area and is
[u] = 0.5+ 25 × 0.001 − (−0.004) ∆z
velocity gradient.
2 Current flowing in the coil of tangent = 0.5+ 0.025+ 0.004
[MLT −2 ]
galvanometer is given by I = Kφ = 0.5+ 0.029 = 0.529 cm ∴ Dimensions of η =
[L ] [LT −1 / L]
2

where, K is a constant called reduction Thus, the diameter of the ball is 0.529 cm. = [ML−1 T −1 ]
factor and φ is the angle of deflection.
Since, deflection has no unit. So, unit of
9 Volume of a sphere, V = 4 πr 3 Hence, Pascal-second has dimensions of
3 coefficient of viscosity.
reduction factor is same as of current, ∆V 3 × ∆r
∴ × 100 = × 100
i.e. ampere. V r 15 Spring constant, surface tension and
∆r surface energy have the same
3 The lumen (lm ) is the SI derived unit of Here × 100 = 2% dimensions, i.e. [ML0 T −2 ]. However,
luminous flux, a meausre of total r
∆V acceleration due to gravity has
quantity of visible light emitted by a ∴ × 100 = 3 × 2% = 6% dimensions [LT −2 ].
source. V
kg -m −1 /s 2
10 Density, ρ = m = m3 16 Pressure gradient = = [ML−2 T −2 ]
4 Young’s modulus, m
v l
stress N m/s
Y = = ∆ρ  ∆m ∆l Velocity gradient = = [M 0 L0 T −1 ]
+ 3  × 100
or pascal
strain m2 ⇒ × 100 = ±  m
ρ  m l 
kg -m2 /s 3 -A
[in SI system]
= ± (4 + 3 × 3)% = ± 13% Potential gradient =
dyne m
Y = [in CGS system]
cm2 11 Here, P =
a 3 b2 = [MLT −3 A −1 ]
Hence, Nm −1 is not the unit of Young’s cd kg -m2 /s 2
Energy gradient = = [MLT −2 ]
modulus. ∆P m
∴ × 100
P Hence option (d) is correct.
5 Unit of Young’s modulus Y in SI units is 3∆a 2 ∆b ∆c ∆d 
Nm −2 . =  + + +  × 100 17 Resistance,
dyne  a b c d 
Here, Y = 18 × 1011 ∆a ∆b R=
Potential difference V
= =
W
cm2 =3 × 100 + 2 × 100 Current I QI
As, 1 dyne = 10−5 N a b
∆c ∆d  W
1 cm = 10−2 m + × 100 + × 100
c d QV = Q 
10−5  
⇒ Y = 18 × 10
11
× Nm −2 =3 × 1 + 2 × 2 + 3 + 4
(10−2 ) 2 So, dimension of R
= 3 + 4 + 3 + 4 = 14%
= 18 × 1010 Nm −2 [Dimension of work] [ML2 T −2 ]
12 Let M ∝ [ F a Lb Tc ] = =
6 All measurements are correct upto two [Dimension of charge] [AT] [A]
places of decimal. However, the Writing dimensions of both sides and [Dimension of current]
absolute error in (a) is 0.01 mm, which using the principle of homogeneity of
is the least of all the four. So, 5.00 mm is dimensions, we have = [ML2 T −3 A −2 ]
most precise. [M1 L0 T 0 ] = K [MLT −2 ] a [L] b [T] c 1
18 We know that, c =
7 Number of significant figures in On comparing the powers both sides µ 0 ε0
23.023 = 5 a = 1, a + b = 0, −2a + c = 0 where, c is speed of light.
Number of significant figures in On solving, we have
Hence, dimensions of ( µ 0 ε0 )−1 /2 is equal
0.0003 = 1 b = − 1, c = 2, a = 1
to that speed of light [LT −1 ].
Number of significant figures in ∴ Units of mass is [FL−1 T2 ].
2.1 × 10−3 = 2 13 As we know that, 19 Dimensional formula of L is [ML2 T −2 A −2 ],
8 Given, least count of screw gauge, arc [ L] C is [M −1 L−2 T 4 A2 ] and that of R is
Angle = = = dimensionless
radius [L] [ML2 T −3 A −2 ]. Thus, dimensional formula
LC = 0.001 cm of L /R is [ T ].
Main scale reading, But unit of angle is radian.
Similarly, dimensional formula of R-C is
MSR = 5mm = 0.5cm 14 Pascal is unit of pressure, hence its [T].
Number of coinciding divisions on the dimensional formula is [ML−1 T −2 ].
∴ L /R and R-C has the dimensions of
circular scale, i.e. Vernier scale reading, ∴Dimensional formula of Pascal-second time.
VSR = 25 is [ML−1 T −1 ].
08 40 DAYS ~ NEET PHYSICS DAY ONE

20 From Einstein’s equation, E = hν Applying principle of homogeneity, we 2 Magnetic moment, M = IA


2 −2
get a= 1, a + 2b = 2 and − a + c = − 2 Thus, dimensions of M = [A][L2 ] = [L2 A]
E [ML T ]
⇒ h= = Solving it, we get q q
ν [T −1 ] 3 Coulomb’s law, F = 1 ⋅ 1 2 2
a = 1, b = 1/2 and c = − 1 4 πε 0 r
Dimensions of Planck’s constant,
∴ E = K [ p1 A1 /2 T −1 ] q1 q2
[h] = [ML T ] 2 −1 ⇒ ε0 =
4 πFr 2
Also, moment of inertia, I = MR2 28 Refractive index, n = A + B2 C2
λ =
⇒ [I] = [ML2 ] Nm2
From principle of homogeneity of
[h] [ML2 T −1 ] dimensions, quantity = C2 m −2 N −1
∴ = = [T −1 ]
[I] [ML2 ] B/λ2 should have the dimensions of n 4 We know that, wavelength λ = h
= dimension of frequency.  B  = [M 0 L0 T 0 ] mv

 λ2  h
21 Surface tension ∴ = mv .
∴ B = [M L T ] × [L2 ] = [M 0 L2 T 0 ]
0 0 0 λ
Force Surface energy
= =
Length Area 29 (i) Dimensions of pressure = [M1 L−1 T −2 ] Hence, h/λ has dimensions of
momentum.
=
[E] −2 −2
= [Ev T ] ∴If a = 1, b = −1, c = −2, then
[v ⋅ T]2 the physical quantity is pressure. 5 Here, l = (16.2 ± 0.1) cm,
(ii) Dimensions of velocity = [M 0 L1 T −1 ] b = (10.1 ± 0.1) cm
22 We know that, F = Ma
Mv Ft Here, a = 0, b = 1, c = −1 A = l × b = 16.2 × 10.1 = 163. 62
⇒ F = or M = Rounding off to one significant figure,
t v (iii) Dimensions of acceleration
= [M 0 L1 T −2 ] A = 164 cm2
∴ Dimension of M = [Fv −1 T] ∆A  ∆l ∆b 
Here, a = 0, b = 1, c = −2 = + 
23 As we know that, A  l b 
−1 −3
Dimension of ε0 = [M L T A ] 4 2
0 .1 0 .1
= +
Dimension of E = [MLT −3 A −1 ] 16.2 10.1
(iv) Dimensions of force = [M1 L1 T −2 ]
So, dimension of
Here, a = 1, b = 1, c = −2 1. 01 + 1. 62
1 = = 2. 63 cm2
ε0 E 2 = [M −1 L−3 T 4 A2 ] × [MLT −3 A −1 ]2 16.2 × 10.1
2 30 According to homogeneity principle,
Rounding off to one significant figure,
= [ML−1 T −2 ] [F ] = [6πη a r b v c ]
∆A = 3cm2
or [MLT −2 ] = [ML−1 T −1 ] a [L]b [LT −1 ] c
24 γ is the ratio of C p to C V which has no ∴ A = (164 ± 3)cm2
or [MLT −2 ] = [ M a L− a + b + c T − a − c ]
unit. 6 Tension = Force = [M1 L1 T −2 ]
Equating the coefficients and powers,
∴ Its dimensional formula is [M 0 L0 T 0 ] .
we get Surface tension
25 Given, y = a sin (ωt + kx ) ∴ a = 1, − a + b + c = 1 Force [M 1 L1 T −2 ]
v and − a−c = −2 = =
Here, ω = angular velocity = Length [L]
r After solving a = 1, b = 1, c = 1
= [M1 L0 T −2 ]
[v ] [LT −1 ]
∴ [ω] = = = [T −1 ] 31 Given critical velocity of liquid flowing So, tension and surface tension does not
[r ] [L] through a tube as expressed as have same dimensional formulae.
= [M 0 L0 T −1 ] v c ∝ η xρy r z
Coefficient of viscocity of liquid,
7 The dimensions of E = [ML2 T −2 ]
26 Using the principle of homogeneity of Dimensions of M = [M]
αz η = [ML−1 T −1 ]
dimensions, the quantity is Density of liquid, ρ = [ML−3 ] Dimensions of L = [ML2 T −1 ]

Radius of a tube r = [L] Dimensions of G = [M −1 L3 T −2 ]
dimensionless.
kθ Critical velocity of liquid ∴ Dimensions of
⇒ α =  E2 L2  [ML2 T −2 ] 2 [ML2 T −1 ] 2
v c = [M 0 L1 T −1 ]
z  M 5G 2  = [M] 5 [M −1 L3 T −2 ] 2
[ML2 T −2 K −1 ] × [K] ⇒ [M L T −1 ] = [ML−1 T −1 ] x [ML−3 ] y [L] z
0 1
 
⇒ [ α] = = [MLT −2 ]
[L] [M 0 L1 T −1 ] = [M x + y L− x −3 y + z T − x ] = [ML2 T −2 ]
α Comparing exponents of M, L and T,
Also, p = 8 van der Waals’ equation,
β we get
x + y = 0, − x − 3 y + z = 1, − x = −1  p + a  (V − b ) = R T
 α  [MLT −2 ]  
⇒ [ β] =   = = [M 0 L2 T 0 ] ⇒ z = −1, x = 1, y = −1  V2 
−1 −2
 p  [ML T ]
a / V 2 should have dimensions of
27 Let energy E ∝ p a A b T c SESSION 2 pressure.
1 We know that, 1 G = 10−4 T ∴ a = pressure × V 2
= K pa A b T c
1 = [ML−1 T −2 ][L3 ]2 =[ML5T −2 ]
where, K is dimensionless constant. ⇒ 1T= G = 104 G b should have dimensions of volume, i.e.
−2 −1 10− 4
⇒ [M L T ] = [MLT
1 2
] a 2 b
[ L] T c
[M 0 L3 T 0 ]
Also, 1 T = 1 Wb/m2
−a + c
=M a a + 2b
L T ⇒ 1 Wb/m2 = 10 4 G
DAY ONE PHYSICAL WORLD AND MEASUREMENT 09

dv 12 If student measures 3.50 cm, it means Using Eqs. (i), (ii) and (iii), we get
9 As, F =−η A 1 /2
dz that there is an uncertainly of order 1  Ge 2 
F 0.01cm.  
∴ η=− c 2  4 πε0 
A  
 dv For Vernier scale with 1 MSD = 1 mm
 dz  = [L−2 T2 ][[ML3 T −2 ][M −1 L3 T −2 ]]1 /2
and 9 MSD = 10 VSD
= [L−2 T2 ][L3 T −2 ]
As F = [MLT −2 ], A = [L2 ] LC of Vernier calliper = 1 MSD – 1VSD = [L]
dv = [LT −1 ], dz = [L]
=  1 −
1 9 1
= cm
[MLT −2 ] [L] 10  10  100 15 Given, time period, T = 2π L
∴ η= 2 g
[L ] [LT −1 ]
13 From Ohm’s law, R = V Thus,changes can be expressed as
= [ML−1 T −1 ] I 2 ∆T ∆L ∆g
= =± ±
⇒ ln R = ln V − ln I T L g
10 Maximum possible % error is
∆R ∆V ∆I
∆X ⇒ = + According to the question, we can write
= × 100 R V I ∆L 0.1cm 1
X = =
∆M ∆L ∆T = 3% + 3% = 6% L 20.0cm 200
=a +b +c 90
M L T e2 e2 Again, time period, T = s and
14 As force, F = 2
⇒ = r2 ⋅ F 100
= aα + bβ + cγ 4πε0 r 4πε0 1 ∆T 1
∆T = s ⇒ =
Putting dimensions of r and F, 200 T 90
11 As, f = Cm x k y
we get, ∆g ∆L 2∆T
∴ = +
∴ (Dimension of f ) g L T
 e2  3 −2
= C (dimension of m ) x ⇒   = [ML T ] …(i)
∆g  ∆L 
 4πε0  or × 100% =   × 100%
× (dimensions of k ) y g  L 
[T ] = C [M] [MT −2 ] y
−1 x Gm2
…(i) Also, force, F = 2∆T 
r2 +   × 100%
Applying the principle of homogeneity  T 
of dimensions, we get [MLT −2 ][L2 ]
⇒ [G ] =
=  × 100 % + 2 ×
1 1
[M2 ] × 100%
x + y = 0, − 2 y = − 1  200  90
1 ⇒ [G ] = [M −1 L3 T −2 ] …(ii) = 2.72% = 3%
or y =
2 Thus, accuracy in the determination of g
and 1= 1
= [L−2 T2 ] ...(iii)
1  c 2  [L2 T −2 ]
∴ x=− is approximately 3 %.
2
DAY TWO

Kinematics
Learning & Revision for the Day
u Frame of Reference u Uniformly Accelerated u Elementary Concept of
u Motion in a Straight Line Motion Differentiation and
Graphs Integration for Describing
u Uniform and Non-uniform u

Motion
Motion

Frame of Reference
The frame of reference is a suitable coordinate system involving space and time used as a
reference to study the motion of different bodies. The most common reference frame is
the cartesian frame of reference involving (x, y, z and t).
(i) Inertial Frame of Reference A frame of reference which is either at rest or moving
with constant velocity is known as inertial frame of reference. Inertial frame of
reference is one in which Newton’s first law of motion holds good.
(ii) Non-Inertial Frame of Reference A frame of reference moving with some
acceleration is known as non-inertial frame of reference. Non-inertial frame of
reference in one which Newton’s law of motion does not hold good.

Motion in a Straight Line


The motion of a point object in a straight line is one dimensional motion. During such a
motion the point object occupies definite position on the path at each instant of time.
Different terms used to described motion are defined below:

Distance and Displacement


l
Distance is the total length of the path travelled by a particle in a given interval of PREP
time. It is a scalar quantity and its SI unit is metre (m).
Displacement
MIRROR
l
Displacement is shortest distance between initial and final Your Personal Preparation Indicator
Y Distance
positions of a moving object. It is a vector quantity and its SI u No. of Questions in Exercises (x)—
unit is metre. A
u No. of Questions Attempted (y)—
From the given figure, mathematically it is expressed as, r1 ∆r u No. of Correct Questions (z)—
B
∆r = r2 – r1 r2 (Without referring Explanations)
l
Displacement of motion may be zero or negative O X u Accuracy Level (z / y × 100)—
but path length or distance can never be negative.
u Prep Level (z / x × 100)—
l
For motion between two points displacement is single valued while distance depends
on actual path and so can have many values. In order to expect good rank in NEET, your
Accuracy Level should be above 85 & Prep
l
Magnitude of displacement can never be greater than distance. However, it can be Level should be above 75.
equal, if the motion is along a straight line without any change in direction.
DAY TWO KINEMATICS 11

Average and Instantaneous Acceleration If velocity of a


Speed and Velocity particle at instant t is v1 and at instant t 2 is v2 , then
l
Speed is defined as the total path length (or actual distance v2 − v1 ∆v
l
Average acceleration, aav = =
covered) by time taken by object. t2 − t 1 ∆t
Distance ∆v dv
Speed = l
Instantaneous acceleration, a = lim =
Time taken ∆t→ 0 ∆t dt
It is scalar quantity. Its SI unit is m/s.
Total distance travelled
l
Average Speed, vav =
∆t
Uniformly Accelerated Motion
l
A motion, in which change in velocity in each unit of time
l
When a body travels equal distance with speeds v1 and v2 , is constant, is called an uniformly accelerated motion. So,
the average speed (v) is the harmonic mean of the two for an uniformly accelerated motion, acceleration is constant.
speeds.
2 1 1
l
For uniformly accelerated motion are given below
= +
v v1 v2 Equations of motion, v = u + at …(i)
1
l
When a body travels for equal time with speeds v1 and v2 , s = ut + at 2 …(ii)
the average speed v is the arithmetic mean of the two speeds. 2
v + v2 and v2 = u2 + 2 as …(iii)
vav = 1
2 where, u = initial velocity, v = velocity at time t
l
Velocity is defined as ratio of displacement and and s = displacement of particle at time t.
corresponding time interval taken by an object. l
Equation of uniformaly accelerated motion under gravity
Displacement are
i.e. velocity =
time interval 1
(i) v = u − gt (ii) h = ut − gt 2 (iii) v2 = u2 − 2 gh
Total displacement x2 − x 1 ∆x 2
l
Average velocity = = =
Total time taken t2 − t 1 ∆t
Here, x2 and x1 are the positions of a particle at the time Elementary Concept of Differentiation
t2 and t 1 respectively, with respect to a given frame of and Integration for Describing Motion
reference.
l
At an instant t , the body is at point P ( x, y, z).
l
For a moving body speed can never be negative or zero
dx
while velocity can be negative and zero. Thus, velocity along X-axis, v x =
dt
l
The instantaneous speed is average speed for infinitesimal
small time interval (i.e. ∆t → 0) dv
Acceleration along X-axis is ax = x
∆s ds dt
i.e. Instantaneous speed v = lim =
∆t → 0 ∆t dt dy
Velocity along Y -axis is v y =
l
The instantaneous velocity (or simply velocity) v of a dt
∆x dx
moving particle is v = lim = dv y
∆t → 0 ∆ t dt Acceleration along Y-axis is a y =
dt
It (at a particular time) can be calculated as the slope (at dz dv
that particular time) of the graph of x versus t. Similarly, vz = and az = z
dt dt
For a accelerating body
Uniform and Non-uniform Motion
l

(i) If ax variable, x = ∫ vx dt ,∫ dvx = ∫ ax dt


l
An object is said to be in uniform motion if its velocity is
uniform i.e. it undergoes equal displacement in equal may (ii) If a y is variable, y = ∫ v y dt , ∫ dv y = ∫ a y dt
be intervals of time, however small these interval.
l
An object is said to be in non-uniform motion if its (iii) If a z is variable, z = ∫ vz dt , ∫ dvz = ∫ az dt
undergoes equal displacement in unequal intervals of
Also, distance travelled by a particle is s = ∫ | v| dt
time., however small these intervals may be.
(i) x-component of displacement is ∆x = ∫ vx dt
Acceleration
Acceleration of an object is defined as rate of change (ii) y-component of displacement is ∆y = ∫ v y dt
of velocity. It is a vector quantity having unit m/s2 or ms −2 . (iii) z-component of displacement is ∆z = ∫ vz dt
It can be positive, zero or negative.
12 40 DAYS ~ NEET PHYSICS DAY TWO

Graphs Velocity-Time Graph


During motion of the particle, its parameters of kinematical (i) Velocity-time graph gives the instantaneous value of
analysis changes with time. These can be represented on the velocity at any instant.
graph, which are given as follows: (ii) The slope of tangent drawn on graph gives instantaneous
acceleration.
Position-Time Graph (iii) Area under v-t graph with time axis gives the value of
(i) Position-time graph gives instantaneous value of displacement covered in given time.
displacement at any instant. (iv) The v-t curve cannot take sharp turns.
(ii) The slope of tangent drawn to the graph at any instant of
Different Cases in Velocity-Time Graph
time gives the instantaneous velocity at that instant.
(iii) The s-t graph cannot make sharp turns. Different Cases v- t Graph The main Features of Graph

Different Cases of Position-Time Graph Uniform motion v (i) θ = 0º


(ii) v = constant
Different Cases s- t Graph The main Features of Graph v = constant (iii) Slope of v-t graph = a = 0

At rest s Slope = v = 0 t
Uniformly v So slope of v-t graph is
accelerated constant u = 0 i.e.
motion with v = at so, a = constant u = 0
t u = 0 and s = 0 i.e. v = 0 at t = 0
at t = 0
Uniform motion s Slope = constant, t
v = constant
Uniformly v Positive constant
a=0 accelerated acceleration because θ is
s =νt v = u + at
motion with u constant and <90º but the
t u ≠ 0 but s = 0 initial velocity of the particle
at t = 0 is positive
Uniformly s u = 0, i.e. t
accelerated Slope of s-t graph v
Uniformly Slope of v-t graphs = – a
motion with 1
s = at 2 at t = 0, should be zero. decelerated (retardation)
u = 0, s = 0 at 2 u
motion
t =0 v = u – at
t

Uniformly s Slope of s-t graph gradually t


t0
accelerated goes on increasing
motion with 1 Non-uniformly v Slope of v-t graph increases
s =ut + at 2 accelerated with time.
u ≠ 0 but s = 0 2
at t = 0 motion θ is increasing, so,
t acceleration is increasing
t
Uniformly s θ is decreasing
retarded motion so, v is decreasing, a is Non-uniformly v θ is decreasing, so
negative decelerating acceleration decreasing
motion
t
t0 t
DAY TWO KINEMATICS 13

DAY PRACTICE SESSION 1

FOUNDATION QUESTIONS EXERCISE


1 A wheel of radius 1m rolls forward half a revolution on a 8 The velocity of a particle at an instant is10 ms −1 . After
horizontal ground. The magnitude of the displacement of 3 s its velocity will become16 ms −1. The velocity at 2 s
the point of the wheel initially in contact with the ground before the given instant, will be
is (a) 6 ms −1 (b) 4 ms −1 (c) 2 ms −1 (d) 1 ms −1
(a) 2 π (b) 2 x (c) π2 + 4 (d) π 9 An object moves, starting from rest through a resistive
2 In one dimensional motion, instantaneous speed v medium such that its acceleration is related to velocity
statisfies 0 ≤ v ≤ v 0. as, a = 3 − 2v . Then,
(a) The displacement x in time T must always take (a) the terminal velocity is 1.5 unit
non-negative values (b) the terminal velocity is 3 unit
(c) the slope of a-v graph is not constant
(b) The displacement x in time T satisfies −v 0T < x < v 0 T
(d) initial acceleration is 2 unit
(c) The acceleration is always a non-negative number
(d) The motion has no turning point 10 A particle moves in a straight line with a constant
acceleration. It changes its velocity from 10 ms −1 to
3 A particle moving in a straight line covers half the 20 ms −1, while passing through a distance 135 m in
distance with speed of 3 m/s. The other half of the t second. The value of t (in second) is
distance is covered in two equal time intervals and with (a) 12 (b) 9
speeds of 4.5 m/s and 7.5 m/s, respectively. The (c) 10 (d) 1.8
average speed of the particle during this motion is
11 A particle moving in a straight line with uniform
(a) 4 m/s (b) 5 m/s (c) 5.5 m/s (d) 4.8 m/s acceleration is observed to be a distance a from a fixed
4 Preeti reached the metro station and found that the point initially. It is at distance b, c, d from the same point
escalator was not working. She walked up the stationary after n, 2n, 3 n seconds. The acceleration of the particle is
c − 2b + a c+b+a
escalator in time t1. On other days, if she remains (a) (b)
n2 9n 2
stationary on the moving escalator, then the escalator
c + 2b + a c −b + a
takes her up in time t 2 . The time taken by her to walk up (c) (d)
on the moving escalator will be j
NEET 2017 4n 2 n2
t1 + t 2 t1t 2 t1t 2 12 A car accelerates from rest at constant rate for first 10 s
(a) (b) (c) (d) t1 − t 2
2 t 2 − t1 t 2 + t1 and covers a distance x. It covers a distance y in next
10 s at the same acceleration. Which of the following is
5 A body travelling along a straight line traversed one-third true?
of the total distance with a velocity 4 ms −1. The remaining (a) x = 3 y (b) y = 3 x (c) x = y (d) y = 2 x
part of the distance was covered with a velocity 2 ms −1
for half the time and with velocity 6ms −1 for the other half 13 A car starts from rest, moves with an acceleration a and
of time. The average velocity over the whole time of then decelerates at a constant rate b for sometime to
motion is come to rest. If the total time taken is t. The maximum
velocity of car is given by
(a) 5 ms −1 (b) 4 ms −1 (c) 4.5 ms −1 (d) 3.5 ms −1
abt a 2t
6 A body is moving with velocity 30 m/s towards East. After (a) (b)
(a + b) (a + b)
10 s its velocity becomes 40 m/s towards North. The
at b 2t
average acceleration of body is (c) (d)
(a + b) (a + b)
(a) 7 m/s 2 (b) 7 m/s 2 (c) 5 m/s 2 (d) 1 m/s 2
14 A particle starts its motion from rest such that its velocity
7 A body initially at rest is moving with uniform acceleration
2 remains constant. If the distance covered in first 10 s is
a m/s . Its velocity after n second is v. The displacement
s1 and that covered in the first 20 s is s2 , then
of the body in last 2 s is
(a) s 2 = 2s1 (b) s 2 = 3s1 (c) s 2 = 4s1 (d) s 2 = s1
2v (n − 1) v (n − 1)
(a) (b)
n n 15 A bullet loses 1/20 of its velocity after penetrating a
v (n + 1) 2v (2n + 1) plank. How many planks are required to stop the bullet?
(c) (d)
n n (a) 6 (b) 9 (c) 11 (d) 13
14 40 DAYS ~ NEET PHYSICS DAY TWO

16 A man throws balls with the same speed vertically (c) non-uniform accelerated motion
upwards one after the other at an interval of 2 s. What (d) Nothing can be said about the motion
should be the speed of the throw, so that more than two 24 A graph of x versus t is shown in figure. Choose incorrect
balls are in the sky at any time? (Given, g = 9.8 ms −2 ) statements from below.
(a)Any speed less than 19.6 ms −1 X
(b)Only with speed 19.6 ms −1
B C
(c)Greater than 19.6 ms −1 A
E
(d)At least 9.8 ms −1
17 A stone falls freely under gravity. It covers distances
h1, h2 and h3 in the first 5 s, the next 5 s and the next 5 s, t
respectively. The relation between h1, h2 and h3 is D
h2 h3
(a) h1 = 2 h2 = 3 h3 (b) h1 = = j NEET 2013 (a) The particle was released from rest at t = 0
3 5
(b) At B, the acceleration a > 0
(c)h2 = 3 h1 and h3 = 3 h2 (d) h1 = h2 = h3
(c) At C, the velocity and the acceleration
18 A boy standing at top of a tower of 20 m height drops a (d) The speed at D exceeds than at E
stone. Assuming g = 10 ms −2 , the velocity with which it 25 A particle shows distance-time curve as given in this
hits the ground is j CBSE AIPMT 2011
figure. The maximum instantaneous velocity of the
(a) 20 m/s (b) 40 m/s (c) 5 m/s (d) 10 m/s particle is around the point
19 A ball is dropped from a high rise platform at t = 0 x
D
starting from rest. After 6 s another ball is thrown

Distance
C
downwards from the same platform with speed v. The
two balls meet at t = 18 s. What is the value of v ? B
j
CBSE AIPMT 2010 A
Time
(a) 74 ms −1 (b) 55 ms −1
(c) 40 ms −1 (d) 60 ms −1 (a) B (b) C
(c) D (d) A
20 A body is thrown vertically up with a velocity u. It passes
26 The position-time graph for a uniform motion is
three points A, B and C in its upward journey with
u u u represented as
velocity , and , respectively. The ratio of separations
2 3 4
x x
between points AB and between BC, i.e. AB is (a) (b)
BC
10 20 t t
(a) 1 (b) 2 (c) (d)
7 7
21 When a ball is thrown up vertically with velocity v 0, it x x
reaches a maximum height of h. If one wishes to triple the (c) (d)
maximum height, then the ball should be thrown with
t t
velocity
(a) 3 v 0 (b) 3 v 0 (c) 9 v 0 (d) 3 /2 v 0 27 Among the four graphs, there is only one graph for which
average velocity over the time interval ( 0 ,T ) can vanish for
22 From the top of a tower two stones, whose masses are in
suitably chosen T. Which one is it?
the ratio 1 : 2 are thrown on straight up with an initial
x
speed u and the second straight down with the same x
speed u. Then, neglecting air resistance,
(a) (b)
(a) the heavier stone hits the ground with a higher speed t
(b) the lighter stone hits the ground with a higher speed
(c) both the stones will have the same speed when they hit x x t
the ground
(d) the speed can not be determined with the given data
(c) (d)
23 The velocity-time graph of particle comes out to be a
non-linear curve. The motion is
(a) uniform velocity motion t t
(b) uniformly accelerated motion
DAY TWO KINEMATICS 15

28 A toy car with charge q moves on a frictionless horizontal 32 A particle of unit mass undergoes one-dimensional
plane surface under the influence of a uniform electric motion such that its velocity varies according to
field E . Due to the force qE , its velocity increases from 0
v ( x ) = β x −2 n
to 6 m/s in are second duration. At that instant, the
direction of the field is reversed. The car continues to where, β and n are constants and x is the position of the
move for two more second under the influence of this particle. The acceleration of the particle as a function of
field. The average velocity and the average speed of the x, is given by j CBSE AIPMT 2015

toy car between 0 to 3 seconds are respectively. (a) −2nβ 2 x −2n − 1 (b) −2nβ 2 x −4n − 1
j NEET 2018 (c) −2β 2 x −2n + 1 (d) −2nβ 2e −4n + 1
(a) 1 m/s, 3.5 m/s (b) 1 m/s, 3 m/s 33 A particle moves a distance x in time t according to
(c)2 m/s, 4 m/s (d) 1.5 m/s, 3 m/s equation x = (t + 5)−1. The acceleration of particle is
29 A particle moving along X -axis has acceleration f , at time proportional to j CBSE AIPMT 2010

 t (a)(velocity) 3 / 2 (b) (distance) 2


t given by f = f0 1 −  , where f0 and T are constants.
 T (c)(distance) −2 (d) (velocity) 2 / 3
The particle at t = 0 has zero velocity. In the time interval 34 If the velocity of a particle is v = At + Bt 2 , where A and B
between t = 0 and the instant when f = 0 , the particle’s are constants, then the distance travelled by it between
velocity (v x ) is 1s and 2s is j NEET 2016

1 2 1
(a) f0T (b) f0T (c) f0T 2 (d) f0T (a) 3 A + 7B (b)
3 7
A+ B
2 2 2 3
30 The position x of a particle with respect to time t along A B 3
(c) + (d) A + 4B
X -axis is given by x = 9 t 2 − t 3 , where x is in metre and t 2 3 2
in second. What will be the position of this particle when 35 Two cars P and Q start from a point at the same time in a
it achieves maximum speed along the + x direction? straight line and their positions are represented by
(a) 32 m (b) 54 m (c) 81 m (d) 24 m X P (t ) = at + bt 2 and X Q (t ) = ft − t 2 . At what time do the
31 An object moving with a speed of 6.25 m/s, is cars have the same velocity? j
NEET 2016
dv a −f a+f
deaccelerated at a rate given by = −2.5 v , where v is (a) (b)
dt 1+ b 2 (b − 1)
the instantaneous speed. The time taken by the object, to a+f f −a
(c) (d)
come to rest would be 2 (1 + b) 2 (1 + b)
(a) 2 s (b) 4 s (c) 8 s (d) 1s

DAY PRACTICE SESSION 2

PROGRESSIVE QUESTIONS EXERCISE


1 The motion of a particle along a straight line is described
by equation, x = 8 + 12t − t 3 , where x is in metre and t in 4
Velocity (m/s)

sec. The retardation of the particle when its velocity 2


0
becomes zero, is 2 4 8
−2 −2 −2
–2
(a) 24 ms (b) zero (c) 6 ms (d) 12 ms t (s)
–4
2 A body is thrown vertically upward in air when air
resistance is taken into account, the time of ascent is t1 (a)12 m, 20 m (b) 20 m, 12 m
and time of descent is t 2 , then which of the following is (c)12 m , 12 m (d) 20 m, 20 m
true? 4 Two bodies A (of mass 1 kg) and B (of mass 3 kg) are
(a) t1 = t 2 (b) t1 < t 2 (c) t1 > t 2 (d) t1 ≥ t 2 dropped from heights of 16 m and 25 m, respectively.
3 A body is moving in a straight line as shown in The ratio of the time taken by them to reach the ground is
velocity-time graph. The displacement and distance (a) 5/4 (b) 12/5
(c) 5/12 (d) 4/5
travelled by body in 8 s are respectively
16 40 DAYS ~ NEET PHYSICS DAY TWO

9 Then, match the following columns and choose the


5 A balloon rises from rest with a constant acceleration .
8 correct option from the codes given below.
A stone is released from it when it has risen to height h.
Column I Column II
The time taken by the stone to reach the ground is
A. Part OA of graph 1. Positive velocity
h h
(a) 4 (b) 2 B. Part AB of graph 2. Object at rest
g g
C. Part BC of graph 3. Negative velocity
2h g
(c) (d) D. Point ‘A’ in the graph 4. Change in direction of motion
g h
A B C D
6 The ratio of distance traversed in successive intervals of (a) 1 2 3 4
time when a body falls freely under gravity from certain (b) 1 3 2 4
height is (c) 2 1 3 4
(d) 4 3 2 1
(a) 1 : 2 : 3 (b) 1 : 5 : 9
(c) 1 : 3 : 5 (d) 1 : 2 : 3 10 The displacement x of a particle varies with time t as
x = ae − αt + beβt , where a, b, α and β are positive
7 Figure shows the time-displacement curve of the particles
P and Q . Which of the following statement is correct? constants. The velocity of the particle will
(a)decrease with time (b) be independent of α and β
x P
Q (c)drop to zero when α = β (d) increase with time
11 A particle is released from rest from a tower of height 3 h.
The ratio of times to fall equal height h, i.e. t1 : t 2 : t 3 is
O t
(a) 3 : 2 : 1 (b) 3 : 2 : 1
(c)9 : 4 : 1 (d) 1 : ( 2 − 1) : ( 3 − 2)
(a) Both P and Q move with uniform equal speed
(b) P is accelerated and Q moves with uniform speed, but 12 A train accelerates from rest at a constant rate α for
the speed of P is more than the speed of Q distance x1 and time t1, it retards to rest at constant rate β
(c) Both P and Q moves with uniform speeds but the speed for distance x 2 and time t 2 . Which of the relation is
of P is more than the speed of Q correct?
(d) Both P and Q moves with uniform speeds but the speed x1 α t1 x1 β t1
(a) = = (b) = =
of Q is more than the speed of P x2 β t 2 x2 α t 2
x α t x β t
8 The velocity (v ) of a particle moving along X -axis varies (c) 1 = = 2 (d) 1 = = 2
with its position x as shown in figure. The acceleration (a ) x 2 β t1 x 2 α t1
of particle varies with position ( x ) as 13 A dust packet is dropped from 5th storey of a
–1
v (ms ) multi-storeyed building. In the first second of its free fall
another dust packet is dropped from 7th storey 15 m
4 below the 9th storey. If both packets reach the ground at
same time, then height of the building is
(a) 25 m (b) 15 m (c) 20 m (d) 16 m
14 A ball is dropped into a well in which the water level is at
x (m) a depth h below the top. If the speed of sound is c, then
O 2
the time after which the splash is heard will be given by
(a) a 2 = x + 3 (b) a = 2 x 2 + 4  2 1  2 1  2 1  2 1
(c) 2a = 3 x + 5 (d) a = 4 x − 8 (a) h  +  (b) h  −  (c) h  +  (d) h  − 
 gh c   gh c  g c  g c 
9 Given graph (x-t ) representing the motion of an object,
match the terms of Column I with the items of Column II 15 The displacement-time graph of a moving particle with
and choose the correct options from the codes given constant acceleration is shown in figure. The
below velocity-time graph is given by
A x (m)

x 5
B C

O t (s)
t 0 1 2
DAY TWO KINEMATICS 17

v v 17 From a tower of height H, a particle is thrown vertically


upwards with a speed u. The time taken by the particle to
(a) (b) hit the ground, is n times that taken by it to reach the
highest point of its path. The relation among H, u and n is
O 1 2 t O 1 2 t (a) 2 gH = n 2u 2 (b) gH = (n − 2)2 u 2
(c) 2 gH = nu 2 (n − 2) (d) gH = (n − 2) u 2
18 A train is moving along a straight path with uniform
v v
acceleration. Its engine passes across a pole with a
(c) (d) velocity of 60 kmh −1 and the end (guard’s van) passes
across same pole with a velocity of 80 kmh −1. The middle
point of the train will pass across same pole with a
O 1 2 t O 1 2 t
velocity
(a) 70 kmh−1 (b) 70.7 kmh−1
16 Water drops fall from a top on the floor 5 m below at (c) 65 kmh−1 (d) 75 kmh−1
regular intervals. The fifth drop is leaving the top at the 19 A ball rolls off the top of stair way with a horizontal
instant, the first strikes the ground. The height at which velocity of magnitude 1.8 m/s. The steps are 0.20 m high
the third drop will be from ground at that instant is and 0.20 m wide. Which step will the ball hit first?
(take, g = 10 ms 2 ) (a) First (b) Second
(a) 1.25 m (b) 2.15 m (c) 2.75 m (d) 3.75 m (c) Third (d) Fourth

ANSWERS
SESSION 1
SESSION 1 1 (c) 2 (b) 3 (a) 4 (c) 5 (b) 6 (c) 7 (a) 8 (a) 9 (a) 10 (b)
11 (a) 12 (b) 13 (a) 14 (c) 15 (b) 16 (c) 17 (b) 18 (a) 19 (a) 20 (d)
21 (a) 22 (c) 23 (c) 24 (b) 25 (b) 26 (a) 27 (b) 28 (b) 29 (d) 30 (b)
31 (a) 32 (b) 33 (a) 34 (b) 35 (d)
SESSION 2
SESSION 2 1 (d) 2 (b) 3 (a) 4 (d) 5 (b) 6 (c) 7 (c) 8 (d) 9 (b) 10 (d)
11 (d) 12 (b) 13 (c) 14 (a) 15 (d) 16 (d) 17 (c) 18 (b) 19 (d)

Hints and Explanations


SESSION 1 = AA ′ 3 If t 1 and 2t 2 are the time taken by the
1 Horizontal distance covered by the = ( πR2 ) + (2R )2 particle to cover first and second half
wheel in half revolution πR. distance, respectively.
=R π + 4
2
x /2 x
t1 = =
A′ Final 3 6
= π2 + 4 [QR = 1 m]
Clearly x1 = 4.5t 2 and x2 = 7.5t 2
O
2R 2 The maximum distance covered in x
So, x1 + x2 = = 4.5t 2 + 7.5t 2
R time T is v 0 T . 2
x
A
πR
Therefore, for the object having or t2 =
one-dimensional motion, the 24
x x x
So, the displacement of the point displacement x in time T satisfies Total time, t = t 1 + 2t 2 = + =
6 12 4
which was initially in contact with − v 0 T < x< v 0 T .
ground So, average speed = 4 m/s
18 40 DAYS ~ JEE MAIN PHYSICS DAY TWO

4 Speed of walking = h = v 1 9 The meaning of terminal velocity is 13 By equation of motion, we have


t1 constant velocity. For constant velocity, v
h acceleration should be zero (i.e. a = 0).
Speed of escalator = = v2
t2 Q a = 3 − 2v
Time taken when she walks over or 0 = 3 − 2v
3 a b
running escalator ∴Velocity, v = = 1.5 unit
h 2
⇒t = Car comes to rest
v1 + v2 10 From equation of motion, t
1 v
⇒ = 1 + 2 =
v 1
+
1 t t
⇒t = 1 2 v 2 = u2 + 2as Since, body starts from rest u = 0.
t h h t1 t2 t1 + t2 v 2 − u2 Let t 1 be time when body accelerates
⇒ a=
2s and t 2 when it decelerates.
5 Let total distance be s and the time taken
to cover first one-third distance as t 1 , (20)2 − (10)2 300 10 ∴ t = t1 + t2 ⇒ t2 = t − t1
= = = ms −2 ∴ v = 0 + at 1 = at 1 … (i)
s /3 s 2 × 135 270 9
then t 1 = =
4 12 From first equation of motion, When car finally comes to rest, v = 0
Now, let t 2 be the time for the rest two v = u + at ∴ 0 = v − b(t − t 1 ) … (ii)
journeys. Then, v − u 20 − 10 10 From Eqs. (i) and (ii), we get
2s ⇒ t = = = = 9s b ab
= 2t 2 + 6t 2 = 8t 2 a 10 / 9 10 / 9 t1 = t and v = t
3 (a + b ) (a + b )
2s s 11 Using equation of motion,
∴ t2 = = 14 We know that from second equation of
1 2
24 12 s = ut + at , we get motion
∴ Average velocity 2
1 2
Total displacement s = ut + at …(i)
= t=0 t=n t = 2n 2
Total time
a Given distance, s = s1 in first 10 sec
=
s
=
s b and distance, s = s2 in first 20 sec.
t1 + 2 t2 s
+
s c and u = 0
12 6 1
b − a = un +
1
An2 So, s1 = a(t 1 )2 …(ii)
12 × 6 2
= = 4 ms −1 2
1
12 + 6 ⇒ 2b − 2a = 2un + An2 …(i) s2 = a(t 2 )2 …(iii)
2
6 Average acceleration 1
Again, c − a = u(2n ) + A(2n ) 2
…(ii) From Eqn. (ii) and (iii), we get
Change in velocity 2 2
= s1  t 1 
2
=   =   =
10 1
Total time On substracting, Eq. (i) from Eq. (ii), we ⇒ s2 = 4s1 .
get s2  t 2   20  4
|v f − v i|
a= , c − a − 2b + 2a = An2
∆t 15 The final velocity after it passes the
c − 2b + a 19u
A= plank is
(30) + (40)
2 2
n2 20
a=
10 Bullet
12 From equation of motion, we have
900 + 1600 s = ut +
1 2
at where, u is initial
=
10 2 19u
u
velocity, t is time and a is acceleration. 20
2500
= = 5 ms −2 Since, car accelerates from rest
10 Let x be the thickness of the plank, the
u = 0, t = 10 s deacceleration due to resistance of plank
7 Displacement in last 2 s 1
∴ s = 0 + × a × (10) 2 = 50a …(i) is given by v 2 = u2 + 2 as
1 2 1 2
= an − a (n − 2) 2 where, v is final velocity, u is initial
2 2 Also, v = u + at
velocity, a is acceleration and s is
= 2a (n − 1) where, v is final velocity.
displacement.
v ∴Velocity after 10 s is 19
Acceleration, a = [Qt = n second ] v = 0 + a × 10 Here, v = u
n 20
s
2v (n − 1) v = 10a = 10 × …(ii) 2
−39 2
∴ 
Displacement in last 2 s = 19 
50 u  = u2 + 2ax ⇒ 2ax = u
n  20  400
In the next 10 s car moves with constant
8 Using equation of motion, acceleration and with initial velocity v. Suppose the bullet is stopped after
v = u + at , we get 1 passing through n such planks. Then,
16 = 10 + 3 a ∴ s ′ = vt + at 2 the distance covered by bullet is nx.
2
⇒ a = 2 ms −2 2
0 =   u2 + 2 anx
s 1 s 19
= × 10 × 10 + × × 100 = 3 s ∴
and 10 = u + 2 × 2  20 
50 2 50
[u = required velocity] 2
−39 2
Given, s = x and s ′ = y ⇒ −   u2 = n ×
19 361
⇒ u = 6 ms −1 u ⇒n = ≈9
∴ y = 3x  20  400 39
DAY TWO KINEMATICS 19

2
16 Time taken by ball to reach maximum 20 Here, for point A = u − u2 = − 2gh1 25 At instant instantaneous velocity of
height v = u − gt 4 particle is given by
At maximum height, final speed is zero u2 ds
for point B = − u2 = − 2gh2 v = = tanθ
i.e. v = 0 9 dt
So, u = gt u2 From figure shows that slop of curve is
In 2s, u = 2 × 9.8 = 19.6 ms −1 and for point C = − u2 = − 2gh3 maximum at point C.
16
If man throws the ball with velocity of C
26 For object at rest, the position-time
19.6 ms −1 , then after 2 s it will reach the graph is a straight line parallel to the
maximum height. When he throws B time axis.
second ball, first is at top. When he h 3 h2 x (m)
throws third ball, first will come to A
ground and second will at the top. h1 40

Therefore, only 2 balls are in air. If he 20


wants to keep more than 2 balls in air, u2  8 3  u2 5
∴ AB = − = ⋅
he should throw the ball with a speed 2g  9 4  2g 36 0
greater than 19.6 ms −1 . 10 20 30 40 t (s)
u2  15 8  u2 7
17 For free fall from a height, u = 0 BC = − = ⋅
2g  16 9  2g 144 (a)
Distance covered by stone in first 5s,
AB 5 144 20
1 25 ∴ = × = 27 In graph (b), one value of displacement
h1 = 0 + g (5)2 = g …(i) AC 36 7 7
2 2 is obtained at two different time.
Distance covered in first 10s, 21 We have, v 2 = u2 − 2 gh Thus, the average displacement of
1 100 Here, v = 0, u = v 0 particle is zero. As average velocity
s2 = 0 + g (10)2 = g Displacement
2 2
∴ 0 = v 20 − 2gh ⇒ v 0 = 2gh =
∴Distance covered in second 5s Time
100g 25g 75g When h ′ = 3 h, then So, average velocity would be zero.
h2 = s2 − h1 = − = …(ii)
2 2 2 v ′0 = 2g × 3 h = 3 2gh = 3 v 0 28 Given condition can be represented
Distance covered in first 15s, through graph also as shown below.
1
s3 = 0 + g (15)2 =
225
g
22 u
2 2 1 v
A
Distance covered in last 5s, 2 (ms–1) + 6
225 100 125
h3 = s3 − s2 = g− g = g h
2 2 2 O′ t=2 t=3
…(iii) O t=1 D t(s)
Ground B
From Eqs. (i), (ii) and (iii) we get
25 g 75 g 125 g Applying equation of motion for both
h1 : h2 : h3 = : : = 1:3: 5
2 2 2 particles 1 and 2 –6
C
h h For particle 1,
⇒ h1 = 2 = 3
3 5 v 2 = u2 + 2as ⇒ v 12 = u2 − 2gh … (i) ∴Displacement in three seconds
For particle 2, v 22 = u2 − 2gh … (ii) = Area under the graph
18 From the equation of motion
= Area of ∆OAO ′ + Area of
v 2 = u2 + 2gh Hence, v 1 = v 2
∆AO ′B − Area of ∆BCD
Given, u = 0, Both the stones will have same speed
1 1 1
∴ v = 2gh when they hit the ground. = ×1× 6+ ×1× 6− × 6×1
2 2 2
Given, g = 10 ms −2 , h = 20 m 23 Velocity-time graph gives the = 3m
−1 instantaneous value of velocity at any
v = 2gh = 2 × 10 × 20 = 20 ms 3
∴ Average velocity = = 1 ms −1 .
instant. For non-uniformly accelerated
3
19 For first ball, u = 0 motion, v-t graph is non-linear.
Total distance travelled, d = 9 m
s1 = ut +
1 2 1 2 1
gt = gt 1 = g(18)2 24 From graph, when t = 0, the particle is 9
2 2 2 released from rest at A, hence, v = 0. At ∴ Average speed = = 3 ms −1
1 3
For second ball, s2 = vt 2 + gt 2 B, the graph is parallel to time axis,
2 hence velocity is constant there. The 29 Acceleration,
⇒ t 2 = 18 − 6 = 12s
= f 0  1 − 
acceleration a is zero. dv t
f =

1
s2 = v × 12 + g (12)2 At C, the graph changes slope, where dt  T
2 velocity and acceleration vanish.
dv = f 0  1 −  dt
t
or …(i)
Here, s1 = s2 Average velocity for motion between A  T
1 1
g (18)2 = 12v + g (12)2 and D is negative, because the value of x On integrating Eq. (i) both sides, we get
2 2 is decreasing with time t. The slope of f t2
⇒ − 74 ms −1
v = 73.5~ graph (which represents speed) is more ∴ v = f0 t − 0 ⋅ +C …(ii)
T 2
at D than at E.
20 40 DAYS ~ JEE MAIN PHYSICS DAY TWO

After applying boundary conditions v = 0 Integrating both sides, we get u g + a0


⇒ t2 = = t1
at t = 0, we get x2 2

C=0
∫x 1
dx = ∫1 ( At + Bt 2 ) dt ( g + a0 ) ( g − a0 ) g − a0
2 2 2 t2 g + a0

f t2
v = f0 t − 0 ⋅ …(iii)
⇒ ∆x = x2 − x1 = A ∫1 t dt + B ∫1 t dt i.e.
t1
=
g − a0
> 1 ⇒ t2 > t1
T 2 2 2
t 2  t 3 
Given, f = f 0  1 − 
t = A  + B   3 Displacement
 T  2 1  3 1 = (2 × 4 − 2 × 2 + 2 × 4) = 12 m
Substituting, f = 0, t = T in Eq. (iii), =
A 2
(2 − 1 ) +
2 B 3
(2 − 13 ) Distance = 2 × 4 + 2 × 2 + 2 × 4 = 20 m
then velocity, 2 3
f T2 1 ∴ Distance travelled between 1s and 2s is 4 For a freely falling body,
v x = f 0T − 0 ⋅ = f 0T A B 1 2
T 2 2 ∆x = × (3) + (7) h= gt
2 3 2
30 Given, x = 9t 2 − t 3 …(i) 3 7 2
= A+ B h1  t 1 
Velocity, v =
dx d
= ( 9t 2 − t 3 ) 2 3 ∴ = 
dt dt h2  t 2 
35 Velocity of each car is given by Given, h1 = 16 m, h2 = 25 m
= 18t − 3t 2 Q x = nx n − 1 
d n dX p (t )
 dx  vP = = a + 2bt t1 h1 16 4
dt ∴ = = =
Also, Acceleration t2 h2 25 5
dX Q (t )
dv d and v Q = = f − 2t
a= = (18 t − 3t 2 ) = 18 − 6 t …(ii) dt 5 From equation, v 2 = u2 + 2as
dt dt
It is given that v P = v Q Given,
9
u = 0, a =
Now, speed of particle is maximum, ⇒ a + 2bt = f − 2t 8
when its acceleration is zero, i.e. a = 0 f −a
We have, v = 2   h
9
i.e. 18 − 6 t = 0 or t = 3 s ⇒ t =
2 (b + 1)  8
Putting in Eq. (i), we obtain position of When the stone released from this
particle at the time SESSION 2 balloon. It will go upward with velocity
gh
x = 9(3) 2 − (3) 3 = 54m v =
1 Given, x = 8 + 12t − t 3 2
31 Given, dv = − 2.5 v We know In this condition, time taken by stone to
dt dx dv
dv v= and acceleration a = reach the ground
⇒ = − 2.5 dt dt dt
v v  2gh 
So, v = 12 − 3t 2 and a = − 6t t = 1 + 1+ 
0 −1 /2 t g  v2 
⇒ ∫6.25 v dv = − 2.5 ∫ dt
0
At t = 2 s, v = 0 and a = − 6 × 2
⇒ −2.5 [t ] t
= [2v 1 /2 ] 06.25 a = − 12 m/s2 gh / 2  2gh 
= 1 + 1+
gh / 4 
0
So, retardation of the particle g 
⇒ Time t = 2 s
= 12 m/s 2
32 a = ∂v = ∂v × ∂x = ∂v × v
2 gh h
= =2
∂t ∂x ∂t ∂x 2 First of all note that, air resistance acts g g
We have v = βx −2 n in direction opposite to the motion of
∂v body. So, when a body is thrown up, 6 The distance, x = ut + 1 at 2
= − 2nβx −2 n − 1 2
∂x then both the deaccelerating forces, i.e.
gravity and air resistance act in same For free fall starting from rest,
Acceleration,
1
a = (−2nβx −2 n − 1 ) × ( βx −2 n ) direction. Thus, total deacceleration is u = 0, a = g ⇒ x = gt 2
2
a1 = g + a0 , where a0 is deacceleration
a = − 2nβ2 x −4 n − 1
due to air resistance which is assumed t 0 T 2T 3T
33 Given, x = ( t + 5)−1 to be constant. g 2 g g
x 0 T ( 4T 2 ) ( 9T 2 )
dx −1 If u be the initial velocity and t 1 be the 2 2 2
∴ v = = …(i)
dt ( t + 5) 2 time of ascent, then
u ∆t 0 to T T to 2T 2T to 3T
d2 x 2 t1 = or u = t 1 (g + a0 )
a= = …(ii) g + a0 ∆x
g 2
T
g
( 4 − 1) T 2
g g
( 9 − 4) T 2 = 5T 2
dt 2 ( t + 5) 3 2 2 2 2
u2
On comparing Eqs. (i) and (ii), we get and h= …(i)
2(g + a 0 ) ∴ Required ratio,
a ∝ (v )3 /2 g 2 g g
Also, t 2 is time of descent, then T : 3T 2 : 5T 2 = 1 : 3 : 5
34 Velocity of the particle is given as 1 2 2 2
h = ut + a2 t 22
v = At + Bt 2 2 7 As x-t graph is a straight line in either
where A and B are constants. As, a2 = g − a0 , during descent case, velocity of both is uniform. As the
dx Q v = dx  u2 1 slope of x - t graph for P is greater,
⇒ = At + Bt 2 ⇒ = 0 + × [g − a0 ] × t 22
dt  dt  2( g + a0 ) 2 therefore velocity of P is greater than
that of Q.
⇒ dx = ( At + Bt 2 ) dt [using Eq. (i)]
DAY TWO KINEMATICS 21

8 Given velocity (v )-position ( x ) graph is Similarly, v 2 = β t 2 Hence, the interval of falling of each
α t2 1
shown in figure. As, α t 1 = βt 2 ⇒ = …(i) water drop is = 0.25 s
β t1 4
v (ms–1) When the fifth drop starts its journey
1
x1 = 0 (t 1 ) + α (t 1 )2 …(ii) towards ground, the third drop would be
4 2 in air for 0.25 + 0.25 = 0.5 s
1
x2 = (αt 1 ) t 2 + (− β ) (t 2 )2 Height (distance) covered by third drop
2 in air is
1 1 1
x2 = ( βt 2 ) (t 2 ) − β (t 2 )2 h1 = gt 2 = × 10 × (0.5)2
2 2 2
x (m) [using Eq. (i)] = 5 × 0.25 = 1.25
O 2 1 Therefore, the third water drop will be
⇒ x2 = βt 22 …(iii)
We know that, acceleration, 2 at a height = 5 − 1.25 = 3.75 m
vdv From Eqs. (ii) and (iii), we get 17 At highest point of path, velocity = 0
a= … (i)
dx x1 αt 12  αt 1   β  t 1 u
= =    = …(iv) ⇒ 0 = u − gt ⇒ t =
From the graph, we have
x v
x2 βt 22  βt 2   α  t 2 g
+ = 1 ⇒ 2x + v = 4 … (ii) [using Eq. (i)] Given, time for the particle to hit the
2 4 From Eqs. (i) and (iv), we get ground = nt …(i)
dv
⇒ = −2 … (iii) α t2 x 1
dx = = 2 Now, − H = u (nt ) − g (nt )2
β t1 x1 2
From Eqs. (i), (ii) and (iii), we get  u  1 2 u2
β t1 x x1 β t ⇒ − H = un   − gn
a = (4 − 2 x ) (− 2) = − 8 + 4x ⇒ = = 1 ⇒ = = 1
α t2 x2 x2 α t 2 g 2 g2
⇒ a = 4x − 8
13 From the relation, u2 n2 u2
9 In (x - t ) graph OA → Positive slope → ⇒ −H =n −
1 2 g 2 g
h= gt …(i)
Positive velocity 2 ⇒ − 2gH = 2nu2 − n2u2
AB → Negative slope → Negative velocity 1
h − 15 = g (t − 1) 2 …(ii)
BC → Zero slope → Object at rest 2 ⇒ 2gH = n2u2 − 2nu2
1 2 1 ⇒ 2gH = nu2 (n − 2)
A gt − 15 = g (t − 1) 2
2 2
x [from Eq. (i)] 18 From v 2 − u2 = 2as, we get
1
C g [ t 2 − ( t − 1) 2 ] = 15 (80)2 − (60)2
B 2 =s
15 × 2 2a
(t + t − 1) (t − t + 1) = =3 6400 − 3600 1400
g ∴ Distance, s = =
O 2a a
t [Qg = 10 m/s2 ] The middle point of the train is to cover
At point ‘A’, there is change in sign of 2t − 1 = 3 ⇒ t = 2 s a distance
velocity, hence the direction of motion 1 s 700
∴ h = × 10 × 2 × 2 = 20 m =
must have changed at ‘A’. 2 2 a
10 Given, x = ae −αt + be βt 14 Time of fall = 2h Again, from v 2 − u2 = 2as,
g 700
dx v 2 − (60)2 = 2a × = 1400
Velocity, v =
dt Time taken by the sound to come out a
h v = 1400 + 3600
2
= − aαe − αt + bβe βt = A + B =
c
where, A = − aαe − αt , B = bβe βt  2 Velocity, v = 5000 = 70.7 kmh − 1
2h h 1
Total time = + =h + 
The value of term A = − aαe − αt g c  gh c 19 Let ball strike the nth step of stairs.
decreases and B = bβe βt increases with
time. As a result, velocity goes on 15 From the given graph, for 0 < t < 1s, Vertical distance travelled
1 2
increasing with time. slope of x-t graph is decreasing, this = ny = n × 0.20 = gt
implies v (velocity) is increasing. 2
11 h = 1 g t 12 ⇒ 2h = 1 g ( t 1 + t 2 ) 2 For 1 < t < 2s, slope of x-t graph is
2 2 Horizontal distance travelled
1 increasing, this implies v is increasing. = nx = ut
and 3 h = g (t1 + t2 + t3 )2 Thus, the above conditions is only
2 nx
⇒ t =
satisfied by the graph in option (d).
i.e. t 1 : (t 1 + t 2 ) : ( t 1 + t 2 + t 3 ) = 1 : 2 : 3 u
16 By the time fifth water drop starts 2
or t 1 : t 2 : t 3 = 1 : ( 2 − 1) : ( 3 − 2 ) 1 n2 x2
gt = g ⋅   = g
1 2 1 nx
falling, the first water drop reaches the ny =
12 Consider the diagram ground. This means, 2 2  u  2 u2
1 2u2 y
x1, t1 x2, t2 u = 0, h = gt 2 ⇒n =
2 g x
v=0 α β v=0 1
⇒ 5 = × 10 × t 2
2 2 × (1.8)2 × 0.2
= ≈4
Velocity, v 1 = 0 + α t 1 = α t 1 ⇒ t =1s 9.8 × (0.2) 2
DAY THREE

Scalar
and Vector
Learning & Revision for the Day
u Scalar and Vector Quantities u Multiplication or Division of a u Relative velocity
u Laws of Vector Addition Vector by a Scalar u Motion in a Plane
u Substraction of Vectors u Product of Vects u Projectile Motion
u Resolution of vector

Scalar and Vector Quantities


A scalar quantity is one whose specification is completed with its magnitude only.
e.g. mass, distance, speed, energy, etc.
A vector quantity is a quantity that has magnitude as well as direction. Not all physical
quantities have a direction. e.g. velocity, displacement, force, etc.

Position and Displacement Vectors


A vector which gives position of an object with reference to the origin of a coordinate
system is called position vector.
The vector which tells how much and in which direction on object has changed its
position in a given interval of time is called displacement vector.

General Vectors and Notation


Zero Vector The vector having zero magnitude is called zero vector or null vector. It
PREP
l

is written as 0. The initial and final points of a zero vector overlap, so its direction is
arbitrary (not known to us). MIRROR
l
Unit Vector A vector of unit magnitude is known as an unit vector. Unit vector for A Your Personal Preparation Indicator
is A$ (read as A cap). u No. of Questions in Exercises (x)—
A=AA Direction u No. of Questions Attempted (y)—
u No. of Correct Questions (z)—
(Without referring Explanations)
Magnitude
l
Orthogonal Unit Vectors The unit vectors along X -axis,s, Y-axis Y u Accuracy Level (z / y × 100)—
and Z-axis are denoted by $i, $j and k$ . These are the orthogonal unit j u Prep Level (z / x × 100)—
vectors. i
X In order to expect good rank in NEET, your
$i = x , $j = y , k$ = z Accuracy Level should be above 85 & Prep
k Level should be above 75.
x y z Z
DAY THREE SCALAR AND VECTOR 23

l
Parallel Vector Two vectors are said to be parallel, if they Thus, vector subtraction is
have same direction but their magnitudes may or may not really a special case of vector A B
be equal. addition. The geometric
l
Antiparallel Vector Two vectors are said to be anti-parallel construction for subtracting
two vectors is shown in the A – B –B
when
(i) both have opposite direction above figure.
(ii) one vectors is scalar non zero negative multiple of If θ be the angle between A
another vector. and B,
l
Collinear Vector Collinear vector are those which act along then | A − B| = A2 + B2 − 2 AB cos θ
same line.
l
Coplanar Vector Vector which lies on the same plane are If the vectors form a closed n sided polygon with all the sides
called coplanar vector. in the same order, then the resultant is zero.
l
Equal Vectors Two vectors A and B are equal, if they have
the same magnitude and the same direction. Multiplication or Division of a
Laws of Vector Addition Vector by a Scalar
The multiplication or division of a vector by a scalar gives a
1. Triangle Law vector. For example, if vector A is multiplied by the scalar
If two non-zero vectors are represented by the two sides of a number 3, the result, written as 3A, is a vector with a magnitude
triangle taken in same order than the resultant is given by the three times that of A, pointing in the same direction as A. If we
closing side of triangle in opposite order, i.e. multiply vector A by the scalar − 3, the result is − 3 A , a vector
R=A+B with a magnitude three times that of A, pointing in the direction
The resultant R can be calculated as opposite to A (because of the negative sign).
| A + B| = R = A2 + B2 + 2 AB cos θ
Product of Vectors
A +B The two types of products of vectors are given below
B sin θ
R= B
α θ
A B cos θ
Scalar or Dot Product
If resultant R makes an angle α with vector A, then The scalar product of two vectors A and B is defined as the
B sin θ product of magnitudes of A and B multiplied by the cosine of
tan α = smaller angle between them. i.e. A ⋅ B = AB cos θ
A + B cos θ

2. Parallelogram Law Properties of Dot Product


B C l
Dot product or scalar product of two
According to parallelogram law of B
vector addition, if two vector acting vectors gives the scalar two vectors given
on a particle are represented in Q R the scalar quantity.
magnitude and direction by two β l
It is commutative in nature.
adjacent side of a parallelogram, then α i.e. A ⋅ B = B ⋅ A .
the diagonal of the parallelogram P A θ
l
Dot product is distributive over the
represents the magnitude and direction O A
addition of vectors. B cos θ
of the resultant of the two vector acting i.e. A ⋅ (B + C) = A ⋅ B + A ⋅ C Component of
as the particle. $i ⋅ i$ = $j ⋅ $j = k$ ⋅ k$ = 1, because angle vector B along A
l

i.e. R = P+Q between two equal vectors is zero.


Magnitude of the resultant R is given by l
If two vectors A and B are perpendicular vectors, then
| R | = P2 + Q2 + 2 PQ cos θ A ⋅ B = AB cos 90 ° = 0 and $i ⋅ $j = $j ⋅ k$ = k$ ⋅ i$ = 0
Q sin θ P sin θ
tan α = ⇒ tan β =
P + Q cos θ Q + P cos θ The Vector Product
The vector product of A and B, written as A × B, produces a
Subtraction of Vectors third vector C whose magnitude is C = AB sin θ. where, θ is the
Vector subtraction makes use of the definition of the negative smaller of the two angles between A and B.
of a vector. We define the operation A − B as vector − B added Because of the notation, A × B is also known as the cross
to vector A. A − B = A + ( − B) product, and it is spelled as ‘A cross B’.
24 40 DAYS ~ NEET PHYSICS DAY THREE

Properties of Cross Product Relative Velocity


l
Vector or cross product of two vectors gives the vector The time rate of change of relative position of one object with
quantity. respect to another is called relative velocity.
l
Cross product of two vectors does not obey the
commutative law. i.e. A × B ≠ B × A ; Different Cases
Here, A × B = − B × A Case I If both objects A and B move along parallel straight
l
Cross product of two vectors is distributive over the lines in the opposite direction, then relative velocity of B
addition of vectors. w.r.t. A is given as,
A × (B + C) = A × B + A × C vBA = vB − (− v A ) = vB + v A
l
Cross product of two equal vectors is given by A × A = 0 If both objects A and B move along parallel staight lines in the
Similarly, i$ × i$ = (1 × 1 × sin 0 ° ) n$ = 0 same direction, then
$j × $j = (1 × 1 × sin 0 ° ) n$ = 0 v AB = v B − v A
k$ × k$ = (1 × 1 × sin 0 ° ) n$ = 0 Case II Crossing the River To cross the river over shortest
distance, i.e. to cross the river straight, the man should swim
l
Cross product of two perpendicular vectors is given as upstream making an angle θ with OB such that, OB gives the
A × B = ( AB sin 90 ° ) n$ = ( AB) n$ direction of resultant velocity ( vmR ) of velocity of swimmer v M
l
For two vectors A = a $i + a $j + a k$
x y z and velocity of river water v R as shown in figure. Let us
consider
and B = b x i$ + b y $j + b z k$ .
A vR B
$i $j k$
A × B = ax a y az
bx b y bz vm vmR
θ
l
Cross product of vectors $i , $j and k$ are following cyclic rules
as follows $i × $j = k,
$ $j × k$ = $i and k$ × $i = $j O
vR
j In ∆OAB, sin θ = and vmR = v2m − v2R
vm
The time taken to cross the river given by
d d
i ⊕ t1 = =
vmR v − v2
2
m R

Case III To cross the river in possible shortest time The


k man should go along OA. Now, the swimmer will be going
Cyclic representation for unit vectors $i , $j and k$ along OB, which is the direction of resultant velocity v mR of
vm and vR .
NOTE • Vector triple product is given by AB vR
A × (B × C) = B (A ⋅ C) − C (A ⋅ B) In ∆OAB, tan θ = =
OA vm

Resolution of a Vector and vmR = v2m + v2R


The process of splitting of a single Y A vR B
vector into two or more vectors in x
different direction is resolution of a A
vectors. Consider a vector A in the X - d vm –vmR
A
Y plane making an angle θ with the y v θ
X -axis. The X and Y components of
A are Ax and A y respectively. downstream
θ upstream O
O X
Thus A = A = ( A cos θ)i$
x xi Ax
Time of crossing the river,
along X -direction
d OB x2 + d 2
A y = A yj = ( A sin θ)$j along Y-direction t = = =
vm vmR v2m + v2R
From triangle law of vector addition
The boat will be reaching the point B instead of point A. If
|A|= | A xi + A yj| = A2x + A2y
AB = x,
Ay  Ay  v x dv
and tan θ = = θ = tan −1   then, tan θ = R = ⇒ x= R
Ax  Ax  vm d vm
DAY THREE SCALAR AND VECTOR 25

gx2
Motion in a Plane l
Equation of trajectory, y = u
2u2 O X
Let the object be at position A y y
2 B 2h
and B at timing t 1 and t 2 , where A l
Time of flight, T =
y P
OA = r1 , and OB = r2 1 g h x vx
β
Suppose O be the origin for r1 r2 2h
measuring time and position of
l
Horizontal range, R = u vy v
g Y
the object (see figure). A B
O x1 x2 l
Velocity of projectile at any
l
Displacement of an object time, v = u2 + g2t 2
form position A to B is
AB = r = r2 − r1 = ( x2 − x1 )i − ( y2 − y1 ) j
r −r
Projectile Motion Up an Inclined Plane
l
Velocity, v = 2 1 Let a particle be projected
t2 − t1
up with speed u from an Y P
X
in α
l
A particle moving in X -Y plane (with uniform velocity) inclined plane which u gs t=T

then, its equation of motion for X and Y axes are makes an angle α with the a x=
v = v $i + v $j, r = x $i + y $j and r = x$i + y$j horizontal and velocity of ay = g cos α
x y 0 0 0
projection makes an angle θ
α g
x = x0 + vx t , y = y0 + v y t θ with the inclined plane. O t = 0
l
A particle moving in xy-plane (with uniform acceleration),
then its equation of motion for X and Y-axes are 2u sin θ
l
Time of flight on an inclined plane T =
vx = ux + ax t , v y = u y + a yt g sin α
1 1
x = x 0 + ux t + ax t 2 , y = y0 + u yt + a yt 2 u2 sin2 θ
2 2 l
Maximum height, h =
2 g cos α
a = a i$ + a i$
x y
2u2 sin θ cos (θ + α )
l
Horizontal range, R =
g cos2 α
Projectile Motion l
Maximum range occurs when θ =
π α

Projectile is an object which Y 2 2
once projected in a given
u cos θ u2
A l
Rmax = when projectile is thrown upwards.
uy = u sin θ

direction with given velocity g(1 + sin α )


and is then free to move under u
gravity alone. The path u2
h l
Rmax = when projectile is thrown downwards.
described by the projectile is g (1 − sin α )
θ
called its trajectory.
X
Let a particle is projected at an O ux = u cos θ D B
angle θ from the ground with R Projectile Motion Down
initial velocity u. an Inclined Plane
Resolving u in two components, we have A projectile is projected down the plane from the point O with
ux = u cos θ, u y = u sin θ, ax = 0, a y = − g. an initial velocity u at an angle θ with horizontal. The angle of
g inclination of plane with horizontal α. Then,
l
Equation of trajectory, y = x tan θ − x2 α
2u2 cos2 θ x sin
u g
u2 sin2 θ θ α)
=
l
Vertical height covered, h = –
2g O 0°
(9 g sin (90°– α)
s α
α

co
u sin 2θ
2
= g cos α
°–

g
90

l
Horizontal range, R = OB = ux T , R = α A
α
g A

Projectile Motion in Horizontal 2u sin(θ + α )


l
Time of flight down an inclined plane, T =
Direction From Height ( h) g cos α
Let a particle be projected in horizontal direction with speed u u2
l
Horizontal range, R = [sin (2θ + α ) + sin α ]
from height h. g cos2 α
DAY PRACTICE SESSION 1

FOUNDATION QUESTIONS EXERCISE


1 Which of the following statement is true? 11 If a vector 2 $i + 3 $j + 8 k$ is perpendicular to the vector
(a) A scalar quantity is the one that is conserved in a 4$j − 4$i + αk$ , then value of α is
1 1
process (a) − 1 (b) (c) − (d) 1
(b) A scalar quantity is one that can never be negative 2 2
values 12 At what angle should the two forces 2P and 2P act, so
(c) A scalar quantity is the one that does not vary from one that the resultant force is P 10?
point to another in space
(a) 45° (b) 60° (c) 90° (d) 120°
(d) A scalar quantity has the same value for observers with
different orientations of the axes 13 A boat is sent across a river with a velocity of 8 km/h. If
the resultant velocity of boat is 10 km/h, then velocity of
2 If two vectors are equal in magnitude and their resultant
river is
is also equal in magnitude to one of them, then the angle
(a) 10 km/h (b) 8 km/h (c) 6 km/h (d) 4 km/h
between the two vectors is
(a) 60° (b) 120° (c) 90° (d) 0° 14 The velocity of a projectile at the initial point A is
( 2$i + 3$j ) m/s. Its velocity (in m/s) at point B is
3 If A = 3 $i + 4 $j and B = 7 $i + 24 $j, the vector having the j NEET 2013
same magnitude as B and parallel to A is
Y
(a) 5 $i + 20 $j (b) 15 $i + 10 $j (c) 20 $i + 15 $j (d) 15 $i + 20 $j
4 Six vectors a through f have the magnitudes and
directions as shown in figure. Which statement is true?
j
CBSE AIPMT 2010
b B X
a c A
d e
f (a) −2 $i − 3 $j (b) −2 i + 3 j (c) 2 i − 3 j (d) 2 i + 3 j
(a) b + c = f (b) d + c = f 15 The x and y coordinates of the particle at any time are
(c) a + e = f (d) b + e = f x = 5 t − 2 t 2 and y = 10 t respectively, where x and y are
5 The component of vector A = 2 $i + 3 $j along the vector in metres and t in seconds. The acceleration of the
$i + $j is particle at t = 2 s is j
NEET 2017

(a)
5
(b) 10 2 (c) 5 2 (d) 5 (a) 0 (b) 5 $i m/s 2 (c) −4 $i m/s 2 (d) −8 $i m/s 2
2
16 A particle has initial velocity ( 3i$ + 4$j ) and has
6 A and B are two vectors and θ is the angle between acceleration ( 0.4 i$ + 0.3 $j ) . Its speed after 10 s is
them, if | A × B | = 3 ( A ⋅ B ), the value of θ is j
CBSE AIPMT 2010
(a) 60° (b) 45° (c) 30° (d) 90°
(a) 7 unit (b) 7 2 unit (c) 8.5 unit (d) 10 unit
7 Given A = 4$i + 6$j and B = 2$i + 3$j. Which of the following
17 A particle is moving such that its position coordinates
is correct?
( x , y ) are (2 m, 3 m) at time t = 0, (6 m, 7 m) at time
(a) A × B = 0 (b) A ⋅ B = 24
| A| 1 t = 2 s and (13 m, 14 m) at time t = 5 s. Average velocity
(c) = (d) A and B are anti-parallel vector ( vav ) from t = 0 to t = 5 s is
|B| 2
1 7 11 $ $
(a) (13 $i + 14 $j ) (b) ($i + $j ) (c) 2 ($i + $j ) (d) (i + j )
8 If A = 4$i + 4$j + 4k$ and B = 3$i + $j + 4k,
$ then angle between 5 3 5
vectors A and B is 18 The horizontal range and maximum height attained by a
(a) 180° (b) 90° (c) 45° (d) 0° projectile are R and H, respectively. If a constant
9 If two vectors 2 $i + 3$j + k$ and −4$i − 6$j − λk$ are parallel to horizontal acceleration a = g /4 is imparted to the
projectile due to wind, then its horizontal range and
each other, then value of λ is
maximum height will be
(a) zero (b) −2 (c) 3 (d) 4
(b)  R +  , 2H
H H
(a)(R + H ),
10 If A ⋅ B = A × B, then the angle between A and B is 2  2
(a) 45° (b) 30° (c) 60° (d) 90° (c)(R + 2H ), H (d) (R + H ), H
DAY THREE SCALAR AND VECTOR 27

19 A projectile is thrown in the upward direction making an 22 The horizontal range and maximum height of a projectile
angle of 60° with the horizontal direction with a velocity of are equal. The angle of projection is j CBSE AIPMT 2012
147 m/s. Then, the time after which its inclination with the
(a) θ = tan  
−1 
1 −1
(b) θ = tan (4)
horizontal is 45°, is  4
(a) 15 s (b) 10.98 s (c) 5.49 s (d) 2.745 s (c) θ = tan−1 (2) (d) θ = 45 °
20 The velocity of a particle is v = v 0 + gt + at . If its position is
3
23 A missile is fired for maximum range with an initial
x = 0 at t = 0, then its displacement after unit time (t = 1) is
velocity of 20 m/s. If g = 10 m/s 2, the range of missile is
g
(a)v 0 = +a (b) v 0 = 2g + 3a j CBSE AIPMT 2011
2
g a (a) 50 m (b) 60 m
(c)v 0 + + (d) v 0 + g + a
2 3 (c) 20 m (d) 40 m
21 A projectile is fired from the surface of the earth with a 24 A particle of mass m is projected with a velocity v making
velocity of 5 m/s and angle θ with the horizontal. Another an angle of 45° with the horizontal. The magnitude of
projectile fired from another planet with a velocity of 3 angular momentum of projectile about the point of
m/s at the same angle follows a trajectory which is projection when the particle is at its maximum height h is
identical with the trajectory of the projectile fired from the mvh
(a)zero (b)
earth. The value of the acceleration due to gravity on the 2
planet is (in m/s 2) (given, g = 9.8 m/s 2) j CBSE AIPMT 2014 mvh 2
(c) (d) None of these
(a) 3.5 (b) 5.9 (c) 16.3 (d) 110.8 2

DAY PRACTICE SESSION 2

PROGRESSIVE QUESTIONS EXERCISE


1 The sum of the magnitudes of two forces acting at a point 6 A boat takes 2 h to travel 8 km and back in still water. If
is 16 N. The resultant of these forces is perpendicular to the velocity of water 4 km/h, the time taken for going up
the smaller forces and has a magnitude of 8 N. If the stream 8 km and coming back is
smaller forces of magnitude x, then the value of x is (a) 2 h (b) 2 h 40 min (c) 1 h 20 min
(a) 2 N (b) 4 N (c) 6 N (d) 7 N (d) Cannot be estimated with the
information given B
2 If the magnitude of sum of two vectors is equal to the
magnitude of difference of the two vectors, the angle 7 A man wants to reach point B on the
between these vectors is j NEET 2016, CBSE AIPMT 1991 opposite bank of a river flowing at a u→
45°
(a) 90° (b) 45° (c) 180° (d) 0° speed as shown in figure. What
minimum speed relative to water A
3 The value of n so that vectors 2 $i + 3 $j − 2 k$ , 5 $i + n $j + k$ should the man have, so that he can
and − $i + 2 $j + 3 k$ may be coplanar, will be reach point B ?
(a) 18 (b) 28 (c) 9 (d) 36 (a) u 2 (b) u / 2 (c) 2u (d) u / 2

4 A projectile is given an initial velocity of (i + 2j ) m/s, when 8 A particle starting from the origin (0, 0) moves in a
i is along the ground and j is along the vetical. straight line in the XY -plane. Its coordinates at a later
If g = 10 m/s 2, the equation of its trajectory is time are ( 3, 3). The path of the particle makes with the
(a) y = x − 5 x 2 (b) y = 2 x − 5 x 2 X -axis an angle of
(c) 4 y = 2 x − 5 x 2 (d) 4 y = 2 x − 25 x 2 (a) 30° (b) 45° (c) 60° (d) 0°
5 A police jeep is chasing a culprit going on a motorbike. 9 A ball is rolled off along the edge of the table with
The motorbike crosses a turning at a speed of 72 km/h. horizontal with velocity 4 m/s. It hits the ground after time
The jeep follows it at a speed of 90 km/h, crossing the 0.4 s. Which of the following statement is wrong.
turning 10 s later than the bike. Assuming that they travel (g = 10 m/s 2)
at constant speeds, how far from the turning will the jeep
(a) The height of table is 0.8 m.
catch up with the bike? (in km)
(b) It hits the ground of an angle of 60° with the vertical.
(a) 1 (b) 2 (c) 3 (d) 4 (c) It covers a horizontal distance 1.6 m from the table.
(d) It hits the ground with vertical velocity 4 m/s.
28 40 DAYS ~ NEET PHYSICS DAY THREE

10 A ship A is moving Westwards with a speed of 10 km/h 12 The position vector of a particle R as a function of time is
and ship B 100 km South of A, is moving Northwards with given by R = 4 sin ( 2πt ) i$ + 4 cos ( 2πt ) $j
a speed of 10 km/h. The time after which the distance
between them becomes shortest, is j CBSE AIPMT 2015 where R is in metre, t is in seconds and $i and $j denote
(a) 0 h (b) 5 h (c) 5 2 h (d) 10 2 h unit vectors along x and y-directions, respectively. Which
one of the following statements is wrong for the motion of
11 Two particles A and B, move with constant velocities v1
particle? j CBSE AIPMT 2015
and v 2. At the initial moment, their position vectors are r1
and r2 respectively. The condition for particles A and B (a) Acceleration is along − R
for their collision is j CBSE AIPMT 2015 v2
(b) Magnitude of acceleration vector is , where v is the
r1 − r2 v − v1 R
(a) = 2 (b) r1 ⋅ v1 = r2 ⋅ v 2 velocity of particle
| r1 − r2 | | v 2 − v1 |
(c) Magnitude of the velocity of particle is 8 m/s
(c) r1 × v1 = r2 × v 2 (d) r1 − r2 = v1 − v 2
(d) Path of the particle is a circle of radius 4 m

ANSWERS
SESSION 1 1 (b) 2 (b) 3 (d) 4 (d) 5 (a) 6 (a) 7 (a) 8 (b) 9 (b) 10 (a)
11 (c) 12 (a) 13 (c) 14 (c) 15 (c) 16 (b) 17 (d) 18 (d) 19 (c) 20 (c)
21 (a) 22 (b) 23 (d) 24 (b)

SESSION 2 1 (c) 2 (a) 3 (a) 4 (b) 5 (a) 6 (b) 7 (b) 8 (c) 9 (b) 10 (b)
11 (a) 12 (c)

Hints and Explanations


SESSION 1 5 Component of A along $i + $j 9 The coefficients of i, j, k should be a
1 A scalar quantity has same value for B (2$i + 3$j ) ⋅ ($i + $j ) 5 constant ratio.
⇒ A ⋅ B$ = A ⋅ = = 2 3 1
observers with different orientation of |B| 2 2 or = = or λ = −2
the axes. Since, value of scalar is −4 −6 λ
independent of the direction of its 6 Given, | A × B | = 3 (A ⋅ B ) 10 Given, A ⋅ B = A × B
observation. ⇒ AB sin θ = 3 AB cos θ ⇒ AB cos θ = AB sin θ ⇒ cos θ = sin θ
2 Given, R = A = B ⇒ tanθ = 3 ⇒ θ = 60° ⇒ tanθ = 1 ⇒ θ = 45°
∴ R2 = R2 + R2 + 2RR cos θ
7 A × B = (4 $i + 6$j ) × (2$i + 3$j ) 11 Let, a = 2 $i + 3 $j + 8k$ ,
1 b = 4 $j − 4 $i + α k$ = –4 $i + 4 $j + α k$
or cos θ = − ; = 12($i × $j ) + 12($j × i)
2 Given, a ⊥ b ⇒ a ⋅ b = 0
∴ θ = 120° = 12($i × $j ) − 12($i × $j ) = 0 ⇒ (2 $i + 3 $j + 8k$ ) (− 4 $i + 4 $j + α k$ ) = 0
3 A vector parallel to A will be Again, A ⋅ B = (4 $i + 6$j ) ⋅ (2 $i + 3 $j ) ⇒ −8 + 12 + 8α = 0 ⇒ 8α = − 4
n A or (3 n $i + 4 n $j ) = 8 + 18 = 26 ⇒ α =−
1
Now, | nA | = | B | is given |A| 16 + 36 1 2
Again, = ≠
Hence, |B| 4+ 9 2 12 Resultant, R = A2 + B 2 + 2 AB cos θ
n 9 + 16 = 49 + 576 1
Also, B= A Given, R = P 10, A = 2P , B = 2P
or n=5 2
∴ nA = 15$i + 20 $j ⇒ A and B are parallel and not ∴ P 10 = 4P 2 + 2P 2 + 4 2 P 2 cos θ
anti-parallel.
4 When two non-zero vectors are ⇒ P 10 = 6 P 2 + 4 2 P 2 cos θ
represented by the two adjacent sides 8 A ⋅ B = AB cos θ
On, squaring both sides, we have
of a parallelogram, then the resultant is Given, A = 4 $i + 4 $j − 4k$ , B = 3 $i + $j + 4k$
given by the diagonal of the 10 P 2 = 6 P 2 + 4 2 P 2 cos θ
⇒ A ⋅ B = (4 $i + 4 $j − 4k$ ) ⋅ (3 $i + $j + 4k$ )
parallelogram passing through the point 4 P 2 = 4 2P 2 cos θ
of intersection of the two vectors = 4 × 3 + 4 − 16 = 0 1
b + e = f. ⇒ A ⋅ B = 0 ⇒ cos θ = 0 ⇒ θ = 90° ⇒ cos θ = ⇒ θ = 45°
2
DAY THREE SCALAR AND VECTOR 29

13 Given, AB = Velocity of boat = 8 km/h 19 Horizontal component of velocity at 24 The angular momentum of a particle is
AC = Resultant velocity of boat = 10 km/h angle 60° = Horizontal component of given by
velocity at 45° r
B C i.e. u cos 60° = v cos 45°
θ
1 1 147
or 147 × = v × or v = m/s r
v cos 45°
2 2 2 h
θ Vertical component of θ
O
147 3 L =r × mv
A u y = u sin 60° = m
2 ∴ L = mvr sin θ
∴ BC = Velocity of river = AC − AB
2 2
Vertical component of From figure,
= (10) − (8) = 6 km/h
2 2 147 1 147
v y = v sin 45° = × = m L = r m (v cos 45° ) sin θ
2 2 2 mv mvh
14 From the figure, the x- component = (r sin θ) =
but v y = u y + at 2 2
remain unchanged, while the 147 147 3
y-component is reverse. Then, the ∴ = − 9. 8 t or t = 5.49 s
velocity at point B is ( 2$i − 3$j ) m/s.
2 2 SESSION 2
20 Velocity v = v 0 + gt + at 2 1 Given, x + y = 16
15 Given, x = 5t − 2t 2 dx
Velocity of the particle, = v 0 + gt + at 2
dt
dx d
vx = = (5t − 2t 2 ) = 5 − 4t Integrate on both sides,
dt dt y
∫ dx = ∫ v 0dt + ∫ gtdt + ∫ at
2 8N
dt
d
Acceleration, ax = v x = − 4 ms −2
dt 1 2 at 3
x = v 0t +gt + +C
Also, y = 10t 2 3
dy Given, x = 0 at t = 0 x
Velocity, vy = = 10 Also, y 2 = 82 + x2
dt ∴ C =0
dv y 1 1 or y 2 = 64 + (16 − y )2 [Q x = 16 − y ]
∴ Acceleration, a y = =0 x = v 0 t + gt 2 + at 3
2 3 or y 2 = 64 + 256 + y 2 − 32 y
dt
∴ Net acceleration of the particle, 1 1 or 32 y = 320 or y = 10 N
At t = 1 second, x = v 0 + g + a
2 3 ∴ x + 10 = 16 or x = 6 N
a net = ax $i + a y$j = (−4 ms −2 ) i$
gx2 2 Suppose two vectors are P and Q.
or a net = − 4 i$ ms −2 21 y = x tan θ −
2u cos 2 θ
2 It is given that| P + Q | = | P − Q |
16 Given, initial velocity (u ) = 3i$ + 4$j For equatorial trajectories for same angle Let angle between P and Q is φ.
Final velocity (v) = ? of projection ∴ P 2 + Q 2 + 2PQ cos φ
Acceleration (a ) = (0.4 i$ + 0.3 $j ) 8 = P 2 + Q 2 − 2PQ cos φ
= constant
Time (t) = 10 s u2 ⇒ 4PQ cos φ = 0
9.8 g ′ ⇒ cos φ = 0 [Q P , Q ≠ 0]
From first equation of motion, ⇒ = 2
v = u + at 52 3 π
9.8 × 9 ⇒ φ= = 90°
v = 3$i + 4$j + 10 (0.4i$ + 03
. $j ) g′ = = 3.528 m/s2 2
25
v = 7i + 7 j ⇒ |v| = 7 2
$ $ 3 For given vectors to be coplanar,
= 3.5 m/s2 A ×B × C = 0
( x2 − x1 ) $i + ( y 2 − y 1 ) $j
17 Velocity, v av =
t2 − t1 22 Given, Range (R) = maximum height (H) A = 2 $i + 3 $j − 2k$ ⇒ B = 5$i + n$j + k$
(13 − 2) $i + (14 − 3) $j u2 (2sin θ cos θ) u2 sin2 θ ∴ C = − $i + 2 $j + 3k$
= Also, R = ,H =
5− 0 g 2g 2 3 −2
11 $i + 11 $j 11 $ $ u2 (2sin θ cos θ) u2 sin2 θ 5 n 1 =0
= = (i + j ) ∴ =
5 5 g 2g −1 2 3
2u y u2y sin θ ⇒ 2(3 n − 2) − 3(15 + 1) − 2 (10 + n ) = 0
18 T = , H = and R = u x T ⇒ 2cos θ =
g 2g 2 ⇒ 6 n − 4 − 45 − 3 − 20 − 2n = 0
⇒ tanθ = 4 ⇒ 4 n = 72, n = 18
When a horizontal acceleration is also
given to the projectile u y, T and H will ⇒ θ = tan −1 (4) 4 The equation of trajectory of a particle,
remains unchanged while the range will fired, with an initial velocity u at an
23 Maximum range of projectile is given by angle of projection θ,
become 2
u
1 R max = gx2
R ′ = u x T + aT 2 g y = x tan θ − 2
2 2u cos 2 θ
Given, u = 20 m/s and g = 10 m/s2
1 g  4u y 
2
gx2
=R+ =R+ H (20)2 400 = x tan θ − 2 sec2 θ
2 4  g 2  ∴ R max = = = 40 m 2u
10 10 gx2
and maximum height will be H. = x tan θ − 2 (1 + tan2 θ)
2u
30 40 DAYS ~ NEET PHYSICS DAY THREE

Now, magnitude of velocity vector Resultant of v and u should be along AB. 1 PS


⇒ =
u = $i + 2$j ⇒ u = (1)2 + (2)2 = 5 m/s Components of v b (absolute velocity of 2 100
boatman) along x and y directions are, 100 100 2
and angle of projection is given by
v x = u − v sin θ PS = = = 50 2
$j component 2 2 2
tan θ = = =2 and v y = v cos θ
i$ component 1 Relative velocity between A and B is
vy
Further, tan 45° = v BA = v 2A + v 2B = 10 2
tanθ = 2 vx
So, from eq (i), we have v cos θ 50 2
or 1 = t =
10 × x2 u − v sin θ 10 2
y = 2x − (1 + 4) = 2 x − 5x2
2× 5 u ⇒ t = 5h
v =
5 v p = 90 km/h = 25m/s sin θ + cos θ 11 For two particles A and B move with
v c = 72 km/h = 20 m/s u constant velocities v 1 and v 2 . Such that
=
2 sin (θ + 45° ) two particles to collide, the direction of
In 10 s culprit reaches point B from A.
Distance covered by culprit, the relative velocity of one with respect
v is minimum at,
S = vt = 20 × 10 = 200 m to other should be directed towards the
θ + 45° = 90° or θ = 45°
At time t = 10 s, the police jeep is 200 m relative position of the other particle.
u
and v min = r − r2
behind the culprit. 2 i.e. 1 → direction of relative
Relative velocity between jeep and |r1 − r2|
culprit is 25 − 20 = 5 m/s 8 Draw the situation Y position of 1 w.r.t. 2.
S 200 as shown. OA v − v2
Time = = = 40 s A Similarly, 1 → direction of
v 5 represents the
(√3,3) |v1 − v2|
[Relative velocity is considered] path of the particle
velocity of 2 w.r.t. 1.
starting from
In 40 s, the police jeep will move from A So, for collision of A and B, we get
origin O(0, 0). θ B
to a distance S X r1 − r2 v − v1
Draw a O (0,0) = 2
where, S = vt = 25 × 40 = 1000 m |r1 − r2| |v2 − v1|
perpendicular
= 1 km away from point A to X -axis. Let path of the
The jeep will catch up with the bike 12 (i) The position vector of a particle R as
particle makes an angle θ with the
1 km far from the turning. X -axis, then a function of time is given by
tanθ = slope of line OA R = 4 sin (2 πt ) $i + 4 cos (2 πt )j$
6 Boat covers distance of 16 km in a still
water in 2 h AB 3 x-component,
= = = 3 or θ = 60°
16 OB 3 x = 4 sin 2 πt
i.e. vB = = 8 km/h …(i)
2
Now, velocity of water v W = 4 km/h
9 Height of table y-component,
h=
1 2 1
gt = × 10 × (0.4)2 = 0.8 m y = 4cos 2 πt …(ii)
Time taken for going upstream
8 8 2 2 Squaring and adding both equations,
t1 = = = 2h Horizontal distance covered = u x t
vB − vW 8− 4 we get
= 4 × 0.4 = 1.6 m x2 + y 2 = 42 [sin2 (2 πt ) + cos 2 (2 πt )]
As water current oppose the motion of
Vertical velocity on reaching ground i.e. x2 + y 2 = 42 i.e. equation of
boat, therefore time taken for going
downstream v y = u y + a yt = 0 + 10 × 0.4 = 4 m/s
circle and radius is 4 m.
8 8 8 Horizontal velocity on reaching ground
t2 = = = h v c = u x = 4 m/s (ii) Acceleration vector,
vG + v W 8 + 4 12
If θ is the angle at which the ball hits the v2
[water current helps a= (− R$ ), while v is velocity of a
ground with the vertical, then R
the motion of boat] v 4
tanθ = x = = 1 ⇒ θ = 45° particle.
∴Total time = t 1 + t 2 vy 4 (iii) Magnitude of acceleration vector,
=  2 +
8
 h v2
 12  10 a=
N R
= 2 h 40 min
(iv) As, we have v x = + 4 (cos 2 πt ) 2 π
7 Let v be the speed of boatman in still and v y = − 4 (sin 2 πt ) 2 π
water, W
O Net resultant velocity,
E
B VA= 10 km/h
v = v 2x + v 2y
P 45° 100 km
v VB = 10 km/h = (8 π)2 (cos 2 2 π t + sin2 2 π t )
vb
45° S
θ v = 8π [Q cos 2 2 πt + sin2 2 πt = 1]
PS
A u sin 45° = So, option (c) is incorrect.
OS
DAY FOUR

Laws
of Motion
Learning & Revision for the Day
u Concept of Forces u Principle of Conservation of u Equilibrium of concurrent
u Inertia Linear Momentum Forces
u Newton’s Laws of Motion u Free Body Diagram u Friction
u Impulse u Connected Motion

Concept of Forces
A push or a pull exerted on any object, is defined to be a force. It is a vector quantity.
Thus, we denote it with an arrow over it, just as we do for velocity and acceleration. Force
can be grouped into two types:
l
Contact forces are the forces that act between two bodies in contact, e.g. tension,
normal reaction, friction etc.
l
Non-contact forces are the forces that act between two bodies separated by a distance
without any actual contact. e.g. gravational force between two bodies and electrostatic
form between two charges etc.

Inertia
The inability of a body to change by itself its state of rest or state of uniform motion along
a straight line is called inertia of the body.
As inertia of a body is measured by the mass of the body. Heavier the body, greater the PREP
force required to change its state and hence greater is its inertia. inertia obviously three
types (i) inertia of rest (ii) inertia of motion (iii) inertia of direction.
MIRROR
Your Personal Preparation Indicator

Newton’s Laws of Motion u No. of Questions in Exercises (x)—


u No. of Questions Attempted (y)—
First Law of Motion (Law of Inertia) u No. of Correct Questions (z)—
(Without referring Explanations)
It states that a body continues to be in a state of rest or of uniform motion along a straight
line, unless it is acted upon by some external force the change the state. This is also u Accuracy Level (z / y × 100)—
called law of inertia. u Prep Level (z / x × 100)—
If F = 0, ⇒ v = constant ⇒ a = 0
l
This law defines force. In order to expect good rank in NEET, your
Accuracy Level should be above 85 & Prep
l
The body opposes any external change in its state of rest or of uniform motion. Level should be above 75.
l
It is also known as the law of inertia given by Galileo.
32 40 DAYS ~ NEET PHYSICS DAY FOUR

Linear Momentum Applications of Principle of


It is defined as the total amount of motion of a body and is
measured as the product of the mass of the body and its
Conservation of Linear Momentum
velocity. The momentum of a body of mass m moving with a The propulsion of rockets and jet planes is based on the
velocity v is given by p = mv. principle of conservation of linear momentum.
u dm
Its unit is kg-ms–1 and dimensional formula is [ML T –1] l
Upward thrust on the rocket, F = − − mg and if effect
dt
u dm
Second Law of Motion of gravity is neglected, then F = − .
dt
The rate of change of momentum of a body is directly l
Instantaneous upward velocity of the rocket
proportional to the applied force and takes place in the m 
direction in which the force acts. v = u ln  0  − gt
 m
dp dp
According to second law, F ∝ or F = k and neglecting the effect of gravity
dt dt
where, k is constant. m  m 
v = u ln  0  = 2.303 u log 10  0 
dp d m dv  m  m
as, = (mv) = ma or = ma
dt dt dt where, m0 = initial mass of the rocket including that of the
i. e., second law can be written as fuel,
dp u = initial velocity of the rocket at any time t ,
F = = ma
dt m = mass of the rocket left,
The SI unit of force is newton (N) and in CGS system is dyne. v = velocity acquired by the rocket,
1 N = 10 5 dyne dm
= rate of combination of fuel.
dt
Impulse
l
Burnt out speed of the rocket is the speed attained by the
Impulse received during an impact is defined as the product
rocket when the whole of fuel of the rocket has been burnt.
of the average force and the time for which the force acts.
Burnt out speed of the rocket
Impulse, I = Fav t
m  m 
Impulse is also equal to the total change in momentum of the vb = u loge  0  = 3.303 u log 10  0 
 mr   mr 
body during the impact.
Impulse, I = p2 − p1
Impulse = Change in momentum Apparent Weight of a Boy in a Lift
Actual weight of the body is mg. Here we consider the
apparent weight of a non stading in a moving lift.
Third Law of Motion l
If lift is accelerating upward at the rate of acceleration a,
To every action, there is an equal and opposite reaction.
then apparent weight of the body is R = m(g + a).
F12 = − F21 l
If lift is accelerating downward at the rate of acceleration a,
l
Action and reaction are mutually opposite and act on two then apparent weight of the body is R = m(g − a)
different bodies. l
If lift is moving upward or downward with constant
l
The force acting on a body is known as action. velocity, then apparent weight of the body is equal to
l
When a force acts on a body, then the reaction acts actual weight.
normally to the surface of the body.
Free Body Diagram
A free body diagram (FBD) consists of a diagramatize
Principle of Conservation of representation of a single body or sub-system of bodies
Linear Momentum isolated from its surroundings showing all forces acting on it.
It states that if no external force is acting on a system, the While sketching a free body diagram the following points
momentum of the system remains constant. should be kept in mind.
dp l
Normal reaction (N) always acts
According to second law of motion, F =
dt normal to the surface on which the A B
If no force is acting, then F = 0 body is kept.
dp l
When two objects A and B are connected by a string, the
∴ = 0 ⇒ p = constant or m1v1 = m2 v2 = constant
dt tension for object A is towards B and for object B, it is
towards A.
DAY FOUR LAWS OF MOTION 33

Tension in the string,


Connected Motion F (m3 + m2 )
If two blocks of masses m1 and m2 are placed on a perfectly T1 = (m2 + m3 )a =
(m1 + m2 + m3 )
l

smooth surface and are in contact, then


F m3
B and tension T2 = m3 a =
A m1 + m2 + m3
F f f
m1 m2
l
For a block system shown in the figure, acceleration of the
system
F F
Acceleration of the blocks, a = a=
m1 + m2 m1 + m2 + m3
and the contact force (acting normally) between the two
T1 T2 m3
F m2 m1 m2
blocks is f = m2 a = .
(m1 + m2 )
l
A block system is shown in the figure Tension in the string,
m1F (m1 + m2 )F
F T1 = and T2 =
m1
m2 m1 + m2 + m3 m1 + m2 + m3

F For a block system suspended freely from a


Acceleration of the blocks a = l
T1
m1 + m2 rigid support as shown in the figure, the
Contact force between two blocks acceleration of the system a = 0. m1 A

m1F String tension, T2


f = m1a = T1 = (m1 + m2 + m3 )g
(m1 + m2 ) m2 B
T2 = (m2 + m3 )g
l
For a block of mass m placed on a fixed, perfectly smooth T3
and T3 = m3 g
inclined plane of angle θ, the forces acting on the block are
as shown in the figure. Obviously, here a = g sin θ m3 C

N
m F l
For a block system and a pulley as shown T
a in the figure, value of the acceleration of
θ
the system
θ
sin (m + m2 − m3 )g T2
mg mg cos θ
θ mg a= 1 T2
(m1 + m2 + m3 ) a
l
If a block of mass m is placed on a smooth movable wedge of 2 m1m3 g m3 m2 B
Tension, T1 = C
mass M, which in turn is placed on smooth surface, then a (m1 + m2 + m3 )
force F is applied on the wedge, horizontally. T1 a
2 m3 (m1 + m2 )g
The acceleration of the wedge and the block is Tension, T2 =
(m1 + m2 + m3 ) m1
A
N cos θ
and tension T
N
θ 4 m3 (m1 + m2 )g
T = 2 T2 =
m (m1 + m2 + m3 )
N sin θ
ma F l
For the pulley and block arrangement as shown in the
figure, we have
θ mg M T2 T1
M
F
a=
(M + m)
T2
T1
Force on the block, F = (M + m)a = (M + m)g tan θ a
l
For a block system shown in the figure, acceleration of the a
m2 (m 1 > m 2 )
system
m1
F
a=
m1 + m2 + m3 Net acceleration,
Net accelerating force (m1 − m2 )g
A B C a= =
F m1 T1 m2 T2
m3 Total mass (m1 + m2 + M )
34 40 DAYS ~ NEET PHYSICS DAY FOUR

(M + 2 m2 )m1g Lami Theorem For three concurrent forces in equilibrium


Tension, T1 = m1(g − a) =
(M + m1 + m2 ) position.
P R
and Tension, T2 = m2 (g + a) β
(M + 2 m1)m2 g
= γ α
(M + m1 + m2 )
l
For the system of block and pulley, with a smooth inclined
plane as shown in the figure, we have
Q
N a P Q R
T = =
sin α sin β sin γ
m2 T
θ
in
gs
θ
m2
θ m2g
m2 g cos θ m1 a
Friction
Whenever an object actually slides or rolls over the surface of
Net acceleration, another body or tends to do, so a force opposing the relative
(m − m2 sin θ)g motion starts acting between these two surfaces in contact. It
a= 1 , is known as friction or the force due to friction. Force of
(m1 + m2 )
friction acts in a tangential direction to the surfaces in contact.
if m1g > m2 g sin θ
(m sin θ − m1)g
and a = 2
m1 + m2
, if m1g < m2 g sin θ Types of Friction
The four types of friction are given below
and tension in the string
1. Static Friction It is the opposing force that comes into
m m (1 + sin θ)
T = m1(g − a) = 1 2 play when one body is at rest and a force acts to move it
(m1 + m2 ) over the surface of another body.
l
For a pulley and block system on a smooth double inclined It is a self adjusting force and is always equal and opposite
plane as shown in the figure, we have to the applied force.
2. Limiting Friction It is the limiting (maximum) value of
N
a T static friction when a body is just on the verge of starting
T
m2
N its motion over the surface of another body.
m1
N
θ1
in a
gs
m2

m1
g
sin

m1g cos θ1 m2g cos θ2 θ2


θ1
θ2

Applied
(m2 sin θ2 − m1 sin θ 1)g f = µN force F
Net acceleration, a = ,
(m1 + m2 )
for θ2 > θ 1, m2 > m1 The force of limiting friction f l between the surfaces of
two bodies is directly proportional to the normal reaction
and tension in the string, at the point of contact. Mathematically,
m m (sin θ 1 + sin θ2 )g
T = 1 2 f
(m1 + m2 ) fl ∝ N = µ l N ⇒ µ l = l
N
where, µ l is the coefficient of limiting friction for the
Equilibrium of Concurrent Forces given surfaces in contact.
3. Kinetic Friction It is the opposing force that comes into
If a number of forces act at the same point, they are called
play when one body is actually slides over the surface of
concurrent forces.
another body. Force of kinetic friction fk is directly
The necessary condition for the equilibrium of a body under f
the action of concurrent forces is that the vector sum of all the proportional to the normal reaction N and the ratio k is
N
forces acting on the body must be zero.
called coefficient of kinetic friction µ k , value of µ k is
Mathematically for equilibrium,
slightly less than µe (µ k < µ l ).
Σ Fnet = 0 or ΣFx = 0, ΣF y = 0
Whenever limiting friction is converted into kinetic
and ΣFz = 0 friction, body started motion with a lurch.
DAY FOUR LAWS OF MOTION 35

4. Rolling Friction It is the opposing force that comes into In limiting condition, f1 = mg sin φ
play when a body of symmetric shape (wheel or cylinder ⇒ N = mg cos φ
or disc, etc.) rolls over the surface of another body. Force f1
of rolling friction fr is directly proportional to the normal = tan φ
N
reaction N and inversely proportional to the radius (r ) of
f1
the wheel. ∴ = µ s = tan φ
N N N
Thus, fr ∝ or fr = µ r
r r
The constant µ r is known as the coefficient of rolling Acceleration of a Block on Applying
friction µ r has the unit and dimensions of length. a Force on a Rough Surface
Magnitudewise µ r << µ k or µ l . l
Acceleration of a block on a horizontal surface is as shown
l
The value of rolling friction is much smaller than the in the figure.
value of sliding friction. N
l
Ball bearings are used to reduce the wear and tear and
energy loss against friction.
f = µN F
Angle of Friction
Angle of friction is defined as the angle θ which the resultant R mg
of the force of limiting friction fl and normal reaction N,
subtends with the normal reaction. F − f F − µmg
a= =
The tangent of the angle of friction is equal to the coefficient m m
of friction. i.e. µ = tan θ =
F
− µg
N m
R
where, µ = coefficient of kinetic friction between the two
surfaces in contact.
θ
fl
l
Acceleration of block sliding down a rough inclined plane
Applied force F as shown in the figure is given by
a = g(sin α − µ cos α )
Angle of Repose N
µN
Angle of repose is the least angle of the inclined plane f=
(of given surface) with the horizontal such that the given body ma
placed over the plane, just begins to slide down, without α α
sin
getting accelerated. mg mg mg cos α
N α
fl
l
Retardation of a block sliding up a rough inclined plane as
shown in the figure is a = g(sin α + µ cos α )
sinφ φ
mg mg mg cos φ N
φ F
The tangent of the angle of repose is equal to the coefficient of ma
friction. α α
sin
Hence, we conclude that angle of friction is (θ) equal to the mg µN mg mg cos α
α
angle of repose (φ).
DAY PRACTICE SESSION 1

FOUNDATION QUESTIONS EXERCISE


1 A ship of mass 3 × 107 kg initially at rest is pulled by a 10 A rigid ball of mass m strikes a rigid wall at 60° and gets
force of 5 × 104 N through a distance of 3 m. Assume reflected without loss of speed as shown in the figure.
that the resistance due to water is negligible, the speed The value of impulse imparted by the wall on the ball will
of the ship is be j NEET 2016

(a) 1.5 ms −1 (b) 60 ms −1 (c) 0.1 ms −1 (d) 5 ms −1 m


2 An open knife edge of mass 200 g is dropped from v
height 5 m on a cardboard. If the knife edge penetrates
distance 2 m into the cardboard, the average resistance 60°
offered by the cardboard to the knife edge is 60°
(a) 7 N (b) 25 N v
(c) 35 N (d) None of these
3 A body with mass 5 kg is acted upon by a force (a) mv (b) 2mv (c) mv / 2 (d) mv / 3
F = ( −3$i + 4$j ) N. If its initial velocity at t = 0 is 11 The force F acting on a particle of mass m is indicated by
v = ( 6$i − 12$j ) m/s. The time at which it will just have a the force-time graph shown below. The change in
velocity along the Y-axis is momentum of the particle over the time interval from 0 to 8
(a) never (b) 10 s (c) 2 s (d) 15 s s is j CBSE AIPMT 2014

4 A block of mass m is moving on a wedge with the 6


acceleration a 0. The wedge is moving with the 3
acceleration a1. The observer is situated on wedge. The
F(N)
0 2
magnitude of pseudo force on the block is 4 6 8
–3
a + a0 
(a) ma0 (b) ma1 (c) m a02 + a12 (d) m  1  t(s)
 2 
(a) 24 N-s (b) 20 N-s (c) 12 N-s (d) 6 N-s
5 A machine gun fires 10 bullets/s, each of mass 10 g, the 12 Three forces start acting simultaneously C
speed of each bullet is 20 cms −1, then force of recoil is
on a particle moving with velocity v.
(a) 200 dyne (b) 2000 dyne These forces are represented in
(c) 20 dyne (d) None of these
magnitude and direction by the three
A B
6 A body, under the action of a force F = 6$i − 8$j + 10k$ , sides of a triangle ABC (as shown). The
−2 particle will now move with velocity
acquires an acceleration of 1 ms . The mass of this
body must be j
CBSE AIPMT 2009 (a) less than v
(a) 2 10 kg (b) 10 kg (c) 20 kg (d) 10 2 kg (b) greater than v
(c) | v | in the direction of largest force BC
7 A disc of mass 100 g is kept floating horizontally in air by (d) v remaining unchanged
firing bullets, each of mass 5g with the same velocity at
13 You are marooned on a frictionless horizontal plane and
the same rate of 10 bullets/s. The bullets rebound with
cannot exert any horizontal force by pushing against the
the same speed in opposite direction, the velocity of
surface. How can you get off
each bullet at the time of impact is
(a) by jumping
(a) 196 cms −1 (b) 9.8 cms −1
(b) by rolling your body on the surface
(c) 98 cms −1 (d) 980 cms −1
(c) by splitting or sneezing or throwing any object
8 A boat of mass 40 kg is at rest. A dog of mass 4 kg (d) by throwing an object in opposite direction
moves in the boat with a velocity of 10 ms −1. What is the
14 A bullet of mass 10 g moving horizontal with a velocity of
velocity of boat?
400 m/s strikes a wood block of mass 2 kg which is
(a) 4 ms −1 (b) 2 ms −1 (c) 8 ms −1 (d) 1 ms −1 suspended by light inextensible string of length 5 m. As
9 A body of mass 0.25 kg is projected with muzzle velocity result, the centre of gravity of the block found to rise a
100 m/s from a tank of mass 100 kg. What is the recoil vertical distance of 10 cm. The speed of the bullet after it
velocity of the tank? emerges of horizontally from the block will be j NEET 2016
(a) 5 m/s (b) 25 m/s (c) 0.5 m/s (d) 0.25 m/s (a) 100 m/s (b) 80 m/s (c) 120 m/s (d) 160 m/s
DAY FOUR LAWS OF MOTION 37

15 An explosion breaks a rock into three parts in a 23 Two blocks of masses m1 = 4 kg and m2 = 2 kg are
horizontal plane. Two of them go off at right angles to connected to the ends of a string which passes over a
each other. The first part of mass 1 kg moves with a massless, frictionless pulley. The total downward thrust
speed of 12 ms −1 and the second part of mass 2 kg on the pulley is nearly
moves with 8 ms −1 speed. If the third part flies off with (a) 27 N (b) 54 N
(c) 2.7 N (d) None of these
4 ms −1 speed, then its mass is j NEET 2013

24 Three blocks of masses m1, m2 and m3 are placed on a


(a) 3 kg (b) 5 kg (c) 7 kg (d)17 kg
horizontal frictionless surface. A force of 40 N pulls the
16 A lift is moving down with acceleration a . A man in the lift system, then calculate the value ofT , if m1 = 10 kg,
drops a ball inside the lift. The acceleration of the ball as m2 = 6 kg, m3 = 4 kg.
observed by the man in the lift and a man standing m1 m2 m3
stationary on the ground are respectively T
10 kg 6 kg 4 kg
(a) g, g (b) g − a,g − a (c) g − a,g (d) a , g
17 A balloon with mass m is descending down with an (a) 40 N (b) 20 N (c) 10 N (d) 5 N
acceleration a (where, a < g). How much mass should be 25 Three identical blocks of masses m = 2 kg are drawn by
removed from it, so that its starts moving up with an a force F = 10 . 2 N with an acceleration of 0.6 ms −2 on a
acceleration a ? j CBSE AIPMT 2014 frictionless surface, then what is the tension (in N) in the
2ma 2ma ma ma string between the blocks B and C ?
(a) (b) (c) (d)
g+a g−a g+a g−a
C B A F
18 A spring balance is attached to the ceiling of lift. A man
hangs his bag on the string and the balance reads 49 N, (a) 9.2 N (b) 7.8 N (c) 4 N (d) 9.8 N
when the lift is stationary. If the lift moves downwards 26 The mass m is placed on a body of
with an acceleration of 5 ms −2 . The reading of the spring mass M. There is no friction. The force
m
balance would be F is applied on M and it moves with F
M
(a) 24 N (b) 74 N (c) 15 N (d) 49 N acceleration a. Then, the net force on
the top body is
19 A person of mass 60 kg is inside a lift of mass 940 kg
(a) F (b) ma
and presses the button on control panel. The lift starts (c) F − ma (d) None of these
moving upwards with an acceleration 1.0 m/s 2 . If
27 Three blocks A, B and C of masses 4 kg, 2 kg and 1kg,
g = 10 m/s2 , the tension in the supporting cable is respectively, are in contact on a frictionless surface as
j
CBSE AIPMT 2011 shown. If a force of 14 N is applied on the 4 kg block,
(a) 9680 N (b) 11000 N (c) 1200 N (d) 8600 N then the contact force between A and B is
20 The mass of a lift is 2000 kg. When the tension in the
supporting cable is 28000 N, then its acceleration is A B
j
CBSE AIPMT 2009 C
−2 −2
(a) 30 ms downwards (b) 4 ms upwards (a) 6 N (b) 8 N (c) 18 N (d) 2 N
(c) 4 ms −2 downwards (d) 14 ms −2 upwards
28 A spring balance is attached to the ceiling of a lift. A man
21 The ratio of the weight of a man in a stationary lift and
hangs his bag on the spring and the spring reads 49 N,
when it is moving downwards with uniform acceleration a
when the lift is stationary. If the lift moves downward with
is 3 : 2, then the value of a is
an acceleration of 5 ms −2 , the reading of the spring
3 g
(a) g (b) balance will be
2 3
2 (a) 24 N (b) 74 N (c) 15 N (d) 49 N
(c) g (d) g F
3 29 Three blocks with masses m, 2m and 3m are
22 A block of mass 10 kg is kept on a horizontal surface. A connected by string as shown in the figure. v
m
force F is acted on the block as shown in figure. For what After an upward force F is applied on block m,
minimum value of F , the block will be lifted up? the masses move upward at constant speed v.
F 2m
What is the net force on the block of mass 2m ?
30° (g is the acceleration due to gravity)
M
j
NEET 2013
3m

(a) 98 N (b) 49 N (a) Zero (b) 2 mg


(c) 200 N (d) None of these (c) 3 mg (d) 6 mg
38 40 DAYS ~ NEET PHYSICS DAY FOUR

30 A marble block of mass 2 kg lying on ice when given a 36 A system consists of three masses m1, m2 and m3
velocity of 6 m/s is stopped by friction in 10 s. Then, the connected by a string passing over a pulley P. The mass
coefficient of friction is m1 hangs freely and m2 and m3 are on a rough horizontal
(a) 0.01 (b) 0.02 (c) 0.03 (d) 0.06
table (the coefficient of friction = µ).
The pulley is frictionless and of negligible mass. The
31 A block of mass 4 kg is kept on a rough horizontal downward acceleration of m1 is
surface. The coefficient of static friction is 0.8. If a force (Assume, m1 = m2 = m3 = m) j CBSE AIPMT 2014

of 19 N is applied on the block parallel to the floor, then m2 m3


the force of friction between the block and floor is p
(a) 32 N (b) 18 N (c) 19 N (d) 9.8 N
32 The upper half of an inclined plane of inclination θ is
perfectly smooth while lower half is rough. A block m1
starting from rest at the top of the plane will again come g (1 – gµ ) 2 gµ
(a) (b)
to rest at the bottom, if the coefficient of friction between 9 3
the block and lower half of the plane is given by g (1 – 2µ ) g (1 – 2µ )
(c) (d)
j NEET 2013 3 2
1 2 37 A block A of mass 100 kg rests on another block B of
(a) µ = (b) µ = (c) µ = 2 tanθ (d) µ = tanθ
tanθ tanθ mass 200 kg and is tied to a wall as shown in the figure.
33 A block of mass m is in contact with the cart C as shown The coefficient of friction between A and B is 0.2 and that
in the figure. j CBSE AIPMT 2010 between B and the ground is 0.3. The minimum force F
required to move the block B is (take, g = 10 ms −2 )
α

A
C m
Wall
F
B
S

(a) 900 N (b) 200 N (c) 1100 N (d) 700 N


The coefficient of static friction between the block and
38 A plank with a box on it at one end is gradually raised
the cart is µ. The acceleration α of the cart that will
about the other end. As the angle of inclination with the
prevent the block from falling satisfies
horizontal reaches 30°, the box starts to slip and slides
mg g g g
(a) α > (b) α > (c) α ≥ (d) α < 4.0 m down the plank in 4.0 s. The coefficients of static
µ µm µ µ
and kinetic friction between the box and the plank will
34 Which one of the following statements is incorrect? be, respectively j
CBSE AIPMT 2015
(a) Frictional force opposes the relative motion j NEET 2018
(b) Limiting value of static friction is directly proportional to
normal reaction
(c) Rolling friction is smaller than sliding friction
(d) Coefficient of sliding friction has dimensions of length mg
35 A block A of mass m1 rests on a horizontal table. A light θ
string connected to it passes over a frictionless pulley at
the edge of table and from its other end another block B (a) 0.6 and 0.6 (b) 0.6 and 0.5
of mass m2 is suspended. The coefficient of kinetic (c) 0.5 and 0.6 (d) 0.4 and 0.3
friction between the block and the table is µk . When the 39 A block is moving up an inclined plane of inclination 60°
block A is sliding on the table, the tension in the string is with velocity of 20 ms −1 and stops after 2.00 s. If
(a)
(m2 – µ k m1) g
(b)
m1m2 (1 + µ k ) g g = 10 ms −2 , then the approximate value of coefficient of
(m1 + m2 ) (m1 + m2 ) friction is
m1m2 (1 – µ k ) g (m2 + µ k m1) g (a) 3 (b) 3.3 (c) 0.27 (d) 0.33
(c) (d)
(m1 + m2 ) (m1 + m2 )
DAY FOUR LAWS OF MOTION 39

DAY PRACTICE SESSION 2

PROGRESSIVE QUESTIONS EXERCISE


1 A body of mass 2 kg travels according to the law 7 A block B is pushed momentarily v
B
x (t ) = pt + qt 2 + rt 3 , where p = 3 m/s, q = 4 m /s 2 and along a horizontal surface with an
r = 5 m /s 3 initial velocity v. If µ is the coefficient of
The force acting on the body at t = 2 s is sliding friction between B and the surface, block B will
(a) 136 N (b) 134 N (c) 158 N (d) 68 N come to rest after a time
v gµ
2 Figure shows a block of mass m kept (a) (b)
kg gµ v
on an inclined plane with inclination m =1
θ = 30°. The tension in the string is g v
(c) (d)
(a) 8 N (b) 10 N µ= 0.8 v g
30°
(c) 0.8 N (d) zero 8 A horizontal force of 10 N is necessary to just hold a
3 Two spheres A and B of masses m1 and m2 respectively block stationary against a wall. The coefficient of friction
collide. A is at rest initially and B is moving with velocity v between the block and the wall is 0.2. The weight of the
v block is
along X-axis. After collision, B has a velocity in a
2
F
direction perpendicular to the original direction. The
mass A moves after collision in the direction 10N
(a) same as that of B (b) opposite to that of B
−1  1 
(c) θ = tan   to the X-axis
 2
−1  −1
(d) θ = tan   to the X-axis (a) 20 N (b) 50 N
 2
(c) 100 N (d) 2 N
4 Four blocks of same mass connected by cords are
pulled by a force F on a smooth horizontal surface, as 9 A heavy uniform chain lies on a horizontal top of table. If
shown in the figure. The tensionsT1, T2 and T3 will be the coefficient of friction between the chain and the table
T1 T2 T3
is 0.25, then the maximum percentage of the length of
F ↔↔ M ↔↔ M ↔↔ M ↔↔ M the chain that can hang over one edge of the table is
1 3 1 1 1 1
(a) T1 = F, T2 = F, T3 = F (b) T1 = F, T2 = F, T3 = F (a) 20% (b) 25%
4 2 4 4 2 2 (c) 35% (d) 15%
3 1 1 3 1 1
(c) T1 = F, T2 = F, T3 = F (d) T1 = F, T2 = F, T3 = F 10 A 0.5 kg ball moving with a speed of 12 ms −1
4 2 4 4 2 2
strikes a hard wall at an angle of 30° with the
5 The coefficient of static

30°
2kg wall. It is reflected with the same speed and at
friction µs , between block A
the same angle. If the ball is in contact with the
A of mass 2 kg and the wall for 0.25 s, the average force acting on the
table as shown in the T
wall is
30°

figure is 0.2. What would


B (a) 48 N (b) 24 N
be the maximum mass (c) 12 N (d) 96 N
value of block B, so that
the two blocks do not 11 Two rods X and Y are attached to a
move? The string and the pulley are assumed to be weight of mass M as shown in X
smooth and massless (take, g = 10 ms −2 ) figure, then
(a) 2.0 kg (b) 4.0 kg (c) 0.2 kg (d) 0.4 kg (a) both X and Y experience
6 A block of mass m is placed on a smooth wedge of compression
Y
inclination θ. The whole system is accelerated (b) both X and Y experience
horizontally, so that the block does not slip on the extension
wedge. The force exerted by the wedge on the block (c) Y experiences extension and X
M
(g is acceleration due to gravity) will be compression
mg (d) Y experiences compression and
(a) mg cosθ (b) mg sin θ (c) mg (d) X extension
cosθ
40 40 DAYS ~ NEET PHYSICS DAY FOUR

12 With what minimum acceleration can a fireman slide 15 A block of mass m is placed on a smooth inclined wedge
2
down a rope while breaking strength of the rope is of ABC of inclination θ as shown in the figure. The wedge is
3 given an acceleration a towards the right. The relation
the weight? between a and θ for the block to remain stationary on the
2 1 wedge is j NEET 2018
(a) g (b) g (c) g (d) Zero
3 3 A
m
13 Figure shows a uniform rod of length 30 cm having a
mass of 3.0 kg. The strings shown in the figure are pulled
a
by constant forces of 20 N and 32 N. Find the force
q
exerted by 20 cm part of the rod on the 10 cm part. All C B
the surfaces are smooth and the strings and pulleys are g
(a) a = g cos θ (b) a =
light j AFMC 2010 sin θ
g
F F (c) a = (d) a = g tan θ
cosec θ
20 N 10 cm 20 cm 32 N 16 Three forces acting on a body are shown in the figure. To
have the resultant force only along the y-direction, the
(a) 36 N (b) 12 N
(c) 64 N (d) 24 N magnitude of the minimum additional force needed is
j CBSE AIPMT 2008
14 Two blocks A and B of masses 3m and m
respectively are connected by a massless 4N
and inextensible string. The whole system is

30°
suspended by a massless spring as shown A 3m
60°
in figure. The magnitudes of acceleration of
B m
A and B immediately after the string is cut,
are respectively j
NEET 2017
g g g g 2N
(a) g, (b) , g (c) g, g (d) ,
3 3 3 3 3
(a) 0.5 N (b) 1.5 N (c) N (d) 3 N
4

ANSWERS
SESSION 1 1 (c) 2 (b) 3 (b) 4 (b) 5 (b) 6 (d) 7 (d) 8 (d) 9 (d) 10 (d)
11 (c) 12 (d) 13 (c,d) 14 (c) 15 (b) 16 (c) 17 (a) 18 (a) 19 (b) 20 (b)
21 (b) 22 (c) 23 (b) 24 (b) 25 (b) 26 (d) 27 (a) 28 (a) 29 (a) 30 (d)
31 (a) 32 (c) 33 (c) 34 (d) 35 (b) 36 (c) 37 (c) 38 (b) 39 (c)

SESSION 2 1 (a) 2 (d) 3 (c) 4 (c) 5 (d) 6 (d) 7 (a) 8 (d) 9 (a) 10 (b)
11 (d) 12 (c) 13 (d) 14 (b) 15 (d) 16 (a)

Hints and Explanations


SESSION 1 2 Velocity acquired in falling through This is total retardation due to gravity
height h and cardboard.
1 v = u + 2as or v = 2as [Qu = 0] …(i)
2 2 2
∴ Retardation due to cardboard
Force on the ship, u = 2gh = 2 × 10 × 5 = 10 ms −2
a′ = g + a = (10 + 25) ms −2 = 35 ms −2
F = ma …(ii) Again, v 2 = u 2 + 2as, Force due to card board = M a ′
From Eqs. (i) and (ii), we get
⇒ (0)2 = (10)2 + 2a × 2, = 200 × 10−3 × 35
2Fs 2 × 5 × 104 × 3
v = = 100 = 7000 × 10−3 = 7 N
m 3 × 107 a= − = − 25 ms −2
2×2
. ms −1
= 01
DAY FOUR LAWS OF MOTION 41

3 Here, m = 5 kg; Force 11 The area under F - t graph gives change 17 Since, balloon is descending
F = (−3 i$ + 4 $j ) N in momentum. B
 −3 i 4j  1
So, a =
F
= + N For 0 to 2 s, ∆ p1 = × 2 × 6 = 6 kg -m/s
m  5 5  2
a mg – B=ma ...(i)
For 2 to 4 s, ∆ p2 = 2 × − 3 = − 6 kg - m/s
Final velocity is along y-axis only when
For 4 to 8 s, ∆ p3 = 4 × 3 = 12 kg - m/s
its x-component of velocity is zero. So,
using vx = u x + ax t So, total change in momentum for 0 to 8 s mg
3 6× 5 ∆ p net = ∆ p1 + ∆ p2 + ∆ p3
⇒ 0= 6− ×t⇒ t = = 10 s
5 3 = (+ 6 − 6 + 12) B
= 12 kg -m/s = 12 N-s
4 The magnitude and direction of pseudo
force depends upon acceleration of frame 12 Resultant force is zero, as three forces a B – (m–m0)g=(m–m0)a ...(ii)
of reference in which observer is situated. acting on the particle has been
It does not depend upon direction and represented in magnitude and direction
by three sides of a triangle in same (m–m0)g
magnitude of acceleration of the block.
order. Hence, by Newton’s second law On adding, Eqs. (i) and (ii), we get
i.e. the magnitude of pseudo force on
 F = m dv  , particle velocity (v) will be ⇒ mg – mg + m 0 g = ma + ma – m 0 a
the block is ma1 .  
 dt  2ma
⇒ m0 =
5 Time taken for 1 bullet = 1 same. g+a
n
Force = the rate of change of momentum 13 One can come off the frictionless surface 18 When lift is stationary,
= mvn following the law of conservation of
R = mg
momentum i.e. by splitting or sneezing
= 10 × 20 × 10 = 2000 dyne or throwing an object in opposite 49 = m × 9.8
49
6 Here, F = 6i$ − 8$j + 10k$ direction of motion. ⇒ m= = 5 kg
9.8
|F| = 36 + 64 + 100 14 According to the law of conservation of If a is downward acceleration of lift
momentum, p i = p f
= 10 2 N R = m(g − a)
⇒ (0.01 ) × 400 + 0 = 2 v + (0.01) v ′ ...(i)
R = 5 (9.8 − 5) = 24 N
a = 1 ms −2 Also, velocity v of the block just after
the collision is 19 Total mass (m) = Mass of lift + Mass of
10 2
∴ m= [QF = ma] v = 2gh = 2 × 10 × 0.1 = 2 ...(ii) person = 940 + 60 = 1000 kg
1 So, from the free body diagram
= 10 2 kg ⇒ From Eqs. (i) and (ii), we have T − mg = ma
7 2m ′ vn = mg v ′ ≈ 120 m/s Hence, T − 1000 × 10 = 1000 × 1
∴ velocity of each bullet 15 Concept Momentum is conserved T = 11000 N
mg 100 × 980
v = = = 980 cms −1 before and after collision. 20 Here, lift is accelerating upward at the
2m ′ n 2 × 5 × 10 We have, rate of a.
p1 + p2 + p3 = 0 [Q p = mv ] Hence, equation of motion is
8 From law of conservation of linear ∴ 1 × 12 $i + 2 × 8$j + p3 = 0 written as R − mg = ma
momentum,
⇒ 12 $i + 16$j + p = 0 28000 − 20000 = 2000a
momentum of boat = momentum of dog 3
⇒ p3 = − (12 $i + 16$j ) [Qg = 10 ms −2 ]
m1 v 1 = m2 v 2
Given, m1 = 40 kg ∴ p3 = (12)2 + (16)2
−1 R
v 1 = ?, m2 = 4 kg, v 2 = 10 ms = 144 + 256 = 20 kg-m/s a
∴ 40 × v 1 = 4 × 10 Now, p3 = m3 v3
4 × 10 p 20
⇒ v1 = = 1 ms −1 ⇒ m3 = 3 = = 5 kg
40 v3 4
9 From law of conservation of momentum mg
16 Acceleration of man in left a man = a k$
m1 v 1 = m2 v 2 8000
Acceleration of ball ⇒ a ball = g k$ ⇒ a= = 4 ms −2 upwards
Given, m1 = 0.25kg, v 1 = 100 m/s, 2000
m2 = 100 kg Here, k$ represents downwards direction
⇒ Relative acceleration 21 According to the question
0.25 × 100 = 100 × v 3 mg
v = 0.25m/s a rel = a base − a man = (g − a) k$ =
2 m (g − a)
∴ Acceleration observed by man
10 As we know that, impulse is imparted g
in lift = g − a 3 g − 3 a = 2g ⇒ a =
due to change in perpendicular 3
components of momentum of ball. Again for man standing outside
a man = 0 k$ 22 F sin30° + N = Mg
J = ∆p = mv f − mv i
= mv cos 60°− (− mv cos 60° ) So, a = g k$ − 0 k$ = g k$
relative The block lifts when N = 0
1 ∴ Acceleration observed by man 10 × 10
= 2mv cos 60° = 2mv × = mv ∴ F = = 200 N
2 standing outside = g 1 /2
42 40 DAYS ~ NEET PHYSICS DAY FOUR

23 Tension 29 Since, all the blocks are moving with 35 a


2m1 m2 2 × 4 × 2 × 10 constant velocity, then the net force on m1 I
T = g =
m1 + m2 4+ 2 the all blocks will be zero. A
160 µm1g
= = 26. 6 ≈ 27 N 30 From equation of motion,
6 v = u − at
Total downward thrust on the pulley
⇒ 0 = u − µgt m1
= 2T = 2 × 27 = 54 N ⇒ Coefficient of friction µm1g a
B
u 6
24 m1 m2 m3 µ = = = 0.06
T gt 10 × 10
m2 g
10 kg 6 kg 4 kg
31 Friction, f = µ mg = 0.8 × 4 × 10 = 32 N For the motion of both blocks
Applied force F < f , therefore answer m2 g – T = m2 a …(i)
From the relation, F = ma
will be (a).
a=
F T – µ k m1 g = m1 a …(ii)
m1 + m2 + m3 32 (m2 – µ k m1 )g
40 R L/2 ⇒ a=
⇒ a= = 2 ms −2 m1 + m2
10 + 6 + 4 F
L/2 For the block of mass m2 ,
40 − T = 10 × 2
T = 20 N θ θ  m – µ k m1 
sin m2 g – T = m2  2
mg mg mg cos θ g
25 The system of masses is shown below θ  m1 + m2 
T2 T1 The block may be stationary, when  m – µ k m1 
C B A F T = m2 g –  2  m2 g
L
mg sin θ ⋅ L = µmg cos θ  m1 + m2 
2
mg sin θ ⋅ L sin θ  m + µ k m1 
From the figure, or µ = =2 = m2 g  1 
F − T1 = ma …(i) mg cos θ
L cos θ  m1 + m2 
and T1 − T2 = ma …(ii) 2 ⇒ Tension in the string
Eq. (i) gives 10.2 − T1 = 2 × 0.6 = 2 tanθ
m1 m2 (1 + µ k )g
⇒ T1 = 10.2 − 1.2 = 9 N Coefficient of friction, µ = 2 tan θ T =
m1 + m2
Again from Eq. (ii), we get
9 − T2 = 2 × 0.6
33 When cart moves F
with some
36 Acceleration
⇒ T2 = 9 − 1.2 = 7.8 N m1 g – µ (m2 + m3 )g m[g – 2µg ]
acceleration a= =
m1 + m2 + m3 3m
26 Since, no force of friction is present. So, towards right, then mα m R
no horizontal force is present on body a pseudo force g
= (1 – 2µ )
of mass m. In vertical direction normal (ma) acts on block 3
force balances weight of the body. towards left. mg a a
Hence, net force on top body must be This force (mα ) is P
zero. action force by a block on cart. Now, µm2g µm3g
block will remain static with respect to
27 Acceleration of system cart, if frictional force a
F net 14 m1
= = = 2 m/s2 µR ≥ mg .
M total 4+ 2+ 1
⇒ µm α ≥ mg [ R = mα] m 1g
g
14 N ⇒ α≥
4 kg 2 kg 1 kg µ 37 Friction force between blocks A and B
and between block B and surface will
34 The opposing force that comes into play oppose F
The contact force between 4 kg and 2 kg when one body is actually sliding over
block will move 2 kg and 1kg block the surface of the other body is called ∴ F = F AB + F BS
with the same acceleration. sliding friction. = µ AB M A g + µ BS (m A + m B ) g
So, Fcontact = (2 + 1) a = 3(2) = 6 N The coefficient of sliding is given as = 0.2 × 100 × 10
N
µS = + 0.3 (100 + 200) × 10
28 In stationary position, F sliding
mg = 49 = 200 + 900 = 1100 N
49 where, N is the normal reaction and This is the required minimum force to
m= = 5 kg
9.8 F sliding is the sliding force. move the block B.
When lift moves downwards reading of As, the dimensions of N and F sliding are 38 Given a plank with a box on its one end
balance same. Thus, µ S is a dimensionless is gradually raised about the end having
quantity. angle of inclination is 30°, the box starts
T = mg − ma to slip and slides down 4 m the plank in
= 5 (9.8 − 5) = 24.0 N Hence, statement(d) is incorrect. 4 s as shown in figure.
DAY FOUR LAWS OF MOTION 43

3 Here, p i = m2 v$i + m1 × 0 6 Let an acceleration to the wedge is given


v towards left, then the block (being in
p f = m2 $j + m1 × v1 non-inertial frame) has a pseudo
2
acceleration to the right because of
mg Law of conservation of momentum
which the block is not slipping
θ v
p i = p f ⇒ m2 v$i = m2 $j + m1 × v1 N
2
The coefficient of static friction, θ
m m v os
µ s = tan 30° =
1
= 0.6 v1 = 2 v i$ + 2 $j aps c
m1 m1 2 θ θ a
3 sin
y mg θ aps
So, distance covered by a plank,
1 mg mg cos θ
s = ut + at 2 v/2 θ
2 B
u=0 v
Here, x ∴ mg sin θ = apseudo cos θ
and a = g (sin θ − µ cos θ) B A (m1) θ
mg sin θ
1 (m2) Rest A u ⇒ apseudo =
∴ 4 = g (sin 30° − µ k cos 30° ) (4)2 (m1) cos θ
2
From this equation, we can find Hence, total force exerted by the wedge
1 3
⇒ 0.5 = 10 × − µ K × 10 ×
= , θ = tan −1   to the
y 1 1 on the block is
2 2 tanθ =
x 2  2 N = N1 + N2
⇒ 5 3 µ k = 4.5
X-axis. = mg cos θ + apseudo sin θ
⇒ µ k = 0.51 mg sin θ
(m2 + m3 + m 4 ) = mg cos θ + × sin θ
Thus, coefficient of kinetic friction 4 T1 = F
cos θ
between the box and the plank is 0.51. m1 + m2 + m3 + m 4
mg cos 2 θ + mg sin2 θ mg
Given, m1 = m2 = m3 = m 4 = m = =
39 Retardation = g (sin 60° + µ cos 60° ) cos θ cos θ
3 7 Retardation due to friction = − µg
∴ T1 = ⋅F
–1

4
ms

Initial velocity = v
(m3 + m 4 )F
20

Similarly, T2 = Now using v = u + at


v=

m1 + m2 + m3 + m 4 Final velocity is zero,


v
Given, m1 = m2 = m3 = m 4 = m ⇒ 0 = v − µgt ⇒ time, t =
µg
1
∴ T2 = ⋅F 8 Force, F = µR = Mg
2
60° ∴ Weight of block = µR = 0.2 × 10 = 2 N
m4 F
Also, T3 =
 3
m1 + m2 + m3 + m 4 9 M lg = µ M ( L − l ) g ⇒ l = ( L − l ) µ
1 L L
= 10  + µ  = 5( 3 + µ )
 2 2 Given, m1 = m2 = m3 = m 4 = m l µ
=
v = u − at ⇒ T3 =
1
F L 1+ µ
0 = 20 − 5( 3 + µ ) × 2 4 0.25  1
= =  × 100 % = 20%
3+ µ =2 5 Let the mass of the block B is M. 1.25  5 
µ = 2 − 1.732 ≈ 0.27 10 Change in momentum
R
µmg T
SESSION 2 2kg
B
1 We have, mass m = 2 kg
30°

x = pt + qt 2 + rt 3 mg T
60° 60°

dx
⇒ = p + 2qt + 3rt 2
O

dt M
30°

d2 x
= 2q + 6 rt = acceleration (a)
dt 2 In equilibrium, T = Mg …(i) A
At t = 2 s,
If block do not move, then T = fs
a = 2q + 12r = 2 × 4 + 12 × 5
where, fs = frictional force ∆p = OB sin 30° − (− OA sin 30° )
= 68 m / s 2
= µ s R = µ s mg = mv sin 30° − (− mv sin 30° )
Now,|F| = |ma| = 2 × 68 = 136 N ∴ T = µ s mg …(ii) = 2 mv sin 30°
2 We know that, tan α = 0.8 Thus, from Eqs. (i) and (ii), we have
Its time rate will appear in the form of
Mg = µ s mg
α = tan −1 (0.8) = 39° average force acting on the wall.
or M = µs m
The given angle of inclination is less ∴ F × t = 2 mv sin 30°
Given, µ s = 0.2, m = 2 kg
than angle of repose. So, 1 kg block has 2mv sin 30°
∴ M = 0.2 × 2 or F =
no tendency to move. Therefore, T = 0. t
= 0.4 kg
44 40 DAYS ~ NEET PHYSICS DAY FOUR

Given, m = 0.5 kg, v = 12 ms −1 , ∴ kx = 4mg Dividing Eq. (i) by Eq (ii), we get


When string is cut at the location as mg sin θ ma a
and t = 0.25 s = ⇒ tan θ =
shown above. mg cos θ mg g
θ = 30°
2 × 0.5 × 12 sin30° Free body diagram for m is
Hence, F = = 24 N or a = g tan θ
0.25 m
∴The relation between a and g for the
11 By writing the equilibrium equations in block to remain stationary on the wedge
horizontal and vertical direction mg is a = g tan θ.
(assuming same angle θ between So,force on mass m = mg 16 y
(0 ≤ θ ≤ 90° ) x and y. We can find that 4 cos 30°+1 sin 60°
∴ Acceleration of mass, m = g
rod x is under extension (tension), while 1N
For mass 3m; free body diagram is
rod y is under compression. 4N
kx=4mg
12 If man slides down with same 30°
acceleration, then its apparent weight 60°
decreases. For critical condition rope a 3m x′ x
2 1 cos 60°+2 sin 30°
can bear only of his weight. If a is 30°
3
minimum acceleration. 3 mg 2N
2
Breaking strength = m (g − a) = mg If a = acceleration of block of mass 3m, y′
3 then F net = 4mg − 3mg
2g g g Breaking all the forces in X and Y-axis.
⇒ a= g − = ⇒ 3m ⋅ a A = mg or a A =
3 3 3 Total force along (+ x ) axis
So, accelerations for blocks A and B are = (1cos 60° + 2sin 30° )
13 F F
aA =
g
and aB = g along (− X ) axis = (4sin 30° )
3 along (+ Y ) axis = (4cos 30° + 1sin 60° )
20 N 10 cm 20 cm 32 N
15. According to the question, the FBD of along (−Y ) axis = (2cos 30° )
Net force on the rod, f = 32 − 20 = 12 N the given condition will be ⇒ Net force along X -axis
F 12 = − (1cos 60° + 2sin 30° ) + 4sin 30°
Acceleration, a = = = 4 ms −2 A
m 3
⇒ −  + 2 ×  + 4 ×
R cosq 1 1 1
Equation of motion of 10 cm apart is R
q
2 2 2
⇒  m ′ = × m = × 3 = 1kg 
1 1 −3 1
 3 3 
ma R sinq ⇒ + 2= +
(Pseudo 2 2
F − 20 = m ′ a = 1 × 4 force)
q
Net force along Y-axis
F = 4 + 20 = 24 N mg q a
= 4cos 30° + 1sin 60° − 2cos 30°
Similarly, equation of motion of 20 cm B C
apart is 32 − F = m ′ a = 2 × 4 = 4×
3
+
3
−2×
3
F = 32 − 8 = 24 N Since, the wedge is accelerating towards 2 2 2
right with a, thus a pseudo force acts in 5 3 2 3
14 Initially system, is in equilibrium with a the left direction in order to keep the = −
total weight of 4mg over spring. 2 2
block stationary. As, the system is in
3 3
equilibrium. =
∴ ΣF x = 0 or ΣF y = 0 2
kx
⇒ R sin θ = ma To have, resultant only in Y-axis we
(3m+m) 1
or mg sin θ = ma …(i) must have N force towards + X -axis,
A 3m 2
Cutting Similarly, R cos θ = mg so that it can compensate the net force
4 mg plane
B m or mg cos θ = mg …(ii) of − X axis.
DAY FIVE

Circular
Motion
Learning & Revision for the Day
u Concept of Circular Motion u Forces in Circular Motion
u Dynamics of Uniform Circular Motion u Applications of Circular Motion

Concept of Circular Motion


Circular motion is a two dimension motion. To bring circular motion in a body it must be
given some initial velocity and a force. Circular motion can be classified into two types-
Uniform circular motion and Non uniform circular motion.
When an object moves in a circular path at a constant speed then the motion is said to
be a uniform circular motion.
When an object moves in a circular path with variable speed, then the motion is said to
be non-uniform circular motion.

Terms Related to Circular Motion


1. Angular Displacement
It is defined as the angle turned by the particle from some reference line. Angular
displacement ∆θ is usually measured in radians. PREP
Finite angular displacement ∆θ is a scalar but an infinitesimally small displacement is
a vector.
MIRROR
Your Personal Preparation Indicator

2. Angular Velocity u No. of Questions in Exercises (x)—


v2
It is defined as the rate of change of the angular u No. of Questions Attempted (y)—
displacement of the body. u No. of Correct Questions (z)—
B (Without referring Explanations)
From figure a particle moving on circular track of radius r is r2
showing angular displacement ∆θ in ∆t time and in this time
∆θ Accuracy Level (z / y × 100)—
period, it covers a distance ∆s along the circular track, then v1 u
O
Prep Level (z / x × 100)—
 ∆θ  dθ
u

Angular velocity, ω = lim   = r1


∆t→ 0  ∆t  dt A In order to expect good rank in NEET, your
It is an axial vector whose direction is given by the right Accuracy Level should be above 85 & Prep
Level should be above 75.
hand rule. Its unit is rad/s.
46 40 DAYS ~ NEET PHYSICS DAY FIVE

3. Angular Acceleration Applications of Circular Motion


It is the rate of change of angular velocity. Some of the most important applications of centripetal and
dω d2θ centrifugal forces are given below
Thus, α= =
dt dt 2
Motion of a Vehicle on a Level Circular Road
Its unit is rad/s2 .
When a vehicle negotiates a circular path, it requires a
centripetal force.
Dynamics of Uniform In such cases the lateral force of friction may provide the
Circular Motion required centripetal force. Thus, for maintaining its circular
path required centripetal force,
If a particle is performing circular motion with a uniform
speed, then motion of the particle is called uniform circular  mv2 
  ≤ frictional force (µ mg)
motion. In such a case,  r 
dv v2 Maximum speed vmax = µrg
= 0 and ac = ω2 r = [Q v = rω]
dt r where, µ = coefficient of friction between road and vehicle
Thus, if a particle moves in a circle of radius r with a uniform tyres and r = radius of circular path.
v2
speed v, then its acceleration is towards the centre. This
r
acceleration is termed as centripetal acceleration. Bending of a Cyclist
When a cyclist goes round turns in a circular track, then angle
NOTE • In non-uniform circular motion Resultant acceleration of
made by cyclist with vertical level is given by
v4
the body is a = ar2 + aT2 = + r2 α2 v2
r2 tan θ =
rg
aT
 v2 
a Tangential θ = tan −1  
acceleration  rg 
O ar
Radial Banking of a Curved Road
acceleration
For the safe journey of a vehicle on a curved (circular) road,
without any risk of skidding, the road is slightly raised
towards its outer end.
Forces in Circular Motion
Let the road be banked at an angle θ from the horizontal, as
In circular motion of an object two kinds of forces occur shown in the figure.
which are described below
If b is width of the road and h is height of the outer edge of the
road as compared to the inner edge, then
Centripetal Force
v2 h
The centripetal force is the force required to move a body tan θ = =
rg b
along a circular path with a constant speed. The direction of
the centripetal force is along the radius, acting towards the
centre of the circle, on which the given body is moving.
R R cos θ
Centripetal force,
mv2 4π 2  1 θ
F = = mrω2 = mr 4π 2 ν2 = mr 2 Qv = R sin θ A
r T 
 T  Outer
edge
Work done by centripetal force is always zero as it is F raised
perpendicular to velocity and hence instantaneous θ mg
O X
displacement. B

If friction is present between road and tyre, then Maximum


Centrifugal Force speed,
Certrifugal force can be defined as the radially directed rg (µs + tan θ)
outward force acting on a body in circular motion, as observed vmax = ,
1 − µs tan θ
by a person moving with the body.
where, µs = coefficient of static friction.
mv2
Mathematically, centrifugal force = = mrω2
r
DAY PRACTICE SESSION 1

FOUNDATION QUESTIONS EXERCISE


1 For a particle in a non-uniform accelerated circular What is the magnitude of the centripetal acceleration of
motion correct statement is the stone while in circular motion?
(a) velocity is radial and acceleration is transverse (a)163 ms −2 (b) 64 ms −2
(b) velocity is transverse and acceleration is radial only (c)15.63 ms −2 (d) 125 ms −2
(c) velocity is radial and acceleration has both radial and 9 A particle moves in a circle of radius 5 cm with constant
transverse components speed and time period 0.2 π s. The acceleration of the
(d) velocity is transverse and acceleration has both radial particle is j CBSE AIPMT 2011

and transverse components


(a) 25 m/s 2 (b) 36 m/s 2
2 An athlete completes one round of a circular track of (c) 5 m/s 2 (d) 15 m/s 2
radius 10 m and time period 40 s. The distance covered
10 In the given figure, a = 15 m/s 2 represents the total
by him in 2 min 20 s is
acceleration of a particle moving in the clockwise
(a) 70 m (b) 140 m (c) 110 m (d) 220 m direction in a circle of radius R = 2.5 m at a given instant
3 A car wheel is rotated to uniform angular acceleration of time. The speed of the particle is j NEET 2016

about its axis. Initially its angular velocity is zero. It


rotates through an angle θ1 in the first 2s, in the next 2s, it
θ
rotates through an additional angle θ 2, the ratio of 2 is 30°
θ1 R
a
(a) 1 (b) 2 O
(c) 3 (d) 5
4 Which of the following statements is false for a particle
moving in a circle with a constant angular speed.
(a) 4.5 m/s (b) 5.0 m/s (c) 5.7 m/s (d) 6.2 m/s
(a) The velocity vector is tangent to the circle.
(b) The acceleration vector is tangent to the circle. 11 A uniform circular disc of radius 50 cm at rest is free to
(c) The acceleration vector point to the centre of the circle turn about an axis which is perpendicular to its plane and
(d) The velocity and acceleration vectors are perpendicular passes through its centre. It is subjected to a torque
to each other.
which produces a constant angular acceleration of
 20 2.0 rad s −2. Its net acceleration a in ms −2 at the end of
5 A particle moves along a circle of radius   m with
 π 2.0 s is a approximately j
NEET 2016
constant tangential acceleration. If the velocity of the (a) 7.0 (b) 6.0 (c) 3.0 (d) 8.0
particle is 80 m/s at the end of the second revolution
12 A particle of mass 100 g tied to a string is rotated along
after motion has begain, the tangential acceleration is
a circle of radius 0.5 m. The breaking tension of string is
(a) 160π m/s 2 (b) 40 m/s 2 (c) 40π m/s 2 (d) 640π m/s 2 10 N. The maximum speed with which particle can be
6 An electric fan has blades of length 30 cm measured rotated without breaking the string is
from the axis of rotation. If the fan is rotating at 120 (a) 10 ms −1 (b) 9.8 ms −1
rev/min, the acceleration of a point on the tip of the (c) 7.7 ms −1 (d) 7.07 ms −1
blade is 13 A proton of mass 1.66 × 10−27 kg goes round in a circular
(a) 1600 m/s 2 (b) 47.4 m/s 2 (c) 23.7 m/s 2 (d) 50.55 m/s 2 orbit of radius 0.10 m under a centripetal force of
7 An aircraft executes a horizontal loop of radius 1 km with 4 × 10−13 N, then the frequency of revolution of the proton
a speed of 900 kmh −1. Ratio of centripetal acceleration to is about
acceleration due to gravity is (a) 0.08 × 108 cycle/s (b) 4 × 108 cycle/s
(a) 12.3 (b) 3.3 (c) 8 × 108 cycle/s (d) 12 × 108 cycle/s
(c) 6.4 (d) None of these 14 If the radius of curvature of the path of two particle of
8 A boy whirls a stone in a horizontal circle of radius 1.5 m same masses are in the ratio 1 : 2, then in order to have
and at height 2.0 m above level ground. The string constant centripetal force, their velocity should be in the
breaks and the stone flies off tangentially and strikes the ratio of
ground after travelling a horizontal distance of 10 m. (a) 1 : 4 (b) 4 : 1 (c) 4 : 2 (d) 1 : 2
48 40 DAYS ~ NEET PHYSICS DAY FIVE

15 One end of the string of length l is connected to a particle of 19 A cyclist goes round a circular path of length 400 m in
mass m and the other end is connected to a small peg on a 20 s. The angle through which he bends from vertical in
smooth horizontal table. If the particle moves in circle with order to maintain the balance is
speed v, the net force on the particle (directed towards center) (a)sin−1 (0.64) (b) tan−1 (0.64)
will be (T represents the tension in the string) j NEET 2017 (c)cos−1 (0.64) (d) None of these
mv 2 mv 2 20 A car of mass 1000 kg negotiates a banked curve of
(a)T (b)T + (c)T − (d) Zero
l l radius 90 m on a frictionless road. If the banking angle is
16 A coin placed on a rotating turn table just slips, if it is 45°, the speed of the car is j CBSE AIPMT 2012

placed at a distance of 8 cm from the centre. If angular (a) 20 ms −1 (b) 30 ms −1 (c) 5 ms −1 (d) 10 ms −1
velocity of the turn table is doubled, it will just slip at a
21 Find the maximum velocity with which a train can be
distance of
moved on a circular track of radius 200m. The banking of
(a) 1 cm (b) 2 cm (c) 4 cm (d) 8 cm
the track is 5.71° (Given, tan 5.71° = 0.1) j AFMC 2011

17 What will be the maximum speed of a car on a road turn (a) 1.4 m/s (b) 14 m/s (c) 140 m/s (d) 0.14 m/s
of radius 30 m, if the coefficient of friction between, the
tyres and the road is 0.4? 22 A cyclist riding the bicycle at a speed of 14 3 m/s takes
a turn a circular road of radius 20 3 m without skidding.
(Take, g = 9.8 m/s 2)
What is his inclination to the vertical?
(a) 10.84 m/s (b) 9.84 m/s
(a) 30° (b) 90° (c) 48° (d) 60°
(c) 8.84 m/s (d) 6.84 m/s
23 A road of width 20 m forms an arc of radius 15 m, its
18 The maximum velocity with which a car driver must
outer edge is 2 m higher than its inner edge. For what
traverse a flat curve of radius 150 m and coefficient of velocity the road is banked?
friction 0.6 to avoid skidding?
(a) 10 ms −1 (b) 14 .7 ms −1
(a) 60 (b) 30
(c) 15 (d) 25 (c) 9.8 ms −1 (d) None of these

DAY PRACTICE SESSION 2

PROGRESSIVE QUESTIONS EXERCISE


1 A cyclist starts from centre O of a circular park of radius 4 A stone of mass 0.25 kg tied to the end of a string is
1 km and moves along the path Q whirled round in a circle of radius 1.5 m with speed
OPRQO as shown in given figure. If R 40 rev/min in a horizontal plane. What is the tension in the
he maintains constant speed of string ? What is the maximum speed with which the stone
10 m/s, what is his acceleration at P can be whirled around, if the string can withstand a
O
point R in magnitude and direction? maximum tension of 200 N?
(a) 0.1 m/s 2 , along RO (b) 0.2 (a) 4.4 N, 32.0 m/s (b) 6.6 N, 34.6 m/s
m/s 2 , along OR (c) 4.4 N, 42.8 m/s (d) 6.6 N, 24.8 m/s
(c) 0.3 m/s 2 , along RO (d) 0.4 m/s 2 , along RO 5 Two stones of masses m and 2m are whirled in horizontal
2 A car rounds a curved road of radius 150 m at a speed r
circles, the heavier one in a radius and the lighter one
of 20 m/s. Calculate the angle of banking, so that there is 2
no side thrust on the tyres. Also, find the elevation of the in radius r. The tangential speed of lighter stone is n
outer wheels over the inner wheels, if the distance times that of the value of heavier stone when they
between them is 1 m. experience same centripetal forces. The value of n is
j
CBSE AIPMT 2015
(a) 15.22°, 0.428 m (b) 15.22°, 0.272 m
(c) 10°, 4.186 m (d) 10°, 3.581 m (a) 2 (b) 3 (c) 4 (d) 1
3 A 200 kg car has to go ever a turn whose radius is 750 m 6 The small spherical balls are free to move on the inner
and the angle of slope is 5°. The coefficient of friction surface of the rotating spherical chamber of radius
between the car wheels and the road is 0.5. What should R = 0. 2 m. Sphere is rotating about the axes XX′ with
be the maximum speed of the car? So, that it may go angular velocity ω. If the balls reach a steady state at
over the turn without slipping. Take g = 9.8 m/s 2. an angular position θ = 45°, the angular speed ω of
(a) 67.2 m/s (b) 60.2 m/s (c) 72.2 m/s (d) 76.2 m/s device is
DAY FIVE CIRCULAR MOTION 49

X ω g g 1 1
3r (a) (b) (c) g (d) g
R sinα R cosα R cosα R sinα
11 A 70 kg man stands in contact against the inner wall of a
R θ R hollow cylindrical drum of radius 3 m rotating about its
θ θ vertical axis with 200 rev/min. The coefficient of friction
between the wall and his clothing is 0.15. When the floor
X′
(a) 8 rad/s (b) 2 rad/s
is suddenly removed, what is the minimum rotational
(c) 3.64 rad/s (d) 9.34 rad/s speed of the cylinder to enable the man to remain stuck
to the wall without falling (approximately)?
7 A heavy small-sized sphere is suspended by a string of
(a) 5 rad/s (b) 10 rad/s
length l. The sphere rotates uniformly in a horizontal
(c) 15 rad/s (d) 20 rad/s
circle with the string making an angle θ with the vertical.
Then, the time-period of this conical pendulum is 12 A national roadway bridge over a canal is in the form of
an arc of a circle of radius 49 m. What is the maximum
l l sin θ
(a) t = 2 π (b) t = 2 π speed with which a car can move without leaving the
g g
ground at the highest point? (take, g = 9.8 ms −2 )
l cos θ l
(c) t = 2 π (d) t = 2 π (a) 19.6 ms −1 (b) 40 ms −1
g g cos θ (c) 22 ms −1 (d) None of these
8 A stone tied to the end of a string of 1 m long is whirled 13 A motorcycle moving with a velocity 72 kmh −1 on a flat
in a horizontal circle with a constant speed. If the stone road takes a turn on the road at a point where the radius
makes 22 revolutions in 44 s, what is the magnitude and of curvature of the road is 20 m. The acceleration due to
direction of acceleration of the stone? gravity is 10 ms −2. In order to avoid skidding, he must not
π2 bent with respect to the vertical plane by an angle
(a) ms −2 and direction along the radius towards the
4 greater than
centre (a) θ = tan−1 (2) (b) θ = tan−1 (6)
(b) π 2 ms −2 and direction along the radius away from centre (c) θ = tan−1 (4) (d) θ = tan−1(25.92)
(c) π 2 ms −2 and direction along the radius towards the 14 A car is moving in a circular horizontal track of radius
centre 10.0 m with a constant speed of 10.0 ms −1. A plumb bob
(d) π 2ms −2 and direction along the tangent to the circle is suspended from the roof of the car by a light rigid rod
9 A cyclist is riding with a speed of 27 km/h. As he of length 1.00 m. The angle made by the rod with the
approaches a circular turn on the road of radius 80 m, he track is (take, g = 10 ms −2). j
AFMC 2010
applies brakes and reduces his speed at the constant (a) zero (b) 30°
rate of 0.5 m/s every second. What is the magnitude of (c) 45° (d) 60°
the net acceleration of the cyclist on the circular turn? 15 A car is negotiating a curved road of radius R. The road
(a) 0.5 m/s 2
(b) 0.86 m/s 2
(c) 0.72 m/s 2
(d) 0.3 m/s 2 is banked at angle θ. The coefficient of friction between
the tyres of the car and the road is µs . The maximum safe
10 A hemispherical bowl of radius R is rotating about its axis
velocity on this road is j
NEET 2016
of symmetry which is kept vertical. A small ball kept in the
bowl rotates with the bowl without slipping on its surface. If  µ + tanθ  g  µ s + tanθ 
(a) gR  s  (b)  
the surface of the bowl is smooth and the angle made by  1 − µ s tanθ  R  1 − µ s tanθ 
the radius through the ball with the vertical is α. Find the g  µ s + tanθ   µ + tanθ 
(c)   (d) gR 2  s 
angular speed at which the bowl is rotating. R 2  1 − µ s tanθ   1 − µ s tanθ 

ANSWERS
SESSION 1 1 (d) 2 (d) 3 (c) 4 (b) 5 (b) 6 (b) 7 (c) 8 (a) 9 (c) 10 (c)
11 (d) 12 (c) 13 (a) 14 (d) 15 (a) 16 (b) 17 (a) 18 (b) 19 (b) 20 (b)
21 (b) 22 (d) 23 (b)

SESSION 2 1 (a) 2 (b) 3 (a) 4 (b) 5 (a) 6 (c) 7 (c) 8 (d) 9 (b) 10 (b)
11 (a) 12 (c) 13 (a) 14 (c) 15 (a)
50 40 DAYS ~ NEET PHYSICS DAY FIVE

Hints and Explanations


SECTION 1 ∴a =
(250) 2
= 62.5 ms −2 and g = 9.8ms −2
QLinear acceleration at the end of 2 s.
1000 a t = αr = 2 × 0.5 ⇒ a t = 1 m/s2
1 In non-uniform circular motion particle
a 62.5
possess both radial as well as transverse ∴ = = 6.4 Therefore, the net acceleration at the
acceleration and velocity of particle is g 9.8 end of 2.0 s is given by
transverse.
8 Given, r = 1.5 m, h = 2 m and d = 10 m a= ac2 + a2t
2 Time period = 40 sec
2h 2×2 a = (8)2 + (1)2 = 65
No of revolution t = = = 0.64 s
=
Total time
=
140
= 3.5 rev g 9.8 ⇒ a ≈ 8 m/s 2

time period 40 10
v = = 15.62 ms −1 12 Given, m = 100 g , r = 0.5 m, F = 10 N
So, distance t
d = 3.5 × 2 πR = 3.5 × 2 π × 10 = 220 m v 2 (1562. )2 mv 2
∴ a= = Centripetal force, F = ,
r 1.5
3 α = ω and ω = θ ⇒ α = θ2
r
− 163 ms −2
= 162.65 ~
t t t v =  rF 
 
but α = constant 9 Acceleration, a = rω2  m
θ1 (2) 2 θ1 1 where, ω is angular frequency given by 0.5 × 10 × 1000
So, = or = =
θ1 + θ2 (2 + 2) 2 θ1 + θ2 4 2π
ω= 100
θ 1 + θ2 4 θ 4 T
= 50 ms −1 = 7.07 ms −1
or = or 1 + 2 = 4 π2
θ1 1 θ1 1 As, a= ω r ⇒a= 2 r
2
T 13 Given M = 1.66 × 10−27 kg
θ2
∴ =3 Given, r = 5cm = 5 × 10−2 m, r = 010
. m
θ1
T = 0.2 π s F = 4 × 10–13 N
4 × π2 × 5 × 10−2
4 The acceleration vector is not tangent to ∴ a= = 5ms −2 mv 2
the circle. (0.2 π )2 Centripetal force F =
r
5 Initial velocity, u = 0 10 Centripetal acceleration of a particle F = m × 4 π2 n2 r
Final velocity, v = 80 m/s moving on a circular path is given by 4 × 10−13 = m × 4 π2 × n2 r
v2
Radius of circle, r =   m
20
ac = Frequency
 π R
4 × 10−13
Distance travelled In the given figure, n= −27
ac = a cos 30° = 15 cos 30° m/s2 1.66 × 10 × 4 × (314
. )2 × 010
.
s = 2 × (2 πr ) = 2 2 π ×  = 80 m
20
 π v2 = 0.08 × 108 cycle/s
⇒ = 15 cos 30°
Now, by applying third equation of R 2

motion 3
14 The centripetal force, F = mv
⇒ v 2 = R × 15 × r
v 2 = u2 + 2as 2 mv 2
(80)2 = 0 + 2 × at × 80 r =
3
= 2.5 × 15 × F
at = 40 m/s2 2 ∴ r ∝ v 2 or v ∝ r
∴ v = 5.7 m/s [If m and F are constant]
6 Centripetal acceleration of rotating body
11 According to given v1 r1 1 1
is given by = = =
question, a v2 r2 2 2
v2 r 2ω2 0.5m
aC = = = rω2 uniform circular
r r disc of radius 15 Consider the string of length l
aC = 4rπ2 v 2 50 cm at rest is connected to a particle as shown in the
Here, r = 30 cm = 30 × 10−2 m = 030
. m free to turn about figure.
120 an axis which is
v = 120 rev/min = rev/s = 2 rev/s peg particle
60 perpendicular to its plane and passes v
aC = (030
. × 4 × 314
. × 314
. × 2 × 2) through its centre. l
= 47.4 m/s2 This situation can be shown by the Speed of the particle is v. As the particle
figure given below is in uniform circular motion, net force
7 Centripetal acceleration is given by
∴ Angular acceleration, α = 2 rad s −2 on the particle must be equal to
v2
a= (given) centripetal force which is provided by
r
Angular speed, ω = αt = 4 rad s −1 the tension (T ).
Given, r = 1 km = 1000 m ∴ Net force = Centripetal force
1000 Q Centripetal acceleration, ac = ω2 r
v = 900 km/h = 900 × ms −1 mv 2
3600 = (4)2 × 0.5 = 16 × 0.5 ⇒ =T
= 250 ms −1
l
ac = 8 m/s2
DAY FIVE CIRCULAR MOTION 51

16 We know that, centrifugal force 23 Condition for velocity on banked road Frequency ν = 40 rev / min
F = mrω2 v2 40 2
∴ tan θ = ⇒v = rg tan θ = rev /s = rev /s
1 rg 60 3
rω2 = constant, ω2 ∝ Centripetal force required for circular
r 2
2
But, tan θ ≈ θ ≈ sin θ = motion is obtained from the tension in
ω  r 20 the string.
Given, ω2 = 2ω1 or  2  = 4 ⇒ 1 = 4 [for small angle]
 ω1  r2 ∴Tension in the string = Centripetal
Given r = 154 m force
∴ r2 = 2 cm [Q r1 = 8 cm]
∴ v = 15 × 9.8 ×
2 mv 2 m(ωr )2
T = = = mrω2
17 If v is the velocity of the vehicle while 20 r r
rounding the curve, the centripetal force = 14.7 ms −1 = mr (2 π ν)2 (Q ω = 2 πν)
mv 2
required = = mr 4 π2 ν2
r SESSION 2 2 2
= 0.25 × 1.5 × 4 ×   ×   = 6.6 N
As this force is provided only by the 22 2
force of fricition, 1 Acceleration of the cyclist at point R  7  3
mv 2 = Centripetal acceleration (ac ) Maximum tension which can be with
≤ µmg ⇒ v 2 ≤ µrg
r v2 (10)2 100 stood by the string
ac = = =
v ≤ µrg ⇒ v max = µrg r 1000 1000 T max = 200 N
∴ = 01
. m/s2 , along RO mv 2max
v max = 0.4 × 30 × 9.8 ∴ T max =
r
= 10.84 m/s 2 Here, v = 20 m/s, r = 150 m; l = 1 m. If T max × r 200 × 1.5
⇒ v max =
2
= = 1200
18 Given, r = 150 m, µ = 0.6 θ is angle of banking, then m 0.25
Maximum velocity v = µrg ∴ v max = 1200 = 34.6 m/s
= 0.6 × 150 × 10 = 30 m/s 5 Given, that two stones of masses m and
19 Given, l = 400 m, t = 20 sec 2 m are whirled in horizontal circles,
mg r
θ the heavier one in a radius and lighter
Total length of circle, l = 2πr 2
mg mv 2 one in radius r as shown in figure.
400 = 2 πr sin θ =
2 cos θ r
 400 
  v2 (20)2 v2
v 2
 20  6.28 ⇒ tan θ = =
Qtanθ = = = = 0.64 rg 150 × 9.8
rg  400  9.8
  g
= 0.2721
r/2
2m
 2π 
∴ −1
θ = tan (0.64) θ = 15.22°
Suppose that, h is the distance through r
2
20 Angle of banking, tanθ = v which the outer part of the wheel has to
m
rg be raised with respect to the inner rail. v1
h
Given, θ = 45° , r = 90 m, g = 10 m/s2 sin θ = As, lighter stone is n times that of the
l value of heavier stone when they
v2
∴ tan 45° = Since, θ is small, sin θ ≈ tan θ experience same centripetal forces, we
90 × 10 h v2
∴ = (Q tan θ = v 2 / rg ) get (Fc )heavier = (Fc )lighter
⇒v = 90 × 10 × tan 45° rg
l 2m (v )2 m (nv )2
= 900 = 30 m/s ⇒ = ⇒ n2 = 4 ⇒ n = 2
v2
20 × 1 2
(r / 2) r
⇒ h= l=
2 rg 150 × 9.8
21 Angle of banking, tanθ = v 6 Given, R = 0.2 m
rg = 0.272 m
Given, r = 200 m, θ = 5.71 ω
3 Given, m = 200 kg, r = 750 m,
⇒ v = rg tanθ
θ = 5° , µ s = 0.5
N sin 45°
= 200 × 10 × tan 5.71 The maximum speed,
1 /2
= 200 × 10 × 0.1  rg (µ s + tan θ) N °
v max =  45
= 14.14 m/s = 14 m/s 
 (1 − µ s tan θ)  mrω2 N cos 45°
1 /2
22 Given, v = 14 3 m/s, r = 20 3  750 × 9.8(0.5 + tan 5° ) R sin 45º
= 3R
 (1 − 0.5tan 5° ) 
v2 
−1 
Angle θ = tan   1 /2
mg
 rg  750 × 9.8(0.5 + 0.087)
=
r
 (14 3 )  2  1 − 0.5 × 0.087  r = 3R + R sin 45°
= tan −1  
 20 3 × 9.8  v max = 672 . m/s N cos 45° = mrω2 and N sin 45° = mg
mg g
= tan −1 ( 3 ) 4 Mass of a stone m = 0.25 kg ⇒ tan 45° = = 2
mrω2 rω
θ = 60° Radius of the string r = 1.5 m
52 40 DAYS ~ NEET PHYSICS DAY FIVE

g g Radius of the circular turn (r) = 80 m 12 Given, R = 49 m,


⇒ ω= =
r R ∴Centripetal acceleration acting on the
3R + v = gR = 9.8 × 49
2 cyclist
−1
= 3.64 rad/s v 2 (15 / 2)2 = 21.9 ms ≈ 22 ms −1
ac = =
r 80
7 Radius of circular path in the horizontal 13 Using the formula for motorcycle not to
225
plane = m/s2 skid
4 × 80  v2 
r = l sin θ θ = tan −1  
Resolving T along the vertical and = 070
. m/s2  rg 
horizontal directions, we get, Tangential acceleration applied by where, r = 20 m
T cos θ = Mg ...(i) brakes a T = 0.5 m/s2
v = 72 kmh −1
T sin θ = Mrω2 = M (l sin θ) ω2 Centripetal acceleration and tangential 5
= 72 × = 20 ms −1
or T = Mlω2 ...(ii) acceleration acts perpendicular to each
18
other.
 20 × 20 
∴ Resultant acceleration, ∴ θ = tan −1  
θθ  20 × 10 
a= a2c + a2T = (07
. )2 + (0.5)2
T l or θ = tan −1 (2)
T cos θ = 0.49 + 0.25 = . = 0.86 m/s2
074
14 Angle of banking is
10 Balancing the forces, we get
mv 2 / r v2
A Tsinθ r B ω tan θ = ⇒ tanθ =
mg rg

Mg Given, v = 10.0 ms –1 , r = 10 m
On dividing Eq. (ii) by Eq. (i), we get l = 1.00 m
R
lω2 g = 10 ms –2
1
= or ω2 =
g N α
cos θ g l cos θ 90 – α (10)2
⇒ tan θ = = 1 ⇒ θ = 45°
2π l cos θ r 10 × 10
∴ Time period, t = = 2π
ω g mg 15 According to question, a car is
negotiating a curved road of radius R.
8 Since, speed is constant throughout the The road is banked at angle θ and the
motion, so it is a uniform circular N sin (90 − α ) = mg …(i)
coefficient of friction between the tyres
motion. Therefore, its radial and, N cos (90 − α ) = m ω2 r
of car and the road is µ s . So, this given
acceleration Also, r = R sin α situation can be drawn as shown in
2
2 π n 4 π2 n2 N = mω 2 R
a = r ω = r 
So, …(ii) figure below.
 =r×
2
 t  t2 N cos θ
From Eq. (i) and (ii), we get
Given, r = 2 m, n = 22, t = 44 sec m ω 2R N g
1 × 4 × π2 × (22)2 = ⇒ω =
= = π2 ms −2 mg N cos α R cos α θ
N sin θ
(44)2
11 Cylinder being vertical, the normal Fl cos θ
and direction along the tangent to the θ
circle reaction of the wall on the man acts fl
horizontally and provides the necessary fl sin θ
a
centripetal force, N = mrω2 θ
mg
The frictional force f , acting upwards
O m balances his weights f = mg
r Considering the case of vertical
Man will remain stuck to the wall after equilibrium
the floor is removed i.e. he will N cos θ = mg + f l sin θ
continue to rotate with the cylinder ⇒ mg = N cos θ − f l sin θ …(i)
9 Speed of the cyclist (v) = 27 km/h without slipping, if f ≤ µ N Considering the case of horizontal
5 Q 1 km/ h = 5 m/s  mg ≤ µ mrω2 equilibrium,
= 27 × m/s
18  18  g g mv 2
ω2 ≥ ⇒ω ≥ N sin θ + f l cos θ = …(ii)
15 µr µr
= m/s R
2 Minimum angular speed of rotation of Divide Eqs. (i) and (ii), we get
aT the cylindrical drum is v2 sin θ + µ s cos θ
= [ f1 = Nµ s ]
ωmin =
g Rg cos θ − µ s sin θ
a µr
θ
 sin θ + µ s cos θ 
O Given, µ = 015
. , r = 3 m, g = 9.8 m/s2 ⇒ v = Rg  
ac  cos θ − µ s sin θ 
9.8
∴ ωmin = = 4.67 rad s −1  tan θ + µ s 
. ×3
015 ⇒ v = Rg  
−1  1 − µ s tan θ 
≈ 5 rad s
DAY SIX

Work, Energy
and Power
Learning & Revision for the Day
u Work u Energy u Work-Energy Theorem u Power
u Conservative and u Law of Conservation of Mechanical u Collision
Non-conservative Force Energy

Work
When a body is displaced actually through some distance in the direction of applied
force, then work is said to be done by the force. The SI unit of work is joule (J) and in
CGS is erg.
1 joule (J) = 107 erg

Work Done by a Constant Force


The work done by the force F in displacing the body through a distance s is
W = (F cos θ)s = Fs cos θ = F ⋅ s
where, F cos θ is the component of the force, acting along the direction of the
displacement produced. SI unit of work is joule (J).
1 J = 1 N-m
Work is a scalar quantity. Work can be of three types
(i) Positive work (ii) Negative work and (iii) Zero work.
PREP
MIRROR
l
Positive work If value of the angle θ between the directions of F and s is either zero or Your Personal Preparation Indicator
an acute angle.
u No. of Questions in Exercises (x)—
l
Negative work If value of angle θ between the directions of F and s is either 180° or u No. of Questions Attempted (y)—
an obtuse angle. u No. of Correct Questions (z)—
l
As work done W = F ⋅ s = F s cos θ, hence work done can be zero, if (Without referring Explanations)
(i) No force is being applied on the body, i.e. F = 0.
u Accuracy Level (z / y × 100)—
(ii) Although the force is being applied on a body but it is unable to cause any
displacement in the body, i.e. F ≠ 0 but s = 0.
u Prep Level (z / x × 100)—

(iii) Both F and s are finite but the angle θ between the directions of force and
In order to expect good rank in NEET, your
displacement is 90°. In such a case Accuracy Level should be above 85 & Prep
W = F ⋅ s = F s cos θ = F s cos 90 ° = 0 Level should be above 75.
54 40 DAYS ~ NEET PHYSICS DAY SIX

Work Done by Variable Force 2. Potential Energy


Work done by a variable force is given by W = ∫ F ⋅ ds Potential energy is the energy stored in a body or a system
by virtue of its position in a field of force or due to its
B configuration. Potential energy is also called mutual
Force
A energy or energy of the configuration.
Value of the potential energy in a given position can be
defined only by assigning some arbitrary value to the
D C reference point. Generally, reference point is taken at
Displacement infinity and potential energy at infinity is taken as zero. In
r
It is equal to the area under the force-displacement graph, that case, U = − W = − ∫ F ⋅ dr

along with proper sign.
Potential energy is a scalar quantity. It may be positive as
Work done = Area of ABCDA
well as negative.
Different types of potential energy are given below.
Conservative and Gravitational Potential Energy
Non-Conservative Force Gravitational potential energy of a body is the energy
A force is said to be conservative if work done by or against possessed by a body by virtue of its position above the surface
the force in moving a body depends only on the initial and of the earth.
final positions of the body and not on the nature of path i.e. Gravitational potential energy = mgh
followed between the initial and the final position. where, m is the mass of a body at a height h above a reference
Gravitational force, force of gravity, electrostatic force are level and g is acceleration due to gravity.
some examples of conservative forces (fields).
A force is said to be non-conservative if work done by or
Potential Energy of a Spring
against the force in moving a body from one positions to Whenever an elastic body (say a spring) is either stretched or
another, depends on the path followed between these two compressed, work is being done against the elastic spring
positions. Force of friction and viscous force are the examples 1
force. The work done is W = kx2 ,
of non-conservative forces. 2
where, k is spring constant and x is the displacement.
1
Energy And elastic potential energy, U = k x2
2
Energy is defined as the capacity or ability of a body to do If spring is stretched from initial position x 1 to final position
work. Energy is scalar and its units and dimensions are the x2 , then
same as that of work. Thus, SI unit of energy is joule (J). work done = Increment in elastic potential energy
Some other commonly used units of energy are 1
= k ( x22 − x21 ).
1 erg = 10 −7 J, 1 cal = 4.186 J ~
− 4.2 J, 2
1 kcal = 4186 J, 1 kWh = 3.6 × 10 6 J
Work Done in Pulling the Chain Against Gravity
and 1 electron volt = (1 eV) = 1.60 × 10 −19 J
If point mass m is pulled through a
height h, then work done W = mgh.
1. Kinetic Energy For a chain, we can consider its
Kinetic Energy (KE) is the capacity of a body to do work by centre of mass at the middle point of L/n
virtue of its motion. A body of mass m, moving with a the hanging part i.e. at a height of
1  L
velocity v, has a kinetic energy, K = mv2 .   from the lower end and the
2  2 n
Kinetic energy of a body is always positive irrespective of M
mass of hanging part of chain, m = .
the sign of velocity v. Negative kinetic energy is n
impossible. Kinetic energy is correlated with momentum as So, workdone to raise the centre of mass of the chain on the
p2 table is given by
K =
2m M L MgL
W = ×g × ⇒W =
or p = 2 mK . n 2n 2 n2
DAY SIX WORK, ENERGY AND POWER 55

Power is a scalar quantity and its SI unit is watt, where


Work-Energy Theorem 1 W = 1 J /s
Accordingly, work done by all the forces (conservative or dW F ⋅ ds
non-conservative, external or internal) acting on a particle or Instantaneous power, Pinst = = = F⋅v
an object is equal to the change in its kinetic energy of the dt dt
particle. Thus, we can write W = ∆K = K f − K i Some other commonly used units of power are
We can also write, K f = K i + W 1 kW = 103 W,1 MW = 10 6 W and 1 HP = 746 W.
Which says that  Kinetic energy after 
 the net work is done
Collision
=  the net work done  +  work done
Kinetic energy before The net
    The physical interaction of two or more bodies in which each
equal and opposite forces upon each others causing the
exchange of energy and momentum is called collision.
Law of Conservation of Collisions are classified as
Mechanical Energy (i) elastic collisions and (ii) inelastic collisions.
The mechanical energy E of a system is the sum of its kinetic
energy K and its potential energy U.
1. Perfectly Elastic Collision in
E = K +U
When the forces acting on the system are conservative in
One Dimension
nature, the mechanical energy of the system remains constant, In a perfectly elastic collision, total energy and total linear
K + U = constant ⇒ ∆K + ∆U = 0 momentum of colliding particles remain conserved.
There are physical situations, where one or more non- Moreover, the forces involved in interaction are
conservative force act on the system but net work done by conservative in nature and the total kinetic energy before
them is zero, then the mechanical energy of the system and after the collision, remains unchanged.
remains constant. If Σ W net = 0
u1 u2 v1 v2
Mechanical energy, E = constant. m1 m2 m1 m2

Motion in a Vertical Circle A


Before Collision
B A
After Collision
B

In this motion body is under the influence of gravity of earth


and total mechanical energy remains conserved In above figure, two bodies A and B of masses m1 and m2
and having initial velocities u1 and u2 in one dimension,
(K. E. converts into P. E and vice-versa).
collide elastically and after collision move with velocities
l
Velocity at any point on vertical loop v1 and v2 , then we find that
v = u2 − 2 g(1 − cos θ)r (i) Relative velocity of approach = Relative velocity of
separation, i.e. u1 − u2 = v2 − v1
where, u is the initial velocity at lowest point and r is
length of the string.  m − m2   2 m2 
(ii) v1 =  1  u1 +   u2
C  m1 + m2   m1 + m2 
 2 m1   m − m1 
and v2 =   u1 +  2  u2
O  m1 + m2   m1 + m2 
D r B
θ

mg cos θ +
mv 2 2. Perfectly Elastic Collision in a Plane
u r
A In a two dimensional (or oblique) collision between two
l
Tension at any point on vertical loop bodies, momentum remains conserved,
v1
mv2 m
T − mg cos θ = or T = [u2 − gr (2 − 3 cos θ)]
r r m1
A
Power A B θ
m1 u1
Power is defined as the rate of doing work. If an agent does u1
work W in time t , then its average power is given by Before collision
m2 φ
W B
Pav =
t m2
The shorter is the time taken by a person or a machine in v2
performing a particular task, the larger is the power of that
After collision
person or machine.
56 40 DAYS ~ NEET PHYSICS DAY SIX

∴ Along the X -axis If 0 < e < 1, the collision is said to be partially elastic. But if
m1u1 + m2u2 = m1v1 cos θ + m2 v2 cos φ … (i) e = 0, then collision is said to be perfectly inelastic.
and along the Y-axis In a perfectly inelastic collision, e = 0 which means that
0 = m1v1 sin θ − m2 v2 sin φ … (ii) v2 − v1 = 0 or v2 = v1.
As the total kinetic energy remains unchanged. It can be shown that for an inelastic collision the final
1 1 1 1 velocities of the colliding bodies are given by
Hence, m1u21 + m2u22 = m1v21 + m2 v22 …(iii)  m − em2  (1 + e) m2
2 2 2 2 v1 =  1  u1 + u2
We can solve these equations provided that either the  m1 + m2  (m1 + m2 )
value of θ or φ is known to us. (1 + e) m1  m − em1 
and v2 = u1 +  2  u2 .
(m1 + m2 )  m1 + m2 
3. Inelastic Collision If a particle of mass m, moving with velocity u, hits an
In an inelastic collision, the total linear momentum as well identical stationary target inelastically, then final velocities of
as total energy remain conserved but total kinetic energy projectile and target are correlated as
after collision is not equal to kinetic energy before collision. v 1 −e
For the type of collision i.e. m1 = m2 = m and u2 = 0; 1 = .
v2 1 + e
m1v1
Common speed v =
m1 + m2
m1m2 (v1 − v2 )2
Rebounding of a Ball on Collision
and loss of kinetic energy ∆K =
2(m1 + m2 ) with the Floor
Here v2 = 0 l
Speed of the ball after the nth rebound
m1m2 v21 vn = e n v0 = e n 2 gh0
∴ ∆K =
2(m1 + m2 ) l
Height covered by the ball after the nth rebound
v1 v2 v hn = e2 n h0
m1 m2 m1 m2 l
Total distance (vertical) covered by the ball before it stops
Before collision v2=0 After collision bouncing
 1 + e2 
H = h0 + 2 h 1 + 2 h2 + 2 h3 + … = h0  
1 − e 
2

Coefficient of Restitution (e) l


Total time taken by the ball before it stops bouncing
For a collision, it is defined as the ratio of relative velocity of
separation to the relative velocity of approach. 2 h0 2 h1 2 h2
T = t 0 + t 1 + t 2 + t 3 + …= +2 +2 + ...
v − v1 g g g
Thus, coefficient of restitution e = 2
u1 − u2
2 h0  1 + e 
=  
For a perfectly elastic collision (e) = 1. g 1 − e
DAY PRACTICE SESSION 1

FOUNDATION QUESTIONS EXERCISE


1 A bicyclist comes to a skidding stop in 10 m. During this 8 Velocity-time graph of
process, the force on the bicycle due to the road is particle of mass 2 kg v (m/s)
200 N and is directly opposed to the motion. The work moving in a straight line is 20
done by the cycle on the road is as shown in figure. Work
(a) +2000 J (b) -200 J (c) Zero (d) -20000 J done by all the forces on
2 A force F acting on an object varies with distance x as the particle is
2 t (s)
shown here. The force is in newton and x is in metre. The (a) 400 J (b) - 400 J
(c) - 200J (d) 200J
work done by the force in moving the object from x = 0 to
x = 6 m is 9 A ball of mass 2 kg and another of mass 4 kg are
dropped together from a 60 ft tall building. After a fall of
F (N)
3 30 ft each towards the earth, their respective kinetic
energies will be in the ratio of
2
(a) 2 : 1 (b) 1 : 4 (c) 1 : 2 (d) 1 : 2
1
10 The momentum of a body is increased by 20%. The
0 x (m) percentage increase in kinetic energy is
1 2 3 4 5 6 7
(a) 54% (b) 44% (c) 100% (d) 50%
(a) 4.5 J (b)13.5 J (c) 9.0 J (d) 18.0 J
11 A body of mass m is accelerated uniformly from rest to a
3 A particle moves from a point (–2 $i + 5 $j ) to ( 4 $j + 3 k$ ) speed v in a time T . The instantaneous power delivered
when a force of ( 4 $i + 3 $j ) N is applied. How much work to the body as a function of time is, given by
has been done by the force? j
NEET 2016 mv 2 mv 2 1 mv 2 1 mv 2 2
(a) t (b) t2 (c) t (d) t
(a) 8 J (b) 11 J (c) 5 J (d) 2 J T 2
T 2
2 T2 2 T2
4 Force F on a particle moving in F (N) 12 The power supplied by a force acting on a particle
a straight line varies with moving in a straight line is constant. The velocity of the
distance d as shown in the particle varies with the displacement x as
figure. The work done on the 2 (a) x1/ 2 (b) x
particle during its displacement (c) x 2 (d) x1/ 3
of 12 m is j
CBSE AIPMT 2011
(a) 21 J (b) 26 J 13 A force F acting on a particle F(N)
O 3 7 12 d(m)
(c) 13 J (d) 18 J varies with the position x as
10
5 A uniform force of ( 3i$ + $j ) N acts on a particle of mass shown in the figure. The work
2 kg. Hence, the particle is displaced from position done by this force in
–2
displacing the particle from x(m)
( 2i$ + k$ ) m to position ( 4$i + 3$j - k$ ) m. The work done by 2
x = - 2 m to x = 0 is
the force on the particle is j
NEET 2013
(a) - 10 J (b) + 5 J –10
(a) 9 J (b) 6 J (c) 13 J (d) 15 J
(c) - 20 J (d) + 40 J
6 A particle moves along the X -axis from x = 0 to x = 5 m
under the influence of a force given by F = 7 - 2x + 3x 2 . 14 A particle is moved from Y
The work done in the process is ( 0 , 0) to (a , a ) under a
(a) 70 J (b) 270 J force F = ( 3 $i + 4 $j ) from P (a, a)
(c) 35 J (d) 135 J two paths. Path 1 is OP
7 The potential energy of a system increases, if work is and path 2 is OQP. Let
done j
CBSE AIPMT 2011 W1 and W2 be the work 45°
done by this force in O Q x
(a) by the system against a conservative force
(b) by the system against a non-conservative force these two paths. Then,
(c) upon the system by a conservative force (a)W1 = W2 (b)W1 = 2W2
(d) upon the system by a non-conservative force (c)W2 = 2W1 (d)W2 = 4W1
58 40 DAYS ~ NEET PHYSICS DAY SIX

15 An engine pumps water continuously through a hose. 23 A block of mass 10 kg, moving in x-direction with a
Water leaves the hose with a velocity v and m is the mass constant speed of 10 ms -1, is subjected to a retarding
per unit length of the water jet. What is the rate at which force F = 0.1x J/m during its travel from x = 20 m to 30 m.
kinetic energy is imparted to water ? Its final KE will be j CBSE-AIPMT 2015

1 3 1 2 1 2 2 (a) 475 J (b) 450 J (c) 275 J (d) 250 J


(a) mv (b) mv3 (c) mv (d) m v
2 2 2
24 A small block of super dense material has mass
16 300 J of work is done in sliding a 2 kg block up an 3 ´ 10 24 kg. It is situated at a height h ( < < Re ) from
inclined plane of height 10 m. Taking g = 10 ms -2 , work where it falls on the earth’s surface. Its speed when its
done against friction is h
(a) 200 J (b) 100 J (c) zero (d) 1000 J height has reduced to is (given mass of the earth is
2
17 The potential energy of a particle in a force field is 6 ´ 10 24 kg)
A B
U= - , where A and B are positive constants and r is 2gh 3
r2 r (a) (b) gh (c) zero (d) gh
3 2
the distance of particle from the centre of the field. For
stable equilibrium, the distance of the particle is 25 A block of mass M is attached to the lower end of a
j CBSE AIPMT 2012 vertical spring. The spring is hung from a ceiling and has
force constant value k. The mass is released from rest
(a) B/2A (b) 2A/B (c) A/B (d) B/A
3 -1
with the spring initially unstretched. The maximum
18 A spring of spring constant 5 ´ 10 Nm is stretched extension produced in the length of the spring will be
initially by 5 cm from the unstretched position. Then, the j CBSE AIPMT 2009
work required to stretch it further by another 5 cm is
(a) Mg/k (b) 2 Mg/k (c) 4 Mg/k (d) Mg/2k
(a) 12.50 N-m (b) 18.75 N-m (c) 25.00 N-m (d) 6.25 N-m
26 The block of mass M M
19 Two similar springs P and Q have spring constants kP moving on the frictionless
and kQ , such that kP > kQ . They are stretched, first by the horizontal surface collides
same amount (case a), then by the same force (case b). with the spring of spring
The work done by the springs WP and WQ are related as, constant k and compresses it by length L. The maximum
in case (a) and case (b), respectively j CBSE AIPMT 2015 momentum of the block after collision is
(a)WP = WQ ; WP > WQ (b)WP = WQ ; WP = WQ kL2 ML2
(c)WP > WQ ; WQ > WP (d)WP < WQ ; WQ < WP (a) Mk L (b) (c) zero (d)
2M k
20. When a bullet of mass 10 g and speed 100 ms -1
27 A body of mass 1 kg is thrown upwards with a velocity
penetrates up to distance 1 cm in a human body in rest.
20 ms-1. It momentarily comes to rest after attaining a
The resistance offered by human body is
height of 18 m. How much energy is lost due to air
(a) 2000 N (b)1500 N (c) 5000 N (d) 1000 N friction? (Take, g = 10 ms -2 ) j
CBSE AIPMT 2009
21 A block is left on a frictionless curve path that connects (a) 20 J (b) 30 J (c) 40 J (d) 10 J
to a horizontal path. At the end of the horizontal path,
28 The string of a pendulum is horizontal. The mass of bob
there is a spring connected. Find the work done by
attached to it is m. Now, the string is released. The
spring, when the block compress it completely.
tension in the string in the lowest position is
m
(a) mg (b) 2mg (c) 3mg (d) 4mg
29 A stone is tied to a string of length l and is whirled in a
k h
vertical circle with the other end of the string as the
centre. At a certain instant of time, the stone is at its
µ=0 lowest position and has a speed u. The magnitude of the
mk mk change in velocity as it reaches a position where the
(a) - (b) -mgh (c) mgh (d)
h h string is horizontal (g being acceleration due to gravity) is
22 Consider a drop of rain water having mass 1 g falling (a) 2 (u 2 - gl ) (b) u 2 - gl (c) v - u 2 - 2gl (d) 2gl
from a height of 1 km. It hits the ground with a speed of
30 A heavy stone hanging from a massless string of length
50 m/s. Take g constant with a value of 10 m/s2 . The work
15 m is protected horizontally with speed 147 m/s. The
done by the (i) gravitational force and the (ii) resistive
speed of the particle at the point where tension in the
force of air is j
NEET 2017
string equals the weight of the particle is
(a) (i) -10 J, (ii) - 8.25 J (b) (i) 1.25 J, (ii) - 8.25 J
(c) (i) 100 J, (ii) 8.75 J (d) (i) 10 J, (ii) - 8.75 J (a) 10 m/s (b) 7 m/s
(c) 12 m/s (d) None of these
DAY SIX WORK, ENERGY AND POWER 59

31 A stone of mass 1 kg is tied of a string 4m long and is 38 Three objects A, B and C are kept
m 2m m
rotated at constant speed of 40 m/s in a vertical circle. in a straight line on a frictionless
A B C
The ratio of the tension at the top and the bottom is horizontal surface. These have
(Take, g = 10 m/s 2 ) masses m, 2m and m, respectively. The object A moves
(a) 11 : 12 (b) 39 : 41 (c) 41 : 39 (d) 12 : 11 towards B with a speed 9 m/s and makes an elastic
5
32 A train of mass 2 ´ 10 kg has a constant speed of collision with it. Thereafter, B makes completely inelastic
æ 1 ö collision with C. All motions occur on the same straight
20 m/s up a hill inclined at θ = sin-1 ç ÷ to the horizontal line. Find the final speed (in m/s) of the object C.
è 50 ø
when the engine is working at 8 ´ 105 W. The resistance (a) 3 m/s (b) 4 m/s (c) 5 m/s (d) 1 m/s
to motion of train is 39 A particle A suffers on oblique elastic collision with a
(a) 400 N (b) 200 N (c) 600 N (d) 800 N particle B, i.e. at rest initially. If their masses are the
same, then after the collision
33 A particle of mass m is driven by a machine that delivers
a constant power k watts. If the particle starts from rest 1. their KE may be equal
2. a continuous to move in the original direction while B
the force on the particle at time t is j CBSE AIPMT 2015

remains at rest
mk -1/ 2
(a) t (b) mk t -1/ 2 3. they will move in mutually perpendicular directions
2
1 4. a comes to rest and B starts moving in the direction of
(c) 2mk t -1/ 2 (d) mk t -1/ 2 the original motion of A
2
(a) 1, 3 (b) 2, 3 (c) 1, 2 (d) 1, 2, 3
34 An engine pumps water through a hose pipe. Water
40 A body of mass ( 4 m ) is lying in xy-plane at rest. It
passes through the pipe and leaves it with a velocity of
suddenly explodes into three pieces. Two pieces each of
2 m s -1. The mass per unit length of water in the pipe is
mass m move perpendicular to each other with equal
100 kg m -1 . What is the power of the engine?
speeds v. The total kinetic energy generated due to
j
CBSE AIPMT 2010
explosion is j
CBSE AIPMT 2014
(a) 400 W (b) 200 W (c) 100 W (d) 800 W 3
(a) mv 2 (b) mv 2 (c) 2 mv 2 (d) 4 mv 2
35 Two particles of masses m1 and m2 move with initial 2
velocities u1 and u 2 . On collision, one of the particles get 41 For inelastic collision between two spherical rigid bodies
excited to higher level, after absorbing energy E . If final (a) the total kinetic energy is conserved
velocities of particles be v1 and v 2 , then we must have (b) the total mechanical energy is not conserved
j
CBSE AIPMT 2015 (c) the linear momentum is not conserved
(d) the linear momentum is conserved
(a) m12u1
+ m22u2
-E = +m12v1 m22v 2
1 1 1 1 42 A moving neutron collides with a stationary α-particle.
(b) m1u1 + m2u2 = m1v12 + m2v 22 - E
2 2
2 2 2 2 The fraction of the kinetic energy lost by the neutron is
1 1 1 1 (a) 16 / 25 (b) 9/25 (c) 3/5 (d) 2/5
c) m1u1 + m2u2 - E = m1v1 + m2v 22
2 2 2
2 2 2 2
43 A block of mass m moving at a velocity v collides with
1 1 1 1
(d) m12u12 + m22u22 + E = m12v12 + m22v 22 another block of mass 2m at rest. The lighter block comes
2 2 2 2 to rest after collision. Find the coefficient of restitution.
36 Two identical balls A and B having velocities of 0.5 m/s (a) 1/2 (b) 1 (c) 1/3 (d) 1/4
and – 0.3 m /s respectively, collide elastically in one 44 A ball is thrown vertically downwards from a height of
dimension. The velocities of B and A after the collision 20m with an initial velocity v 0. It collides with the ground,
respectively will be j
NEET 2016 loses 50 % of its energy in collision and rebounds to the
same height. The initial velocity v 0 is (Take, g = 10 ms -2 )
(a) –0.5 m/s and 0. 3 m/s (b) 0.5 m/s and –0. 3 m/s j
CBSE AIPMT 2015
(c) –0. 3 m/s and 0.5 m/s (d) 0. 3 m/s and 0.5 m/s
(a) 14 ms - 1 (b) 20 ms - 1
37 An explosion breaks a rock into three parts in a horizontal (c) 28 ms - 1 (d) 10 ms - 1
plane. Two of them go off at right angles to each other.
45 A bullet of mass m moving with velocity v strikes a block
The first part of mass 1kg moves with a speed of 12 ms -1
of mass M at rest and gets embedded into it. The kinetic
and the second part of mass 2 kg moves with 8 ms -1
energy of the composite block will be
speed. If the third part flies off with 4 ms -1 speed, then its
1 m 1 M
mass is j
NEET 2013 (a) mv 2 ´ (b) mv 2 ´
2 (m + M ) 2 (m + M )
(a) 3 kg (b) 5 kg 1 (M + m) 1 m
(c) 7 kg (d) 17 kg (c) mv 2 ´ (d) Mv 2 ´
2 M 2 (m + M )
60 40 DAYS ~ NEET PHYSICS DAY SIX

DAY PRACTICE SESSION 2

PROGRESSIVE QUESTIONS EXERCISE


1 A body of mass 10 kg is moving on an inclined plane of 7 What is the minimum velocity with which a body of mass
inclination 30° with an acceleration 2 m s -2 . The body m must enter a vertical loop of radius R so that it can
starts from rest, the work done by force of gravity in 2 s is complete the loop? j NEET 2016

(a) 10 J (b) zero (c) 98 J (d) 196 J (a) 2gR (b) 3gR
2 A body of mass 0.5 kg travels in a straight line with (c) 5gR (d) gR
velocity v = a x 3 / 2 , where a = 5 ms -2 . The work done by 8 A body of mass 1 kg begins to move under the action of
the net force during its displacement from x = 0 to x = 2 a time dependent force F = ( 2t i$ + 3t 2 $j ) N, where $i and $j
m is j NCERT Exemplar
are unit vectors along X and Y axes. What power will be
(a) 1.5 J (b) 50 J (c) 10 J (d) 100 J developed by the force at the time (t )? j NEET 2016

3 A particle free to move along X -axis has potential energy (a) (2 t 2 + 4 t 4 ) W (b) (2 t 3 + 3 t 4 ) W
-X 2 (c) (2 t 3 + 3 t 5 ) W (d) (2 t + 3 t 3 ) W
given as U( X ) = k (1 - e ), where k is a positive
constant of appropriate dimension. Then, for - a < X < ¥ 9 On a frictionless surface, a block of mass M moving at
(a) at points away from the origin, the particle is in unstable speed v collides elastically with another block of same
equilibrium mass M which is initially at rest. After collision the first
(b) for any finite non-zero value of x, there is a force block moves at an angle θ to its initial direction and has a
directed away from the origin v
k speed . The second block's speed after the collision is
(c) if its total mechanical energy is , it has its minimum 3
2 j
CBSE AIPMT 2015
kinetic energy at the origin
k 2 2 3
(d) if its total mechanical energy is , it has its maximum (a) v (b) v
2 3 4
kinetic energy at origin 3 3
(c) v (d) v
2 2
4. A particle of mass 10 g moves along a circle of radius
6.4 cm with a constant tangential acceleration. What is 10 A moving block having mass m, collides with another
the magnitude of this acceleration, if the kinetic energy of stationary block having mass 4m. The lighter block
the particle becomes equal to 8 ´ 10-4 J by the end of the comes to rest after collision. When the initial velocity of
second revolution after the beginning of the motion? the lighter block is v, then the value of coefficient of
(a) 0.15 m/s 2 . m/s 2
(b) 018 j
NEET 2016 restitution (e ) will be j
NEET 2018
(c) 0.2 m/s 2 (d) 0.1 m/s 2 (a) 0.8 (b) 0.25
(c) 0.5 (d) 0.4
5 A block of mass 10 kg is moving in x-direction with a
constant speed of 10 ms -1. It is subjected to a retarding 11 A mass m moves with a velocity v and collides
force F = - 0.1 x Jm -1, during its travel from x = 20 m to inelastically with another identical mass. After collision
x = 30 m. Its final kinetic energy will be v
the first mass moves with velocity in a direction
(a) 475 J (b) 450 J (c) 275 J (d) 250 J 3
6 A body initially at rest and sliding along a frictionless perpendicular to the initial direction of motion. Find the
track from a height h (as shown in the figure) just speed of the second mass after collision.
completes a vertical circle of diameter AB = D. The
height h is equal to j
NEET 2018 v

√3
v At rest

B m m
h
Before collision After collision
2 v
A (a) v (b)
3 3
7 3 5
(a) D (b) D (c) D (d) D (c) v (d) 3 v
5 2 4
DAY SIX WORK, ENERGY AND POWER 61

12 Water falls from a height of 60 m at the rate of 15 kg/s to (a) 2Avρ cos θ (b) 2 Av 2ρ cos θ
operate a turbine. The losses due to frictional forces are (c) 2 Av 2ρ (d) 2Avρ
10% of energy. How much power is generated by the 14 A mass of 100 g strikes the wall with speed 5 m/s at an
turbine? ( Take, g = 10 m /s 2 ) j CBSE AIPMT 2008
angle as shown in figure and it rebounds with the same
(a) 8.1 kW (b) 10.2 kW speed. If the contact time is 2 ´ 10-3 sec, then what is the
(c) 12.3 kW (d) 7.0 kW force applied on the mass by the wall?
13 An intense stream of water of cross-sectional area A
strikes a wall at an angle θ with the normal to the wall and
returns back elastically. If the density of water is ρ and its
velocity is v, then the force exerted in the wall will be 60°

60°

θ 100 g

(a) 250 3 N to right (b) 250 N to right


(c) 250 3 N to left (d) 250 N to left

ANSWERS
SESSION 1 1 (c) 2 (b) 3 (c) 4 (c) 5 (a) 6 (d) 7 (a) 8 (b) 9 (c) 10 (b)
11 (a) 12 (d) 13 (a) 14 (a) 15 (a) 16 (b) 17 (b) 18 (b) 19 (c) 20 (c)
21 (b) 22 (d) 23 (a) 24 (a) 25 (b) 26 (a) 27 (a) 28 (c) 29 (a) 30 (b)
31 (b) 32 (d) 33 (a) 34 (d) 35 (c) 36 (c) 37 (b) 38 (b) 39 (b) 40 (b)
41 (d) 42 (a) 43 (a) 44 (b) 45 (a)
SESSION 2 1 (d) 2 (b) 3 (d) 4 (d) 5 (a) 6 (d) 7 (c) 8 (c) 9 (a) 10 (b)
11 (a) 12 (a) 13 (b) 14 (c)

Hints and Explanations


SESSION 1 4 Work done = Area under (F -d ) graph 7 The potential energy of a system
1 As the road does not move at all, the 1 increases, if work is done by the
= 2 ´ ( 7 - 3) + ´ 2 ´ (12 - 7)
work done by cycle on road is zero. 2 system against a conservative force.
1 - DU = Wconservative force
= 8 + ´ 10 = 8 + 5 = 13 J
2 Work done in moving the object from 2
x = 0 to x = 6 m is given by 8 Work done by all forces = Change in
W = area of rectangle + area of 5 Given, force F = 3i$ + $j kinetic energy
r1 = (2$i + k$ )m 1 1
triangle = m (v 2f - v 2i ) = ´ 2(0 - 400)
1 and r2 = (4 $i + 3 $j - k$ )m 2 2
= 3 ´ 3 + ´ 3 ´ 3 = 9 + 4.5 = 13.5 J = - 400 J
2 \ Ds = r2 - r1 = (4 $i + 3 $j - k$ ) - (2 $i + k$ )
3 Position vectors of the particles are = (2 $i + 3 $j - 2 k$ ) m 9 v 1 = v 2 = v at a 30 ft from falling point.
r1 = -2$i + 5$j and r2 = 4$j + 3 k $ W = F × ds = (3 i$ + $j ) × (2 i$ + 3 $j - 2 k$ ) Here, m1 = 2 kg, m2 = 4 kg.
\ Displacement of the particle, = 3 ´ 2 + 3 + 0 = 6 + 3 = 9J 1
m1 v 2
K1
Ds = r - r1
2 6 Work done, dW = F × dx Thus, = 2
K2 1
= 4$j + 3 k
$ - (-2$i + 5$j ) = 2 $i - $j + 3 k$ 5 2 m2 v 2
Force on the particle, F = 4i + 3j N Þ W = ò 0 (7 - 2 x + 3 x )dx 2
\ Work done, W = F × Ds Þ W = [7x - x 2
+ x3 ] 50 m1 2 1
= = =
= (4$i + 3$j ) × (2$i - $j + 3 k
$ ) = 8 - 3 = 5J 2
\ W = 7 ´ 5 - (5) + (5) = 135 J 3 m2 4 2
62 40 DAYS ~ NEET PHYSICS DAY SIX

p2 17 Given, the potential energy of a particle 1


10 Increase in KE = ⇒ W air resistance = mv 2 - mgh
2m A B 2
in a force field, U = -
r2 r1 1
New momentum = p +
p 6p
= = 10-3 æç ´ 50 ´ 50 - 10 ´ 103 ö÷
5 5 dU è2 ø
For stable equilibrium, F = - =0
2 dr = -8.75J
æ 6 pö
ç ÷ dU
è5 ø 36 p2 = = - 2 Ar -3 + Br -2 23 Work done by force = DKE = KE f - KE i
KE f = = dr
2m 25 2m 2A B
0= - + æ As -dU = 0ö DW + KE i = KE f
36 p2 p2 11 p2 ç ÷
DKE = × - = r3 r2 è dr ø 30 1 2
25 2m 2m 25 ´ 2 m 2A ò20 - 0.1 x dx + 2 ´ 10 ´ 10 = KE f
or =B
r 30
Percentage increase = 44% x2 ½
The distance of particle from the centre -0.1 ½ + 500 = KE f
11 F = ma = mv 2A
2 ½20
T of the field r = 0.1
B - (900 - 400) + 500 = KE f
Instantaneous power = Fv = mav 2
mv mv v mv 2 18 W1 = 1 kx12 = 1 ´ 5 ´ 103 ´ (5 ´ 10-2 )2 -0.1 ´
500
+ 500 = KE f
= × at = × ×t = ×t 2 2 2
T T T T2 = 6.25 J -25 + 500 = KE f = 475 J
æ 2 ö 1
W2 = k ( x1 + x2 ) 2
12 QP = Fv = (ma) v = m ç d 2x ÷ æç dx ö÷ 2 24 Change in potential energy
è dt ø è dt ø 1 h h
= ´ 5 ´ 103 (5 ´ 10-2 + 5 ´ 10-2 )2 DPE = m1 g æç h - ö÷ = m1 g
Since, power is constant, 2 è 2ø 2
2
æ d 2 x ö æ dx ö d æ dx ö = 25J 1 2 1 2
\ ç ÷ ç ÷ = k or ç ÷ =k KE = m1 v 1 + m2 v 2
2 Net work done = W2 - W1 2 2
è dt ø è dt ø dt è dt ø
= 25 - 6.25 = 18.75 J = 18.75 N-m m
dx
2
dx m1 v 1 = - m2 v 2 Þ v 2 = - 1 v 1
or æç ö÷ = k1 t or = k1 t = k2 t 1 /2 m2
è dt ø dt 19 k P > kQ
From conservation principle, DPE = gain
3 /2 2 /3
or x = k3 t Þ t µ x In Case a Elongation ( X ) is same. in KE
2
dx 1 1 h 1 1 æm ö
Hence, µ t 1 /2 µ x1 /3 \ W P = k P X 2 , WQ = kQ X 2 m1 g = m1 v 12 + m2 ç 1 v 1 ÷
dt 2 2 2 2 2 m
è 2 ø
13 From the diagram, it is clear that from \ W P > WQ 2 m1 2 1 3
gh = v 1 + v 1 = v 1 + v 12 = v 12
2
x = - 2 m to x = 0, displacement of In Case b Force of elongation is same. m2 2 2
particle is along positive x-direction F F
\ X1 = , X2 = 2gh
while force acting on the particle is kP kQ Þ v1 =
along negative x-direction. Hence, work 3
1 1 F2
done is negative and is given by the area WP = k1 X 12 = 25 Let x be the extension in the spring.
under (F-x) graph 2 2 kP
Applying conservation of energy,
1 1 1 F2 2 Mg
W = - ´ 2 ´ 10 = - 10 J k2 X 22 = 1
2
WQ = Mgx - kx2 = 0 - 0 Þ x =
2 2 kQ 2 k
14 Given force is a constant force and work [Q k1 = k P , k2 = kQ ] 26 According to conservation of energy,
done by a constant force is always path \ W P < WQ 1 2 1 (Mv ) 2
independent. kL = Mv 2 Þ kL2 =
20 W = DK 2 2 M
15 As m is the mass per unit length, then 1 1 MkL2 = p2 [ p = Mv ]
rate of mass per second Þ - Fs = mv 2 - mu2
mx 2 2 \ p = L Mk
= = mv 1
t = 0 - ´ 10 ´ 10-3 ´ 10000 So, just after collision momentum of
1 1 2 block (maximum) would be L MK .
\ Rate of KE = (mv)v2 = mv3 1
2 2 Þ - F ´ 10-2 = - ´ 10 ´ 10-3 ´ 104
2 27 Initially body posseses only kinetic
16 Net work done in sliding a body up to a \ F = 5000 N energy and after attaining a height, the
height h on inclined plane kinetic energy is zero.
= Work done against gravitational force 21 Work done by spring + work done by Therefore, loss of energy = KE – PE
+ Work done against gravity = DKE 1
= mv2 - mgh
frictional force Ws + W g = 0 - 0 2
Þ W = Wg + W f …(i) Ws = -W g Þ Ws = - mgh 1
= ´ 1 ´ 400 - 1 ´ 18 ´ 10
but W = 300 J 2
22 Work done by gravitational force,
W g = mgh = 2 ´ 10 ´ 10 = 200 J = 200 - 180 = 20 J
On, putting the value of W g in Eq. (i), W g = mgh = 10-3 ´ 10 ´ 1 ´ 103 = 10 J
we get Now, from work-KE theorem, we have 28 When string is released, then according
300 = 200 + W f DK = Wgravity + W air resistance to conservation of energy
1
Þ W f = 300 - 200 = 100 J

1
´ mv 2 = mgh + W air resistance mv 2 = mgh Þ v 2 = 2gh = 2gR
2 2
DAY SIX WORK, ENERGY AND POWER 63

Now, when it is at the bottom 32 Power = Fv 37 We have, p1 + p2 + p3 = 0 [Q p = mv ]


mv 2 P \ 1 ´ 12 $i + 2 ´ 8 $j + p3 = 0
T - mg cos 0° = Þ F =
R v
m Þ 12 $i + 16 $j + p = 0 3
T - mg = (2gR )
R Þ p3 = - (12 i$ + 16 $j)
T = mg + 2mg = 3mg θ \ p3 = (12)2 + (16)2
s in
29 When stone is at its lowest position, a)
+ φ = 144 + 256
(g
stone has only kinetic energy given by, =
1 a net = 20kg m/s
K = mu2 Now, p3 = m3 v 3
2 Given, P = 8 ´ 105 W, v = 20 m/s p 20
At the horizontal position, it has energy Þ m3 = 3 = = 5 kg
8 ´ 105 v3 4
1 Þ F = = 4 ´ 104 N
E = U + K ¢ = mu ¢2 + mg l 20
2 38 u1=9 m/s u2=0 v1 v2
At constant speed, the forces acting on m 2m m 2m
the train are in equilibrium. The force A B A B
O parallel to the hill is Before elastic After elastic
1 collision collision
F = R + (2 ´ 105 )g ´
l 50 2m1 u1 2´ m´ 9
v2 = = = 6 m/s
u 1
5 m1 + m2 m + 2m
F = R + (2 ´ 10 ) ´ 9.8 ´
According to conservation of energy 50
i.e. After elastic collision B strikes
K =E
Þ R = 4 ´ 104 - 39200 = 800 N to C with velocity of 6 m/s.
1 1
mu2 = mu ¢2 + mgl Now, collision between B and C is
2 2
1 1
33 Fv = constant = k Þ m dv v = k perfectly inelastic.
mu ¢ 2 = mu2 - mgl dt
2 2 6 m/s v
k v2 k 2m m 3m
2 2 2 ò v dv = ò m dt, 2 = m t
u ¢ = u - 2gl Þ u ¢ = u - 2gl Rest
So, the magnitude of change in velocity 2k dv
⇒ v = t, F = m By the law of conservation of
|Du|= |u|= u ¢2 + u2 + 2uu ¢ cos 90° m dt
momentum,
2k 1 -1 /2 2m ´ 6 + 0 = 3 m ´ v C
|D u|= u ¢2 + u2 - 2 = 2(u2 - gl ) = m t
m 2 ⇒ v C = 4 m/s
2 mk -1 /2
30 Equation of force, mg - mg cos θ = mv = t
39 If A suffers on oblique elastic collision
l 2
with a particle B, i.e. is at rest initially.
v2
= g (1 - cos θ) 34 Given, Velocity of water v = 2 m /s If their massess are the same then after
l collision B remains at rest while A
Mass per unit length of water in the pipe
Þ v = gl (1 - cos θ)
2
…(i) = 100 kg / m continuous to move in the original
Applying law of conservation of energy So, power = (mass per unit length of direction or they will move in mutually
1 1 water in pipe) ´ v3 perpendicular directions.
mgl = mv 2 + mgl(1 - cos θ)
2 2 m 40 Initial momentum = p i = 0
= ´ v3 = 100 ´ 2 ´ 2 ´ 2 = 800 W
v 2 = gl - 2gl (1 - cos θ) …(ii) l Final momentum,
On equating Eqs. (i) and (ii) and then
solving, we get 35 Total initial energy = 1 m1 u12 + 1 m2 u22 p f = 0 = mvi + mvj + p3
2 2 ⇒ p3 = mv 2
2
θ = cos -1 æç ö÷ Total final energy p32 1 1
è3 ø 1 1 Total KE = + mv 2 + mv 2
= m2 v 22 + m1 v 12 + E 2 ´ 2m 2 2
From Eq. (ii), we get 2 2
2m2 v 2
æ gl ö 9.8 ´ 15 From conservation of energy, = + mv 2
v = ç ÷= = 7 m/s 1 1 4m
è3ø 3 m1 u12 + m2 u22
2 2 3 mv 2
mv 2 =
1 1 2
31 T top = - mg = m1 v 12 + m2 v 22 + E
l 2 2
41 For inelastic collision between two
mv 2 ⇒
1 1
m1 u12 + m2 u22 - E
T bottom = + mg spherical rigid bodies, total linear
l 2 2 momentum is conserved.
æv 2 ö 1 1
- g÷ æ 40 ´ 40 - 10ö = m1 v 12 + m2 v 22
ç ç ÷ 2 2 42 Fraction of lost KE
T top è l ø è 4 ø 4x 4´ 4
= = = =
T bottom æ v2 ö æ 40 ´ 40 + 10ö 36 It is given that mass of balls are same (1 + x )2 (1 + 4)2
ç + g ÷ çè ÷
ø and collision is perfectly elastic (e = 1),
è l ø 4 16
so their velocities will be interchanged. =
400 - 10 390 39 Thus, v ¢A = v B = - 0.3 m/s, 25
= = =
400 + 10 410 41 v B¢ = v A = 0.5 m/s
64 40 DAYS ~ NEET PHYSICS DAY SIX

43 From conservation of linear momentum, 2 Here, m = 0.5 kg, v = ax3 /2 , where 1


F × dx = K f - ´ 10 ´ (10) 2
we can see that velocity of 2 m will -2 2
a = 5ms , W = ?
v
become after collision (as mass is dv d Þ F × dx = K f - 500
2 Acceleration, A = = (ax3 /2 )
dt dt x = 30
doubled)
3
= a ´ x1 /2
dx 3 a 1 /2
= x (ax3 /2 )
Þ òx = 20 (- 0.1) x dx = K f - 500
m 2m
2 dt 2 x = 30
v
3 a2 2 é x2 ù
A= x or - 0.1 ê ú = K f - 500
m 2m 2 ë 2 ûx = 20
v/ 2 3 a2 2 3 a2 2
⇒ Force, F = mA = 0.5 ´ x = x é (30)2 (20) 2 ù
Rest 2 4 - 0.1 ê - ú = K f - 500
Work done, ë 2 2 û
Relative velocity of separation 2 2
Now, e = 3 a2 2 Þ K f - 500 = - 25
Relative velocity of approach W = ò F × dx = ò x dx
4
v /2 1 0 0 Þ K f = 500 - 25 = 475 J
= = 2
v 2 3 a2 é x3 ù 3 6 According to the conservation of
= ê ú = (5)2 [23 - 0]
44 Suppose a ball rebounds with speed v, 4 ë 3 û0 4 ´ 3 mechanical energy,
v = 2gh = 2 ´ 10 ´ 20 W = 50 J (TE )initial = (TE )final
Þ (KE )i + (PE )i = (KE ) f + (PE ) f
= 20 m /s 3 The graph of U ( x ) with x is as shown,
1
Energy of a ball just after rebound, potential energy is zero at x = 0 and 0 + mgh = mv 2A + 0
1 maximum at x = ± a. 2
E = mv 2 = 200 m
2 v 2A v2
U(x) Þ gh = or h = A …(i)
As, 50% of energy loses in collision means 2 2g
just before collision energy is 400 m. In order to complete the vertical circle,
According to law of conservation of the velocity of the body at point A
energy, we have should be v A = v min = 5gR
1
mv 20 + mgh = 400 m x where, R is the radius of the body.
2 –a O +a AB D
1 Here, R= =
Þ mv 20 + m ´ 10 ´ 20 = 400 m k
Mechanical energy has fixed value ,
2 2
2 2 5
Þ v 0 = 20 m/s Þ v min = v A = gD
kinetic energy has to be maximum at 2
45 By conservation of momentum, x = 0 and minimum at x = ± a. Substituting the value of v A in Eq. (i),
mv + M ´ 0 = (m + M ) v ¢ 4 Given, mass of particle m = 0.01 kg. we get
2
Velocity of composite block Radius of circle along which æ æ5 öö
æ m ö ç ç gD ÷ ÷
v¢ = ç ÷v particle is moving, r = 6.4 cm. è è2 ø ø 5gD 5
èm + M ø Q Kinetic energy of particle, h= = = D
1 2g 2 ´ 2g 4
KE of composite block KE = 8 ´ 10-4 J Þ mv 2 = 8 ´ 10-4 J
1 2 7 According to question, we have
= (M + m ) v 2 16 ´ 10-4
2
2 Þ v = = 16 ´ 10-2 …(i) Let the tension at point A be T A . So,
2 0.01 force equation,
1 æ m ö 2
= (M + m ) ç ÷ v As it is given that KE of particle is equal
2 èM + mø to 8 ´ 10-4 J by the end of second mv c2
T A - mg =
1 æ m ö R
mv 2 ç revolution after the beginning of motion
= ÷ 1
2 èm + M ø of particle. It means, its initial velocity Energy at point A = mv 20 …(i)
(u ) is 0 m/s at this moment. 2
QBy Newton’s 3rd equation of motion, Energy at point C is
SESSION 2 v 2 = u2 + 2a t s
1
mv c2 + mg ´ 2R …(ii)
1 We have, s = 1 at 2 = 1 ´ 2 ´ 4 = 4 m Þ v 2 = 2a t s or v 2 = 2a t (4 πr )
2
2 2 C
(Q particle covers 2 revolutions)
N v2 16 ´ 10-2
Þ at = = R TC
8πr 8 ´ 314 . ´ 6.4 ´ 10-2 B
3 0° m (Qfrom Eq. (i), v 2 = 16 ´ 10-2 )
TB
sin
mg 60°
\ . m/s2
a t = 01
s 30° mg mg cos 30° A
v0
5 Work done
\ Work done by force of gravity 1 1 mg
= mv 2 - mu2 = K f - K i
W = mg × s = mgscos 60° 2 2 At point C,
1 1 mv c2
= 10 ´ 9.8 ´ 4 ´ = 196 J \ F × dx = K f - mv 2i Tc + mg =
2 2 R
DAY SIX WORK, ENERGY AND POWER 65

To complete the loop Tc ³ 0 10 Since, the collision mentioned is an From the conservation of vertical linear
mv c2 elastic head-on collision. Thus, momentum
So, mg = mv v
R according to the law of conservation of - mv ¢ sin θ = 0 ⇒ = v ¢ sin θ
Þ v c = gR …(iii) linear momentum, we get 3 3
...(iv)
From Eqs. (i) and (ii) by conservation of m1u1 + m2u2 = m1 v 1 + m2 v 2
On squaring and adding Eqs. (iii) and
energy where, m1 and m2 are the masses of the
(iv), we get
1 1 two blocks, respectively and u1 and u2
mv 20 = mv c2 + 2mgR v2
2 2 are their initial velocities and v 1 and v 2 v2 + = v ¢2 (sin2 θ + cos 2 θ)
are their final velocities, respectively. 3
Þ mv 20 = mgR + 2mgR ´ 2 Here, m1 = m, m2 = 4m 4v 2 2
u1 = v ,u2 = 0 ⇒ = v ¢2 ⇒ v ¢ = v
[Q v c = gR ] 3 3
and v1 = 0
Þ v 20 = gR + 4gR 12 Pgenerated = Pinput ´ 90
mv + 4m ´ 0 = 0 + 4mv 2
Þ v0 = 5gR 100
Þ mv = 4mv 2
mgh 90 15 ´ 10 ´ 60 90
v = ´ = ´
8 According to question, a body of mass 1 or v2 = …(i) t 100 1 100
4
kg begins to move under the action of = 8.1 kW
As, the coefficient of restitution is given
time dependent force,
as,
F = (2ti$ + 3t 2 $j ) N relative velocity of separation 13 Linear momentum of water striking per
where i$ and $j are unit vectors along X after collision
e = second to the wall
relative velocity of approach
and Y-axes. p i = mv = Avρv = Av 2ρ,
F v
Q F = ma Þ a = -0 similarly linear momentum of reflected
v2 - v1
m = = 4 [from Eq. (i)] water per second
2$ u2 - u1 0- v
(2t$i + 3t j ) p r = Av 2ρ.
Þ a= (Q m = 1 kg ) 1
1 = Y
4
Þ a = (2ti$ + 3t $j ) m / s 2
2
\ e = 0.25
dv
Q acceleration, a =
dt 11 Let mass A moves with velocity v and pi
Þ dv = a dt …(i) collides inelastically with mass B, which
is at rest. pr cosθ θ pi cosθ
Integrating both sides, we get X
θ θ
2 pr
ò dv = ò a dt = ò (2ti$ + 3t $j ) dt —
v
v = t 2 $i + t 3 $j A m √3 pr sinθ pi
pi sinθ
Q Power developed by the force at the
A B
time t will be given as v
m m Now, making components of
P = F × v = (2t$i + 3t 2 $j ) × (t 2 $i + t 3 $j ) θ
momentum along X and Y-axes. Change
At rest
= (2t .t 2 + 3t 2 × t 3 ) in momentum of water per second
P = (2t 3 + 3t 5 ) W m
B v′ = p i cos θ + p r cos θ = 2 Av 2ρ cos θ
9 By definition of force, force exerted on
v/ 3 According to problem mass A moves in a
M the wall = 2 Av 2ρ cos θ
perpendicular direction and let the mass
B moves at angle θ with the horizontal 14 Force = Rate of change of momentum
M v M u2=0 º θ with velocity v.
Initial momentum
Initial horizontal momentum of system
p1 = mv sin θ $i + mv cos θ $j
M (before collision) = mv ...(i)
Final horizontal momentum of system Final momentum
v2 p2 = - mv sin θ $i + mv cos θ $j
(after collision) = m v ¢ cos θ ...(ii)
According to law of conservation of From the conservation of horizontal Dp -2mv sin θ
kinetic energy, we have linear momentum \ F= =
1 1 v
2
1
Dt 2 ´ 10-3
Mv 2 + 0 = M æç ö÷ + Mv 22 mv = mv ¢cos θ ⇒ v = v ¢ cos θ ...(iii)
2 2 è3ø 2 On substituting, m = 0.1 kg ,
Initial vertical momentum of system
v = 5m/s, θ = 60°
2 v2 v2 8v 2 (before collision) is zero.
Þ v = + v 22 Þ v 2 - = v 22 Þ
9 9 9 Final vertical momentum of system Force on the ball, F = - 250 3 N to left
Velocity of second block after collision, mv
- mv ¢ sin θ. Negative sign indicates direction of the
2 2 3 force.
v2 = v
3
DAY SEVEN

System of Particles
and Rigid Body
Learning & Revision for the Day
u Centre of Mass u Radius of Gyration u Moment of Inertia for Simple
u Momentum Conservation u Theorems on Moment of Geometrical Objects
u Rigid Bodies Inertia
u Moment of Inertia u Radius of Gyration

Centre of Mass
Centre of mass of a system (body) is a point that moves when external forces are applied
on the body as though all the mass were concentrated at that point and all external forces
were applied there.

Centre of Mass of Two Particle System


Centre of mass of a two particles system consisting of two particles of masses m1, m2 and
respective position vectors r1 , r2 is given by
m r + m2 r2
rCM = 1 1
m1 + m2
r1 + r2
If m1 = m2 = m (say), then rCM =
2

Centre of Mass of n-Particle System


Centre of mass of n-particles system which consists n-particles of masses m1, m2 ,..., mn
with r1, r2 ,..., rn as their position vectors at a given instant of time. PREP
The rCM of the system at that instant is given by
n
MIRROR
Σ mi ri Your Personal Preparation Indicator
m r + m2 r2 + K + mn rn i= 1
rCM = 1 1 = No. of Questions in Exercises (x)—
m1 + m2 + K + mn
u
M
u No. of Questions Attempted (y)—
No. of Correct Questions (z)—
Cartesian Components of the Centre of Mass
u

(Without referring Explanations)


The position vectors rCM and ri are related to their cartesian components by
u Accuracy Level (z / y × 100)—
rCM = xCM i$ + yCM $j + zCM k$ and ri = x i i$ + yi $j + zi k$
u Prep Level (z / x × 100)—
The cartesian components of rCM are given by
n n n In order to expect good rank in NEET, your
Σ mi x i Σ mi yi Σ mi zi Accuracy Level should be above 85 & Prep
i= 1 i= 1 i= 1
xCM = , yCM = and zCM = Level should be above 75.
M M M
DAY SEVEN SYSTEM OF PARTICLE AND RIGID BODY 67

Motion of Centre of Mass Rigid Bodies


The position vector rCM of the centre of mass of n particle A rigid body is defined as that body which does not undergo
system is defined by any change in shape (or) volume when external forces are
m r + m2 r2 + m3 r3 + ... + mn rn applied on it. When a force is applied on a rigid body, the
rCM = 1 1 distance between any two particles of the body will remain
m1 + m2 + m3 ... mn
unchanged, however, larger the forces may be.
1
= (m1r1 + m2 r2 + m3 r3 + ... + mn rn ) Coordinates of centre of mass of a rigid body are
M 1
drCM 1  dr dr dr  X CM = x dm,
=  m1 1 + m2 2 + ... + mn n  M
dt M  dt dt dt  1
YCM = y dm
1 M
Velocity of centre of mass vCM = (m1v 1 + m2 v2 + ... mn v n )
M 1
n and ZCM = zdm
Σ mi v i M
i= 1
vCM =
M Centre of Mass of Some Rigid Bodies
l
Similarly, acceleration of centre of mass is given by l
The centre of mass of a uniform rod is located at its
n
Σ mi a i mid-point.
i= 1
a CM =
M
CM
l
From Newton’s second law of motion,
l
Centre of mass of a uniform rectangular, square or
M a CM = F1 + F2 + ... Fn ⇒ M a CM = FExt circular plate lies at its centre.
For an isolated system, if external force on the body is zero.
d
F = Ma CM = M (vCM ) = 0 ⇒ vCM = constant
dt
i.e. Centre of mass of an isolated system moves with uniform CM
CM CM
velocity along a straight line path and momentum remain
conserved.
l
Centre of mass of a uniform semi-circular ring lies at a
NOTE • If some mass or area is removed from a rigid body, then the 2R
distance of h = from its centre, on the axis of
position of centre of mass of the remaining portion is π
obtained from the following formula symmetry, where R is the radius of the ring.
m r − m2r2 A r − A2r2
rCM = 1 1 or rCM = 1 1
m1 − m2 A1 − A2
R CM 2R
Momentum Conservation π
Let us consider a system of particles of masses m1, m2 ... mn are O
having respective velocities v 1, v2 ... v n . The total linear
momentum of the system would be the vector sum of the l
Centre of mass of a uniform
momentum of the individual particles. semi-circular disc of radius R
4R CM 4R
i.e. p = p 1 + p2 + p3 ... p n = m1v 1 + m2 v2 + .....+ mn v n lies at a distance of h = R
3π 3π
Velocity of centre of mass of a system
from the centre on the axis of O
1
vCM = (m1v 1 + m2 v2 + ...... + mn v n ) symmetry as shown in figure.
M
l
Centre of mass of a
where, M is the total mass of the system, therefore
hemispherical shell of radius
p = MvCM R
Thus, total linear momentum of a system of particles is equal to R lies at a distance of h = R CM R
2 2
the product of the total mass of the system and the velocity of
from its centre on the axis of
its centre of mass. O
symmetry as shown in figure.
dp dv
Again =M = Ma = Fext l
Centre of mass of a solid
dt dt
dp hemisphere of radius R lies
If Fext = 0, then = 0, i.e. p = constant 3R R CM 3R
dt at a distance of h = from
8 8
If external force of a system is zero, then momentum of system
its centre on the axis of O
of particle remain constant.
symmetry.
68 40 DAYS ~ NEET PHYSICS DAY SEVEN

Moment of Inertia 2. Theorem of Perpendicular Axes


Moment of inertia of a rotating body is its property to oppose The sum of moment of inertia of a plane laminar body about
any change in its state of uniform rotation. two mutually perpendicular axes lying in its plane is equal to
If in a given rotational system particles of masses m1, m2 , m3 ,K its moment of inertia about an axis passing through the point
be situated at normal distances r1, r2 , r3 ,... from the axis of of intersection of these two axes and perpendicular to the
rotation, then moment of inertia of the system about the axis plane of laminar body.
of rotation is given by Z
I = m1r12 + m2 r22 + m3 r32 + ... = Σ mr 2 Planar Body
For a rigid body having continuous mass distribution
I = ∫ dm r 2
O N
Y
2
SI unit of moment of inertia is kg m . It is neither a scalar nor M
P(x, y)
a vector i.e. it is a tensor.
X

Radius of Gyration If I x and I y be moment of inertia of the body about two


Radius of gyration of a given body about a given axis of perpendicular axes in its own plane and I z be the moment of
rotation is the normal distance of a point from the axis, where inertia about an axis passing through point O and
if whole mass of the body is placed, then its moment of inertia perpendicular to the plane of lamina, then
will be exactly same as it has with its actual distribution of Iz = Ix + I y
mass. Thus, radius of gyration In theorem of perpendicular axes, the point of intersection of
1/ 2
I  r 2 + r22 + r32 + K + rn2  the three axes ( x, y z) may be any point on the plane.
K = or K =  1 
M  n 
Moment of Inertia for Simple
m3
Geometrical Objects
m1 r3 Uniform Ring of Mass M and Radius R About an axis
r1 l

m4 M passing through the centre and perpendicular to plane of


r4
m2 1
r2 P ring, I = MR2 . About a diameter, I = MR2
K
2
Axis of rotation
l
Uniform Circular Disc of Mass M and Radius R About an
axis passing through the centre and perpendicular to plane
SI unit of radius of gyration is metre. 1 1
of disc, I = MR2 . About a diameter, I = MR2
Radius of gyration depends upon shape and size of the body, 2 4
position and configuration of the axis of rotation and also on l
Thin Uniform Rod of Mass M and Length l About an axis
distribution of mass of body w.r.t. axis of rotation.
passing through its centre and perpendicular to the rod,
1
Theorems on Moment of Inertia I = Ml 2
12
There are two theorems based on moment of inertia are given l
Uniform Solid Cylinder of Mass M, Length l and Radius R
below: 1
About its own axis, I = MR2 . About an axis passing
2
1. Theorem of Parallel Axes through its centre and perpendicular to its length,
I ICM
Moment of inertia of a body about a  l 2 R2 
given axis I is equal to the sum of I =M +
moment of inertia of the body about a 12 4 
parallel axis passing through its centre of A 2
mass ICM and the product of mass of
CM l
Uniform Solid Sphere About its diameter, I = MR2 .
5
body (M ) and square of normal distance 7
between the two axes. Mathematically, About its tangent, I = MR2
5
I = ICM + Md2 d
DAY SEVEN SYSTEM OF PARTICLE AND RIGID BODY 69

DAY PRACTICE SESSION 1

FOUNDATION QUESTIONS EXERCISE


1 In the HCl molecule, the separation between the nuclei of their speeds are v and 2 v at any instant, then the speed
the two atoms is about 1.27 Å (1 Å =10−10 m). Given that, of centre of mass of the system will be j CBSE AIPMT 2010
chlorine atom is about 35.5 times as massive as a (a) 2v (b) 0 (c) 1.5 v (d) v
hydrogen atom and all the mass of the atom is 9 Two persons of masses 55 kg and 65 kg respectively,
concentrated in its nucleus. The approximate location of are at the opposite ends of a boat. The length of the boat
centre of mass w.r.t. H atom is
is 3 m and weighs 100 kg. The 55 kg man walks upto the
(a) 1 Å (b) 1.24 Å 65 kg man and sits with him. If the boat is in still water
(c) 10−10 Å (d) 10−5 Å the centre of mass of the system shifts by
l
2 A T shaped object with dimensions A B j CBSE AIPMT 2012
shown in the figure, is lying on a (a) 3 m (b) 2.3 m
smooth floor. A force F is applied at P (c) zero (d) 0.75 m
F
the point P parallel to AB, such that 2l
10 Two spherical bodies of mass M and 5 M and radii R and
the object has only the translational 2 R respectively are released in free space with initial
motion without rotation. Find the C separation between their centre equal to 12 R. If they
location of P with respect to C. attract each other due to gravitational force only, then the
4
(a) l (b) l distance covered by the smaller body just before
3
2 3 collision is
(c) l (d) l
3 2 (a) 1.5 R (b) 2.5 R
(c) 4.5 R (d) 7.5 R
3 If linear density of a rod of length 3m varies as λ = 2 + x,
11 Point masses m1 and m2
then the position of the centre of gravity of the rod is 0 ω
are placed at the m1 m2
7 12
(a) m (b) m opposite ends of a rigid
3 7 x (L-x)
10 9 rod of length L and
(c) m (d) m
7 7 negligible mass. The rod
is to be set rotating about an axis perpendicular to it. The
4 Two bodies of masres 1 kg and 3 kg have position position of point P on this rod through which the axis
vectors $i + 2 $j + k$ and − 3 $i − 2 $j + k,
$ respectively. The
should pass, so that the work required to set the rod
centre of mass of this system has a position vector rotating with angular velocity ω 0 is minimum is given by
j
CBSE AIPMT 2009 m1L m1
(a) x = (b) x = L
(a) − 2 $i + 2 k$ (b) − 2 i − j + k$
$ $ m1 + m2 m2
(c) 2 $i − $j − 2 k$ (d) − $i + $j + k$ m
(c) x = 2 L (d) x =
m2L
m1 m1 + m2
5 Three masses are placed on the X -axis : 300 g at origin,
500 g at x = 40 cm and 400 g at x = 70 cm. The distance 12 From a disc of radius R and mass M, a circular hole of
of the centre of mass from the origin is diameter R, whose rim pases through the centre is cut.
(a) 40 cm (b) 45 cm (c) 50 cm (d) 30 cm What is the moment of inertia of the remaining part of the
6 Three masses of 2 kg, 4 kg and 4 kg are placed at three disc about a perpendicular axis passing through the
points (1, 0, 0), (1, 1, 0) and (0, 1, 0) respectively. The centre? j
NEET 2016
position vector of its centre of mass is 13MR 2 11MR 2 9MR 2 15MR 2
(a) (b) (c) (d)
3$ 4$ 2$ 4$ 1$ 4 $ 32 32 32 32
(a) i+ j (b) 3 $i + $j (c) i+ j (d) i+ j
5 5 5 5 5 5 13 Point masses 1 kg, 2 kg, 3 kg and 4 kg are lying at the
7 Three bodies having masses 5 kg, 4 kg and 2 kg is point (0, 0, 0), (2, 0, 0), (0, 3, 0) and ( −2, − 2, 0) respectively.
moving at the speed of 5 m/s, 4 m/s and 2 m/s The moment of inertia of this system about X axis will be
respectively along X -axis. The magnitude of velocity of (a) 43 kg-m 2 (b) 34 kg-m 2
CM is (c) 27 kg-m 2 (d) 72 kg-m 2
(a) 1.0 m/s (b) 4 m/s (c) 0.9 m/s (d) 1.3 m/s
8. Two particles which are initially at rest, move towards
each other under the action of their internal attraction. If
70 40 DAYS ~ NEET PHYSICS DAY SEVEN

14 Two masses are joined with a light rod and the system is 21 Three particles each of mass m gram situated at the
rotating about the fixed axis as shown in the figure. The vertices of an equilateral triangle ABC of side l cm
MI of the system about the axis is (as shown in the figure). The moment of inertia of the
system about a line AX perpendicular to AB and in the
plane of ABC in gcm 2 units will be
M/2 M/2 X
m C
l/2 l/2

l l

(a) Ml2 / 2 (b) Ml2 / 4 A


B
Y
(c) Ml2 (d) Ml2 / 6 m l m
(a)   ml 2 (c)   ml 2 (d)   ml 2
3 5 3
15 Four identical thin rods each of mass M and length l, (b) 2ml 2
 4  4  2
form a square frame. Moment of inertia of this frame
about an axis through the centre of the square and 22 Radius of gyration of uniform thin rod of length L about
perpendicular to its plane is j CBSE AIPMT 2009
an axis passing normally through its centre of mass is
4 2 2 2 13 2 1 2 L L
(a) Ml (b) Ml (c) Ml (d) Ml (a) (b) (c) 12 L (d) 12L
3 3 3 3 12 12
16 The moment of inertia of a thin uniform circular disc
23 The radius of gyration of a body about an axis at a
about one of its diameter is l. The moment of inertia about
an axis perpendicular to the circular surface and passing distance 6 cm from its centre of mass is 10 cm. Then, its
through its centre is radius of gyration about a parallel axis through its centre
(a) 2I (b) I / 2 of mass will be
(c) 2 I (d) I/2 (a) 800 cm (b) 8 cm (c) 0.8 cm (d) 80 cm
17 The moment of inertia of a uniform circular disc of radius 24 The moment of inertia of a thin uniform rod of mass M
R and mass M about an axis passing from the edge of and length L about an axis passing through its mid-point
the disc and normal to the disc is and perpendicular to its length is l0. Its moment of inertia
1 about an axis passing through one of its ends and
(a) MR 2 (b) MR 2
2 perpendicular to its length is j
CBSE AIPMT 2011
ML2 ML2
7
(c) MR 2 (d)
3
MR 2 (a) l 0 + (b) l 0 + 2ML2 (c) l 0 + ML2 (d) l 0 +
2 2 4 2
X
18 A thin wire to length l and mass m is bent in the form of a 25 Three identical spherical shells, each of
semi-circle. Its moment of inertia about an axis joining its mass m and radius r are placed as
free ends will be shown in figure. Consider an axis XX ′
which is touching to two shells and
(a) zero (b) ml 2
passing through diameter of third shell.
ml 2 ml 2
(c) 2 (d) Moment of inertia of the system
π 2 π2 X′
consisting of these three spherical shells
19 Two disc of the same material and thickness have radii about XX ′ axis is j
CBSE AIPMT 2015
0.2 m and 0.6 m. Their moment of inertia about their axes 11 2 16 2
(a) mr (b) 3 mr 2 (c) mr (d) 4 mr 2
will be in the ratio 5 5
(a) 1 : 81 (b) 1 : 27 26 The moment of inertia of a thin rod of mass M and length
(c) 1 : 91 (d) 1 : 3 L
L, about an axis perpendicular to the rod and distance
20 A solid cylinder has mass M, length L and radius R. The 4
momentum of inertia of this cylinder about a generator is from one end is
 L2 R 2  ML2 1 3 ML2 ML2 7ML2 7ML2
(a) M  +  (b) (c) MR 2 (d) MR 2 (a) (b) (c) (d)
 12 4  4 2 2 6 12 24 48
DAY SEVEN SYSTEM OF PARTICLE AND RIGID BODY 71

DAY PRACTICE SESSION 2

PROGRESSIVE QUESTIONS EXERCISE


1 Four particle of masses, m, 2 m, 3 m and 4 m are 6 One quarter sector is cut from a uniform
arranged at the corners of a parallelogram with each circular disc of radius R. This sector has
side equal to a and one of the angle between two mass M. It is made to rotate about a line
adjacent sides is 60°. The parallelogram lies in the xy perpendicular to its plane and passing 90°
plane with mass m at the origin and 4 m on the X -axis. through the centre of the original disc. Its
The centre of mass of the arrangement will be located at moment of inertia about the axis of
 3   3  rotation is
(a)  a, 0.95 a  (b)  0.95a, a
 2   4  1 1 1
(a) mR 2 (b) mR 2 (c) mR 2 (d) 2MR 2
 3a a  2 4 8
(d)  ,
a 3a 
(c)  ,  
 4 2 2 4  7 Four thin rods of same mass M and same length l, form a
2 A point P on the rim of wheel is initially at rest and in square as shown in figure. Moment of inertia of this
contact with the ground. Find the displacement of the system about an axis through centre a and
point P, if the radius of the wheel is 5 m and the wheel perpendicular to its plane is
rolls forward through half a revolution. l
(a) 5 m (b) 10 m (c) 2.5 m (d) 5 (π 2 + 4) m

3 A uniform square plate has a small piece Q of an


irregular shape removed and glued to the centre of the l O l
plate leaving a hole behind. The moment of inertia about
the Z-axis is the
Y Y l
2
4 2 Ml Ml2 2 2
Hole (a) Ml (b) (c) (d) Ml
3 3 6 3
Q X X
Q 8 Moment of inertia of circular loop of radius R about the
axis of rotation parallel to horizontal diameter at a
(a) increased (b) decreased distance R /2 from it is
(c) the same 1
(d) changed in unpredicated manner (a) MR 2 (b) MR 2
2
3
4 For spheres each of mass M and radius R are placed (c) 2MR 2 (d) MR 2
with their centres on the four corners A , B, C and D of a 4
square of side b. The spheres A and B are hollow and C 9 What is the moment of inertia of a ring about a tangent to
and D are solids. The moment of inertia of the system the periphery of the ring?
about side AD of square is 1 3
(a) MR 2 (b) MR 2
8 8 2 2
(a) MR 2 + 2Mb 2 (b) MR 2 + 2Mb 2
3 5 (c) MR 2 (d) MR 2 / 9
32
(c) MR 2 + 2Mb 2 (d) 32MR 2 + 4Mb 2 10 A uniform cylinder has a radius R and length L. If the
15
moment of inertia of this cylinder about an axis passing
5 The surface density of a circular disc of radius a through its centre and normal to its circular face is equal
depends on the distance as ρ (r ) = A + Br . The moment of to the moment of inertia of the same cylinder about an
inertia about the line perpendicular to the plane of the axis passing through its centre and perpendicular to its
disc is length, then
(a) πa 4  +
A 2a 
(b) πa 4  +
A 2B  (a) L = R (b) L = 3R
B 
2 5  2 5  R 3
(c) L = (d) L = R
(c) 2 πa 3  +
A Ba  3 2
 (d) None of these
2 5 
72 40 DAYS ~ NEET PHYSICS DAY SEVEN

11 From a circular disc of radius R and mass 9M, a small 12 A uniform circular disc of radius a is taken. A circular
R portion of radius b has been removed from it as shown in
disc of radius is removed from the disc, the moment of
3 the figure. If the centre of holes is at a distance c from
inertia of the remaining disc about an axis perpendicular the centre of the disc, the distance x 2 of the centre of
to the plane of the disc and passing through O is mass of the remaining part from the initial centre of mass
O is given by
R/3

a
b
R O O O1
O2 X-axis
x2 c

(a) 4MR 2 (b)


40
MR 2 (c) 10MR 2 (d)
37
MR 2 πb 2 cb 2 πc 2 ca 2
(a) (b) (c) (d)
9 9 (a − c )
2 2
(a − b )
2 2
(a − b )
2 2
(c − b 2 )
2

ANSWERS
SESSION 1 1 (b) 2 (a) 3 (b) 4 (b) 5 (a) 6 (a) 7 (b) 8 (b) 9 (c) 10 (d)
11 (d) 12 (a) 13 (a) 14 (b) 15 (a) 16 (a) 17 (d) 18 (d) 19 (a) 20 (d)
21 (c) 22 (a) 23 (b) 24 (a) 25 (d) 26 (d)
SESSION 2 1 (b) 2 (d) 3 (b) 4 (c) 5 (a) 6 (a) 7 (a) 8 (d) 9 (b) 10 (b)
11 (a) 12 (b)

Hints and Explanations


1 Let centre of mass be at a distance of x m1 r1 + m2 r2 m2l + 2ml 4 The position vector of centre of mass
rcm = =
cm from H atom. m1 + m2 m + 2m m1 r1 + m2 r2
r=
4ml 4 m1 + m2
= = l
H CI 3m 3 1(i$ + 2 $j + k$ ) + 3(− 3 $i − 2 $j + k$ )
=
3 Linear density of 1+ 3
x CM
dx 1
the rod varies with = (− 8 i − 4 j + 4 k$ )
$ $
1.27A° distance 4
dm = − 2 $i − $j + k$
If x1 and x2 be the position vectors of H = λ [given]
dx x m1 x1 + m2 x2 + m3 x3
and Cl atoms w.r.t. the centre of mass as 5 xCM =
origin, then ∴ dm = λ dx m1 + m2 + m3
m1 x1 + m2 x2 300 × (0) + 500(40) + 400(70)
xCM = 0 = Position of centre of mass =
m1 + m2 3 300 + 500 + 400
xCM =
∫ dm × x = ∫0 (λ dx ) × x =
500 × 40 + 400 × 70
⇒ m1 x1 + m2 x2 = 0
⇒ m (− x ) + 35.5m (1.27 − x ) = 0 ∫ dm 3
∫0 λ dx
1200
xCM = 40 cm
⇒ mx = 35.5m (1.27 − x ) 3
 2 x3  6 For centre of mass
⇒ x + 35.5x = 35.5 × 1.27 3
x + 3 
⇒ x(1 + 35.5) = 45.08 ∫
= 0
(2 + x ) × xdx
=  0 xCM =
2×1+ 4×1+ 4× 0
=
6
=
3
36.5x = 45.08
3 3 2+ 4+ 4 10 5
∫0 (2 + x ) dx 2 x + x 
2
2× 0+ 4×1+ 4×1 8 4
⇒ x=
45.08
= 124
. Å 2 0 y CM = = =
36.5  2+ 4+ 4 10 5
9 + 9 36 12 Position vector of its centre of mass
= = = m
2 Position vector of centre of mass of the 9 21 7 3 4
6+ = $i + $j.
system 2 5 5
DAY SEVEN SYSTEM OF PARTICLE AND RIGID BODY 73

m1 v 1 + m2 v 2 + m3 v 3
7 VCM = I remain =
13MR2 19 I = 1 MR2 = 1 ( πR2t × ρ) × R2 ⇒ I = R2
m1 + m2 + m3 32 2 2
4
5 × 5 + 4 × 4 + 2 × 2 45 R 
4
= 
= = 13 I . 
02 1
Y ∴ 1 =  1  =
5+ 4 + 2 11 I2  R2   0.6  81
= 4. 09 = 4 m/s
1 kg (0, 0, 0) (2, 0, 0) 20 Generator axis of a cylinder in a line
8 As initially both the particles were at rest B 2 kg
X
lying on it’s surface and parallel to axis
therefore velocity of centre of mass was A
of cylinder. By parallel axis theorem,
zero and there is no external force on the
MR2 3
system so speed of centre of mass remains I = + MR2 = MR2

0)
constant i.e it should be equal to zero. 2 2

2,
,–
21 Moment of inertia of the system about

(–2
9 Here, on the entire system net external D (0
, 3,
4 kg 0) AX is given by
force on the system is zero hence centre
of mass remains unchanged. C 3 kg X
10 As the spherical bodies have their own I AB = I X = Σm i r i2 C
size, so the distance covered by both the D
body 12R − 3 R = 9 R, but individual = m1 r12 + m2 r22 + m3 r32 + m 4 r 42
l l
distance covered by each body depends = (1)(0)2 + 2(0)2 + 3(3)2 + 4(−2)2
upon their masses. 30°
= 27 + 16 = 43 kg-m2
5M
M 14 Total moment of inertia is A B
2 2 l
R 9R 2R I =
M  l + M  l
    = m A r 2A + m B r B2 + mC rC2
12 R 2  2 2  2
9R–x Ml 2 Ml 2 Ml 2 = m (0)2 + m (l )2 + m (l sin 30° )2
x = + =
8 8 4 m l2 5
= m l2 + = m l2
We know that bodies are moving under 4 4
15 Moment of inertia of rod about an axis
the effect of mutual attraction only, so 2
through its centre of mass and
22 ML = Mk 2 ⇒ k = L
their position of centre of mass remains perpendicular to rod = (mass of rod) × 12 12
unaffected. (perpendicular distance between two
Let smaller body cover distance x just axes) 23 From theorem of parallel axis,
before collision from m1 r1 = m2 r2 , we 2 A
Ml 2 Ml 2
+ M   =
l
get, Mx = 5M (9 R − x ) ⇒ x = 7.5R =
12  2 3
11 The position of point P on rod through Moment of inertia of the system
which the axis should pass, so that the
Ml 2 4 h
work required to set the rod rotating = × 4 = Ml 2
with minimum angular velocity ω 0 is 3 3
their centre of mass, we have 2 2

m2 L 16 I = Id = MR and I p = MR CM B
m1 x = m2 (L − x ) ⇒ x = 4 2
I = IG + Mh2 ⇒ I AB = ICM + Mh2
m1 + m2
Hence, I p = 2I
MK 2AB = MK CM
2
+ Mh2
12 Considering the I 17 Moment of inertia of disc passing ⇒ K 2AB = K CM
2
+ h2
information given through its centre of gravity and
in the question, let perpendicular to its plane is Given, K AB = 10 cm, h = 6 cm
us draw the figure R R/2 ∴ K CM = 8 cm
1
If the above figure I AB = MR2
2 24 The theorem of parallel axis for moment
is considered, then
Using theorem of parallel axes, we have of inertia.
moment of inertia of disc will be given as
ICD = I AB + MR2 I = ICM + Mh2 ,
I = I remain + I( R /2 ) ⇒ I remain = I − I( R /2 )
2
ML2
I = I 0 + M   = I 0 +
Putting the values, we get 1 3 L
= MR2 + MR2 = MR2
 M  R 2  2 2  2 4
   2
MR 2
4  2 M  R  18 For semi-circle,
= − +    25 X
2  2 4  2  πr = l
  l r dr
  ∴ r = 1
π
MR2  MR2 MR2 
= − + Moment of inertia
2  32 16  mr 2
I = a 2 3
r r
MR2  MR2 + 2MR2  2
= −  2
2  32  m l  m l2 X′
=   =
2
16MR − 3MR
2 2 2 2  π 2 π2
=
MR

3MR
= I xx ′ = I1 + I2 + I3
2 32 32
74 40 DAYS ~ NEET PHYSICS DAY SEVEN

mr 2 +  mr 2 + mr 2 
2 2 3 According to the theorem of The distance between these two parallel
=
3 3  perpendicular axes, I Z = I X + I Y with axes (in figure) is R, the radius of the
ring.
+  mr 2 + mr 2 
 2 the hole, I x and I y both decrease and
3  gluing the removed piece at the centre Using the parallel axes theorem,
I xx ′ = 2mr 2 + 2mr 2 = 4 mr 2 of square plate does not affect I z . Hence, Tangent
I z decreases, overall. (axis-1)
26 I ICM
A b D
4 Moment of inertia of a R
L hollow sphere of radius Diameter (axis-2)
4 R about the diameter b C
passing through D is
According to theorem of parallel axes, 2
2 l A = MR2 C
ML2
+ M  
L 3 B
I = ICM + Mx2 = MR2
12  4 Moment of inertia of solid sphere about I tangent = Idia + MR2 = + MR2
2
ML2 ML2 7ML2 diameter,
= + = 2 3
=MR2
12 16 48 l D = MR2 2
5
Moment of inertia of whole system 10 Moment of inertia of a cylinder about its
SESSION 2 about side AD is = l A + l D + l B + l C MR2
1 Let m1 = m, m2 = 2m, m3 = 3 m, centre and parallel to its length =
= MR2 + MR2 +  Mb 2 + MR2 
2 2 2 2
m4 = 4 m 3 5  3  R

+  Mb + MR2 

Y 2 2
L
2m 3m  5 
32
= MR + 2Mb
2 2
15 Moment of inertia about its centre and
a sin 60°

a 5 dM = (2πr )dr (ρ) = ( A + Br )(2πrdr ) perpendicular to its length


a πAa4 2 πBa5  L2 R2 
l = ∫0 dMr =
+
2
=M + 
2 5  12 4 
= πa4  +
60° X A 2Ba 
m 4m 
2 5  ML2 MR2 MR2
a cos 60° ∴ + = ⇒L = 3 R
12 4 2
r1 = 0i + 0$j
$ 6 Mass of the entire disc would be 4 M
a$ a 3 $ and its moment of inertia about the 11 As the mass is uniformly distributed on
r2 = acos 60i$ + asin 60$j = i+ j 1 the disc, so mass density (per unit area)
2 2 given axis would be (4M )R2 . 9M
r3 = (a + acos 60) $i + asin 60$j 2 =
3 a 3$ For the given section the moment of πR 2
= a$i + j
2 2 inertia about the same axis would be Mass of removed portion
1 2
r4 = ai + 0 j × π   = M
$ $ one quarter of this, i.e. MR2 . 9M R
=
On substituting above value in the
2 πR 2  3
following formula 7 Moment of inerita of A l B So, moment of inertia of the removed
m r + m2 r2 + m3 r3 + m 4 r4 rod AB about point P portion about the stated axis by theorem
r = 1 1 of parallel axis.
m1 + m2 + m3 + m 4 and perpendicular to
O 2 2
Ml 2 1  R
M   + M 
2R 
= 0. 95a i$ +
3 $
aj the plane = I1 = 
4 12 2  3  3 
So, the location of centre of mass M.I. of rod AB about If the disc would not have been
2
Ml2 Ml2
+ M   =
 removed, then the moment of inertia of
3 
l
point O ′ =
 0.95a, 4 a. 12  2  3 complete disc about the stated axis.
  1
But the system consists of four rods of I2 = 9 M (R )2
2 P′ similar type so by the symmetry 2
 Ml 2  4 2 So, the moment of inertia of the disc
Isystem = 4   = Ml about required axis. I = I2 − I1
 3  3 1 2R  
2 2
= 9M (R )2 −  M   + M 
1 R
2R  
8 Applying theorem of parallel axis 2  2  3   3  

2
I = ICM + M  
R I = 4 MR2
 2
P πR 12 Centre of mass of whole system was at
1 MR2 3
= MR2 + = MR2 point O.
Displacement of point P after half 2 4 4
revolution, Hence, x2 (area of remaining portion)
9 The tangent to the ring in the plane of = c (area of removed disc)
PP ′ = ( πR )2 + (2R )2 the ring is parallel to one of the
cb 2
= R π2 + 4 = 5 π2 + 4 m diameters of the ring. x2 ( πa2 − πb 2 ) = c ( πb 2 ) ⇒ x2 =
a2 − b 2
DAY EIGHT

Rotational
Motion
Learning & Revision for the Day
u Concept of Rotational u Moment of Force or Torque u Equilibrium of a Rigid Bodies
Motion u Angular Momentum u Rigid Body Rotation
u Equation of Rotational u Law of Conservation
Motion of Angular Momentum

Concept of Rotational Motion


In rotation of a rigid body about a fixed axis, every particle of the body moves in a circle,
which lies in a plane perpendicular to the axis and has its centre on the axis.
l
Rotational motion is characterised by angular displacement dθ and angular velocity

ω= .
dt
l
If angular velocity is not uniform, then rate of change of angular velocity is called the
angular acceleration.

Angular acceleration, α = .
dt
SI unit of angular acceleration is rad/s2. PREP
l
Angular acceleration α and linear tangential acceleration a t are correlated MIRROR
as a t = α × r. Your Personal Preparation Indicator

u No. of Questions in Exercises (x)—

Equation of Rotational Motion u

u
No. of Questions Attempted (y)—
No. of Correct Questions (z)—
If angular acceleration α is uniform, then equations of rotational motion may be written (Without referring Explanations)
as
1 Accuracy Level (z / y × 100)—
(i) ω = ω 0 + α t (ii) θ = ω 0 t + α t 2
u

2 u Prep Level (z / x × 100)—


α
(iii) ω − ω 0 = 2 αθ
2 2
(iv) θ nth = ω 0 + (2 n − 1) In order to expect good rank in NEET, your
2 Accuracy Level should be above 85 & Prep
Level should be above 75.
76 40 DAYS ~ NEET PHYSICS DAY EIGHT

Moment of Force or Torque Law of Conservation


Torque (or moment of a force) is the turning effect of a force of Angular Momentum
applied at a point on a rigid body about the axis of rotation.
According to the law of conservation of angular momentum, if
n = r F sin θ ^
Mathematically, torque, τ = r × F = | r × F|^ n no external torque is acting on a system, then total vector sum
where, ^n is a unit vector along the axis of F of angular momentum of different particles of the system
rotation. Torque is an axial vector and its remains constant.
dL
SI unit is newton-metre (N-m). We know that, = τext
r dt

O
l
The torque about axis of rotation is θ A dL
independent of choice of origin O, so B Hence, if τext = 0, then = 0 ⇒ L = constant
dt
long as it is chosen on the axis of
rotation AB. Therefore, in the absence of any external torque, total angular
momentum of a system must remain conserved.
l
Only normal component of force
contributes towards the torque. Radial component of force Comparison of Linear and Rotational Motion
does not contribute towards the torque. Linear Motion Rotational Motion
l
A torque produces angular acceleration in a rotating body. 1. Linear momentum, p = mv Angular momentum, L = Iω, L = 2 IE
Thus, torque, τ = Iα
2. Force, F = ma Torque, T = Iα
l
Moment of a couple (or torque) is given by product of
3. Kinetic energy, E = 1 mv 2 Rotational energy, E =
1 2
position vector r between the two forces and either force F. Iω
2 2
Thus, τ = r × F
l
If under the influence of an external torque, τ the given
body rotates by dθ, then work done, dW = τ ⋅ dθ. Equilibrium of a Rigid Bodies
l
In rotational motion, power may be defined as the scalar
product of torque and angular velocity, i.e. Power P = τ ⋅ ω. For mechanical equilibrium of a rigid body, two condition
need to be satisfied.

Angular Momentum 1. Translational Equilibrium


The moment of linear momentum of a given body about an axis A rigid body is said to be in translational equilibrium, if it
of rotation is called as its angular momentum. If p be the linear remains at rest or moving with a constant velocity in a
momentum of a particle and r is its position vector from the particular direction. For this, the net external force or the
point of rotation, then vector sum of all the external form acting on the body must be
zero,
Angular momentum, L = r × p = r psin θ ^
n
i.e. F = 0 or F = ΣFi = 0
where, ^
n is a unit vector in the direction of rotation. Angular
momentum is an axial vector and its SI unit is kg-m2s −1 or J-s.
2. Rotational Equilibrium
l
For rotational motion of a rigid body, angular P
A rigid body is said to be in rotational equilibrium, if the body
momentum is equal to the product of angular does not rotate or rotates with constant angular velocity. For
velocity ω and moment of inertia of the body I this, the net external torque or the vector sum of all the
about the axis of rotation. r torques acting on the body is zero.
Mathematically, L = Iω.
For the body to be in rotational equilibrium,
l
According to the second law of rotational dL
motion, the rate of change of angular momentum of a body τext = 0, = 0 or L = constant
dt
is equal to the external torque τ applied on it and takes
place in the direction of torque. Thus,
dL d dω  dω  Rigid Body Rotation
τ= = (Iω) = I = Iα Qα =
 dt 
dt dt dt
Spinning
l
Total effect of a torque applied on a rotating body in a When the body rotates in such a manner that its axis of rotation
given time is called angular impulse. Angular impulse is does not move, then its motion is called spinning motion.
equal to total change in angular momentum of the system 1
in given time. Thus, angular impulse, In spinning rotational kinetic energy is given by, K R = Iω2 .
2
∆t
J =∫ τ dt = ∆L = L f − Li Rotational kinetic energy is a scalar having SI unit joule (J).
0 Rotational kinetic energy is related to angular momentum as
l
The angular momentum of a system of particles about the per relation,
n L2
origin is L = ∑ ri × pi KR = or L = 2 IK R
2I
DAY EIGHT ROTATIONAL MOTION 77

Pure Rolling Motion Rolling on an Inclined Plane


Let a rigid body, having symmetric surface about its centre of When a body of mass m and radius R
R
mass, is being spined at a certain angular speed and placed rolls down on inclined plane of
on a surface, so that plane of rotation is perpendicular to the height h and angle of inclination θ, it S h
surface. If the body is simultaneously given a translational loses potential energy. However, it
motion too, then the net motion is called rolling motion. acquires both linear and angular θ
speeds and hence gain kinetic energy
ω
of translation and that of rotation.
CM v 1  K2 
By conservation of mechanical energy, mgh = mv2 1 + 2 
2  R 
2 gh
The total kinetic energy in rolling motion,
l
Velocity at the lowest point v =
K2
K = KT + K R 1+ 2
R
1 1 1 1  v2 
= mv2 + Iω2 = mv2 + mv2  2  [Q v = rω] l
Acceleration in motion From second equation of motion,
2 2 2 2 R  v2 = u2 + 2 as
1  v2  h 2 gh
K = mv2 1 + 2  By substituting u = 0, s = and v = , we get
2  R  sin θ K2
1+ 2
R
g sin θ
Rolling Without Slipping a=
K2
If the given body rolls over a surface such that there is no 1+ 2
relative motion between the body and the surface at the point R
of contact, then the motion is called rolling without slipping. l
Time of descent From first equation of motion, v = u + at
By substituting u = 0 and value of v and a from above
1 2h  K2 
expressions t = 1+ 2 
sin θ g  R 
P
Rω v
From the above expressions, it is clear that,
1 1 K2
v∝ ;a ∝ ; t ∝ 1 +
K2 K2 R2
Impure Rolling Motion 1+ 1+ 2
In impure rolling motion, the point of contact of the body with R2 R
the platform is not relatively at rest w.r.t. platform on which,
it is performing rolling motion, as a result sliding occurs at Important Terms Related to Inclined Plane
point of contact.  K2 
l
Here, factor  2  is a measure of moment of inertia of a
For impure rolling motion, v AB ≠ 0 i.e. v − Rω ≠ v0 R 
If platform is stationary, i.e. v0 = 0, then v ≠ Rω body and its value is constant for given shape of the body
and it does not depend on the mass and radius of a body.
l
Velocity, acceleration and time of descent (for a given
K2
ω v ⇒ inclined plane) all depends on 2 . Lesser the moment of
R
A K2
v = Rω inertia of the rolling body lesser will be the value of 2 . So,
R
v0 v0
B
greater will be its velocity and acceleration and lesser will
be the time of descent.
78 40 DAYS ~ NEET PHYSICS DAY EIGHT

DAY PRACTICE SESSION 1

FOUNDATION QUESTIONS EXERCISE


1 When a ceiling fan is switched on, it makes 10 revolutions 9 A constant torque of 31.4 N-m is exerted on a pivoted
in the first 3s. Assuming a uniform angular acceleration, wheel. If angular acceleration of wheel is
how many rotation it will make in the next 3s? 4π rad/sec 2 , then the moment of inertia of the wheel is
(a) 10 (b) 20 (c) 30 (d) 40 (a) 2.5 kg-m 2 (b) 3.5 kg-m 2
(c) 4.5 kg-m 2 (d) 5.5 kg-m 2
2 The drive shaft of an automobile rotates at 3600 rpm and
transmits 80 HP up from the engine to the rear wheels. 10 A force of −F k$ acts on Oi the origin of the coordinate
The torque developed by the engine is system. The torque about the point (1, − I ) is
(a) 16.58 Nm (b) 0.022 Nm (a) −F ($i + $j) (b) F ($i + $j)
(c) 158.31 Nm (d) 141.6 Nm (c) −F ($i − $j) (d) F ($i − $j)
3 The moment of the force, F = 4$i + 5$j − 6k$ at ( 2, 0, − 3), 11 A solid sphere is rotating in free space. If the radius of
about the point ( 2, − 2, − 2), is given by j NEET 2018 the sphere is increased keeping mass same which one
(a) −7 i$ − 8 $j − 4 k$ (b) −4 i$ − $j − 8 k$ of the following will not be affected?
(c) −8 $i − 4 $j − 7 k$ (d) −7 $i − 4 $j − 8 k$ (a) Moment of inertia (b) Angular momentum
(c) Angular velocity (d) Rotational kinetic energy
4 An automobile moves on a road with a speed of
Y
54 kmh − 1. The radius of its wheels is 0.45 m and the 12 A particle of mass m in the
moment of inertia of the wheel about its axis of rotation is XY -plane with a velocity v along B
A
3 kg m2 . If the vehicle is brought to rest in 15 s, the the straight line AB. If the angular
magnitude of average torque transmitted by its brakes to momentum of the particle with
the wheel is j
CBSE AIPMT 2015 respect to origin O is LA when it is
O X
(a) 6.66 kg m2 s − 2 (b) 8.58 kg m2 s − 2 at A and LB when it is at B, then
(c) 10.86 kg m2 s − 2 (d) 2.86 kg m2 s − 2 (a) LA > LB
(b) LA = LB
5 ABC is an equilateral triangle with A
(c) the relationship between LA and LB depends upon the
O as its centre. F1, F2 and F3 F3 slope of the line AB
represent three forces acting (d) LA < LB
along the sides AB, BC and AC, O
B F2 13 In an orbital motion, the angular momentum vector is
respectively. If the total torque C
about O is zero, then the magnitude F (a) along the radius vector
1 (b) parallel to the linear momentum
of F3 is j
CBSE AIPMT 2012
(c) in the orbital plane
F1 + F2
(a) F1 + F2 (b) F1 − F2 (c) (d) 2 (F1 + F2 ) (d) perpendicular to the orbital plane
2
14 When a mass is rotating in a plane about a fixed point, its
6 The instantaneous angular position of a point on a rotating
angular momentum is directed along j
CBSE AIPMT 2012
wheel is given by the equation Q (t ) = 2 t − 6 t . The 3 2
(a) a line perpendicular to the plane of rotation
torque on the wheel becomes zero at j CBSE AIPMT 2011
(b) the line making an angle of 45° to the plane of rotation
(a) t = 0.5 s (b) t =0.25 s (c) t = 2 s (d) t = 1s (c) the radius
7 If F is the force acting on a particle having position (d) the tangent to the orbit
vector r and τ be the torque of this force about the origin, 15 A round disc of moment of inertia I 2 about its axis
then j
CBSE AIPMT 2009 perpendicular to its plane and passing through its centre
(a) r ⋅ τ ≠ 0 and F ⋅ τ = 0 (b) r ⋅ τ > 0 and F ⋅ τ < 0 is placed over another disc of moment of inertia I1
(c) r ⋅ τ = 0 and F ⋅ τ = 0 (d) r ⋅ τ = 0 and F ⋅ τ ≠ 0 rotating with an angular velocity ω about the same axis.
8 A string is wound round the rim of a mounded flywheel of The final angular velocity of the combination of discs is
I 2ω
mass 20 kg and radius 20 cm. A steady pull of 25 N is (a) (b) ω
applied on the cord. Negleating friction and mass of the I1 + I 2
string. The angular acceleration of the wheel is Iω (I1 + I 2 )ω
(c) 1 (d)
2 2 2 2 I1 + I 2 I1
(a) 50 rad/s (b) 25 rad/s (c) 12.5 rad/s (d) 6.25 rad/s
DAY EIGHT ROTATIONAL MOTION 79

16 A mass m moves in a circle on a v0 22 Two rotating bodies A and B of masses m and 2m with
smooth horizontal plane with velocity v 0 m moments of inertia I A and IB (IB > I A ) have equal kinetic
at a radius R 0. The mass is attached to energy of rotation. If LA and LB be their angular momenta
a string which passes through a respectively, then j NEET 2016

smooth hold in the plane as shown in LB


(a) L:A = (b) LA = 2LB (c) LB > LA (d) LA > LB
figure. 2
The tension in the string is increased gradually and 23 A solid sphere of mass m and radius R is rotating about
R its diameter. A solid cylinder of the same mass and same
finally m moves in a circle of radius 0 . The final value of
2 radius is also rotating about its geometrical axis with
the kinetic energy is j CBSE AIPMT 2015
angular speed twice that of the sphere. The ratio of their
(a)
1
mv 02 (b) 2mv 02 (c)
1
mv 02 (d) mv 02  E Sphere 
4 2 kinetic energies of rotation   will be
 E Cylinder  j NEET 2016
17 A solid cylinder of mass 50 kg and radius 0.5 m is, free
to rotate about the horizontal axis. A massless string is (a) 2 : 3 (b) 1 : 5 (c) 1 : 4 (d) 3 : 1
wound round the cylinder with one end attached to it and 24 If the angular momentum of any rotating body increasing
other hanging freely. Tension in the string required to by 200%, then the increase in its kinetic energy
produce an angular acceleration of 2 revolution s −2 is (a) 400% (b) 800% (c) 200% (d) 100%
j CBSE AIPMT 2014
25 A small object of uniform density rolls up a curved
(a) 25 N (b) 50 N (c) 78.5 N (d) 157 N
surface with an initial velocity v′. It reaches upto a
18 A thin circular ring of mass M and radius R is rotating in a 3v 2
horizontal plane about an axis vertical to its plane with a maximum height of with respect to the initial
4g
constant angular velocity ω. If two objects each of mass
position. The object is j
NEET 2013
m be attached gently to the opposite ends of a diameter
of the ring, the ring will then rotate with an angular (a) ring (b) solid sphere
(c) hollow sphere (d) disc
velocity j
CBSE AIPMT 2009, 1998
ω(M − 2 m) ωM ω(M + 2 m) ωM 26 An inclined plane makes an angle of 30° with the
(a) (b) (c) (d)
M + 2m M + 2m M M +m horizontal. A solid sphere rolling down this inclined plane
from rest without slipping has a linear acceleration equal
19 Three objects, A : (a solid sphere), B : (a thin circular disk)
to
and C : (a circular ring), each have the same mass M and
g 2g 5g 5g
radius R. They all spin with the same angular speed ω (a) (b) (c) (d)
3 3 7 14
about their own symmetry axes. The amounts of work (W )
required to bring them to rest, would satisfy the relation 27 A circular disc rolls down an inclined plane. The ratio of
j
NEET 2018 rotational kinetic energy to total kinetic energy is
(a) WB > WA > WC (b) WA > WB > WC 1 1 2 3
(a) (b) (c) (d)
(c) WC > WB > WA (d) WA > WC > WB 2 3 3 4
20 Initial angular velocity of a circular disc of mass M is ω1. 28 The ratio of the acceleration for a solid sphere (mass m
Then, two small spheres of m are attached gently to two and radius R) rolling down an incline of angle θ without
diametrically opposite points on the edge of the disc. slipping and slipping down the incline without rolling is
What is the final angular velocity of the disc? j
CBSE-AIPMT 2014
M + m M + m
(a)  ω (b)  ω
(a) 5 : 7 (b) 2 : 3 (c) 2 : 5 (d) 7 : 5
 M  1  m  1
29 A rod PQ of mass M and
 M   M 
(c)   ω1 (d)   ω1 length L is hinged at end P.
 M + 4m   M + 2m 
The rod is kepts horizontal
21 Two discuss of same moment of inertia rotating about by a massless string tied to P Q
point Q as shown in figure. L
their regular axis passing through centre and
perpendicular to the plane of disc with angular velocities When string is cut, the initial angular acceleration of the
ω1 and ω 2 . They are brought into contact face to face rod is j
NEET 2013
coinciding the axis of rotation. The expression for loss of
3g g
energy during this process is j
NEET 2017 (a) (b)
1 1 2L L
(a) I (ω1 + ω2 )2 (b) I (ω1 − ω2 )2 2g 2g
2 4 (c) (d)
I L 3L
(c) I (ω1 − ω2 )2 (d) (ω1 − ω2 )2
8
80 40 DAYS ~ NEET PHYSICS DAY EIGHT

DAY PRACTICE SESSION 2

PROGRESSIVE QUESTIONS EXERCISE


1 A uniform rod of length l and mass m is free to rotate in a 7 A thin circular ring of mass M and radius r is rotating
vertical plane about A. The rod initially in horizontal about its axis with a constant angular velocity ω. Four
position is released. The initial angular acceleration of objects each of mass m, are kept gently to the opposite
 ml 2  ends of two perpendicular diameters of the ring. The
the rod isMoment of inertia of rod about A is 
 3  angular velocity of the ring will be
Mω  (M + 4m)ω (M − 4m)ω Mω 
l (a)   (b) (c) (d)  
A B  M + 4m  M M + 4m  4m 
3g 2l 8 A thin and circular disc of mass M and radius R is
(a) (b)
2l 3g rotating in a horizontal plane about an axis passing
3g l through its centre and perpendicular to its plane with an
(c) (d) mg
2 l2 2 angular velocity ω. If another disc of same dimensions
Y but of mass M/4 is placed gently on the first disc
2 A uniform rod of mass M and
length L is free to rotate in XY coaxially, then the new angular velocity of the system is
5 2 4 3
plane i.e. about Y -axis. If a (a) ω (b) ω (c) ω (d) ω
4 3 5 2
force of F = ( 3$i + 2$j + 6 k$ ) N X
is acting on rod at (L/2, 0, 0) in 9 A sphere of mass m and radius r falls on a horizontal
the situation as shown in plane without slipping with the speed u. Now if it falls up
Z
figure. The angular vertically, the maximum height it would attain will be
acceleration of rod is (take, M = 6 kg, L = 4 m) 3u 2 5u 2 7u 2 u2
(a) (b) (c) (d)
4g 2g 10g 2g
3 $ 1 $ 3 $ 1$
(a) − i + k (b) − j (c) k (d) 4 $j
2 2 2 2 10 A solid sphere is rotating freely about its symmetry axis
in free space. The radius of the sphere is increased
3 A rope is wound around a hollow cylinder of mass 3 kg
keeping its mass same. Which of the following physical
and radius 40 cm. What is the angular acceleration of the
quantities would remain constant for the sphere?
cylinder, if the rope is pulled with a force of 30 N? j
NEET 2018
(a) 25 m/s2 (b) 0.25 rad/s2 (c) 25 rad/s2 (d) 5 m/s2
(a) Rotational kinetic energy (b) Moment of inertia
4 A tube of length L is filled completely with an (c) Angular velocity (d) Angular momentum
incompressible liquid of mass M and closed at both the
11 A tangential force F acts at the top of a thin spherical
ends. The tube is then rotated in a horizontal plane about
shell of mass m and radius R. The acceleration of the
one of its ends with a uniform angular velocity ω. The
shell, if it rolls without slipping is
force exerted by the liquid at the other end is
F
(a) Mω2L / 2 (b) M ω2L
(c) Mω2 L / 4 (d) Mω2L2 / 2
R
5 A particle performing uniform circular motion has angular
momentum L. If its angular frequency is doubled and its
kinetic energy halved, then the new angular 5F 6F 7m 2m
(a) (b) (c) (d)
momentum is 6m 5m 2F 7F
L L
(a) (b) 2L (c) 4L (d) 12 A disc and a sphere of same radius but different masses
4 2
role off an two inclined planes of the same altitude and
6 A couple consisting of two forces F1 and F2 each equal to length. Which ones of the objects gets to the bottom of
5 N is acting at the rim of a disc of mass 2 kg and radius the plane first? j
NEET 2016
1
m for 5 s. Initially, the disc is at rest, the final angular (a) Sphere
2
(b) Both reach all the same time
momentum (in Nm-s) of the disc is
(c) Depends on their masses
(a) 15 (b) 20
(d) Disc
(c) 25 (d) 30
DAY EIGHT ROTATIONAL MOTION 81

ANSWERS
SESSION 1 1 (c) 2 (c) 3 (d) 4 (a) 5 (a) 6 (d) 7 (c) 8 (c) 9 (a) 10 (b)
11 (b) 12 (b) 13 (d) 14 (a) 15 (c) 16 (b) 17 (d) 18 (b) 19 (c) 20 (c)
21 (b) 22 (c) 23 (b) 24 (b) 25 (d) 26 (d) 27 (b) 28 (a) 29 (a)

SESSION 2 1 (a) 2 (b) 3 (a) 4 (a) 5 (a) 6 (c) 7 (a) 8 (c) 9 (c) 10 (d)
11 (b) 12 (a)

Hints and Explanations


SESSION 1 v = 54 km/h = 54 ×
5
= 15m/s where, α is the angular acceleration
18 1
1 Angle turned in 3s, θ3s = 2 π × 10 = 20 π 25 ×
Angular velocity of a vehicle, v = ω 0 r τ FR 5 = 5 = 12.5 rad/s2
rad α = = =
v 15 100 I I 0.4 0.4
1 2 ⇒ ω0 = = = rad/s
From θ = ω 0t + αt R 0.45 3 τ 31.4
2 So, angular acceleration of an 9 I = = = 2. 5 kg-m2
1 40 π α 4π
⇒ 20 π = 0 + α × (3)2 ⇒α = rad/s2 automobile,
2 9 100 10 F = − F k$ or r = ($i − $j )
0−
Now angle turned in 6 s from the starting ∆ω ω f − ω 0 3 τ = r × F = (i$ − $j ) × (− F k$ )
α = = =
40 π 
θ 6s = ω 0 t + αt 2 = 0 + × 
1 1 t t 15
 × (6)
2
= − F($i × k$ ) + F($j × k$ ) = − F(− $j ) + F ($i )
2 2  9  − 100
= rad/s 2 = F($i + $j )
= 80πrad 45
∴ angle turned between t = 3 s to t = 6 s Thus, average torque transmitted by its 11 Taking the same mass of sphere, if
θlast 3 s = θ 6s − θ3 s = 80 π − 20 π = 60 π brakes to wheel radius is increased, then moment of
100
Number of revolution τ = Iα ⇒ 3 × = 6.66 kg m2 s −2 inertia, rotational kinetic energy and
60 π 45 angular velocity will change but
= = 30 revolution
2π 5 If we take clockwise torque according to law of conservation of
momentum, angular momentum will
2 From P = τω and τ = P τ net = τ F1 + τ F2 − τ F3
not change. So, the reaction at the other
ω 0 = F1 r + F2 r − F3 r
It is given, end will be equal to FC .
⇒ F3 = F1 + F2
P = 80 HP = 80 × 746 W = 59680 N 12 From the definition of angular
ms −1 6 According to question, torque τ = 0 momentum,
3600 It means that, α = 0
ω = 3600 rpm = × 2 π rad s −1 y
60 d 2θ
α =
= 120 π rad s −1 dt 2 φ B
A
So, τ = 158.31 Nm Given, θ(t ) = 2t 3 − 6t 2
P r
3 Given, F = 4i$ + 5$j − 6k$ dθ d 2θ
So, = 6t 2 − 12t ⇒ α = = 12t − 12 d
r1 = 2 $i + 0$j − 3k$ dt dt 2 x
O
⇒ r = 2 $i − 2$j − 2k$
2 12t − 12 = 0 ⇒ t = 1 s
Moment of force = r × F L = r × p = rmv sin φ (− k )
7 τ = r × F, where r = position vector
= (r1 − r2 ) × F Therefore, the magnitude of L is
= [(2 $i + 0$j − 3k$ ) − (2 $i − 2$j − 2k$ )] F = force ⇒ τ = |r ||F |sin θ n$ L = mvr sin φ = mvd
× [4i$ + 5$j − 6k$ ] Torque is perpendicular to both r and F. where, d = r sin φ is the distance of
= [0 i + 2 j − 1k ] × [4i + 5$j − 6k$ ]
$ $ $ $ So, dot product of two vectors will be closest approach of the particle to the
$i $j k$ zero. origin. As d is same for both the
∴ τ ⋅ r = 0 ⇒ F⋅ τ = 0 particles, hence L A = L B .
= 0 2 −1
4 5 −6 8 Here, M = 20 kg, R = 20 cm = 1 m 13 If p be the linear momentum of a
5 particle and r is its position vector from
= i$[(−6 × 2 ) − (−1 × 5)] Moment of inertia of flywheel about its the point of rotation, then angular
− $j [(−6 × 0) − (−1 × 4)] axis is momentum L = r × p = rp sinθ n, $
+ k$ [(0 × 5) − 2 × 4] 2 where n$ is unit vector in the direction
MR2 = × 20 ×   = 0.4 kg-m2
1 1 1
I = of rotation. Hence, angular momentum
= − 7$i − 4$j − 8k$ 2 2  5
vector is perpendicular to orbital plane.
4 As velocity of an automobile vehicle, τ = Iα 14 As we know that
82 40 DAYS ~ NEET PHYSICS DAY EIGHT

Angular momentum, L = m(r × v ) 20 Conservation of angular momentum 2gh


25 As, v =
So, here angular momentum is directed gives K2
1+
MR2ω1 =  MR2 + 2mR2  ω2
along a line perpendicular to the plane 1 1 r2
of rotation. 2 2 
3v 2
⇒ MR2ω1 = R2 (M + 4 m )ω2 Given, h=
15 The angular momentum of a disc of 4g
moment of inertia I1 and rotating about its  M 
axis with angular velocity ω is ∴ ω2 =   ω1 2gh 2g (3v 2 ) 6gv 2
v2 = = =
 M + 4m  K   K2 
2 2
L 1 = I1ω 1+
K
4g  1 +  4g  1 + 2 
When a round disc of moment of inertia 21 Angular momentum before contact r2 
2
r   r 
I2 is placed on first disc, then angular = I1ω1 + I2ω2 3 K2 1 1
momentum of the combination is
Angular momentum after the discs 1= ⇒ = ⇒ K 2 = r2
L2 = (I1 + I2 )ω ′  K 2
r 2 2 2
brought into contact 2 1 + 
2
In the absence of any external torque,
= I net ω = (I1 + I2 ) ω  r 
angular momentum remains conserved [equation of disc]
i.e. L1 = L2 So, final angular speed of system = ω Hence, object is disc.
I1 ω I ω + I2ω2 g sin θ g sin θ g / 2 5g
I1ω = (I1 + I2 )ω ′ ⇒ ω ′ = = 1 1 26 a = = = =
I1 + I2 I1 + I2 k2 2 7 / 5 14
1+ 1 +
Now, to calculate loss of energy, we R2 5
16 Angular momentum remains constant subtract initial and final energies of  K 2 2
because of the torque of tension is zero. system. as,θ = 30° and 2 = 5
R
⇒ L i = L f ⇒ mv 0 R = mv ⇒ v = 2v 0 ⇒ Loss of energy  R 

1
2
2
1 1 1
= Iω21 + Iω22 − (2I ) ω1ω2 27 Rotational kinetic energy, K R = 1 Iω2
⇒ KE f = m (2v 0 ) = 2 mv 0
2
2 2 2 2
2 1 1 MR2 2 1
= I(ω1 − ω2 ) 2 KR = × × ω = mv 2
17 Tr = Iα 2 2 2 4
T Translational kinetic energy,
r
22 As we know that, the kinetic energy of a 1
rotating body, K T = mv 2
2
O 1 1 I 2ω2 L2 Total kinetic energy, = K T + K R
KE = Iω2 = =
r 2 2 I 2I 1 1 3
= mv 2 + mv 2 = mv 2
Also, angular momentum, L = I ω 2 4 4
Iα mr 2 α mrα 1
T = = × = Thus, K A = KB mv 2
Rotational Kinetic energy 1
r 2 r 2 2 = 4 =
1 L2A 1 L2B L  I Total Kinetic energy 3 3
50 × 0.5 × 2 × 2 π ⇒ = ⇒ A = A mv 2
= N = 157 N 2 IA 2 IB  LB  IB 4
2 28 a slipping = g sinθ
LA IA g sin θ 5 a rolling
18 According to question by applying ⇒ = ⇒L ∝ I a rolling = = g sin θ ⇒ =
5
LB IB 2 7 a slipping 7
conservation of angular momentum K
1+
I1ω1 = I2ω2 ∴ L A < LB [Q I B > I A ] r2
In the given case 23 KE of sphere, 29 Torque on the rod = Moment of weight
I1 = MR2 or I2 = MR2 + 2 mR2 1 2 12 1
KS = Iω1 = mR2ω21 = mR2ω21 of the rod about P
ω1 = ω 2 2 5 5 L/ 2
I M KE of cylinder, P
Then, ω2 = 1 ω = ω 11 1
I2 M + 2m KC = mR2ω22 = mR2ω22
22 4 mg
19 Work done required to bring an object mR2ω21 L
1 τ = mg …(i)
to rest is given as W = Iω2 K 5 4 ω21 4 ω21 1 2
2 ∴ S = = = =
KC mR2ω22 5 ω22 5 (2ω1 ) 2 5 ∴ Moment of Inertia of rod about
where, I is the moment of inertia and ω
4 (given, ω2 = 2ω1 ) ML2
is the angular velocity. P = …(ii)
Since, here all the objects spin with the 2 3
L2 E L  As τ = Iα
same ω, this means, W ∝ I 24 E = or E ∝ L2 or 2 =  2 
2 2I E1  L1  From Eqs. (i) and (ii), we get
As, I A (for a solid sphere) = MR2
5 L2 = 200% of (L1 ) + L1 L ML2
Q Mg = α
1 = 2L1 + L1 = 3L1 2 3
I B (for a thin circular disk) = MR2
2 2 3g
E2  3L1  ∴ α =
IC (for a circular ring) = MR2 ∴ =  or E2 = 9E1 2L
E1  L1 
∴ W A : W B : WC = I A : I B : IC
2 1 2 1 Increment in kinetic energy SESSION 2
= MR2 : MR2 : MR2 = : : 1
5 2 5 2 ∆E = E2 − E1 = 9E1 − E1
= 4 : 5 : 10 ∆E = 8E1 1 The moment of inertia of the uniform
⇒ W A < W B < WC ∆E rod about an axis through one end and
∴ = 8 or percentage increase = 800% perpendicular to length is
E1
DAY EIGHT ROTATIONAL MOTION 83

m l2 6 Torque acting on disc is, constant. But other quantities like


I = 1 1 moment of inertia, rotational kinetic
3 τ = F1 r + F2 r = 5 × + 5×
where, m is mass of rod and l its length. 2 2 energy and angular velocity changes.
Torque (τ = Iα ) acting on centre of = 5Nm 11 Let f be force of friction between the
gravity of rod is given by From angular impulse-momentum
shell and horizontal surface.
l l m l2 l theorem,
τ = mg or Iα = mg or α = mg tf
F
2 2 3 2 ∫t i
τ dt = L f − Li
3g
∴ α = ⇒ 5× 5= L − 0⇒L = 25 Nms
2l f f
f
2
6× 4× 4 7 Initial angular momentum of ring,
2 I = ML = = 8 kg m −2 For translational motion,
12 12 L = Iω = Mr 2ω F + f = ma …(i)
From τ = r × F = [2 i$ × (3 i$ + 2$j + 6 k$ )] Final angular momentum of ring and For rotational motion,
four particles system a
= 4 k$ − 12$j FR − fR = Iα = I
L = (Mr 2 + 4mr 2 )ω ′ R
As the rod can rotate along Y-axis, so
As there is no torque on the system, [a = Rα for pure rolling]
τ y = Iα ⇒ − 12$j = 8α
therefore angular momentum remains a
3$ ⇒ F − f =I …(ii)
⇒ α =− j rad s −2 constant. R2
2 Mr 2ω = (Mr 2 + 4 mr 2 )ω ′ On, adding Eqs. (i) and (ii), we get
3 Torque (τ ) acting on a body and angular Mω
⇒ ω′ =
2F =  m +
I   2  5
acceleration (α ) produced in it are M + 4m  a =  m + m  a = ma
 R2   3  3
related as τ = Iα.
8 According to conservation of angular 5 2
Consider a hollow cylinder, around ⇒ F = ma [∴ I shell = mR2 ]
momentum 6 3
which a rope is wounded as shown in
1 6F
the figure. I1ω1 = I2ω2 = MR2ω ⇒ a=
2 5m
Rope 1 
=  MR2 +   R2  ω2
F=30 N 1 M
2 2 4   12 Acceleration of an object rolling down
4 an inclined plane is given by
r ∴ ω2 = ω g sin θ
5 a=
1 + I / mr2
9 The rolling sphere has rotational as well
where, θ = angle of inclination of the
as translational kinetic energy.
Hollow cylinder inclined plane
I 1
∴Kinetic energy = mu2 + Iω2 m = mass of the object
Torque acting on the cylinder due to the 2 2
I = moment of inertia about the axis
force F is τ = Fr I 12 2 2
= mu +  mr  ω
2
through centre of mass
Now, we have τ = Iα 2 25 
I 1 / 2 mr 2 1
where, I = moment of inertia of the 1 mu2 7 For disc, = =
cylinder about the axis through the =mu2 + = mu2 mr 2
mr 2 2
2 5 10
centre = mr 2 For solid sphere,
∴Potential energy = kinetic energy
α = angular acceleration I 2 / 5mr 2 2
7 7u2 = =
τ Fr F 30 mgh = mu2 or h = mr 2 mr 2 5
⇒ α = = = = 10 10g
I mr 2 mr 3 × 40 × 10−2 For hollow sphere,
10 Moment of inertia of a rotating solid I 2 / 3mr 2 2
100 = =
= = 25 rad / s 2 sphere about its symmetrical (diametric) mr 2
mr 2 3
4
2 g sin θ 2
axis is given as, I = mR2 ∴ a disc = = g sin θ
4 The centre of mass is at L distance from 5 1+
1 3
2 Rotational kinetic energy of solid sphere 2
the axis. 1 1 2 1 = 0.66 g sin θ
is K r = Iω2 = × mR2ω2 = mR2ω2
Hence, centripetal force, FC = M   ω2 g sin θ 5
L
2 2 5 5 a solid sphere = = g sin θ
 2 2 7
Angular velocity, ω = V cm R 1+
5
5 Angular momentum, As, we know that external torque, = 0.71 g sin θ
L = Iω …(i) dL g sin θ 3
1 2 1 τ ext = ahollow sphere = = g sin θ
Kinetic energy, KE = Iω = Lω dt 2 5
2 2 1+
where, L is the angular momentum. 3
[from Eq. (i)] = 0.6 g sin θ
2 KE Since, in the given condition, τ ext = 0
∴ L= dL Clearly, asolid sphere > adisk > ahollow sphere
ω ⇒ = 0 or L = constant
dt Type of sphere is not mentioned in the
2 
KE 
 Hence, when the radius of the sphere is question. Therefore, we will assume the
 2  L
Now, L′ = ⇒ L′ = increased keeping its mass same, only given sphere as solid sphere.
2ω 4 ∴ asolid sphere = ahollow sphere > adisk
the angular momentum remains
EXAM BITES

This Pdf Is
Downloaded From
www.exambites.in

Visit www.exambites.in for


More Premium Stuffs,Latest
Books,Test Papers,Lectures etc.
jeeneetadda
jeeneetadda_official
jeeneetadda

VISIT NOW !!
DAY NINE

Gravitation
Learning & Revision for the Day
u Universal Law of Gravitation u Gravitational Potential Energy u Geostationary Satellite
u Acceleration due to Gravity u Escape Velocity u Kepler’s Laws of Planetary
u Gravitational Field u Artificial Satellite Motion
u Gravitational Potential

Universal Law of Gravitation


In this universe, each body attracts other body with a force that is directly proportional
to the product of their masses and inversely proportional to the square of the distance
between them.
Let m1 and m2 be the masses of two bodies and r be the separation between them.
m m
F = G 12 2 .
r
The proportionality constant G is called universal gravitational constant. In SI system,
value of gravitational constant G is 6.67 × 10 −11 Nm2 kg−2 . Dimensional formula of G is
[M −1L3 T −2 ].

Acceleration due to Gravity


The acceleration of an object during its free fall towards the earth is m
called acceleration due to gravity.
If M is the mass of earth and R is the radius, the earth attracts a mass
m on its surface with a force F given by R
PREP
F=
GMm
Centre MIRROR
R2 Your Personal Preparation Indicator
This force imparts an acceleration to the mass m which is known as u No. of Questions in Exercises (x)—
acceleration due to gravity (g).
u No. of Questions Attempted (y)—
By Newton’s law, we have u No. of Correct Questions (z)—
GMm (Without referring Explanations)
F 2 GM
Acceleration (g) = = R = 2
m m R u Accuracy Level (z / y × 100)—
GM u Prep Level (z / x × 100)—
On the surface of earth, g = 2
R
In order to expect good rank in NEET, your
Substituting the values of G, M, R, we get g = 9.81 ms−2 . Accuracy Level should be above 85 & Prep
Level should be above 75.
Mass of the earth m = 6 × 10 24 kg and radius of the earth R = 6.4 × 106 m.
DAY NINE GRAVITATION 85

Gravitational field intensity due to a solid sphere


Variation in g with Altitude and Depth
l

(e.g. earth) of mass M and radius R at a point distant r from


The value of g is variable and can vary in same cases as GM
its centre (r > R) is E = 2
mentioned below r
l
Value of acceleration due to gravity ( g ) at a height ( h ) from GM
and at the surface of solid sphere, E = 2 .
the surface of the earth is given by R
gR2 However, for a point r < R, we find that
g′ = GMr
(R + h)2 E= 3
R
 2h 
If h << R, then g′ = g 1 −
 R 
l
Due to a body in the form of uniform shell gravitational
field intensity at a point outside the shell (r > R) is given
l
Value of acceleration due to gravity (g) at a depth (d) from GM
by E= 2
the surface of the earth is given by r
 d But at any point inside the shell, gravitational intensity is
g′ = g  1 − 
 R zero.
At the centre of the earth d = R and hence, g′ = 0.
Gravitational Potential
Variation in the Value of (g) Due to Gravitational potential at any point in a gravitational field is
Rotation of the Earth defined as the work done in bringing a unit mass from
infinity to that point.
Due to rotation of the earth, the value of g decreases as the W
speed of rotation of the earth increases. The value of Gravitational potential, V = lim .
m 0→ 0 m0
acceleration due to gravity at a latitude is g′λ = g − Rω2 cos2 λ
GM
Following conclusions can be drawn from the above Gravitational potential due to a point mass is V = − .
r
discussion
Gravitational potential is always negative. It is a scalar term
The effect of centrifugal force due to rotation of the earth is
and its SI unit is J kg −1.
l

to reduce the effective value of g.


l
The effective value of g is not truely in vertical direction. For Solid Sphere
At the equators, λ = 0° l
At a point outside the solid sphere, (e.g. earth), i.e.
Therefore, g′ = g − Rω2 (minimum value) GM
r > R, V = − .
l
At the poles, λ = 90 ° r
Therefore, g′ = g (maximum value) l
At a point on the surface of spherical,
GM
V =− .
Gravitational Field R
l
At a point inside the sphere, (r < R).
The space surrounding a material body in which its
gravitational force of attraction can be experienced is GM GM  r2 
V =− (3 R2 − r 2 ) = − 3 − 2 
called its gravitational field. 2 R3 2R  R 
At the centre of solid sphere,
Gravitational Field Intensity
l

3GM 3
Gravitational field intensity at any point is defined as the V =− = Vsurface
2R 2
force experienced by any test mass devided by the magnitude
of test mass when placed at the desired point. For Spherical Shell
Mathematically, l
At a point outside the shell,
F
Gravitational field intensity, E = GM
m0 V =− where, r > R.
r
where, m0 is a small test mass. The SI unit of gravitational l
At a point on the surface of spherical shell,
intensity is N kg −1. GM
V =−
l
Gravitational intensity at a point situated at a distance r R
from a point mass M is given by l
At any point inside the surface of spherical shell
GM GM
E= 2 V =− = Vsurface
r R
86 40 DAYS ~ NEET PHYSICS DAY NINE

Relation between Gravitational Field Escape Velocity


and Gravitational Potential It is the minimum velocity with which a body must be
projected from the surface of the earth so that it escapes the
If r1 and r2 are position of two points in the gravitation field
gravitational field of the earth. We can also say that a body,
(I), then change in gravitational potential projected with escape velocity, will be able to go to a point
r2
V (r2 ) − V (r1) = − ∫ I ⋅ dr which is at infinite distance from the earth.
r1
The value of escape velocity from the surface of a planet of
⇒ dV = − I ⋅ dr = I x $i + I y $j + I z k$ , then mass M, radius R and acceleration due to gravity g is
and dr = dx $i + dy $j + dz k$ 2GM
vescape = = 2gR
R
= (I $i + I $j + I k),
x y
$
z
vescape = 2 vorbital
dV = − I.dr = −I x dx − I y dy − I z dz
∂V $ ∂V $ ∂V $ Escape velocity does not depend upon the mass or shape or
Thus, I=− i− j− k size of the body as well as the direction of projection of the
∂x ∂y ∂z
body. For earth value of escape velocity is 11.2 kms−1.
Remember that partial differentiation indicates that variation
of gravitational potential in counter along the variation of
x-coordinate, then other coordinates (i.e. y and z) are Artificial Satellites
assumed to be constant.
Artificial satellites are man made satellites launched from the
earth. The path of these satellites are elliptical with the centre
Gravitational Potential Energy of earth at a foci of the ellipse. However, as a first
Gravitational potential energy of a body or system is negative approximation we may consider the orbit of satellite as
of work done by the conservative gravitational forces F in circular.
bringing it from infinity to the present position.
Mathematically, gravitational potential energy
Orbital Velocity of Satellite
r Orbital velocity of a satellite is the velocity required to put
U = − W = − ∫ F ⋅ dr
∞ the satellite into its orbit around the earth. The orbital
velocity of satellite is given by
l
The gravitational potential energy of two particles of
masses m1 and m2 separated by a distance r is given by GM gR2 gR2
vo = = =
Gm1m2 r r (R + h)
U=−
r If h << R or r ~
− R, then
l
The gravitational potential energy of mass m at the surface GM
vo = = gR = 7.9 kms – 1
of the earth is R
GMm
U=−
R Period of Revolution
l
Difference in potential energy of mass m at a height h from It is the time taken by a satellite to complete one revolution
the earth’s surface and at the earth’s surface is around the earth.
U( R + h ) − UR =
mgh 2 πr r3 r3 3π
h Revolution period, T = = 2π = 2π =
1+ vo GM gR2 G. e
R
R
− mgh, if h << R
~ − R, then T = 2π
If r ~ = 84.6 min.
g
l
For three particles system,
Gm1m2 Gm1m3 Gm2 m3  Height of Satellite in Terms of Period
U = − + + 
 r 12 r 13 r23  The height of the satellite (from the earth planet) can be
determined by its time period and vice-versa.
n (n − 1)
l
For n-particles system, pairs form and total As the height of the satellite in terms of time period,
2
1/ 3
potential energy of the system is sum of potential energies  gR2T 2 
of all such pairs. h=r −R= 2 
− R.
 4π 
DAY NINE GRAVITATION 87

Energy of Satellite These laws are


1 GMm 1. Law of Orbits All the planets move around the sun in an
Kinetic energy of satellite, K = mv20 = . elliptical orbit with sun at one of the focus of ellipse.
2 2r
GMm Minor axis
Potential energy of satellite, U = −
r
GMm
and total energy of satellite E = K + U = − = − K. Planet Swept area
2r
Major axis
Binding Energy of Satellite Perihelion
S
Apehelion
It is the energy required to remove the satellite from its orbit or Perigee or Apogee
and take it to infinity. a
GMm
Binding energy = − E = + 2. Law of Areas The line joining the sun to the planet sweeps
2r out equal areas in equal intervals of time, i.e. areal
velocity of the planet w.r.t. sun is constant. This is called
Angular Momentum of Satellite the law of area, which indicates that a planet moves faster
Angular momentum of a satellite, L = mv0r = m2GMr near the sun and slowly when away from the sun.
B
A
Geostationary Satellite C I

If an artificial satellite revolves around the earth in an II


equatorial plane with a time period of 24 h in the same sense D
as that of the earth, then it will appear stationary to the
observer on the earth. Such a satellite is known as a
geostationary satellite or parking satellite. 3. Law of Periods The square of the planet’s time period of
revolution is directly proportional to the cube of
semi-major axis of its orbit.
Kepler’s Laws of Planetary Motion T 2 ∝ a3
Kepler discovered three empirical laws which accurately where a is the semi-major axis.
describe the motion of planets.

DAY PRACTICE SESSION 1

FOUNDATION QUESTIONS EXERCISE


1 The magnitude of the force of gravity between two covered by the smaller body before collision is
identical objects is given by F0. If the mass of each object j
CBSE AIPMT 2015
is doubled but the distance between them is halved, then (a) 2.5 R (b) 4.5 R (c) 7.5 R (d) 1.5 R
the new force of gravity between the object will be 4 Force between two objects of equal masses is F . If 25%
(a) 16F0 (b) 4 F0 (c) F0 (d) F0 / 2 mass of one object is transferred to the other object, then
2 Two astronauts are floating in gravitational free space the new force will be
after having lost contact with their spaceship. The two will F 3F 15F
(a) (b) (c) (d) F
j
NEET 2017 4 4 16
(a) keep floating at the same distance between them 5 Let F1 is the gravitational force experienced by a particle
(b) move towards each other 3 −1
at a height Re above the earth’s surface and F2 is
(c) move away from each other 2 2
(d) will become stationary the gravity force experienced by a particle at a depth
3 −1
3 Two spherical bodies of masses M and 5M and radii R R e below the earth’s surface, then
and 2R are released in free space with initial separation 2 2
between their centres equal to 12 R. If they attract each (R e is radius of earth)
other due to gravitational force only, then the distance (a) F1 = F2 (b) F1 > F2
(c) F1 < F2 (d) None of these
88 40 DAYS ~ NEET PHYSICS DAY NINE

6 The depth d at which the value of acceleration due to 16 Dependence of intensity of gravitational field (E ) of earth
1 with distance (r ) from centre of earth is correctly
gravity becomes times the value at the surface,
n represented by j CBSE-AIPMT 2014

(R = radius of the earth)


n − 1  n 
(b) R 
R R E E
(a)  (c) (d) R  
n  n  n2  n + 1 (a) O R (b) O
R r
r
7 If the mass of the Sun were ten times smaller and the
universal gravitational constant were ten times larger in
magnitude, which of the following is not correct?
E E
(a) Time period of a simple pendulum on the Earth would R
decrease (c) O (d) O
r R r
(b) Walking on the ground would become more difficult
(c) Raindrops will fall faster
(d) ‘g’ on the Earth will not change
17 A thin rod of length L is bent to form a semi-circle. The
8 If the value of g at the surface of the earth is 9.8 m/s 2 , mass of rod is M. What will be the gravitational potential
then the value of g at a place 480 km above the surface at the centre of the circle?
of the earth will be (radius of the earth is 6400 km) GM GM πGM πGM
(a) − (b) − (c) − (d) −
(a) 8.4 m/s 2 (b) 9.8 m/s 2 (c) 7.2 m/s 2 (d) 4.2 m/s 2 L 2 πL 2L L
9 A what depth below the surface of the earth, the value of 18 Infinite number of bodies, each of mass 2 kg are situated
g is the same as that at a height of 5 km? on x-axis at distance 1 m, 2 m, 4 m, 8 m respectively
(a) 5 km (b) 2.5 km (c) 10 km (d) 6 km from the origin. The resulting gravitational potential due to
10 Assuming the earth to be a sphere of uniform mass this system at the origin will be j
NEET 2013
density, how much would body weight half way down to 8 4
(a) −G (b) − G (c) − G (d) −4 G
the centre of earth, if it weighed 250 N on the surface? 3 3
(a) 150 N (b) 175 N (c) 125 N (d) 200 N 19 At what height from the surface of earth the gravitation
11 The acceleration due to gravity at a height 1 km above potential and the value of g are −5.4 × 107 J kg −2 and
the earth is the same as at a depth d below the surface 6.0 ms −2 respectively? Take, the radius of earth as
of earth. Then j
NEET 2017 6400 km. j
NEET 2016
1 3
(a) d = km (b) d = 1km (c) d = km (d) d = 2 km (a) 1600 km (b)1400 km (c) 2000 km (d) 2600 km
2 2
20 A particle of mass M is situated at the centre of a
12 The height at which the weight of a body becomes
spherical shell of same mass and radius a. The
1/16th, its weight on the surface of the earth (radius R), is
gravitational potential at a point situated at a /2 distance
j
CBSE AIPMT 2012
from the centre, will be j
CBSE AIPMT 2010
(a) 5 R (b) 15 R (c) 3 R (d) 4 R
3 GM 2 GM GM 4GM
13 A spherical planet has a mass M p and diameter Dp . A (a) − (b) − (c) − (d) −
a a a a
particle of mass m falling freely near the surface of this
planet will experience an acceleration due to gravity, 21 A body of mass m taken from the earth’s surface to the
equal to j
CBSE AIPMT 2012 height equal to twice the radius (R) of the earth. The
(a) 4 GM p / Dp2 (b) GM pm / Dp2 (c) GM p / Dp2 (d) 4 GM pm / Dp2 change in potential energy of body will be j
NEET 2013
2 1
14 A mass m is at a distance x from one end of a uniform (a) mg 2R (b) mgR (c) 3mgR (d) mgR
rod of length l and mass M. The gravitational force on the 3 3
mass due to the rod is 22 When a body is lifted from surface of the earth to a height
M m equal to the radius of the earth, then the change in its
potential energy is
l x 1
(a) mgR (b) 2mgR (c) mgR (d) 4 mgR
GMm GMm GMm GmM 2
(a) (b) (c) (d)
(x + l)2 x (l + x) x2  l + x 
2
23 The orbital velocity of an artificial satellite in a circular
2  orbit just above the earth’s surface is v. For a satellite
15 In a gravitational field, if a body is bound with the earth, orbiting at an altitude of half the earth’s radius the orbital
then total mechanical energy has velocity is
(a) positive (b) zero (c) negative 3 3 2 2
(a) v (b) v (c) v (d) v
(d) may be positive, negative or zero 2 2 3 3
DAY NINE GRAVITATION 89

24 Two satellites of earth, S1 and S 2 are moving in the same (a) the angular momentum of S about the centre of the earth
orbit. The mass of S1 is four times the mass of S 2 . Which changes in direction, but its magnitude remains constant
one of the following statements is true? (b) the total mechanical energy of S varies periodically with
time
(a) The time period of S1 is four times that of S2
(c) the linear momentum of S remains constant in
(b) The potential energies of earth and satellite in the two
magnitude
cases are equal
(d) the acceleration of S is always directed towards the
(c) S1 and S2 are moving with the same speed
centre of the earth
(d) The kinetic energies of the two satellites are equal
25 A remote sensing satellite of earth revolves in a circular 34 A planet moving along an elliptical orbit is closest to the
orbit at a height of 0.25 × 10 m above the surface of
6 sun at a distance r1 and farthest away at a distance of r2 .
If v1 and v 2 are the linear velocities at these points
earth. If earth’s radius is 6.38 × 106 m and g = 9.8 ms − 2 ,
v
then the orbital speed of the satellite is j CBSE AIPMT 2015 respectively, then the ratio 1 is
v2 j CBSE AIPMT 2011
(a) 7.76 kms − 1 (b) 8.56 kms − 1 (c) 9.13 kms − 1 (d) 6.67 kms − 1
(a) r2 /r1 (b) (r2 /r1)2 (c) r1 / r2 (d) (r1 /r2 )2
26 The radii of circular orbits of two satellites A and B of the
earth are 4 R and R, respectively. If the speed of satellite 35 The condition for a uniform spherical mass m of radius r
A is 3 v, then the speed of satellite B will be to be a black hole is
j CBSE AIPMT 2010
(G = gravitational constant and g = acceleration due to
gravity)
(a) 3v/4 (b) 6 v (c) 12 v (d) 3v/2 1/ 2 1/ 2
(a) 
2 Gm 
(b) 
2 gm 
27 The minimum energy required to send a 3000 kg  ≤C  =C
 r   r 
spacecraft from the earth to a far away location where 1/ 2 1/ 2
(c) 
2 Gm 
(d) 
gm 
earth’s gravity is negligible, would be (Me = 6 × 10 24 kg ,  ≥C  ≥C
 r   r 
Re = 6400 km)
. × 1011 J (b) 9 × 1010 J (c) 1 × 1011 J
(a) 188 (d) 6 × 1011 J 36 The velocity with which a projectile must be fired to
28 For a satellite moving in an orbit around the earth, the escape from the earth does not depend upon
ratio of kinetic energy to potential energy is (a) mass of earth (b) mass of projectile
1 1 (c) radius of earth (d) None of these
(a) 2 (b) (c) (d) 2
2 2
37 A body is dropped from a height R e (radius of earth)
29 Network done on a satellite revolving in an elliptical orbit above the earth surface. It strikes the earth with speed vo ,
in one complete revolution v
if ve is the escape velocity from earth’s surface, then o is
(a) is zero (b) is non-zero ve
(c) may be zero (d) None of these (a) 2 : 1 (b) 1 : 2 (c) 1 : 2 (d) 2 : 1
30 An artificial satellite moving in a circular orbit around the 38 Escape velocity on the earth is 11.2 kms . What would −1
earth has a total (kinetic + potential) energy E 0. Its be the escape velocity on a planet whose mass is
potential energy is 1000 times and radius is 10 times that of earth?
(a) − E 0 (b) 1.5 E 0 (c) 2E 0 (d) E 0 (a) 112 kms −1 (b) 11.2 kms −1 (c) 1.12 kms −1 (d) 3.7 kms −1
31 A satellite of mass m is orbiting the earth (of radius R) at 39 A black hole is an object whose gravitational field is so
a height h from its surface. The total energy of the strong that even light cannot escape from it. To what
satellite in terms of g 0, the value of acceleration due to
approximate radius would earth (mass = 5.98 × 1024 kg)
gravity at the earth’s surface is j
NEET 2016
have to be compressed to a black hole?
mg0R 2 mg0R 2 2mg0R 2 2mg0R 2
(a) (b) − (c) (d) − j
CBSE AIPMT 2014
2(R + h) 2(R + h) R+h R+h −9 −6 −2
(a) 10 m (b) 10 m (c) 10 m (d) 100 m
32 An earth satellite is moved from one stable circular orbit 40 The ratio of escape velocity at earth (ve ) to the escape
to another higher stable circular orbit. Which one of the
velocity at a planet (v p ) whose radius and mean density
following quantities increase for the satellite as a result of
are twice as that of earth is j
NEET 2016
this change?
(a) 1 : 2 2 (b) 1: 4 (c) 1 : 2 (d) 1: 2
(a) Angular momentum (b) Kinetic energy
(c) Angular velocity (d) Linear orbital speed 41 If the distance between the earth and sun were half its
present value, the number of days in a year would have
33 A satellite S is moving in an elliptical orbit around the
been
earth. The mass of the satellite is very small as compared
to the mass of the earth. Then, j
CBSE AIPMT 2015 (a) 64.5 (b) 129 (c) 182.5 (d) 730
90 40 DAYS ~ NEET PHYSICS DAY NINE

42 The ratio of mean distances of three planets from the sun 45 The figure shows elliptical orbit m v
C
are 0. 5 : 1 : 1. 5 , then the square of time periods are in the of a planet m about the sun S. B
ratio of The shaded area SCD is twice A S
(a) 1 : 4 : 9 (b) 1 : 9 : 4 (c) 1 : 8 : 27 (d) 2 : 1 : 3 the shaded area SAB. If t1 is D
43 Kepler’s third law states that square of period of the time for the planet to move
revolution (T ) of a planet around the sun, is proportional from C to D and t 2 is the time to
to third power of average distance r between sun and move from A to B, then j CBSE AIPMT 2009
planet (a) t1 > t 2 (b) t1 = 4 t 2 (c) t1 = 2 t 2 (d) t1 = t 2
46 How long will a satellite, placed in a circular orbit of
i.e. T 2 = Kr 3 . th
 1
Here, K is constant. radius that is   the radius of a geostationary satellite,
 4
If the masses of sun and planet are M and m take to complete one revolution around the earth
respectively, then as per Newton’s law of gravitation (a) 12 h (b) 6 h (c) 3 h (d) 4 h
force of attraction between them is 47 The planet neptune travels around the sun with a period
GMm of 165 yr. What is the radius of orbit approximately, if the
F = , here G is gravitational constant.
r2 orbit is considered as circular?
The relation between G and K is described as (a) 20R1 (b) 30R1 (c) 25 R1 (d) 35 R1
j CBSE AIPMT 2015 48 At its aphelion, the planet mercury is 6 .99 × 1010 m from
(a) GK = 4 π 2
(b) GMK = 4 π 2 the sun and at its perihelion it is 4 .6 × 1010 m from the sun.
(c) K = G (d) K =
1 If its orbital speed of aphelion is 3.88 × 104 ms −1, then
G its perihelion orbital speed would be
44 A geostationary satellite is orbiting the earth at a height of (a) 3 .88 × 104 ms−1 (b) 5 .90 × 104 ms−1
5 R above that surface of the earth, R being the radius of (c) 5 .00 × 104 ms−1 (d) 5 .5 × 104 ms−1
the earth. The time period of another satellite in hour at a 49 The time period of moon’s revolution is 27.3 days and
height of 2 R from the surface of the earth is radius of the earth is 6.37 × 106 m, distance to the moon is
j
CBSE AIPMT 2012 3.84 × 108 m, then the mass of the earth is
(a) 5 (b) 10 (c) 6 2 (d) 6/ 2 (approximately)
(a) 1024 kg (b) 1016 kg (c) 1016 kg (d) 105 kg

DAY PRACTICE SESSION 2

PROGRESSIVE QUESTIONS EXERCISE


1 The kinetic energies of a planet in 3 The distance of the centres of moon and earth is D. The
B
an elliptical orbit about the Sun, at mass of the earth is 81 times the mass of the moon. At
positions A, B and C are K A , KB A C what distance from the centre of the earth, the
S
and KC , respectively. AC is the gravitational force on a particle will be zero?
major axis and SB is D 2D 4D 9D
(a) (b) (c) (d)
perpendicular to AC at the position of the Sun S as 2 3 3 10
shown in the figure. Then 4 A geostationary satellite orbits around the earth in a
(a) KB < K A < KC (b) K A > KB > KC circular orbit of radius 36000 km. Then, the time period of
(c) K A < KB < KC (d) KB > K A > KC a spy-satellite orbiting a few 100 km above the earth’s
surface (R = 6400 km ) will approximately be
2 Imagine a new planet having the same density as that of 1
earth but it is 3 times bigger than the earth in size. If the (a) h (b) 1 h (c) 2 h (d) 4 h
2
acceleration due to gravity on the surface of earth is g
and that on the surface of the new planet is g′ , then
5 In the figure given below, equal A B
m m
g
mass m are hung in a balance.
(a) g ′ = 3 g (b) g ′ = h1 h2
9 The error in weighing, if h1 > h2 and
(c) g ′ = 9 g (d) g ′ = 27 g R is the radius of the earth is
DAY NINE GRAVITATION 91

2mg 2mg 13 The binding energy of a particle (mass = 50 kg) and


(a) (h 1 − h 2 ) (b) (h 2 − h 1)
R R earth system is (G = 6.67 × 10−11 Nm 2 kg − 2 ;
 2h   h + h1 
(c) mg  1 −  (d) 2mg  2  Me = 6 × 1024 kg; Re = 6400 km)
 R 1   hh 
  1 2  (a) 3.86 × 1012 J (b) 4 × 106 J
R (c) 3.13 × 109 J (d) 1.56 × 109 J
6 There is a crater of depth on the surface of the moon
100 14 A 500 kg satellite is in a circular orbit at an altitude of
of radius R. A projectile is fired vertically upwards from 500 km above the earth’s surface. Because of air friction,
the crater with a velocity equal to the escape velocity v the satellite eventually falls to the earth’s surface, where it
from the surface of the moon. Maximum height attained
hits the ground with a speed of 2 kms −1. The work done
by the projectile is
R R by air friction is (Me = 6 × 1024 kg, Re = 6400 km )
(a) 2R (b) 99.5 R (c) (d)
2 995
. . × 1011 J
(a) 167 (b) 6 × 109 J
(c) − 6 × 109 J (d) − 1. 67 × 1011 J
7 An object of mass m is raised from the surface of the
earth to a height equal to the radius of the earth, that is, 15 The earth circles around the sun once a year. The work
taken from a distance R to 2R from the centre of the which would have to be done on the earth to bring it to
earth. What is the gain in its potential energy? rest relative to the sun is (Ignore the rotation of earth
1 1 1 1
(a) mg R (b) mg R (c) mg R 2 (d) mg R about its own axis) given that the mass of the earth.
2 4 2 6
(a) 2.7 × 1034 J (b) 3.7 × 1033 J
8 An asteroid of mass m is approaching the earth, initially (c) − 2.7 × 1033 J . × 1023 J
(d) 67
at a distance of 10 R e with speed vi . It hits the earth with
a speed vf (R e and M e are radius and mass of the earth), 16 The mass and diameter of a planet are twice those of
then earth. The period of oscillation of pendulum on this planet
2 Gm  2 GM e 
 1 −  (b) vf = vi +
1 1 will be (if it is a second’s pendulum on earth)
(a) =
vf2 +vi2 2 2
1 + 
M eR  10  Re  10  1
(a) s (b) 2 2 s
2 GM e  2
 1 −  (d) vf = vi +
2 Gm 
 1 − 
1 1
(c) vf2 = vi2 + 2 2
1
Re  10  Re  10  (c) 2 s (d) s
2
9 The earth is assumed to be a sphere of radius R. A
platform is arranged at a height R from the surface of the
17 The weight of a body on the surface of the earth is 63 N.
earth. The escape velocity of a body from this platform is What is the gravitational force on it due to the earth at a
fve , where ve is its escape velocity from the surface of the height equal to half the radius of earth?
earth. The value of f is (a) 34 N (b) 28 N
1 (c) 39 N (d) 42 N
(a) 2 (b)
2
1 1 18 Two bodies of mass m and M are placed a distance d
(c) (d) apart. The gravitational potential at the position, where
3 2
the gravitational field due to them is zero isV . Then,
10 The escape velocity from earth is ve . A body is projected
G Gm
with velocity 2ve . With what constant velocity will it move (a)V = − (m + M ) (b)V = −
in the inter planetary space? d d
GM G
(a) ve (b) 2 ve (c)V = − (d)V = − ( m + M) 2
(c) 3 ve (d) 5 ve d d

11 The radius of a planet is R. A satellite revolves in a circle 19 A particle hanging from a spring stretches it by 1 cm at
of radius r with angular speed ω. The acceleration due to earth’s surface. How much will the same particle stretch
gravity on planet’s surface would be the spring at a place 800 km above the earth’s surface?
r 3ω r 2ω3 (Take, radius of the earth = 6400 km)
(a) (b)
R 2R (a) 1 cm (b) 2 cm
r 3ω2 r 2ω2 (c) 0.53 cm (d) 0.79 cm
(c) (d)
R2 R 20 The escape velocity of a body on the Earth’s surface is
12 A particle is fired vertically upwards with a speed of ve . A body is thrown up with a speed 5 ve . Assuming
−1 that the sun and planets do not influence the motion of
15 kms from the surface of earth. The speed with which
it moves in intersteller space, is the body, the velocity of the body at infinite distance is
v
(take escape speed from surface of earth as v ∞ . Then, the value of ∞ is
11.2 kms −1) ve
(a) zero (b) 3.8 kms −1 (a) zero (b) 1
(c) ≈ 10 kms −1 (d) ≈ 4 kms −1 (c) 2 (d) 3
92 40 DAYS ~ NEET PHYSICS DAY NINE

ANSWERS
SESSION 1 1 (a) 2 (b) 3 (c) 4 (c) 5 (c) 6 (b) 7 (d) 8 (a) 9 (c) 10 (c)
11 (d) 12 (c) 13 (a) 14 (b) 15 (c) 16 (a) 17 (d) 18 (d) 19 (d) 20 (a)
21 (b) 22 (c) 23 (c) 24 (c) 25 (a) 26 (b) 27 (a) 28 (b) 29 (a) 30 (c)
31 (b) 32 (a) 33 (d) 34 (a) 35 (c) 36 (b) 37 (c) 38 (a) 39 (c) 40 (a)
41 (b) 42 (c) 43 (b) 44 (c) 45 (c) 46 (c) 47 (b) 48 (b) 49 (b)
SESSION 2 1 (b) 2 (a) 3 (d) 4 (c) 5 (a) 6 (b) 7 (a) 8 (c) 9 (b) 10 (c)
11 (c) 12 (c) 13 (c) 14 (d) 15 (c) 16 (b) 17 (b) 18 (d) 19 (d) 20 (c)

Hints and Explanations


G × (2m1 ) (2m2 )
1. Fnew = 6 At depth g ′ = g  1 − h  or g  1 − d  8 The value of g on the surface of the
(r / 2) 2  R  R  1
earth, g ∝
g′ 1 
= =  1 − 
Gm1 m2 d R2
= 16 = 16 F 0
r2 g n  R At height h from the surface of the
n − 1 earth,
2 In the space, there is no external gravity. or d = R  . 1
Due to masses of the astronauts, there  n  g′ ∝
will be small gravitational attractive (R + h2 )
force between them. Thus, these 7 Let the original mass of Sun was Ms and R2 9.8 × (6400)2
∴g ′ = g =
astronauts will move towards each gravitational constant G’. (R + h ) 2
(6400 + 480)2
other. According to the question,
M = 8.4 m/s2
3 Suppose, the smaller body cover a New mass of Sun, Ms′ = s
distance x before collision, then 10
9 Acceleration due to gravity at depth d
New gravitational constant, G ′ = 10G below the surface of earth,
5M
g d = g  1 − 
M As, the acceleration due to gravity is d
…(i)
given as  R
2R GM E
R g = …(i) Acceleration due to gravity at height h
R2 from the surface of the earth,
9R
g h = g  1 −
where, M E is the mass of Earth and R is 2h
 …(ii)
12R the radius of the Earth.  R 
Mx = 5M (9R − x ) or x = 45R − 5x Now, new acceleration due to gravity, Here, g h = gd
∴ g  1 −
45R G ′M E 10 M E G 2h  d 2h d
or x= = 7.5 R g′ = = …(ii)  = g 1 −  ⇒ =
 R   R R R
6 R2 R2
GM1 M2 ∴ g ′ = 10 g [from Eqs. (i) and (ii)] ∴ d = 2h
4 F = = G × 100 × 100 Thus, d = 2 × 5 = 10 km
R This means the acceleration due to
G 1 gravity has been increased. Hence, force 10 Weight of the body at earth’s surface,
=
R 10000 of gravity acting on a body placed on or w = mg = 250 N …(i)
G G surface of the Earth increases.
F ′ = × 125 × 75 = × 9375 Acceleration due to gravity at depth h
R R Due to this, rain drops will fall faster, from earth’s surface
F 15  h
F′ = × 9375⇒ F ′ = F walking on ground would become more
g ′ = g 1 −
100 16 
difficult.  Re 
5 At height h above earth’s surface, As, time period of the simple pendulum Here, h = R e /2
2
 Re  is  R /2
∴g ′ = g  1 − e  = g  1 −  ⇒ g ′ =
GM 1 g
g′ = =g ×  l
(R e + h ) 2  R + h T = 2π  Re   2 2
 e 
g
At depth h below earth’s surface, ∴ Weight of the body at depth h,
1 mg
GM (R e − h ) Re − h or T ∝ w ′= mg =
g ′′ = =g   g 2
R3e  R e  250
Thus, time period of the pendulum also Using Eq. (i), we get w ′= = 125N
As, g ′ < g ′′, then ⇒ F1 < F2 decreases with the increase in g. 2
So, weight of the body will be 125 N.
DAY NINE GRAVITATION 93

11 g h = Acceleration due to gravity at 15 Total mechanical energy of any closed 21 Change in potential energy,
−  −
height h above earth’s surface system is always negative. GMm GMm 
∆U = − 
2 R + 2R  R 
 R 
=g   2h  16 E in = − GMr
 = g 1 −  R3 E GMm GMm
 R + h  R R r =− +
GM O 3R R
g d = Acceleration at depth d below Eout = −
r2 2GMm 2 Q g = GM 
earth’s surface = = mgR
17 πR = L 3R 3  R2 
= g  1 − 
d
 R L mgh
∴ R= , 22 ∆U = ,h= R [given]
Given, when h = 1 km, g d = g h π 1 + h 
πGM  
 R
or g  1 −  = g  1 −
d 2h  V =−
GM
=−

 R  R R L mgR
∴ ∆U =
⇒ d = 2h or d = 2 km 18 The resulting gravitational potential, 2
GM 1 v r1
23 v = or v ∝ ⇒ 2 =
V = −2G  + + + +…
12 According to the question, 1 1 1 1
r v1 r2
 1 2 4 8  r
GMm 1 GMm
= r1 R 2
(R + h )2 16 R2 ∴ v2 = v1 = v = v
V = −2G 1 + + + …
1 1 1
⇒ r2 R 3
where, m = mass of the body  2 22 23  R+
2
GM −1
= gravitational acceleration 24 Orbital velocity of satellite is v 0 = gR
V = −2G  1 − 
and 1
R2 ⇒
 2 which is independent of mass of satellite.
1 1
= 2G −2G Option (c) is correct.
(R + h )2 16R2 ⇒ v =− = = − 4G
1 − 1  1 25 Given, height of a satellite
R 1 R+ h    
or = or =4  2  2 h = 0.25 × 10 6 m
R+ h 4 R
h = 3R 19 Gravitational potential at some height h
from the surface of the earth is given h
13 According to Newton’s law of by
gravitation force, GM R
GMm V =− …(i)
F = R+ h
R2
And acceleration due to gravity at some
Force on planet of mass M P and body of height h from the earth surface can be
mass m is given by given as
GM P m g′ =
GM Earth’s radius, Re = 6.38 × 106 m
F = …(ii)
(D P /2)2 (R + h )2 For the satellite revolving around the
From Eqs. (i) and (ii), we get earth, orbital velocity of the satellite
 where, D P = diameter of planet 
and R = radius of planet = D P  |V | (R + h )2 GMe GMe
=
GM
× v0 = =
 P
2  g′ (R + h ) GM Re  h
Re 1 + 
4GM P m  Re
F = ⇒
|V |
=R+ h (iii)
D 2P g′ gRe
⇒ v0 =
h
As we know that, F = ma Q . × 107 J kg −2
V = − 54 1+
F Re
So, acceleration due to gravity a = and g ′ = 6.0 ms −2
m Substitutes the values of g , Re and h, we
4GM P Radius of earth, R = 6400 km. get
= Substitute these values in Eq. (iii), we
D 2P v 0 = 60 × 106 m/s
get
v 0 = 776
. × 103 m/s = 776
. km/s
14 Let a small element dy at a distance y . × 107
54
=R+ h 26 Orbital velocity of satellite is given by,
from the mass m be taken, then force 6.0
GM
due to this element ⇒ 9 × 10 = R + h
6 v=
r
dy m ⇒ h = (9 − 6.4) × 106 = 2.6 × 106 m Ratio of orbital velocities of A and B is
⇒ h = 2600 km given by,
y vA rB
20 Gravitational potential at point a/2 ⇒ =
Gravitational force on the mass due to vB rA
distance from centre is given by,
rod, R 1
x + l GMdym V =−
GM GM
− = =
F =∫ a a/2 4R 2
x
ly 2 ∴
vA
=
3v
=
1
3GM
GMm  1 1  GMm =− vB vB 2
= − = a
l  x x+ l  x ( x + l ) ∴ v B = 6v
94 40 DAYS ~ NEET PHYSICS DAY NINE

27 Minimum energy required is given by GMe On putting the value of v 2 from Eq. (i),
37 vo =
Re 4 π2 r 2
mv 2 GMe m we get T 2 =
− =0 2GMe v 1  GM 
2 Re while, v e = ⇒ o = = 1 : 2.  
Re ve 2  r 
GMe m 4 π2 r 3
K min = = 1.88 × 1011 J T2 = ...(ii)
Re 38 v e = 2gR = 2 GM
2
R or v e ∝
M
GM
R R
28 Potential energy of satellite, T = kr
2 3
...(iii)
Mass is 1000 times and radius is 10
From Eqs. (ii) and (iii), we get
GMe m times. Therefore, escape velocity will
|U | = 4 π2
Re become 10 times. = k ⇒ GMK = 4 π2
⇒ v e = 11.2 × 10 = 112 kms −1 GM
Kinetic energy of satellite,
2GM
44 From Kepler’s third law
1 GM e m 39 Ve = =C
K = R T 2 ∝ r3
2 Re
2GM  where,T = time period of 
⇒R =
K 1 GM e m Re 1 C2  satellite 
Thus, = × =  r = radius of elliptical orbit 
|U | 2 R e GM e m 2 2 × 6.67 × 10−11 × 598
. × 1024  (semi major a xis) 
=
(3 × 108 )2
29 The only force acting on satellite is Hence, T12 ∝ r13 and T22 ∝ r23
2 × 6.67 × 598
.
gravitational force which is conservative = × 10−3 m
9 T22 r23 (3R )3 T22 1
in nature and the work done by So, = = or =
= 8.86 × 10−3 m ≈ 10−2 m T12 r13 3
(6R ) T12 8
conservative force along a closed loop is
1 2 24
zero. 40 Since, the escape velocity of earth can be T22 = T1 ⇒ T2 = = 6 2h
given as 8 2 2
30 Potential energy = 2 × ( Total energy)
8 45 The line joining the sun to the planet
= 2E 0 v e = 2gR = R πGρ
3 sweeps out equal areas in equal time
GMm
Because we know that, U = − . [ρ = density of earth] interval i.e. areal velocity is constant.
r dA A A
8 = constant or 1 = 2
GMm ⇒ ve = R πGρ …(i) dt t1 t2
E0 = − 3
2r where, A1 = area under SCD
31 QTotal energy of a satellite at height h is
As it is given that the radius and mean A2 = area under ABS
density of planet are twice as that of A
= KE + PE =
GMm

GMm ⇒ t1 = 1 t2
2(R + h ) (R + h ) earth. So, escape velocity at planet A2

−GMm − mg 0 R2 will be Given, A1 = 2 A2


= = Q g = GM 
 0  8 ∴ t 1 = 2t 2
2(R + h ) 2(R + h )  R2  v p = 2R πG 2ρ …(ii)
3 46 According to Kepler’s, third law
32 L = mvr or L ∝ vr , v ∝ 1 Divide, Eq. (i) by Eq. (ii), we get T 2 ∝ R3
r
8 T2 (R2 )3 /2 (R1 )3 /2
∴ L ∝ r1 /2 R πGρ = =
ve 3 v 1
= ⇒ e = T1 (R1 ) (4R1 )3 /2
i.e. with increase in r , L will increase. vp 8 vp 2 2
2R πG 2ρ T 24
3 T2 = 1 = = 3h
33 As we know that, force on satellite is 8 8
only gravitational force which will 41 From Kepler’s law, T 2 ∝ r 3 . 47 T1 = T earth = 1 yr
always be towards the centre of earth.
Thus, the acceleration of S is always  T1  r 
2 3 2 3 T2 = T neptune = 165 yr
 365 days  =  2 
directed towards the centre of the earth.   =  1 ⇒     Let R1 and R2 be the radii of the circular
 2
T  2
r  T   1
orbits of the earth and neptune
34 From the law of conservation of angular
respectively
momentum, L1 = L2 ⇒ T ≈ 129 days
T12 R3 R3T 2
So, mr1 v 1 = mr2 v 2 42 T 2 ∝ r 3 , r1 : r2 : r3 = 1 : 1 : 3 2
= 1 ∴ R23 = 1 2
3
 where, m = mass the of planet 2 2 T2 R2 T12
 r = radius of orbit  1 27
v = velocity of the planet ∴ T12 : T22 : T32 = :1: = 1 : 8 : 27 R13 × (165)2
 8 8 or R23 =
r1 v 1 = r2 v 2 12
GMm mv 2
v1 r 43 = = Centripetal force ∴R23 = 1652 R13 or R2 = 30 R1
⇒ = 2 r2 r
v2 r1 GM
v2 ⇒ ...(i) 48 From conservation of angular
1 /2 r momentum, mv A r A = mv P r P
35 
2 GM 
 ≥C 2πr
 r  T = 3.88 × 104 × 6.99 × 1010
v ⇒ vP =
4.6 × 1010
36 v e = 2gR , v e is independent of mass of 4 π2 r 2
⇒ T =
2
= 5.90 × 104 ms −1
projectile. v2
DAY NINE GRAVITATION 95

49 Since, moon is satellite of the earth, 3 Force will be zero at the point of zero KE + PE (at A) = PE (at B )
from Kepler’s third law, we have intensity
1  2GM  GmM  2  99  
2

4 π2 R 3 4 π2 R 3 m  − 3R −   R2 
T2 = ⇒ ME = 2  R  2R3   100  
GM E GT 2 d GMm
=−
Putting the values, we have (R + h )

4 × 3.14 × 3.14 × (3.84)3 × 1024 ⇒ R+ h=


2R
⇒ h = 99. 5 R
ME =
6.67 × 10–11 × (27.3 × 24 × 60 × 60)2 x P 0.0199
m1 m2
M E = 6.02 × 1024 kg ⇒ M E ∝ 1024 kg 7 Potential energy of the object at the
GMe m
surface of the earth = −
m1 R
SESSION 2 x= d
m1 + m2 PE of the object at a height equal to the
1 According to the question, GMm
radius of the earth = −
81 M D 2R
vC =
B 81 M + M ∴ Gain in PE of the object
S GMm  GMm 
A C 9D =− − − 
= 2R  R 
10
GMm gR2 × m
vA =+ = [QGM = gR2 ]
4 By Kepler’s law of planetary motion, 2R 2R
The figure above shows an ellipse traced T 2 ∝ r3 =
1
mg R
by a planet around the Sun, S. The ∴ T 2 ∝ (36000)3 2
closed point A is known as perihelion
and (T ′ )2 ∝ (6400 + h )3 8 Applying law of conservation of energy
(perigee) and the farthest point C is
3 for asteroid at a distance 10 R e and at
6400 + h 
Therefore, (T ′ )2 = T 2 
known as aphelion (apogee).
earth’s surface,
Since, as per the result the Kepler’s  36000 
second law of area, that the planet will Ki + Ui = K f + U f …(i)
3
move slowly (v min ) only when it is 6400 
2
>T
farthest from the Sun and more rapidly  36000  vi
(v max ) when it is nearest to the Sun.
3 10 Re
v A = v max , v C = v min
> (24)2  
Thus, 8
Therefore, we can write  45 vf

v A > v B > vC …(i) 24 × 8 × 8


∴ T ′> > 1.8 h Re
Kinetic energy of the planet at any point 45 × 45
is given as,
1 So, T ′≈ 2 h
K = mv 2 Me
2  2h1  Earth
5 W1 = mg  1 + ,
1
Thus, at A, K A = mv 2A  g 
2
W2 = mg  1 +
2h2 
1  Ki =
1
mv 2i
At B, K B = mv 2B  R  Now,
2 2
∆W = W1 − W2 = mg  1 − 2 
2h 2h GM e m
1
At C, K C = mv C2  R R  and Ui = −
2 10R e
2mg
From Eq. (i), we can write = (h1 − h2 ) 1
R Kf = mv 2f
K A > K B > KC 2
6 Let a particle be projected from A which and Uf =−
GM e m
.
2 The acceleration due to gravity on the reaches B at a height h from the surface. Re
new planet can be found using the
It is projected with Substituting these values in Eq. (i), we
relation
GM B get
g = …(i) 1 GM e m 1 GMe m
R2 h mv 2i − = mv 2f −
4 2 10R e 2 Re
But M = πR3ρ, ρ being density.
3 1 1 GM e m GM e m
R/100 ⇒ mv 2f = mv 2i + −
Thus, Eq. (i) becomes R 2 2 Re 10R e
4 A
G × πR3ρ 2GM e 2GM e
3 4 ⇒ v 2f = v 2i + −
∴g = = G × πRρ ⇒ g ∝ R
R2 3 Re 10R e
Similarly, g ′ ∝ R ′ 2GM e  1
g′ R′ g ′ 3R ∴ v 2f = v 2i + 1 − 
∴ = ⇒ = = 3 ⇒ g ′ = 3g 2GM Re  10 
g R g R ve =
R
96 40 DAYS ~ NEET PHYSICS DAY NINE

9 Escape energy = Binding energy of From work-energy theorem,


and d − x=
M
d
sphere mv 2f mv 2  GMe m GMe m  m + M
− = − − +
or
1
mv ′2e =
GMm 2 2  Re r  Gm GM
2 R+ h V =− −
+ Wair friction x d − x
2GM 2GM mv 2f GMe m
or v ′e = = [Q h = R] ⇒ Wair friction = − Gm ( m + M) GM ( m + M)
R+ h 2R =− −
2 2r m ⋅d M ⋅d
GMe m GMe m
But at surface of earth, − + G
Re r =− ( m + M )2
2GM d
ve = 500 × (2 × 103 )2
R = + 6.67 19 Let the mass of the
2
As given, v ′e = f v e particle be ‘m’ and the
× 10−11 × 6 × 2024
2GM 2GM spring constant of the
Hence, = f  1 1 
2R R × 500  − spring is k. The
6 kx
 2 × 6.9 × 10 6.4 × 10 
6
1 acceleration due to
∴ f = gravity at earth’s
x
m
2 = − 1.67 × 1011 J m
GM
surface is g = . m
10 U i + K i = U f + K f 15 ω = 2 π = 2π
R2
mg
T 365 × 24 × 3600 Let extension of spring be ‘x’.
GMm 1 1
− + m (2v e ) 2 = 0 + mv 2 = 1.99 × 10−7 rad/s Since, at equilibrium
R 2 2
1 mg
GM 1 W = K f − K i = 0 − mv 2 [Q v = ωR] kx = mg ⇒ x=
or − + 2v e2 = v 2 2 k
R 2
1 GmM
2GM 8GM = × 6 × 1024 × (1.5 × 1011 ⇒ 1 cm = …(i)
or − + = v2 2 kR2
R R
× 1.99 × 10−7 )2 At a height ‘h’ = 800 km, the extension is
= 3 
6GM 2GM 
or v =  = − 27
. × 1033 J given by
R  R  GMm
GM x′ = …(ii)
= 3 (2 gR ) 16 Gravity, g = k (R + h )2
R2
= 3 ve Me R2p x′ R2

g earth
= × ⇒
ge
=
2 From Eqs. (i) and (ii), =
v 1 cm (R + h )2
11 ω= , g planet M p R2e gp 1
r (6400 km)2
1 T gp ⇒ = 0.79
GM Also, T ∝ ⇒ e = (7200 km)2
So , v = , we get g Tp ge
r ∴ x ′ = 0.79 cm
2 1
1
GM GM ⇒ = , Tp = 2 2 s
⇒ ω= ⇒ ω2 = Tp 2 20 Applying conservation of mechanical
r r r3 energy,
⇒ ω2 r 3 = g × R2 ⇒ g =
r ω
3 2
17 Given, h = Re Energy at initial position = Energy at
2 infinity
R2
Acceleration due to gravity at altitude h 1 GmM
⇒ mv 2i −
2 mv 2f is given by
12 mv − GMm = , g g
2 R
2 R 2 g′= = 1
= mv 2∞ + GPE at infinity
2 2
where, v is the launching speed.  h  R e /2 2
1 +  1 + 
2 GMm  Re   Re  or
⇒ mv 2 − = mv 2f
m( 5 v e )2 − 
1 GM  1
 mR = mv ∞ + 0
g g 4 2
R = = = g …(i) 2  R2  2
2 2 9
⇒ v 2 − v e2 = v 2f , 1 + 1 (3 / 2)
  1 1
 2 or m (5v e2 ) − mgR = mv 2∞
2GM 2 2
where, v e = is the escape speed Weight of the body at earth’s surface,
5 v e2 1 2
R w = mg = 63 N …(ii) ⇒ − gR = v ∞
Weight of the body at altitude h = R e / 2, 2 2
vf = v 2 − v e2 ≈ 10 kms −1
w ′= mg ′= mg
4 5 v e2 2 gR 1 2
…(iii) ⇒ − = v∞
9
13 Binding Energy (BE) = GMe m Using Eq. (ii), we get
2 2 2
Re 5 v e2 v e2 1 2
4 ⇒ − = v ∞ (Q v e2 = 2gR )
w ′= × 63 = 28 N 2 2 2
6.67 × 10−11 × 6 × 1024 × 50 9
= ⇒
1 1 2
(4 v e ) = v ∞
2
6.4 × 106 18 Enet = 0 2 2
= 3.13 × 10 9 J Gm GM 2
∴ =  v∞ 
x2
(d − x ) 2 ⇒   =4
 ve 
14 Orbital speed of satellite, v = GMe where, x is distance from m.
r v∞
x m m ⇒ =2
mv 2 GMe m ∴ = , x= d ve
Initial KE, Ki = = . d − x M m + M
2 2r
DAY TEN

Unit Test 1
(Mechanics)
1 Imagine a light planet revolving around a very massive 7 Suppose, the kinetic energy of a body oscillating with
star in a circular orbit of radius R with a period of amplitude A and at a distance x is given by
revolutionT . If the gravitational force of attraction Bx
KE =
between the planet and the star is inversely proportional x 2 + A2
to R 5 / 2, then T 2 is proportional to
3 The dimensions of B are the same as that of
(a) R (b) R 7 / 2
(c) R 3/ 2
(d) R 3. 75 (a) work /time (b) work × distance
−1
(c) work /distance (d) work × time
2 The escape velocity from the earth’s surface is 11kms .
A certain planet has a radius twice that of the earth, but 8 A soap bubble oscillates with time periodT , which in turn
its mean density is the same as that of the earth. The depends on the pressure ( p ), density (ρ ) and surface
value of the escape velocity from this planet would be tension ( σ ). Which of the following correctly represents
the expression for T 2 ?
(a) 22 kms −1 (b) 11 kms −1
(c) 5.5 kms −1 (d) 16.5 kms −1 (a) ρσ 2 /p 3 (b) ρp 3 /σ (c) p 3 σ/p (d) ρ /p 3 σ

3 A ball projected from ground at an angle of 45° just 9 A boat which has a speed of 5 kmh −1 in still water
across a wall in front. If point of projection is 4 m from crosses a river of width 1 km along the shortest path in
the foot of wall and ball strikes the ground at a distance 15 min. The velocity of the river (in kmh −1) is
of 6 m on the other side of the wall, the height of the (a) 1 (b) 3 (c) 4 (d) 41
wall is 10 The velocity of a particle at an instant is 10 ms −1 and
(a) 4.4 m (b) 2.4 m after 5 s the velocity of the particle is 20 ms −1. The
(c) 3.2 m (d) 1.6 m velocity 3 s before (in ms −1) was
4 The product of pressure and volume have the same units (a) 8 (b) 4 (c) 6 (d) 7
as the product of 11 A particle moving in a straight line covers half the
(a) charge and potential distance with speed of 3 ms −1. The other half of the
(b) electric field strength and distance distance is covered in two equal time intervals with
(c) electromotive force and capacitance
speed of 4.5 ms −1 and 7.5 ms −1, respectively. The
(d) magnetic moment and magnetic induction
average speed of the particle during this motion is
5 The radius of a ball is ( 5 . 2 ± 0 . 2) cm. The percentage (a) 4.0 ms −1 (b) 5.0 ms −1 (c) 5.5 ms −1 (d) 4.8 ms −1
error in the volume of the ball is approximately
12 What will be the ratio of the distance moved by a freely
(a) 11% (b) 4%
(c) 7% (d) 9% falling body from rest in 4th and 5th seconds of journey?
(a) 4 : 5 (b) 7 : 9 (c) 16 : 25 (d) 1 : 1
6 At time t = 0 s, particle starts moving along the X -axis. If
its kinetic energy increases uniformly with time t, then the 13 A particle moves in xy-plane according to the equation
net force acting on it must be proportional to x = 4 t 2 + 5 t + 16 and y = 5 t
(a) t (b) t −1 The acceleration of the particle is
1
(c) t (d) (a) 8 (b) 13 (c) 14 (d) 16
t
98 40 DAYS ~ NEET PHYSICS DAY TEN

14 The figure shows the position-time ( x - t ) graph of 22 Three solids of masses m1, m2 and m3 are connected
one-dimensional motion of a body of mass 0.4 kg. The with weightless string in succession and are placed on a
magnitude of each impulse is frictionless table. If the mass m3 is dragged with a force,
T the tension in the string between m2 and m3 is
2 m2 m3
(a) T (b) T
m1 + m2 + m3 m1 + m2 + m3
x (m)
m1 + m2 m2 + m3
(c) T (d) T
m1 + m2 + m3 m1 + m2 + m3
0 2 4 6 8 10 12 14 16
t (s) 23 A body of mass m slides down an incline and reaches
the bottom with a velocity v, if the same mass were in the
(a) 0.4N - s (b) 0.8 N - s (c) 1.6N - s (d) 0.2 N - s
form of a ring which rolls down this incline, the velocity of
15 The minimum velocity (in ms −1) with which a car driver the ring at bottom would have been
must traverse a curve of radius 150 m and coefficient of 1
(a) v (b) 2v (c) v (d) 2 /5 v
friction 0.6 to avoid skidding is 2
(a) 60 (b) 30 (c) 15 (d) 25
24 A bullet is shot from a rifle. As a result, the rifle
16 A particle is dropped vertically from rest, from a height. recoils. The kinetic energy of rifle as compared to that of
The time taken by it to fall through successive distances bullet is
of 1 m each will be, then (a) less (b) greater
(a) all equal, being equal to 2 /g second (c) equal (d) cannot be concluded
(b) in the ratio of the square roots of the integers 1, 2, 3,...
25 Two bodies of masses m1 and m2 and having velocities
(c) in the ratio of the difference in the square roots of v1 and v 2 respectively, moving along a straight line are
integers, i.e. 1, ( 2 − 1), ( 3 − 2 ), ( 4 − 3 ),K brought to rest by equal resistance forces. If one moves
(d) in the ratio of the reciprocals of square roots of the twice as that of the duration of other, but goes only 1/2 of
1 1 1 1 the distance covered by the other before coming to rest,
integers, i.e. , , , ,…
1 2 3 4 the ratio v1/v 2 and m1/m2 are
1 1 1 1 1 1 1
17 Let A = $i A cos θ + $j A sin θ be any vector. Another (a) ,8 (b) , (c) , (d) ,
4 2 4 8 2 4 6
vector B which is normal to A is
(a) $i B cosθ + $i B sinθ (b) $i B sinθ + $j B cosθ 26 The angle between the two vectors A = 3 $i + 4$j + 5 k$ and
(c) $i B sinθ − $j B cosθ (d) $i B cosθ − $j B sinθ B = 3 $i + 4$j − 5 k$ will be

18 A particle moves from position r1 = 3$i + 2$j − 6 k$ to (a) 90° (b) 0° (c) 60° (d) 45°

position r2 = 14$i + 13$j + 9 k$ under the action of force 27 If | A × B | = 3 A ⋅ B, then the value of | A + B | is
1/ 2
( 4$i + $j + 3 k$ ) N. The work done will be  AB 
(a) (A 2 + B 2 + AB)1/ 2 (b)  A 2 + B 2 + 
 3
(a) 100 J (b) 50 J (c) 200 J (d) 75 J
(c) (A + B) (d) (A 2 + B 2 + 3 AB )1/ 2
19 An elevator car whose floor to ceiling distance is equal to
2.7 m starts ascending with constant acceleration of 28 A force acts on a 3.0 g particle in such a way that the
1.2 ms −2 . After 2 s of the start, a bolt begins falling from position of the particles as a function of time is given by
the ceiling of the car. The free fall time of the bolt is x = 3 t − 4 t 2 + t 3 , where x is in metre and t is in second.
(a) 6 s (b) 0.7 s (c) 1 s (d) 0.54 s The work done during the first 4 s is
20 Two balls of masses m1 and m2 are separated from each (a) 570 mJ (b) 450 mJ (c) 490 mJ (d) 530 mJ
other by a powder charge placed between them. The 29 The kinetic energy of a particle moving along a circle of
whole system is at rest on the ground. Suddenly, the radius R depends upon the distance s as K = as 2 . Then,
powder charge explodes and masses are pushed apart. the force acting on particle is
The mass m1 travels a distance s1 and stops. If the 1/ 2
coefficients of friction between the balls and ground are s2  s2 
(a) 2a (b) 2as 1 + 2 
same, the mass m2 stops after travelling the distance R  R 
m1 m m2 m2 (c) 2as (d) 2a
(a) s 2 = s1 (b) s 2 = 2 s1 (c) s 2 = 12 s1 (d) s 2 = 22 s1
m2 m1 m2 m1 30 A flywheel rotating about a fixed axis has a kinetic
21 A block slides from an inclination of 45°. If it takes time energy of 360 J when its angular speed is 30 rads −1.
twice with friction than to that without friction, then The moment of inertia of the flywheel about the axis of
coefficient of friction for surface is given by rotation is
(a) 1 (b) 0.75 (c) 0.5 (d) 0.25 (a) 0.15 kg-m 2 (b) 0.75 kg-m 2
(c) 0.60 kg-m 2 (d) 0.80 kg-m 2
DAY TEN UNIT TEST 1 (MECHANICS) 99

31 A body of mass m is thrown upwards at an angle θ with 39 For a satellite, escape velocity is 11 kms −1. If the satellite
the horizontal with velocity v. While rising up the velocity is launched at an angle of 60° with the vertical, then
of the mass after t seconds will be escape velocity will be
(a) 11 kms −1 (b) 11 3 kms −1
(a) (v cosθ)2 + (v sinθ)2 (b) (v cosθ − v sin θ)2 − gt 11
(c) kms −1 (d) 33 kms −1
(c) v + g t − 2v sinθ gt
2 2 2
(d) v + g t − 2v cos θgt
2 2 2 3
40 Two bodies of masses m1 and m2 are initially at rest at
32 A tube of length L is filled completely with an infinite distance apart. They are then allowed to move
incompressible liquid of mass M and closed at both the towards each other under mutual gravitational attraction
ends. The tube is then rotated in a horizontal plane about with the escape velocity. Their relative velocity of
one of its ends with a uniform angular velocity ω . The approach at a separation distance r between them is
1/ 2 1/ 2
force exerted by the liquid at the other end is (m1 − m2 )  (m1 + m2 ) 
(a)  2 G (b)  2 G
(a) Mω2L / 2 (b) Mω2L (c) Mω2L /4 (d) Mω2L2 / 2  r   r 
1/ 2 1/ 2
 
(d)  (m1m2 )
33 Two racing cars of masses m1 and m2 are moving in r 2G
(c)
 2 G (m m )   r 
circles of radii r1 and r2 , respectively. Their speeds are  1 2 
such that each makes a complete circle in the same time
t. The ratio of the angular speeds of the first to the 41 The moon’s radius is 1/4 that of the earth and its mass is
second car is 1/80 times that of the earth. If g represents the
(a) 1 : 1 (b) m1 : m2 (c) r1 : r2 (d) m1m2 : r1 r2 acceleration due to gravity on the surface of the earth,
then on the surface of the moon its value is
34 A wheel is rotating at 900 rpm about its axis. When the
(a) ge /4 (b) ge /5 (c) ge /6 (d) ge /8
power is cut-off it comes to rest in 1 min. The angular
retardation (in rad s −2 ) is 42 A circular road of radius 1000 m has banking angle 45° .
(a) π / 2 (b) π /4 (c) π /6 (d) π /8 Calculate the maximum safe speed of a car having mass
200 kg will be, if the coefficient of friction between tyre
35 A long horizontal rod has a bead which can slide along and road is 0.5.
its length and initially placed at a distance L from one
(a) 172 ms −1 (b) 124 ms −1 (c) 99 ms −1 (d) 86 ms −1
end A of the rod. The rod is set in angular motion about
A with constant angular acceleration α. If the coefficient 43 A body of mass m hangs at one end of a string of length
of friction between the rod and the bead is µ and gravity l, the other end of which is fixed. It is given a horizontal
is neglected, then the time after which the bead starts velocity, so that the string would just reach, where it
slipping is makes an angle of 60° with the vertical. The tension in
µ µ 1 the string at bottom most point position is
(a) (b) (c) (d) infinitesimal
α α µα (a) 2mg (b) mg (c) 3mg (d) 3 mg
44 Three blocks of masses 4 kg,
36 A metallic sphere of diameter 10 cm and mass 0.5 kg is 14N
4 kg 2 kg
2 kg, 1 kg respectively are in 1kg
fixed to one end of a thin rod of length 50 cm and mass
1 kg as shown in the given figure. The rod with the contact on a frictionless table
sphere will balance horizontally, when it is supported at a as shown in the figure. If a
point whose distance from the free end of the rod is force of 14 N is applied on the 4 kg block, the contact
force between the 4 kg and the 2 kg block will be
10 cm (a) 2 N (b) 6 N (c) 8 N (d) 14 N
50 cm 45 A solid sphere of radius R / 2 is cut-out of a solid sphere
(a) 45 cm (b) 40 cm (c) 35 cm (d) 30 cm of radius R such that the spherical cavity, so formed
37 Two bodies of mass 10 kg and 5 kg moving in concentric touches, the surface on one side and the centre of the
orbits of radius R and r such that their periods are the sphere on the other side, as shown in figure. The initial
same. Then, the ratio between their centripetal mass of the solid sphere was M. The gravitational field of
acceleration is the left over part of the sphere at a very far off point P
(a) R /r (b) r /R located as shown below.
(c) R 2 /r 2 (d) r 2 /R 2
38 The mean radius of the earth is R, its angular speed on
its own axis is ω and the acceleration due to gravity at O R/2
earth’s surface is g. What will be the radius of the orbit of P
a geostationary satellite? R
(a) (R 2g/ ω2 )1/ 3 (b) (Rg/ ω2 )1/ 3
(c) (R 2ω2 / g)1/ 3 (d) (R 2g/ ω)1/ 3 8 GM 7 GM GM 6 GM
(a) (b) (c) (d)
5 R2 8 R2 8R 2
7 R2
100 40 DAYS ~ NEET PHYSICS DAY TEN

46 A wheel of radius 0.4 m can rotate freely about its axis as 48 If the angular momentum of any rotating body increases
shown in the figure. A string is wrapped over its rim and by 200%, then the increase in its kinetic energy
a mass of 4 kg is hung. An angular acceleration of (a) 400% (b) 800% (c) 200% (d) 100%
8 rad s −2 is produced in it due to the torque. Then,
49 A body is rolling down an inclined plane. If KE of rotation
moment of inertia of the wheel is (take, g = 10 ms −2 )
is 40% of KE in translatory state, then the body is a
(a) ring (b) cylinder (c) hollow ball (d) solid ball
50 Two discs of moment of inertia I1 and I 2 and angular
speeds ω1 and ω 2 are rotating along collinear axes
passing through their centre of mass and perpendicular
to their plane. If the two are made to rotate combindly
4 kg along the same axis the rotational KE of system will be
2 I1ω1 + I 2ω2 (I1 + I 2 ) (ω1 + ω2 )2
(a) 2 kg-m (b) 1 kg-m 2 (a) (b)
(c) 4 kg-m 2 (d) 8 kg-m 2 2 (I1 + I 2 ) 2
(I1ω1 + I 2 ω2 )2
47 A uniform rod AB of length l and mass m is free to rotate (c) (d) None of these
2 (I1 + I 2 )
about point A. The rod is released from rest in horizontal
position. Given that, the moment of inertia of the rod 51 A cord is wound round the circumference of wheel of
ml 2 radius r. The axis of the wheel is horizontal and moment
about A is , the initial angular acceleration of the rod
3 of inertia about it is I. A weight mg is attached to the end
will be of the cord and falls from the rest. After falling through a
distance h, the angular velocity of the wheel will be
l 1/ 2
2gh  2mgh 
A B (a) (b)  2
I + mr  I + mr 
1/ 2
2g l 3 3g  2mgh 
(a) (b) mg (c) gl (d) (c)  2
(d) 2gh
3l 2 2 2l  I + 2mr 

ANSWERS
1 (b) 2 (a) 3 (c) 4 (a) 5 (a) 6 (d) 7 (b) 8 (a) 9 (b) 10 (b)
11 (d) 12 (b) 13 (a) 14 (b) 15 (b) 16 (c) 17 (c) 18 (a) 19 (c) 20 (c)
21 (b) 22 (c) 23 (c) 24 (a) 25 (a) 26 (a) 27 (a) 28 (d) 29 (b) 30 (d)
31 (c) 32 (a) 33 (a) 34 (a) 35 (a) 36 (c) 37 (a) 38 (a) 39 (a) 40 (b)
41 (b) 42 (a) 43 (a) 44 (b) 45 (b) 46 (a) 47 (d) 48 (b) 49 (d) 50 (c)
51 (b)

Hints and Explanations


1 From universal law of gravitation, 4 3 ∴ u = 10 ms −1 (as, g = 10 ms −2 )
2 ∴ves = w 2GM / R = 2G × πR ρ / R
mv2 Mm 3 1 gx2
= G 5/ 2 Q y = x tan θ −
R R = (8 /3) GπR ρ 2
2 2 v 0 cos2 θ
2

GM Hence, ves ∝ R 1 g × 16
Hence, v2 = 3 / 2 = 4 tan 45 ° −
R ⇒ ves = 2 × 11 = 22 kms −1 2 2 v20 cos2 45 °
2πR 10 × 16
∴ T = u2 sin 2 θ = 4×1 −
1
v 3 As, range = = 10 2 1
Hence, g 2 × 10 × 10 ×
2
4π 2 R2 10 g
⇒ u2 = ⇒ u2 = 10 g = 4 − 0.8 = 3.2
2 7 sin 2 × 45 °
4π R 2 2
GM = 4π R2
T2 = = v 4 ∴ [Pressure × Volume]
v2 R3 /2 GM
N
i.e. T ∝ R
2 7 /2
45º
Wall = × m3 = N-m = J
m2
4m 6m
10 m
DAY TEN UNIT TEST 1 (MECHANICS) 101

J 10 Using equation of motion, v = u + at 16 Time taken to cover n metre is


∴ [Charge × Potential] = ×C = J
C Here, 20 = 10 + a × 5 given by
5 Radius of ball = 5.2 cm This gives a = 2 ms −2 1 2 2n
n= gt n or t n =
4 −1 2 g
∴ V = πR3 and hence v′ = 10 + (2) (−3) = 4 ms
3 x /2 Time taken to cover (n + 1) metre is
 ∆V   ∆R  11 As, t 1 = ; x1 = 4.5 × t 2 ,
  = 3   3 given by
V  R
x2 = 7.5 × t 2 2 (n + 1)
 ∆V   0.2  x tn + 1 =
  × 100 = 3   × 100 = 11% Also, x1 + x2 = = (4 .5 + 7.5) t 2 g
V  5.2 2
x 2 (n + 1) 2n
dK ⇒ t2 = tn + 1 − tn = −
6 Given, = constant ⇒ K ∝ t 24 g g
dt
x x 5 2
⇒ v∝ t t = t1 + t2 = + = x = ( n + 1 − n)
6 24 24 g
dK
Also, p=Fv = = constant v= =
x 24
ms −1 = 4 .8 ms −1
dt This gives ratio as
t 5
1 1 1 ,( 2 − 1 ), ( 3 − 2 ), etc.
⇒ F ∝ ⇒F ∝ 12 From the relation,
v t
a 17 For normal vectors A ⋅B = 0. This is
xn = (2 n − 1)
7 ∴ [K ] = work 2 the case with the vector in option (c).
and [Bx] = [K ] [ x2 + A 2 ] g
(2 × 4 − 1) Given, A = i$ A cos θ + $j A sin θ. From
x4 2 7
[ x2 + A 2] So, = = the options we can choose,
or [B] = [K ] x5 g 9
x (2 × 5 − 1) B = $i B sin θ − $j B cos θ
2
= work × distance ⇒ A ⋅ B = ($i A cos θ + $j A sin θ)
13 Acceleration of the particle is given
8∴ T 2 = p aρb σc as ⋅ (i$ B sin θ − $j B cos θ)
= [ML−1 T−2 ] a[ML−3 ] b [MT−2 ] c a = a2x + a2y = AB sin θ cos θ − AB sin θ cos θ = 0
= [M a + b + c −a − 3b
L T −2 a − 2c
] 2 [Qi$ ⋅ $i = $j ⋅ $j = 1]
d x
Here, ax = =8
Hence, a + b + c = 0, − a − 3 b = 0 dt 2 18 Work done is given by
and − 2a − 2 c = 2 and a y = d 2 y/ dt 2 = 0 W = F⋅r
On solving, we get Substituting F = 4 $i + $j + 3 k$
⇒ a = a2x + a2y = (8)2 + 0 = 8
a = − 3, b = 1 and c = 2 and r = (r2 − r1 )
As, T 2 = p aρb σc 14 From the graph, it is a straight line, = [(14$i + 13 $j + 9 k$ ) − (3 $i + 2 $j − 6 k$ )]
ρσ2 so motion is uniform, because of = 11 i$ + 11 $j + 15 k$ , we get
⇒ T2 = 3 impulse direction of velocity
p W = (4 i$ + $j + 3 k$ ) ⋅ (11 i$ + 11 $j + 15 k$ )
changes as can be seen from the
9 According to the given condition, slope of the graph. = 44 + 11 + 45 = 100 J
2 19 Apparent acceleration,
Initial velocity, v1 = = 1 ms −1
2 a = 9.8 + 1. 2 = 11 ms −2
Boat
Resultant velocity Final velocity, v 2 = −2 / 2 = −1 ms −1 Initial velocity of the car or bolt
velocity
∴ pi = mv1 = 0.4 N - s = 0 + 1.2 × 2 = 2.4 ms −1
and p f = mv2 = −0.4 N - s 1
Using x = ut + at 2
Now, impulse, 2
Stream velocity J = p f − pi = − 0.4 −0.4 As, u = − 2.4 ms−1 , a = 11 ms −2
Resultant velocity of boat = − 0.8 N - s and x = 2.7 m, we get
1 km ⇒ | J| = 0.8 N - s 1
2.7 = − 2.4 × t + × 11 × t 2
= = 4 kmh −1
0.25 h 15 Using the relations, 2
Also, (velocity of river) 2 mv2 ⇒ 5.5 t 2 − 2.4 t − 2.7 = 0
= µR, R = mg
+ (resultant velocity) 2
r ⇒ t~−1s
= (velocity of boat) 2 mv2 20 Here, m1 v1 = m2 v2
⇒ = µmg or v2 = µrg
∴ Velocity of river r 1  1 
or m1 m v2 = m2 m v2
−1 ∴ v2 = 0.6 × 150 × 10  2 1 1   2 2 2 
= (5) − (4) = 3 kmh
2 2

or v = 30 ms −1
102 40 DAYS ~ NEET PHYSICS DAY TEN

1
m1 v12 = µm1 gs1 25 Change in energy = Work done mv2 2 as2
Also, Centripetal force, Fc = =
2 1 F s R R
Hence, m1 v12 = and
1 2 2 ∴ Force acting on the particle is
and m2 v22 = µm2 gs2
2 1  2 as 
2
m2 v22 = F s
F = F t2 + Fc2 = (2 as)2 + 
m2 s2 m1 m12
2  R 
Q = ⇒ s2 = ⋅ s1 m1 v12 1
m1 s1 m2 m22 This gives, = …(i) = 2 as 1 + s2 / R2
m2 v22 2
21 Acceleration without friction, 1
a1 = g sin θ. Also, change in momentum = impulse 30 As, K r = I ω2 . Here, K r = 360 J and
2
i.e. m1 v1 = F × 2 t
Acceleration with friction, ω = 30 rads −1 ⇒ l = 0.80 kg-m2
a2 = g sin θ − µg cos θ and m2 v2 = F × t
2s 31 Instantaneous velocity rising mass
Also, t = This gives,
m1 v1
=2 …(ii) after t second will be
a m2 v2
t1 a2 v t = v2x + v2y
Therefore, = On solving Eqs. (i) and (ii), we get
t2 a1 where, v x = v cos θ = horizontal
v1 1 m
= and 1 = 8 Component of velocity,
2 g sin θ v2 4 m2
Thus, = v y = v sin θ − g t = vertical
1 g sin θ − µ g cos θ A ⋅B Component of velocity,
3 3 26 ∴ cos θ =
This gives, µ = tan θ = × tan 45 ° | A | ⋅ | B|
4 4 v t = (v cos θ)2 + (v sin θ − gt )2
(3 i$ + 4$j + 5 k$ ) ⋅ (3 i$ + 4$j − 5 k$ )
3 3 =
= × 1 = = 0. 75 9 + 16 + 25 9 + 16 + 25 v t = v2 + g 2t 2 − 2 v sin θ g t
4 4
9 + 16 − 25 L
22 The tension pulls m2 and m1 with = =0 32 The centre of mass is at distance
50 2
acceleration, a =
⇒ cos θ = 0 from the axis. Hence, centripetal
T / (m1 + m2 + m3 ) force,
m1 + m2 ∴ θ = 90 °  L
Hence, F = T Fc = M   ω2
m1 + m2 + m3 27 Q | A × B| = 3 A ⋅ B  2

23 When the body slides down the ⇒ AB sin θ = 3 AB cos θ So, the reaction at the other end will
incline, its kinetic energy is given as be equal to Fc .
⇒ tan θ = 3 ⇒ θ = 60 °
1 33 As we know, angular speed,
KE = mv2
2 ∴| A + B | = A 2 + B2 + 2 AB cos 60 ° 2π ω 2 π / T1
ω= ⇒ 1 = .
However, in the case of ring, total T ω2 2 π / T2
1
kinetic energy will be equal to the = A 2 + B2 + 2 AB × Here, T1 = T2 = t
sum of rotational and translational 2
ω1
1 1 = ( A + B + AB)
2 2 1 /2
∴ =1
kinetic energies = Iω2 + mv′2 ω2
2 2 2
d x
1 1 28 ∴ Acceleration = = − 8 + 6t 34 Using ω = ω 0 + at
= mR ω + mv′
2 2 2
[Q I = mR2 ] dt 2
2 2 Here, ω 0 = 900 rpm
d2 x
1
= mv2 + mv′2
1 ∴ W = ∫ Fdx = ∫ m 2 dx = (2 π × 900) / 60 rad/s
dt
2 2 ω = 0 and t = 60 s
d 2 x dx
Since, kinetic energy due to =∫m 2 dt So, ω0 = π / 2
translational will be equal to that dt dt
4 3
35 Bead starts moving when the
due to rotation.
∴W = ∫ (− 8 + 6 t ) centripetal force is equal to the
1 v 0 1000
∴ mv2 = mv′2 or v′ = friction. Hence, friction, f = µm a t .
2 2 (3 − 8 t + 3 t 2 ) dt where, a t is tangential acceleration.
24 In the given case, momentum would On solving, we get It will be given by
be conserved. W = 528 mJ ≈ 530 mJ v
a t = , where v is the linear speed.
Hence, pb = pr or p2b = p2r 1 t
29 Here, K = mv2 = as2 ⇒ mv2 = 2 as2 =Lω
2
Since, p2 = 2MK, we have Lω
On differentiating w.r.t. time t, Hence, f = µm
Mb K b = M r K r dv ds t
M ⇒ 2 mv ⋅ = 4 as = 4 asv Also, centripetal force = m Lω2
i.e. K r = b Kb dt dt
Mr dv µ m Lω
Because, Mb < M r . Hence, K r < K b ⇒ m⋅ = 2 as Hence, mL ω2 =
dv t
DAY TEN UNIT TEST 1 (MECHANICS) 103

µ 43 ∴Velocity at mean position, L2


i.e. t = , also ω = α t 48 As, E =
ω v = 2 gl (1 − cos θ) 2I
This gives, t = µ/α ∴Tension in the string, E2  L 2
2
∴ E ∝ L2 ⇒ = 
36 For balance of moments about G, mv2 E1  L1 
T = mg +
(centre of mass), which is x cm form l 2
E2  L1 + 200% of L1 
free end is m =
= mg + [2 gl (1 − cos 60 ° )] E1  L1 

(50 − x + 5 ) × 0.5 × g = ( x − 25) l
2
×1 × g = mg + mg = 2 mg  L + 2 L1 
= 1  = (3)
2

⇒ x = 35 44 According to the given condition,  L1 


N ⇒ E2 = 9 E1
aR v2R r v2 r 2 R 14N N
37 Q = ⋅ 2 = R2 ⋅ 2 ⋅ 4 kg
2 kg 1kg Increment in kinetic energy,
ar R vr R vr r
∆ E = E2 − E1 = 9 E1 − E1
2
 v   2 nr  R
2
∴ F = ma ⇒ 14 = 7 a ∆ E = 8 E1
= R  ⋅  ⋅ ∆E
 2 nR   v r  r ⇒ a = 2 m/s2 ∴ =8
FBD of 4 kg block, E1
aR TR2 R R
⇒ = ⋅ = [QTR = Tr ] or percentage increases = 800%
ar Tr2 r r 14N
4 kg N 1  K 2 1
2 πr 2 πr 2 πr 3 /2 2 π 49 mv2  2  = 40% mv2
38 ∴T = = = = 2 R  2
v0 2
( gR / r ) 1 /2
gR2 ω 14 = N + ma
K2 40 2
∴Contact force, N = 14 – 4 × 2 = 6 N ⇒ = =
gR 2
gR2
3 R2 100 5
Hence, r 3 /2 = or r 3 = M 4π  R  M
ω ω2 45 We have, M ′ = × = i.e. The body is solid sphere (ball).
4π 3 3  2  8
or r = (gR2 / ω2 ) 1 /3 R 50 Conservation of angular momentum,
3
39 Escape velocity is same for all angles ∴Gravitational field at P I 1 ω1 + I 2 ω2 = ( I 1 + I 2 ) ω
of projection. GM GM M  1 Angular velocity of system,
= 2 − 2 = G × 2 1 − 
40 Velocity of each body will be equal R 8R R  8
I 1 ω1 + I 2 ω2
to the escape velocity w.r.t. the 7 GM ω=
= I1 + I2
other. Hence, v1 = (2 Gm2 / r ) 1 / 2 and 8 R2
∴ Rotational kinetic energy
v2 = (2 Gm1 / r ) 1 /2 . 46 Given, r = 0.4 m, α = 8 rad s −2 , 1
Relative velocity, = ( I 1 + I 2 ) ω2
m = 4 kg, I = ? 2
2G(m1 + m2 ) ∴ Torque, τ = Iα = mgr  I ω + I 2 ω2 
2
v1 − v2 = 1
r ⇒ 4 × 10 × 0.4 = I × 8 = (I 1 + I 2 )  1 1 
2  I1 + I2 
41 Acceleration due to gravity, 16
⇒ I = = 2 kg-m2 (I1 ω1 + I2 ω2 )2
ge =
GMe
, gm =
GMe / 80 8 =
2(I1 + I2 )
R2e (Re / 4) 2
47 Weight of the rod will produce the
g 2 gh
∴ gm = e
torque. 51 We know that, v =
5 K2
l 1+
42 The maximum velocity for a banked r2
road with friction, v 2 gh
l/2 ∴ ω= =
 µ + tan θ  r r2 + K 2
v2 = gr   mg
 1 − µ tan θ  2 mgh 2 mgh
τ = Iα ⇒ ω= =
 0.5 + 1  mr 2 + mK 2 mr 2 + I
⇒ v = 9.8 × 1000 × 
2
 l ml2
 1 − 0.5 × 1  ⇒ mg × = ×α
2 3 2 mgh
=
⇒ v = 172 ms −1 3g I + mr 2
Angular acceleration, α =
2l
DAY ELEVEN

Properties
of Matter
Learning & Revision for the Day
u Elastic Behaviour u Thermal Stress and Strain u Surface Tension
u Stress u Strain u Viscosity u Surface Energy
u Hooke’s Law u Streamline and Turbulent Flow u Angle of Contact
u Work Done (or Potential u Equation of Continuity u Capillary Rise or Capillarity
Energy) in a Stretched Wire u Bernoulli’s Theorem

Elastic Behaviour
Elasticity is the property of body by virtue of which a body regains or tends to regain its
original configuration (shape as well as size), when the external deforming forces acting
on it, is removed.

Stress
The internal restoring force per unit area of cross-section of the deformed body is called
stress.
Thus Stress, σ=
Restoring force F
=
PREP
Area A MIRROR
SI unit of stress is Nm −2 or pascal (Pa). Your Personal Preparation Indicator

Different types of stresses are given below u No. of Questions in Exercises (x)—
u No. of Questions Attempted (y)—
1. Normal or Longitudinal Stress u No. of Correct Questions (z)—
If area of cross-section of a rod is A and a deforming force F is applied along the length (Without referring Explanations)
of the rod and perpendicular to its cross-section, then in this case, stress produced in
the rod is known as normal or longitudinal stress. u Accuracy Level (z / y × 100)—
F u Prep Level (z / x × 100)—
Longitudinal stress = n
A
In order to expect good rank in NEET, your
Accuracy Level should be above 85 & Prep
Level should be above 75.
DAY ELEVEN PROPERTIES OF MATTER 105

Longitudinal stress is of two types


Stress-Strain Relationship
(i) Tensile stress When length of the rod is increased on
For a solid, the graph between stress (either tensile or
application of deforming force over it, then stress
compressive) and normal strain is shown in figure.
produced in rod is called tensile stress.
Plastic region
(ii) Compressive stress When length of the rod is Breaking D
decreased on application of deforming force, then the strength C
E
stress produced is called compressive stress. B (Fracture point)
Elastic limit
A
Proportional limit
2. Volumetric Stress
When a force is applied on a body such that it produces a Stress
change in volume and density, shape remaining same O' Strain

(i) at any point, the force is perpendicular to its surface. l


In the above graph, point A is called proportional limit.
(ii) at any small area, the magnitude of force is directly Till this point, stress and strain are proportional to each
proportional to its area. other.
Then, force per unit area is called volumetric stress.
l
From point A to B, stress and strain are not proportional, B
F is called elastic limit and OB is elastic region.
∴ Volumetric stress = v l
Beyond point B, strain increases without increase in stress,
A
it is called plastic behaviour. Region between point C and
3. Shearing or Tangential Stress D is called plastic region.
When the force is applied tangentially to a surface, then it is l
Finally, at point D, wire may break, maximum stress
called tangential or shearing stress. corresponding to point D is called breaking stress.
F The materials of the wire, which break as soon as stress is
Tangential stress = t
A increased beyond the elastic limit are called brittle.
Graphically, for such materials the portion of graph between
It produces a change in shape, volume remaining same.
B and E is almost zero. While the materials of the wire, which
have a good plastic range (portion between B and E) are called
F
ductile.

Hooke’s Law
According to the Hooke’s law, for any body, within the elastic
Fixed face limit, stress developed is directly proportional to the strain
produced.
Stress ∝ Strain
Strain Stress = E × Strain
Strain is the ratio of change in configuration to the original The ratio of stress to strain, within the elastic limit, is called
configuration of the body. the coefficient (or modulus) of elasticity for the given
Being the ratio of two similar quantities, strain is a unitless material.
and dimensionless quantity. Depending on the type of stress applied and resulting strain,
(i) When the deforming force causes a change in length, it we have the following three of elasticity given as,
is called longitudinal strain. For a wire or rod, Stress
longitudinal strain is defined as the ratio of change in E =
Strain
length to the original length.
Change in length (∆L) There are three modulus of elasticity.
∴ Longitudinal strain =
Original length (L) 1. Young’s Modulus
(ii) When the deforming force causes a change in volume, Young’s modulus of elasticity (Y ) is defined as the ratio of
the strain is called volumetric strain. normal stress (either tensile or compressive stress) to the
Change in volume (∆V ) longitudinal strain within a elastic limit.
Volumetric strain =
Original volume (V ) Young’s modulus,
(iii) When the deforming force, applied tangentially to a Normal stress
Y =
surface, produces a change in shape of the body, the Longitudinal strain
strain developed is called shearing strain or shear. F/A FL
x = =
Shearing strain, φ = ∆ L / L A∆ L
L
106 40 DAYS ~ NEET PHYSICS DAY ELEVEN

= Energy stored in the wire (U)


2. Bulk Modulus
l
Energy stored per unit volume (or energy density)
It is defined as the ratio of the normal stress to the volumetric
U 1 F ∆L 1
strain. Coefficient of volume elasticity, = = = × stress × strain
F/A pV V 2 AL 2
B=− =− 1 Y
∆V / V ∆V = (stress)2 = (strain)2
2Y 2
F
where, p = = the pressure or stress negative sign signifies
A
that for an increase in pressure, the volume will decrease. Thermal Stress
Reciprocal of bulk modulus is called compressibility and is When a body is allowed to expand or contract with increasing
denoted by C. temperature or decreasing temperature, no stresses are
induced in the body.
3. Modulus of Rigidity (Shear modulus) But if the deformation of the body is prevented, some stresses
It is defined as the ratio of tangential stress to shearing stress. are induced in the body. Such stresses are called thermal
Tangential stress F / A F FL stresses or temperature stresses.
η= = = =
Shearing strain φ Aφ Ax The corresponding strains are called thermal strains or
temperature strains.
NOTE Breaking force depends upon the area of cross-section of
the wire. l, a , Y, A
B A
∴ Breaking force ∝ A ∆l
Breaking force = P × A
Here, P is a constant of proportionality and knowing as ∆l
By definition, coefficient of linear expansion α =
breaking stress. lθ
∆l
thermal strain = α ∆θ
Poisson’s Ratio l
For a long bar, the Poisson’s ratio is defined as the ratio of So thermal stress = Yα ∆θ
lateral strain to longitudinal strain. Tensile or compressive force produced in the body
Lateral strain ∆ D/ D ∆ r / r F = YAα ∆θ
∴ Poisson’s ratio, σ = = =
Longitudinal strain ∆ L / L ∆ L / L
Poisson’s ratio is a unitless and dimensionless term. Its value
depends on the nature of the material. Theoretically, value of
Viscosity
σ must lie between −1 and + 0.5 but for most metallic solids Viscosity is the property of a fluid due to which it opposes
0 < σ < 0.5. the relative motion between its different layers.
Force between the layers opposing the relative motion is
Inter-Relations Between Elastic Constants called viscous force.
dv
Y = Young’s modulus, η = Rigidity modulus, F = −ηA .
dr
B = Bulk modulus, σ = Poisson’s ratio
Here, the constant η is called the coefficient of viscosity of the
The inter relation between elastic constants are given fluid. SI unit of coefficient of viscosity is Nsm–2 or Pa-s.
Y = 2 η (1 + σ), Y = 3 B (1 − 2 σ)
9 3 1 9 Bη 3B − 2 η
= + or Y = or σ =
Y η B η + 3B 6B + 2 η Terminal Velocity
If a small spherical body is dropped in a fluid, then initially it
is accelerated under the action of gravity. However, with an
Work Done (or Potential Energy) in increase in speed, the viscous force increases and soon it
a Stretched Wire balances the weight of the body.
Work is done against the internal restoring forces, while Now, the body moves with a constant velocity, called the
stretching a wire. This work is stored as elastic potential terminal velocity. Terminal velocity vt is given by
energy. The work done is given by 2 r 2 (ρ − σ)g
vt =
1 9 η
l
Work done, W = × stretching force × elongation
2 where, r = radius of the falling body,
1 1 YA ρ = density of the substance of falling body
= F∆ L = (∆ L)2
2 2 L and σ = density of the fluid.
DAY ELEVEN PROPERTIES OF MATTER 107

Stokes’ Law Energy of a Flowing Liquid


Stokes proved that for a small spherical body of radius r There are three types of energies in a flowing liquid.
moving with a constant speed v called terminal velocity l
Pressure Energy If p is the pressure on the area A of a
through a fluid having coefficient of viscosity η, the viscous
fluid, and the liquid moves through a distance l due to this
force F is given by F = 6πηrv
pressure, then
It is known as the Stokes’ law. Pressure energy of liquid = work done
= force × displacement = pAl
Streamline and Turbulent Flow The volume of the liquid is Al.
Flow of a fluid is said to be streamlined, if each element of pAl
Hence, pressure energy per unit volume of liquid = =p
the fluid passing through a particular point travels along the Al
same path, with exactly the same velocity as that of the l
Kinetic Energy If a liquid of mass m and volume V is
preceding element. A special case of streamline flow is 1
laminar flow. flowing with velocity v, then the kinetic energy = mv2
2
A turbulent flow is the one in which the motion of the fluid 1  m 1
particles is disordered or irregular. ∴ Kinetic energy per unit volume of liquid =   v2 = ρv2
2 V  2
Here, ρ is the density of liquid.
Critical Velocity l
Potential Energy If a liquid of mass m is at a height h
For a fluid, the critical velocity is that limiting velocity of the from the reference line (h = 0), then its potential energy is
fluid flow upto which the flow is streamlined and beyond mgh.
which the flow becomes turbulent.
∴ Potential energy per unit volume of the liquid
Value of critical velocity for the flow of liquid of density ρ
and coefficient of viscosity η, flowing through a horizontal  m
=   gh = ρgh
η V 
tube of radius r is given by vc ∝ .
ρr

Reynolds’ Number (NR ) Bernoulli’s Theorem


Reynold’s Number as the ratio of the inertial force per unit According to the Bernoulli’s theorem for steady flow of an
area to the viscous force per unit area for a fluid. incompressible, non-viscous fluid through a tube/pipe, the
total energy (i.e. the sum of kinetic energy, potential energy
v2 ρ ρvr
NR = = and pressure energy) per unit volume (or per unit mass too)
ηv / r η
remains constant at all points of flow provided that there is
A smaller value of Reynolds’ number (generally N R ≤ 1000) no source or sink of the fluid along the flow.
indicates a streamline flow but a higher value (N R ≥ 1500) 1
indicates that the flow is turbulent and between 1000 to 1500, Mathematically, we have p + ρgh + ρv2 = constant
2
the flow is unstable.
p v2
Dividing this equation by ρg, we have + + h = constant
ρg 2 g
Equation of Continuity
Let us consider the streamline flow of an ideal, non-viscous v2 p
In this expression, is velocity head and is pressure head.
fluid through a tube of variable cross-section. 2g ρg
Let at the two sections, the cross-sectional areas be
A1 and A2 , respectively and the fluid flow velocities are
v1 and v2 , then according to the equation of continuity
Velocity of Efflux
l
If a liquid is filled in a vessel up to a height H and a small
A1v1 ρ 1 = A 2 v2 ρ2
orifice O is made at a height h, then from Bernoulli’s theorem
If the fluid which is flowing is incompressible, then ρ 1 = ρ2 . it can be shown that velocity of efflux v of the liquid from the
So, equation of continuity is simplified as vessel is v = 2 g(H − h).
A1v1 = A2 v2
Q
l
The flowing fluid describes a parabolic path and hits the
P base level at a horizontal distance (called the range)
v2 R = 2 h(H − h).
v1
A2 H
The range is maximum, when h = and in that case
2
A1 Rmax = H .
108 40 DAYS ~ NEET PHYSICS DAY ELEVEN

Value of the angle of contact depends on the nature of


Surface Tension
l

liquid and solid both.


Surface tension is the property of a liquid due to which its free l
For a liquid having concave meniscus, angle of contact θ is
surface behaves like a stretched elastic membrane and tends to
acute (θ < 90 ° ) but for a convex meniscus, the angle of
have the least possible surface area.
contact is obtuse (θ > 90 ° ).
F
Surface tension, S = l
Value of angle of contact θ decreases with an increase in
l
temperature.
Here, F is force acting on the unit length of an imaginary line
drawn of the surface of liquid.
SI unit of surface tension is Nm−1 or Jm−2 . It is a scalar and its
Excess Pressure Over a Liquid Film
If a free liquid surface film is plane, then pressure on the
dimensional formula is [MT−2 ].
liquid and the vapour sides of the film are the same,
otherwise there is always some pressure difference. Following
Surface Energy cases arise.
Surface energy of a liquid is the potential energy of the l
For a spherical liquid drop of radius r, the excess pressure
molecules of a surface film of the liquid by virtue of its 2S
inside the drop p =
position.When the surface area of a liquid is increased, work r
is done against the cohesive force of molecules and this work
where, S = surface tension of the liquid.
is stored in the form of additional surface energy. Increase in
2S
surface potential energy l
For an air bubble in a liquid, excess pressure p =
r
∆U = Work done (∆W ) = S∆ A
4S
where, ∆ A is the increase in surface area of the liquid. l
For a soap bubble in air, excess pressure p =
r
l
Work done in Blowing a Liquid Drop If a liquid drop is
blown up from a radius r1 to r2 , then work done in the
process,
Capillary Rise or Capillarity
W = S ( A2 − A1) = S × 4π(r22 − r12 ) Capillarity is the phenomenon of
rise or fall of a liquid in a capillary
(drop has only one free surface) tube as compared to that in a
l
Work done in Blowing a Soap Bubble As a soap bubble surrounding liquid. h α
l
has two free surfaces, hence, work done in blowing a soap
The height h up to which a liquid
bubble so as to increase its radius from r1 to r2 , is given by
will rise in a capillary tube is given
W = S × 8 π(r22 − r12 ) (Bubble has two free surfaces) 2 S cos θ 2 S
by h = =
l
Work done in Splitting a Bigger Drop into n Smaller rρg r ρg
Droplets If a liquid drop of radius R is split up into n
smaller droplets, all of the same size, then radius of each where, r = radius of the capillary
r
droplet tube and R = = radius of liquid
cosθ
r = R(n)−1/ 3 meniscus.
1
and work done
l
The rise in capillary tube, h ∝ ⋅ (Jurin law)
r
W = S × 4π(nr 2 − R2 ) = S × 4πR2 (n1/ 3 − 1) l
If a capillary tube, dipped in a liquid is tilted at an angle α
l
Coalescence of Drops If n small liquid drops of radius r from the vertical, the vertical height h of the liquid column
each, combine together so as to form a single bigger drop of remains the same. However, the length of the liquid
radius R = n1/ 3 r , then in the process, energy is released. column (l ) in the capillary tube increases to l =
h
.
Release of energy is given by cos α
∆U = S × 4π (nr 2 − R2 ) = S × 4πr 2 n (1 − n−1/ 3 ) l
If the capillary tube is of insufficient length, the liquid rises
up to the upper end of the tube and then the radius of its
meniscus changes from R to R′ such that hR = h′ R′ , where,
Angle of Contact h′ = insufficient length of the tube.
Angle of contact for a given l
After connection due to the weight of liquid contained in
liquid-solid combination is defined θ
θ the meniscus, the formula for the height is given by
as the angle subtended between
2S r
the tangents to the liquid surface h= −
and the solid surface, inside the ρrg 3
liquid, the tangents are drawn at the point of contact. This is known as ascent formula.
DAY PRACTICE SESSION 1

FOUNDATION QUESTIONS EXERCISE


. × 108 N/m 2 is applied to a steel rod of
1 A stress of 318 9 The breaking stress for a copper wire is 2.2 × 10 8 Nm −2 .
length 1m along its length. Its young’s modulus is 2 × 10 11
The maximum length of the copper wire which when
N/m 2 , then the elongation produced in the rod in mm is suspended vertically for which the wire will not break
(a) 3.18 (b) 6.38 (c) 5.18 (d) 1.59 under its own weight, will be (Density of copper
2 A wire is stretched through 1 mm by certain load. The = 8.8 × 10 3 kgm −3 )
extension produced in the wire of same material with (a) 25000 m (b) 2500 m (c) 250 m (d) 25 m
double the length and double the radius will be 10 A load of 25 kg-wt is applied on a wire of diameter
(a) 4 mm (b) 3 mm (c) 1 mm (d) 0.5 mm 0.4 cm due to which its length increases from 100 cm to
3 The volume of water changes from 100 L to 99.5 L under 102 cm, then the Young’s modulus of wire will be
a pressure of 100 atmosphere. The bulk modulus of (a) 9.75 × 109 dyne cm−2 (b) 9.75 × 109 N m−2
elasticity of water will be (c) 5.79 × 108 dyne cm−2 (d) 5.79 × 108 N m−2
(a) 1.013 × 105 Nm−2 (b) 1.013 × 109 Nm−2 11 A 10 m long thick rubber pipe is suspended from one of
(c) 2.026 × 105 Nm−2 (d) 2.026 × 109 Nm−2 its ends. The extension produced in the pipe under its
−2
4 A copper wire (Y = 1 × 10 Nm ) of length 6 m and a
11 own weight will be (Y = 5 × 106 N m −2 and density of
steel wire (Y = 2 × 1011 Nm −2 ) of length 4 m each of rubber = 1500 kg m −3 )
cross-section10−5 m 2 are fastened end to end and (a) 1.5 m (b) 0.15 m (c) 0.015 m (d) 0.0015 m
stretched by a tension of 100 N. The elongation 12 A rigid bar of mass M is supported symmetrically by
produced in the copper wire is three wires each of length l. Those at each end are of
(a) 0.2 mm (b) 0.4 mm (c) 0.6 mm (d) 0.8 mm copper and the middle one is of iron. The ratio of their
5 A body of mass 1 kg is fastened to one end of a steel diameters, if each is to have the same tension is equal to
wire of cross-sectional area 3 × 10−6 m 2 and is rotated j
NCERT Exemplar
in horizontal circle of radius 20 cm with constant Yiron Y 2iron Yiron
(a)Ycopper / Yiron (b) (c) 2 (d)
speed 2 m/s. The elongation of the wire is (Y = 2 × 1011 Ycopper Y copper Ycopper
N/m 2 )
(a) 0.33 × 10−5 m (b) 0.67 × 10−5 m
13 The correct graph verifying Hooke’s law is
(c) 2 × 10−5 m (d) 4 × 10−5 m Extension

6 The approximate depth of an ocean is 2700 m. The


Extension

compressibility of water is 45.4 × 10−11 Pa −1 and density (a) (b)


of water is 103 kg/m 3 . What fractional compression of
water will be obtained at the bottom of the ocean? Load
Load
j
AIPMT 2015
(a) 0.8 × 10−2 (b) 1.0 × 10−2 (c) 1.2 × 10−2 (d) 1.4 × 10−2
Extension
Extension

7 A copper rod of length L and radius r is suspended from (c) (d)


the ceiling by one of its ends. What will be elongation of
the rod due to its own weight when ρ and Y are the Load
Load
density and Young’s modulus of the copper,
respectively? 14 The work done in increasing the length of a wire of area
ρ gL
2 2
ρgL
2
ρg L
2 2 2
ρgL of cross-section 0.1 mm 2 by 1% will be (Y = 9 × 1011 Pa)
(a) (b) (c) (d)
2Y 2Y 2Y 2Y (a) 2 × 10 2 J (b) 4.5 × 10 2 J (c) 3 × 10 2 J (d) 6 × 10 2 J
8 The following four wires are made of the same material. 15 The length of a wire increases by l due to a force F
Which of these will the largest extension when the same applied on it. Then, the work doneW in stretching the
tension is applied? j
NEET 2013 wire will be
(a) Length = 50 cm, diameter = 0.5 mm F ⋅l
(a)W = F ⋅ l (b)W =
(b) Length = 100 cm, diameter = 1mm 4
(c) Length = 200 cm, diameter = 2 mm F ⋅l F ⋅l
(c)W = (d)W =
(d) Length = 300 cm, diameter = 3 mm 2 3
110 40 DAYS ~ NEET PHYSICS DAY ELEVEN

16 Two wires are made of the same material and have the 25 A rectangular film of liquid is extended from (4 cm × 2 cm)
same volume. The first wire has cross-sectional area A to (5 cm × 4 cm). If the workdone is 3 × 10−4 J. The value
and the second wire has cross-sectional area 3A. If the of the surface tension of the liquid is j NEET 2016

length of the first wire is increased by ∆l on applying a (a) 0.250 N/m (b) 0.125 N/m (c) 0.2 N/m (d) 8.0 N/m
force F , how much force is needed to stretch the second
26 A soap bubble of radius r is blown up to form a bubble of
wire by the same amount? j NEET 2018

radius 2r under isothermal conditions. IfT is the surface


(a) 4F (b) 6F (c) 9F (d) F
tension of soap solution, the energy spent in the blowing is
17 A good lubricant should have
(a) 3 πTr 2 (b) 6 πTr 2 (c) 12 πTr 2 (d) 24 πTr 2
(a) high viscosity (b) low viscosity
(c) moderate viscosity (d) high density 27 8 mercury drops coalesce to form one mercury drop, the
18 In which one of the following cases will the liquid flow in a energy changes by a factor of
pipe be most streamlined? (a) 1 (b) 2 (c) 4 (d) 6
(a) Liquid of high viscosity and high density flowing through 28 The lower end of a capillary tube of diameter 2.00 mm is
a pipe of small radius dipped 8.00 cm below the surface of water in a beaker.
(b) Liquid of high viscosity and low density flowing through What is the pressure required in the tube in order to blow
a pipe of small radius a hemispherical bubble at its end in water?
(c) Liquid of low viscosity and low density flowing through a
(a) 2 × 105 Pa (b) 1.01784 × 105 Pa
pipe of large radius
(c) 3 × 10 3 Pa (d) 2.438 × 105 Pa
(d) Liquid of low viscosity and high density flowing through
a pipe of large radius 29 The angle of contact between glass and water is 0° and
19 Two water pipes of diameters 2 cm and 4 cm are it rises in a capillary upto 6 cm when its surface tension
connected with the main supply line. The velocity of flow is 70 dyne cm −1. Another liquid of surface tension
of water in the pipe of 2 cm diameter is 140 dyne cm −1, angle of contact 60° and relative density
(a)4 times that in the other pipe 2 will rise in the same capillary by
1
(b) times that in the other pipe (a) 12 cm (b) 24 cm (c) 3 cm (d) 6 cm
4
(c)2 times that in the other pipe 30 A frame made of a metallic wire enclosing a surface area
1 A is converted with a soap film. If the area of the frame of
(d) times that in the other pipe
2 metallic wire is reduced by 50%, the energy of the soap
20 Aerofoils are so designed that the speed of air film wire be changed by
(a)on top side is more than on lower side (a) 100% (b) 78% (c) 50% (d) 25%
(b)on top side is less than on lower side
(c)is same on both sides (d) is turbulent 31 A certain number of spherical drops of a liquid of radius r
coalescence to form a single drop of radius R and
21 A wind with speed 40 m/s blows parallel to the roof of a volume V. If T is the surface tension of the liquid, then
house. The area of the roof is 250 m 2 . Assuming that the
(a) energy = 4VT  −  is released
1 1
pressure inside the house is atmospheric pressure, the
j
AIPMT 2014
r R 
force exerted by the wind on the roof and the direction of
(b) energy = 3VT  +  is absorbed
1 1
the force will be (ρ air = 1.2 kg/m 3 ) j
AIPMT 2015 r R 
(a) 4.8 × 105 N, downwards (b) 4.8 × 105 N, upwards
(c) energy = 3VT  −  is released
1 1
(c) 2.4 × 105 N, upwards (d) 2.4 × 105 N, downwards
r R 
22 A cylinder of height 20 m is completely filled with water. (d) energy is neither released nor absorbed
The velocity of efflux of water (in ms −1) through a small
hole on the side wall of the cylinder near its bottom is 32 A liquid rises in a copillary tube if the angle of contact is
(a) 10 (b) 20 (c) 25.5 (d) 5 (a) acute (b) obtuse (c) πl 2 (d) π

23 A small sphere of radius r falls from rest in a viscous 33 The wettability of surface by a liquid depends
liquid. As a result, heat is produced due to viscous force. primarily on j
NEET 2013
The rate of production of heat when the sphere attains its (a) viscosity (b) surface tension (c) density
terminal velocity, is proportional to j
NEET 2018 (d) angle of contact between the surface and the liquid
(a) r 5 (b) r 2 (c) r 3 (d) r 4 34 A capillary tube of radius 0.25 mm is submerged
vertically in water, so that 25 mm of its length is outside
24 A 10 cm long wire is placed horizontaly on the surface of
−2 water. The radius of curvature of the meniscus will be
water and gently pulled up with a force of 2 × 10 N to
(ST of water = 75 × 10−3 Nm −1)
keep the wire in equilibrium. The surface tension in N/m.
of water (a) 0.2 mm (b) 0.4 mm
(c) 0.6 mm (d) 0.8 mm
(a) 0.1 (b) 0.2 (c) 0.001 (d) 0.002
DAY ELEVEN PROPERTIES OF MATTER 111

DAY PRACTICE SESSION 2

PROGRESSIVE QUESTIONS EXERCISE


1 The bulk modulus of a sphertical object is B. If it is increased by 10%, then the rate of flow of liquid will
subjected to uniform pressure P, the practical decrease change nearly by
in radius is j NEET 2017
(a) + 10% (b) + 46% (c) − 10% (d) − 40%
P B 3P P
(a) (b) (c) (d) 8 Two equal drops of water are falling through air with a
B 3P B 3B
steady velocity v. If the drops coalescenced, what will be
2 A cylindrical vessel of radius r containing a liquid is the new velocity?
rotating about a vertical axis through the centre of 1 3 2 1

circular base. If the vessel is rotating with angular (a) (2) 3 v (b) (2) 2 v (c) (2) 3 v (d) (2) 4 v
velocity ω, then what is difference of the heights of liquid 9 In drops of a liquid, each with surface energy E , join to
at centre of vessel and edge form a single drop. In this process
rω r 2ω2 ω2 (a)some energy will be absorbed
(a) (b) (c) 2grω (d)
2g 2g 2gr 2 (b)energy absorbed is E (n − n 2 / 3 )
(c)energy released will be E (n − n 2 / 3 )
3 Three liquids of densities ρ1, ρ 2 and ρ 3 (with ρ1 > ρ 2 > ρ 3 ) (d)energy released will be E (2 2 / 3 − 1)
having the same value of surface tensionT , rise to the
same height in three identical capillaries. The angle of 10 A small uniform tube is bent into a circle of radius r
contact θ1, θ 2 and θ 3 obey. j
NEET 2016
whose plane is vertical. The equal volumes of two fluids
π π whose densities are ρ and σ (ρ > σ ), fill half the circle.
(a) > θ1 > θ2 > θ3 > 0 (b) θ ≤ θ1 < θ2 < θ3 < Find the angle that the radius passing through the
2 2
π π interface makes with the vertical where : OO′ is line
(c) < θ1 < θ2 < θ3 < π (d) π > θ1 > θ2 >
2 2 passing at fluid interface
4 A liquid of density ρ 0 is filled in a wide tank to a height h.
A solid road of length L, cross-section area A and density
ρ is suspended freely in the tank. The lower end of the
O
road touches the base of the tank and h = L (where,
η θ σVg
η > 1). Then, the angle of inclination of the rod with the O′
horizontal in the equilibrium position is ρVg

1 ρ− σ ρ− σ
1 ρ0  ρ (a) cotθ = (b) tanθ =
(a) θ = sin−1   (b) θ = sin−1   ρ+ σ ρ+ σ
η ρ η ρ0 
ρ+ σ ρ
(c) sin θ = (d) sinθ =
 ρ   ρ  ρ− σ σ
(c) θ = sin−1  η 0  (d) θ = sin−1  0 
 ρ  ρ 11 A force of 200 N is applied at one end of a wire of length
5 A solid cylindrical rod of radius 3 mm gets depressed 2 m and having area of cross-section10−2 cm 2 . The
under the influence of a load through 8 mm. The other end of the wire is rigidly fixed. If coefficient of linear
depression produced in an identical hollow rod with outer expansion of the wire α = 8 × 10−6/C and Young’s
and inner radii of 4 mm and 2 mm respectively, will be modulus Y = 2.2 × 1011 N/m 2 and its temperature is
(a) 2.7 mm (b) 1.9 mm (c) 3.2 mm (d) 7.7 mm increased by 5°C, then the increase in the tension of the
wire will be
6 If the shear modulus of a wire material is
(a) 4.2 N (b) 4.4 N (c) 2.4 N (d) 8.8 N
5.9 × 1011 dyne/cm 2 , then the potential energy of a wire
of 4 × 10−3 cm in diameter and 5 cm long twisted through 12 The average depth of Indian ocean is about 3000 m. The
an angle of 10°, is ∆v
fractional compression. of water at the bottom of the
V
(a) 1.253 × 10−12 J (b) 2 × 10−12 J
(c) 1 × 10−12 J (d) 0.8 × 10−12 J ocean (given that the bulk modulus of the water
= 2.2 × 109N/m 2 and g = 10 m/s 2 ) is
7 A capillary tube is attached horizontally to a constant
heat arrangement. If the radius of the capillary tube is (a) 0.82% (b) 0.91% (c) 1.36% (d) 1.24%
112 40 DAYS ~ NEET PHYSICS DAY ELEVEN

13 The cylindrical tube of a spray pump has a cross-section from the mid-point of the wire, strain in the wire is
of 8 cm 2 , one end of which has 40 fine holes each of j NCERT Exemplar
area 10−8 m 2 . If the liquid flow inside the tube with a 2L
speed of 0.15 m/min, the speed with which the liquid is x
ejected through the holes is
m
(a) 50 m/s (b) 5 m/s (c) 0.05 m/s (d) 0.5 m/s
2
x x
14 A raindrop with radius 1.5 mm falls a cloud at a height (a) (b)
2L2 L
1200 m from ground. The density of water is 1000 kg/m 3
x2 x2
and density of air is 1.2 kg/m 3 . (c) (d)
L 2L
Assume the drop was spherical throughout the fall and 16 A body of mass m = 10 kg is attached to a wire of length
there is no air dry, the impact speed of the drop will be 0.3 m. The maximum angular velocity with which it can
(a) 27 km/h (b) 151.5 km/h be rotated in a horizontal circle is
(c) zero (d) 129 km/h (Breaking stress of wire = 4.8 × 107 N/m 2 and
15 A mild steel wire of length 2L and cross-sectional area A area of cross-section of wire = 10−6 m 2 )
is stretched, well within elastic limit, horizontally between (a) 4 rad/s (b) 8 rad/s
two pillars as shown in figure. A mass m in suspended (c) 1 rad/s (d) 2 rad/s

ANSWERS
SESSION 1 1 (d) 2 (d) 3 (d) 4 (c) 5 (b) 6 (c) 7 (b) 8 (a) 9 (b) 10 (a)
11 (b) 12 (b) 13 (c) 14 (b) 15 (c) 16 (c) 17 (a) 18 (b) 19 (a) 20 (a)
21 (c) 22 (b) 23 (a) 24 (a) 25 (b) 26 (d) 27 (c) 28 (b) 29 (c) 30 (c)
31 (c) 32 (b) 33 (d) 34 (c)
SESSION 2 1 (d) 2 (b) 3 (b) 4 (a) 5 (a) 6 (a) 7 (b) 8 (c) 9 (c) 10 (b)
11 (d) 12 (c) 13 (b) 14 (b) 15 (a) 16 (a)

Hints and Explanations


1 Y =
F/A
4 L Cu =
F L Cu 20 × 20 × 10−2
∴ ∆l =
∆l / l YCu A 3 × 10−6 × 2 × 1011
F 100 × 6
= stress = 318
. × 108 N/m2 = = 0.6 mm 2
= × 10−5 m
A 1011 × 10−5 3
lF / A 318. × 108
∆l = = 2
1 × 22 = 0.67 × 10−5 m
Y 2 × 1011 5 T = mv = −2
= 1.59 × 10−3 m = 1. 59 mm r 20 × 10 6 Pressure at the bottom,
MgL =
4
=
40
= 20 N p = ρ gd
2 Y = 0.2 2 = 103 × 10 × 2700
πr l 2
T 20
2 Stress = = = 27 × 106 Pa
L l1  L1   r2  A 3 × 10−6
∴ l∝ ⇒ =  ×  ∴ Fractional compression
r2 l2  L2   r1  T = Compressibility × Pressure
= 45.4 × 10−11 Pa −1 × 27 × 106 Pa
2
l1  L   2r 
=  × 
l2  2 L   r  stress 20 = 1.2 × 10−2
∴ Strain = =
∴l2 = 0.5mm Y 3 × 10−6 × 2 × 1011
7 The weight of the rod can be assumed to
V∆ p ∆l 20
3 K =− ⇒ = act at its mid-point.
∆V l 3 × 10−6 × 2 × 1011 Now, the mass of the rod is
100 × 1.013 × 105 ∴ ∆l =
20
×l M = Vρ ⇒ M = ALρ …(i)
= 100 ×
(100 − 99.5) 3 × 10−6 × 2 × 1011 where, A = Area of cross-section
−2 Given, l = 20 × 10−2 m L = Length of the rod
= 2.026 × 10 Nm 9
DAY ELEVEN PROPERTIES OF MATTER 113

Now, we know that the Young’s 16 According to question 20 The aerofoils are so designed that,
modulus For wire 1 p top side < p lower side
MgL Area of cross section = A1
so that the aerofoils get a lifting force in
Y = 2 Force applied = F1
A⋅ l upward direction.
Increase in length = ∆l
L From the relation of Young’s modulus of According to Bernoulli’s theorem, where
[here, L = , l = extension] the pressure is large, the velocity will be
2 elasticity
MgL Fl minimum or vice-versa.
Y =
⇒ l= 2 A∆ l Thus, v top side > v lower side
AY Substituting the value for wire 1 in the
MgL
21 By Bernoulli’s theorem,
or l= above relation, 1 1
2 AY F l p1 + pv 12 = p2 + ρv 22 [assuming that
we get Y1 = 1 1 14243 142243
2
On putting the value of M from Eq. (i), A1 ∆l
inside outside
we get For wire 2 root width is very small]
ALρ ⋅ gL ρgL2 Area of cross-section = A2
l= or l = . Pressure difference,
2 AY 2Y Force applied = F2 1
p1 − p2 = ρ(v 22 − v 12 )
Increase in length = ∆l
FL L  πd 2  2
8 ∆L = or ∆L ∝ Q A =  F l
AY d2 4  Similarly Y2 = 2 2 1
p1 − p2 = × 1.2(402 − 02 )
 A2 ∆l 2
Therefore, ∆L will be maximum for that Volume, V = Al 1
L = × 1.2 ×1600
wire which is maximum. V
l= 2
A A = 960 N/m2
Mg ALδg Substituting the value of l in Eqs (i) and
9 Breaking stress, S = = Force acting on the roof,
A A (ii), we get
f = 960 × 250 = 24 × 104
FV FV
S 2.2 × 108 Y1 = 1 and Y2 = 2
∴ L = = = 2500 m A1 ∆l
2
A22 ∆l = 2.4 × 105 N (upwards)
ρg 8.8 × 104
As it is given that the wires are made up 22 v = 2gh = 2 × 10 × 20 = 20 ms −1
MgL
10 Y = of same material
πr 2 l Y1 = Y2 23 The rate of heat generation is equal to
25000 × 980 × 100 F1V FV the rate of work done by the viscous
Y = = 2
3.14 × (0.2)2 × 2 A12 ∆l A22 ∆l force which in turn is equal to its power.
= 9.75 × 109 dyne cm −2 F1 A2 A2 1 Rate of heat produced,
dQ
= F × νT
= 1 = = dt
ALρg  
2
L F2 A2 9 A2 9
MgL  2 L2ρ g F2 = 9F1 = 9F
where, F is the viscous force and v T is
11 l = = = the terminal velocity.
AY AY 2Y
17 A good lubricant generally possesses the As, F = 6πηr ν T
100 × 1500 × 10 following characteristics: dQ
= = 0.15 m ⇒ = 6πηr ν T × ν T = 6 πηr ν2T …(i)
2 × 5 × 106 (i) A high viscosity index dt
4 Fl (ii) Thermal stability From the relation for terminal velocity,
12 Y=
π D 2 ∆l (iii) Hydraulic stability 2 r 2 (ρ − σ )
νT = g , we get
4Fl 1 (iv) A high boiling point and low 9 η
∴ D = or D ∝
π. ∆lY Y freezing point νT ∝ r 2 …(ii)
Dcopper Yiron 18 For streamline flow, Reynold’s number From Eq. (ii), we can rewrite Eq. (i) as
Hence, =
D iron Ycopper rρ dQ dQ
NR ∝ ∝ r ⋅ (r 2 )2 or ∝ r5
η dt dt
13 By the relation of Hooke’s law
Stress should be less. For less value of N R , 24 Surface tensions,
= constant radius, density should be small and F 2 × 10−2
Strain
viscosity should be high. S = = = 01
. N/m
2l 2 × 10 × 10−2
2
9 × 1011 × 10−7 × 1
14 W = YAl = 19 d A = 2cm and d B = 4cm
2L 2 × 100 25 Increase in surface energy
∴ r A = 1 cm and r B = 2 cm = Work done in area × surface tension
= 4.5 × 10 J 2
From equation of continuity, QIncrease in surface area
15 Work done = Elastic potential energy av = constant ∆A = (5 × 4 − 4 × 2) × 2
stored vA a π(r B ) 2 = (20 − 8) × 2 = 24 cm2
∴ = B =
1 vB aA π(r A ) 2
or W = × stress × strain × volume So, work done W = T ⋅ ∆A
2
3 × 10−4 = T × 24 × 10−4
2
=  
1 F l 1 2
or W = × × × AL = Fl  1 1
2 A L 2 T = = 0125. N/m
⇒ v A = 4 vB 8
114 40 DAYS ~ NEET PHYSICS DAY ELEVEN

26 Energy spent 33 The value of angle of contact determines 3 According to ascent formula for
= T × increase in surface area whether a liquid will spread on the capaillary tube,
= T × 2[4 π(2r )2 − 4 πr 2 ] = 24 πTr 2 J. surface. 2S cos θ
h=
2S 2 × 75 × 10−3 ρgr
27 As volume remains constant therefore 34 h= = cos θ1 cos θ2 cos θ3
r ρg 0.25 × 10− 3 × 103 × 10 = = =K
R = n1 /3 r ρ1 ρ2 ρ3
Energy of big drop 4 πR2T = 60 mm
= Thus, cos θ ∝ K
Energy of small drop 4 πr 2T h ′ r ′ = hr
Given, ρ1 > ρ2 > ρ3
R2 hr
= = (8)2 /3 = 4 or r′ = ∴ cos θ1 > cos θ2 > cos θ3
r2 h′ π
θ ≤ θ1 < θ2 < θ3 <
60 × 025
. 2
28 The excess pressure in a bubble of gas in = = 0.6 mm
25 4 Here, L = length of rod
a liquid is given by 2S / r , where S is the
surface tension of the liquid-gas
SESSION 2 l = immersed length of rod
interface. There is only one liquid
surface in this case. (For a bubble of θ = angle of inclination of rod with
1 The object is spherical and the bulk
liquid in a gas, there are two liquid horizontal
modulus is represented by B. It is the
surfaces, so the formula for excess ratio of normal stress to the volumetric
pressure in that case is 4S / r ). The B
strain.
radius of the bubble is r. Now, the r
F/A
pressure outside the bubble po equals Hence, B = FB
∆V / V
atmospheric pressure plus the pressure
∆V P
due to 8.00 cm fo water column. That is ⇒ =
p 0 = (1.01 × 105 Pa + 0.08 m V B
∆V P L
× 1000 kg m − 3 × 9.80 ms −2 ) ⇒ = h=
V B η
w
= 1.01784 × 105 Pa Here, P is applied pressure on the object A
θ ρ0
2S cos θ ∆V
29 h= and is volume strain.
rdg V N

h2 S 2 cos θ2 d 1 r Fractional decrease in volume


∴ = × × × 1 ∆V ∆R If the rod is in equilibrium, then the net
h1 S 1 cos θ1 d2 r2 =3
V R torque about the point A is zero i.e.
h2 140 cos 60° 1 1 L l
= × × ×1= Volume of the sphere decreases due to τ A = ( ρALg ) cos θ − ( ρ0 Alg ) cos θ = 0
h1 70 cos 0° 2 2 the decrease in its radius 2 2
h1 ∆V 3∆R P L2 l2
⇒ h2 = = 3 cm Hence, = = ⇒ =
2 V R B ρ0 ρ
∆R P
30 Surface energy = Surface tension = h L 1 ρ0
R 3B
Q sinθ = = =
× Surface area l ηl η ρ
E = T × 2A 2 From Bernoulli’s theorem, 1 ρ0 
Now, surface energy θ = sin −1  .
ω η ρ 
E1 = T × 2   = T × A
A
2 P 5 Depression produced in a beam with
h
% decrease in surface energy circular cross-section
A r C B
E − E1 wl3
= × 100 δ1 = [for solid rod]
E 12 πr 4 Y
2TA − TA
= × 100 = 50% wl3
2TA and δ2 =
4 4
1 2 1 12 π(r2 − r1 )Y
31 Energy released = ( A f − A i )T pA + ρV A + ρgh A = p B + ρV B2 +ρghB
2 2 [for hollow rod]
3 R 3 3V 1
A f = 4 πR 2 = 4π = Here, h A = hB ∴ δ1 ∝
3 R R 1 1 r4
V 3V p A + ρV 2A = p B + ρV B2
A i = n × 4 πr =
2
4 πr 2 = 2 2 1
4 3 and δ2 ∝
πr r 1 4
r2 − r1
4
3 ⇒ p A − p B = ρ( V B − V 2A )
2
2 δ2 r4
⇒ Energy released = −3VT  −  ∴ =
1 1 Now, V A = 0, V B = rω
 r R  δ1 r2 − r14
4
p A − p B = hρ g
Here, negative sign shows that amount 1 δ2 34
hρg = ρr 2ω2 or =
of energy is released. 2 8 4 − 24
4

32 If θ is obtuse, i.e. θ > 90°, then liquid r 2ω2 or δ2 = 2.7 mm


h=
meniscus will be convex upwards. 2g
DAY ELEVEN PROPERTIES OF MATTER 115

6 To twist the wire through an angle dθ 2  (21 /3 r ) 2 ( ρ − σ ) g  For holes (40 × 10−8 )v = a2 v 2
v′=   …(ii)
dW = τdθ 9 η  a2 v 2 = a1 v 1
10 π π 8 × 10−4 × 015
and θ = 10′ = × = rad From Eqs. (i) and (ii), we get 40 × 10−8 × v =
.
60 180 1080 2 60
θ η πr θdθ
4
η πr 4θ2 v′
θ
W = ∫ τ dθ = ∫ = = (2)3 8 × 10−4 × 015.
0 0 2l 4l v v =
2 40 × 10−8 × 60
5.9 × 1011 × 10−5 × π (2 × 10−5 )4 π2 v ′ = (2) 3 v = 5m/s
W = or
10−4 × 4 × 5 × 10−2 × (1080)2
9 Energy is released in the process 14 Impact speed = 2gh
= 1.253 × 10−12 J
∴ ∆E = n × 4 π r 2 S − 4 π R2S = 2 × 9.8 × 1200
7 Volume of liquid flowing through 4 4 18
But πR3 = n × πr 3 or R = n1 /3 r = 1533
. ×
capillary per second is given by 3 3 5
Poiseuille’s formula as, ∴ ∆E = n × 4 πr 2S − 4 π n2 /3 r 2S = 551.5 km/h
4
πpr 4 V r  or ∆E = 4 πr 2S (n − n2 /3 ) = E (n − n2 /3 )
V = ⇒ 2 =  2 15 Increase in length = BO + OC − BC
8 ηl V1  r1 
4 10 Taking torque about the point O, L L
V2 = V1 
110 
∴  = V1 (11
. )4 ρVr sin (45 − θ) g = σVr sin(45 + θ) g B C
 100  D
x
ρ− σ
= 1.4641 V ⇒ tanθ =
ρ+ σ O
∆V V2 − V1 1.4641 V − V
∴ = =
V V V 11 Increase in tension of wire = YAα ∆θ ⇒ ∆L = 2BO − 2L
= 0.46 or 46% = 8 × 10−6 × 22
. × 1011 × 10−6 × 5 = 2 [L2 + x2 ]1 /2 − 2L
8 Let r be the radius of the each drop. The = 8.8 N 1 /2
 x2 
terminal velocity of drop will be given 12 The pressure exerted by a 3000 m or ∆ L = 2 L 1 +  − 2L
2 r2 (ρ − σ ) g  L2 
by v = …(i) coloumn of water on the bottom layer is
9 η P = hρg = 3000 × 1000 × 10 = x2 / L
where, ρ is density of drop and σ is = 3 × 107
∆L x2 / L x2
∆V P Strain = = =
density of viscous medium of coefficient Fractional compression = = 2L 2L 2 L2
of viscosity η. When two drops each of V B
radius r coalescence to form a new drop, 3 × 107 −2 16 Breaking strength = tension in wire
= . × 10 = 136
= 136 . %
then the radius of coalescenced drop 22. × 109 = mrω2
1
will be R= (2)3 r 13 According to equation of continuity, 4.8 × 107 × 10−6 = 10 × 03
. × ω2
Hence, new terminal velocity of area × velocity = constant 48
ω2 = = 16
For tube 8 × 10−4 × 
coalescenced drop will be . 
015 . × 10
03
 = a1 v 1
 60  ⇒ ω = 4 rad/s
DAY TWELVE

Transfer
of Heat
Learning & Revision for the Day
u Heat u Calorimetry u Stefan’s Law
u Temperature u Latent Heat u Newton’s Law of Cooling
u Thermal Expansion u Heat Transfer u Wein’s Displacement Law
u Specific Heat u Perfectly Black Body u Green House Effect

Heat
Heat is a form of energy which characterises the thermal state of matter. It is transferred
from one body to the other due to temperature difference between them.
Heat is a scalar quantity with dimensions [ML2 T−2 ] and its SI unit is joule (J) while
practical unit is calorie (cal); 1 cal = 4.18 J
If mechanical energy (work) is converted into heat then, the ratio of work done (W ) to
heat produced (Q) always remains the same and constant.
W
i.e. = constant = J or W = JQ
Q
The constant J is called mechanical equivalent of heat.
PREP
Temperature MIRROR
Your Personal Preparation Indicator
The factor that determines the flow of heat from one body to another when they are in
contact with each other, is called temperature. Its SI unit is kelvin. u No. of Questions in Exercises (x)—
u No. of Questions Attempted (y)—
No. of Correct Questions (z)—
Thermometer u

(Without referring Explanations)


An instrument used to measure the temperature of a body is called a thermometer. For
construction of thermometer, two fixed reference point ice point and steam point are u Accuracy Level (z / y × 100)—
taken. Some common types of thermometers are as follows: u Prep Level (z / x × 100)—

1. Liquid (mercury) thermometer Range of temperature: −50°C to 350°C


In order to expect good rank in NEET, your
2. Gas thermometer (Nitrogen gas) Range of temperature: −200°C to 1600°C Accuracy Level should be above 85 & Prep
Level should be above 75.
3. Pyrometers Range of temperature: −800°C to 6000°C
DAY TWELVE TRANSFER OF HEAT 117

Scales of Temperature NOTE • The coefficients α , β and γ for a solid are related to each
other.
Three most common scales are Celsius scale or Centigrade
β γ
scale, Fahrenheit scale and Kelvin scale (Absolute scale). α= =
2 3
Ice point / lower Steam point / Upper • As temperature increases, density decreases according to
Scale Unit
reference point reference point relation,
ρ0
Celsius 0 100 °C ρ=
1 + γ∆T
Fahrenheit 32 212 °F
or ρ = ρ0 ( 1 − γ ∆T ) [valid for small ∆T]
Kelvin 273.15 373.15 K

Relation between C, F and K scales is Thermal Expansion of Liquid


C F − 32 K − 273.15 Liquids do not have linear and superficial expansion but
= =
5 9 5 these only have volume expansion.
Temperature of X − Ice point of X Liquid have two coefficient of volume expansion
In general,
Steam point of X − Ice point of X 1. Coefficient of apparent expansion,
Temperature of Y − Ice point of Y Apparent expansion in volume
= γa =
Steam point of Y − Ice point of Y Initial volume × ∆T
(∆V )a
=
V × ∆T
Thermal Expansion
2. Coefficient of real expansion,
Almost all substances (solid, liquid and gas) expand on
heating and contract on cooling. The expansion of a Real expansion in volume (∆V )r
γr = =
substance on heating is called thermal expansion of Initial volume × ∆T V × ∆T
substance.
Anomalous/Exceptional Behaviour of Water
Thermal Expansion of Solids Generally, density of liquids decreases with increase in
temperature but for water as the temperature increases from 0
Thermal expansion in solids is of three types:
to 4°C, its density increases and as temperature increases
1. Linear Expansion Thermal expansion along a single beyond 4°C, the density decreases.
dimension of a solid body is defined as the linear
expansion. If a rod is having length l0 at temperature T, The variation in the density of water with temperature is
then elongation in length of rod due to rise in shown in the figure given below.
temperature by ∆T is,
∆l
∆l = l0 α ∆T or α = Density
l0 × ∆T
where, α is the coefficient of linear expansion whose
value depends on the nature of the material.
Final length, l f = l0 + l0α ∆T = l0(1 + α ∆T ) Temperature
4°C
2. Superficial Expansion or Areal Expansion Anomalous Behaviour of Water
Expansion of solids along two dimensions of solid objects
is defined as superficial expansion.
∆A Thermal Expansion of Gases
Coefficient of superficial expansion, β =
A 0 × ∆T Gases have no definite shape, therefore, gases have only
volume expansion.
Final area, A f = A0(1 + β∆T )
1. The coefficient of volume expansion at constant
where, A0 is the area of the body at temperature T. ∆V 1
pressure, α= ×
3. Volume or Cubical Expansion Expansion of solids along V0 ∆T
three dimensions of solids objects is defined as cubical
expansion. Final volume, V ′ = V (1 + α ∆T )
∆V 2. The coefficient of pressure expansion at constant
Coefficient of volume or cubical expansion, γ = ∆p 1
V0 × ∆ T volume, β= ×
p ∆T
Final volume, V = V0 (1 + γ∆ T ).
Final pressure, p′ = p (1 + β ∆T ).
where, V0 is the volume of the body at temperature T.
118 40 DAYS ~ NEET PHYSICS DAY TWELVE

The heat required to change the phase of a system


Specific Heat is proportional to the mass of the system i.e.
The quantity of heat required to raise the temperature of unit Q∝m
mass of a substance by 1°C is called specific heat.
Q = mL
Q
Specific heat, s = where, L is the latent heat, which is defined as the amount of
m × ∆T
heat required to change the phase of the unit mass of a
The SI unit of specific heat is J kg −1 k −1. substance at given temperature.
l
Specific heat capacity can have any value from 0 to ∞. For l
In case of ice, the latent heat of fusion of ice is
some substances under particular situations, it can have 80 cal/gm.
negative values also. l
In case of water, the latent heat of vapourisation is
l
The product of mass of the body and specific heat is 539 cal/gm.
termed as heat capacity, C = m × s.

Molar Heat Capacity Heat Transfer


The heat can be transferred from one body to the other body,
The amount of heat required to change the temperature of a
through the following modes
unit mole of substance by 1°C is termed as its molar heat
capacity,
Q
1. Conduction
Cm = The process of heat-transmission in which the particles of the
µ∆T
body do not leave their position is called conduction.
Generally, for gases, two molar heat capacities are very
common—molar heat capacity at constant pressure (C p) and Thermal Conductivity
molar heat capacity at constant volume (CV ). The amount of heat transmitted through a conductor is given
KA∆T t
by Q =
Water Equivalent of a Substance l
Water equivalent of certain amount of substance is defined as where, A = area of cross-section,
the amount of water, which when replaced by the substance ∆T = temperature difference = T2 − T1,
requires the same amount of heat for the same rise in t = time elapsed,
temperature.
K = thermal conductivity
mS
mw = , and l = length of conductor
Sw
The rate of transmission of heat by conduction is given by
where, mw = water equivalent of substance whose mass is m, ∆Q KA∆T
S = specific heat capacity of substance H= =
∆t l
and S w = specific heat capacity of water The unit of thermal conductivity is Wm−1K −1.

Thermal Resistance
Calorimetry
∆Q KA ∆T
Calorimetry means measurement of heat. When a body at |H|= = ⋅ ∆T =
higher temperature is brought in contact with another body at ∆t l l / KA
lower temperature, the heat lost by the hot body is equal to l
The term is generally called the thermal resistance (R).
the heat gained by the colder body and provided no heat is KA
allowed to escape to the surrounding. A device in which heat l
Equation for rate of heat conduction can be written as
measurement can be made is called a calorimeter.
Q ∆T
If temperature changes, heat exchanged is given by H = =
t Rthermal
Q = ms∆T
As temperature of the body increases, it means heat is taken It is equivalent/analysis to ohm’s law which states that
V
by the body, otherwise given out by the body. I =
R(electrical)
Q
Latent Heat where, H = is equivalent of electric current and called as
t
In case of phase change, heat is consumed during melting and heat, ∆T is equivalent of voltage (PD) and Rthermal is
boiling while released during freezing and condensation. equivalent of Relectrical .
DAY TWELVE TRANSFER OF HEAT 119

Combination of Metallic Rods Interaction of Radiation with Matter


1. Series Combination In a series combination of two When radiant energy Q is incident on a body, a part of it Qa is
metal rods, equivalent thermal conductivity is given by absorbed, another part Qr is reflected back and yet another
l + l2 part Qt is transmitted such that
Ks = 1
l1
+ 2
l Q = Qa + Qr + Qt
K 1 K2 Qa Qr Qt
or + + = 1 or a + r + t = 1
K1 K2 Q Q Q
Q
where, a = a = absorbing power or absorptance,
Q
l1 l2 Q
r = r = reflecting power or reflectance
2K 1K 2 Q
or Ks = [if l 1 = l2 ] Qt
K 1 + K2 and t = = transmitting power or transmittance
Q
If temperature of the interface of the series combination
be T, then
K T + K 2T2
T = 1 1
Perfectly Black Body
K 1 + K2 A perfectly black body is the one which completely absorbs
the radiations of all the wavelengths that are incident on it.
2. Parallel Combinations In a parallel combination of two
Thus, absorbing power of a perfectly black body is 1 (i.e
metal rods, thermal conductivity is given by a = 1).
A1 A2 When perfectly black body is heated to a suitable high
temperature, it emits radiation of all possible wavelengths.
e.g. temperature of the sun is very high (6000 k approx.) it
K1 K2
emits all possible radiations. So, it is an example of black
body.
l
For perfectly black body, a = 1, r = t = 0
K 1 A1 + K 2 A2
Kp = For a perfect reflector, a = t = 0, r = 1
A1 + A2
l

K 1 + K2
l
For a perfect transmitter, a = r = 0, t = 1.
or Kp = [if A1 = A2 ]
2
Emissive Power and Emissivity
Formation and Growth of Ice on a Lake
Total emissive power of a given surface at a given
Time required for the thickness of the layer of ice to increase temperature is defined as the total amount of radiant energy
from d1 to d2 will be emitted per unit surface area per unit time by the body. SI
ρL f 2 unit of emissive power is Wm−2 .
t = (d2 − d21 )
2 KT Emissive power of a surface depends on the nature of the
where, ρ = density of ice, surface and its temperature.
L f = latent heat of fusion of ice Emissivity of a body at a given temperature is defined as the
ratio of the total emissive power of the black body (e) to the
and K = thermal conductivity of ice
total emissive power of perfectly black body (E) at that
temperature
2. Convection i.e. ε=
e
The process of heat-transmission in which the particles of the E
fluid move is called convection. A perfectly black body is also the perfect emitter i.e. it emits
radiations of all possible wavelengths at that temperature.
3. Radiation
The process of the transfer of heat from one place to another Absorptive Power
place without heating the intervening medium is called The ratio of the radiant energy absorbed by a surface in a
radiation. given time to the total radiant energy incident on the surface
in the same time, is called the absorptive power.
120 40 DAYS ~ NEET PHYSICS DAY TWELVE

Kirchhoff ’s Law of Radiation Newton’s Law of Cooling


Kirchhoff’s law of radiation states that the ratio of emissive According to the Newton’s law of cooling, rate of cooling of a
power to absorptive power of a body, is same for all body is directly proportional to the temperature difference
surfaces at the same temperature and is equal to the between the body and the surroundings. The law holds good
emissive power of a perfectly black body at that
temperature. only for small temperature difference.
e e dT
Mathematically, 1 = 2 = K = E = 1 or e = a Mathematically, − ∝ (T − T0)
a1 a2 dt
dT
Kirchhoff’s law implies that ‘a good absorber is a good or − = k (T − T0)
dt
emitter (or radiator) too’.
where, k is a constant.
Fraunhoffer’s lines (dark lines observed in solar
spectrum) can be easily explained on the basis of If a body cools by radiation through a small temperature
Kirchhoff’s laws. difference from T1 to T2 in a short time t when the surrounding
temperature is T0, then

Stefan’s Law dT ~ T1 − T2 T1 + T2
− − and T =
dt t 2
According to the Stefan’s law, the emissive power of a
perfectly black body (energy emitted by black body per The Newton‘s law of cooling becomes
unit surface area per unit time) is directly proportional to T1 − T2  T + T 
= k 1 2 − T0 .
the fourth power of its absolute temperature.  t   2 
Mathematically,
E ∝ T 4 or E = σT 4 Black Body Spectrum
where, σ is a constant known as the Stefan’s constant and The black body spectrum is Eλ
its value is 5.67 × 10 −8 Wm−2 K −4. a continuous spectrum as T4 > T3
shown in the figure. At a
For a body, whose emissivity is ε, Stefan’s law is modified
given temperature, initially
as, e = εσT 4. T3 > T2
the intensity of thermal
The total radiant energy Q emitted by a body of surface radiation increases with an
area A in time t, is given by T2 > T 1
increase in wavelength and
Q = Ate = AtεσT 4. reaches a maximum value
at a particular wavelength
The radiant power (P), i.e. energy radiated by a body per T1
λ m . On increasing the
unit time is given by λ
wavelength beyond λ m , the λm4 λ λ λ
Q m3 m2 m1
P = = AεσT 4 intensity of radiation E λ
t starts decreasing.
If a body at temperature T is surrounded by another body The total area under E λ -λ curve gives the total intensity of
at temperature T0 (where, T0 < T ), then Stefan’s law is radiation at that temperature.
modified as,
The area, in accordance with the Stefan’s law of radiation, is
E = σ (T 4 − T04) [black body] directly proportional to the fourth power of the temperature.
and e = εσ (T 4 − T04) [any body]

Wien’s Displacement Law


Solar Constant According to Wien‘s law, the product of wavelength
The amount of heat received from the sun by one square corresponding to maximum intensity of radiation and
centimeter area of a surface placed normally to the sun temperature of body is constant i.e. λ mT = constant = b, where b
rays at mean distance of the earth from the sun is known is known as the Wien’s constant and its value is 2.89 × 10 −3 mK.
as solar constant. It is denoted by S.
2
 r
S =   σT4
 R Green House Effect
The carbon dioxide concentration in the atmosphere has
where, r is the radius of sun and R is the mean earth’s
increased which may be attributed to the increase in the
distance from sun value of solar constant S = 1.937
temperature of atmosphere. This effect is known as the green
cal/cm2 /min.
house effect.
DAY TWELVE TRANSFER OF HEAT 121

DAY PRACTICE SESSION 1

FOUNDATION QUESTIONS EXERCISE


1 The metal sheet as shown in figure with two holes cut-off 7 2 g of steam condenses, when passed through 40 g of
unequal diameters d1 and d 2 (d1 > d 2 ). If the sheet is water initially at 25°C. The condensation of steam raises
heated, the temperature of water to 54.3°C. What is the latent
heat of steam?
(a) 540 cal/g (b) 536 cal/g
d2 (c) 270 cal/g (d) 480 cal/g
d1 8 Steam at 100°C is passed into 20 g of water at 10°C.
When water acquires temperature of 80°C, the mass of
water present will be
(a) both d1 and d2 will decrease
(b) both d1 and d2 will increase (Take, specific heat of water = 1 cal g−1 ° C−1 and latent
(c) d1 will increase, d2 will decrease heat of steam = 540 cal g−1 ) j CBSE AIPMT 2014

(d) d1 will decrease, d2 will increase (a) 24 g (b) 31.5 g (c) 42.5 g (d) 22.5 g
2 What will be the force developed in steel rod of 9 A piece of ice falls from a height h so that it melts
cross-sectional area 150 mm 2 , which is fixed between completely. Only one-quarter of the heat produced is
two fixed points, if temperature is increased by 20°C? absorbed by the ice and all energy of ice gets converted
(Assume, α = 10 −5 /°C and Y = 200 × 10 11N/m 2 ) into heat during its fall. The value of h is [Latent heat of
ice is 3.4 × 105 J/kg and g = 10 N / kg] j
NEET 2016
(a) 200 kN (b) 400 kN (c) 600 kN (d) 800 kN
(a) 544 km (b) 136 km (c) 68 km (d) 34 km
3 The value of coefficient of volume expansion of glycerin
10 In a steady state, the temperature at the ends A and B of
is 5 × 10− 4 K − 1. The fractional change in the density of
20 cm long rod AB are 100°C and 0°C, respectively. The
glycerin for a rise of 40°C in its temperature is
temperature of a point 9 cm from A is
j
CBSE AIPMT 2015
(a) 45°C (b) 60°C (c) 55°C (d) 65°C
(a) 0.015 (b) 0.020
(c) 0.025 (d) 0.010 11 The heat is flowing through a rod of length 50 cm and
area of cross-section 5 cm 2 . Its ends are respectively at
4 A clock with a metal pendulum beating seconds keeps
25°C and 125°C. The coefficient of thermal conductivity
correct time at 0°C. If it loses 12.5 s a day at 25°C, the
of the material of the rod is 0.092 kcal/ms. The
coefficient of linear expansion of metal pendulum is
1 1 1 1 temperature gradient in the rod is
(a) /° C (b) /° C (c) /° C (d) /° C
86400 43200 14400 28800 (a) 2°C/cm (b) 2°C/m (c) 20°C/cm (d) 20°C/m

5 In similar calorimeters, equal volumes of water and alcohol 12 Three rods of same dimensions have thermal
when poured take 100 s and 74 s respectively to cool from conductivities 3 K, 2 K and K. They are arranged as
50°C to 40°C. If the thermal capacity of each calorimeter is shown, with their ends at 100°C, 50°C and 0°C. The
numerically equal to volume of either liquid, then the temperature of their junction is
specific heat capacity of alcohol is 2K 50°C
(Given, relative density of alcohol as 0.8 and specific 3K
heat capacity of water as 1 cal/g/°C) 100°C
K 0°C
(a) 0.8 cal/g°C (b) 0.6 cal/g°C
200 100
(c) 0.9 cal/g°C (d) 1 cal/g°C (a) 75°C (b) °C (c) 40°C (d) °C
3 3
6 Two identical bodies are made of a material for which the
heat capacity increases with temperature. One of these 13 The two ends of a metal rod are maintained at
is at 100°C, while the other one is at 0°C. If the two temperatures 100°C and 110°C. The rate of heat flow in
bodies are brought into contact, then assuming no heat the rod is found to be 4.0 J/s. If the ends are maintained
loss, the final common temperature is j
NEET 2016 at temperatures 200°C and 210°C, the rate of heat flow
will be j
CBSE AIPMT 2015
(a) 50°C (b) more than 50°C
(c) less than 50°C but greater than 0°C (a) 44.0 J/s (b) 16.8 J/s (c) 8.0 J/s (d) 4.0 J/s
(d) 0°C
122 40 DAYS ~ NEET PHYSICS DAY TWELVE

14 The ratio of the emissive power to the absorptive power of 22 A sphere, a cube and a thin circular plate, all of same
all substances for the particular wavelength is the same at material and of same mass are initially heated to same
given temperature. The ratio is known as high temperature
(a)the emissive power of a perfectly black body (a)plate will cool fastest and cube the slowest
(b)the emissive power of any type of body (b)sphere will cool fastest and cube the slowest
(c)the Stefan’s constant (c)plate will cool fastest and sphere the slowest
(d)the Wien’s constant (d)cube will cool fastest and plate the slowest
15 A black body is at 727°C. It emits energy at a rate which 23 A black body at 227°C radiates heat at the rate of 7
is proportional to cal-cm −2 s −1. At a temperature of 727°C, the rate of heat
2 4 2 4 radiated in the same units will be j CBSE AIPMT 2009
(a) (727) (b) (1000) (c) (1000) (d) (727)
16 A sphere has a surface area of 1.0m and a temperature 2 (a) 60 (b) 50 (c) 112 (d) 80
of 400 K and the power radiated from it is 150 W. 24 The initial temperature of a body is 80°C. If its
Assuming the sphere is a black body radiator, the power temperature falls to 64°C in 5 minutes and in 10 minutes
in kilowatt radiated when the area expands to 2.0 m 2 to 52°C, then the temperature of surrounding will be
and the temperature changes to 800 K is (a) 26°C (b) 49.6°C (c) 35°C (d) 42°C
(a) 6.2 (b) 9.6 25 A body cools from a temperature 3T to 2T in 10 minutes.
(c) 4.8 (d) 16
The room temperature isT . Assume that Newton’s law of
17 Two spheres of the same material have radii 1 m and cooling is applicable. The temperature of the body at the
4 m, temperature 4000K and 2000K, respectively. Then, end of next 10 minutes will be j NEET 2016

the ratio of energy radiated per second by the first 7 3


(a) T (b) T
sphere as compared to that by the second is 4 2
4
(a) 4 : 1 (b) 2 : 1 (c) T (d) T
(c) 1 : 1 (d) 1 : 4 3

18 If a black body radiates 10 cal s −1 at 227°C, it will radiate 26 The two ends of a rod of length L and a uniform
at 727°C cross-sectional area A are kept at two temperatures
dQ
(a) 10 cal s −1
(b) 80 cal s −1 T1 and T2 (T1 > T2 ). The rate of heat transfer, , through
dt
(c) 160 cal s −1 (d) None of these
the rod in a steady state is given by j
CBSE AIPMT 2009
19 The surface temperature of the sun isT K and the solar
dQ KL (T1 − T2 ) dQ K (T1 − T2 )
constant for a plate is S. The sun subtends an angle θ at (a) = (b) =
dt A dt LA
the planet. Then,
dQ dQ KA(T1 − T2 )
(a) S ∝ T 4 (b) S ∝ T 2 (c) = KLA(T1 − T2 ) (d) =
dt dt L
(c) S ∝ θ2 (d) S ∝ θ
27 There are two identical vessel filled with equal amount of
20 If the radius of a star is R and it acts as a black body,
ice. The vessels are of different metals. If the ice melts in
what would be the temperature of the star in which the
the two vessels in 20 min and 35 min respectively, the
rate of energy production is Q ? j
CBSE AIPMT 2012
1/ 2
ratio of the coefficient of thermal conductivity of the two
Q  Q  metals is
(a) (b)  
4 πR σ 2  4 πR 2 σ 
1/ 4
(a) 4 : 7 (b) 7 : 4
 4 πR 2Q   Q 
1/ 4
(c) 16 : 49 (d) 49 : 16
(c)   (d)  
 σ   4 πR 2 σ  28 A black body at a temperature of 2600 K has the
wavelength corresponding to maximum emission 1200 Å
where, σ stands for Stefan’s constant.
assuming the moon to be perfectly black body. The
21 The total radiant energy per unit area, normal to the temperature of the moon, if the wavelength
direction of incidence, received at a distance R from the corresponding to maximum emission is 5000 Å, is
centre of a star of radius r, whose outer surface radiates (a) 7800 K (b) 624 K
as a black body at a temperature T K is given by (c) 5240 K (d) 3640 K
j
CBSE AIPMT 2011 29 Certain quantity of water cools from 70°C to 60°C in the
σr T
2 4
σr T
2 4
first 5 minutes and to 54°C in the next 5 minutes. The
(a) (b)
R2 4 πr 2 temperature of the surroundings is j
CBSE AIPMT 2014
σr 2 T 4 4 πσr 2 T 4 (a) 45°C (b) 20°C
(c) (d)
r 4
R2 (c) 42°C (d) 10°C
DAY TWELVE TRANSFER OF HEAT 123

30 If a body coated black at 600 K surrounded by then it can be concluded from the above observations
atmosphere at 300 K has cooling rate r0, the same body that j CBSE AIPMT 2015

at 900 K, surrounded by the same atmosphere, will have (a)TP > TQ > TR (b)TP > TR > TQ
cooling rate equal to (c)TP < TR < TQ (d)TP < TQ < TR
16 8
(a) r0 (b) r0 (c) 16r0 (d) 4 r0 34 If λ m denotes the wavelength at which the radiative
3 16
emission from a black body at a temperature T K is
31 The temperature of sun is 5500 K and it emits maximum
maximum, then
intensity radiation in the yellow region (5.5 × 10−7 m). The
(a) λm ∝ T 4 (b) λm is independent ofT
maximum radiation from a furnance occurs at
(c) λm ∝ T (d) λm ∝ T −1
wavelength 11 × 10−7 m. The temperature of furnace is
(a) 1125 K (b) 2750 K
35 A piece of iron is heated in a flame. If first becomes dull
(c) 5500 K (d) 11000 K red then becomes reddish yellow and finally turns to
white hot. The correct explanation for the above
32 A black body is at a temperature of 5760 K. The energy of
observations is possible by using j NEET 2013
radiation emitted by the body at wavelength 250 nm isU1,
at wavelength 500 nm isU2 and that at 1000 nm isU3 . (a) Stefan’s law (b) Wien’s displacement law
(c) Kirchhoff’s law (d) Newton’s law of cooling
Wien's constant, b = 2.88 × 106 nmK. Which of the
following is correct ? j NEET 2016 36 Which of the following is the λ m -T graph for a perfectly
(a) U 3 = 0 (b) U1 > U 2 black body?
(c) U 2 > U1 (d) U1 = 0
33 On observing light from three different stars P, Q and R, (a) λm (b) λm
it was found that intensity of violet colour is maximum in
the spectrum of P, the intensity of green colour is T T
maximum in the spectrum of R and the intensity of red
colour is maximum in the spectrum of Q. If TP , TQ and TR (c) λm (d) λm
are the respective absolute temperatures of P , Q and R,
T T

DAY PRACTICE SESSION 2

PROGRESSIVE QUESTIONS EXERCISE


1 At temperature T0, two metal strips of length l0 and [2 + (α A + αB ) ∆T ]d [2 − (α A + αB ) ∆T ]d
(a) R = (b) R =
thickness d, is bolted, so that their ends coincide. The 2 (α A − αB ) ∆T 2 (α A − αB ) ∆T
upper strip is made up of metal A and have coefficient of [2 + (α A − αB ) ∆T ]d [2 − (α A − αB ) ∆T ]d
(c) R = (d) R =
expansion α A and lower strip is made up of metal B with 2 (α A − αB ) ∆T 2 (α A − αB ) ∆T
coefficient of expansion α B (α A > α B ). When temperature
2 The specific heat of a substance at temperature t°C is
of their blastic strip is increased from T0 to (T0 + ∆T ), one
s = at 2 + bt + c. The amount of heat required to raise the
strip becomes longer than the other and blastic strip is
temperature of m kg of the substance from 0°C to t 0°C is
bend in the form of a circle as shown in figure. Calculate
mt 03a bt 02 mt 03a mbt 02
the radius of curvature R of the strip. (a) + + ct 0 (b) + + mct 0
3 2 3 2
3 2
mt a mbt 0
(c) 0 + (d) None of these
A 3 2
l0
B 3 A spherical black body with a radius of 12 cm radiates
A d
450 W power at 500 K. If the radius were halved and the
B
temperature doubled, the power radiated in watt would
R RB RA be j
NEET 2017
q
(a) 225 (b) 450
(c) 1000 (d) 1800
O
124 40 DAYS ~ NEET PHYSICS DAY TWELVE

4 A wall has two layers A and B, each made of different 10 Assuming the sun to have a spherical outer surface of
materials. Both the layers have the same thickness. The radius r, radiating like a black body at temperature t ° C,
thermal conductivity for A is twice that of B and under the power received by a unit surface, (normal to the
steady condition, the temperature difference across the incident rays) at a distance R from the centre of the sun
wall is 36°C. The temperature difference across the layer is (where, σ is the Stefan’s constant.)
A is 4 πr 2t 4 r 2 σ (t + 273)4
(a) (b)
(a) 6°C (b) 12°C (c) 24°C (d) 18°C R 2
4π R2
5 Ice starts forming in lake with water at 0°C and when the 16 π 2r 2 σ t 4 r σ (t + 273)4
2
(c) (d)
atmospheric temperature is −10°C. If the time taken for R2 R2
1 cm of ice be 7 h, then the time taken for the thickness 11 A black body calorimeter filled with hot water cools from
of ice to change from 1 cm to 2 cm, is 60°C to 50°C in 4 min and 40°C to 30°C in 8 min. The
(a) 7 h (b) 14 h approximate temperature of surrounding is
(c) less than 7 h (d) more than 7 h
(a) 10°C (b) 15°C (c) 20°C (d) 25°C
6 A wall has two layers A and B each made of different 12 The power radiated by a black body is P and it radiates
materials. The layer A is 10 cm thick and B is 20 cm maximum energy at wavelength, λ 0. If the temperature
thick. The thermal conductivity of A is thrice that of B. of the black body is now changed, so that it radiates
Under thermal equilibrium, temperature difference 3
maximum energy at wavelength λ 0, the power radiated
across the wall is 35°C. The difference of temperature 4
across the layer A is by it becomes nP. The value of n is j NEET 2018

(a) 20°C (b) 10°C (c) 15°C (d) 5°C 256 4 3 81


(a) (b) (c) (d)
7 A cylindrical metallic rod in thermal contact with two 81 3 4 256
reservoirs of heat at its two ends conducts an amount of 13 A black body at 1227°C emits radiations with maximum
heat Q in time t. The metallic rod is melted and the intensity at a wavelength of 5000 Å. If the temperature of
material is formed into a rod of half the radius of the the body is increased by 1000°C, the maximum intensity
original rod. What is the amount of heat conducted by the will be observed at
new rod when placed in thermal contact with the two (a) 4000 Å (b) 5000 Å (c) 6000 Å (d) 3000 Å
reservoirs in time t? j
CBSE AIPMT 2010
14 In the figure, the distribution of energy density of the
(a) Q / 4 (b) Q /16 radiation emitted by a black body at a given temperature
(c) 2Q (d) Q / 2 is shown. The possible temperature of the black body at
8 In the figure, ABC is a B λ m = 1.5 µm is
conducting rod whose lateral
2K
surfaces are insulated. The E
length of the section AB is 3K
A
one-half of that of BC and the
respective thermal
conductivities of the two C
0 1.0 1.5 2.0 2.5 λ (in µm)
sections are as given in the figure. If the ends A and C
(a) 1500 K (b) 2000 K (c) 2500 K (d) 3000 K
are maintained at 0°C and 70°C respectively, the
2
temperature of junction B in the steady state is 15 A non-conducting body floats in a liquid at 20°C with of
3
(a) 30°C (b) 40°C (c) 50°C (d) 60°C
its volume immersed in the liquid. When liquid
9 Two identical square rods of metal are welded end to 3
temperature is increased to 100°C, of body’s volume is
end as shown in Fig. (i), 20 cal of heat flows through it in 4
4 min. If the rods are welded as shown in Fig. (ii), the immersed in the liquid. Then, the coefficient of real
same amount of heat will flow through the rods in expansion of the liquid is (neglecting the expansion of
container of the liquid)
0°C 100°C
0°C 100°C (a)15.6 × 10−4 ° C −1
Fig. (i) Fig. (ii) (b) 156 × 10−4 ° C −1
. × 10−4 ° C −1
(c)156
(a) 1 min (b) 2 min (c) 4 min (d) 16 min (d) 0156
. × 10−4 ° C −1
DAY TWELVE TRANSFER OF HEAT 125

16 Two slabs A and B of different materials but of the same 17 An experiment takes 10 min to raise temperature of water
thickness are joined end to form a composite slab. The from 0°C and 100°C and another 55 min to convert it
thermal conductivities of A and B are K1 and K 2 totally into steam by a stabilised heater. The latent heat
respectively. A steady temperature difference of 12°C is of vaporisation comes out to be
K
maintained across the composite slab. If K1 = 2 , the (a) 530 cal/g
2 (b) 540 cal/g
temperature difference across slabs A is (c) 550 cal/g
(a) 4°C (b) 6°C (d) 560 cal/g
(c) 8°C (d) 10°C

ANSWERS
SESSION 1 1 (b) 2 (c) 3 (b) 4 (a) 5 (b) 6 (b) 7 (a) 8 (d) 9 (b) 10 (c)
11 (a) 12 (b) 13 (d) 14 (a) 15 (b) 16 (c) 17 (c) 18 (c) 19 (a) 20 (d)
21 (a) 22 (c) 23 (c) 24 (b) 25 (b) 26 (d) 27 (b) 28 (b) 29 (a) 30 (a)
31 (b) 32 (c) 33 (b) 34 (d) 35 (b) 36 (d)

SESSION 2 1 (a) 2 (b) 3 (d) 4 (b) 5 (d) 6 (d) 7 (b) 8 (a) 9 (a) 10 (d)
11 (b) 12 (a) 13 (d) 14 (b) 15 (a) 16 (c) 17 (c)

Hints and Explanations


SESSION 1 4 Number of second lost in a day 6 Heat lost by Ist body = heat gained by
1 IInd body. Body at 100°C temperature
1 When a sheet having positive coefficient ∆t = α ∆θ × 86400 has greater heat capacity than body at
of expansion is heated, diameters of 2
0°C so final temperature will be closer to
holes present in the sheet will increase. The coefficient of linear expansion of
100°C. So, Tc > 50° C.
metal pendulum,
2 Since, the support is fixed, so its length
2∆t 2 × 12.5 7 Let L be the latent heat and using
won’t change due to increase in α = =
∆θ × 86400 25 × 86400 principle of calorimetry,
temperature. Instead of expansion, the
rod shall undergo compressive stress σ, 2L + 2(100 − 543. ) = 40 × (543
. − 25)
1
such that ∆lexpansion + ∆l stress = 0 α = /°C L = 5403. cal/g
86400
σl
⇒ α l . ∆T − =0 8 Heat given by water = Heat lost by
λ 5 Let, V be volume of either liquid, mass steam
⇒ σ = α λ . ∆T of water = V × 1 g 20 × 1 × (80 − 10)
⇒ σ = 10−5 × 200 × 1011 × 20 Mass of alcohol = V × 0.8 = 0.8 Vg = m × 540 + m × 1 × (100 − 80)
∴ Force = σ A = 150 × 10−6 × 10−5 Rate of cooling of the water calorimeter ⇒ 1400 = 560 m
1
× 200 × 1011 × 20 = [ V × (50° − 40° ) ⇒ m = 2.5 g
100
Total mass of water = 20 + 2.5 = 22.5 g
= 600000 N = 600 kN + V × 1 × (50° − 40° )]
1 9 According to question as conservation of
3 Given, the value of coefficient of volume = V cal/s
−4 −1 5 energy, energy gained by the ice during
expansion of glycerin is 5 × 10 K . its fall from height h is given by
Rate of cooling of alcohol calorimeter
As, orginal density of glycerin, 1 E = mgh
= [ V × (50° − 40° )
ρ = ρ0 (1 + Y∆T ) 74 As given, only one quarter of its energy
⇒ ρ − ρ0 = ρ0 Y∆T + 0.8V × s (50° − 40° )] is absorbed by the ice.
Thus, fractional change in the density of 1 mgh mL f × 4
= (10V + 8Vs ) cal/s So, = mL f ⇒ h =
glycerine for a rise of 74 4 mg
40°C in its temperature, As, rate of cooling of both is same Lf × 4
ρ − ρ0 3.4 × 105 × 4
= Y∆T = 5 × 10−4 × 40 V 1 = =
= (10V + 8Vs ) g 10
ρ0 5 74
= 200 × 10 −4
= 0.020 ⇒ s = 0.6 cal/g°C = 13.6 × 104
= 136000 m = 136 km
126 40 DAYS ~ NEET PHYSICS DAY TWELVE

100 − 0 100 18 E ∝ T 4 , T1 = 227°C = 500 K 23 According to Stefan’s law


10 H = = Heat current =
R AB R AB E = σT 4
and T2 = 727°C = 1000 K
Hence, T2 = 2T1 σ = Stefan’s constant
100 − TC = HR AC
100 ∴ E2 = (2)4 E1 T = temperature
= R AC Given, T1 = 227°C and T2 = 727°C
R AB = 16 E1 = 160 cal s −1
E1 = 7 cal cm −2 s −1
100 × 9
= = 45 19 Let radius of the sun = R 4
20 E 1  T1 
Distance of the earth from the sun = d =
TC = 55° C E2  T2 
Power radiated from the sun
4 4
11 Temperature gradient P = (4 πR2 ) σT 4  273 + 727   1000 
dT (125 − 25) ⇒ E2 = 7   = ×7
= = 2°C/cm Power received/area = S =
P  273 + 227   500 
dx 50cm
4 πd 2 = 112 cal-cm2 s −1
12 Suppose T be the temperature of T4
= 4 πR 2 σ 24 According to Newton’s law,
junction. H 1 , H 2 and H 3 the heat 4 πd 2 T1 − T2  T + T2 
currents. Then, =k 1 − T0
R2 t  2 
50°C = σT 4
 80 − 64   80 + 64
− T 0 
H2
H1 d2 Initially,   = k
2
 5   2 
σT 4 
100°C 1 2R 
H3 0°C =  ⇒ 3.2 = k (72 − T 0 ) …(i)
4  d 
Finally,
H1 = H2 + H3 64 − 52 64 + 52
= k  − T 0 
Angle subtended by sun at the earth
100 − T T − 50 T −0 α =
2R
. 10  2 
or = +
 l   l   l  d
. = k (58 − T 0 )
12 …(ii)
     
 3K A   2K A   K A S = constant × T 4 × α2 On solving Eqs. (i) and (ii), we get
or 3(100 − T ) = 2(T − 50) + T T 0 = 49.6° C
∴ S ∝T4
200
⇒ T = °C 25 According to Newton’s law of cooling,
3 20 From Stefan’s law, ∆T = ∆T 0 e −λt
13 In both cases the temperature difference E =σT 4
⇒ 3T − 2T = (3T − T ) e − λ × 10
…(i)
between the ends of the rod is 10°C. Again for next 10 minutes
So, the rate of energy production,
∴ Rate of heat flow is also 4 J/S in the
Q =E × A T ′ − T = (2T ) × e − λ ( 20 ) …(ii)
second case. From Eqs. (i) and (ii), we get
Q =σT 4
× 4 πR 2

14 Kirchhoff’s law of radiation states that T ′ − T = (2T ) (e −λ × 10 )2


1/4
the ratio of emissive power to  Q  2
Temperature of star, T =  = (2T )   =
 1 T
absorptive power of a body is same for  4 πR 2 σ   2 2
all surface at the same temperature and
T 3T
is equal to the emissive power of a 21 If r is the radius of the star and T is its ∴ T′=T + =
perfectly black body at that temperature, then the energy emitted by 2 2
temperature. the star per second through radiation in 26 For a rod of length L and area of
accordance with Stefan’s law will be cross-section A whose faces are
15 According to Stefan’s law,
given by maintained at temperaturesT1 and T2
E ∝T 4
A σ T 4 = 4 πr 2 σT 4 respectively.
or E = σT 4 ,
Then in steady state the rate of heat
Here, E ∝ ( 727 + 273)4 In reaching a distance R, this energy flowing from one face to the other face
[σ = Stefan’s constant] will spread over a sphere of radius R, so in time t is given by
the intensity of radiation will be given dQ KA(T1 − T2 )
⇒ E ∝ (1000) 4
by =
dt L
P 4 πr 2 σ T 4
16 P ∝ AT 4 S = = kA (T1 − T2 )t
4 πR 2
4 πR 2 27 Q =
Area and temperature both are double. l
Hence, power will become ( 2)5 or 32 r σT 4
2
k1 t 1 = k2 t 2 = constant
=
times. R2 k1 t 1 35 7
= = =
32 × 150 k2 t 2 20 4
∴ P ′ = 32P = kW 22 Rate of loss of heat from a body is
1000 directly proportional to the surface area
28 From Wien's law,
= 4.8 kW of the body for a given mass of material,
the surface area of circular plate is λ1T1 = λ2T2
P A T4 4 π (1)2 (4000)4 1 maximum and of sphere is least. Hence, λT 1200 × 2600
17 1 = 1 14 = = T2 = 1 1 =
P2 A 2T2 4 π (4)2 (2000)4 1 plate will cool the fastest and sphere λ2 5000
will cool the slowest.
T2 = 624 K
DAY TWELVE TRANSFER OF HEAT 127

29 Newtons law of cooling 35 The equation of Wien’s displacement ∴ Power radiated,


T1 − T2  T + T2  law, i.e. P ′ = σ   (2T )4
A
=K 1 − T0
∆t  2   4
λ mT = constant
70 − 60 16
First ⇒ = K [65 − T 0 ] 36 For perfectly black body, = ⋅ (σAT 4 )
5 4
1
⇒ 2 = K [65 − T 0 ] ...(i) λm ∝ = 4P = 4 × 450
60 − 54 T
Next ⇒ = K [57 − T 0 ] ...(ii) = 1800 W
So, graph is rectangular hyperbola as
5
shown in option (d). 4 K A = 2K B
On dividing Eqs. (i) and (ii), we get
5 65 − T 0 ∴ RA = B
R Q R = l 
= SESSION 2
3 57 − T 0 2  KA 
⇒ 285 − 5T 0 = 195 − 3T 0 1 At temperature T 0 + ∆T , increase in Suppose, R A = R,
⇒ 2T 0 = 90 length then, R B = 2R
T 0 = 45° C L A = l0 (1 + α A ∆T ) …(i)

30 Cooling rate ∝ T − 4
T 04 L B = l0 (1 + α B ∆T ) …(ii)
Let R A and R B be the radius of strips,
 (900)4 − (300)4  L A = θR A
A B
r = …(iii)
4 0
r
 (600) − (300) 
4
L B = θR B …(iv)
16 as θ is the common angle.
= r0
3 From Eqs. (i) and (ii), we get
Heat current,
L A − L B = θ (R A − R B )
31 λ m1 T1 = λ m2 T2 H =
36
=
36 12
=
L − LB R + 2R 3R
5.5 × 10 −7 −7
× 5500 = 11 × 10 T ⇒ θ= A R
R A − RB
T = 550 × 5K = 2750 K ∴ Temperature difference across A
l0 (α A − α B ) ∆T 12
= …(v) = HR A = ×R
32 Given, temperature, T1 = 5760 K d R
Since, it is given that energy of Similarly from Eqs. (i), (ii), (iii), and = 12° C
radiation emitted by the body at (iv), we get ρL f
wavelength 250 nm in U 1 , at 2l + l0 (α A + α B ) ∆T
5 t= (d 22 − d 12 )
wavelength 500 nm is U 2 and that at R A + RB = 0 2KT
θ …(vi) ⇒ t ∝ (d 22 − d 12 )
1000 nm is U 3 .
Q According to Wien’s law, we get R A + RB t (d 22 − d 12 )
Mean radius, R = ⇒ =
λ mT = b 2 t ′ (d 2 ′2 −d 1 ′2 )
where, b = Wien’s constant 2l + l0 (α A + α B ) ∆T
= 0 …(vii) 7 (12 − 02 )
= 2.88 × 106 nmK 2θ ⇒ =
t ′ (22 − 12 )
b 2.88 × 106 nmK Putting the value of θ in Eq. (iii),
⇒ λm = ⇒ λm = we get ⇒ t ′ = 21 h
T 5760 K
6 RA =
10 Q R = l 
⇒ λ m = 500 nm [2l0 + l0 (α A + α B ) ∆T ]
R= 3 KA  KA 
 ∆T  
Q λ m = wavelength corresponding to 2 l0 (α A − α B )  
maximum energy, so, U 2 > U 1 .   d  
33 We know from Wien’s displacement [2 + (α A + α B ) ∆T ] d A
⇒ R= 3K B K
law, 2 (α A − α B ) ∆T
λ mT = constant t
So, T ∝
1 2 ∆H = ∫ ms dt = m ∫ 0 (at 2 + bt + c ) dt
0 10 cm 20 cm
λm  at 3 
bt 2
As, λr > λg > λv = m  0 + 0 + ct 0 
3 2 20 R A 1
  RB = , =
So, Tr < Tg < Tv KA R B 6
Given, P → v max , Q → r max , R → g max mt 30 a mbt 20
= + + mct 0 So, let R A = R, then R B = 6R
Hence, TQ < T R < T p 3 2
Heat current,
34 According to Wien’s displacement law, 3 Radiated power of a black body, Temperature difference
the wavelength (λ m ) of maximum H =
P = σAT 4
Total resistance
intensity of emission of black body 35 5
radiation is inversely proportional to where, A = surface area of the body = =
T = temperature of the body 7R R
absolute temperature (T ) of the black
body. Therefore, Wien’s law is and σ = Stefan’s constant Now, temperature difference across A
5
λ mT = constant When radius of the sphere is halved, = H RA = × R = 5° C
new area, R
or λ m ∝ T −1
A
A′ =
4
128 40 DAYS ~ NEET PHYSICS DAY TWELVE

7 In steady state the amount of heat T1 − T2  T + T2  λ m = 5000 Å


11 =α 1 − T0
flowing from one face to the other face t  2  1500
Hence, λ m′ = × 5000 = 3000 Å
in time t is given by 2500
KA (θ1 − θ2 ) t Here, T 0 is the temperature of
Q = , surroundings. 14 According to Wien’s law, λ mT = b
l
Substituting the values, we have where, b = 2.89 × 10−3 mK
where K is coefficient of thermal
conductivity of material of rod 60 − 50  60 + 50 
=α − T0 ...(i) ⇒ T =
b
4  2 
Q A r2 λm
⇒ ∝ ∝ …(i)
t l l 40 − 30  40 + 30 T  2.89 × 10−3
=α − 0 ...(ii) =
As the metallic rod is melted and the  
8 2 1.5 × 10−6
material is formed into a rod of half the
Solving these two equations, we get = 2000 K
radius
V1 = V2 T 0 = 15°C 15 Coefficient of real expansion,
 r  V2 − V1
π r1 l 1 = πr22 l 2
2
Q r2 = 1 12 According to Wien’s law, γr =
 2  V1 (T2 − T1 )
1
l2 λ max ∝ 3 2
⇒ l1 = …(ii) T Here, V2 = , V1 =
4 i.e. λ maxT = constant 4 3
Now, from Eqs. (i) and (ii) where, λ max is the maximum and ( t 2 − t 1 ) = (100 − 20) = 80°C
Q1 r2 l wavelength of the radiation emitted at  3 − 2
= 1 × 2  
Q2 l1 r22 temperature T .  4 3
∴ γr =
∴ λ max 1 T1 = λ max 2 T2 2
(80)
r12 4l 1
= × T1 λ max 2 3
l1 (r1 / 2)2 or = …(i) 1
T2 λ max 1 =
⇒ Q 1 = 16 Q 2 640
3
Q Q Here, λ max 1 = λ 0 and λ max 2 = λ0 = 15.6 × 10−4 °C −1 .
⇒ Q2 = 1 = [QQ 1 = Q ] 4
16 16 Substituting the above values in
16 The given situation can be shown as,
8 Heat currents in both the rods are equal. Eq. (i), we get
3
i.e. H CB = H BA λ0 A B
T1 3 T 3
= 4 = or 1 = …(ii) 12°C K1 x K2 0°C
TC − T B T −TA T2 λ0 4 T2 4
or = B
 2l   l  As we know that, from Stefan’s law, the Rate of flow of heat will be equal in
   
 3KA   2KA  power radiated by a body at both the slabs
3 temperature T is given as ∴ (12 − x )K 1 = K 2 ( x − 0)
∴ (70 − T B ) = 2(T B − 0)
2 P = σAεT 4  K 
12 − x = 2 x Q K1 = 2
i.e. P ∝ T 4 (Qthe quantity σ Aε is  2 
or T B = 30° C
constant for a body) ⇒ x = 4°C
9 Q = KA∆T = ∆T =
∆T
4 The temperature difference across slab A
t l  l  R P1 T14 T 
  ⇒ = =  1 = (12 − x )
 KA  P2 T24  T2 
= (12 − 4)
[R = Thermal resistance] From Eq. (i), we get = 8° C
⇒ t ∝R [QQ and ∆T are same] P1  3 
4
81
tp Rp R /2 1 =  = 17 Heat given for raising the temperature of
⇒ = = = P2  4  256 W gram of water from 0°C to 100°C
ts Rs 2R 4
Given, P1 = P and P2 = nP = W × 1 × 100 cal
⇒ tp =
ts 4
= = 1 min ⇒
P1
=
P
=
81 Time taken = 10 × 60 s
4 4 P2 nP 256 ∴Heat given per second
256 W × 1 × 100
10 From Stefan’s law, the rate at which or n= = cal
energy is radiated by sun at its surface 81 10 × 60
is P = σ × 4 πr 2T 4 13 According to Wien’s law, Heat given out to convert W gram to
where, r is the radius of the sun so, the steam = W × L
λ mT = constant This is the heat supplied in 55 × 60 s.
power received by unit surface at a
where, λ m is wavelength corresponding ∴Heat given
distance R from the centre of the sun.
to maximum intensity of radiation and 55 × 60
P σ × 4 πr 2T 4 T is temperature of the body in kelvin. = 100 × W × = WL
I = = 10 × 60
4 πR 2
4 πR 2 λ m′ T
∴ = 100 × 55 × 60
σr 2T 4 λm T′ ∴ L =
= 10 × 60
R2 Given, T = 1227 + 273 = 1500 K, = 100 × 5.5
σr ( t + 273)
2 4 T ′ = 1227 + 1000 + 273 = 2500 K, L = 550 cal/g
=
R2
DAY THIRTEEN

Behaviour of
Perfect Gas and
Kinetic Theory
Learning & Revision for the Day
u Ideal Gas or Perfect Gas u Kinetic Theory of Gases u Specific Heat Capacities of
u Gas Law u Various Speeds of Gas Molecule Gases
u Equation of State of a u Degree of Freedom (f) u Mean Free Path
Perfect Gas u Law of Equipartition of Energy

Ideal Gas or Perfect Gas


An ideal gas or perfect gas is that gas which strictly obeys the gas laws such as Boyle’s
law, Charles’ law, Gay Lussac’s law, etc. at all value of temperature and pressure.

Gas Law
Some important gas laws are given below in table form.
p
1
Boyle’s law V ∝ or pV = constant
p
V PREP
V
V
MIRROR
V ∝ T or = constant Your Personal Preparation Indicator
Charles’ law
T
u No. of Questions in Exercises (x)—
T
u No. of Questions Attempted (y)—
p u No. of Correct Questions (z)—
p (Without referring Explanations)
Gay-Lussac’s law p ∝ T or = constant
T
T
u Accuracy Level (z / y × 100)—
u Prep Level (z / x × 100)—
1 1 r M2 ρ2
Grahm’ law of diffusion r∝ ∝ or 1 = =
ρ M r2 M1 ρ1 In order to expect good rank in NEET, your
Accuracy Level should be above 85 & Prep
Level should be above 75.
Daltan’s law of partial pressure P = P1 + P2 + P3 + P4 + .....+ Pn
130 40 DAYS ~ NEET PHYSICS DAY THIRTEEN

Equation of State of a Perfect Gas Kinetic Energy and Temperature


The equation which relates the pressure (P), volume (V ) and In ideal gases, the point particles can have only translational
temperature (T ) of the given state of an ideal gas is known as motion and thus only translational energy.
ideal or perfect gas equation. 1 3
pV
l
Translational KE of a molecule = mc 2 = kT
For 1 mole of gas, = R (constant) 2 2
T 3
l
Mean KE per molecule = kT
pV = RT 2
where, R is universal gas constant. The SI unit of gas constant l
Mean kinetic energy per gram mole is given by
is J/mol-K. Its value is 8.314 J/mol-K or 8.314 × 107 erg/mol-K 1  3 3
KEmole =  mc 2  N = kTN = RT
or 2 cal/mol-K. The dimensions of R are [ML2 T−1θ −1]. 2  2 2
Work 3
Moreover, gas constant R =
l
Average kinetic energy of gas = pV
Moles × Temperature 2
3 pV 3 RT
l
The perfect gas equation for 1 molecule of gas is
l
KE per molecule = =
2N 2N
pV = kT 3
l
KE per mole = kT
l
Boltzmann’s constant is represented by per mole gas 2
3
constant l
KE per volume = p
R 8.31 2
i.e. k = = = 1.38 × 10 −23 J K −1
N 6.023 × 1023
Its dimensions are [ ML2 T −2θ −1]. Concept of Pressure
Pressure p exerted by a perfect gas on the walls of container is
given by
Kinetic Theory of Gases p=
1 mN c 2 1 M 2
= c
3 V 3 V
Kinetic theory of gases relates the macroscopic properties of
pV = (ρc 2 ) =  ρc 2  =  ρc 2  = E
1 1 2 2 1 2
gases (such as pressure, temperature etc.) to the microscopic
3 32  32  3
properties of gas molecules (such as speed, momentum,
kinetic energy of molecules etc). Here, m = mass of each molecule, c = root mean square
velocity of molecules, ρ = density of gas, M = mass of gas
enclosed in volume V of container, and E = Total KE of the
Assumptions of Kinetic ideal gas.
Theory of Gases
l
Every gas is composed of tiny particles known as Various Speeds of Gas Molecules
molecules. The size of molecules is much smaller than the l
Root mean square speed It is defined as the square root
intermolecular spacing. of mean of squares of the speed of different molecules
l
The molecules of a gas are identical, spherical, rigid and
v21 + v22 + v32 + K
perfectly elastic point masses. i.e. vrms = = v2
N
l
Molecules are in a state of random rapid motion. They
1
collide with each other. There is no loss of energy during From the expression of pressure, p = ρv2rms
collision. Only the direction of motion is changed. 3
3p 3 pV 3 RT 3 kT
l
The time spent in collision between two molecules is vrms = = = =
negligible in comparison to time between two successive ρ Mass of gas M m
collisions. l
Most probable speed It is defined as the speed which is
l
The number of collisions per unit volume in a gas remains possessed by maximum fraction of total number of
constant. No attractive or repulsive force acts between gas molecules of the gas.
molecules. 2p 2 RT 2 kT
vmp = = =
l
Gravitational attraction among the molecules is ineffective ρ M m
due to extremely small masses and very high speed of
molecules.
l
Average speed It is the arithmetic mean of the speeds of
molecules in a gas at given temperature.
l
Molecules constantly collide with the walls of container v1 + v2 + v3 + v 4 + K
due to which their momentum changes. The change in v av =
N
momentum is transferred to the walls of the container.
8p 8 RT 8 kT
Consequently, on the walls of container pressure is exerted Average speed, vav = = =
by gas molecules. The density of gas is constant at all πρ π M πm
points of the container.
DAY THIRTEEN BEHAVIOUR OF PERFECT GAS AND KINETIC THEORY 131

For one mole of gas,


Degree of Freedom (f ) M (∆Q) p
The term degree of freedom of a system refers to the possible ∴ C p = Mc p = …(i)
m∆T
independent motions a system can have 1 ( ∆Q ) p
=
l
for monoatomic gas, ( f ) = 3 n ∆T
l
for diatomic gas, ( f ) = 5 Specific heat of a gas at constant pressure is greater than the
l
for triatomic gas, ( f ) = 6(non-linear) specific heat at constant volume i.e. C p > CV .
l
for triatomic (linear) gas, ( f ) = 7 C p and CV are related to each other according to relation,
l
for N-atomic molecule ( f ) = 6 N – 3 C p − CV =
R
…(ii)
J
l
for N-atomic linear molecule ( f ) = 6 N – 5
Eq. (ii) is called Mayer’s relation.
If C p and CV are measured in the units of work and R is also in
Law of Equipartition of Energy
the units of work (or energy), then Eq. (ii) becomes
According to law of equipartition of energy for any system in C p − CV = R.
thermal equilibrium, the total energy is equally distributed
among its various degree of freedom and each degree of
1 Specific Heat in Terms of Degree of
freedom is associated with energy kT (where,
2 Freedom
k = 1.38 × 10 −23 J/K and T = absolute temperature of the f
For a gas at temperature T, the internal energy U = nRT .
system). 2
f 1
Change in energy, ∆U = n R∆T ⇒ CV = f R
2 2
Specific Heat Capacities of Gases f 
The specific heat of gas can have many values, but out of them Specific heat at constant pressure, C p =  + 1 R.
2 
following two values are important
f 
 + 1 R
Cp 2  2
Specific Heat at Constant Volume Ratio of C p and CV , γ =
CV
=
f
=1 +
f
.
The specific heat of a gas at constant volume is defined as the R
2
quantity of heat required to raise the temperature of unit mass
of gas through 1°C or 1 K when its volume is kept constant i.e.
(∆Q )V
Mean Free Path
cV =
m∆T The distance travelled by a gas molecule between two
For one mole of gas, successive collision is known as free path.
M (∆Q )V 1 (∆Q )V Q n = m 
Total distance covered
C V = Mc V = = Mean free path =
m∆T n ∆T  M  Number of collisions
The mean free path of a gas molecule is the average distance
between two successive collisions.
Specific Heat at Constant Pressure It is represented by λ.
The specific heat of a gas at constant pressure is defined as the 1
quantity of heat required to raise the temperature of unit mass λ=
of gas through 1K, when its pressure is kept constant i.e. 2 π σ2 n
(∆Q) p Here, σ = diameter of the molecule and n = number of
cp = . molecules per unit volume.
m∆T
132 40 DAYS ~ NEET PHYSICS DAY THIRTEEN

DAY PRACTICE SESSION 1

FOUNDATION QUESTIONS EXERCISE


1 The volume of a given mass of a gas at 27°C, 1 atm is 10 Two molecules of a gas have speeds of 9 × 106 m/s and
100 cc. What will be its volume at 327°C? 1 × 106 m /s, respectively. What is the root mean square
(a) 50 cc (b) 100 cc (c) 150 cc (d) 200 cc speed of these molecules?
2 A vessel contains 1 mole of O 2 gas (molar mass 32) at (a) 39 × 106 m/s (b) 6 × 106 m/s
(c) 41 × 106 m/s (d) 8 × 106 m/s
temperature T . The pressure of the gas is p. An identical
vessel containing 1 mole of He gas (molar mass 4) at 11 For a gas at a temperature T the root-mean-square
temperature 2T has a pressure of velocity v rms , the most probable speed vmp and the
p average speed v av obey the relationship
(a) (b) p (c) 2 p (d) 8p
8 (a) v av > vrms > vmp (b) vrms > v av > vmp
3 In the given (V -T ) diagram, what is the relation between (c) vmp > v av > vrms (d) vmp > vrms > v av
pressures p1 and p2 ? 12 Root mean square speed of the molecules of ideal gas is
V v. If pressure is increased two times at constant
temperature, then the rms speed will become
p2
v
(a) (b) v (c) 2v (d) 4v
p1 2

θ2 13 The curve between absolute temperature and v 2rms is


v 2rms v 2rms
θ1
T
j
NEET 2013
(a) p2 = p1 (b) p2 > p1 (a) (b)
(c) p2 < p1 (d) Cannot be predicted
4 A balloon is filled at 27°C and 1 atm pressure by 500 m 3
T T
He. At − 3°C and 0.5 atm pressure, the volume of He gas
v 2rms v 2rms
will be j
AIIMS 2012
(a) 700 m3 (b) 900 m3 (c) 1000 m3 (d) 500 m3
5 At what temperature, the mean kinetic energy of O 2 will (c) (d)
be the same as that of H 2 molecules at − 73°C?
(a) 127°C (b) 527°C (c) − 73 ° C (d) −173 °C T T

6 Relationship between p, V and E for a gas is 14 The degrees of freedom of a molecule of a triatomic gas
(E = total translational kinetic energy) are
3 2 3 2 (a) 2 (b) 4 (c) 6 (d) 8
(a) p = EV (b)V = Ep (c) pV = E (d) pV = E
2 3 2 3 15 The temperature of argon, kept in a vessel, is raised by
7 When temperature of an ideal gas is increased from 1°C at a constant volume. The total heat supplied to the
27°C to 227°C. Its rms speed changed from 400 m/s gas is a combination of translational and rotational
to v s . The v s is energies. Their respective shares are
(a) 516 m/s (b) 450 m/s (c) 310 m/s (d) 746 m/s (a) 60% and 40% (b) 40% and 60%
(c) 50% and 50% (d) 100% and 0%
8 The ratio of the speed of sound in nitrogen gas to that in
helium gas, at 300 K is 16 One mole of ideal monoatomic gas (γ =5/3) is mixed with
one mole of diatomic gas (γ =7/5). What is γ for the
(a)   (b)  
2 1 3 6
(c) (d)
7 7 5 5 mixture? (γ denotes the ratio of specific heat at constant
pressure, to that at constant volume)
9 The molecules of a given mass of gas have rms speed
(a) 3/2 (b) 23/15 (c) 35/23 (d) 4/3
200 ms −1 at 27°C and105 Nm −2 pressure. When the
absolute temperature is doubled and the pressure is 17 The ratio of specific heats ( γ ) of an ideal gas is given by
1 R Cp
halved, the rms speed of the molecules of the same gas is (a) (b) 1 + (c) (d) All of these
(a) 200 ms −1
(b) 400 ms −1 1−
R CV Cp − R
(c) 200 2 ms −1 (d) 400 2 ms −1 Cp
DAY THIRTEEN BEHAVIOUR OF PERFECT GAS AND KINETIC THEORY 133

18 The molar specific heat at constant pressure of an ideal 24 For the specific heat of 1 mole of an ideal gas at
gas is (7/2)R. The ratio of specific heat at constant constant pressure (CP ) and at constant volume (Cv ),
pressure to that at constant volume is which is correct
(a) 7/5 (b) 8/7 (c) 5/7 (d) 9/7 5 7
(a) CP of hydrogen gas is R (b) Cv of hydrogen gas is R
2 2
19 The heat required to increase the temperature of 4 moles (c) H2 has very small values of CP and CV
of a monoatomic ideal gas from 273 K to 473 K at
(d) CP − CV = 199
. cal/mole-K for H2
constant volume is
(a) 200 R (b) 400 R 25 If the internal energy of n1 moles of He at temperature
(c) 800 R (d) 1200 R 10 T is equal to the internal energy of n 2 mole of
n
20 4.0 g of a gas occupies 22.4 L at NTP. The specific heat hydrogen at temperature 6T. The mole of 1 is
n2
capacity of the gas at constant volume is
3 5
5.0 JK − 1 mol− 1. If the speed of sound in this gas at NTP (a) (b) 2 (c) 1 (d)
5 3
is 952 ms − 1, then the heat capacity at constant pressure
is (Take gas constant R = 8.3 JK − 1 mol− 1) 26 40 calories of heat is needed to raise the
temperature of 1 mole of an ideal monoatomic gas
CBSE AIPMT 2015
j

−1
(a) 8.0 JK mol −1
(b) 7.5 JK mol − 1
−1 from 20°C to 30°C at a constant pressure. The
(c) 7.0 JK − 1 mol − 1 (d) 8.5 JK − 1 mol − 1 amount of heat required to raised its temperature
Cp over the same interval of a constant
21 The ratio of the specific heats = γ in terms of degrees volume (R = 2 calorie mole −1K −2 ) is
CV
(a) 20 cal (b) 40 cal (c) 60 cal (d) 80 cal
of freedom (n ) is given by j
CBSE AIPMT 2015
27 A gas mixture consists of 2 moles of O2 and 4 moles of
(a)  1 +  (b)  1 +  (c)  1 +  (d)  1 + 
1 n 2 n Ar at temperature T. Neglecting all vibrational modes, the
 n  3  n  2 total internal energy of the system is j
NEET 2017
22 The amount of heat energy required to raise to (a) 4RT (b) 15RT (c) 9RT (d) 11RT
temperature of 1 g of helium at NTP, from T1K to T2K is 28 One mole of an ideal monatomic gas undergoes a process
j
NEET 2013
described by the equation pV 3 = constant. The heat
3 3
(a) NakB (T2 − T1) (b) NakB (T2 − T1) capacity of the gas during this process is j
NEET 2016
8 2 3 5
3 3 T  (a) R (b) R (c) 2R (d) R
(c) NakB (T2 − T1) (d) NakB  2  2 2
4 4  T1 
29 The molar specific heats of an ideal gas at constant
23 When an ideal monoatomic gas is heated at constant pressure and volume are denoted by Cp and CV
pressure, fraction of heat energy supplied which Cp
increases the internal energy of gas, is respectively. If γ = and R is the universal gas
CV
2 3
(a) (b) constant, then CV is equal to j
NEET 2013
5 5
3 3 1+ γ R (γ − 1)
(c) (d) (a) (b) (c) (d) γR
7 4 1− γ (γ − 1) R

DAY PRACTICE SESSION 2

PROGRESSIVE QUESTIONS EXERCISE


1 The mean free path of molecules of a gas, (radius r) is 3 Two different isotherms representing the relationship
p
inversely proportional to j
CBSE-AIPMT 2014 between pressure p and volume V
3 2 at a given temperature of the same
(a) r (b) r (c) r (d) r
2 The equation of state for 5g of oxygen at a pressure p ideal gas are shown for masses m1
and temperature T , when occupying a volumeV , will be and m2 , then m2
(a) m1 > m2
(a)pV =   RT
5
(b) pV = 5 RT (b) m1 = m2
m1
 32  V
(c) m1 < m2
(c)pV =   RT (d) pV =   RT
5 5 (d) Nothing can be predicted
 2  16 
134 40 DAYS ~ NEET PHYSICS DAY THIRTEEN

pV 9 If temperature of the atmosphere varies with height as


4 The figure below shows, the plot of versus p for
nT T = (T0 − ah ), where a and T0 are positive constants, then
oxygen gas at two different temperatures. the pressure as a function of height h is (assume
Read the following statements concerning the above atmospheric pressure at sea level (h = 0) is p 0 and
curves: molecule mass M of the air and acceleration due to
gravity g be constant)
Mg /Ra 2Mg /Ra
 T − ah   T − ah 
T2 (a) p = p0  0  (b) p = p0  0 
(Jmol–1 K–1)

T1  T0   T0 
3Mg /Ra 4Mg /Ra
 T − ah   T − ah 
(c) p = p0  0  (d) p = p0  0 
 T0   T0 
pV
nT

10 Two different masses m and 3 m of an ideal gas are


p
heated separately in a vessel of constant volume. The
I The dotted line corresponds to the ideal gas behaviour
pressure p and absolute temperatureT graphs for these
II T1 > T2
two cases are shown in the figure as A and B. The ratio of
pV
III The value of at the point, where the curves meet on slopes of curves B to A is
nT
theY-axis is the same for all gases p

Which of the above statements is true? B


(a)Only (I) (b) Both (I) and (II)
(c)All of these (d) None of these
A
5 The temperature at which the root mean square speed of 3m
m
a gas will be half its value at 0°C is (assume the pressure O T
remains constant)
(a) 3 : 1 (b) 1 : 3 (c) 9 : 1 (d) 1 : 9
(a)− 86. 4 ° C (b) − 204 .75 ° C
(c)− 104 .75 ° C (d) − 68 . 25 ° C 11 The quantity of heat required to raise one mole
TA T through one degree kelvin for a monoatomic gas at
6 Let A and B be two gases and given : = 4⋅ B , constant volume is
MA MB
3 5 7
where, T is the temperature and M is molecular mass. If (a) R (b) R (c) R (d) 4R
2 2 2
C
CA and CB are the rms speed, then the ratio A will be 12 Two mole of oxygen is mixed with eight mole of
CB
helium. The effective specific heat of the mixture at
equal to
constant volume
(a) 2 (b) 4 (c) 1 (d) 0.5
(a) 1. 3 R (b) 1. 4 R (c) 1.7 R (d) 1. 9R
7 A cylindrical tube of
13 At 10°C the value of the density of a fixed mass of an
uniform cross-sectional Wire ideal gas divided by its pressure is x. At 110°C this ratio
area A is fitted with two
is j
CBSE AIPMT 2008
air tight frictionless
383 10 283
pistons. The piston are (a) x (b) x (c) x (d) x
283 110 383
connected to each other by a metallic wire. Initially, the
pressure of the gas is p0 and temperature is T0, 14 A cylinder contains 20 kg of N 2 gas (M = 28 kg kmol −1) at
atmospheric pressure is also p0. Now, the temperature a pressure of 5 atm. The mass of hydrogen (M = 2 kg
of the gas is increased to 2T0, the tension of the wire will kmol −1) at a pressure of 3 atm contained in the same
be cylinder at same temperature is
p0 A (a) 1.08 kg (b) 0.86 kg
(a) 2 p0 A (b) p0 A (c) (d) 4 p0 A (c) 0.68 kg (d) 1.68 kg
2
8 Two vessels separately contain two ideal gases A and B 15 At what temperature will the rms speed of oxygen
at the same temperature, the pressure of A being twice molecules become just sufficient for escaping from the
that of B. Under such conditions, the density of A is found earth’s atmosphere?
to be 1.5 times the density of B. The ratio of molecular (Take, mass of oxygen molecule, m = 2.76 × 10−26 kg,
weight of A and B is j
CBSE AIPMT 2015 Boltzmann’s constant kB = 1.38 × 10−23 J K −1 ) j NEET 2018
2 3 1 (a) 5.016 × 104 K (b) 8.326 × 104 K
(a) (b) (c) 2 (d)
3 4 2 (c) 2.508 × 104 K (d) 1.254 × 104 K
DAY THIRTEEN BEHAVIOUR OF PERFECT GAS AND KINETIC THEORY 135

ANSWERS
SESSION 1 1 (d) 2 (c) 3 (c) 4 (b) 5 (c) 6 (c) 7 (a) 8 (c) 9 (c) 10 (c)
11 (b) 12 (b) 13 (b) 14 (c) 15 (d) 16 (a) 17 (c) 18 (a) 19 (d) 20 (a)
21 (c) 22 (a) 23 (b) 24 (d) 25 (c) 26 (a) 27 (d) 28 (d) 29 (b)

SESSION 2 1 (b) 2 (a) 3 (c) 4 (c) 5 (b) 6 (a) 7 (b) 8 (b) 9 (a) 10 (a)
11 (a) 12 (c) 13 (d) 14 (b) 15 (b)

Hints and Explanations


SESSION 1 5 Mean kinetic energy of molecules 11 As we know that
1 If pressure of a given mass of the gas is depends upon temperature only. For O2 3kT 2KT
v rms = , v mp = ,
kept constant, then it is same as that of H2 at the same m m
temperature of −73° C.
V ∝T 8kT
v av =
V V V 6 The relationship between p, V and E for πm
⇒ = constant ⇒ 1 = 2
T T1 T2
a gas is pV =
3
E Hence, v rms > v av > v mp
T  2
⇒ V2 = V1  2  12 Root mean square speed,
 T1  v2 T2 v 273 + 227 5
7 = ⇒ s = = v rms =
3kT
=
3p
Here, T1 = 273 + 27 = 300 K v1 T1 400 273 + 27 3 m ρ
T2 = 273 + 327 = 600 K vs 5
= 400 = 516 m/s On increasing the pressure by the two
But V1 = 100 cc 400 3 times, the density also increases in the
V2 = V1 
600  same ratio, so that rms speed remains
 γ RT
 300  8 Speed of sound in a gas v = same.
M
V2 = 2V1
v N2 γ N2 M He 13 As we know that,
V2 = 2 × 100 = 200 cc = ×
γ He 3kT 2
v He M N2 v rms = , v rms ∝ T
2 pV = nRT m
(7 / 5) 4 3
⇒ p ∝ nT [QV and R = constants] = × = Hence, option (b) is correct.
p2 n T (5/ 3) 28 5
⇒ = 2 × 2 14 Non-linear triatomic
p1 n1 T1 9 v rms ∝ T . If temperature is doubled molecule In this
p He 1 2T rms speed will become 2 v rms times. molecule, the three
⇒ = × ⇒ p He = 2 p
p 1 T ⇒ v rms = 200 2 ms −1 atoms are present at the
vertices of triangle.
3 V -T graph is a straight line passing 10 By definition, Here, A = 3 and R = 3
through origin. v 12 + v 22 ∴ N = 3×3−3 = 6
v rms = , where v 1 and v 2 are So, a non-linear triatomic molecule has
Hence, process is isobaric. 2
six degree of freedom.
 nR  the individual velocities of the two
V = T
 p  particles. 15 Argon is a monoatomic gas, so it has
nR Given, v 1 = 9 × 106 m/s only translational energy.
Slope =
p and v 2 = 1 × 10 m/s
6 µ 1 γ1 µ γ
+ 2 2
Slope of 2 > Slope of 1 γ1 − 1 γ2 − 1
(9 × 106 )2 + (1 × 106 )2 16 γ mix =
∴ v rms = µ1 µ2
∴ p2 < p1 2 +
γ1 − 1 γ2 − 1
p1V1 pV
= 2 2 81 × 1012 + 1 × 1012
4 = 5 7
T1 T2 2 1× 1×
3 + 5
p1V1T2 1 × 500 × (273 − 3) (81 + 1) × 10 12
or V2 = = =  5 − 1  7 − 1
p2T1 0.5 × (273 + 27) 2    
3  5  3
1 × 500 × 270 = =
= 82 × 1012 1 1 2
= +
0.5 × 300 2  5 − 1  7 − 1
   
V2 = 900 m3 = 41 × 106 m/s 3  5 
136 40 DAYS ~ NEET PHYSICS DAY THIRTEEN

17 C p − C V = R f 29 As we know that
...(i) 22 We know that, Q = nR∆T
On dividing Eq. (i) by C p , we get
2 C p − CV = R
∴ Amount of heat required, C p = R + CV
C R 1 R  Cp 
1− V = or 1 − = = γ 3 1
Q = × × k B N a ∆T Cp
γ C p  C V
Q
Cp Cp  2 4 and =γ (given)
CV
Cp 3
1
⇒ = 1−
R
or γ = = N a k B (T2 − T1 ) R + CV
γ Cp Cp − R 8 So, =γ
CV
C
18 We have given molar specific heat at 23 For monoatomic gas, γ = p = 5 ⇒ γCV = R + CV ⇒ γCV − CV = R
CV 3
constant pressure, R
7 ⇒ CV =
Cp = R We know that, γ −1
2
Mayer’s relation is ∆Q = µC p ∆T and ∆U = µC V ∆T
5 ∆U C V 3
SESSION 2
CV = C p − R = R = =
2 1 1 1
∆Q C p 5 1 λ= = ⇒λ∝
Hence, required ratio is λd 2 n 2 4 πr 2 n 2 r2
 7 R i.e. fraction of heat energy to increase the
  2 Number of moles,
Cp  2 7 3
γ = = = internal energy be . m 5
CV  5 R 5 5 n= =
  molecular weight 32
 2 24 For any gas So, from ideal gas equation,
19 Specific heat for a monoatomic gas, C P − C V = 1.99 ≈ 2 cal mol −1 K −1 5
pV = nRT ⇒ pV = RT
3
CV = R 32
25 n1C V1 ∆T1 = n2C V2 ∆T2
2
3 pV = nRT = m RT
∴ Heat, dQ = µC V ∆T n1 ×
3 5
R × 10 = n2 × R × 6 M
3 2 2 For first graph,
= 4 × × R (473 − 273) ` n1 m RT
2 =1 p= 1 …(i)
[Q µ = 4] M V1
n2
3
= 4 × × R × 200 p
2 26 At constant pressure,
∴ dQ = 4 × 300R (∆Q ) p = µC p ∆T
= 1200 R = 1 × C p × (30 − 20) = 40
20 Given, M = 4 gm, V = 22.4 L, C p = 4 cal mol −1 K −1 m2

C V = 5 JK −1 mol −1 CV = C p − R m1

v sound = 952 m / s, C p = ? = 4 − 2 = 2 cal mol −1 K −1 V1 V2


V

As, velocity of sound, Now, (∆Q )V = µc V ∆T = 1 × 2 × (30 − 20)


For IInd graph,
γ pV M 2 Cp = 20 cal m RT
v sound = ⇒γ = v sound = p= 2 …(ii)
M pV CV 27 Total internal energy of system M V2
So, heat capacity at constant pressure, = Internal energy of oxygen On equating Eqs. (i) and (ii), we get
M  2 molecules + Internal energy of m1 V1
C p = CV  = ⇒m ∝V
 v sound argon molecules m2 V2
 pV 
f1 f2 V2 > V1 ⇒ m1 < m2
 4 × 10−3  = n1 RT + n2 RT As,
= 5 (952)2
−3 
2 2
 10 5
× 22.4 × 10  5 3 4 (i) The dotted line in the diagram
= × 2RT + × 4RT = 11RT shows that there is no deviation
20 2 2
= × (952)2 × 10−5 pV
22.4 in the value of for different
28 As we know that for polytropic process
= 809200 × 10−5 = 8.09 J/mol K of index α specific heat capacity
nT
temperatures T1 and T2 for
− 8.0 JK −1 mol −1
~ = CV +
R
1−α increasing pressure so, this gas
21 C V = n R behaves ideally.
2 Q Process, pV 3 = constant ⇒ α = 3
n (ii) At high temperature, the deviation
Q C p = CV + R = R + R R
∴ C = CV + of the gas is less and at low
2 1−α
temperature the deviation of gas is
∴ C p =  + 1 R
n
fR
2  where, C V = more. In the graph, deviation for T2
2 is greater than for T1 .
 n + 1 R
  fR R
Cp 2  n+2 = + (iii) The two curves intersect at
Now, γ = = = 2 1−3
CV n n pV
R 3R dotted line, so the value of at
2 For monatomic gas, f = 3 = nT
2 2
γ =1+ 3R R
that point on the y-axis is same
n ⇒ C = − =R for all gases.
2 2
DAY THIRTEEN BEHAVIOUR OF PERFECT GAS AND KINETIC THEORY 137

5 v rms ∝ T , rms speed will remain half, dp  pM  ρ 1


9 = − ρg = −   g ∴ ∝
1 dh  RT  p T
if temperature becomes th or
4  pM 
 273 − 273 ° C or − 204.75° C. =− g Molecular mass M and universal gas

 4

  R (T 0 − ah ) constant R remains same for a gas.
dp  Mg  dh So, for two different situations i.e. at two
TA T TA TB =− 
6 =4 B ⇒ =2 p  R  T 0 − ah different temperatures and densities
MA MB MA MB ρ1 / p1 T2 x 383 K
p dp  Mg  h dh ∴ = ⇒ =
3RT A 3RT ∫p =−
 R 
 ∫0 (T ρ2 / p2 T1 ( ρ2 / p2 ) 283 K
⇒ =2 p 0 − ah )
0
MA MB ρ2 283
Mg ⇒ = x
 T − ah  Ra p2 383
⇒ C A = 2C B p = p0  0  .
 T0  14 V = nR
CA
⇒ =2 T p
CB 10 For a gas, pV = nRT = m RT V and T for both cases is same.
M
n1 n m1 m2
7 Volume of the gas is constant p Hence, = 2 or =
i.e. V = constant p1 p2 p1 M1 p2 M2
B p M
F or m2 = 2 2 m1
p1 M1
3m A (3) (2)
m = 20 = 0.86 kg
pA (5) (28)
p0 A T
15 (b) The minimum velocity with which
Slope of graph A, the body must be projected vertically
∴ p ∝T  p m R
  = ...(i) upwards, so that it could escape from
i.e. pressure will be T  M V the Earth’s atmosphere, is its escape
doubled temperature is doubled Similarly, slope of graph B, velocity (v e ).
∴ p = 2 p0 p 3m R As, v e = 2gR
= …(ii)
Let F be the tension in the wire. Then, T M V Substituting the value of g (9.8 ms−2 )
equilibrium of any one piston gives 3m R and radius of Earth (R = 6.4 × 106 m), we
Slope of curve B 3
F = ( p − p 0 )A ∴ = M V = get
Slope of curve A m R 1
= (2 p 0 − p 0 )A = p 0 A M V v e = 2 × 9.8 × 6.4 × 106
≅ 11.2 km s −1
8 Vessel-I Vessel-II 11 (∆Q )V = µC V ∆T
(∆Q )V = 1 × C V × 1 = 11200 m s−1
Ideal Ideal Let the temperature of molecule be T
= CV
Gas A Gas B when it attains v e .
For monoatomic gas,
T T 3 According to the question,
CV = R v rms = v e
ρ A = 1.5 ρB 2
p A = 2 pB 3 where, v rms is the rms speed of the
∴ (∆Q )V = R oxygen molecule.
According to ideal gas equation, we 2
have 3k B T
12 For mixture of gases, let specific heat be ⇒ = 11.2 × 103
ρRT mO 2
Pressure, p = , where M is CV.
M n1 (C V )1 + n2 (C V )2
CV = (11.2 × 103 ) 2 (m O 2 )
molecular weight of ideal gas. n1 + n2 or T =
p RT ρRT (3 k B )
Such that, = ⇒M = 2×
5R
+ 8×
3R
ρ M P 2 2 = 17R = 17 Substituting the given values, i.e.,
= . R
where, R and T are constants. 2+ 8 10 R B = 1.38 × 10 −23 JK −1 and
ρ
So, M ∝ 13 Ideal gas equation m O 2 = m = 2.76 × 10 −26 kg
p pV = nRT
M A ρA p 1 pV 1 Q n = m  We get,
⇒ = × B = 1.5 × =  
MB ρB pA 2
RT
 M (11.2 × 10 3 ) 2 (2.76 × 10 −26 )
m M T =
3 p RT  V 1 (3 × 1.38 × 10 −23 )
= 075
. = = Q = 
4 ρ M  m ρ = 8.3626 × 10 4 K
DAY FOURTEEN

Thermodynamics
Learning & Revision for the Day

u Zeroth Law of u Thermodynamic Processes u Heat Engine


Thermodynamics u Second Law of Thermodynamics u Carnot Engine and Its
u First Law of u Reversible and Irreversible Efficiency
Thermodynamics Processes u Refrigerator

Zeroth Law of Thermodynamics


(Concept of Temperature)
When there is no exchange of heat between two systems placed in contact, then both
are called in thermal equilibrium.
According to this law, if two systems A and B , separated by an adiabatic wall, are
separately and independently in thermal equilibrium with a third system C, then the
systems A and B are also in a state of thermal equilibrium with each other.

A B

Adiabatic wall

Basic Terms Used in Thermodynamics PREP


Heat It is the energy, which is transferred between system and surroundings due to the
temperature difference.
MIRROR
Your Personal Preparation Indicator
Internal Energy Internal energy of a system is defined as the sum of the total kinetic
energy of all its constituent particles and sum of all the potential energies of u No. of Questions in Exercises (x)—
interaction among these particles. u No. of Questions Attempted (y)—
The internal energy of an ideal gas is totally kinetic and it is given by u No. of Correct Questions (z)—
(Without referring Explanations)
3
U = µRT
2 u Accuracy Level (z / y × 100)—
3 Prep Level (z / x × 100)—
and change in internal energy ∆U = µR∆T .
u

2
In order to expect good rank in NEET, your
NOTE • For non-ideal gases, internal energy depends not only on the temperature but also on the Accuracy Level should be above 85 & Prep
Level should be above 75.
pressure.
DAY FOURTEEN THERMODYNAMICS 139

Work
T1>T2
Consider a system in a cylinder with movable piston, whose
volume can be changed (a gas, liquid or solid). Suppose, the p
cylinder has a cross-sectional area A and pressure exerted by T1
system on the piston face is p. T2

The work done by the system on the surroundings for small


displacement dx is dW = pAdx. V
Vf
p-V graph for isothermal process
W = ∫ dW = ∫ pdV
Vi Molar specific heat of a gas under isothermal condition
∆Q ∆Q
i.e. work done in a finite change of volume from Vi to V f . C= = =∞
m∆T m (0)
NOTE • Work done by the system depends on the initial and final dp p
states. Slope of p-V curve at any point is =− ⋅
dV V
• If volume of the system increases, then work is done by the
Work done in an isothermal process
system and it is taken as positive work done.
Vf  Vf 
• If volume of the system decreases, then work is done on ∆W = ∫ pdV = nRT ln  
the system and it is taken as negative work done. Vi  Vi 
where, n = number of moles, R = gas constant
First Law of Thermodynamics and T = temperature.
V f and Vi are final and initial volume of the gas
According to this law, the heat given to a system (∆Q) is equal
respectively.
to the sum of increase in its internal energy (∆U) and the work
done (∆W ) by the system against the surroundings. As per first law of thermodynamics, since, ∆T = 0 , hence,
∆U = 0 for an ideal gas and we have ∆Q = ∆W .
Mathematically, ∆Q = ∆U + ∆W
Thus, heat supplied to the system in an isothermal
process is entirely used to do work against external
Sign Convention surroundings.
∆Q = + ve when heat supplied = − ve when heat is rejected
∆U = + ve when temperature increases 2. Adiabatic Process
= − ve when temperature decreases It is that process in which there is no exchange of heat of
∆W = + ve when work is done by the system (expansion) the system with its surroundings. Thus, in an adiabatic
= − ve when work is done on system (compression)
process p, V and T change but ∆Q = 0 or entropy remains
First law of thermodynamics is based on the energy  ∆Q 
conservation. constant  ∆S = = 0 .
 T 

Thermodynamic Processes Q2 Q1
A thermodynamic process is the process of change of state of
a system involving change of thermodynamic variables, e.g. p Q1 > Q2
p, V , T etc.
When a system undergo a thermodynamic change, then work
done either by system on surrounding or by surroundings on
system is called external work. V
V2 p-V graph for adiabatic process
Wext = ∫ p dV = area under p-V curve.
V1 The equation of state for an adiabatic process is
pV γ = constant or T V γ − 1 = constant or T γ p1 − γ = constant
1. Isothermal Process
l
Molar specific heat of a gas under adiabatic condition
It is that process in which temperature remains constant.
Here, exchange of heat with the surroundings is allowed. ∆Q 0
C= = =0
As temperature T remains constant in an isothermal m ⋅ ∆T m ⋅ ∆T
process, hence as per Boyle’s law dp p
l
Slope of an adiabatic at a point is =−γ .
1 dV V
p ∝ or pV = constant
V
140 40 DAYS ~ NEET PHYSICS DAY FOURTEEN

l
Work done in an adiabatic process l
Indicator diagram for an isobaric process is a straight line
Vf µR µR parallel to X -axis.
∆W = ∫ p dV = (Ti − T f ) = − ∆T
Vi (γ − 1) (γ − 1)
During an adiabatic expansion ∆W = + ve, hence,
temperature of gas falls, i.e. an adiabatic expansion is p p
always accompanied with cooling.
V V
As per first law of thermodynamics, since, ∆Q = 0 in an (a) (b)
adiabatic process hence,
∆U = − ∆W Graph (a) represent isobaric expansion, graph (b) represent
isobaric compression.
l
Free expansion is an adiabatic process in which ∆W = 0.
Hence, in accordance with first law of thermodynamics
l
Work done in an isobaric process
Vf
∆U = 0 i.e. the final and initial values of the internal energy ∆W = ∫ p dV = p ∫ dV = p(V f − Vi ) = p∆V
Vi
are equal in free expansion.
l
Molar specific heat of a gas under isobaric condition
3. Isochoric Process
f 
It is that thermodynamic process in which volume C p =  + 1 R = CV + R
2 
remains constant.
In an isochoric process for a given mass of gas
p Second Law of Thermodynamics
p ∝ T or = constant
T Two most common statements of second law of
l
Indicator diagram for an isochoric process is a straight line thermodynamics are given below
parallel to p-axis.
Clausius Statement
It is impossible for a self-acting machine, working in a cyclic
process to transfer heat from a colder body to a hotter body
without the aid of an external agency.
p p
Kelvin-Planck’s Statement
V V
(a) (b) It is impossible to design an engine which extracts heat from
a reservoir and fully converts it into work without producing
Graph (a) shows isometric heating graph in which pressure any other effect.
increases, temperature increases, ∆Q is positive and ∆U is
positive.
Similarly, Graph (b) shows isometric cooling graph in
Reversible and Irreversible
which pressure decreases, temperature decreases, ∆Q is Processes
negative and ∆U is negative. A reversible process is one which can be reversed in such a
l
Molar specific heat of a gas under isochoric condition way that all changes taking place in the direct process are
f exactly repeated in inverse order and in opposite sense, and
CV = R, where f is the number of degrees of freedom per
2 no changes are left in any of the bodies taking part in the
molecule. process or in the surroundings. Any process which is not
reversible exactly is an irreversible process.
l
Work done in an isochoric process ∆W = ∫ p dV = 0

As ∆W = 0 hence, according to first law of thermodynamics, Cyclic Process


we have
µR In cyclic process, if the process takes the path AxB, it returns
(∆Q)V = ∆U = µCV ∆T = ∆T via ByA, the initial and final points are same.
(γ − 1) p
B
4. Isobaric Process x
It is that process in which pressure remains constant.
y
As in an isobaric process for a given mass of gas A
V ∝T
V V
or = constant A cyclic process
T
DAY FOURTEEN THERMODYNAMICS 141

W Q T
Heat Engine η=
Q1
=1− 2 =1− 2
Q1 T1
Any cyclic device by which heat is converted into mechanical
work is called a heat engine. The efficiency does not depend on the nature or quantity of
There are three main parts in an engine: the working substance.
(i) a hot body source
(ii) a working substance
Refrigerator
(iii) a cold body called sink.
A refrigerator or heat pump is basically a heat engine running
The efficiency of heat engine, in reverse direction. It takes heat from colder body (sink) and
W Q1 − Q2 Q after doing some work gives the rest heat to the hotter body
η= = =1− 2
Q1 Q1 Q1 (source). An ideal refrigerator can be regarded as Carnot’s
ideal heat engine working in the reverse direction.

Carnot Engine and Its Efficiency Source Q1 Working Q2 Sink


Carnot engine is a theoretical, ideal heat engine working in a T1 substance T2
reversible cyclic process operating between two temperatures
T1 (heat source) and T2 (heat sink).
W
The Carnot’s cycle consists of two isothermal
processes connected by two adiabatic processes as shown in Refrigerator
the figure.

A Coefficient of Performance of a
T1
p Q1
T1>T2 Refrigerator
B
p It is defined as the ratio of quantity of heat removed per cycle
D (Q2 ) to the work done (W ) on the working substance per cycle
T2 T1
Q2 C T2 to remove this heat.
Q Q2 T2 1−η
V β= 2 = or β = =
V W Q1 − Q2 T1 − T2 η
Carnot’s cycle Isothermal process

The efficiency of a Carnot’s cycle is given by

DAY PRACTICE SESSION 1

FOUNDATION QUESTIONS EXERCISE


1 1 cc of water at its boiling point (100°C) absorbs 540 cal Given that initial internal energy of the sample was
of heat to become steam with a volume of 1671 cc. If the 20 J,what will be the final internal energy?
atmospheric pressure is 1.013 × 10 5 Nm −2 , the energy (a) 72 J (b) 32 J (c) 28 J (d) 12 J
spent (in cal) to overcome molecular attractive forces is 4 A gas is compressed at a constant pressure of 50 Nm −2
nearly (mechanical equivalent of heat = 4.2 J/cal) from a volume of 10 m 3 to a volume of 4 m 3 . Energy of
(a) 540 (b) 500 (c) 40 (d) zero 100 J is then added to the gas by heating. Its internal
2 When one mole of monoatomic gas expands at constant energy is
pressure, the ratio of the heat supplied that increases the (a)increased by 400 J (b) increased by 200 J
internal energy of the gas and that is used in expansion (c)increased by 100 J (d) decreased by 200 J
is 5 The internal energy change in a system that has
3 2
(a) ∞ (b) zero (c) (d) absorbed 2 kcal of heat and done 500 J of work is
2 3 j
CBSE AIPMT 2009
3 In a thermodynamic process with 2 moles of gas, 30 J of (a) 8900 J (b) 6400 J (c) 5400 J (d) 7900 J
heat is released and 22 J of work is done on the gas.
142 40 DAYS ~ NEET PHYSICS DAY FOURTEEN

6 Ten moles of an ideal gas at constant temperature 600 K 12 An ideal gas at 27°C is compressed adiabatically to
is compressed from 100 L to 10 L. The work done in the 8/27 of its original volume. The rise in temperature is
process is (take, γ = 5/3)
(a) 4.11 × 104 J (b) − 4.11 × 104 J (a) 475°C (b) 150°C
. × 104 J
(c)114 (d) − 114
. × 104 J (c) 275°C (d) 402°C
7 One mole of an ideal gas goes from an initial state A to 13 A monoatomic gas ( γ = 5 / 3) at pressure p is suddenly
final state B via two processes. It first undergoes 1
compressed to th of its volume adiabatically. Then,
isothermal expansion from volumeV to 3V and then its 64
volume is reduced from 3V to V at constant pressure. pressure of gas is
The correct p-V diagram representing the two processes (a) 8 p (b) 42/3 p
is j CBSE AIPMT 2011 (c) 256 p (d) 1024 p
B A 14 A gas is compressed adiabatically till its temperature
is doubled. The ratio of its final volume to initial volume
A will be
(a) p (b) p B
(a)1/2 (b) more than 1/2
V V (c) less than 1/2 (d) between 1 and 2
V 3V V 3V
A A 15 One mole of an ideal gas at an initial temperature ofT K
does 6 R J of work adiabatically. If the ratio of specific
(c) p B (d) p B heats of this gas at constant pressure and at constant
volume is 5/3, the final temperature of gas will be
V
V 3V
V
V 3V (a) (T + 2.4) K (b) (T − 2.4) K
(c) (T + 4) K (d) (T − 4) K
8 In thermodynamic processes, which of the following
16 During an adiabatic process, the pressure of a gas is
statements is not true ? j
CBSE AIPMT 2009
found to be proportional to the cube of its temperature.
(a)In an adiabatic process the system is insulated from the Cp
surroundings The ratio of for the gas is
(b)In an isochoric process pressure remains constant
CV j
NEET 2013
(c)In an isothermal process the temperature remains 4 5 3
(a) (b) 2 (c) (d)
constant 3 3 2
(d) In an adiabatic process pV γ = constant 17 A monoatomic gas at a pressure p, having a volume V
9 During an isothermal expansion, a confined ideal gas expands isothermally to a volume 2V and then
does –150 J of work against its surroundings. This adiabatically to a volume 16V. The final pressure of the
implies that j
CBSE AIPMT 2011  5
gas is  take γ = 
(a) 300 J of heat has been added to the gas  3 j
CBSE AIPMT 2014
(b)no heat is transferred because the process is isothermal (a) 64p (b) 32p
(c) 150 J of heat has been added to the gas p
(d) 150 J of heat has been removed from the gas (c) (d) 16p
64
10 If ∆U and ∆W represent the increase in internal energy 18 Which of the following statements is correct for any
and work done by the system respectively in a thermodynamic system?
thermodynamical process, which of the following is true?
(a)The internal energy changes in all processes
j
CBSE AIPMT 2010
(b)Internal energy and entropy are state functions
(a) ∆U = − ∆W , in an adiabatic process (c)The change in entropy can never be zero
(b) ∆U = ∆W , in an isothermal process (d)The work done in an adiabatic process is always zero
(c) ∆U = ∆W , in an adiabatic process
(d) ∆U = − ∆W , in an isothermal process 19 We consider a thermodynamic system. If ∆U represents
11 A sample of perfect gas is compressed isothermally to the increase in its internal energy andW the work done
half its volume. If it is compressed adiabatically to the by the system, which of the following statements is true?
same volume, the final pressure of the gas will be (a) ∆U = − W in an adiabatic process j
CBSE AIPMT 2010
(a) more (b) less (c) same (b) ∆U = W in an isothermal process
(d)more or less depending on the initial temperature of the (c) ∆U = − W in an isothermal process
gas (d) ∆U = W in an adiabatic process
DAY FOURTEEN THERMODYNAMICS 143

20 3 moles of an ideal monoatomic gas performs a cycle as 26 Thermodynamic processes are indicated in the following
shown in figure. The gas temperaturesTA = 400 K, diagram j NEET 2017
TB = 800 K, TC = 2400 K, TD = 1200 K, work done by the P
gas is
p IV
f
B I III
C
II f
f 700 K
A f 500 K
D 300 K
V

T
Match the following :
(a) 10 kJ (b) 24 kJ (c) 30 kJ (d) 40 kJ Column-I Column-II
21 Figure below shows two paths that may be taken by a P. Process I a. Adiabatic
gas to go from a state A to a state C. Q. Process II b. Isobaric
B R. Process III c. Isochoric
6×104 C
p (in Pa) S. Process IV d. Isothermal
(a) P → a, Q → c, R → d, S → b
A (b) P → c, Q → a, R → d, S → b
2×104 (c) P → c, Q → d, R → b, S → a
(d) P → d, Q → b, R → a, S → c

2×10–3 4×10–3 27 Which of the following processes is reversible?


V(in m3 ) (a) Transfer of heat by radiation
(b) Electrical heating of a nichrome wire
In process AB, 400 J of heat is added to the system and (c) Transfer of heat by conduction
in process BC, 100 J of heat is added to the system. The (d) Isothermal compression
heat absorbed by the system in the process AC will be
28 A thermodynamic system is taken through the cycle
j
CBSE AIPMT 2015
PQRSP. The net work done by the system is
(a) 380 J (b) 500 J (c) 460 J (d) 300 J
22 In an isobaric process of an ideal gas. The ratio of heat
 Q S R
supplied and work done by the system i.e.    is 200 kPa
 W  
γ −1 γ
(a) (b) γ (c) (d) 1
γ γ −1 100 kPa Q
P
23 In thermodynamic process, which of the following
statements is not ture?
100 cc 300 cc
(a)In an adiabatic process, the system is insulated from the (a) 20 J (b) − 20 J (c) 400 J (d) − 374 J
surroundings
(b)In an isochoric process, pressure remains constant 29 A thermodynamics system undergoes cyclic process
(c)In an isothermal process, the temperature remains ABCDA as shown in figure. The work done by the system
constant in the cycle is j
CBSE-AIPMT 2014
(d) In an adiabatic process, pvr = constant
24 Two gram mole of a gas, which are kept at constant 3p0
C
B
temperature of 0°C, are compressed from 4 L to 1 L. The
work done will be p 2p E
0
(a) −6500 J (b) −3000 J (c) −5800 J (d) −6359 J
25 A sample of gas expands from volume V1 to V2 . The p0 D
A
amount of work done by the gas is greatest when the
expansion is V0 2V0
V
(a)adiabatic (b) isobaric
p0V0
(c)isothermal (d) equal in all three cases (a) p0V0 (b) 2 p0V0 c) (d) Zero
2
144 40 DAYS ~ NEET PHYSICS DAY FOURTEEN

30 A gas is taken through the cycle A → B → C → A, as 35 The efficiency of an ideal heat engine working between
shown in figure. What is the net work done by the gas? the freezing point and boiling point of water is
j NEET 2013 j NEET 2018

p(105 Pa) (a) 6.25% (b) 20% (c) 26.8% (d) 12.5 %
36 An ideal gas heat engine operates in Carnot cycle
7 between 227°C and 127°C. It absorbs 6 × 104 cal of heat
6 B
at higher temperature. Amount of heat converted to work
5
is
4
(a)2.4 × 104 cal (b) 6 × 104 cal
3
A (c)1.2 × 104 cal (d) 4.8 × 104 cal
2 C
1 37 A reversible engine takes in heat from a reservoir of heat
0 V(10–3 m3) at 527°C and gives it to the sink at 127°C. How many
2 4 6 8 calorie/s it shall take from the reservoir to do a work of
(a) 2000 J (b) 1000 J (c) Zero (d) − 2000 J 750 W.
31 A thermodynamic system is taken through the cycle (a) 257 cal s−1 (b) 357 cal s−1
ABCD as shown in figure. Heat rejected by the gas (c) 1500 cal s−1 (d) None of these
during the cycle is j CBSE AIPMT 2012
38 In refrigerator one removes heat from a lower
temperature and deposits to the surroundings at a higher
2p D C
temperature. In this process, mechanical work has to be
done, which is provided by an electrical motor. If the
Pressure

motor is of 1kW power and heat is transferred from −3°C


p to 27°C, find the heat taken out of the refrigerator per
A B
second assuming its efficiency is 50% of a perfect
engine.
V 3V
Volume (a) 12 kJ (b) 19 kJ (c) 10 kJ (d) 7 kJ
1 1
(a) 2 pV (b) 4 pV (c) pV (d) pV 39 A Carnot engine having an efficiency of as heat
2 10
32 One mole of an ideal diatomic gas undergoes a transition engine, is used as a refrigerator. If the work done on the
from A to B along a path AB as shown in the figure. system is 10 J, the amount of energy absorbed from the
reservoir at lower temperature is j
NEET 2017
A (a) 1 J (b) 90 J (c) 99 J (d) 100 J
5
40 If the coefficient of performance of a refrigerator is 5 and
P (in kPa)

2 6 operates at the room temperature (27°C), find the


B
temperature inside the refrigerator.
(a) − 23°C (b) −20°C (c) −15°C (d) −31°C
4 6 41 A refrigerator works between 4°C and 30°C. It is required
V (in m3) to remove 600 calories of heat every second in order to
The change in internal energy of the gas during the keep the temperature of the refrigerated space constant.
transition is j
CBSE AIPMT 2015 The power required is (Take, 1 cal = 4 . 2 Joules)
(a) 20 kJ (b) −20 kJ (c) 20 J (d) −12 kJ j
NEET 2016

33 A bread gives a boy 5000 cal. How much height he can (a) 23.65 W (b) 236.5 W
(c) 2365 W (d) 2.365 W
climb by using this energy, if his efficiency is 28%?
(Mass of the boy = 60 kg) 42 The temperature inside a refrigerator is t 2 °C and the
(a) 2.5 m (b) 5 m (c) 10 m (d) 15 m room temperature is t1 ° C. The amount of heat delivered
to the room for each joule of electrical energy consumed
34 A person of mass 60 kg wants to lose 5 kg by going up
ideally will be j
NEET 2016
and down a 10 m high stairs. Assume he burns twice as
t1 t1 + 273
much fat while going up than coming down. If 1 kg of fat is (a) (b)
t1 − t 2 t1 − t 2
burnt on expanding 7000 kilo calories, how many times t + 273 t1 + t 2
must be go up and down to reduce his weight by 5 kg? (c) 2 (d)
t1 − t 2 t1 + 273
(a) 10.5 × 103 (b) 24.3 × 103 (c) 16.3 × 103 (d) 9 × 103
DAY FOURTEEN THERMODYNAMICS 145

43 The coefficient of performance of a refrigerator is 5. If the 44 An ideal refrigerator has a freezer at a temperature of
temperature inside freezer is − 20° C, the temperature of − 13°C. The coefficient of performance of the engine is 5.
the surroundings to which it rejects heat is The temperature of the air (to which heat is rejected) will
j CBSE AIPMT 2015 be
(a) 31°C (b) 41°C (c) 11°C (d) 21°C (a) 325°C (b) 325 K (c) 39°C (d) 320°C

DAY PRACTICE SESSION 2

PROGRESSIVE QUESTIONS EXERCISE


1 One mole of oxygen is expanded from a volume 1 L to 6 A Carnot engine whose sink is at 300K has an efficiency
5 L at a constant temperatureT = 280 K. The change in of 40%. By how much should the temperature of source
internal energy is be increased so as to increase its efficiency by 50% of
(a) 0.22 kJ (b) 22 J (c) 0.11 kJ (d) 11 J original efficiency?
2 The heat absorbed by the system in going through the (a) 275 K (b) 325 K (c) 250 K (d) 380 K
process as shown in figure 7 In the given p-V diagram, the path (2) from A to B is
kPa
zig-zag path, but (1) is simple path. Then,
A
300
1
p
B

100

2
200 400 cc
Volume
V
(a) 100 J (b) 200 J (c) 31.4 J (d) 68.3 J (a)W1 = W2 (b) ∆U1 = ∆U 2
3 The p-V diagram of a system undergoing thermodynamic (c)W1 > W2 (d) Both (b) and (c) are true
transformation as shown in figure. The work done by the 8 Efficiency of a Carnot engine is 50% when temperature
system in going from a → b → c is 50 J and 20 cal heat of outlet is 500 K. In order to increase efficiency upto
is given to the system. The change in internal energy 60% keeping temperature of intake the same, what is
between a and c is temperature of outlet?
p c (a)200 K (b) 400 K (c) 600 K (d) 800 K
9 A gas is compressed isothermally to half its initial
volume. The same gas is compressed separately
a b through an adiabatic process until its volume is again
V reduced to half. Then j
NEET 2016
(a) 34 J (b) 70 J (c) 84 J (d) 134 J (a)compressing the gas through adiabatic process will
4 Consider a Carnot’s cycle operating betweenT1 = 500 K require more work to be done.
and T2 = 300 K producing 1 kJ of mechanical work per (b) compressing the gas isothermally or adiabatically will
require the same amount of work.
cycle. Find the heat transferred to the engine by the
(c) which of the case (whether compression through
reservoirs. isothermal or through adiabatic process) requires more
(a)1500 J (b) 1000 J work will depend upon the atomicity of the gas.
(c)2000 J (d) None of these (d) compressing the gas isothermally will require more work
1 to be done.
5 An engine has an efficiency of . When the temperature of
6 10 An ideal gas is compressed to half its initial volume by
sink is reduced by 62°C, its efficiency is doubled. means of several process. Which of the process results in
Temperature of the source is the maximum work done on the gas? j CBSE AIPMT 2015
(a) 124°C (b) 37°C (c) 62°C (d) 99°C (a) Adiabatic (b) Isobaric (c) Isochoric (d) Isothermal
146 40 DAYS ~ NEET PHYSICS DAY FOURTEEN

11 A steam engine whose source is at 400 K, takes 200 cal in the cycle is 5J, the work done by the gas in the
of heat and rejects 150 cal to the sink. What is the process C → A is
temperature of the sink?
V (m3)
(a)800 K (b) 400 K C B
2
(c)300 K (d) None of these
12 1 mm 3 of a gas is compressed at 1 atmosphere
pressure and temperature 27°C to 627°C. What is the
final pressure under adiabatic condition? 1
A
(Take, γ for the gas = 1. 5) 10p (N/m2)
(a)27 × 105 N/m 2 (b) 80 × 105 N/m 2
(c) 36 × 105 N/m 2 (d) 56 × 105 N/m 2 (a) − 5 J (b) −10 J (c) −15 J (d) −20 J
1
13 When 1 g of water at 0°C and 1 × 10 N/m pressure is
5 2
17 A Carnot engine, having an efficiency of η = as heat
3 10
converted into ice of volume 1.091 cm , the external
engine is used as a refrigerator. If the work done on the
work done will be
system is 10 J, the amount of energy absorbed from the
(a) 0.0091 J (b) 0.0182 J (c) − 0.0091 J (d) −0.0182 J
reservoir at lower temperature is j CBSE AIPMT 2015

14 The temperature-entropy diagram of a reversible engine (a) 100 J (b) 99 J (c) 90 J (d) 1 J
cycle is given in the figure. Its efficiency is
18 A Carnot engine has an efficiency 50% when its sink is at
a temperature of 27°C. The temperature of the source is
2T0
(a) 273°C (b) 300°C (c) 327°C (d) 373°C
19 The volume (V ) of a monoatomic gas varies with its
temperature (T ), as shown in the graph. The ratio of work
T0
done by the gas, to the heat absorbed by it, when it
undergoes a change from state A to state B, is j NEET 2018
S0 2S0
1 2 1 1 V
(a) (b) (c) (d)
3 3 2 4
15 Work done in the given p-V diagram in the cyclic process is
B
p A
(2p, V) (2p, 2V )
O T
1 2 2 2
(a) (b) (c) (d)
3 3 5 7

( p, V ) ( p, 2V ) 20 A sample of 0.1 g of water at 100° C and normal pressure


V (1.013 × 105 Nm −2 ) requires 54 cal of heat energy to
pV
(a) pV (b) 2pV (c) (d) 3pV convert to steam at 100° C. If the volume of the steam
2
produced is 167.1 cc, the change in internal energy of
16 An ideal gas is taken through the cycle. A → B → C → A the sample, is j
NEET 2018
as shown in the figure. If the hot heat supplied to the gas (a) 42.2 J (b) 208.7 J (c) 104.3 J (d) 84.5 J

ANSWERS
SESSION 1 1 (b) 2 (c) 3 (d) 4 (a) 5 (d) 6 (d) 7 (b) 8 (b) 9 (c) 10 (a)
11 (a) 12 (d) 13 (d) 14 (b) 15 (d) 16 (d) 17 (c) 18 (b) 19 (c) 20 (b)
21 (c) 22 (c) 23 (b) 24 (d) 25 (d) 26 (b) 27 (d) 28 (b) 29 (d) 30 (b)
31 (a) 32 (b) 33 (c) 34 (c) 35 (c) 36 (c) 37 (b) 38 (b) 39 (b) 40 (a)
41 (b) 42 (b) 43 (a) 44 (c)
SESSION 2 1 (c) 2 (c) 3 (a) 4 (a) 5 (d) 6 (c) 7 (d) 8 (b) 9 (c) 10 (a)
11 (c) 12 (a) 13 (a) 14 (a) 15 (a) 16 (a) 17 (c) 18 (c) 19 (c) 20 (b)
DAY FOURTEEN THERMODYNAMICS 147

Hints and Explanations


SESSION 1 7 According to question first gas goes 14 In adiabatic process,
from volume V to 3V and after this
1 Work done against the atmospheric pV γ = constant
pressure volume is reduced from 3V to V at
constant pressure. In isothermal Since, pV = RT
W = p∆V expansion p-V curve is rectangular Hence, T V γ − 1 = k
= 1.013 × 105 × (1671 − 1) × 10−6 J hyperbola. ⇒ T1V1γ − 1 = T2V2γ − 1
= 1.013 × 167 J
8 For an adiabatic process, there should γ −1
 V2  T  1
not be any exchange of heat between ⇒   =  1 =
1.013 × 167  V1   T2  2
= = 40 cal
cal ~ the system and its surroundings. All
4.2 1
walls of the container must be perfectly V2  1  γ − 1
Water absorbs 540 cal of heat out of ⇒ = 
insulated. In adiabatic changes, gases V1  2 
which 40 cal is used in doing work obey Poisson’s law, i.e., pV γ = constant. 1
against the atmospheric pressure. Hence, ratio will be more than .
Therefore, work done to overcome the In an isochoric process, volume remains 2
molecular attraction constant and for isobaric process, 15 Work done in an adiabatic process
= 540 − 40 = 500 pressure remains constant. nR
W = (T i − T f ) …(i)
γ −1
2 We know that, ∆Q = ∆U + p∆V 9 According to first law of
thermodynamics Here, W = 6 R J, n = 1 mol,
n × C p × ∆T = ∆U + R ∆T 5
5 ∆U = ∆Q + ∆W R = 8.31 Jmol −1 K −1 , γ = , T i = T K
1 × R × ∆T = ∆U + R ∆T For isothermal process, ∆U = 0
3
2 Substituting given values in Eq. (i), we
⇒ ∆U = 1.5 R ∆T So, ∆Q = − ∆W
∆U 3 get
Therefore, = 1.5 = Given, ∆W = − 150 J
R
R ∆T 2 So, ∆Q = + 150 J ∴ 6R = (T − T f )
 5 − 1
When Q is positive, the heat is added to  
3 ∆Q = ∆U + ∆W 3 
the gas.
= U 2 − U 1 + ∆W ⇒ 6R =
3R
(T − T f )
Here, ∆Q = − 30 J, ∆W = − 22 J 10 From first law of thermodynamics 2
U 1 = 20 J ∆Q = ∆U + ∆W ⇒ T − Tf = 4
Hence, For adiabatic process, ∆Q = 0 ∴ T f = (T − 4) K
U 2 = ∆Q − ∆ W + U 1 Q ∆Q = 0 16 According to question, p ∝ T 3
= − 30 J − (− 22 J ) + 20 J So, ∆U = − ∆W
= 12 J pV = nRT and p ∝ T 3
11 For isothermal compression, p ∝ ( pV )3
4 Work done by the = p∆ V Also,
p1V1
p2 = ⇒ p V = constant
2 3
= − 50 (10 − 4) = − 300 J V2
Using dQ = dU + dW ⇒ pV 3 /2 = constant …(i)
For adiabatic compression,
⇒ dU = dQ − dW γ Q pV = constant …(ii)
V 
= 300 + 100 = 400 J p2 = p1  1  Comparing eqn (i) and (ii)
Increased by 400 J  V2  3
∴ γ =
5 According to first law of Cp 2
Since, γ = > 1, hence p2 will be
thermodynamics CV 17 In isothermal expansion
∆Q = ∆U + ∆W or ∆U = ∆Q − ∆W more as compared to that of isothermal pV = p2 2V
compression. p
∆U = change in internal energy ⇒ p2 =
2
Q = heat given to system 12 We know that, pV γ = constant
In adiabatic expansion p2V2γ = p3V3γ
W = work done TV γ − 1 = constant
∴ ∆U = 2 × 4.2 × 1000 − 500 p
2 /3 ⇒ (2V )5/3 = p3 (16V )5/3
300 =  
= 8400 − 500 = 7900 J 8 2
T2
 27  5/3 5/3
6 Work done in an isothermal process p  2V  p  1 p
⇒ T2 = 675 K = 402° C ⇒ p3 =   = ×  =
2  16V  2  8 64
V 
W = 23026
. nRT log 10  2  13 p1V1γ = p2V2γ
 V1  18 Internal energy does not change in
 10  5/3 isothermal process. ∆S can be zero for
= p2  
= 23026 × 10 × 83
. × 600 log 10 V
.   pV 5/3
[Q p1 = p] adiabatic process. Work done in
 100   64 
adiabatic process may be non-zero.
= − 11.4 × 104 J ⇒ p2 = 1024 p
148 40 DAYS ~ NEET PHYSICS DAY FOURTEEN

19 From first law of thermodynamics, 25 The p -V diagram for isobaric, 28


33 Mgh = × 5000 × 4.2
∆Q = ∆U + ∆W isothermal and adiabatic process of an 100
For adiabatic process, ∆Q = 0 ideal gas is shown in graph below. 28 × 4.2 × 50 ~
⇒ height h = = 10 m
∆U = − ∆W 60 × 9.8
20 W BC = 3R(TC − T B ) 34 5 × 7000 × 103 × 4.2 J
W AB = WCD = 0 = 60 × 15 × 10 × N
Because, the processes are isochoric 21 × 7 × 106 147
N = = × 103
W DA = 3 R (T A − T D ) 9000 9
∴ Total work done = 163
. × 103 times.
W BC + W DA = 3R(T A + TC − T B − T D )
= 3R (400 + 2400 − 800 − 1200) In thermodynamics, for same change in 35 Efficiency of an ideal heat engine is
volume, the work done is maximum for given as
= 2400 R = 24 kJ T2
the curve having maximum area η = 1−
21 As initial and final points are same, so enclosed with the volume axis. T1
∆U A → B → C = ∆U A → C where, T1 is the temperature of the
A → B is isochoric process 26 In isochoric process, the curve is source and T2 is the temperature of the
parallel to y-axis because volume is
dW A → B = 0 sink.
constant.Isobaric is parallel to x-axis T1 = 100 + 273 = 373 K
dQ A → B = dU A → B = 400 J
because pressure is constant. Along the T2 = 0 + 273 = 273 K
B → C is isobaric process curve, it will be isothermal because 273 373 − 273
dQ B → C = dU B → C + dW B → C temperature is constant. Q η = 1− =
100 J = dU B → C + 6 × 104 (2 × 10−3 ) 373 373
So, P → c ⇒ Q → a ⇒ R → d 100
⇒ S→ b = = 0.268
100 J = dU B → C + 12 × 10 373
dU B → C = 100 − 120 = − 20 J 27 Isothermal compression is reversible. η% = 0.268 × 100 = 26.8%
So, ∆U A → B → C = ∆U A → C Q T
∆U A → B = ∆U B → C = dQ A → C 28 Work done = area enclosed in indicator 36 From the relation, 2 = 2
diagram PQRSP Q 1 T1
− dW A → C
= ∆p ∆V = ( p p − ps ) (VQ − V P ) Given, Q 1 = 6 × 104 cal,
400 − 20 = dQ A → C
 2 × 10+4 × 2 × 10−3 + 1  = (1 × 105 − 2 × 105 ) (3 × 10−4 −1 × 10−4 ) T1 = 227 + 273 = 500 K
− 
2  = − 20 J T2 = 127 + 273 = 400 K
 −3 +4
 × 2 × 10 × 4 × 10  Q2 400
29 Work done by the system ∴ =
380 J = dQ A → C − (40 + 40) 6 × 104 500
W = Area of BCE + Area of ADE
dQ A → C = 380 + 80 = 460 J 4
= − W0 + W0 = 0 ⇒ Q2 = × 6 × 104 = 4.8 × 104 cal
22 From ideal gas equation, 5
pV = nRT …(i)
30 Net work done area enclosed in p-V Now, heat converted to work
curve, i.e. ∆ABC = Q 1 − Q 2 = 6.0 × 104 − 4.8 × 104
where, p is pressure, V is the volume, R 1
= × 5 × 10−3 × 4 × 105 J = 1.2 × 104 cal
is the gas constant, T is the absolute 2
temperature.
= 103 J = 1000 J 37 Efficiency of a Carnot’s engine is given
For isobaric process, by
Q = nC p ∆T …(ii) 31 For given cyclic process, ∆U = 0 W T  T 
η= = 1 − 2 ⇒ W = 1 − 2  Q
W = p∆V …(iii) ∴ ∆Q = ∆W Q T1  T1 
where, C p is specific heat at constant Also, W = area enclosed by the curve Substituting
pressure. = AB × AD = ( p − 2 p ) (3V − V ) T2 = 127 + 273 = 400 K
From Eqs. (i), (ii) and (iii), we get = − p × 2V
nC p ∆T C p T1 = 527 + 273 = 800 K
Q γ ∴Heat rejected = 2 pV
= = = W 2 × 750
W nR∆T R γ −1 Q = = = 3571
.
32 Since, ∆U = nC V dT  T2  4.2
1 − 
23 In an isochoric process, pressure  T1 
= n 
5R 
remains constant.  (T − T A )
 2  B Cal/s
24 We know that work done in an 5nR  p B V B p V  38 η = 1 − 270 = 1 . Efficiency of
isothermal process is =  − A A
2  nR nR  300 10
V 1
W = 2303
. µRT log 10 2 5 refrigerator = 0.5 η =
V1 = ( p B V B − p AV A ) 20
2
1 If Q is the heat/s transferred at higher
W = 2303
. × 2 × 8.4 × 273 log 10 5
4 = (2 × 103 × 6 − 5 × 103 × 4) temperature, then
W
=
1
or
2 Q 20
W = 2303
. × 2 × 8.4 × 273
5
× (log 10 1 − log 10 4) = (−8 × 103 ) Q = 20 W = 20 kJ
2 and heat removed from lower
[Q log 10 4 = 0.6021]
= − 20 kJ temperature = 19 kJ.
∴ W ≈ −6359 J
DAY FOURTEEN THERMODYNAMICS 149

39 Consider schematic diagram for a 42 For a refrigerator, we know that 2 Heat absorbed = πr 2
Carnot engine as shown below. Q1 Q1 T1
= = = 3.14 × (100 × 103 ) × (100 × 10−6 )
W Q 1 − Q 2 T1 − T2
T2 Low temperature = 31.4 J
reservoir where,
q2 Q 1 = amount of heat delivered to the 3 Heat added to system is given by
room ∆Q = ∆U + ∆W
W E W = electrical energy consumed Here, ∆Q = 20 cal = 84 Jand ∆W = 50 J
T1 = room temperature = t 1 + 273 ⇒ ∆U = 84 − 50 = 34 J
q1 T2 = temperature of sink = t 2 + 273 4
Q 2 T2 3
= = ,Q 1 − Q 2 = 103 J
High temperature Q1 t 1 + 273 Q 1 T1 5
T1 ∴ =
reservoir 1 t 1 + 273 − (t 2 + 273)
Q 1  1 −
3
 = 10 J
3
In case of engine, engine efficiency t + 273 
⇒ Q1 = 1 5
work W t1 − t2
= = 5
heat absorbed q1 ⇒ Q1 = × 103 J = 2500 J,
43 Given, coefficient of performance of a 2

W
=
1

10 J
=
1 refrigerator, β = 5 Q 2 = 1500 J
q1 10 q1 10 Temperature of surface, i.e. inside
freezer, 5 Efficiency of engine is given by
or q1 = 100 J
T2 = − 20°C = − 20 + 273 = 253 K T2
When this engine is reversed, it takes in η=1−
Temperature of surrounding, i.e. heat T1
work W and heat q2 from cold reservoir
rejected outside T1 = ? T2 1 5
and ejects 100 J of heat to hot reservoir. ∴ =1− η=1− = …(i)
T2 253 T1 6 6
∴ W + q2 = q1 ⇒ 10 + q2 = 100 So, β= ⇒ 5=
or q2 = 90 J T1 − T2 T1 − 253 In other case,
⇒ 5 T1 − 1265 = 253 T2 − 62 2 2
40
Q2
= 5, Q 2 = 5W, Q 1 = 6 W = 1 − η = 1 − = …(ii)
W ⇒ 5 T1 = 1518 T1 6 3
1518
T2 5 T
= = 2 ,T2 = 250 K = − 23° C T1 = = 303.6 K Using Eq. (i), we get
T1 6 300 5 2 2 6 1
T1 = 303.6 − 273 = 31° C T2 − 62 = T1 = × T2 or T2 = 62
3 3 5 5
41 Given, temperature of source, 44 Given that, the temperature of freezer, ∴ T2 = 310 K
T1 = 30°C T2 = − 13° C ⇒ T2 = − 13 + 273 = 260 K 6 6
Here, T1 = T2 = × 310
= 30 + 273 Coefficient of performance, β = 5 5 5
= 303 K The coefficient of performance is = 372 K = 372 − 273 = 99° C
Temperature of sink, defined as,
T2 = 4° C = 4 + 273 = 277 K 6 The efficiency of Carnot engine
T2 260
β= or 5 = T2
As, we know that T1 − T2 T1 − 260 η=1− .
Q 1 T1 T1
= 260
Q 2 T2 ∴ T1 − 260 = or T1 − 260 = 52 Here, T1 is the temperature of source
5 and T2 is the temperature of sink.
Q2 + W T
⇒ = 1 [Q W = Q 1 − Q 2 ] or T1 = (52 + 260) K = 312 K 40
Q2 T2 = (312 − 273°)C As given, η = 40% = = 0.4
100
where Q 2 is the amount of heat drawn ⇒ T1 = 39°C and T2 = 300 K
from the sink (at T2 ), W is workdone on 300 300
working substance, Q 1 is amount of heat SESSION 2 So, 0.4 = 1 − ⇒ T1 = = 500 K
T1 1 − 0.4
rejected to source (at room temperature V2  δU  dV
T1 ). 1 ∆Q = ∫V 1
 
 δV 
Let temperature of the source be
increased by x, then efficiency becomes
⇒ WT2 + T2Q 2 = T1Q 2
⇒ WT2 = T1Q 2 − T2Q 2 From second law of thermodynamics η′ = 40% + 50% of η
⇒ WT2 = Q 2 (T1 − T2 ) 40 50
 ∂U  = T  ∂S  − p = + × 0.4
T      100 100
 ∂V  T  ∂V  T
⇒ W = Q 2  1 − 1 = 0.4 + 0.5 × 0.4 = 0.6
T 
 2  ∂p 
= T   − p, Hence, 0.6 = 1 −
300
 ∂T  V
W = 600 × 4.2 ×  − 1
303 500 + x

 277  RT a 300
p= − ⇒ = 0.4
V − b V2
W = 600 × 4 .2 × 
26  500 + x

 277  ∂p  ∂U 
T  and  ∴ x = 750 − 500 = 250 K
RT a
W = 236.5 Joules  =  =
 ∂T  V V − b  ∂V  T V 2
Work done 7 Work done depends upon the path
Power = Given, V1 = 1 L, V2 = 5 L followed by process
Time
W 236.5 1 1 So, W1 ≠ W2
= = ∆Q = a  −  = 0.11 kJ
t 1  V1 V2  Since, change in internal energy
= 236.5 W depends upon the initial and final state,
150 40 DAYS ~ NEET PHYSICS DAY FOURTEEN

it does not depend upon the path Q 1 − Q 2 200 − 150 1 T1 − T2 T


11 η= = = 18 η= =1− 2
followed by the process, Q1 200 4 T1 T1
∆U 1 = ∆U 2 1
η= =1−
T2
⇒ T1 =
T2
=
300
= 600 K = 327 ° C
The area under p-V diagram gives the 4 T1 1 − η 1/2
work done by the gas. Here, path (1) is This gives
19 According to the given graph,
greater than path (2), so W1 > W2 . 3 3
T2 = T1 = × 400 K = 300 K
Hence, option (d) is true. 4 4 V
T 1 500
8 η = 1− 2 ⇒ = 1− 12 Given, p1 = 10 N/m , 5 2
B
T1 2 T1
T1 = 27 + 273 = 300 K, VB
500 1
= …(i) T2 = 627 + 273 = 900 K, γ = 1.5 A
T1 2
Tγ VA
60 T′ T′ 2 For adiabatic change = = constant
⇒ = 1− 2 ⇒ 2 = …(ii) p γ −1
100 T1 T1 5 1 /2 3 /2
O TA TB T
1 /2 3 /2
 p2  T   p 
= 
900 
On dividing Eq. (i) by Eq. (ii), we get   =  2 ⇒ 2   Volume (V ) ∝ Temperature (T )
500 5  p1   T1   105   300 
= or
V
= constant
T2′ 4 ∴ p2 = 27 × 105 N/m T
∴ T2′ = 400 K Thus, the process is isobaric.
13 It is an isothermal process. Hence, work
9 The solution of this question can be done ∴ Work done, ∆W = p∆V
understood by plotting a p-V graph for = p(V2 − V1 ) = 1 × 105(1.091 − 1) × 10−6
the compression of a gas isothermally = nR∆T = nR(T B − T A ) …(i)
= 0.0091 J Heat absorbed, ∆Q = nC p ∆T
and adiabatically simultaneously to half
of its initial volume. i.e. 14 Q 1 = T 0S 0 + 1 T 0S 0 = 3 T 0S 0 = nC p (T B − T A ) …(ii)
2 2 γR  2
Q 2 = T 0S 0 and Q 3 = 0 As, C p = ,  where, γ = 1 + 
Adiabatic curve γ −1  f
T For a monoatomic gas, f = 3
Isothermal curve
C p =  R + R  = R
3 5
p 2T0 ⇒
 2  2

Q3 Q1 Substituting the value of C p in Eq. (ii),


V/2 V we get
∆Q = n  R  (T B − T A )
Volume V 5
T0
Since, the isothermal curve is less Q2 2 
steeper than the adiabatic curve. So, ∆W nR (T B − T A ) 2
Hence, = =
area under the p-V curve for adiabatic S0 2S0 ∆Q  5  5
n  R  (T B − T A )
process has more magnitude than W Q − Q2 Q 2 1 2 
isothermal curve. Hence, work done in η= = 1 = 1− 2 = 1− =
Q1 Q1 Q1 3 3 20 According to the question,
adiabatic process will be more than in
isothermal process. 15 Work done = Area of closed pV diagram Heat spent during the conversion of
10 Given, ideal gas is compressed to half = (2V − V ) × (2 p − p ) = pV sample of water at 100°C to steam is,
V ∆Q = 54 cal = 54 × 4.18 J
its initial volume i.e. V 0 = 16 For cyclic process, total work done
2 = 225.72 J
= W AB + W BC + WCA
∆W AB = p∆V Normal pressure, p = 1.013 × 10 5 Nm −2
= 10(2 − 1) = 10 J Net work done during the conversion
and ∆W AB = 0 (as = constant) would be given as
Ad

Iso
From First law of thermodynamics, ∆W = p∆V = p[V steam − V water ]
ia

P th
ba

er
m ∆Q = ∆U + ∆W
. cc = 167.1 × 10 −6 m 3
tic

al Here, V steam = 1671


∆U = 0
Isobaric (process ABCA is cyclic) V water = 0.1 g = 0.1cc = 0.1 × 10 −6 m 3
∆Q = ∆W AB + ∆W BC + ∆WCA ∴∆W = 1.013 × 10 5 [(167.1 − 0.1) × 10 −6 ]
V/2 V
5 = 10 + 0 + ∆WCA = 1.013 × 167 × 10 −1 = 16.917 J
V
The isochoric process is one in which ∆WCA = −5J
Now, by the first law of
volume is kept constant, meaning that Q 2 T2 Θ + Ω Τ2
17 = ⇒ 1 = thermodynamics, ∆Q = ∆U + ∆W
work done by the system will be zero. Q 1 T1 Θ1 Τ1
i.e. W isochoric = 0 where, ∆U is the change in internal
Q 1 + 10 10
As we know, work done on the gas = ⇒ = energy of the sample. ⇒ ∆U = ∆Q − ∆W
Q1 9
Area under curve, i.e. Substituting the values in the above
Wadiabatic > W isothermal > W isobaric ⇒ 9Q 1 + 90 = 10Q 1 equation, we get
⇒ Q 1 = 90 J ∆U = 225.72 − 16.917 = 208.7 J
DAY FIFTEEN

Unit Test 2
(General Properties of Matter)
1 A thick rope of density 1. 5 × 103 kgm −3 and Young’s 8 The work done in increasing the size of a soap film from
modulus 5 × 106 Nm −2 , 8 m in length when hung from the 10 cm × 6 cm to 10 cm × 11 cm is 3 × 10−4 J. The surface
ceiling of room, the increase in its length due to its own tension of the film is
weight is (a) 1.5 × 10−2 Nm −1 (b) 3.0 × 10−2 Nm −1
(a) 9.6 × 10 m−5
(b) 19. 2 × 10 −7
m (c) 6. 0 × 10−2 Nm −1 . × 10−2 Nm −1
(d) 110
(c) 9.6 × 10−2 m (d) 9.6 m 9 Figure below shows a cyclic
2 A vessel contains oil (density = 0.8 g cm −3 ) over mercury process abcda. If ∆Q be the heat p c
(density = 13.6 g cm −3 ). A homogeneous sphere floats supplied to the system, ∆U be the
b
with half of its volume immersed in mercury and the other change in internal energy and
half in oil. The density of material of the sphere in (gcm −3 ) ∆W be the work done by the d
is system, then which of the a
(a) 3.3 (b) 6.4 (c) 7.2 (d) 2.8 following relations is correct? V
3 Two wires A and B are of the same material. Their (a) dQ − dU = 0 (b) dQ − dW = 0
lengths are in the ratio 1 : 2 and the diameter are in the (c) dU + dW = 0 (d) None of these
ratio 2 : 1. If they are pulled by the same force, their 10 How much force is required to produce an increase of
increase in length will be in the ratio 0.2% in the length of a brass wire of diameter 0.6 mm?
(a) 2 : 1 (b) 1 : 4 (c) 1 : 8 (d) 8 : 1 [Young’s modulus for brass = 0.9 × 1011 Nm −2 ]
4 Two capillary tubes P and Q are dipped in water. The (a) Nearly 17 N (b) Nearly 51 N
height of water level in capillary P is 2/3 to the height in Q (c) Nearly 34 N (d) Nearly 68 N
capillary. The ratio of their diameters is 11 If two drops of same radius are falling through air with a
(a) 2 : 3 (b) 3 : 2 (c) 3 : 4 (d) 4 : 3 velocity of 5 cm s −1. If the two drops coalescence to form
5 For a given material the Young’s modulus is 2.4 times one drop, the terminal velocity of the drop is
that of rigidity modulus. Its poission’s ratio is (a) 2.5 cms −1 (b) 10 cms −1
(a) 2.4 (b) 1.2 (c) 5 2 cms −1 (d) 5 × 41/ 3 cms −1
(c) 0.4 (d) 0.2 12 The pressure and density of a diatomic gas ( γ = 7/5)
6 The work done when n smaller equal size spherical changes adiabatically from ( p, ρ ) to ( p′ , ρ′ ). If ρ′ /ρ = 32,
drops combine to form a bigger size single spherical then ( p′ /p ) should be
drop of water is proportional to (a) 1/128 (b) 32 (c) 128 (d) None of these

13 One mole of monoatomic gas  γ =  is mixed with one


−2 /3 −1/ 3 5
(a) (n − 1) (b) (n − 1)
(c) (n1/ 3 − 1) (d) (n 4 / 3 − 1)  3
 7
7 For adiabatic expansion of a monoatomic perfect gas, mole of diatomic gas  γ =  . What will be the value of γ
 5
the volume increases by 2.4%. What is the percentage
for the mixture?
decrease in pressure?
(a) 1.5 (b) 1.54 (c) 1.4 (d) 1.45
(a) 2.4% (b) 4.0% (c) 4.8% (d) 7.1%
152 40 DAYS ~ NEET PHYSICS DAY FIFTEEN

14 70 cal of heat is required to increase, the temperature of 24 Newton’s law of cooling is the special case of Stefan’s
2 moles of an ideal gas from 30°C to 35°C at constant law, when
pressure. The amount of heat required to increase the (a)temperature of body is high
temperature of the same gas through the same (b)temperature of surroundings is high
temperature range (30°C to 35°C) at constant volume will (c)temperature difference of body and surroundings is
be [(Gas constant R = 2 cal/(mol −1K −1)]. small
(a) 30 cal (b) 50 cal (c) 70 cal (d) 90 cal (d)temperature difference of body and surroundings is high
15 125 ml of gas A at 0.60 atm and 150 ml of gas B at 25 It takes 10 min to cool a liquid from 61°C to 59°C. If room
0.80 atm pressure at same temperature is filled in a temperature is 30°C, then time taken in cooling from 51°C
vessel of 1 L volume. What will be total pressure of to 49°C is
mixture at the same temperature? (a) 10 min (b) 11 min
(a) 0.195 atm (b) 0.212 atm (c) 13 min (d) 15 min
(c) 0.120 atm (d) 0.140 atm 26 A planet of radius r radiates heat at a rate proportional to
16 At constant temperature, on increasing the pressure of a the fourth power of its surface temperatureT . The
gas by 5%, its volume will decrease by temperature of the planet is such that, this loss is exactly
(a) 5% (b) 5.26% compensated by the heat gained from the sun. If d is the
(c) 4.26% (d) 4.76% mean distance of planet from the sun, then the
17 The density of hydrogen is 0.09 kgm −3 . What is its root temperature T of planet is directly proportional to
mean temperature velocity at NTP ? (a) d (b) d 2 /3
1 1
(a)10/3 ms −1 (b) 10 /3 kms −1 (c) (d) 2 / 3
d d
(c) 10/ 3 ms −1 (d) (10/3) cm−1
27 A body when fully immersed in a liquid of specific gravity
18 The gas having average speed four times as that of SO 2 1.2 weight 44 gwt. The same body when fully immersed
(molecular mass = 64) is in water weight 50 gwt. The mass of the body is
(a)He (molecular mass 4) (b) O 2 (molecular mass 32)
(a) 36 g (b) 48 g
(c)H2 (molecular mass 2) (d) CH4 (molecular mass 16)
(c) 64 g (d) 80 g
19 Which of the graphs shown in the figure correctly 28 How much should the pressure of the gas be increased
represents the cooling of a body due to radiations?
to decrease the volume by 10% at a constant
(a) I (b) II (c) III (d) None of these temperature?
20 The temperature of a given mass is increased from 27°C (a) 10% (b) 9.5%
to 327°C. The rms velocity of the molecules increases by (c) 11.11% (d) 5.11%
(a) 2 times (b) 2 times (c) 2 2 times (d) 4 times 29 A black body at 227°C radiates heat at a rate of 7 cal/cm 2 s.
21 The quantity of heat required to heat 1 mole of a At a temperature of 727°C, the rate of heat radiated in
monoatomic gas through 1 K at constant pressure is the same units will be
(a) 3.5 R (b) 2.5 R (a) 112 (b) 105
(c) 1.5 R (d) None of these (c) 101 (d) 89
22 The radiant energy from the sun, incident normally at the 30 The figure show the graph of pressure versus density for
surface of the earth is 20 kcal m −2 min –1. What would an ideal gas at two temperatures T1 and T2 , then
have been the radiant energy, incident normally on the
earth, if the sun had a temperature, twice of the present T1
one?
(a)160 kcal m−2 min −1 (b) 40 kcal m−2 min −1
(c)320 kcal m−2 min −1 (d) 80 kcal m−2 min −1 p
T2
23 A solid ball of density half that of water falls freely under
gravity from a height of 19.6 m and then enters water.
Neglecting air resistance and viscosity effect in water,
the depth upto which the ball will go is ρ
( take, g = 9.8 m /s 2 ) (a)T1 > T2 (b)T1 = T2
(a) 19.6 m (b) 28.4 m (c) 9.8 m (d) 14.7 m (c)T1 < T2 (d) None of these
DAY FIFTEEN UNIT TEST 5 (GENERAL PROPERTIES OF MATTER) 153

31 One end of a conducting rod is maintained at Direction (Q. Nos. 36-40) In each of the following questions
temperature 50°C and at the other end ice is melting at a statement of Assertion is given followed by a corresponding
0°C. The rate of melting of ice is doubled, if statement of Reason just below it. Of the statements mark the
correct answer as
(a) the temperature is made 200°C and the area of
(a) If both Assertion and Reason are true and the Reason is
cross-section of the rod is doubled
(b) the temperature is made 100°C and length of the rod is the correct explanation of the Assertion
made four times (b) If both Assertion and Reason are true but the Reason is
(c) area of cross-section of rod is halved and length is not the correct explanation of the Assertion
doubled (c) If Assertion is true but Reason is false
(d) the temperature is made 100°C and area of (d) If both Assertion and Reason are false
cross-section of rod and length both are doubled
36 Assertion (A) If length of a rod is doubled the breaking
32 A small steel ball falls through a syrup at a constant
load remains unchanged.
speed of 1.0 m/s. If the steel ball is pulled upwards with
a force equal to twice its effective weight, how fast will it Reason (R) Breaking load is equal to the elastic limit.
move upward?
37 Assertion (A) The number of degrees of freedom of
(a) 1.0 m/s (b) 2.0 m/s (c) 0.5 m/s (d) zero triatomic molecules is 6.
33 A solid ball of density ρ1 and radius r falls vertically Reason (R) Triatomic molecules have three translational
through a liquid of density ρ 2 . Assume that the viscous
degrees of freedom and three rotational degrees of
force acting on the ball is F = krv , where k is a constant
freedom.
and v its velocity. What is the terminal velocity of the ball?
4 πgr 2 (ρ1 − ρ2 ) 2 πr (ρ1 − ρ2 ) 38 Assertion (A) If the same load is attached to lead and
(a) (b)
3k 3gk rubber wires of the same cross-sectional area, the strain
2 πg (ρ1 + ρ2 ) of lead is very much less than that of rubber.
(c) (d) None of these
3gr 2k Reason (R) Lead is more elastic than rubber.
34 A block of mass M is suspended from a wire of length L, 39 Assertion (A) The coefficient of real expansion of liquid is
area of cross-section A and Young’s modulus Y. The independent of the nature of container.
elastic potential energy stored in the wire is
Reason (R) γa = γr + γv
1 M 2g 2L 1 Mg 1 M 2g 2 A 1 MgY
(a) (b) (c) (d) where, γa = coefficient of apparent expansion,
2 AY 2 ALY 2 YL 2 AL
γr = coefficient of real expansion
35 22320 cal of heat is supplied to 100g of ice at 0° C. If the
latent heat of fusion of ice is 80 cal g −1 and latent heat of and γv = coefficient of expansion of vessel.
vaporisation of water is 540 cal g −1, the final amount of
40 Assertion (A) Thermodynamic processes in nature are
water, thus obtained and its temperature respectively are
irreversible.
(a) 8g, 100°C (b) 100g, 100°C
(c) 92g, 100°C (d) 82g, 100°C Reason (R) Dissipative effects cannot be eliminated.

ANSWERS
1 (c) 2 (c) 3 (c) 4 (b) 5 (d) 6 (c) 7 (b) 8 (b) 9 (b) 10 (b)
11 (d) 12 (c) 13 (a) 14 (b) 15 (a) 16 (d) 17 (b) 18 (a) 19 (c) 20 (a)
21 (b) 22 (c) 23 (a) 24 (c) 25 (d) 26 (c) 27 (d) 28 (c) 29 (c) 30 (a)
31 (d) 32 (a) 33 (a) 34 (a) 35 (a) 36 (c) 37 (a) 38 (a) 39 (d) 40 (a)
154 40 DAYS ~ NEET PHYSICS DAY FIFTEEN

Hints and Explanations


1 Weight of the rope F l 150 × 0.80
10 ∴ Y = pB = = 0120
. atm
W = 8 × A × 1.5 × 10 × 10 N 3 A ∆ l 1000
∆ l 2 l 1000 So, p = 0.075 + 0120 = 0195
where, A is area of cross-section. The Here, = 0.2% = ⇒ = . . atm
weight can be assumed to act at half the l 1000 ∆ l 2
length of the rope. i.e. L = 4 m. 2 16 According to Boyle’s law,
2
 0.6 ×10−3 
A = πr 2 = π   = 3.14 × 
d ⇒ p1V1 = p2V2 …(i)
Hence, ∆L =
W
×
L
 2 
 2  p1 21
A Y Here, p2 = p1 + or p2 = p1
2 20 20
= 8 × 1.5 × 103 × 10 × 4 / 5 × 106  0.6 × 10−3 
0.9 × 1011 × 3.14 ×   ×2 Substituting it in Eq. (i), we get
= 9.6 × 10−2 m  2  20
∴F = V2 = V1 or V2 = 0.9524 V1
2 Let the volume of the sphere be V . Then, 1000 21
weight of mercury displaced + weight of = 50868 × 10−3 = 50.868 ≈ 51 N Thus, V2 is 95.24% of V1 .
oil displaced = Weight of the sphere. In other words, volume decreases
V V 11 Let r = Radius of small drop by 4.76%.
× 13. 6 × g + × 0.8 × g = V × ρ × g R = Radius of big drop
2 2 1 /2
17 We know that, c = 
Then, volume of 2 small drops = Volume 3p 
This gives ρ = 7.2 g cm −3 of one big drop, i.e.

 ρ 
.
4F 4 4
3 Y= F × L = 2 × L πR 3 = 2 × πr 3 Here, p = 1.0 × 105 Pa ≅ 105 Pa
A∆L πd ∆L 3 3
1 /2
∆L 1 F1 d 22 ⇒ R = 21 /3 r  3 × 105  10
Hence, = ×
L Y
× 1 × 2 2 2
Therefore,c =   = kms −1
∆L 2 F2 d 12 L 2 Y1 Now,
v1
=
r
=
r  0.09  3
2 v2 R2 (21 /3 r )2
×   ×   ×
F 1 1 Y 1 1 v M SO2
= = Therefore, v 2 = 5 × 41 /3 cms −1 18 v av ∝ ⇒ Gas =
F  2  2 Y 8 M v SO2 M Gas
4 h = 2T cos θ/ρrg , 12 For adiabatic process, 4 64
⇒ = ⇒ MGas = 4 i.e. gas is He.
1 r hQ h 3 pV γ = constant 1 M Gas
r ∝ ⇒ P = = =
2 7
h rQ hP h 2 ⇒ pρ− γ = constant and γ =
3 5 19 Temperature falls with time. As rate of
For diatomic gas, loss of heat by radiation is directly
Y
5 Y = 2η (1 + σ ) or 1 + σ = proportional to T 4 , so rate of loss of
2η pρ− γ = p ′ ρ′ − γ
heat and time have inverse relation. So,
Y 2.4
⇒ σ = −1 = − 1 = 0. 2  ρ′ 
−γ 7/5 the graph will be represented by
=  
2η p 1 1
2 ⇒ =  = hyperbolic curve III.
p′  ρ   32  128
−1 /3
6 Here, R = n r ⇒ r = n
1 /3
R 20 The rms speed of a gas molecule is
⇒ p ′ = 128 p
and W = (4 πr × n − 4 πR2 )T
2
given by
13 For monoatomic gas, C V = 3 / 2 R
= ( 4 πR n 2 1 /3
− 4 πR )T
2
3RT c T1
For diatomic gas, C V = 5/ 2 R c= or c ∝ T ⇒ 1 =
= 4 πR2 (n1 /3 − 1)T M c2 T2
Hence, for mixture,
W ∝ (n1 /3 − 1) 3 / 2 R + 5/ 2 R Substituting
CV = =2R
γ 2 T1 = 27° C = 27 + 273 = 300 K
7 pV = K
Therefore, C p for mixture = 3 R T2 = 327° C = 327 + 273 = 600 K, we get
Therefore, ∆ pV γ + pγ V γ − 1 ∆V = 0 [QC p − C V = R] c1 1
= ⇒ c2 = 2 c1
Hence, ∆p/ p = − γ ∆V /V . C p 3R c2 2
⇒ γ = = = 1.5
For monoatomic gas, γ = 5/3 CV 2R
∆p 5 21 Molar specific heat at constant pressure
Hence, = × 2.4% = 4.0%
p 3 14 Q = nC p ∆T is given by Q = m C p ∆T
8 Change in area 70 = 2 C p (35 − 30) ⇒ C p = 7 cal where, m = mass of one mole gas,
= 220 × 10−4 − 120 × 10−4 = 10−2 m2 CV = C p − R = 7 − 2 = 5 C p = specific heat of a one mole
W Q = n C V ∆T = 2 × 5 × (35 − 30) of gas at constant pressure,
Surface tension, T =
Change in area = 50 cal ∆T = change in temperature.
3 × 10 −4
15 Here, p = p A + p B [Dalton’s law] On substituting m = 1 mol,
= = 3.0 × 10−2 Nm −1
−2 5
10 125 × 0.60 C p (for monoatomic gas) = R,
and pA = = 0.075 atm 2
9 According to first law of 1000
thermodynamics, dQ = dU + dW ∆T = 1 K
 V 
In cyclic process, dU = 0 Q p x = i × p i  5
V we get, Q= R = 2.5R
⇒ dQ − dW = 0  f  2
DAY FIFTEEN UNIT TEST 5 (GENERAL PROPERTIES OF MATTER) 155

9 MgL 1
22 E ∝T4 [Stefan’s law] Therefore, volume becomes = V 34 ∆ l = FL = ; U = k (∆l )2
10 AY AY 2
4 4
E 1  T1  20  T1  Pressure =
10
p where, k =
YA
⇒ =  ⇒ = 
E 2  T2  E2  2T1  9 L
 10 p  2
M2g2L
 − p 1  YA   MgL 
⇒ E2 = 320 kcal m −2 min −1 ∴U =     =
% increase =  9  × 100 = 11.11% 2  L   AY  2 AY
 p 
23 Velocity of ball on reaching the water   35 Heat required to convert ice to water at
surface is v = 2gh = 2 × 9.8 × 19.6 100° C,
29 According to Stefan’s law E = σT 4

= 19.6 m/s so initially water is at 0°C.


∴ 7 = σ (227 + 273)4 = σ × (500)4
If ρ be the density of ball, then density So, Q 1 = Heat required to convert 100 g
and x = σ (727 + 273)4 = σ × (1000)4 ice to water
of water 2ρ.
If a is the retardation of the ball in x (1000)4 = mL = 100 × 80 = 8000 Cal
Hence, = = 16 Q 2 = Heat required to convert the
water, then 7 (500)4
upward thrust − weight temperature 0°C to 100°C for 100 g
a= ⇒ x = 16 × 7 = 112 cal/cm2 s water
mass
v (2ρ) g − v ρg 30 As, pV = nRT = m RT = mδ∆T = 100 × 1 × (100°−0° )
= M = 100 × 100 = 10,000 Cal

where, M = molecular weight. So, Q = Q 1 + Q 2 = 18000 Cal
⇒ a = g = 9.8 m/s2 p RT p RT p
⇒ = ⇒ = ⇒ ∝T So remaining heat for vaporisation
So, the depth is m /V M ρ M ρ = 22320 − 18000 = 4320 Cal
v 2 19.6 × 19.6 (Qfor a given gas, M is constant) So amount of water vaporised by
d = = = 19.6 m
2a 2 × 9.8 Temperature is directly proportional to remaining heat,
the slope of p - ρ graph. m × L vapor = 4320
24 Newton’s law of cooling is valid when 4320
So, T1 > T2 . m= = 8 g(steam)
the temperature difference of body and
540
surroundings is small. In such a case, 31 Rate of melting of ice ∝ rate of heat So the remaining water after
transfer 
dQ 
Newton’s law can be derived from  vaporisation is,
 dt 
Stefan’s law. m = 100 − 8 = 92 g at 100°C
dQ temperature difference
θ1 − θ2
Further, = So, option (c) is correct.
 θ + θ2   l 
25 Q =K  1 − θ0  dt
 
t  2   kA  36 Breaking load depends on the area of
61° − 59° cross-section and is independent of
 61° + 59°  dQ (temperature difference)
=K  − 30°  or ∝ A length of rod.
10  2  dt l
Hence, breaking load
1 If temperature difference, A and l are all
K = dQ = breaking stress × cross-sectional area.
150 doubled, then and hence rate of
Again dt 37 A non-linear molecule can rotate about
melting of ice will be doubled. any of three coordinate axis. Hence, it
51° − 49° 1  51° + 49° 
= − 30° 
150  32 w e = effective weight has 6 degrees of freedom: 3 translational
t 2 
and 3 rotational.
t = 15min kv = w e ...(i)
In equilibrium 38 On applying load on lead and rubber
26 Energy received per second 2w e − w e = kv ′ ...(ii) wires of same cross-sectional area, the
Q Qr 2 strain produced in lead is much less
= πr 2 = From Eqs. (i) and (ii), we get
4 πd 2 4d 2 than rubber wires. From the relation,
v ′ = v = 1.0 m/s
Energy emitted per second = 4 πr 2 (σT 4 ) Stress
kV 2 We Young’s modulus (Y) =
For equilibrium to exist, Strain
Qr 2 The Young’s modulus of elasticity is
= 4 πr 2 ( σ T 4 )
4d 2 V V greater for lead wires.
1 1
⇒ T4∝ or T ∝ 39 Coefficient of real expansion,
d2 d We
γr = γa + γv
We kv
27 Balancing equation, Here, γ v is coefficient of cubical
w = mg – Vδg 33 Net force on the ball = 0 expansion of vessel (container).
Hence, 44 = m − 1.2V …(i) (when terminal velocity is attained) Thus, real expansion coefficient
50 = m − V …(ii) Hence, depends on nature of vessel.
On solving Eqs. (i) and (ii), m = 80 g Weight = upthrust + viscous force 40 The thermodynamic process is
4 3 4 irreversible, as there always occurs a
28 At constant temperature, pV = constant ∴ πr ρ1 g = πr 3ρ2 g + krv T
3 3 loss of energy due to energy spent in
As volume decreased by 10%, so 4 πgr 2 working against the dissipative force
∴ vT = (ρ1 − ρ2 )
pressure has to increase to keep the 3k which is not recovered back.
product of pV constant.
DAY SIXTEEN

Oscillations
Learning & Revision for the Day
u Periodic Motion u Simple Pendulum
u Simple Harmonic Motion u Oscillations of a Spring
u Force and Energy in SHM u Free, Damped, Forced and Resonant Vibrations

Periodic Motion
A motion which repeats itself over a regular interval of time is called a periodic motion.
A periodic motion in which a body moves back and forth repeatedly about a fixed point
(called mean position) is called oscillatory or vibratory motion.

Displacement as a Function of Time


In a periodic motion, each displacement value is repeated after a regular interval of
time, displacement can be represented as a function of time y = f (t ).

Periodic Function
A function which repeats its value after a fix interval of time is called a periodic
function. y(t ) = y(t + T )
where, T is the period of the function.
Trigonometric functions sin θ and cos θ are simplest periodic functions having period of
2π.
PREP
Simple Harmonic Motion MIRROR
Your Personal Preparation Indicator
Simple Harmonic Motion (SHM) is that type of periodic motion in which the particle
moves to and fro or back and forth about a fixed point under a restoring force, whose u No. of Questions in Exercises (x)—
magnitude is directly proportional to its displacement u No. of Questions Attempted (y)—
i.e. F ∝ x or F = − kx u No. of Correct Questions (z)—
where, k is a positive constant called the force constant or spring factor and x is (Without referring Explanations)
displacement.
u Accuracy Level (z / y × 100)—
d2 y
Differential equations of SHM, for linear SHM, + ω2 y = 0, u Prep Level (z / x × 100)—
dt 2
d2 θ In order to expect good rank in NEET, your
for angular SHM, + ω2 θ = 0 Accuracy Level should be above 85 & Prep
dt 2 Level should be above 75.
DAY SIXTEEN OSCILLATIONS 157

Time Period The time taken by a particle to complete one


Terms Related to SHM
l

oscillation is called time period. It is denoted by T.


The few important terms related to simple harmonic motion ∴ Time period of SHM,
are given as
2π | y| Displacement
Displacement The a
Displacement T = = 2π = 2π
ω
l
| a| Acceleration
displacement of a particle
T/2 T Frequency and Angular Frequency It is defined as the
executing SHM is, in l

O Time
general, expressed as number of oscillations executed by body per second. SI
y = A sin (ωt + φ). –a unit of frequency is hertz.
where, A is the amplitude of Angular frequency of a body executing periodic motion is
 2π  equal to product of frequency of the body with factor 2π.
SHM, ω is the angular frequency  where ω = = 2 πν and
 T  Angular frequency, ω = 2 πn.
φ is the initial phase of SHM. However, displacement may
also be expressed as x = A cos (ωt + φ). Force and Energy in SHM
l
Amplitude The maximum displacement on either side of l
Force For an object executing SHM, a force always acts on
mean position is called amplitude of SHM.
it, which tries to bring it in mean position, i.e. it is always
l
Velocity The velocity of a particle executing SHM at an directed towards mean position.
instant is defined as the time rate of change of its The equation of motion, F = ma ,
displacement at that instant.
∴ F = − mω2 x [Q a = − ω2 x ]
dy
Velocity, = v = ω A2 − y 2 
dt k 
= − kx Qω =
At the mean position ( y = 0), during its motion  m 
v = Aω = vmax and at the extreme positions ( y = ± A), v = 0.
Here, negative sign shows that direction of force is always
Velocity amplitude = v max = Aω opposite to the direction of displacement.
Velocity l
Energy If a particle of mass m is executing SHM, then at a
T
displacement x from the mean position, the particle
T/2 possesses potential and kinetic energy.
O Time At any displacement x,
1 1
Potential energy, U = m ω2 x2 = k x2
2 2
l
Acceleration The acceleration of a particle executing SHM 1 1
Kinetic energy, K = m ω ( A − x2 ) = k ( A2 − x2 )
2 2
at an instant is defined as the time rate of change of 2 2
velocity at that instant. 1
Total energy, E = U + K = m ω A = 2 π 2 mν2 A2
2 2
d2 y 2
Acceleration, = a = − ω2 y
dt 2 If there is no friction, the total mechanical energy,
The acceleration is also a variable. E = K + U, of the system always remains constant even
At the mean position ( y = 0), acceleration a = 0 and at the though K and U change.
extreme position ( y = ± A), the acceleration is amax = − Aω2 .
∴ Acceleration amplitude amax = Aω2 Simple Pendulum
Acceleration A simple pendulum, in practice, consists of a heavy but small
sized metallic bob suspended by a light, inextensible and
flexible string. The motion of a simple pendulum is simple
T/2 T harmonic for very small angular displacement (α) whose time
O Time period and frequency are given by
l 1 g
T = 2π and ν =
g 2π l
l
Phase Phase is that physical quantity which tells about the
position and direction of motion of any particle at any where, l is the effective length of the string and g is
moment. It is denoted by φ. acceleration due to gravity.
l
Phase Difference If two particles perform S.H.M and their
l
If a pendulum of length l at temperature θ° C has a time
equations are period T, then on increasing the temperature by ∆θ° C its
time period changes to T × ∆T ,
y1 = a sin(ωt + φ 1) and y2 = a sin(ωt + φ2 )
∆T 1
phase difference ∆φ = (ωt + φ2 ) − (ωt + φ 1) = φ2 − φ 1 where, = α ∆θ
T 2
158 40 DAYS ~ NEET PHYSICS DAY SIXTEEN

where, α is the temperature coefficient of expansion of the l


The graph between T 2 and g is a rectangular hyperbola.
string. y
l
A second’s pendulum is a pendulum whose time period is
2s. At a place where g = 9.8 ms −2 , the length of a second’s T2
pendulum is 0.9929 m (or 1 m approx).
l
If the bob of a pendulum (having density ρ) is made to
oscillate in a non-viscous fluid of density σ, then it can be O x
g
shown that the new period is
l
T = 2π
 σ Oscillations of a Spring
g 1 − 
 ρ
If the mass is once pulled, so as to stretch the spring and is
l
If a pendulum is in a lift or in some other carriage moving then released, then a restoring force acts on it which
vertically with an acceleration a, then the effective value of continuously tries to restore its mean position.
the acceleration due to gravity becomes (g ± a) and hence Elongation
l
T = 2π x1
(g ± a)
Here, positive sign is taken for an upward accelerated Natural position
motion and negative sign for a downward accelerated
Compression
motion.
x2
l
If a pendulum is made to oscillate in a freely falling lift or
an orbiting satellite, then the effective value of g is zero and Restoring force F = − k l ,
hence, the time period of the pendulum will be infinity and where k is force constant and l is the change in length of the
therefore pendulum will not oscillate at all. spring.
l
If the pendulum bob of mass m has a charge q and is Here, x 1 = x2 = l
oscillating in an electrical field E acting vertically l
The spring pendulum oscillates simple harmonically
l
downwards, then T = 2π having time period and frequency given by
 qE 
g ±  T = 2π
m and ν = 1 k
 m
k 2π m
l
If pendulum of charge q is oscillating in an electric field E l
If the spring is not light but has a mass ms , then
acting horizontally, then m + 1 / 3 ms
T = 2π
l k
T = 2π
q2 E 2 l
If two masses m1 and m2 ,
g2 + m1 m2
m2 connected by a spring, are made
to oscillate on a horizontal
l
The graphs l–T and l–T 2 intersect at T = 1 s. surface, then its period will be
y µ
T = 2π
T = 1s k
l–T m1m2
where, µ = = reduced mass of the system.
m1 + m2

l–T2
O x
Combination of Springs
l
If two springs of spring constants k 1 and k2 are
l
The graph between T 2 and 1/g is a straight line. joined in series (horizontally and vertically), then
y
their equivalent spring constant k s is given K1
1 1 1
by = +
T2 k s k 1 k2
k 1k2 K2
⇒ ks =
k 1 + k2
m
O x m m(k 1 + k2 )
1/g ∴ T = 2π = 2π (a)
ks k 1k2
DAY SIXTEEN OSCILLATIONS 159

l
If the two springs of spring constants k 1 and k2 are joined where, v is the velocity of the oscillator and b is a damping
in parallel as shown, then their equivalent spring constant constant.
k p = k 1 + k2 The displacement of the oscillator is given by
hence, T = 2π
m
= 2π
m x(t ) = Ae −bt / 2 m cos(ω′t + φ)
kp (k 1 + k2 )
where, ω′ = the angular frequency
k b2
ω′ = −
m 4 m2
k1 k2
The mechanical energy E of the oscillator is given by
1
E (t ) = kA2e −bt / m
2
m A

ing
mp
(b ) x

a
all d
t

Sm
Free, Damped, Forced and Larger
damping
Resonant Vibrations ω0 ω
(a) (b)
Some of the vibrations are described below

Free Vibrations Forced Vibrations


If a body, capable of oscillating is slightly displaced from its The vibrations in which a body oscillates under the effect of an
position of equilibrium and then released, it starts external periodic force, whose frequency is different from the
oscillating with a frequency of its own. natural frequency of the oscillating body, are called forced
Such oscillations are called free vibrations.The frequency vibrations.
with which a body oscillates is called the natural frequency In forced vibrations, the oscillating body vibrates with the
and is given by frequency of the external force and amplitude of oscillations is
1 k generally small.
ν0 =
2π m
Here, a body continues to oscillate with a constant Resonant Vibrations
amplitude and a fixed frequency. It is a special case of forced vibrations in which the frequency
of external force is exactly same as the natural frequency of the
oscillator.
Damped Vibrations
As a result, the oscillating body begins to vibrate with a large
The oscillations in which the amplitude decreases gradually
amplitude leading to the phenomenon of resonance to occur.
with the passage of time are called damped vibrations.
Resonant vibrations play a very important role in music and in
Damping force, Fd = − bv tuning of station/channel in a radio/TV, etc.
DAY PRACTICE SESSION 1

FOUNDATION QUESTIONS EXERCISE


1 An elastic ball is dropped from a certain height and Y
returns to the same height after elastic collision on the
floor. What is the nature of repeated motion of the ball? P (t = 0)
(a) Simple harmonic, oscillatory and periodic Q (t = 30s)
(b) Simple harmonic, oscillatory but not periodic B
(c) Simple harmonic, periodic, but not oscillatory
(d) Oscillatory, periodic but not simple harmonic X
O
2 The displacement of a particle varies with time according
to the relation y = a sin ω t + b cos ω t
(a)The motion is oscillatory but not SHM
(b)The motion is SHM with amplitude a + b 2 πt  πt
(a)x (t ) = B sin   (b) x (t ) = B cos 
(c)The motion is SHM with amplitude a 2 + b 2  30   15 
πt π πt π
(d)The motion is SHM with amplitude a 2 + b 2 (c)x (t ) = B sin  +  (d) x (t ) = B cos + 
 15 2  15 2
3 The graph between restoring force and time in case of
SHM is 8 The displacement of a particle is represented by the
π 
(a)a straight line (b) a circle equation y = 3 cos  − 2ω t  . The motion of the
(c)a parabola (d) a sine curve 4 
particle is
4 Out of the following functions representing motion of a
particle which represents SHM? j
CBSE AIPMT 2011 (a) simple harmonic with period 2 π /ω
(b) simple harmonic with period π /ω
I. y = sin ωt − cos ωt II. y = sin3 ωt (c) periodic but not simple harmonic
(d) non-periodic
 3π 
III. y = 5 cos  − 3 ωt  IV. y = 1 + ωt + ω 2t 2 9 When two displacements represented by y1 = a sin (ωt )
 4 
and y 2 = b cos (ωt ) are superimposed the motion is
(a) Only (IV) does not represent SHM j
CBSE AIPMT 2015
(b) (I) and (III)
(c) (I) and (II) (a) not a simple harmonic
a
(d) Only (I) (b) simple harmonic with amplitude
b
5 The displacement of SHM is given by (c) simple harmonic with amplitude a 2 + b 2
y = 5 cos (10 t + 0.6) (a + b)
(d) simple harmonic with amplitude
What is the initial phase of the SHM? 2
(a)5 rad (b) 10 rad 10 The displacement of a particle along the X -axis is given
(c)0.6 rad (d) None of these
by x = a sin2 ω t . The motion of the particle corresponds
6 A particle executes simple harmonic oscillation with an to j
CBSE AIPMT 2010
amplitude a. The period of oscillation isT . The minimum ω
(a)simple harmonic motion of frequency
time taken by the particle to travel half of the amplitude π
from the equilibrium position is 3ω
(b)simple harmonic motion of frequency
T T 2π
(a) (b)
4 8 (c)non-simple harmonic motion
ω
T T (d)simple harmonic motion of frequency
(c) (d) 2π
12 2
11 The time period of a SHM is 16 s. It starts its motion from
7 Figure shows the circular motion of a particle. The radius
the equilibrium position. After 2 s its velocity is π ms − 1.
of the circle, the period, sense of revolution and the initial
What is its displacement amplitude?
position are indicated on the figure. The simple harmonic
motion of the x-projection of the radius vector of the (a) 2 m (b) 2 2 m
(c) 4 2 m (d) 8 2 m
rotating particle P is
DAY SIXTEEN OSCILLATIONS 161

12 The maximum velocity of a simple harmonic motion Which one of the following graphs shows
 π correctly the variation a with t ? j CBSE AIPMT 2014
represented by y = 3 sin 100t +  is given by
 6
3π π a a
(a) 300 (b) (c) 100 (d)
6 6 (a) (b) O T
O T t t
13 A particle is executing SHM along a straight line. Its
velocities at distances x1 and x 2 from the mean position
are v1 and v 2 , respectively. Its time period is
j CBSE AIPMT 2015 a a
(c) (d) O
x12 + x 22 x 22 − x12 O T t T t
(a) 2 π (b) 2 π
v12 + v 22 v12 − v 22
v12 + v 22 v12 − v 22 Here, a = acceleration at time t
(c) 2 π (d) 2 π
x12 + x 22 x12 − x 22 T = Time period
14 A simple pendulum performs simple harmonic motion 20 Which one of the following equations of motion
about x = 0 with an amplitude a and time period T. The represents simple harmonic motion? j CBSE AIPMT 2009
a
speed of the pendulum at x = will be (a) Acceleration = − k0 x + k1x 2
2 j CBSE AIPMT 2009
(b) Acceleration = − k (x + a)
πa 3 πa 3 π 2a πa 3 (c) Acceleration = k (x + a)
(a) (b) (c) (d)
2T T T T (d) Acceleration = kx
(where, k, k0 , k1 and a are all positive.)
15 The amplitude of a particle executing SHM is 4 cm. At the
21 Two simple harmonic motions of angular frequency 100
mean position, the speed of the particle is 16 cm/s. The
rad s −1 and 1000 rad s −1 have the same displacement
distance of the particle from the mean position at which
the speed of the particle becomes 8 3 cm/s will be amplitude. The ratio of their maximum accelerations is
(a) 2 3 cm (b) 3 cm (c) 1cm (d) 2 cm (a) 1 : 10 (b) 1 : 102 (c) 1 : 103 (d) 1 : 104

16 If at any time the displacement of a simple pendulum be 22 A system is subjected to two SHMs given by
0.02 m, then its acceleration is 2 ms . What is the−2 y1 = 6 cos ω t and y 2 = 8 cos ω t
angular speed of the pendulum at that instant? The resultant amplitude of SHM is given by
(a) 100 rads −1 (b) 10 rads −1 (c) 1 rads −1 (d) 0.1 rads −1 (a) 2 (b) 10 (c) 14 (d) 20

17 The x-t graph of a particle undergoing simple harmonic 23 A pendulum is hung from the roof of a sufficiently high
motion is shown below. The acceleration of the particle at building and is moving freely to and fro like a simple
t = 4 / 3 s is harmonic oscillator. The acceleration of the bob of the
pendulum is 20 m / s 2 at a distance of 5 m from the mean
1 position. The time period of oscillation is j
NEET 2018
x (cm)

(a) 2 s (b) π s (c) 2 π s (d) 1 s


4 8 12 t (s) 24 A particle executes linear simple harmonic motion with
an amplitude of 3 cm. When the particle is at 2 cm from
–1 the mean position, the magnitude of its velocity is equal
3 2 − π2 to that of its acceleration. Then, its time period in
(a) π cm/s 2 (b) cm/s 2
32 32 seconds is j
NEET 2017
π2 3 2
(c) cm/s 2 (d) − π cm/s 2 (a)
5
(b)
5
32 32 π 2π
18 A particle moves such that its acceleration, a is given by 4π 2π
(c) (d)
a = − bx , where x is the displacement from equilibrium 5 3
position and b is a constant. The period of oscillation is 25 A particle executing simple harmonic motion of amplitude
(a) 2 π b (b) 2 π/ b (c) 2 π/b (d) 2 π/b 5 cm has maximum speed of 31.4 cms −1. The frequency of
19 The oscillation of a body on a smooth horizontal surface its oscillation is
is represented by the equation, x = A cos (ωt ) (a) 3 Hz (b) 2 Hz (c) 4 Hz (d) 1 Hz

where, x = displacement at time t 26 A particle is executing a simple harmonic motion. Its


maximum acceleration is α and maximum velocity is β.
ω = frequency of oscillation
Then, its time period of vibration will be j CBSE AIPMT 2015
162 40 DAYS ~ NEET PHYSICS DAY SIXTEEN

β2 α (a)Amplitude of B greater than A


(a) (b)
α2 β (b)Amplitude of B smaller than A
β2 2 πβ (c)Amplitude will be same
(c) (d) (d) None of the above
α α
36 The potential energy of a simple harmonic oscillator
27 A particle executes linear simple hormonic motion with
an amplitude of 2 cm. When the particle is at 1 cm from when the particle is half way to its end point is
the mean position the magnitude of its velocity is equal to
j CBSE AIPMT 2003
that of its acceleration. Then, its time period in second is 1 1 2 1
(a) E (b) E (c) E (d) E
4 2 3 8
1 2π 3
(a) (b) 2 π 3 (c) (d) (where, E is the total energy)
2π 3 3 2π
37 A body of mass m is attached to the lower end of a
28 A point performs simple harmonic oscillation of periodT spring whose upper end is fixed. The spring has
 π
and the equation of motion is given by x = a sin ω t +  . negligible mass. When the mass m is slightly pulled
 6
down and released, it oscillates with a time period of 3 s.
After the elapse of what fraction of the time period, the
When the mass m is increased by 1 kg, the time period
velocity of the point will be equal to half of its maximum
velocity ? of oscillations becomes 5 s. The value of m in kg is
T T T T
jNEET 2016
(a) (b) (c) (d) 3 4 16 9
8 6 3 12 (a) (b) (c) (d)
4 3 9 16
29 A particle executes SHM with a period of T second and
amplitude A metre. The shortest time it takes to reach 38 A mass of 0.5 kg moving with a speed of 1.5 ms −1 on a
A horizontal smooth surface, collides with a nearly
point metre from its mean position in seconds is
2 weightless spring of force constant k = 50 Nm −1. The
T T T maximum compression of the spring would be
(a)T (b) (c) (d)
4 8 16
30 A SHM has an amplitude A and time period T . The time
A
required by it to travel from x = A to x = is
2
(a)T /6 (b)T /4 (c)T / 3 (d)T / 2 (a) 0.15 m (b) 0.12 m (c) 1.5 m (d) 0.5 m
31 The total energy of SHM is E . What will be the kinetic 39 A block of mass m is suspended by different springs of
energy of the particle, when displacement is half of the force constant shown in figure.
amplitude?
3 E 3E E K
(a) E (b) (c) (d)
2 2 4 3
32 A body executing SHM has amplitude of 4 cm. What is
the distance at which the body has equal value of both k
KE and PE?
(a) 2 2 cm (b) 1/ 2 cm (c) 2 cm (d) 2 / 6 cm m
m
33 If the length of a pendulum is quadrupled, its time period (i) (ii)
is
(a) quadrupled (b) halved
2k
(c) doubled (d) unchanged
2k 2k m
34 A tunnel is bored along the diameter of the earth and a
stone is dropped into it. What happens to the stone? 2k
(a) It oscillates between the two ends of the tunnel m
(b) It comes to rest at the centre of the earth (iii) (iv)
(c) It will go out of the other end of the tunnel
(d) It will come to a permanent stop at the other end of the Let time period of oscillation in these four positions be
tunnel T1, T2 , T3 and T4 . Then, which of the following statement is
35 Two simple pendulums first of bob mass M1 and length correct?
L1, second of bob mass M 2 and length L2 . M1 = M 2 and (a)T1 = T2 = T4 (b)T1 = T2 and T3 = T4
L1 = 2L2 . If the vibrational energies of both are same. (c)T1 = T2 = T3 (d)T1 = T3 and T2 = T4
Then, which is correct? j
AIIMS 2010
DAY SIXTEEN OSCILLATIONS 163

40 A girl is sitting on the roof of a flat toy car of mass 6 kg. If (a) ν = ν1 + ν2 + ν3 (b) ν = ν1 + ν2 + ν3
no slipping takes place between car and the girl, then 1 1 1 1 1 1 1 1
(c) = + + (d) = + +
what should be the mass of the child in order to have ν ν1 ν2 ν3 ν ν1 ν2 ν3
period of system equal to 0.758 s?
44 One-fourth length of a spring of force constant k is cut
away. The force constant of the remaining spring will be
3 4
(a) k (b) k (c) k (d) 4k
k = 600 N/m 4 3
45 The period of oscillation of a mass M suspended from a
spring of negligible mass is T . If along with it another
(a) 9 kg (b) 2.74 kg (c) 6 kg (d) 7.28 kg mass M is also suspended, the period of oscillation will
41 Two springs of force constants k1 and k 2 are connected now be j CBSE AIPMT 2010

in series. The spring constant of the combination is T


(a)T (b) (c) 2T (d) 2T
k + k2 k + k2 kk 2
(a) k1 + k2 (b) 1 (c) 1 (d) 1 2
2 k1k2 k1 + k2 46 A spring is vibrating with frequency n under some mass.
If it is cut into two equal parts and same mass is
42 As shown in figure, a simple harmonic k
k suspended, then the new frequency is
motion oscillator having identical four
springs has time period. (a) n/ 2 (b) n (c) n 2 (d) n / 2
k
k 47 The amplitude of a damped oscillator becomes half in
m 1
(a)T = 2 π (b) m 1 min. The amplitude after 3 min will be times the
4k X
m
T = 2π original, where X is
2k
m 2m (a) 2 × 3 (b) 2 3 (c) 3 2 (d) 3 × 2 2
(c)T = 2 π (d)T = 2 π
k k 48 The damping force on an oscillator is directly
proportional to the velocity. The units of the constant of
43 When a string is divided into three segments of lengths
proportionality are j
CBSE AIPMT 2012
l1, l2 and l3 , the fundamental frequencies of these three
segments are ν1, ν 2 and ν 3 , respectively. The original (a) kg ms −1 (b) kg ms −2
(c) kg s −1 (d) kg s
fundamental frequency (ν) of the string is
j
CBSE AIPMT 2012

DAY PRACTICE SESSION 2

PROGRESSIVE QUESTIONS EXERCISE


1 A mass of 2.0 kg is put on a flat pan attached to a If they act on a particle simultaneously, the trajectory of
vertical spring fixed on the ground as shown in the the particle is
figure. The mass of the spring and the pan is negligible. y2 x2 4x 2
(a) + =1 (b) y 2 = (a 2 − x 2 )
When pressed slightly and released the mass executes a a2 4a2 a2
simple harmonic motion. The spring constant is
(c)y 2 = 2a x (d) None of these
200 Nm −1. What should be the minimum amplitude ofmthe
motion, so that the mass gets detached from the pan? 3 A tunnel is made along a chord inside the earth and a
(take, g = 10 ms − 2 ) ball is released in it. What will be the time period of
(a) 8.0 cm oscillation of the ball?
(b) 10.0 cm R R 2R R
(a) 2 π (b) 2 π (c) 2 π (d) π
(c) Any value less than 12.0 cm 2g g g g
(d) 4.0 cm
4 A cylindrical piston of mass M slides smoothly inside a
2 The equations of two linear SHM’s are long cylinder closed at one end, enclosing a certain
x = a sin ωt , along X -axis mass of gas. The cylinder is kept with its axis horizontal.
If the piston is disturbed from its equilibrium position, it
y = a sin 2 ωt , along Y-axis oscillates simple harmonically. Its period is
164 40 DAYS ~ NEET PHYSICS DAY SIXTEEN

h 9 The mass M shown in the figure oscillates in simple


p harmonic motion with amplitude A. The amplitude of the
M point P is
A

k1 k2
Mh MA M
(a)T = 2 π (b)T = 2 π P
pA ph
M k1A k2 A
(c)T = 2 π (d) None of these (a) (b)
pAh k2 k1
kA k A
5 From the ceiling of a train, a pendulum of length l is (c) 1 (d) 2
k1 + k2 k1 + k2
suspended. The train is moving with an acceleration a 0
on horizontal surface. The period of oscillation of the 10 What will be the force constant of the spring system as
pendulum is shown in the figure?
l l
(a)T = 2 π (b)T = 2 π
g a02 + g2
K1 K1
l l
(c)T = π (d)T = π
a02 + g 2 g

6 A wire of length l, area of cross-section A and K2


Young’s modulus of elasticity Y is suspended
from the roof of a building. A block of mass m is m
attached at lower end of the wire. If the block is −1
displaced from its mean position and released, k1  1 1
(a) + k2 (b)  + 
then the block starts oscillating. Time period of 2 2
 1k k 2
−1
these oscillations will be 1 1 2 1
(c) + (d)  + 
Al AY 2k1 k2  1
k k1
(a) 2 π (b) 2 π
mY ml
ml m 11 In the given figure, the spring has a force constant k. The
(c) 2 π (d) 2 π
YA YAl pulley is light and smooth the spring and string are light.
The suspended block has a mass m. If the block is
7 On a smooth inclined plane a body of mass M is
slightly displaced from its equilibrium position and then
attached between two springs. The other ends of the
released. The period of its vertical oscillation is
springs are fixed to firm supports. If each spring has a
force constant k, the period of oscillation of the body is
(assuming the spring as massless)
k
k
M

k
m x
θ

M 2M
(a) 2 π (b) 2 π m m
2k k (a) 2 π (b) 4 π
k k
M sin θ 2M sin θ
(c) 2 π (d) 2 π k k
2k k (c) 2 π (d) 4 π
m m
8 A rectangular block of mass m and area of cross-section 12 The particle executing SHM has a kinetic energy
A floats in a liquid of density ρ. If it is given a small K 0 cos 2 ωt . The maximum values of the potential energy
vertical displacement from equilibrium it undergoes and the total energy respectively are
oscillation with a time periodT . Then, K0
1 1 1 (a)0 and 2K 0 (b) and K 0
(a)T ∝ ρ (b)T ∝ (c)T ∝ (d)T ∝ 2
A ρ m (c)K 0 and 2K 0 (d) K 0 and K 0
DAY SIXTEEN OSCILLATIONS 165

13 A load of mass m falls from a height h on 15 The potential energy of a long spring when stretched by
to the scale pan hung from a spring as 2 cm is U. If the spring is stretched by 8 cm, the potential
k
shown in figure. If the spring constant is k energy stored in it is
and mass of the scale pan is zero and U
(a) 4U (b) 8 U (c) 16 U (d)
the mass m does not bounce relative to 4
the pan, then the amplitude of vibration is m 16 A simple harmonic oscillator consists of a particle of mass
mg m and an ideal spring with spring constant k. The particle
(a) h
k oscillates with a time periodT . The spring is cut into two
mg 2hk
(b) 1+ equal parts. If one part oscillates with the same particle,
k mg
the time period will be j AIIMS 2012

mg mg 1 + 2hk
(c) + T T
k k mg (a) 2T (b) 2T (c) (d)
2 2
(d) None of these
17 A particle moves with simple hormonic motion in a
14 The angular frequency and amplitude of a simple
straight line. In first τ s, after starting from rest it travels a
pendulum are ω and A, respectively. At the
distance a and in next τ s it travel 2a, in same direction,
displacement y from the mean position, the kinetic
then
energy is K and potential energy is U. What is the ratio of
(a) Amplitude of motion is τa
K/U ?
(b) Time period of oscillation is 8τ
(a)MA 2ω2 sin2 ωt (b) MA 2ω2 cos2 ωt (c) Amplitude of motion is 4a
(c)(A − y ) /y
2 2 2
(d) y 2 /(A 2 − y 2 ) (d) Time period of oscillation is 6τ

ANSWERS
SESSION 1 1 (d) 2 (d) 3 (d) 4 (b) 5 (c) 6 (c) 7 (a) 8 (b) 9 (c) 10 (c)
11 (d) 12 (a) 13 (b) 14 (d) 15 (d) 16 (b) 17 (d) 18 (b) 19 (c) 20 (b)
21 (b) 22 (c) 23 (b) 24 (c) 25 (d) 26 (d) 27 (c) 28 (d) 29 (c) 30 (a)
31 (c) 32 (a) 33 (c) 34 (a) 35 (b) 36 (a) 37 (d) 38 (a) 39 (b) 40 (b)
41 (d) 42 (c) 43 (c) 44 (b) 45 (d) 46 (c) 47 (b) 48 (c)
SESSION 2 1 (b) 2 (b) 3 (b) 4 (a) 5 (b) 6 (c) 7 (a) 8 (b) 9 (d) 10 (b)
11 (b) 12 (d) 13 (b) 14 (c) 15 (c) 16 (c) 17 (d)

Hints and Explanations


SESSION 1 3 F = Ma = Mω2 x = Mω2 A sin ωt . Hence, 6 Let displacement equation of particle
1 Here, the ball moves under constant graph between F and t is a sine curve. executing SHM is y = a sin ω t
acceleration and hence displacement is As particle travels half of the amplitude
4 For a simple harmonic motion, from the equilibrium position, so
not proportional to acceleration. So, the
d2 y a
motion cannot be simple harmonic. a∝ ∝− y y =
dt 2 2
2 y = a sin ω t + b cos ω t ...(i) Hence, equations y = sin ωt − cos ωt a
Therefore, = a sin ω t
Let a = A cos θ and b = A sinθ  3π  2
and y = 5 cos  − 3 ω t  are
Then,  4  1 π π
or sin ω t = = sin or ω t =
a2 + b 2 = A2 (cos 2 θ + sin2 θ) = A2 satisfying this condition and equation 2 6 6
y = 1 + ωt + ω2t 2 is not periodic and π π  2π 
or A= a2 + b 2 or t = or t = as ω =
y = sin3 ωt is periodic but not simple 6ω  2π  T 
From Eq. (i), 6 
hormonic motion. T 
y = A cos θ sin ω t + A sin θ cos ω t T
= A sin(ω t + θ) 5 In the equation of SHM, or t =
y = A cos (ω t + φ), the initial phase is 12
It is an equation of SHM with amplitude
the phase at t = 0. Since, the phase is Hence, the particle travels half of the
A= a +b .
2 2
T
ωt + φ, therefore at t = 0, we have amplitude from the equilibrium in s.
φ = 0.6 rad. 12
166 40 DAYS ~ NEET PHYSICS DAY SIXTEEN

7 Given, T = 30 s, OQ = B . The projection Again differentiating, we get 16 a = − ω2 y ⇒ a2 = ω 4 y 2


of the radius vector on the diameter of 2
d x a 2
the circle when a particle is moving = a = 2aω2 [cos 2 ωt − sin2 ωt ] Hence, ω = = = 100
with uniform angular velocity (ω) on a dt 2 y 0.02
circle of reference is SHM. Let the
= 2aω2 cos 2ωt [Q y = 0.02, a = 2]
particle go from P to Q in time t.
= 10 rad s −1
The given equation does not satisfy the
Y
condition for SHM [Eq. (i)]. Therefore, 17 From given graph,
P (t = 0) amplitude (a) = 1 cm
motion is not simple harmonic.
Q (t = 30 s) Time period (T ) = 8 s
ωt 11 Here, y = A sin ω t and 2π π
∴ ω= = Hz
ωt 8 4
v = Aω cos ω t
X 2π Acceleration, A = − ω2 a sin ωt
O x (t) R Since, ω= [QT = 16s]
16 4
at t = s,
Therefore, 3
2 π 2π − π2  π 4
v = A  cos ×2 A= × 1 × sin  × 
 16  16 16  4 3
Then, ∠POQ = ω t = ∠OQR. The 2π
π = A× ×
1
[Q v = π m/s] − π2  π
projection of radius OQ on X-axis will be ⇒ A= sin  
16 2 16  3
OR = x(t ) say.
Hence, A = π ×
16
× 2= 8 2m − 3 2
x(t ) ⇒ A= π cm/s2
In ∆OQR, sin ω t = 2π 32
OQ
2π 12 Maximum velocity, 18 a = − ω2 x ...(i)
or x( t ) = B sin ω t = B sin t
T v max = aω = 3 × 100 = 300 ∴ a = − bx ...(ii)

= B sin t [∴ a = 3, ω = 100] 2π
30 ∴ ω = b ;ω = b ;
2
= b
13 v 12 = ω2 (a2 − x12 ) ...(i) T
8 Given, y = 3cos  π − 2ω t  ...(i) v 22 = ω2 (a2 − x22 ) ...(ii) ⇒ T =

4 
b
On subtracting Eq. (ii) from Eq. (i), we
dy π 
Velocity, v = = 3 × 2ω sin  − 2ω t  get 19 As, x = A cos ω t
dt 4 
v 12 − v 22 dx
v 12 − v 22 =ω 2
( x22 − x12 ), ω = ∴ v = = − Aω sin ω t …(i)
Acceleration, x22 − x12 dt
dv π 
A= = − 4ω2 × 3cos  − 2ω t  2
d x
dt 4  2π x22 − x12 and a = = − Aω2 cos ω t …(ii)
T = = 2π dt 2
ω v 12 − v 22
As, A ∝ y and negative sign shows that We can find the correct graph by putting
it is directed towards equilibrium (or 14 As we know, the velocity of body different values of t in Eq. (ii).
mean position), hence particle will executing SHM is given by At t = 0, a = − Aω2
execute SHM. Comparing Eq. (i) with dx
equation y = r cos (φ − ω ′ t ), we have v = = aω cos ωt = aω 1 − sin2 ωt At t = ,
T
dt 4
ω ′ = 2ω
2π = ω a2 − x2  2π T 
or = 2ω a = − Aω2 cos  ×  =0
T′ a T 4
Here, x=
π 2 T  2π T 
or T′= At t = , a = − Aω2 cos  × 
ω a2 3a2 2 T 2
∴ v =ω a − =ω2

9 y 1 = a sin (ω t ) 4 4 = − Aω2 cos π = + Aω2


y 2 = b sin (ω t + π / 2) 2 π a 3 πa 3 At t =
3T
= = ,
T 2 T 4
y R = y1 + y2 = a + b sin (ω t + θ)
2 2
 2 π 3T 
15 At the mean position, velocity is a = − Aω2 cos  ×  =0
SHM with amplitude a2 + b 2 . T 4
maximum
v max = aω At t = T ,
10 For a particle executing SHM,  2π 
v 16 a = − Aω2 cos  × T  = − Aω2
acceleration, ω = max = =4 [Q v max = 16] T 
a 4
a ∝ − ω2 displacement ( x ) …(i)
∴ v = ω a2 − y 2 This condition is represented by graph
Given, x = asin2 ω t …(ii) in option (c).
Differentiating the above equation w.r.t. 8 3 = 4 (4)2 − ( y )2 [Q v = 8 3]
20 As we know that, the condition for a
t, we get 192 = 16(16 − y ) 2
body executing SHM is F = − kx
dx F k
= 2aω (sin ω t ) (cos ω t ) 12 = 16 − y 2 So, a= =− x
dt m m
y = 2 cm
DAY SIXTEEN OSCILLATIONS 167

or a = − ω2 x 25 Maximum speed of a particle executing 2π


But ω=
SHM is T
Acceleration ∝ − (displacement) T
u max t =
A∝− y u max = aω = a (2 π ν) ⇒ ν = Therefore,
2 πa 8
A = − ω2 y
Here, u max = 31.4 cms −1 , a = 5 cm 30 Here, y = A cos ω t .
k
A=− y
m Substituting the given values, we have When y = A /2, we find
A = − ky 31.4 1 π
ν= = 1 Hz cos ω t = = cos .
Here, y = x+ a 2 × 3.14 × 5 2 3
∴ Acceleration = − k ( x + a) π
Hence, ω t =
26 For a particle executing SHM, we have 3
21 Maximum acceleration of body executing maximum acceleration, π π T
SHM is given by α max = ω2 a or t = = =
α = Aω2 …(i) 3ω 3 × 2 π / T 6
So, for two different cases, where, A is maximum amplitude and ω
α max1 ω2 is angular velocity of a particle. 31 K = 1 / 2 mω2 [ A2 − y 2 ]
= 1 (Qa is same)
α max2 ω22 Maximum velocity, β = Aω …(ii) 1  y2 
Dividing Eq. (i) by Eq. (ii), we get = mω2 A2 1 − 
(100) 2
1 2  A2 
= = α Aω2 α 2π
(1000)2 102 = ⇒ = ω=  y2  3
β Aω β = 1 − E = E
2
T
2πβ  A  4
22 The phase difference between the i.e. T =
SHM’s is φ = 0. Therefore, resultant α
32 Here, KE = PE
amplitude is given by Thus, its time period of vibration,
1 1
T =
2πβ
. mω2 ( A2 − y 2 ) = mω2 y 2
A= A 12 + A 22 + 2 A 1 A 2 cos φ α 2 2
A
y = ⇒ y = 2 2 cm
27 Velocity v = ω A2 − x2 2
= A 12 + A 22 + 2 A 1 A 2
and Acceleration = ω2 x l
= A 1 + A 2 = 6 + 8 = 14 33 We know that, T = 2π
Now given, ω2 x = ω A2 − x2 g
23 The acceleration of particle/body ω2 ⋅ 1 = ω (2)2 − (1)2 T = 2π
l
⇒ T ′ = 2π
4l
executing SHM at any instant (at g g
ω= 3
position x) is given as T′
2π 2π ⇒ = 2 ⇒ T ′ = 2T
a = − ω2 x Time period, T = =
ω 3 T
where, ω is the angular frequency of the When l is made four times, the time
body. 28 According to the question, equation of period is doubled.
⇒ | a | = ω2 x …(i) motion of SHM is
 π 34 The acceleration due to gravity at a
Here, x = 5m, | a | = 20 ms−2 x = a sin  ω t + 
 6 depth d is
Substituting the given values in Eq. (i), velocity of body is given by  d R − d 
g d = g 1 −  = g  . But
we get 20 = ω2 × 5 dx  π  R  R 
20 v = = aω cos  ω t + 
⇒ ω2 = = 4 or ω = 2 rad s −1 dt  6 R − d = y is the distance of the point
5 aω π from the centre of the earth. Then,

As, we know that = aω cos  ωt +  g
2  6 gd = y . So, the acceleration of the

Time period, T = …(ii) R
ω 1  π
= cos  ω t +  body in the tunnel is proportional to y.
∴Substituting the value of ω in 2  6 And the motion is SHM with time
Eq. (ii), we get π π  π 1 period
ωt + = Qcos 3 = 2 
2π R
1 /2
T = = πs 6 3
2 T = 2π  
π g 
⇒ ωt =
24 Given, when x = 2 cm 6 Hence, it oscillates between the two
2π π T
| v|=|a | t = ⇒ t = ends of the tunnel.
T 6 12
⇒ ω A − x =ω x
2 2 2 g
35 Frequency, n = 1 or n ∝
1
29 Let y = A sin ω t 2π L L
A − x
2 2
9− 4
⇒ ω= = A n1 L2 L2 n1 1
x 2 i.e. = A sin ω t ∴ = = , =
2 n2 L1 2L2 n2 2
5
⇒ Angular velocity ω = 1 π
2 ⇒ sin ω t = = sin n2 = 2n1
2 4
∴ Time period of motion ⇒ n2 > n1
2π 4π π
T = = s i.e. ωt = Energy, E =
1
mω2 a2 = 2 π2 mn2 a2
ω 4 2
5
168 40 DAYS ~ NEET PHYSICS DAY SIXTEEN

1 39 Effective force constant in case (iii) and 45 Time period of spring pendulum
and a2 ∝ [QE is same]
mn2 (iv) is K eff = 2k + 2k = 4k M
a12 m2 n22 m T = 2π
∴ = Therefore, T1 = T2 = 2 π k
a22 m1 n12 k
When mass is doubled,
Given, n2 > n1 and m1 = m2 ⇒ a1 > a2 m m
and T3 = T 4 = 2 π = π T ′ = 2π
2M
So, amplitude of B smaller than A. 4k k k
36 Potential energy of a simple harmonic 40 Time period of the system is T′
∴ = 2
oscillator m1 + m2 T
T = 2π
1 ⇒ T ′ = 2T
U = mω2 x2 k
2 Given, m1 = 6 kg, Hence, option (d) is true.
Kinetic energy of a simple harmonic m2 = m = mass of girl
oscillator k = 600 N/m, T = 0758
. s 46 T = 2π m ⋅ Since, T = 1
1 k n
K = mω2 (a2 − x2 ) 6+ m n∝ k
2 ∴ T = 2π = 0758
. Therefore,
600 1
Here, x = Displacement from mean Also, for a spring k ∝
position On squaring both sides, we get x
a = Maximum displacement 6 + m (0758
. )2 k2 x
= ⇒ m = 274
. kg Thus, = 1 =2
(or amplitude) from mean position 600 4 π2 k1 x2
Total energy is E = U + K 41 For series combination of springs, Therefore, new frequency
1 1 1
= mω2 x2 + mω2 (a2 − x2 ) = mω2 a2 1
=
1
+
1
, k =
k1 k2 k2
2 2 2 n2 = n1 × =n 2
k k1 k2 k1 + k2 k1
When the particle is half way to its end
point i.e. at half of its amplitude, then 42
a 47 Amplitude of damped oscillator
x=
2 P Q A = A 0 e − λt ; λ = constant, t = time
Hence, potential energy
2 For t = 1 min,
1  a 1 1  R S
U = mω2   =  mω2 a2  A0
2  2 4 2  k = A 0 e − λt ⇒ e λ = 2
m 2
E
⇒U = For t = 3 min,
4 A0 A0
Spring P and Q, R and S are in parallel. A
37 As we know that A = A 0e − λ × 3 = = = 0
x = k + k = 2k λ 3
Then, [for P, Q] (e ) (2)3 X
m
Time period, T = 2π and y = k + k = 2k [for R, S] ⇒ X = (2)3
k x and y both in series
Case I T1 = 2 π
m
…(i) ∴
1 1
= +
1
=
1 48 Given, Damping force ∝ velocity
k k′ x y k F ∝v
Case II When the mass m is increased m m F
Time period, T = 2 π = 2π ⇒ F = kv ⇒ k =
m+1 k′ k v
by 1 kg, then, T2 = 2 π …(ii)
k unit of F kg - ms −2
43 The fundamental frequency of string Unit of k = =
From Eqs. (ii) and (i), we get
1 T k
unit of v ms −1
T2 m+1 ν= =
= 2l m l = kg s −1
T1 m
∴ ν1 l1 = ν2 l2 = ν3 l3 = k …(i)
5 m+1 25 m + 1 k k k
⇒ = ⇒ = From Eq. (i) l1 = , l2 = , l3 = , SESSION 2
3 m 9 m ν1 ν2 ν3
25 1 1 16 1 Let the minimum amplitude of SHM is
⇒ =1+ ⇒ = Original length l =
k
9 m m 9 a. Restoring force on spring
ν
9 F = ka
∴ m= kg Here, l = l1 + l2 + l3 mg
16 k k k k ∴ ka = mg or a =
= + + k
38 By the law of conservation of energy, ν ν1 ν2 ν3
Here, m = 2 kg, k = 200 Nm −1 ,
Kinetic energy of mass = Energy stored 1 1 1 1
= + + . g = 10 ms −2
in spring ν ν1 ν2 ν3
1 1
mv 2 = k x2 2 × 10 10
i.e. ∴ a= = m
2 2 44 By using k ∝ 1 200 100
l
mv 2  mv 2  10
∴ x2 = ⇒x=   Since, one-fourth length is cut away, so = × 100 cm = 10 cm
k  k  3 100
remaining length is th, hence, k
 0.5 × 1.5 × 1.5 4 Hence, minimum amplitude of the
⇒ x=   = 0.15 m 4 4
 50  becomes times i.e. k ′ = k . motion should be 10 cm, so that the
3 3 mass gets detached from the pan.
DAY SIXTEEN OSCILLATIONS 169

2Q x = a sin ω t 6 Force constant of a wire is When block is displaced a small


x YA m ml distance x in downward direction from
∴ sin ωt = K = or T = 2 π = 2π
a l K YA equilibrium position, tension in the
a2 − x2 string is T .
cos ωt = 7 K eff = 2k
a mg − T = ma
M
and y = a sin 2ω t = 2a sin ω t cos ω t ∴ T = 2π  x kx
2k 2T = k  x 0 +  = k x 0 +
 2 2
x a2 − x2 2x a2 − x2 Period (or K eff ) is independent of θ.
= 2a =  k x
a a a mg −  mg +  = ma
8 Let block is displaced through x, then  4
4 x2
∴ y =2
(a − x )
2 2 weight of displaced water or upthrust kx
a2 (upwards) = − A xρg − = ma
4
3 Force on the mass m along the tunnel where, A is area of cross-section of the  k 
block and ρ is its density. This must be ⇒ a= −   x
will be  4m
equal to force (= ma) applied, where m is
 GMm  mass of the block and a is acceleration.  k 
F =− y  sin θ ⇒ ω2 (− x ) = −  x
 R3  ∴ ma = − A xρg  4m
 x Aρg
 From fig. sinθ =  or a= − x = − ω2 x k
 y  m ω=
4m
This is the equation of SHM.
 GMm   x  GMm 2π
=− y   = − x Time period of oscillation ⇒ =
k
⇒ T = 4π
m
.
 R3   y R3
2π m 1 T 4m k
T = = 2π ⇒T ∝
ω Aρg A
12 In SHM, the total energy of the particle
P is constant at all instants which is
9 Extensions in springs are x1 and x2 , then
totally kinetic when particle is passing
x k1 x1 = k2 x2 and x1 + x2 = A
θ through the mean position and is totally
O k x k x
y ⇒ x2 = 1 1 ⇒ x1 + 1 1 = A potential when particle is passing
m k2 k2 through the extreme position.
k2 A
⇒ x1 = Energy
k1 + k2 Total energy
mg  GM  10 In series combination PE
or F =− x Q g = R2 
R   KE
2K1 t
It is a SHM with k = mg/R T/4 2T/4 3T/4 4T/4
m R K1 K1 2K1
Hence, T = 2 π =2 π ,
k g The variation of PE and KE with time is
where g is the acceleration on the shown in figure by dotted parabolic
K2 curve and solid parabolic curve,
surface of the earth. K2 respectively.
2
4 Frequency, f = 1 BA
m
Figure indicates that maximum values
2π MV 0 m of total energy, KE and PE of SHM are
B = bulk modulus of elasticity −1 equal.
1 1 1  1 1
MV 0 M (hA ) Mh = + ⇒ kS =  +  Now, E K = K 0 cos 2 ω t
T = 2π = 2π = 2π kS 2k1 k2 2
 1k k2
BA2 pA2 pA ∴ (E K ) max = K 0
11 Let tension in spring at equilibrium So, (E P ) max = K 0
5 Let train is moving towards right. The positions for block is T 0
and (E )total = K 0
Pseudo force is acting in left direction x
k x0 +
= ma0 2 13 From conservation principle,
k x0
1 2
mgh = k x 0 − mgx 0
T q a0 2
ma 0
where, x 0 is maximum elongation in
g spring.
mg
√a 2 + g2 1 2
0 ⇒ kx 0 − mgx 0 − mgh = 0
2
T0 T0 T T 2mg 2mg
Effective acceleration, ⇒ x20 − x0 − h=0
a eff = a20 + g 2 ∴ mg = T 0 k k
2
T = 2π
l
= 2π
l From force diagram of pulley, 2mg  2mg  2mg
±   + 4× h
aeff a20 + g 2 2T 0 = k x 0 ⇒ 2mg = k x 0 k  k  k
x0 =
2
170 40 DAYS ~ NEET PHYSICS DAY SIXTEEN

Amplitude = elongation in spring for 15 Potential energy of stretched string is 17 x = A cos ωt


lowest extreme position – elongation in 1 Displacement in t time = A − A cos ωt
U = kx2
spring for equilibrium position 2 For t = τ ⇒ A [1 − cos ωt ] = a …(i)
mg 2hk where, k is spring constant or force For t = 2τ ⇒ A [1 − cos 2ωt ] = 3a …(ii)
= x 0 − x1 = 1+
k mg constant. Divide eq (i) by eq (ii)
 mg  U1 x2 1 − cos ωt 1
∴ = 1 =
Q x1 = k 
…(i)
U2 x22 1 − cos 2ωt 3
 
1 − cos ωt 1
Hence, option (b) is true. Given, U 1 = U , x1 = 2 cm, and x2 = 8 cm =
2sin2 ωt 3
On putting these values in Eq. (i), we get
14 Let y = A sin ω t say x = cos ωt
U (2) 2 4 1 1− x
1 = = = =
1
Then, K = MA2ω2 cos 2 ω t and U 2 (8) 2 64 16 2(1 − x2 ) 3
2
∴ U 2 = 16 U 1 1
1
U = MA2 ω2 sin2 ω t ⇒ =
2 16 Mass of the particle = m 2(1 + x ) 3
⇒ 3 = 2 + 2x
Therefore, Spring constant = k
1
K cos 2 ω t 1 − sin2 ω t m ⇒ x = = cos ωt
= = The time period of oscillator, T = 2π 2
U sin2 ω t sin2 ω t k π
1 A = 2a, ωt =
1 − y 2 / A2 As k ∝ [where, l is the length of spring] 3
=
y /A2 2 l 2π π
⇒ τ =
k ′ = 2k T 3
A − y2
2
m 1
= ∴ T ′ = 2π = T ⇒ T = 6τ
y2 2k 2
DAY SEVENTEEN

Waves
Learning & Revision for the Day
u Wave Motion u Principle of u Standing or Stationary
u Speed of Waves Superposition of Waves Waves
u Sound Waves u Intensity u Power u Beats
u Displacement Relation for a u Reflection and Transmission u Doppler’s Effect
Progressive or Harmonic Wave of Waves

Wave Motion
Wave motion involves transfer of disturbance (energy) from one point to the other with
particles of medium oscillating about their mean positions i.e. the particles of the
medium do not travel themselves along with the wave. Instead, they oscillate back and
forth about the same equilibrium position as the wave passes by. Only the disturbance is
propagated.
1. Longitudinal Waves When particles of the medium vibrate parallel to the direction
of propagation of wave, then wave is called longitudinal wave. These waves
propagate in the form of compressions and rarefactions. They involve changes in
pressure and volume. The medium of propagation must possess elasticity of volume.
They are set up in solids, liquids and gases.
2. Transverse Waves When the particles of the medium vibrate in a direction
perpendicular to the direction of propagation of wave, then wave is called transverse
waves. These wave propagates in the form of crests and troughs. These waves can be
set up in solids, on surface of liquids but never in gases.

Terms Used in Wave Motion


Angular Wave Number Number of wavelength in the distance 2π is called the wave
PREP
MIRROR
l

number or propagation constant.


2π Your Personal Preparation Indicator
K = rad/m
λ
u No. of Questions in Exercises (x)—
l
Particle velocity It is the velocity of the particle executing simple harmonic motion. u No. of Questions Attempted (y)—
dy
i.e. v= u No. of Correct Questions (z)—
dt (Without referring Explanations)
where, y denotes displacement at any instant.
u Accuracy Level (z / y × 100)—
l
Wave Velocity The velocity of transverse wave motion is given by
u Prep Level (z / x × 100)—
Distance travelled by wave
v=
Time taken In order to expect good rank in NEET, your
λ ω Accuracy Level should be above 85 & Prep
i.e. v= = or v = νλ Level should be above 75.
T k
172 40 DAYS ~ NEET PHYSICS DAY SEVENTEEN

Differential Equation of Wave Motion


Factors Affecting Speed of Sound
l

d2 y 1 d2 y
2
= 2 Effect of Temperature on Velocity With rise in
v dt 2
l
dx
temperature, the velocity of sound increases as
γRT v2 T
Speed of Waves v= ; = 2 i.e. v∝ T
M v1 T1
Speed of waves is divided in two types as per the nature of Speed of sound in air increases by 0.61 m/s for every 1°C
wave, these are given below rise in temperature.
l
Effect of Pressure for Gases Medium Pressure has no effect
1. Speed of Transverse Wave on the velocity of sound, provided temperature remains
The expression for speed of transverse waves in a solid and in constant.
case of a stretched string can be obtained theoretically l
Effect of Humidity When humidity in air increases, its
η density decreases and so velocity of sound increases.
l
In solids, v =
d Y K
For solids, v = . For liquids, v =
where, η is the modulus of rigidity and d is the density of D D
the medium. where, Y = Young’s modulus of elasticity
In a stretched string, v =
T Mg K = bulk modulus of elasticity.
l
=
m πr 2 d
where, T = the tension in the string,
m = the mass per unit length of the string,
Sound Waves
The longitudinal waves which can be heard are called sound
M = mass suspended from the string,
waves. They are classified into following categories
r = radius of the string and
l
Infrasonics The longitudinal waves having frequencies
d = density of the material of the string.
below 20 Hz are called infrasonics. These waves cannot be
heard. These waves can be heard by snakes.
2. Speed of Longitudinal Wave l
Audible waves The longitudinal waves having the
(or Sound Wave) frequency between 20 Hz and 20000 Hz are called audible
waves. Human can hear these waves.
Following are the expressions for the speed of longitudinal
waves in the different types of media l
Ultrasonics The longitudinal waves having the
frequencies above 20000 Hz are called ultrasonics. These
l
If the medium is solid,
waves are also called supersonic waves or supersonics.
4
B+ η
v= 3
ρ Displacement Relation for a
where B, η and ρ are values of bulk modulus, modulus of Progressive or Harmonic Wave
rigidity and density of the solid respectively.
The equation of a plane progressive or simple harmonic wave
If the solid is in the form of a long rod, then
travelling along positive direction of x-axis is
2π  T
v=
Y y = a sin (ωt − kx) ⇒ y = a sin t − x  .
ρ T  λ
2π  x
where, Y is the Young’s modulus of the solid material. ⇒ y = a sin (vt − x) ⇒ y = a sin ω t − 
λ  v
l
In a liquid,
B t x
v= ⇒ y = a sin 2π  −  .
ρ T λ

where B is the bulk modulus of the liquid. l


If maximum value of y = a, i.e. a is amplitude, then
l
According to Newton’s formula, speed of sound in a gas is dy / dt = velocity of particle
obtained by B replaced by initial pressure p of the gas i.e., dy 2π
v= = aω cos ⋅ (vt − x)
B = p. dt λ
p  dy  2πva
v=   = = 2πna = ωa [where, n = frequency]
ρ  dt  max λ
DAY SEVENTEEN WAVES 173

l
Acceleration of particle Constructive and Destructive Interference
d2 y 2π
= − ω2 a sin (vt − x) l
When the wave meet a point with some phase,
dt 2 λ constructuve interference is obtained at that point.
l
For a wave, velocity of sound wave (i) Phase difference between the waves at the point of
v = frequency (n) × wavelength (λ ) observation φ = 0 ° or 2πn.
⇒ v = nλ (ii) Resultant amplitude at the point of observation will
2π 2 πv be maximum, Amax = A1 + A2 .
l
Angular speed, ω = 2 πn = ⇒ω =
T λ l
When the waves meet a point with opposite phase,
destructive interference is obtained at that point.
(i) Phase difference between the waves at the point of
Relation between Phase Difference, observation φ = 180 ° or (2 n − 1)π.
Path Difference and Time Difference (ii) Resultant amplitude at the point of observation will
2π be minimum, Amin = A1 − A2 .
l
Phase difference (φ) = × path difference ( x)
λ
2πx φλ Intensity
⇒ φ= ⇒x=
λ 2π The intensity of waves is the average amount of energy
2π transported by the wave per unit area per second normally
l
Phase difference (φ) = × time difference (t )
T across a surface at the given point.
2πt Tφ Intensity (I 1) ∝ (Amplitude A)2
⇒ φ= ⇒t =
T 2π 2
T I 1  A1 
l
Time difference (t ) = × path difference ( x) ∴ = 
λ I2  A2 
Tx λt If I 1 and I2 are intensities of the interfering waves and φ is
⇒ t = ⇒x=
λ T the phase difference, then resultant intensity is given by
I = I 1 + I2 + 2 I 1I2 cos φ
Principle of Superposition I max = I 1 + I2 + 2 I 1I2 = ( I 1 + I2 )2 , for φ = 2 πn
of Waves and I min = I 1 + I2 − 2 I 1I2
Two or more waves can traverse the same space I min = ( I 1 − I2 )2, for φ = (2 n + 1) π
independently of one another. The resultant displacement of
each particle of the medium at any instant is equal to the
vector sum of displacements produced by the two waves Power
separately. This principle is called principle of superposition If P is power of a sound source, then intensity (I ) follows
of waves. inverse square law of distance (d).
y = y1 + y2 + y3 + ..... P
I =
4πd2
Interference of Waves Reflection and Transmission
When two waves of same frequency (or A2 A
same wavelength) travelling along same
of Waves
path superimpose each other, there When sound waves are incident on a boundary separating two
φ
occurs redistribution of energy in the media, a part of it is reflected back into the initial medium while
medium. If at a given position (x being θ
the remaining is partly absorbed and partly transmitted into the
constant) displacement due to two A1 second medium.
waves be
y1 = A1 sin ωt Standing or Stationary Waves
and y2 = A2 sin (ωt + φ) Standing or stationary wave is formed due to superposition of
Then, resultant displacement two progressive waves of same nature, same frequency (or
y = y1 + y2 = A sin (ωt + φ) same wavelength), same amplitude travelling with same
speed in a bounded medium in mutually opposite directions.
where, A= A21 + A22 + 2 A1 A2 cos φ
If the incident wave be represented as y1 = A sin(ω t − kx)
A2 sin φ
and tan θ = and the reflected wave as y2 = A sin (ω t + kx),
A1 + A2 cos φ
then y = y1 + y2 = A sin(ω t − kx) + A sin(ω t + kx)
174 40 DAYS ~ NEET PHYSICS DAY SEVENTEEN

⇒ y = 2 A cos kx sin ωt λ1 v
l
First harmonic l = ⇒ f1 =
The resultant wave does not represent a progressive wave. 2 2l

Standing Waves in String A1


N
A2

Consider a string of length L stretched under tension T λ


between two fixed points (i.e. clamped at its ends). Transverse l= 1
2
wave is set up on the string whose speed is given by v = T /µ, (a)

where µ is the mass per unit length of the string. l


Second harmonic or first overlone l = λ 2 ; f =
2v
Different modes of vibration of stretched string are discuss 2l
below
A1 A2 A3
l
Let only one anti-node A is A N1 N2
formed at the centre and l= λ2
string vibrates in one A (b)
Fundamental or first harmonic
segment only, it is called 3λ3 3v
fundamental mode, then l
Third harmonic or second overtone l = ;f =
2 2l
λ
L = 1 or λ 1 = 2 L
2 A1 A2 A3 A4
Frequency of vibration in fundamental mode N1 N2 N3
v 1 T 3λ3
ν1 = = l=
λ 1 2L µ 2
(c)
It is known as the fundamental frequency or first
harmonic. l
All harmonics are present in open pipe with their
A A frequencies in the ratio 1 : 2 : 3 : 4 . . .. and
l
If string vibrates in two
segments, then N ratio of overtones = 2 : 3 : 4 : 5 K
A A λ 3λ 5λ
L = λ2 Second harmonic Position of nodes from one end x = , , K
4 4 4
v 1 T
and ν2 = = = 2ν 1 Position of anti-nodes from one end
λ2 L µ λ 3λ
x = 0, , λ , K
It is known as first overtone or second harmonic. 2 2
l
If the string vibrates in three segments, 2. Closed Organ Pipe
3λ A A A
then L = 3 In a closed organ pipe, always node is formed at the closed
2
N N end. Various mode of vibration of air column in a closed
v A A A
and ν3 = = 3ν 1 Third harmonic organ pipe are shown below
λ3 l
First harmonic
It is called second overtone or third harmonic. λ A N
l= 1
l
In general, if a string vibrates in p segments [i.e. have 4
( p + 1) nodes and p antinodes], then v λ1
f = l=
4
p T 4l
ν pth = = pν 1 and it is known as pth harmonic or (a)
2L µ l
Third hormonic
( p − 1)th overtone. (first overtone) A1 A2 N2
3λ N1
l= 3
Standing Waves in Organ Pipes 4 l=
3λ2
3v 4
(Air Columns) f3 = (b)
4l
Organ pipes are those cylindrical pipes which are used for l
Fifth hormonic
producing musical (longitudinal) sounds. The standing waves
(second overtone) A1 A2 A3 N3
in both organ pipes (i.e. open organ pipe and closed organ
5λ 5 N1 N2
pipe) are described below. l= ;
4 5λ3
l=
1. Open Organ Pipe 5v 4
f5 = (c)
In an open organ pipe, always anti-node is formed at both 4l
open ends. Various modes of vibration of air column in an l
In closed organ pipe only odd harmonics are present. Ratio
open organ pipe are shown below of harmonic is n1 : n3 : n5 = 1 : 3 : 5.
DAY SEVENTEEN WAVES 175

Ratio of overtones = 3 : 5 : 7
Doppler’s Effect
l

λ 3λ
l
Position of nodes from closed end x = 0, , λ, , ... The phenomena of apparent change in frequency of source
2 2
due to a relative motion between the source and observer is
λ 3λ 5λ
l
Position of antinodes from closed end x = , , , ... called Doppler’s effect.
4 4 4 l
When Source is Moving and Observer is at Rest When
source is moving with velocity vs , towards an observer at
rest, then apparent frequency
Beats  v  vs
When two sound waves of nearly equal (but never equal) or n′ = n  
 v − vs  S v O
slightly different frequencies and equal or nearly equal
amplitudes travelling along the same direction superimpose If source is moving away from observer, then
at a given point, the resultant sound intensity alternately rises  v 
n′ = n  
and falls. This alternate rise and fall of sound at a given  v + vs 
position is called beats. l
When Source is at Rest and Observer is Moving When
l
Number of beats formed per second is called the frequency observer is moving with velocity vo , towards a source at
of beats. If two sound waves of frequencies ν 1 and ν2 rest, then apparent frequency.
superimpose, then frequency of beats = (ν 1 ~ ν2 ),  v + vo  vo
n′ = n  
i.e. either (ν 1 − ν2 ) or (ν2 − ν 1).  v  S O
v
l
For formation of distinct beats, then difference between the When observer is moving away from source, then
frequencies of two superposing notes should be less than
 v − vo  vs vo
10 Hz. n′ = n  
 v  S O
l
Our perception of loudness is better co-related with the v
second level measured in decibel (dB) and defined as l
When Source and Observer Both are Moving
follows (a) When both are moving in same direction along the
 I direction of propagation of sound, then
β = 10 log 10   , where I 0 = 10 −12 Wm2 at 1kHz.
 I0  v − vo 
n′ = n  
 v − vs 
Tuning Fork S
v
O
The tuning fork is a metallic device that produces sound of a vs vo
single frequency.
Suppose, a tuning fork of known frequency nA is sounded (b) When both are moving in same direction opposite to
together with another tuning fork of unknown frequency (nB ) the direction of propagation of sound, then
and x beats heard per second.  v + vo 
n′ = n  
There are two possibilities to know frequency of unknown  v + vs 
tuning fork
vs vo
nA − nB = x …(i)
S O
nB − nA = x …(ii) v

We can find true frequency of tuning fork B from a pair of (c) When both are moving towards each other, then
tuning forks A and B, in which frequency of A is known,
 v + vo 
where x is the beats per second. n′ = n  
 v − vs 
When B is loaded When B is filled v
(its frequency decreases) (its frequency increases) S O
vs vo
(i) If x increases, then (i) If x increases, then
nB = nA − x nB = nA + x (d) When both are moving in opposite direction, away
(ii) If x decreases, (ii) If x decreases, then from each other, then
then nB = nA + x nB = nA − x  v − vo 
n′ = n  
(iii) If x remains same, (iii) If x remains same, then  v + vs 
then nB = nA + x nB = nA − x v
vo
(iv) If x becomes zero, (iv) If x becomes zero, then vs
O S
then nB = nA + x nB = nA − x
176 40 DAYS ~ NEET PHYSICS DAY SEVENTEEN

DAY PRACTICE SESSION 1

FOUNDATION QUESTIONS EXERCISE


1 With propagation of longitudinal waves through a (a) Increases by a factor 20 (b) Increases by a factor 10
medium, the quantity transmitted is (c) Decreases by a factor 20 (d) Decreases by a factor 10
(a) matter (b) energy 10 Velocity of sound in the atmosphere of a planet is
(c) energy and matter
(d) energy, matter and momentum 500 ms −1. The minimum distance between the source of
sound and the obstacle to hear the echo should be
2 The waves produced by a motorboat sailing in water are
(a) 17 m (b) 20 m (c) 25 m (d) 50 m
(a)transverse (b) longitudinal
(c)longitudinal and transverse 11 The displacementY of a particle in a medium can be
(d)stationary expressed as
y = 10−6 sin (100 t + 20x + π /4) m
3 A transverse wave is represented by y = A sin (ωt − kx ).
For what value of the wavelength is the wave velocity equal where, t is in second and x in metre.
to the maximum particle velocity? j CBSE AIPMT 2010 The speed of the wave is
(a) πA/ 2 (b) πA (c) 2 πA (d) A (a) 2000 ms −1 (b) 5 ms −1
(c) 20 ms −1 (d) 5 π ms −1
4 A transverse wave propagating along X -axis is
represented by 12 A wave in a string has an amplitude of 2cm. The wave
travels in the positive direction of x-axis with a speed of
 π
y ( x , t ) = 8. 0 sin  0.5 πx − 4πt −  128 ms−1 and it is noted that 5 complete waves fit in 4 m
 4
length of the string. The equation describing the wave is
where, x is in metre and t is in second. The speed of the j
CBSE AIPMT 2009
wave is (a) y = (0.02) m sin (7.85 x + 1005 t )
π
(a) 4 π ms −1 (b) 0.5 π ms −1 (c) ms −1 (d) 8 ms −1 (b) y = (0.02) m sin (15.7 x − 2010 t )
4 (c) y = (0.02)m sin (15.7 x + 2010 t )
5 Sound waves of wavelength λ travelling in a medium with (d) y = (0.02) m sin (7.85x − 1005 t )
a speed of v m/s enter into another medium where its 13 A wave is represented by the equation
speed is 2v m/s. Wavelength of sound waves in the
 π
second medium is y = 7 sin 7πt − 0 . 04x + 
λ  3
(a) λ (b) (c) 2λ (d) 4λ
2 where, x in metre and t in second. The speed of the wave
6 Velocity of sound in a gaseous medium is 330 ms −1. If the is
pressure is increased by 4 times without change in (a) 175 π ms −1(b) 49 π ms −1 (c) 49/π ms −1 (d) 0. 28 π ms −1
temperature, the velocity of sound in the gas is
14 In the given progressive wave equation,
(a) 330 ms −1 (b) 660 ms −1 (c) 156 ms −1 (d) 990 ms −1
y = 0.5 sin (10 π t − 2 x ) cm
7 The velocity of sound in air at NTP is 330 m/s. What will What is the maximum velocity of particle?
be its value when temperature is doubled and pressure is
(a) 5 ms −1 (b) 5 π ms −1 (c) 10 ms −1 (d) 10.5 ms −1
halved?
(a) 330 m/s (b) 165 m/s 15 The phase difference between two waves, represented
(c) 330 2 m/s (d) 320/ 2 m/s by
8 A wave of frequency 500 Hz travels between X and Y ,   x 
y1 = 10−6 sin 100 t +   + 0 . 5 m
distance of 600 m in 2 s. How many wavelength are there
  50  
in distance XY ?
  x 
(a) 1000 (b) 300 y 2 = 10−6 cos 100 t +    m
(c) 180 (d) 2000   50 
9 Sound waves travel at 350 m/s through a warm air and at
where, x is expressed in metre and t is expressed in
3500 m/s through brass. The wavelength of a 700 Hz
second, is approximately
acoustic wave as it enters brass from warm air
j
CBSE AIPMT 2011 (a) 1.07 rad (b) 2.07 rad (c) 0.5 rad (d) 1.5 rad
DAY SEVENTEEN WAVES 177

 x 25 When a string is divided into three segments of lengths


16 The equation y = A cos 2 2πnt − 2π represents a
 λ  l1, l2 and l3 , the fundamental frequencies of these three
wave with segments are ν1, ν 2 and ν 3 , respectively. The original
(a) amplitude A/2, frequency 2n and wavelength λ fundamental frequency (v) of the string is
(b) amplitude A/2, frequency 2n and wavelength λ/2 (a) ν = ν1 + ν2 + ν3 (b) ν = ν1 + ν2 + ν3
(c) amplitude A, frequency n and wavelength λ 1 1 1 1 1 1 1 1
(c) = + + (d) = + +
(d) amplitude A, frequency 2n and wavelength 2λ ν ν1 ν2 ν3 ν ν1 ν2 ν3

17 A wave travelling in the positive x-direction having 26 An organ pipe, open from both ends produced 5 beats/s
displacement along y-direction as 1 m, wavelength 2π m when vibrated with a source of frequency 200 Hz in its
1 fundamental mode. The second harmonic of the same
and frequency of Hz is represented by
π pipe produces 10 beats/s with a source of frequency
(a) y = sin (x − 2 t ) (b) y = sin (2 πx − 2 πt ) 420 Hz. The fundamental frequency of pipe is
(c) y = sin (10 πx − 20 πt ) (d) y = sin (2 πx + 2 πt ) (a) 195 Hz (b) 205 Hz (c) 190 Hz (d) 210 Hz
18 Ratio of amplitude of two interfering waves is 2:1, then 27 The fundamental frequency of a closed pipe is 220 Hz.
ratio of amplitude of maxima to minima is 1
If of the pipe is filled with water, the frequency of the Ist
(a) 4 : 1 (b) 9 : 1 (c) 3 : 1 (d) 9 : 4
4
overtone of the pipe now is
19 If two waves of amplitude a produce a resultant wave of
(a) 220 Hz (b) 440 Hz (c) 880 Hz (d) 1760 Hz
amplitude a, then the phase difference between them
will be 28 An organ pipe closed at one end has fundamental
frequency of 1500 Hz. The maximum number of
(a) 60° (b) 90° (c) 120° (d) 180°
overtones generated by this pipe which a normal person
20 A point source emits sound equally in all directions in a can hear is?
non-absorbing medium. Two points P and Q are at (a) 4 (b) 13 (c) 6 (d) 9
distance of 2 m and 3 m respectively, from the source.
The ratio of the intensities of the waves at P and Q is 29 The fundamental frequency of a closed organ pipe of
length 20 cm is equal to the second overtone of an organ
(a) 9 : 4 (b) 2 : 3
pipe open at both the ends. The length of organ pipe
(c) 3 : 2 (d) 4 : 9
open at both the ends is j
AIPMT 2015
21 The equation of a stationary wave is (a) 80 cm (b) 100 cm (c) 120 cm (d) 140 cm
 πx 
y = 0.8 cos   sin 200 πt 30 The number of possible natural oscillations of air column
 20 
in pipe closed at one end of length 85 cm whose
where x is in cm and t is in second. The separation frequencies lie below 1250 Hz are (velocity of sound
between consecutive nodes will be = 340 ms −1 ) j
AIPMT 2014
(a) 4 (b) 5 (c) 7 (d) 6
(a) 20 cm (b) 10 cm (c) 40 cm (d) 30 cm
22 Standing waves are produced in 10 m long stretched 31 The fundamental frequency in an open organ pipe is
equal to the third harmonic of a closed organ pipe. If the
string. If the string vibrates in five segments and wave
length of the closed organ pipe is 20 cm, the length of
velocity is 20 ms −1, then its frequency will be
the open organ pipe is j
NEET 2018
(a) 5 Hz (b) 2 Hz (c) 10 Hz (d) 15 Hz
(a) 12.5 cm (b) 8 cm (c) 13.3 cm (d) 16 cm
23 The length of a sonometer wire AB is 110 cm. Where
32 The two nearest harmonics of a tube closed at one end
should the two bridges be placed from A to divide the
and open at other end are 220 Hz and 260 Hz. What is
wire in three segments whose fundamental frequencies the fundamental frequency of the system? j
NEET 2017
are in the ratio of 1 : 2 : 3.
(a) 10 Hz (b) 20 Hz
(a) 30 cm and 90 cm (b) 40 cm and 80 cm (c) 30 Hz (d) 40 Hz
(c) 60 cm, 30 cm and 20 cm (d) 30 cm and 60 cm
33 A uniform rope of length L and mass m1 hangs vertically
24 If n1, n 2 and n 3 are the fundamental frequencies of three from a rigid support. A block of mass m2 is attached to
segments into which a string is divided, then the original the free end of the rope. A transverse pulse of
fundamental frequency n of the string is given by wavelength λ 1 is produced at the lower end of the rope.
j
AIPMT 2014 The wavelength of the pulse when it reaches the top of
1 1 1 1 1 1 1 1 the rope is λ 2 . The ratio λ 2 / λ 1 is j
NEET 2016
(a) = + + (b) = + +
n n1 n2 n3 n n1 n2 n3 m1 + m2 m2 m1 + m2 m1
(a) (b) (c) (d)
(c) n = n1 + n2 + n3 (d) n = n1 + n2 + n3 m2 m1 m1 m2
178 40 DAYS ~ NEET PHYSICS DAY SEVENTEEN

34 The second overtone of an open organ pipe has the 42 A tuning fork of frequency 512 Hz makes 4 beat/s with
same frequency as the first overtone of a closed pipe L the vibrating string of a piano. The beat frequency
metre long. The length of the open pipe will be decreases to 2 beat/s when the tension in the piano string
j NEET 2016 is slightly increased. The frequency of the piano string
(a) L (b) 2L (c) L/ 2 (d) 4L before increasing the tension was j CBSE AIPMT 2010

(a) 510 Hz (b) 514 Hz (c) 516 Hz (d) 508 Hz


35 If we study the vibration of a pipe open at both ends,
which of the following statements is not true? j NEET 2013 43 A sound source is moving towards stationary listener with
(a) Open end will be antinode 1/10th of the speed of sound. The ratio of apparent to
(b) Odd harmonics of the fundamental frequency will be real frequency is
2 2
(b)   (c)  
generated 11 11 9 10
(a) (d)
(c) All harmonics of the fundamental frequency will be 10  10   10  9
generated
44 A tuning fork is used to produce resonance in a glass
(d) Pressure change will be maximum at both ends
tube. The length of the air column in this tube can be
36 Two sound waves with wavelengths 5.0 m and 5.5 m adjusted by a variable piston. At room temperature of
respectively, each propagate in a gas with velocity 330 ms −1. 27° C, two successive resonances are produced at 20 cm
We expect the following number of beats per second and 73 cm of column length. If the frequency of
(a) 12 (b) 0 (c) 1 (d) 6 the tuning fork is 320 Hz, the velocity of sound in air at
37 If two waves of wavelengths 50 cm and 51 cm produced 27° C is j NEET 2018

12 beats/s, the velocity of sound is (a) 350 m/s (b) 339 m/s (c) 330 m/s (d) 300 m/s
−1 −1 −1 −1 45 A sonometer wire resonates with a given tuning fork
(a) 360 ms (b) 306 ms (c) 331 ms (d) 340 ms
38 Two wave of wavelengths 99 cm and 100 cm both forming standing waves with five antinodes between the
−1
travelling with velocity 396 ms are made of interfere. two bridges when a mass of a 9 kg is suspended from a
wire. When this mass is replaced by a mass M, the wire
The number of beats produced by them per second are
resonates with the same tuning fork forming three
(a) 1 (b) 2 (c) 4 (d) 8
antinodes for the same positions of the bridges. The
39 Three sound waves of equal amplitudes have value of M is
frequencies (n −1), n, (n + 1). They superimpose to give (a) 25 kg (b) 5 kg (c) 12.5 kg (d) 1/25 kg
beats. The number of beats produced per second will be
46 A train is approaching the platform with a speed of
j
NEET 2016
4 ms −1. Another train is leaving the platform with the same
(a) 1 (b) 4 speed. The velocity of sound is 320 ms −1. If both the
(c) 3 (d) 2
trains sound their whistles at frequency 280 Hz, the
40 Two sources of sound placed closed to each other, are number of beats heard per second will be
emitting progressive waves given by y1 = 4 sin 600πt and (a) 6 (b) 7 (c) 8 (d) 10
y 2 = 5 sin 608πt
47 A speeding motorcyclist sees traffic jam ahead of him.
An observer located near these two sources of sound will He slows down to 36 km/hour. He finds that traffic has
hear j
CBSE AIPMT 2012 eased and a car moving ahead of him at 18 km/h is
(a) 4 beat/s with intensity ratio 25 : 16 between waxing and honking at a frequency of 1392 Hz. If the speed of sound
waning is 343 m/s, the frequency of the honk as heard by him will
(b) 8 beat/s with intensity ratio 25 : 16 between waxing and be j
AIPMT 2014
waning (a) 1332 Hz (b) 1372 Hz (c) 1412 Hz (d) 1454 Hz
(c) 8 beat/s with intensity ratio 81 : 1 between waxing and 48 Doppler’s effect in sound in addition of relative velocity
waning between source and observer, also depends while
(d) 4 beat/s with intensity ratio 81 : 1 between waxing and source and observer or both are moving. Doppler effect
waning
in light depend only on the relative velocity of source and
41 A source of unknown frequency gives 4 beat/s when observer. The reason of this is
sounded with a source of known frequency 250 Hz. The (a) Einstein mass-energy relation
second harmonic of the source of unknown frequency (b) Einstein theory of relativity
gives 5 beat/s when sounded with a source of frequency (c) Photoelectric effect (d) None of the above
513 Hz. The unknown frequency is j
NEET 2013
49 Two cars moving in opposite directions approach each
(a) 254 Hz (b) 246 Hz
other with speed of 22 m/s and 16.5 m/s respectively.
(c) 240 Hz (d) 260 Hz
The driver of the first car blows a horn having a
DAY SEVENTEEN WAVES 179

frequency 400 Hz. The frequency heard by the driver of as shown in the figure. The observer is at rest. The
the second car is [velocity of sound 340 m/s] j NEET 2017 apparent frequency observed by the observer (velocity
(a) 350 Hz (b) 361 Hz of sound in air is 330 ms − 1), is j CBSE AIPMT 2015

(c) 411 Hz (d) 448 Hz vs


50 A siren emitting a sound of frequency 800 Hz moves away
from an observer towards a cliff at a speed of 15ms −1.
Then, the frequency of sound that the observer hears in 60°
the echo reflected from the cliff is
S O
(Take, velocity of sound in air = 330 ms −1) j NEET 2016 (a) 100 Hz (b) 103 Hz (c) 106 Hz (d) 97 Hz
(a) 800 Hz (b) 838 Hz (c) 885 Hz (d) 765 Hz 52 The driver of a car travelling with speed 30 ms −1 towards
51 A source of sound S emitting waves of frequency 100 Hz a hill sounds a horn of frequency 600 Hz. If the velocity of
and an observer O are located at some distance from sound in air is 330 ms −1 , the frequency of reflected sound
each other. The source is moving with a speed of as heard by driver is j CBSE AIPMT 2009
19.4 ms− 1at an angle of 60° with the source observer line
(a) 550 Hz (b) 555.5 Hz (c) 720 Hz (d) 500 Hz

DAY PRACTICE SESSION 2

PROGRESSIVE QUESTIONS EXERCISE


1 An observer standing on the sea coast finds that 54 5 The frequency changes by 10% as the source
ripples reach the surface per minute. If the wavelength of approaches a stationary observer with constant speed
the ripples be 10 m, what is the wave velocity? vs . What would be the percentage change in frequency
(a) 54 m/s (b) 9 m/s as the source recedes the observer with the same
(c) 10 m/s (d) 6 m/s speed? Given, that vs << v (v = speed of sound in air)
2 A wave travelling in positive x-direction with (a) 14.3% (b) 20%
A = 0 . 2 m, velocity = 360 ms −1 and λ = 60 m, then (c) 16.7% (d) 10%
correct expression for the wave is 6 The graph between distance of source and observer
 x  and apparent frequency in the case of Doppler’s effect
(a) y = 0.2 sin 2 π  6 t + 
  60   will be
 x 
(b) y = 0.2 sin π  6 t + 
  60   (a) (b)
s s n
 x 
(c) y = 0.2 sin 2 π  6 t −  n
  60  
 x  (c) (d)
(d) y = 0.2 sin π  6 t −  s
  60  
s
n n
3 In a resonance pipe the 1st and 2nd resonances are
7 An open pipe is suddenly closed at one end with the
obtained at depths 22.7 cm and 70.2 cm, respectively.
result that the frequency of third harmonic of the closed
What will be the end correction?
pipe is found to be higher by 100 Hz, then the
(a) 1.05 cm (b) 1115.5 cm (c) 92.5 cm (d) 113.5 cm
fundamental frequency of open pipe is
4 A car is moving towards a high cliff. The car driver (a) 480 Hz (b) 300 Hz
sounds a horn of frequency f . The reflected sound heard (c) 240 Hz (d) 200 Hz
by the driver has a frequency 2f . If v be the velocity of
8 An open pipe resonates with a tunning fork of frequency
sound, then the velocity of the car, in the same velocity
500 Hz. It is observed that two successive node are
units will be
v v formed at distance 16 cm and 46 cm from the open end.
(a) (b) The speed of sound in air in the pipe is
2 3
v v (a) 230 m/s (b) 300 m/s
(c) (d)
4 2 (c) 320 m/s (d) 350 m/s
180 40 DAYS ~ NEET PHYSICS DAY SEVENTEEN

9 Two tuning forks P and Q when set vibrating, give 13 Two points on a travelling wave having frequency 500 Hz
4 beats/s. If a prong of the fork P is filed, the beats are and velocity 300 m/s are 60° out of phase, then the
reduced to 2/s. What is frequency of P, if that of Q is minimum distance between the two points is
250 Hz? (a) 0.2 (b) 0.1
(a) 246 Hz (b) 250 Hz (c) 254 Hz (d) 252 Hz (c) 0.5 (d) 0.4
10 An observer standing at station observes frequency 14 A longitudinal wave is represented by
219 Hz when a train approaches and 184 Hz when a train  x
x = x 0 sin 2π nt − 
goes away from him. If velocity of sound in air is 340 m/s,  λ
then velocity of train and actual frequency of whistle will be The maximum particle velocity will be four times the wave
(a) 15.5 m/s, 200 Hz (b) 19.5 Hz, 205 Hz velocity, if
(c) 29.5 m/s, 200 Hz (d) 32.5 Hz, 205 Hz πx 0
(a) λ = (b) λ = 2 πx 0
11 A boy is sitting on a swing 4
πx 0
and blowing a whistle at a (c) λ = (d) λ = 4 πx 0
2
30°

frequency of 1000 Hz. The


2m
swing is moving to an 15 A source of sound S is moving with a velocity of 50 m/s
angle of 30° from the towards a stationary observer. The observer measures
vertical. The boy is at 2m the frequency of the source as 1000 Hz.
h
from the point of support What will be the apparent frequency of the source when it
of swing and a girl stands Girl is moving away from the observer after crossing him?
in front of swing. Then, the maximum frequency she will The velocity of the sound in the medium is 350 m/s.
hear is (given velocity of sound = 330 m/s).
(a) 750 Hz (b) 857 Hz
(a) 1000 Hz (b) 1001 Hz (c) 1143 Hz (d) 1333 Hz
(c) 1007 Hz (d) 1011 Hz
16 While measuring the speed of sound by performing a
12 A transverse wave propagating on a stretched string of resonance column experiment, a student gets the first
linear density 3 × 10−4 kgm −1 is represented by the resonance condition at a column length of 18 cm during
equation y = 0.2 sin (1.5x + 60 t ) winter. Repeating the same experiment during summer,
where, x is in metres and t is in seconds. The tension in she measures the column length to be x cm for the
the string (in newton) is second resonance. Then,
(a) 0.24 (b) 0.48 (c) 1.20 (d) 1.80 (a) 18 > x (b) x > 54 (c) 54 > x > 36 (d) 36 > x > 18

ANSWERS
SESSION 1 1 (b) 2 (a) 3 (c) 4 (d) 5 (c) 6 (a) 7 (c) 8 (a) 9 (b) 10 (d)
11 (b) 12 (d) 13 (a) 14 (b) 15 (a) 16 (b) 17 (a) 18 (c) 19 (c) 20 (a)
21 (a) 22 (a) 23 (c) 24 (a) 25 (c) 26 (b) 27 (c) 28 (c) 29 (c) 30 (d)
31 (c) 32 (b) 33 (a) 34 (b) 35 (d) 36 (d) 37 (b) 38 (c) 39 (a) 40 (d)
41 (a) 42 (d) 43 (d) 44 (b) 45 (a) 46 (b) 47 (c) 48 (b) 49 (d) 50 (b)
51 (b) 52 (c)
SESSION 2 1 (b) 2 (c) 3 (a) 4 (b) 5 (d) 6 (d) 7 (d) 8 (b) 9 (b) 10 (c)
11 (c) 12 (b) 13 (b) 14 (c) 15 (a) 16 (b)
DAY SEVENTEEN WAVES 181

Hints and Explanations


1 Propagation of longitudinal waves 8 Velocity (v ) is given by 13 Compare with y = A sin (ω t − k x + φ)
through a medium leads to transmission Distance 600 Here, ω = 7 π and k = 0.04
v= = = 300 ms −1
of energy through the medium. Time 2 Therefore,
2 Transverse waves are produced by Further, wavelength is given by ω
v c = = 7 π / 0.04 = 175 π ms −1
a motorboat sailing in water. λ= k
ν
dy
3 Given, y = A sin(ωt − k x ) Substituting v = 300 ms −1 , ν = 500 Hz, 14 = 0.5 × 10 π cos (10 πt − 2 x )
dt
As we know that wave velocity is given 3
we get λ= m = 5π cos (10 πt − 2 x )
by 5
 dy 
λ ωλ = 5 π ms −1
…(i) T =
2π  Therefore, number of wavelengths in  
vw = =  dx  max
T 2π  ω  600 m is given by
and maximum particle velocity is given Distance x 600 15 The given waves are
n= = = = 1000
Wavelength λ  
y 1 = 10−6 sin 100 t +   + 0.5 m
by 3 /5 x
v ρ = Aω …(ii)   50  
 A = amplitude 
9 Velocity of sound v = nλ
 x 
ω = angular frequency  v1 n λ and y 2 = 10−6 cos 100 t +   m
= 1 1 [but n1 = n2 ]   50  
So, as Eq. (i) is equal to Eq. (ii), v2 n2 λ2
 π
⇒y 2 = 10−6 sin 100 t +   +
x
ωλ v m
Aω =

, λ = 2π A λ2 = λ1 2 = λ1 × 10   50  2 
v1
 π 
λ2 = 10λ1 Qsin  + θ = cos θ
4 The given equation is  2  
π
y ( x, t ) = 8.0 sin  0.5 πx − 4 πt −  10 The time taken by the reflected wave to Hence, the phase difference between the
 4 return to the source should not be more waves is
π  3.14 0.5 rad
∆φ =  − 0.5 rad = 
…(i) than 0.1s. Hence, total distance travelled
− 
by the sound in going and coming back 2   2 
The standard wave equation is should not be more than 50 m.
y = a sin (kx − ω t + φ) …(ii) = 1.07 rad
11 The general equation of the wave in
On comparing Eqs. (i) and (ii), 16 y = A cos2 2 π nt − 2 π x 
we get k = 0.5 π, ω = 4 π negative X -axis direction is given by  λ 
∴ Speed of transverse wave y = A sin (ωt + kx + φ) …(i)  1 − cos 2 (2 πnt − 2 π x/λ )
ω 4π = A
v = = = 8 ms −1 Given equation is  2 
k 0.5 π  2π x 
y = 10−6 sin(100t + 20 x + π/4) m y = A / 2 1 − cos (2 π (2n )) t −
5 In the first medium, frequency …(ii)  λ/ 2 
v On comparing Eq. (ii) with Eq. (i), we …(i)
ν=
λ get Now, general wave equation is
as ν′ = v [remain same] ω = 100, k = 20 y = A cos (ωt − kx )
2π 

v ′ 2v
=
v
= ⇒ λ′ = 2 λ Hence, wave velocity is given by = A cos  2 πνt − x …(ii)
λ′ λ′ λ ω 100  λ 
c= = = 5ms −1
6 We know the velocity of sound is given k 20 On comparing Eqs. (i) and (ii),
γ π we get Amplitude = A /2,
by v = 12 Given, amplitude of wave, A = 2 cm frequency ν = 2n and wavelength = λ /2
ρ
direction = +ve x direction
At constant temperature, Velocity of wave v = 128 ms −1 17 Given, a = 1 m, k = 2 π , λ = 2 π
pV = constant λ
Volume of gas, and length of string, 5λ = 4 As y = a sin (kx − ωt )
2 πx
= sin  − 2 π × t  = sin ( x − 2t )
M pM We know that, 1
V= ⇒ = constant
ρ ρ 2π 2π × 5  2π π 
k = = = 7.85
As M is constant, therefore λ 4 a2 2
p ω 18 We have, =
= constant and v = = 128 ms −1 a1 1 2
ρ k  a2 
2  + 1
[ω = Angular frequency] I max  a2 + a1  a1
Hence, change in pressure has no effect =  = 
on velocity of sound. ⇒ ω = v × k = 128 × 7.85= 1005 I min  a2 − a1   a2
− 1 

So, option (a) is correct.  a1 
As, the wave travelling towards + x-axis 2
7 As there is no effect of change in  3
=   = 9:1
is given by
pressure on velocity of sound in air, and  1
velocity ∝ T , therefore, when y = A sin (kx − ωt )
So, y = 2 cm sin (7.85 x − 1005 t ) a max 9 3
temperature is doubled velocity ∴ = = = 3:1
becomes 330 2 m/s. y = (0.02) m sin (7.85 x − 1005 t ) a min 1 1
182 40 DAYS ~ NEET PHYSICS DAY SEVENTEEN

19 Here, R2 = a2 + b2 + 2ab cos θ ∴ ν1 l1 = ν2 l2 = ν2 l3 = k Fundamental frequency of open organ


v
Hence, a = a + a + 2 aa cos φ
2 2 2 From Eq. (i), pipe n 0 =
2l ′
k k k
1
⇒ cos φ = − or φ = 120° l1 = , l2 = , l3 = Since, fundamental frequency of closed
2 v1 v2 v3
organ pipe is equal to the second
k overtone of an organ pipe open at both
20 Intensity of sound Original length l =
2 ν end.
P 1 I1  r2  3v
I= or I ∝ or =  Here, l = l1 + l2 + l3 ∴ n ′ = 3 n0 ⇒ n ′ =
4 πr 2 r2 I2  r1  2l ′
k k k k
Here, r1 = 2 m , r2 = 3 m = + + v 3v
ν ν1 ν2 ν3 Q nc = n ′ ⇒ =
Substituting the values, we have 4l 2l ′
1 1 1 1 l ′ = 6l = 6 × 20 = 120 cm
I1  3  9
2
= + +
=  = ν ν1 ν2 ν3
I2  2  4 30 lc = 0.85 m
2π π 26 Initially number of beats per second = 5 v 340 ms −1
21 = ⇒ λ = 40 cm ∴ Frequency of pipe f0 = = = 100 Hz
λ 20 4lc 4 × 0.85 m
= 200 ± 5 = 195Hz or 205 Hz …(i)
Separation between two consecutive f n = (2n + 1) f 0 = f 0 , 3 f 0 , 5 f 0 ,
λ 40 Frequency of second harmonic of the
nodes = = = 20 cm pipe = 2n and number of beats in this
7 f 0 , 9 f 0 , 11 f 0 , 13 f 0
2 2 = 100 Hz, 300 Hz, 500 Hz,
case = 10
22 Five segments means five loops. One ∴ 2n = 420 ± 10 700 Hz, 900 Hz, 1100 Hz
λ
loop length is . ⇒ n = 205 Hz or 215Hz …(ii) 31 Fundamental frequency for an open
2 From Eqs. (i) and (ii), it is clear that v
5λ organ pipe is given as f =
Hence, = 10 m n = 205 Hz 2L
2 where, L is the length of the open organ
∴ λ= 4m 27 Fundamental frequency of closed pipe
pipe.
v 20 v
Now, f = = = 5 Hz = = 220 Hz Third harmonic for a closed organ pipe
λ 4 4l 3v
⇒ v = 220 × 4l is given as f ′ =
23 Fundamental frequency 4L ′
1
1 If of the pipe is filled with water, then where, L ′ is the length of closed organ
f ∝ 4
pipe.
l 3l
remaining length of air column is . According to the question,
4
60 cm 30 cm 20 cm f = f′
Now, fundamental frequency
v 3v 2
v v = ⇒ L = L′
= = 2L 4L ′ 3
Given, f1 : f2 : f3 = 1 : 2 : 3  3l  3l
4 
 4 Given, L ′ = 20 cm
1 1 1
or : : = 1:2:3 2
⇒ L = × 20 cm =
40
cm = 13.3 cm
l1 l2 l3 First overtone 3 3
1 1 1 = 3 × fundamental frequency
or l1 : l2 : l3 = : : 3v v 220 × 4l 32 Thinking Process Frequency of nth
1 2 3 = = = = 880 Hz
3l l l harmonic in a closed end tube
or l1 : l2 : l3 = 6 : 3 : 2 (2n − 1) v
6 28 The frequency of note emitted from the ⇒ f = ; n = 1, 2, 3, K
l1 = × 110 = 60 cm 4l
11 pipe for v being velocity of sound in air, Also, only odd harmonics exists in a
3 is
l2 = × 110 = 30 cm closed end tube.
f ′ = n  
11 v
Now, given two nearest harmonics are of
and l3 =
2
× 110 = 20 cm  4l  frequency 220 Hz and 260 Hz.
11 and l is length of pipe. (2n − 1) v
∴ = 220 Hz ...(i)
24 l1 l2 l3 f ′ = n × fundamental frequency 4l
n1 n2 n3 We know that, human ear can hear Next harmonic occurs at
frequencies upto 20000 Hz. (2n + 1) v
1 T 1 T 1 T = 260 Hz ...(ii)
n1 = , n2 = , n3 = 20000 = n × 1500 4l
2l1 µ 2l2 µ 2l3 µ 20000 On subtracting Eq. (i) from Eq. (ii), we
⇒ n= ≈ 13
1 T 1500 get
or n = [l = l1 + l2 + l3 ]
2l µ Maximum possible harmonics obtained {(2n + 1) − (2n − 1)} v
= 260 − 220
1 2l 1 1 1 are 1, 3, 5, 7, 9, 11, 13 4l
∴ = = + +
2   = 40 ⇒
Hence, man can hear upto 13th v v
n T n1 n2 n3 = 20 Hz
µ harmonic = 7 − 1 = 6  4l  4l
So, number of overtones heard = 6 ∴ Fundamental frequency of the system
25 The fundamental frequency of string v
29 Fundamental frequency of closed organ = = 20 Hz
1 T 4l
ν= pipe n c =
v
2l m 4l
DAY SEVENTEEN WAVES 183

33 According to question, we have 38 Number of beats is given by where, ν is the frequency.


n = ν1 − ν2 ⇒ v = ν[2(l 2 − l 1 )] [ from Eq. (i)]
Wavelength of transverse pulse
c c 1 1  Here, ν = 320 Hz, l 2 = 0.73 m, l 1 = 0.20
v or n = − or n = c  −
λ= …(i) m
f λ1 λ2  λ1 λ2 
(v = velocity of the wave; ⇒ v = 2[320(0.73 − 0.20)]
Substituting c = 396 ms −1 , λ1 = 99 cm = 2 × 320 × 0.53
f = frequency of the wave)
= 0.99 m; λ2 = 100 cm = 1 m, we get n = 339.2 ms −1 ~ − 339 ms −1
T
As we know v = …(ii) =4
µ 45 The frequency of vibration of a string
(T = tension in the spring; µ = mass per 39 As we know that n=
P T
unit length of the rope) Beat frequency = f1 ~ f2 = n − (n − 1) = 1 2l m
From Eqs. (i) and (ii), we get and similarly for n and n + 1 Also number of loops
1 T Beat frequency = n + 1 − n = 1 = Number of antinodes
λ= ⇒ λ∝ T
f µ 40 Given, y 1 = 4sin 600 πt Hence, with 5 antinodes and hanging
So, for two different case, we get mass of 9 kg
and y 2 = 5 sin 608 πt
we have, P = 5and T = 9 g
λ2 T2 m1 + m2 Comparing with general equation
= = 3 9g
λ1 T1 m2 y = asin2πft ⇒ n1 =
2l m
We get, f1 = 300 Hz and f2 = 304 Hz
with 3 antinodes and hanging
34 For an open organ pipe, So,number of beats = f2 − f1 = 4 s −1
n mass M.
νn = v , where n = 1, 2, 3, … We know that, we have, P = 3 and T = Mg
2L 2 2
For second overtone n = 3, I max  a1 + a2   4 + 5 n2 =
B Mg
=  =  = 81
3 I min  a1 − a2   4 − 5 2l m
v2 O = v
2L1 5 9g 3 Mg
41 Given, n1 = n2 ⇒ =
L1 = length of open organ pipe …(i) 2l m 2l m
For closed organ pipe 254 508 5 be M = 25kg
at
 2n + 1  v 46 νa′ = c
ν=
320
× 280
νn =   250 513
 4L  c −u 320 − 4
where, n = 0, 1, 2, 3, ... eat 320
246 492 21 b = × 280 Hz
Ist overtone for closed organ pipe, n = 1 316
3 Hence, unknown frequency is 254 Hz. c 320
ν1C = v …(ii) ν′ r = ν= × 280
4L c +u 320 + 4
3v 3
42 Let n p be the frequency of piano 320
Q ν2 O = ν1C ⇒ = v ⇒ L1 = 2L = × 280 Hz
2L1 4L As (n p ∝ T ) 324
ν′ a − ν′ r = 320 × 280 
1 1 
n f = frequency of tuning fork = 512 Hz −
35 Statement (d) is not true, because at the  316 324 
open ends pressure change will be zero. x = Beat frequency = 4 beats/s, which is 320 × 280
decreasing (4 → 2) after changing the = × 8=7
36 Let λ1 = 5.0 m, v = 330 ms −1 316 × 324
tension of piano wire.
and λ2 = 5.5 m
Also, tension of piano wire is increasing 47 v 0 = 36 km/h = 10 m/s O
v
We have, v = n λ ⇒ n = …(i) so n p ↑
λ v s = 18 km/h = 5m/s S f =1392 Hz
The frequency corresponding to Hence, n p ↑ − n f = x ↓ → wrong
v + v0 
wavelength λ1 , f′= f  
n f − n p ↑ = x ↓ → correct  v + vs 
v 330
n1 = = = 66 Hz  343 + 10 
λ1 5.0 n p = n f − x = 512 − 4 = 508 Hz
= 1392 ×   Hz
c  343 + 5 
The frequency corresponding to 43 ν′ = ν
wavelength λ2 , c − vs = 1392 ×
353
Hz = 1412 Hz
v 330 1 348
n2 = = = 60 Hz Substituting νs = c, we get
λ2 5.5 10 48 According to Einstein theory of
ν′ 10
Hence, number of beats per second = relativity, the velocity of the observer is
= n1 − n2 = 66 − 60 = 6 ν 9 neglected w.r.t. the light velocity.

37 Given, n1 − n2 = 12 44 For first resonance, l 1 = λ 49 Thinking Process When both source and
4 observer are moving towards each other,
v

v
= 12 3λ
For second resonance, l 2 = apparent frequency is given by
λ1 λ2 4  v + v0 
v v 100 v 3λ λ fa = f0  
− = 2v − ∴ (l2 − l1 ) = −  v − vs 
50 51 51 4 4
100 100 or λ = 2( l 2 − l 1 ) …(i) where, f 0 = original frequency
2v As, velocity of sound wave is given as, of source
⇒ = 12 ⇒ v = 306 ms −1
51 v = νλ v s = speed of source
184 40 DAYS ~ NEET PHYSICS DAY SEVENTEEN

v 0 = speed of observer  330   330  4 When the sound is reflected from the
= 100  = 100 3203
v = speed of sound  300 − 97
.  .  cliff, it approaches the driver of the car.
Frequency of the horn, Therefore, the driver acts as an observer
= 103.02 Hz ≈ 103 Hz
f 0 = 400 Hz and both the source (car) and observer
Speed of observer in the second car, 52 Concept Use Doppler’s effect. are moving.
v 0 = 16. 5 m/s According to Doppler's effect, whenever Hence, apparent frequency heard by the
there is a relative motion between a observer (driver) is given by
vs = 22 m/s
source of sound and the observer  v + vo 
v0 f′= f   …(i)
=16.5 m/s (listener), the frequency of sound heard  v − vs 
by the observer is different from the
Source observer where, v = velocity of sound,
actual frequency of sound emitted by
Speed of source, v o = velocity of car = v s
source.
v s = speed of first car = 22 m/s Thus, Eq. (i) becomes
hill
 v + vo 
Frequency heard by the driver in the ∴ 2f = f  
second car Case I n n′  v − vo 
 v + v0  30 ms–1
fa = f0   340 + 16.5  s or 2v − 2v o = v + v o or 3v o = v
 = 400   O
 v − vs   340 − 22  v
or vo =
= 448 Hz n′ 3
Case II n′′
30 ms–1
50 According to question, situation can be O 5 When the source approaches the
v observer
drawn as follows. [for case I] n′ = n …(i) −1
v − 30  v   v 
f1 = f   = f 1 − s 
 n = frequency emitted by car   v − vs   v 
 v = velocity of sound 
≈ f  1 + s 
Cliff v
15 m/s
v + 30  v 
Source [for case II] n ′ ′ = n′ …(ii)
Observer v  f1 − f  v
Frequency of sound that the observer or   × 100 = s × 100 = 10
hear in the echo reflected from the cliff  n′ ′ = frequency heard by   f  v
is given by  the driver after reflection  In second case when the source recedes
 v  From Eqs. (i) and (ii), we get the observer
f′=   v + 30 360 −1
 v − vs  n′ ′ = n= × 600 = 720 Hz  v   v 
v − 30 300 f2 = f   = f 1 + s 
where, f = original frequency of source;  v + vs   v 
v = velocity of sound
≈ f  1 − s 
v
v s = velocity of source SESSION 2  v 
 330  1 Frequency = 54  f2 − f 
So, f′=   800 = 838 Hz ∴
v
 330 − 15 60  × 100 = − s × 100 = − 10%
 f  v
54
and c = fλ= × 10 = 9 m/s
51 Given, as a source of sound S emitting 60 In the first case observed frequency
waves of frequency 100 Hz and an increases by 10% while in the second
observer O are located at some distance. 2 General equation for a plane progressive case observed frequency decreases by
Such that, source is moving with a wave is given by 10%.
speed of 19.4 m/s at angle 60° with y = A sin (ωt − kx )
6 Since, the apparent frequency does not
= A sin 2π  νt − 
source-observer line as shown in figure. x
…(i)
vs sin 60°  λ depend on the distance between source
vs −1 and observer, hence option (d) is
Now, v = 360 ms , λ = 60 m [given]
correct.
v
⇒ ν = = 6 Hz 7 Fundamental frequency of open organ
λ
v
pipe =
Substituting A = 0.2 ms −1 , ν = 6 Hz, 2l
λ = 60 m in Fundamental of third harmonic of
60° 3v
Eq. (i), we get closed pipe =
S Doppler 4l
 x 
vs cos 60° line . sin 2 π  6t −
y = 02  3v v 3v 2v
  60   ∴ = 100 + ⇒ − = 100
The apparent frequency heared by 4l 2l 4l 4l
observer v v
3 For end correction x, ⇒ = 100 ⇒ = 200 Hz
 v  l2 + x 3λ / 4 4l 2l
f 0 = fs   = =3
 v − v s cos 60 °  l1 + x λ/4 8 Distance between two consecutive
  l − 3l1 λ
x= 2 nodes, = 46 − 16
 330  2 2
= 100 
1 . − 3 × 227
702 . λ = 60 cm = 0.6 m
 330 − 19.4 ×  = = 1.05 cm
 2 2 v = nλ = 500 × 0.6 = 300 m/s
DAY SEVENTEEN WAVES 185

9 There are four beats between P and Q, Potential energy = kinetic energy = 2 π nx 0 cos 2 π  nt − 
x
1  λ
therefore the possible frequencies of P mv 2 = 2mg (1 − cos 30° )
are 246 or 254 (i.e. 250 ± 4) Hz. 2
Particle velocity will be maximum when
v = 2.3 m/s
cos 2π  nt −  is maximum i.e. 1.
When the prong of P is filed, its x
v + v0 
frequency becomes greater than the n max = ×n λ
original frequency. If we assume that the v − vs λ
original frequency of P is 254, then on 330 + 0 Wave velocity = = nλ
= × 1000 T
filing its frequency will be greater than 330 − 2.3 Now, 2 πnx 0 = 4nλ
254. The beats between P and Q will be
= 1007 Hz 2 π nx 0 πx 0
more than 4. But it is given that the λ= = .
4n 2
beats are reduced to 2, therefore 254 is 12 Equation of wave y = 0.2sin (1.5x + 60t )
not possible. Comparing with standard equation, we 15 When the source is coming to the
Therefore, the required frequency must get y = A sin(kx + ω t ) stationary observer,
be 246 Hz. (This is true, because on k = 1.5 and ω = 60  v 
n′ =   n
filing the frequency may increase to ω 60  v − vs 
248 Hz, giving 2 beats with Q of ∴Velocity of wave v = =
k 1.5  350 
frequency 250 Hz). = 40 m/s or 1000 =   n
 350 − 50 
10 When train is approaching frequency Velocity of wave in a stretched string,
n =  1000 ×
T 300 
heared by observer is v = or  Hz
 v   350 
m
na = n   When the source is moving away from
 v − vs  where, m is linear density and
T is tension in the string. the stationary observer.
 340   v 
219 = n   …(i) So, T = v 2 m = (40)2 × 3 × 10−4 = 0.48
 340 − v s  n ′′ =   n
 v + vs 
When train is goes away frequency 13 As v = nλ  350   1000 × 300 
heard by the observer is V 300 3 =   
∴ λ= = = m  350 + 50   350 
 v  n 500 5
nr = n   = 750 Hz
 v + vs  Now, phase difference
 340  π v1
= 2 × path difference 16 Here, l1 = 18 cm, f =
184 = n   …(ii) λ 4l1
 340 + v s 

On solving Eqs. (i) and (ii), we get 60° = × path difference ⇒ f =
3 v2
λ 4 l2
n = 200 Hz and v s = 29.5 m/s 60°× π 2 π × 5
or = × path difference where, l2 = x according to given
11 The maximum velocity of swing will be 180° 3
3 × 60° × π situation and also v 1 < v 2 as during
when it crosses the lowest point and ∴ Path difference = = 01
. summer temperature would be higher.
minimum velocity when it is at a height 2 π × 5 × 180° 3v 2 v v
of h, = 1 ⇒ l2 = 3l1 × 2
14 Particle velocity = d ( x ) 4l2 4l1 v1
i.e. at its maximum displacement the dt
⇒ x = 54 × (a quantity greater than 1)
potential energy = mgh = mgL (1 − cos θ) d  x 
= x 0 sin 2 π  nt −  ∴ x > 54
= 2mg (1 − cos 30° ) dt   λ  
DAY EIGHTEEN

Unit Test 3
(Waves and Oscillations)

1 When an observer moves towards a stationary source 5 Two sources of sound A and B produces the wave of
with a certain velocity, he observes an apparent 350 Hz, in the same phase. The particle P is vibrating under
frequency f . When he moves away from the source with the influence of these two waves, if the amplitudes at the
same velocity, the observed frequency is 0.8 f . If the point P produces by the two waves is 0.3 mm and 0.4 mm,
velocity of sound is v, then the speed of the observer is then the resultant amplitude of the point P will be when
v v AP − BP = 25 cm and the velocity of sound is 350 ms −1.
(a) (b)
4 8 (a) 0.7 mm (b) 0.1 mm (c) 0.2 mm (d) 0.5 mm
v
(c) (d) None of these 6 Equation of a plane progressive wave is given by
9
 x
2 A transverse wave y = 0. 05 sin ( 20πx − 50 π t ) in metre, y = 0 . 6 sin 2π t −  . On reflection from a denser
 2
is propagating along positive X -axis on a string. A light
medium, its amplitude becomes 2/3 of the amplitude of
insect starts crawling on the string with the velocity of
the incident wave. The equation of the reflected wave is
5 cms −1 at t = 0 along the positive X -axis from a point
(a)y = 0. 6 sin 2 π  t +  (b) y = − 0. 4 sin 2 π  t + 
x x
where x = 5 cm. After 5s the difference in the phase of its
 2  2
position is equal to
(c)y = 0. 4 sin 2 π  t +  (d) y = − 0. 4 sin 2 π  t − 
x x
(a) 150 π (b) 250 π  2  2
(c) 10 π (d) 5 π
3 Equation of a progressive wave is given by 7 Two travelling waves y1 = A sin [k ( x − ct )] and
π y 2 = A sin [k ( x + ct )] are superimposed on string. The
y = 0.2 cos π  0.04 t + 0.02 x −  distance between adjacent nodes is
 6
c c π π
The distance is expressed in cm and time in second. (a) (b) (c) (d)
π 2π 2k k
What will be the minimum distance between two particles
π 8 A source of sound is moving with constant velocity of
having the phase difference of ?
2 20 ms −1 emitting a note of frequency 1000 Hz. The ratio
(a) 4 cm (b) 8 cm of frequencies observed by a stationary observer while
(c) 25 cm (d) 12.5 cm the source is approaching him and after it crosses him
will be (speed of sound, v = 340 ms −1)
4 Two waves represented by the following equations are
travelling in the same medium (a) 9 : 8 (b) 8 : 9 (c) 1 : 1 (d) 9 : 10
y1 = 5 sin 2π (75 t − 0.25x ) 9 A table is revolving on its axis at 5 rev/s. A sound source
y 2 = 10 sin 2π (150 t − 0.50 x ) of frequency 1000 Hz is fixed on the table at 70 cm from
the axis. The minimum frequency heard by a listener
I1
The intensity ratio of the two waves is standing at a distance very far from the table will be
I2
(speed of sound, v = 352 ms −1)
(a) 1 : 2 (b) 1 : 4 (a) 1000 Hz (b) 1066 Hz
(c) 1 : 8 (d) 1 : 16 (c) 941 Hz (d) 352 Hz
DAY EIGHTEEN UNIT TEST 3 (WAVES AND OSCILLATIONS) 187

10 The wavelength is 120 cm when the source is stationary. 19 The displacement of a particle varies with time according
If the source is moving with relative velocity of 60 ms −1 to the relation y = a sin ωt + b cos ωt .
towards the observer, then the wavelength of the sound (a)The motion is oscillatory but not SHM
wave reaching to the observer will be (b)The motion is SHM with amplitude a + b
(velocity of sound, v = 330 ms −1) (c)The motion is SHM with amplitude a 2 + b 2
(a) 98 cm (b) 140 cm (c) 120 cm (d) 1440 cm (d)The motion is SHM with amplitude a 2 + b 2

11 An observer moves towards a stationary source of sound, 20 The length of a simple pendulum executing SHM is
with a velocity one-fifth of the velocity of sound. What is increased by 21%. The percentage increase in the time
the percentage increase in the apparent frequency? period of the pendulum of increased length is
(a) 5% (b) 20% (c) zero d) 0.5% (a) 10% (b) 11% (c) 21% (d) 42%
12 In the experiment for the determination of the speed of 21 Two tuning forks when sounded together produce
sound in air using the resonance column method, the 10 beats/s. The first fork has the frequency 5% less than
length of the air column that resonates in the the standard fork and the second has the frequency 8%
fundamental mode, with a tuning fork is 0.1 m. When this higher than a standard one. What are the frequencies of
length is changed to 0.35 m, the same tuning fork the forks?
resonates with the first overtone. Calculate the end
(a)73 Hz, 83 Hz (b) 83 Hz, 93 Hz
correction. (c)103 Hz, 93 Hz (d) 113 Hz, 103 Hz
(a) 0.012 m (b) 0.025 m (c) 0.05 m (d) 0.024 m
22 A source of sound gives five beats per second, when
13 Two bodies M and N of equal masses are suspended sounded with another source of frequency100 s −1. The
from two separate springs of constants k1 and k 2 , second harmonic of the source, together with a source of
respectively. If they oscillate with equal maximum frequency 205 s −1 gives five beats per second. What is
velocities, then the amplitudes of M and N are in the ratio the frequency of the source?
of
(a) 105 s −1 (b) 205 s −1 (c) 95 s −1 (d) 100 s −1
(a) k1/ k2 (b) k2 / k1 (c) k1/ k2 (d) k2 / k1
23 A column of air at 88° C and a tuning fork produce
14 Two simple pendulums of length 0.5 m and 20 m 6 beats/s when sounded together. When temperature is
respectively are given small linear displacement in one 16° C the two produce only one beat per second. What is
direction at the same time. They will again be in the the frequency of tuning fork?
phase when the pendulum of shorter length has (a) 53.9 Hz (b) 97.7 Hz (c) 41.5 Hz (d) 87.7 Hz
completed, how many oscillations? 24 A steel wire of length 80 cm and area of cross-section
(a) 5 (b) 1 (c) 2 (d) 3 10−5 m 2 is joined with an aluminium wire of length 60 cm
15 A particle is executing SHM with an amplitude of 4 cm. and area of cross-section 3 × 10−5 m 2 . The composite
At the mean position the velocity of the particle is 10 cms −1. string is stretched by a tension 100 N. What is minimum
The distance of the particle from the mean position when frequency of tuning fork, which can produce standing
its speed becomes 5 cms −1 is wave in it with node at the joint?
(a) 3 cm (b) 5 cm (c) 2 3 cm (d) 2 5 cm (Given, ρsteel = 7800 kg m −3 and ρal = 2600 kg m −3 )
(a) 87.5 Hz (b) 97.8 Hz (c) 67.9 Hz (d) 89.5 Hz
16 In a Kundt’s tube experiment, the distance between
−1 25 A string of mass 2.5 kg is under a tension of 200 N. The
nodes was 8 cm. If velocity of sound in air is 330 ms
and total length of sounding rod is 160 cm, then what is length of the stretched string is 20.0 m. If the transverse
the velocity of sound in rod? jerk is struck at one end of the string, the disturbance will
(a) 6800 ms −1 (b) 7200 ms −1 reach the other end in
(c) 6200 ms −1 (d) 6600 ms −1 (a) 1 s (b) 0.5 s
17 A particle executes SHM between x = − A and x = + A. (c) 2 s (d) Data insufficient
The time taken for it to go from 0 to A /2 is T1 and to go 26 A spring of force constant k is cut into two pieces such
from A /2 to A is T2 . Then, that one piece is doubled the length of the other. Then,
(a)T1 < T2 (b)T1 > T2 (c)T1 = T2 (d)T1 = 2T2 the longer piece will have a force constant
18 A simple pendulum has a time period T1 when on the (a) 2k / 3 (b) 3 k / 2 (c) 3k (d) 6k
earth’s surface and T2 when taken to a height R above 27 The potential energy of a simple harmonic oscillator
the earth’s surface, where R is the radius of the earth. when the particle is half way to its end point is
The value ofT2 / T1 is 1 1 1 2
(a) E (b) E (c) E (d) E
(a) 1 (b) 2 (c) 4 (d) 2 8 4 2 3
188 40 DAYS ~ NEET PHYSICS DAY EIGHTEEN

28 An elastic ball is dropped from a height h on an elastic 36 When a certain volume of water is subjected to increase
floor. It collides against the floor elastically and rebounds of 100 kPa pressure, the volume of water decreases by
to the same height. What is the nature of its motion and 0.005%. The speed of sound in water must be
time period of return to the starting point? (a) 140 m/s (b) 300 m/s (c) 1400 m/s (d) 5000 m/s
(a) SHM, 8h /g (b) Oscillatory, 8h /g 37 A progressive wave moving along X -axis is represented
(c) SHM, 2 h /g (d) Oscillatory, 2 h /g  2π 
by y = A sin ( vt − x ) . The wavelength ( λ ) at which
 λ 
29 In a transverse progressive wave of amplitude A , the
maximum particle velocity is four times the wave velocity. the maximum particle velocity is 3 times the wave
Then, the wavelength of the wave is velocity is
πA πA
(c)   πA (d)   πA
A 2A 3 2
(a) (b) (c) π A (d) 2 π A (a) (b)
2 4 3 (3 π )  4  3
30 When a mass m is connected individually to two springs S1 38 The frequency of the first overtone of a closed pipe of
and S 2 , the oscillation frequencies are ν1 and ν 2 . If the length l1 is equal to that of the first overtone of an open
same mass is attached to the two springs as shown in pipe of length l2 . The ratio of their lengths ( l1 : l2 ) is
figure the oscillation frequency would be (a) 2 : 3 (b) 4 : 5
S1 m S2 (c) 3 : 5 (d) 3 : 4
39 A particle is executing SHM. Then, the graph of velocity
−1 as a function of displacement is a/an
1 1
(a) ν1 + ν2 (b) ν12 + ν22 (c)  +  (d) ν12 − ν22 (a) straight line (b) circle
 ν1 ν2 
(c) ellipse (d) hyperbola
31 The equation of a simple harmonic is given by
π 40 A long spring, when stretched by a distance x, has
y = 3 sin ( 50 t − x ),
2 potential energy U. On increasing the stretching to nx,
the potential energy of the spring will be
where, x and y are in metres and t is in seconds. The ratio
U
of maximum particle velocity to the wave velocity is (a) (b) nU
n
3 2
(a) 2 π (b) π (c) 3 π (d) π (c) n 2 U (d)
U
2 3 n2
32 A solid cylinder of mass 3 kg is rolling on a horizontal
surface with velocity 4ms −1. It collides with a horizontal 41 If the maximum velocity and acceleration of a particle
spring of force constant 200 Nm −1. The maximum executing SHM are equal in magnitude the time period
compression produced in the spring will be will be
(a) 0.5 m (b) 0.6 m (c) 0.7 m (d) 0.2 m (a) 1.57 s (b) 3.14 s
33 The amplitude of a particle executing SHM is 4 cm. As (c) 6.28 s (d) 12.56 s
the mean position, the speed of the particle is 16 cm/s. 42 Two pendulums have time periodT and 5T / 4. They start
The distance of the particle from the mean position at SHM at the same time from the mean position. What will
which the speed of the particle becomes 8 3 cm/s will be the phase difference between them after the bigger
be pendulum completed one oscillation?
(a) 2 3 cm (b) 3 cm (c) 1 cm (d) 2 cm (a) 45° (b) 90°
34 A simple harmonic oscillator consists of a particle of (c) 60° (d) 30°
mass m and an ideal spring with spring constant k. The 43 A sound has an intensity of 2 × 10−8 W/m 2 . Its intensity
particle oscillates with a time periodT . The spring is cut level (in decibel) is (log10 2 = 0 . 3)
into two equal parts. If one part oscillates with the same (a) 23 (b) 3
particle, the time period will be (c) 43 (d) 4.3
T T
(a) 2T (b) 2T (c) (d) 44 Two waves having the intensities in the ratio of 9:1
2 2
produce interference. The ratio of maximum to minimum
35 A train moving at a speed of 220 ms −1 towards a
intensity is equal to
stationary object, emits a sound of frequency 1000 Hz.
Some of the sound reaching the object gets reflected (a) 10 : 8 (b) 9 : 1
(c) 4 : 1 (d) 2 : 1
back to the train as echo. The frequency of the echo as
detected by the driver of the train is (speed of sound in 45 Which of the following is independent from others?
air is 330 ms −1) (a) Wavelength (b) Velocity
(a) 3500 Hz (b) 4000 Hz (c) 5000 Hz (d) 3000 Hz (c) Frequency (d) Amplitude
DAY EIGHTEEN UNIT TEST 3 (WAVES AND OSCILLATIONS) 189

Direction (Q. Nos. 46-50) In each of the following 48 Assertion (A) A tuning fork is in resonance with a closed
questions a statement of Assertion is given followed by a pipe. But the same tuning fork cannot be in resonance
corresponding statement of Reason just below it. Of the with an open pipe of the same length.
statements mark the correct answer as Reason (R) The same tuning fork will not be in resonance
(a) If both Assertion and Reason are true and Reason is the with open pipe of same length due to end correction of
correct explanation of assertion pipe.
(b) If both Assertion and Reason are true but the Reason is
49 Assertion (A) In everyday life, the Doppler’s effect is
not the correct explanation of Assertion
observed readily for sound waves than light waves.
(c) If Assertion is true but Reason is false
(d) If both Assertion and Reason are false Reason (R) Velocity of light is greater than the sound.
46 Assertion (A) Simple Harmonic Motion (SHM) is not a 50 Assertion (A) Velocity of particles while crossing mean
uniform motion. position (in stationary waves) varies from maximum at
anti-nodes to zero at nodes.
Reason (R) It is the projection of uniform circular motion.
Reason (R) Amplitude of vibration at anti-nodes is
47 Assertion (A) A hole were drilled through the centre of
maximum and at nodes, the amplitude is zero and all
earth and a ball is dropped into the hole at one end, it
particles between two successive nodes cross the mean
will not get out of other end of the hole.
position together.
Reason (R) It will come out of the other end normally.

ANSWERS
1. (c) 2. (d) 3. (c) 4. (d) 5. (d) 6. (b) 7. (d) 8. (a) 9. (c) 10. (a)
11. (b) 12. (b) 13. (d) 14. (b) 15. (c) 16. (d) 17. (a) 18. (d) 19. (d) 20. (a)
21. (a) 22. (a) 23. (c) 24. (d) 25. (b) 26. (b) 27. (b) 28. (b) 29. (a) 30. (b)
31. (b) 32. (b) 33. (d) 34. (c) 35. (c) 36. (c) 37. (d) 38. (d) 39. (c) 40. (c)
41. (c) 42. (b) 43. (c) 44. (c) 45. (d) 46. (b) 47. (c) 48. (c) 49. (b) 50. (a)

Hints and Explanations


1 Suppose, f 0 be the actual frequency. π 6 By reflection from a denser medium
∆φ =
2 phase reversal = 180°
Then,
2
Hence, path difference between them, New amplitude = × 0.6 = 0.4
v + vo 
f = f 0   …(i) λ λ π
3
 v  ∆x = × ∆φ = × Hence, new equation for the reflected
2π 2π 2 wave
 v − vo  λ 100
0.8 f = f 0   y = 0.4sin 2 π t + + 180° 
and …(ii) = = = 25 cm x
 v 
4 4  2 
On solving these two equations, we get, = − 0.4 sin 2 π ( t + x / 2 )
2
A 2ω 2 2
5  2 π × 75 
= 1 1 =   
v I1 1
vo = 4∴  = 7 The distance between adjacent nodes
9 I2 A2 ω2
2 2  10   2 π × 150  16
λ
x=
2 ∆φ = 2 π ∆x = k (∆x ) = k (vt ) 5 ∴ λ = v = 350 = 1 m = 100 cm
2
λ 2π
f 350 Also, k=
Here, vt = distance travelled by insect in λ
Also, path difference (∆ x ) between the π
given time interval. waves at the point of observation is Hence, x =
k
or ∆φ = (20 π ) (5 × 10−2 × 5) = 5π AP − BP = 25cm. Hence,
f a v + vs 340 + 20 9
2π 2 π  25  π 8 = = =
3 On comparing with ⇒ ∆φ = (∆ x ) = ×  = fr v − vs 340 − 20 8
λ 1  100  2
y = a cos (ω t + kx − φ), we get 9 For source vs
⇒ A = (a1 ) 2 + (a2 ) 2
k=

= 0.02 π . × 2 π × 5 = 22 ms −1
= rω = 070
λ = (03
. ) 2 + (0.4) 2 = 0.5 mm
⇒ λ = 100 cm
190 40 DAYS ~ NEET PHYSICS DAY EIGHTEEN

Minimum frequency is heard when the Velocity of sound in solid 21 Let, frequency of standard fork = ν
16
source is receding the man. It is given Velocity of sound in air 95
Frequency of first fork, ν1 = ν
by Total length of rod 100
v =
f min = f Distance between nodes Frequency of second fork, ν2 =
108
ν
v + vs
v solid 160 × 10−2 100
352 =
= 1000 × = 941 Hz 330 8 × 10−2 Number of beats = ν2 − ν1
352 + 22 ⇒ ν2 − ν1 = 10
Therefore, v solid = 6600 ms −1 108 95
 v  or ν− ν = 10
10 f′= f   17 Let the equation of SHM be 100 100
 v − vs  13
y = A sin ω t or ν = 10
 v − vs  100
⇒ λ′ = λ   A
Then, in the first case = A sinω T1
 v  2 or ν = 76.92 ≅ 77 Hz
 330 − 60  1 π
ω T1 = sin −1 = ν1 =
95
× 77 = 73 Hz
⇒ λ′ = 120   = 98 cm i.e. Therefore,
 330  2 6 100
π 108
Therefore, T1 = ν2 = × 77 = 83 Hz
11 When observer moves towards 6ω 100
stationary source, then apparent T 2π π π
frequency Also, T2 = − T1 = − = 22 The frequency of source can be 100 + 5
4 4ω 6ω 3ω
v + vo   v + v /5 and 100 − 5
f′=  f = f Therefore, T1 < T2
 v   v  i.e. 105 or 95
6 18 Time period of simple pendulum is 2nd harmonic is 210 or 200.
=f = 1.2 f given by
5 Since, 5 beats with 105 are possible, if
Increment in frequency = 0.2 f . T = 2π
l
= 2π
l source frequency is 210.
So, percentage change in frequency g GM /r 2 ∴Original frequency is 105 s −1 .
0.2f
= × 100 = 20% l 23 We know that ν ∝ v and v ∝ T
f = 2π r
GM
ν 88 v 88 273 + 88 19
12 Let x be the end correction, then l Therefore, = = =
Hence, T1 = 2πR ν16 v 16 273 + 16 17
according to question. GM
v
=
3v As ν16 < ν 88, hence frequency of tuning
l fork ν is less than the frequency of air
4 (l1 + x ) 4 (l2 + x ) And T2 = 2 π (2R )
GM column.
⇒ x = 2.5 cm = 0.025 m
Therefore,
T2
=2 Therefore, ν 88 = ν + 6
13 v max = Aω, for mass attached to spring ω T1 and ν16 = ν + 1
= k /M . Here, masses are equal. ν + 6 19
19 y = asin ω t + b cos ω t ∴ =
Therefore, A1 ω1 = A 2 ω2 ν+1 17
or A 1 k1 = A 2 k2 Let a = A cos θ ; b = A sinθ On solving, we get ν = 41.5 Hz
1 /2 ⇒ a2 + b 2 = A2 (cos 2 θ + sin2 θ ) = A2
A1  k2  k2 24 The frequency of the stretched string is
i.e. = =
A2  k1  A = a2 + b 2
or given by
k1
n T
Hence, Y = A cos θ sin ω t + A sin θ cos ω t ν=
14 nTs = (n − 1) T1 2l µ
= A sin(ω t + θ )
0.5 20 Therefore,
n 2π = (n − 1) 2 π = a2 + b 2 sin( ω t + θ )
g g n1 100
νsteel = …(i)
It is SHM with amplitude A = a2 + b 2 2 × 80 × 10−2 7800 × 10−5
or n = (n − 1) 40 ≅ 6 (n − 1)
Hence, n= 6/ 5≅ 1 20 Time period of a simple pendulum is and
given by n2 100
15 Here, v = ω [ A2 − y 2 ]1 /2 νal =
l 2 × 60 × 10−2 3 × 10−5 × 2600
or v 2 = ω2 [ A2 − y 2 ] T = 2π …(i)
g …(ii)
Also v max = ω A 21 121 On solving Eqs. (i) and (ii), we get
i.e. 10 = ω × 4 21% increase l ⇒ l ′ = l + l= l
100 100 n1 4
=
⇒ ω = 2.5 l′ 121 /100l n2 3
Putting v = 5 cms −1 , ω = 2.5 Therefore, T ′ = 2π = 2π
g g 4 100
and A = 4 cm νsteel = = 89.5 Hz
11 2 × 0.8 7800 × 10−5
(5)2 = (2.5)2 [(4)2 − ( y )2 ] = T [using Eq. (i)]
10 [from Eq. (i)]
25
⇒ = (4)2 − ( y )2 Hence, percentage increase in the time In composite string, different harmonics
2.5 × 2.5 period, give rise to a common frequency.
⇒ y = 16 − 4
2
11
∴ νsteel = νal = 89. 5 Hz
T −T
T′−T
⇒ y 2 = 12 ⇒ y = 2 3 cm × 100 = 10 × 100 = 10%
T T
DAY EIGHTEEN UNIT TEST 3 (WAVES AND OSCILLATIONS) 191

2. 5 30 Let k1 and k2 be the spring constants of 36 Volume elasticity


25 m = mass per unit length = kg/m
20 S 1 and S 2 , respectively. ∆p 100 × 103
B= = = 2 × 109
T = 200 N 1 k1 1 k2 ∆V / V 0.005 / 100
Then, ν1 = and ν2 =
200 × 200 2π m 2π m B
v=
T
=
200
= Speed of sound, v =
m 2.5 / 20 25 keq = k1 + k2 d
1 keq 1 k1 + k2
200 ν= = 2 × 109
= = 40 m/s 2π m 2π m =
5 103
1 k1 k2 1
i 20 = + = 4 π ν1 + 4 π2 ν22
2 2
∴ Required time = = = 0.5 s 2π m m 2π = 1400 m/s
v 40
⇒ ν= ν21 + ν22 37 Progressive wave, y = A sin  2π (vt − x )
26 As the spring is cut into two parts of  λ 
l 2l
length , 31 v max = aω and v = nλ

3 3 v max aω a(2 π n ) 2 πa v= vA
mg = = = λ
We know that, k = , here x is v nλ nλ λ
x 2 πa π 3π According to the question,
= = ka = × 3 = v max = 3 v
extension 2π / k 2 2
l l 2π
For spring of length extension = x 32 Loss in KE = Gain in spring energy ∴ 3v = vA
3 3 λ
1  k2  1 2
2l 2l mv 2 1 +  = k x max 2π
For spring of length extension = x 2  R2  2 ⇒ λ= A
3 3 3
where, k is the force constant.
For spring of length
2l
, we have spring Given, v = 4 m/s, m = 3 kg and 38 Here, v 1 = 3 v
3 k = 200 N/m 4 l1
3 2
constant k k2 1 v2 = v
2 For solid cylinder, =
R2 2 2l2
27 Potential energy, E = 1 /2m ω2 y 2 Given, v 1 = v 2
∴ × 3 × (4)2 1 +  = × 200 × x2max
1 1 1
At end point, y = A 2  2  2 The ratio of lengths
l1 3
=
Therefore, E = 1 / 2 m ω2 A2 …(i) The maximum compression in the l2 4
At half way y ′= A / 2 spring
39 From the relation, v 2 = ω2 (a2 − x2 )
x max = 0.6 m
Therefore, E ′ = 1 / 2m ω2 ( y ′)2
⇒ v 2 + ω2 x2 = ω2 a2
2
33 At mean position, velocity is maximum
A
= 1 / 2 m ω2 v max = aω v2 x2
…(ii) ⇒ +
=1
4 v 16 aω 2 2
a2
ω = max = =4
On comparing Eqs. (i) and (ii), we get a 4 Which is the equation of an ellipse.
E
E′ = ∴ v = ω a2 − y 2 ⇒ 8 3 = 4 42 − y 2
4
192 = 16(16 − y 2 ) ⇒ 12 = 16 − y 2 40 Potential energy of the spring,
28 The motion is under constant 1
y = 2 cm U = kx2
acceleration and therefore y is not 2
proportional to displacement. Hence, 34 Mass of the particle = m 1
the motion is oscillatory but not SHM. and U ′ = k (nx )2
Spring constant = k 2
Time of fall = Time of rise = t (say) 1
The time period of oscillator, U ′ = n2 kx2 = n2U
The time of rise is given by 2
m
1
− h = 0 × t − gt 2 T = 2π
2 k 41 According to question,
1 |v max |=|a max |i.e. ω2 a = a,
 using s = ut + 1 at 2  As k ∝ [where, l is the length of spring]
 2  l 2π
Q k ′ = 2k ⇒ ω =1=
2h T
Hence, t = m 1
g ∴ T ′ =2π = T ∴ T = 2π
2k 2 ⇒ T = 2 × 314
.
Therefore, time period of oscillation
35 From Doppler’s shifts, we knows for this ⇒ T = 6.28 s
2h 8h
=2 = case 42 When bigger pendulum of time period
g g
 v + vs  (5T / 4) completes one vibration, the
n′ = n  
29 ω A = 4 × v (wave velocity)  v − vs  smaller pendulum will complete (5/4)
vibrations. It means the smaller
or ω A = 4v = 4n λ  330 + 220   550 
= 1000   = 1000   pendulum will be leading, the bigger
or 2π nA = 4 nλ  330 − 220   110  pendulum by phase T /4 second = π / 2
πA rad = 90°.
or λ= = 5000 Hz
2
192 40 DAYS ~ NEET PHYSICS DAY EIGHTEEN

 I  46 In Simple Harmonic Motion (SHM), it cannot be in resonance with the given


43 L = 10 log 10  
 I0   v 
v = ω (a − y )
2 2 tuning fork of frequency ν  = .
 2 × 10−8   4L 
L = 10 log 10   As y changes, velocity v will also
−12
 10  49 Doppler’s effect is observed readily in
change. So, Simple Harmonic Motion
−12 sound waves due to the larger
[Q I 0 = 10 W/m ]2
(SHM) is not a uniform motion. wavelength. This effect is not followed
L = 10 log 10 (2 × 104 ) Simple Harmonic Motion (SHM) may be with light due to shorter wavelength.
L = 10 [ log 10 2 + log 10 (10)4 ] defined as the projection of uniform The velocity of light is 3 × 108 ms −1 and
L = 10 (43
. ) = 43 dB circular motion along one of the 332 ms −1 velocity of sound.
diameters of the circle.
I1 a12 9 50 Stationary wave is represented as
44 = =
I2 a22 1 47 The ball will not go out of the other end shown in figure.
of the hole, because it will execute
a1 9 3 A A
⇒ = = SHM. On reaching the other end of the
a2 1 1 hole, its velocity becomes zero and N
N
acceleration of ball will be maximum
I max (a1 + a2 ) (3 + 1 )
2 2 N
Then, = = and will be directed towards the centre
I min (a1 − a2 )2 (3 − 1 )2 of earth.
It is quite clear from figure that at nodes
(4)2
16 4
48 If a closed pipe of length L is in the amplitude is zero and velocity of
= = = resonance with a tuning fork of particle is also zero and at anti-nodes
(2)2 4 1 v the amplitude is maximum.
frequency ν, then ν =
Thus, I max : I min = 4 : 1 4L So that the velocity of particle is also
45 Amplitude is independent of An open pipe of same length L produces maximum and all particles cross mean
wavelength, velocity and frequency of v position between two successive modes.
vibrations of frequency . Obviously,
oscillation. 2L
DAY NINETEEN

Electrostatics
Learning & Revision for the Day
u Electric Charge u Motion of A Charged Particle u Electric Potential Energy
u Coulomb’s Law of Forces in An Electric Field u Equipotential Surface
between Two Point Charges u Electric Dipole u Conductors and Insulators
u Superposition Principle u Electric Flux (φE ) u Electrical Capacitance
u Electric Field u Gauss Law u Capacitor
u Continuous Charge Distribution u Electric Potential

If the charge in a body does not move, then the frictional electricity is known as static
electricity. The branch of physics which deals with static electricity is called
electrostatics.

Electric Charge
Electric charge is the property associated with matter due to which it produces and
experiences electric and magnetic effects.

Conservation of Charge
We can neither create nor destroy electric charge. The charge can simply be transferred
from one body to another. There are three modes of charge transfer:
(a) By friction (b) By conduction (c) By induction

Quantisation of Charge
Electric charge is quantised. The minimum unit of charge, which may reside
PREP
independently is the electronic charge e having a value of 1.6 × 10 –19 C, i.e. Q = ± ne, MIRROR
Your Personal Preparation Indicator
where, n is any integer.
Important properties of charges are listed below u No. of Questions in Exercises (x)—
l
Like charges repel while opposite charges attract each other. u No. of Questions Attempted (y)—
u No. of Correct Questions (z)—
l
Charge is invariant i.e. charge does not change with change in velocity. (Without referring Explanations)
l
According to theory of relativity, the mass, time and length change with a change in
velocity but charge does not change. u Accuracy Level (z / y × 100)—
l
A charged body attracts a lighter neutral body.
u Prep Level (z / x × 100)—

l
Electronic charge is additive, i.e. the total charge on a body is the algebraic sum of all In order to expect good rank in NEET, your
the charges present in different parts of the body. For example, if a body has different Accuracy Level should be above 85 & Prep
charges as + 2q, + 4q, − 3 q, − q, then the total charge on the body is + 2 q. Level should be above 75.
194 40 DAYS ~ NEET PHYSICS DAY NINETEEN

Coulomb’s Law of Forces between Properties of electric field lines are given below
Electric field lines come out of a positive charge and go into
Two Point Charges l

the negative charge.


l
If q 1 and q2 be two stationary point charges in free space l
No two electric field lines intersect each other.
separated by a distance r, then the force of attraction /
repulsion between them is
l
Electric field lines are continuous but they never form a
closed loop.
K |q 1||q2| 1 |q 1||q2|  1 
F = = ⋅ K = 4πε  l
Electric field lines cannot exist inside a conductor. Electric
r2 4πε 0 r2  0
shielding is based on this property.
9 × 10 9 × | q 1|| q2|
= [K = 9 × 10 9 N-m2 /c2 ]
r2
l
If some dielectric medium is completely filled between the
Continuous Charge Distribution
given charges, then the Coulomb’s force between them The continuous charge distribution may be one dimensional,
becomes two dimensional and three-dimensional.
1 q 1q2 1 q 1q2  ε  1. Linear charge density (λ ) If charge is distributed along a
Fm = = Q ε = ε r or k  line, i.e. straight or curve is called linear charge
4πε r 2 4πε 0ε r r 2  0  distribution. The uniform charge distribution q over a
1 q q length L of the straight rod.
= ⋅ 1 2
4πkε 0 r 2 Then, the linear charge density, λ =
q
L
Its unit is coulomb metre −1 (Cm–1).
Superposition Principle 2. Surface charge density (σ) If charge is distributed over a
It states that, the net force on any one charge is equal to the surface is called surface charge density, i.e. σ = q / A
vector sum of the forces exerted on it by all other charges. If Its unit is coulomb m–2 (Cm–2 )
there are four charges q 1, q2 , q3 and q 4, then the force on q 1 3. Volume charge density (ρ) If charge is distributed over
(say) due to q2 , q3 and q 4 is given by F1 = F12 + F13 + F14,
the volume of an object, is called volume charge density,
where F12 is the force on q 1 due to q2 , F13 that due to q3 and q
F14 that due to q 4. i. e., p = . Its unit is coulomb metre −3 (Cm–3 ).
V

1. Electric Field due to a Point Charge


Electric Field l
Electric field at a distance r from a point charge q is
The space surrounding an electric charge q in which another 1 q
charge q 0 experiences a force of attraction or repulsion, is E = ⋅ 2
4πε 0 r
called the electric field of charge q. The charge q is called the
source charge and the charge q 0 is called the test charge. The l
If q 1 and q2 are two like point charges, separated by a distance
test charge must be negligibly small so that it does not modify r, a neutral point between them is obtained at a point distant
the electric field of the source charge. r
r1 from q 1, such that r1 =
 q2 
Intensity (or Strength) of Electric Field (E) 1 + q 
 1
The intensity of electric field at a point in an electric field is l
If q 1 and q2 are two charges of opposite nature separated by
the ratio of the forces acting on the test charge placed at that a distance r, a neutral point is obtained in the extended line
point to the magnitude of the test charge. joining them, at a distance r1 from q 1, such that,
F r
E= , where F is the force acting on q 0. r1 =
q0  q2 
Electric field intensity (E) is a vector quantity.  q − 1
 1 
The direction of electric field is same as that of force acting on
the positive test charge. Unit of E is NC−1 or Vm −1. 2. Electric Field due to Infinitely Long Uniformly
Charged Straight Wire
Electric Field Lines Electric field at a point situated at a normal distance r,
An electric field line in an electric field is a smooth curve, from an infinitely long uniformly charged straight wire
tangent to which, at any point, gives the direction of the having a linear charge density λ, is
electric field at that point. λ
E =
2 πε 0 r
DAY NINETEEN ELECTROSTATICS 195

3. Electric Field due to a Charged Cylinder Fx = 0


l
For a conducting charged cylinder of linear charge density Y E
λ and radius R, the electric field is given by
λ P (x, y)
E = , for r > R,
2 πε 0 r
λ O
u
X
E = , for r = R
2 πε 0 R
∴Acceleration of the particle along Y-axis is given by
and E = 0, for r < R
Fy qE
l
For a non-conducting charged cylinder, for r ≤ R, ay = =
m m
λr
E = The initial velocity is zero along Y-axis (u y = 0).
2 πε 0R2
∴The deflection of charged particle along Y-axis after time t
λ 1
and E = , for r > R is given by y = u y t + a y t 2
2 πε 0 r 2
qE 2
E Emax E Emax = t
2m
r>R Along X -axis there is no acceleration, so the distance
R

covered by particle in time t along X -axis is given by x = ut


r<

r>R
r

E ∝ 12

r Eliminating t , we have
E

O  qE 
O r=R r r=R r y= x2 
 2 mu2 
(a) Variation of electric field (b) Variation of electric field
with distance for conducting with distance for y ∝ x2
cylinder non-conducting cylinder
This shows that the path of charged particle in
perpendicular field is a parabola.
4. Electric Field due to a Uniformly Charged
Infinite Plane Sheet
Electric field near a uniformly charged infinite plane sheet
Electric Dipole
having surface charge density σ is given by An electric dipole consists of two equal and opposite charges
σ separated by a small distance.
E=
2 ε0 A p B
–q +q
5. Electric Field due to a Uniformly Charged 2a
Electric Dipole
Thin Spherical Shell
For a charged conducting sphere/ E The dipole moment of a dipole is defined as the product
r<R of the magnitude of either charges and the distance
shell of radius R and total charge
Q, the electric field is given by between them. Therefore, dipole moment
Case I E = 0, for r < R p = q(2a)
Q O
Case II E = , for r = R R Electric Field due to a Dipole
4πε 0R 2 Variation of electric field
Q with distance for uniformly l
At a point distant r from the centre of a dipole, along its
Case III E = , for r > R charge spherical shell 1 2 pr
4πε 0r 2 axial line E = ⋅ 2
4πε 0 (r − a2 )2
[direction of E is the same as that of p]
Motion of a Charged Particle in For a short dipole, E =
1 2p
⋅ [r > > a]
4πε 0 r 3
an Electric Field l
At a point distant r from the centre of a dipole, along its
Let a charged particle of mass m and charge q, enters the
equatorial line
electric field along X -axis with speed u. The electric field E
1 p
is along Y-axis is given by E=− ⋅
F y = qE 4πε 0 (r 2 + a2 )3 / 2
and force along X -axis remains zero, i.e. [direction of E is opposite to that of p]
196 40 DAYS ~ NEET PHYSICS DAY NINETEEN

1 p
For a short dipole E = − ⋅
4πε 0 r 3
[r > > a] Electric Flux (φE )
It is a measure of the flow of electric field through a surface. It
l
At a point distant r from the centre of a short dipole, along
can be defined as the total number of lines of electric field
a line inclined at an angle θ with the dipole axis
1 p passes through a surface placed perpendicular to direction of
E = ⋅ 3 cos2 θ + 1 field.
4πε 0 r 3
i.e. φ E = ∫ EdS cos θ = ∫ E ⋅ dS = ∫ E ⋅ n$ dS
E Er
β E

EQ P (r, θ) ∆s n

z
A surface placed perpendicular
θ to electric field
–q O +q
2d
Electric Field at (r, θ) position Gauss Law
ds
l
E subtends an angle β from r such that The total electric flux linked with a E
1 1
tan β = tan θ closed surface is equal to times,
2 ε0
θ
the net charge enclosed by that
Torque on a Dipole in a Uniform surface. Thus,
1
Electric Field φ E = ∫ E ⋅ dS = [Qenclosed ]
S ε0 Illustration on Gauss Law
When a dipole is placed in an external electric field, making i= n
an angle θ with the direction of the uniform electric field E, it where, Q enclosed Σ qi is the algebraic sum of all the charges
i= 1
experiences a torque given by
inside the closed surface.
τ = qE × AC
τ=p×E
τ = pE sin θ
Electric Potential
The amount of work done in bringing a unit positive charge,
or qE × 2 d sin θ = (q × 2 d) E sin θ
without any acceleration, from infinity to that point, along
A any arbitrary path.
qE
+q W
E V =
2d θ q0
Electric potential is a state function and does not depend on
the path followed.
qE – C
q B
Rotation of Electric Dipole 1. Electric Potential Due to a Point Charge
Potential due to a point charge Q, at a distance r is given
1 Q
Work Done in Rotating a Dipole by V = ⋅
4πε 0 r
If an electric dipole initially kept in an uniform electric field E,
making an angle θ 1, is rotated so as to finally subtend an angle 2. Electric Potential Due to a System of Charges
θ2 , then the work done for rotating the dipole is, If a number of charges q 1, q2 , q3 ,… are present in space,
W = pE (cos θ 1 − cos θ2 ) then the electric potential at any point will be
V = V1 + V2 + V3 +…
Potential Energy of a Dipole 1 q 1 q2 q3 
= + + +…
4πε 0  r1
It is the amount of work done in rotating an electric dipole
from a direction perpendicular to electric field to a particular
r2 r3 
n
direction. 1  qi 
Hence, U = − pE cos θ or U = − p ⋅ E
=
4πε 0
∑  
 ri 
i= 1
DAY NINETEEN ELECTROSTATICS 197

3. Electric Potential Due to an Electric Dipole In general, for a system of n charges, the electric potential
energy is given by
1 p cos θ
At any general point, V = 1 q iq j
4πε 0 r2 U= Σ ,i ≠ j
1 p 2 4πε 0rij
On the dipole axis, θ = 0 ° and V = ⋅
4πε 0 r 2 1 
is used as each term in summation will appear twice 
On the equitorial axis θ = 90 ° and V = 0 2 

4. Electric Potential due to Some Common Relation between E and V


Charge Distributions Because E is force per unit charge and V is work per unit
Potential at a point distant r charge. E and V are related in the same way as work and force.
from an infinitely long wire V r < R Work done against the field to take a unit positive charge from
having linear charge density λ, P
infinity (reference point) to the given point VP = − ∫ E ⋅ dr volt
is ∞
r>R
λ where, the negative sign indicates that the work is done
V = ⋅ ln r
2 πε 0 against the field.
For a charged conducting O r=R r
sphere/shell having total charge
Q and radius R, the potential at
Equipotential Surface
a point distant r from the centre Equipotential surface is an imaginary
surface joining the points of same
of the sphere/shell is
potential in an electric field. So, we
Q 1 Q
(i) V = , for r > R (ii) V = , for r = R can say that the potential difference +q
4πε 0r 4πε 0 r between any two points on an
Q equipotential surface is zero.
(iii) V = , for r ≤ R
4πε 0R The electric lines of force at each
For a charged non-conducting (dielectric) sphere of radius R, point of an equipotential surface are
the charge Q is uniformly distributed over the entire volume. normal to the surface. Figure shows An Equipotential Surface
Q Q the electric lines of force due to point
Hence, (i) V = , for r > R (ii) V = , for r = R charge +q. The spherical surface will be the equipotential
4πε 0r 4πε 0R
surface and the electrical lines of force emanating from the
Q 3 R2 − r 2  point charge will be radial and normal to the spherical
and (iii) V = , for r < R
4πε 0  2 R3  surface.
At the centre of the sphere (r = 0) Regarding equipotential surface, following points are worth
noting
3Q 3  Q 
V = = Vs Vs = 4πε R  (i) Equipotential surface may be planar, solid etc. But
8 πε 0 R 2  0 
equipotential surface can never be point size.
V r< (ii) Equipotential surface is single valued. So, equipotential
R surfaces never cross each other.
(iii) Electric field is always perpendicular to equipotential
r>R surface.
(iv) Work done to move a point charge q between two points
O on equipotential surface is zero.
r=R r
(v) The surface of a conductor in equilibrium is an
equipotential surface.
Electric Potential Energy
The electric energy of a system of charges is the work that has
been done in bringing those charges from infinity to near each
Conductors and Insulators
other to form the system. For two point charges q 1 and q2 Conductors are those materials through which electricity
separated by distance r12 , the potential energy is given by can pass through easily. e.g. metals like copper, silver, iron
1 q 1q2 etc. Insulators are those materials through which electricity
U= . cannot pass through, e.g. rubber, ebonite, mica etc.
4πε 0 r12
198 40 DAYS ~ NEET PHYSICS DAY NINETEEN

The equivalent capacitance Cs is given by


Dielectrics and Polarisation i= n
1 1 1 1 1
Dielectrics are insulating materials which transmit electric = + + +… = Σ
Cs C1 C2 C3 i = 1 Ci
effect without actually conducting electricity.
e.g. mica, glass, water etc.
C1 C2 C3
When a dielectric is placed in an external electric field, so +Q + – –Q +Q + – –Q +Q + – –Q
+ – + – + –
the molecules of dielectric gain a permanent electric dipole + – + – + –
moment. This process is called polarisation. + – + – + –
V1 V2 V3

Electrical Capacitance + –

Capacitance of a conductor is the amount of charge needed in V


order to raise the potential of the conductor by unity. Three Capacitor in Series
Q
Mathematically, Capacitance C = 2. Parallel Grouping
V
In a parallel arrangement, Q = Q1 + Q2 + Q3 +…
Sharing of Charges and Q1 : Q2 : Q3… = C1 : C2 : C3…
l
Let two charged conductors having charges Q1 and Q2 (or The equivalent capacitance is given by
potentials V1, V2 and capacitances C1, C2 respectively). If i= n
these are joined together. In such case C p = C1 + C2 + C3 + K = Σ Ci
i= 1
Q + Q2 C1V1 + C2V2
Common potential, V = 1 = C1
C1 + C2 C1 + C2 +Q1 + – –Q1
+ –
l
During sharing of charges, there is some loss of + –
+ –
electrostatic energy, which in turn reappears as heat or
C2
light. The loss of electrostatic energy +Q2 + – –Q2
+ –
C1C2
∆U = Ui − U f = (V1 − V2 )2 + –
2(C1 + C2 ) + –
+Q3 C3 –Q3
l
When charges are shared between any two bodies, their + –
+ –
potential become equal. The charges acquired are in the + –
+ –
ratio of their capacities.
+ –

Capacitor V
Three Capacitor in Parallel
A capacitor is a device which stores electrostatic energy. It
consists of conductors of any shape and size carrying charges
of equal magnitudes and opposite signs and separated by an
insulating medium.
Capacitance of a Parallel Plate
A B
Capacitor
+Q –Q
1. Capacitor without Dielectric Medium
between the Plates
If the magnitude of charge on each plate of a parallel plate
capacitor be Q and the overlapping area of plates be A,
then
d
σ Q
A Capacitor l
Electric field between the plates, E = =
ε0 ε0 A
There are two types of combination of capacitors: l
Potential difference between the plates
σd Qd
1. Series Grouping V = E ⋅d = = , where d = separation between the two
ε0 ε0 A
In a series arrangement, V = V1 + V2 + V3 +…
plates.
1 1 1
and V1 : V2 : V3… = : : :… Q ε A
C1 C2 C3 l
Capacitance, C = = 0
V d
DAY NINETEEN ELECTROSTATICS 199

2. Capacitor with Dielectric Medium between σ2 A Q2 CV 2


F = = =
the Plates 2ε 0 2 Aε 0 2d
l
If a dielectric medium of dielectric constant K is completely l
The energy density between the plates of a capacitor
filled between the plates of a capacitor, then its capacitance U 1
u= = ε0 E2
becomes, Volume 2
Kε 0 A ′  ε A′ 
C′ = = KC 0 where, C 0 = 0 
d  d  Energy Stored in a Capacitor
l
If a dielectric slab/sheet of thickness t (where, t < d) is If a capacitor of capacity C is charged to a potential V , the
introduced between the plates of the capacitor, then electrostatic energy stored in it is,
ε0 A 1 1 1 Q2
C′ = U= CV 2 = QV =
d − t + t  2 2 2 C
 
 K
l
If a metallic slab/plate of thickness t (where, t < d), is Energy Loss During Parallel
inserted between the plates of capacitor, then Combination
Kε 0 A ′
C′ = = KC 0 When capacitor of capacitance C1 charge to potential V1,
d whereas another of C2 charge to potential of V2 , then after
l
Magnitude of the attractive force between the plates of a parallel combination.
parallel plate capacitor is given by 1 C1C2
Loss in energy = (V1 − V2 )2
2 C1 + C2

DAY PRACTICE SESSION 1

FOUNDATION QUESTIONS EXERCISE


1 When a glass rod rubbed with silk is brought near the 5 Two identical coins having similar charges are placed
gold leaf electroscope, the leaves diverge. What is the 4.5 m apart on a table. Force of repulsion between them
charge on the leaves? 40
is N. The value of charge on each coin is
(a) Negative (b) Zero 9
(c) Positive (d) Either positive or negative (a) 100 µC (b) 200 µC (c) 300 µC (d) 400 µC

2 Identify the wrong statement in the following Coulomb’s 6 A small uncharged metallic sphere is positioned exactly
law correctly described the electric force that at a point midway between two equal and opposite point
(a) binds the electrons of an atom to its nucleus charges. If the sphere is slightly displaced towards the
(b) binds the protons and neutrons in the nucleus of positive charge and released, then
an atom (a) it will oscillate about its original position
(c) binds atoms together to form molecules (b) it will move further towards the positive charge
(d) binds atoms and molecules to form solids (c) its electric potential energy will decrease and kinetic
energy will increase
3 The excess (equal in number) of electrons that must be (d) its total energy remains constant but it non-zero
placed on each of two small spheres spaced 3 cm
apart, with force of repulsion between the spheres to be 7 A charge Q is divided into two parts of q and − q. If the
10−19N, is coulomb repulsion between them when they are
Q
(a) 25 (b) 225 (c) 625 (d) 1250 separated is to be maximum, the ratio of should be
q
4 When air is replaced by a dielectric medium of constant 1 1
(a) 2 (b) (c) 4 (d)
K. The maximum force of attraction between two charges 2 4
separated by a distance
8 Two positive ions, each carrying a charge q, are
(a)increases K −1 times (b) increases K times separated by a distance d. If F is the force of repulsion
(c)decreases K times (d) remains constant between the ions, the number of electrons missing from
200 40 DAYS ~ NEET PHYSICS DAY NINETEEN

each ion will be (e being the charge on an electron) (c) The dipole will experience a force towards left
j CBSE AIPMT 2010 (d) The dipole will experience a force upwards
4 πε0Fd 2 4 πε0Fe 2 4 πε0Fd 2 4 πε0Fd 2 15 An electric dipole has the magnitude of its charge as q
(a) 2
(b) 2
(c) 2
(d)
e d e q2 and its dipole moment is p. It is placed in a uniform
electric field E. If its dipole moment is along the direction
9 The electric field due to a charge at a distance of 3 m
the field, the force on it and its potential energy are
from it is 500 NC −1. The magnitude of the charge is
respectively
[1/ 4 πε0 = 9 × 109 Nm2C −2 ]
(a) 2.5 µC (b) 1.0 µC (a)2qE and minimum (b) qE and pE
(c) 2.0 µC (d) 0.5 µC (c)zero and minimum (d) qE and maximum

10 An electron is sent in electric field of intensity 16 The electric potential at a point ( x , y , z ) is given by
−1
9.1 × 10 NC . The acceleration produced is
6 V = − x 2 y − xz 3 + 4. j CBSE AIPMT 2009
(mass of electron = 9.1 × 10 −31 kg) The electric field E at that point is
(a)1.6 ms −2 (b) 1.6 × 1018 ms −2 (a) E = (2 xy + z 3 )$i + x 2 $j + 3 xz 2 k$
(c)3.2 × 1018 ms −2 (d) 0.8 × 1018 ms −2 (b) E = 2 xy $i + (x 2 + y 2 )$j + (3 xz − y 2 )k$
(c) E = z 3 $i + xyz$j + z 2 k$
11 A charged particle of mass 0.003 g is held stationary (d) E = (2 xy − z 3 )$i + xy 2 $j + 3 z 2 xk$
in space by placing it in a downward direction of
17 Which one of the following graphs represents the variation
electric field of 6 × 104 NC −1. Then, the magnitude of
of electric field strength E with distance r from the centre
the charge is
of a uniformly charged non-conducting sphere?
(a) 5 × 10−4 C (b) 5 × 10−10 C (c) 18 × 10−6 C (d) −5 × 10−9 C
12 A point charge +q, is placed at a distance d from an
isolated conducting plane. The field at a point P on the (a) E (b) E
other side of the plane is
(a) directed perpendicular to the plane and away from the
plane R r R r
(b) directed perpendicular to the plane but towards the
plane
(c) directed radially away from the point charge (c) E (d) E
(d) directed radially towards the point charge
13 A , B and C are three points in a uniform electric field. The
R r R r
electric potential is j
NEET 2013
18 An electric dipole of moment p is placed in an electric field
A
of intensity E . The dipole acquires a position such that the
B
E axis of the dipole makes an angle θ with the direction of
C the field. Assuming that the potential energy of the dipole
to be zero when θ = 90° , the torque and the potential
(a) maximum at A (b) maximum at B
energy of the dipole will respectively be j CBSE AIPMT 2012
(c) maximum at C
(d) same at all the three points A, B and C (a) pE sinθ, − pE cosθ (b) pE sinθ, − 2 pE cosθ
(c) pE sinθ, 2 pE cosθ (d) pE cosθ, − pE sinθ
14 Figure shows electric field lines in which an electric
19 The electric flux for Gaussian surface A that enclose the
dipole p is placed as shown in figure. Which of the
charged particles in free space is
following statements is correct? (Given q1 = − 14 nC, q 2 = 78.85 nC, q 3 = − 56 nC)

Gaussian surface A
q1

–q P q3
+q q2 Gaussian surface B

(a)103 Nm 2 C −1 (b) 103 CN −1m −2


(a) The dipole will not experience any force (c)6.32 × 103 Nm 2 C −1 (d) zero
(b) The dipole will experience a force towards right
DAY NINETEEN ELECTROSTATICS 201

20 Consider the charge configuration and a


q2
29 Four point charges − Q , − q , 2q and 2Q are placed, one
spherical Gaussian surface as shown in +q1
at each corner of the square. The relation between Q and
figure. When calculating the flux of the q for which the potential at the centre of the square is
electric field over the spherical surface, –q1 zero, is j CBSE AIPMT 2012

the electric field will be due to 1 1


(a) Q = − q (b) Q = − (c) Q = q (d) Q =
(a) q2 (b) only the positive charges q q
(c) all the charges (d) + q1 and − q1
30 A hollow metallic sphere of radius R is given a charge Q.
21 If the electric flux entering and leaving an enclosed Then, the potential at the centre is AFMC 2010
j

surface respectively is φ1 and φ 2 , the electric charge 1 Q 1 2Q 1 Q


(a) zero (b) (c) (d)
inside the surface will be 4 πε0 R 4 πε0 R 4 πε0 2R
(a) (φ1 + φ 2 ) ε0 (b) (φ 2 − φ1) ε 0
(c) zero (d) φ 2 / ε 0 31 Four electric charges + q , + q , − q and − q are placed at
the corners of a square of side 2L as see figure. The
22 What is the total flux from the surface of cylinder of radius
electric potential at point A , midway between the two
R and length L which is placed in a uniform electric field
charges + q and + q , is j CBSE AIPMT 2011
E parallel to cylinder axis?
+q –q
(a) 2 πR 2E (b) πR 2 E
(c) (πR + πR 2 ) /E (d) zero
L
23 A hemisphere surface of radius R is placed in uniform A
electric field of intensity E parallel to the axis of its
circular plane. What will be the electric flux φ through the L
hemisphere surface?
+q 2L –q
(a) 2 πR E (b) 2 πR 2E
(c) πR 2E (d) (4 /3) πR 3E
1 2q 1 + 1  1 2q 1 − 1 
(a)   (b)  
4 πε0 L  5 4 πε0 L  5
24 What is the total electric flux leaving a spherical surface 1 2q
(c) zero (d) (1 + 5)
of radius 1 cm and surrounding at electric dipole? 4 πε0 L
(a) q/ε0 (b) zero (c) 2 q/ε0 (d) 8 πr q/ε0
2
32 The electric charge on drop A
25 A charge Q is enclosed by a gaussian spherical surface B is 1.6 × 10 −19 C and its
10 mm
of radius R. If the radius is doubled, then the outward mass is 1.6 × 10 −14 kg. If the
electric flux will j
CBSE AIPMT 2011 B
drop is in equilibrium, then
(a)be reduced to half (b) remain the same the potential difference between the plates will be
(c)be doubled (d) increase four times
(where, density d = 10 × 10−3 kgm −3 )
26 A square surface of side L metre (a) 5 × 105 V (b) 104 V (c) 2 × 104 V (d) 105 V
in the plane of the paper is placed 33 A conducting sphere of radius R is given a charge Q.
in a uniform electric field E (V/m) E
The electric potential and the electric field at the centre of
acting along the same place at an θ
the sphere respectively are j
CBSE AIPMT 2014
angle θ with the horizontal side of Q Q
(a) zero and (b) and zero
the square as shown in figure. The 4 πε0R 2 4 πε0R
electric flux linked to the surface in Q Q
(c) and (d) Both are zero
unit of V-m, is j
CBSE AIPMT 2010 4 πε0R 4 πε0R 2
2 2 2
(a) EL (b) EL cosθ (c) EL sinθ (d) 0
34 Which of the following is not the property of equipotential
27 What is the flux through a cube of side a if a point charge surfaces?
of q is at one of its corner? j
CBSE AIPMT 2012
(a) They do not cross each other
2q q
(a) (b) (b) They are concentric spheres for uniform electric field
ε0 8 ε0
(c) Rate of change of potential with distance on them is
q q
(c) (d) 6a 2 zero
ε0 2 ε0
(d) They can be imaginary spheres
28 There exists an electric field of 1 N/C along y-direction. 35 The potential of a spherical conductor of radius 3 m
The flux passing through the square of 1m placed in xy is 6 V. The potential at its centre is
plane inside the electric field is (a) zero (b) 2V (c) 6V (d) 18V
(a) 1 Nm 2 /C (b) 10 Nm 2 /C (c) 2 Nm 2 /C (d) zero
202 40 DAYS ~ NEET PHYSICS DAY NINETEEN

36 A parallel plate capacitor has an electric field of 10 5 Vm −1 43 A parallel plate capacitor is located horizontally such that
between the plates. If the charge on the capacitor plates one of the plates is submerged in a liquid while the other
is 1 µC, the force on each capacitor plate is is above the liquid surface. When plates are charged, the
(a)0.5 N (b) 0.05 N level of liquid
(c)0.005 N (d) None of these
37 If a battery is disconnected from the capacitor and then a
dielectric substance between two plates of condenser is
introduced the capacity, potential and potential energy Charged Liquid
respectively
(a) increases, decreases, decreases
(a) rises
(b) decreases, increases, increases
(b) falls
(c) increases, increases, increases
(d) decreases, decreases, decreases (c) remains unchanged
(d) may rise or fall depending on the amount of charge
38 Two identical metal plates are given positive charge Q1
and Q 2 such that Q 2 < Q1. If they are now brought close 44 A parallel plate condenser has a uniform electric field
V 
to each other to form a parallel plate capacitor of E   in the space between the plates. If the distance
 m
capacitance C, the potential difference between them is
A B between the plates is d(m) and area of each plate is
σ1+ +σ2 A (m 2 ). The energy (joule) stored in the condenser is
+ + j CBSE AIPMT 2011
EA P + Q R EA 1
+ (a) ε0E 2 (b) ε0EAd
EB EB EA EB 2
+ + 1 Ad
(c) ε0E 2 Ad (d) E 2
+ + 2 ε0
Q1 + Q2 Q1 + Q2
(a) (b) 45 Two capacitors of capacitances 3 µF and 6 µF are
2C C
Q − Q2 Q − Q2 charged to a potential of 12 V each. They are now
(c) 1 (d) 1
C 2C connected to each other, with the positive plate of each
joined to the negative plate of the other. The potential
39 A 0.2 µF capacitor is charged to 600 V. After removing it difference across each will be
from battery it is connected to another capacitor 1.0 µF in (a) 4 V (b) 6 V
parallel. The voltage on the capacitor will become (c) zero (d) 3 V
(a) 300 V (b) 600 V
46 Three capacitors each of capacitance C and of
(c) 100 V (d) 120 V
breakdown voltage V are joined in series. The
40 A thin aluminium sheet is placed between the plates of a capacitance and breakdown voltage of the combination
parallel plate capacitor. Its capacitance will will be j
CBSE AIPMT 2009
(a)increase (b) decrease C V V
(a) , (b) 3 C,
(c)remain same (d) become infinite 3 3 3
C
(c) , 3V (d) 3C, 3V
41 Work done in placing a charge of 8 × 10−18 C on a 3
condenser of capacity 100 µF is
47 Two thin dielectric slabs of dielectric constants K1 and K 2
(a) 16 × 10−32 J (b) 31 × 10−26 J
(K1 < K 2 ) are inserted between plates of a parallel plate
(c) 4 × 10−10 J (d) 32 × 10−32 J
capacitor, as shown in the figure.
42 A number of condensers, each of the capacitance 1µF P + – Q
and each one of which gets punctured, if a potential
+ –
difference just exceeding 500 V is applied, are
+ –
provided. An arrangement suitable for giving
+ –
capacitance of 2 µF across which 3000 V may be
+ –
applied requires at least.
+ –
(a) 6 component capacitors –
+
(b) 12 component capacitors –
+
(c) 72 component capacitors K1 K2
(d) 2 component capacitors
DAY NINETEEN ELECTROSTATICS 203

The variation of electric field E between the plates with 51 What will be the equivalent capacitance of the system as
distance d as measured from plate P is correctly shown shown in the figure, where two spherical conductors A
by j CBSE AIPMT 2014 and B of radii a and b (b > a ) are placed concentrically in
air with a charge + Q on A and B being earthed?
E E B
A
(a) (b) + + +
+ +
0 0 + a +
+ +
d d + +
+ +
b + + +
Q
E E
 ab 
(c) (d) (a)4 πε0   (b) 4 πε0 (a + b)
b − a
0 0  b2 
d d (c)4 πε0 b (d) 4 πε0  
b − a
48 A capacitor having capacity of 2.0 µF is charged to 200 V
and then the plates of the capacitor are connected to a 52 The following figure shows seven capacitors, a switch S
resistance wire. The heat produced in joule will be and a source of emf connected together. Initially S is
(a) 2 × 10−2 (b) 4 × 10−2 open and all capacitors are uncharged.
(c) 4 × 104 (d) 4 × 1010 59 V S
+ –
49 The equivalent capacitance of the following combination
is 21µF 3 µF 4 µF

7µF
10 mF A
5 mF
10 mF 28 µF 2 µF 1 µF

What will be the potential difference in volts across the


(a) 10 µF (b) 4 µF
(c) 25 µF (d) 15 µF plates of the capacitor A, if S is closed afterwards and a
steady state is attained?
50 Two condensers, one of capacity C and the other of (a) 12 (b) 15
C
capacity , are connected to aV volt battery, as shown (c) 17 (d) 19
2
in figure. 53 A capacitor of capacitance C has charge Q and stored
energy is W . If the charge is increased to 2 Q, the stored
C energy will be
V C W W
2 (a) (b)
4 2
(c) 2W (d) 4W
The work done in charging fully both the condensers is 54 Two capacitors of10 µF and 20 µF are connected in
2 1 series with a 30 V battery. The charge on the capacitors
(a) 2 CV (b) CV 2
4 will be respectively
3 1
(c) CV 2 (d) CV 2 (a)100 µC, 100 µC (b) 200 µC, 100 µC
4 2
(c)200 µC, 200 µC (d) 100 µC, 200 µC
204 40 DAYS ~ NEET PHYSICS DAY NINETEEN

DAY PRACTICE SESSION 2

PROGRESSIVE QUESTIONS EXERCISE


1 In figure, two positive charges q 2 and q 3 fixed along the
y-axis, exert a net electric force in the +x direction on a
charge q1 fixed along the x-axis. If a positive charge Q is +q
added at ( x , 0), the force on q1
y y (ii)
q2 q2

Q
x x
q1 q1 (x, 0) +q

q3 q3
(iii)
(a) (b)

(a) shall increase along the positive x-axis


(b) shall decrease along the positive x-axis +q
(c) shall point along the negative x-axis
(d)shall increase but the direction changes, because of the
(iv)
intersection of Q with q2 and q3
(a) Fig. (i) (b) Fig. (ii)
2. Three charges are placed at the vertices of an equilateral (c) Fig. (iii) (d) Fig. (iv)
triangle of side a as shown in the figure. The force
experienced by the charge placed at the vertex A in a 5 A hollow cylinder has a charge q coulomb within it. If φ is
direction normal to BC is the electric flux in unit of V-m associated with the curved
surface B, the flux linked with the plane surface A in unit of
A
+Q V-m will be
B

C A

–Q +Q
B C
1 q  q
(a) Q 2 /(4 πε0a 2 ) (b) −Q 2 /(4 πε0a 2 ) (a)  − φ (b)
2  ε0  2 ε0
(c) zero (d) Q 2 /(2 πε0a 2 )
φ q
(c) (d) −φ
3 The ratio of electrostatic and gravitational forces acting 3 ε0
between electron and proton separated by a distance
5 × 10 −11m, will be (Charge of electron = 1.6 × 10 −19 C, 6 Two infinitely long parallel conducting plates having
mass of electron = 9.1 × 10 −31kg, mass of proton surface charge densities + σ and − σ respectively, are
= 1.6 × 10 −27 kg, G = 6.7 × 10 −11N-m 2 kg −2 ) separated by a small distance. The medium between the
(a) 2.36 × 1039 (b) 2.36 × 1040 plates is vacuum. If ε 0 is the dielectric permittivity of
(c) 2.34 × 104 (d) 2.34 × 1042 vacuum, then the electric field in the region between the
4 A point positive charge is brought near an isolated plates is
conducting sphere, in figure shown. The electric field (a) zero (b) σ/2 ε0 Vm−1
is best given by (c) σ/ε0 Vm−1 (d) 2 σ/ε0 Vm−1
7 When n identical mercury droplets charged to the same
potential V coalescence to form a single bigger drop. The
+q potential of the new drop will be
V
(a) (b) nV (c) nV 2 (d) n 2 / 3V
(i) n
DAY NINETEEN ELECTROSTATICS 205

8 Charges + q and − q are placed at points A and B between the plates d and area A of the plates
respectively which are a distance 2L apart, C is the respectively are
mid-point between A and B. The work done in moving a (a)6.6 × 10−6 m ; 103 m2 (b) 6.6 × 10−5 m ; 104 m2
charge + Q along the semi-circle CRD is (c)6.6 × 10−4 m ; 105 m2 (d) 6.6 × 10−6 m ; 102 m2
qQ qQ 14 A parallel plate air capacitor of capacitance C is
(a) (b)
4 πε0L 2 πε0L connected to a cell of emfV and then disconnected from
qQ qQ it. A dielectric slab of dielectric constant K, which can
(c) (d) −
6 πε0L 6 πε0L just fill the air gap of the capacitor, is now inserted in it.
Which of the following statements is incorrect?
9 Two charges q1 and q 2 are placed 30 cm apart, as j CBSE AIPMT 2015

shown in the figure. A third charge q 3 is moved along the


(a)The energy stored in the capacitor decreases K times.
arc of a circle of radius 40 cm from C to D. The change in
(b)The charge in energy stored is CV 2  – 1
1 1
q K 
the potential energy of the system is 3 k , where k is 2
4πε 0 (c)The charge on the capacitor is not conserved
C q3 (d)The potential difference between the plates decreases K
times.
15 A capacitor having capacitance 1 µF with air, is filled with
40 cm two dielectrics as shown in figure. How many times
capacitance will increase?
q1 q2

A 30 cm B D

(a) 8 q2 (b) 8 q1 K1 = 8 K2 = 4
(c) 6q2 (d) 6q1
10 The electrostatic potential inside a charged spherical ball
is given by V = ar 2 + b, where r is the distance from the (a) 12 (b) 6
centre and a, b are constants. Then, the charge density (c) 8/3 (d) 3
inside the ball is 16 A series combination of n1 capacitors, each of value
(a) − 3 a ε0r (b) − 6a ε0 C1, is charged by a source of potential difference 4V .
(c) + 3 a ε0 r (d) zero
When another parallel combination of n 2 capacitors, each
11 Three point charges + q , − 2 q and + q are placed at of value C2 , is charged by a source of potential difference
points ( x = 0 , y = a, z = 0), ( x = 0 , y = 0 , z = 0) and V , it has the same (total) energy stored in it, as the first
( x = a, y = 0, z = 0), respectively. The magnitude and combination has. The value of C2 , in terms of C1, is then
direction of the electric dipole moment vector of this j
CBSE AIPMT 2010
charge assembly are 2C1 n n 16C1
(a) (b) 16 2 C1 (c) 2 2 C1 (d)
(a) 2 aq along + y-direction n1n2 n1 n1 n1n2
(b) 2 aq along the line joining points (x = 0, y = 0, z = 0)
and (x = a, y = a, z = 0) 17 The equivalent capacitance between A and B for the
(c)qa along the line joining points (x = 0, y = 0, z = 0) combination of capacitors shown in figure, where all
and (x = a, y = a, z = 0) capacitances are in microfarad is
(d) 2 aq along + x-direction 1 mF
12 27 small drops each having charge q and radius r 4 µF
coalesce to form big drop. How many times charge and
capacitance will become?
(a) 3, 27 (b) 27, 3 3 µF
A 6 mF
(c) 27, 27 (d) 3, 3 B
13 A parallel plate capacitor of value 1.77 µF is to be
designed using a dielectric material (dielectric constant 8 µF
= 200), breakdown strength of 3 × 10 6 Vm −1 . In order to 2 µF
make such a capacitor which can withstand a potential
(a) 6.0 µF (b) 4.0 µF (c) 2.0 µF (d) 3.0 µF
difference of 20 V across the plates, the separation
206 40 DAYS ~ NEET PHYSICS DAY NINETEEN

2
18 Charge Q is divided into two parts which are then kept (a) k = k1 + k2 + k3 + 3k4 (b) k = (k1 + k2 + k3 ) + 2k4
some distance apart. The force between them will be 3
2 3 1
maximum, if the two parts are having the charge (where (c) = + (d) None of these
k k1 + k2 + k3 k4
e = electronic charge )
Q Q 3Q 24 The electric field in a certain region is acting radially
(a) each (b) and
2 4 4 outward and is given by E = Ar . A charge contained in a
Q 2Q
(c) and (d) e and (Q − e) sphere of radius ‘a’ centred at the origin of the field' will
3 3
be given by j CBSE AIPMT 2015

19 A capacitor is charged by a battery. The battery is (a) 4 πε0 Aa 2 (b) Aε0a 2


removed and another identical uncharged capacitor is (c) 4 πε0 Aa 3 (d) ε0 Aa 3
connected in parallel. The total electrostatic energy of
25 If potential (in volts) in a region is expressed as
resulting system j NEET 2017

V ( x , y , z ) = 6xy − y + 2yz , the electric field (in N/C) at


(a) increases by a factor of 4
(b) decreases by a factor of 2 point (1, 1, 0) is j CBSE AIPMT 2015

(c) remains the same (a) − (3 $i + 5 $j + 3 k$ ) (b) − (6$i + 5 $j + 2 k$ )


(d) increases by a factor of 2 (c) − (2 $i + 3 $j + k$ ) (d) − (6$i + 9$j + k$ )
20 Suppose the charge of a proton and an electron differ 26 A parallel plate air capacitor has capacity C, distance of
slightly. One of them is −e and the other is (e + ∆e ). If the separation between plates is d and potential differenceV
net of electrostatic force and gravitational force between is applied between the plates. Force of attraction
two hydrogen atoms placed at a distance d (much between the plates of the parallel plate air capacitor is
greater than atomic size) apart is zero, then ∆e is of the j CBSE AIPMT 2015
order (Given mass of hydrogen, mh = 1.67 × 10−27 kg) C 2V 2 CV 2
j
NEET 2017 (a) (b)
2d 2d
(a) 10−20 C (b) 10−23 C (c) 10−37 C (d) 10−47 C CV 2 C 2V 2
(c) (d)
21 A capacitor of 2 µF is charged as shown in the figure. d 2d 2
When the switch S is turned to position 2, the percentage
27 In a region, the potential is represented by V ( x , y , z )
of its stored energy dissipated is j
NEET 2016
= 6x − 8xy − 8y + 6yz, where V is in volts and x, y, z are
1 2 in metres. The electric force experienced by a charge of
S 2 C situated at point (1, 1, 1) is j
CBSE AIPMT 2014
(a) 6 5 N (b) 30 N
V 2 µF (c) 24 N (d) 4 35 N
8 µF
28 Two pith balls carrying equal charges are suspended from
a common point by strings of equal length, the equilibrium
(a) 20% (b) 75% (c) 80% (d) 0% separation between them is r. Now, the strings are rigidly
22 An electric dipole is placed at an angle of 30° with an clamped at half the height. The equilibrium separation
electric field intensity 2 × 105 N/C. It experiences a torque between the balls now becomes j
NEET 2013
equal to 4 Nm. The charge on the dipole, if the dipole
length is 2 cm, is j
NEET 2016
(a) 8 mC (b) 2 mC (c) 5 mC (d) 7 µC y

23 A parallel-plate capacitor of area A, plate separation d y/2


and capacitance C is filled with four dielectric materials r r
having dielectric constants k1, k 2 , k 3 and k 4 as shown in
2
(a) 
1 
(b) 
the figure below. If a single dielectric material is to be r 
 
used to have the same capacitance C in this capacitor,  2  2
(c) 
2r 
(d)  
then its dielectric constant k is given by j
NEET 2016 2r

 3  3
A/3 A/3 A/3
29 Two identical charged spheres suspended from a
k1 k2 k3 d/2 common point by two massless strings of lengths l, are
d initially at a distance d (d < < l) apart because of their
k4 mutual repulsion. The charges begin to leak from both
the spheres at a constant rate. As a result, the spheres
A
approach each other with a velocity v. Then, ν varies as a
DAY NINETEEN ELECTROSTATICS 207

function of the distance x between the sphere, as (b) linearly proportional to the distance between the plates
j NEET 2016 (c) independent of the distance between the plates
1 1 (d) inversely proportional to the distance between the

(a) v ∝ x (b) v ∝ x 2 (c) v ∝ x −1 (d) v ∝ x 2 plates
30 An electron falls from rest through a vertical distance h in 32 A toy car with charge q moves on a frictionless horizontal
a uniform and vertically upward directed electric field E . plane surface under the influence of a uniform electric
The direction of electric field is now reversed, keeping its field E. Due to the force q E, its velocity increases from 0
magnitude the same. A proton is allowed to fall from rest to 6 m/s in one second duration. At that instant, the
in it through the same vertical distance h. The time of fall direction of the field is reversed. The car continues to
of the electron, move for two more seconds under the influence of this
in comparison to the time of fall of the proton is field. The average velocity and the average speed of the
(a) 10 times greater (b) 5 times greater toy car between 0 to 3 seconds are respectively
(c) smaller (d) equal (a) 1 m/s, 3.5 m/s
(b) 1 m/s, 3 m/s
31 The electrostatic force between the metal plates of an
(c) 2 m/s, 4 m/s
isolated parallel plate capacitor C having a charge Q and (d) 1.5 m/s, 3 m/s
area A, is j NEET 2018

(a) proportional to the square root of the distance between


the plates

ANSWERS
SESSION 1 1 (c) 2 (b) 3 (c) 4 (c) 5 (a) 6 (b) 7 (a) 8 (c) 9 (d) 10 (b)
11 (b) 12 (a) 13 (b) 14 (c) 15 (c) 16 (a) 17 (c) 18 (a) 19 (a) 20 (c)
21 (b) 22 (d) 23 (c) 24 (b) 25 (b) 26 (c) 27 (b) 28 (d) 29 (a ) 30 (b)
31 (b) 32 (b) 33 (b) 34 (d) 35 (c) 36 (b) 37 (c) 38 (c) 39 (c) 40 (c)
41 (d) 42 (c) 43 (a) 44 (c) 45 (a) 46 (c) 47 (c) 48 (b) 49 (a) 50 (c)
51 (a) 52 (a) 53 (d) 54 (c)
SESSION 2 1 (a) 2 (c) 3 (a) 4 (a) 5 (a) 6 (c) 7 (d) 8 (d) 9 (a) 10 (b)
11 (b) 12 (b) 13 (a) 14 (c) 15 (b) 16 (d) 17 (a) 18 (a) 19 (d) 20 (c)
21 (c) 22 (b) 23 (d) 24 (c) 25 (b) 26 (b) 27 (d) 28 (b) 29 (b) 30 (c)
31 (c) 32 (b)
208 40 DAYS ~ NEET PHYSICS DAY NINETEEN

Hints and Explanations


1 The glass rod acquires positive charge 8 From Coulomb’s law, force of repulsion is 14 In figure spacing between electric lines
on rubbing. When the rod is brought qq
1 qq of force increases from left to right.
F =K ⇒F =
near the disc of the electroscope, the d 2 4 π ε0 d 2 Therefore, E on left is greater than E on
disc will acquire negative charge and right. Force on +q charge of dipole is
where, q = ne
there will be positive charge on the far smaller and to the right. Force on − q
end, i.e. on the leaves. 1 n2e 2 4 πε0 Fd 2 charge of dipole is bigger and to the left.
∴ F = ⋅ ⇒n=
4 πε 0 d 2 e2 Hence, the dipole will experience a net
2 Coulomb’s law is applicable for charge force towards the left.
particles, it is not responsible to bind 9 Electric field by the charges.
the protons and neutrons in the nucleus 1 q q 15 Force on the charge q is F2 = qE along
of an atom. E = 500 = 9 × 109 ×
4 πε0 r 2 (3) 2 the direction of E and force on charge
3 Coulomb’s law is given by −9 − q is F1 = − qE in the direction opposite
⇒ 500 × 10 =q
q1 q2 to E.
F =k or q = 0.5 × 10−6 C, q = 0.5 µC Since, forces on the dipole are equal and
r2
opposite, so net force on the electric
Substituting q1 = q2 = ne , we get 10 Electric field intensity is given by dipole is zero.
F F
(ne )(ne ) n2e 2 Fr 2 E = = [here, q = e ) Now, potential energy of the dipole.
F =k =k ⇒ n2 = q e
r 2
r 2
ke 2
U = − pE cos θ
⇒ F = eE
On putting
or ma = eE [because, F = ma] where, θ is the angle between direction
F = 10−19 N , r = 3cm = 3 × 10−2 m, eE of electric field and direction of dipole
⇒ a= moment.
k = 9 × 109 Nm2 C −2 , e = 1.6 × 10−19 C, m ∴ θ = 0°
we get n = 625 Substituting Hence, U = − pE cos 0° = − pE
e = 1.6 × 10−19 C , E = 9.1 × 106 NC −1 , [minimum]
4 Coulomb’s force between them,
1 q1 q2 q1 q2 F and m = 9.1 × 10− 31 kg, we get 16 Potential gradient relates with electric
F = . = = 0
4 πε0 r 2 4 πε0 kr 2 k a = 1.6 × 1018 ms −2 field according to the following relation,
− dV
E =
11 Weight of charged particle dr
q q
5 Given, F = 40 or 1 1 2 2 = 40 = magnitude of force of electric field
E= −
dV
9 4 πε0 r 9 i.e. mg = qE dr
9 × 109 × (q )2 40 Given, m = 3 × 10−6 kg As V = − x2 y − xz3 + 4
or =
(4.5)2 9 g = 10 ms −2 and E = 6 × 104 NC −1 So, E = −
dV $ dV $ dV $
i− j− k
[because q1 = q2 = q] dx dy dz
Substituting the values, we get
⇒ q 2 = 10−8 E = (2 xy + z3 )$i + x2 $j + 3 xz2 k$
q = 5 × 10−10 C
q = 10 −4 C = 100µC
12 When a point charge +q is placed at a 17 At r ≤ R, E ∝ r and at r > R, E ∝ 12
6 Initially, the force on the sphere is equal distance (d) from an isolated conducting r
due to both negative and positive plane, some negative charge develop on 18 Here, torque, τ = pE sin θ
charges. the surface of the plane towards the
∴ Net force = 0 charge and an equal positive charge +q
On displacing the sphere towards the developes on opposite side of the plane.
positive charge, force on sphere due to Hence, the field at a point P on the other θ→E
positive charge will be more than due to side of the plane is directed
the negative charge, because it is nearer. perpendicular to the plane and away
So, sphere will move further to the from the plane as shown in figure.
positive charge. –q
– + Potential energy of the dipole,
7 Let distance between two charges is x.
– + U = ∫ τ dθ
The force between them,
1 q (Q − q ) P 0 θ
F =
4 πε 0 x2 +q
– + = ∫π /2 pE sin θdθ = − pE [cos θ]π /2
– + = − pE cos θ
x is constant, so for maximum force q
– + 19 Electric flux (φ) =
q (Q − q ) should be maximum. ε0
d
[q (Q − q )] = 0 (− 14 + 78.85 − 56) × 109
dq 13 The electric field is maximum at B, =
8.85 × 10−12
Q because electric field is directed along
Q − 2q = 0 ⇒ =2 8.85 × 10−9
q decreasing potential. Hence, = = 103 Nm2 C −1
V B > VC > V A . 8.85 × 10−12
DAY NINETEEN ELECTROSTATICS 209

20 At any point over the spherical gaussian 29 If potential at centre is zero, then 38 Let plate A and plate B be carrying
surface, net electric field is the vector V1 + V2 + V3 + V 4 = 0 charge Q 1 and Q 2 , respectively.
sum of electric field due to + q1 , − q1 –q 2q Q1 Q2
and q2 . Hence, electric field will be due
to all the charges.
21 For a closed surface outward flux is
taken as positive and inward flux is – Q2 – Q1
O
taken as negative.
Total charge enclosed
Net flux, φ =
ε0 –Q 2Q A B
1
= ×Q
ε0 −
kQ

kq
+
k 2Q
+
k 2q
=0
When they are brought closer, they
∴ Q = ε 0 ( φ2 − φ1 ) r r r r induce equal and opposite charge on
− Q − q + 2q + 2Q = 0 each other, i.e. − Q 2 on plate A and − Q 1
22 Electric field is perpendicular to area Q = −q on plate B.
vector. Hence, the total flux, Therefore, net charge on plate
30 Potential at the centre of a hollow
φ=0 [Q φ = E ds cos θ] A = Q 1 − Q 2 and net charge on plate
metallic sphere
1 Q B = − (Q 1 −`Q 2 ). So, the charge on
23 The electric flux through any surface is V = capacitor is given as
equal to the product of electric field 4 πε 0 R
Q1 − Q2
intensity at the surface and component 1 q
31 Electric potential, V = Potential difference between the plates
of the surface perpendicular to electric Q − Q2
4 πε 0 r is V = 1
field
C
= E × π R 2 = πR 2 E Here, V = 2V+ ve + 2V – ve
1  2q 2q  39 Charge on capacitor plates is given by
24 From Gauss’ theorem, V = − q = CV
Net charge enclosed by the surface 4 πε0  L L 5 
φ= Substituting C = 0.2 µF and V = 600 V,
ε0 V =
2q 1 − 1  we get q = 120 µC …(i)
 
The net charge due to a dipole is zero, 4 πε 0 L  5
When 0.2µF capacitor is joined to
hence φ = 0 1.0 µF capacitor in parallel combination,
32 As E = v / d and mg = qE .
Net enclosed charge then equivalent capacitance.
25 Total flux (φ T ) = So,
ε0 Now, voltage across the capacitor is
mgd 1.6 × 10−14 × 9.8 × 10 × 10−3 q 120µC
V = = given by V ′ = = = 100 V
Hence, we can say the electric flux q 1.6 × 10−19 C′ 1.2µF
depends only on net enclosed charge by
V = 104 V
the surface. 40 The combination shows two capacitors
33 In a conducting sphere charge is present connected in series. Resultant
26 Flux of electric field E through any area capacitance is
A is defined as φ = E A cos θ. Now, angle on the surface of the sphere. So, electric
1 1 1
between normal and field lines is (90 − θ). field inside will be zero and potential = +
remains constant from centre to surface C ′ C1 C2
So, φ = EA cos (90 − θ)
1 Q Kε 0 A
= EA sin θ of sphere and is equal to . C = , where A is area, d is the
4 πε0 R d
= EL sin θ
2

34 Spherical equipotential surface exists distance between the plates and K is


27 According to Gauss’ law, the electric only for point charges. dielectric constant (= 1). Therefore,
flux through a closed surface is equal to ε0 A ε A
1 35 Electric potential inside a charged C1 = ,C2 = 0
times the net charge enclosed by the d /2 d /2
ε0 sphere is everywhere same as that on
the surface. 1 d /2 d /2 ε A
surface. ∴ = + ⇒C′ = 0 = C
C ′ ε0 A ε0 A d
As the charge enclosed = q / 8 36 Net charge on the parallel plate
q capacitor is 1µC Hence, on inserting aluminium sheet the
So, electric flux = enclosed ⇒ Charge on each plate of the capacitance remains the same.
ε0
capacitor, q = 0.5 µC
⇒ φ=
q 41 Here, q = 8 × 10−18 C,
Now, force is given by F = qE
8ε0 C = 100 µF = 10−4 F
Substitutingq = 0.5 µC = 0.5 × 10−6C and
28 The flux passing through the square of E = 105 Vm −1 , we get F = 0.05 N q 8 × 10−18
V = = = 8 × 10−14 V
1m placed in xy plane inside the C 10−4
electric field is zero. Because surface 37 If a battery is disconnected from the 1
capacitor and then a dielectric substance Work done = qV
area is parallel to the electric field. 2
between the two plates of condenser, so
So, E⋅d S = 0 1
= × 8 × 10−18 × 8 × 10−14
capacity, potential and potential energy
2
⇒ ∫ E⋅d S = 0 are increased at saturated point and then
= 32 × 10−32 J
it will decreases.
210 40 DAYS ~ NEET PHYSICS DAY NINETEEN

42 Minimum number of condensers in each 47 Graph (c) will be the right graph, the If q be the charge on each capacitor,
3000 q q q q q
row = =6 electric field inside the dielectrics will then + + + + = 59
500 be less than the electric field outside the 21 7 28 3 7
If Cs is capacity of 6 condensers in a dielectrics. The electric field inside the or q = 84µC
row, dielectrics could not be zero. ∴ Potential difference across A
1 1 1 1 1 1 1 q 84
= + + + + + =6 = = = 12V
Cs 1 1 1 1 1 1 7 7
1 E 2
Cs = µF 53 Stored energy, W = Q
6 2C
Let there be m such rows in parallel, (2Q )2 4Q 2
Total capacity = m × Cs W1 = = = 4W
2C 2C
1 0
2= m× d
6 54 The equivalent capacitance, for
∴ m = 12 As K 2 > K 1 , the drop in electric field for capacitors in series is
Total number of capacitors = 6 × 12 = 72 K 2 dielectric must be more than K 1 . 1 1 1
= +
Cs C1 C2
43 When plates of the capacitor are 48 Heat produced in wire
= Energy stored in capacitor 1 1
charged, then opposite charges are = +
1 1 10 20
induced on water, so due to attractive
= C V 2 = × (2 × 10−6 ) (200)2 20
force water level will rise. 2 2 ⇒ Cs = µF
3
44 The energy stored in the condenser = 4 × 10−2 J
10 µF 20 µF
1 Aε
CV 2 ⇒ U =  0  (Ed )2
1
U = 49 Both 10µF capacitances are in series.
2 2 d 
Hence, equivalent capacitance can be
QC = Aε0 and V = Ed  calculated as, i.e.
 d  1 1 1 1
= + = ⇒ C1 = 5µF
1 C1 10 10 5
U = ε0 E 2 Ad
2 Now, C1 = 5 µF and C2 = 5 µF, both are 30 V
45 Common potential is given by in parallel. 20
Hence, equivalent capacitance, Also, q = CV =
× 30 = 200 µC
C V − C2V2 3
V = 1 1 C = C1 + C2 = 5 + 5 = 10 µF
C1 + C2 This charge on the two capacitors in
50 The two condensers in the circuit are in series is same.
Substituting
parallel order, hence Hence, q ′ = 200 µC, q ′′ = 200 µC
C1 = 3µF, C2 = 6 µF, V1 = V2 = 12 V, C 3C
C′=C + =
we get, potential difference, V = 4 V 2 2 SESSION 2
46 In series arrangement charge on each The work done in charging the 1 As q2 , q3 are positive charges and net
plate of all the capacitors have same equivalent capacitor is stored in the force on q1 is along + x-direction,
magnitude. The potential difference is form of potential energy. therefore q1 must be negative as shown
distributed inversely in the ratio of 1
Hence, W = U = C ′ V 2 in figure.
capacitors, 2
When a positive charge Q is added at
i.e. V = V1 + V2 + V3 1  3C  2 3
=   V = CV
2
( x, 0), it will attract (− q1 ) along + x
[QV1 = V2 = V3 = V ] 2 2  4 direction, in figure. Therefore, force on
Here, V = 3V 51 If the outer sphere is earthed, then q1 will increase along the positive
inside of outer sphere and outside of x-axis.
The equivalent capacitance Cs is given by
inner sphere constitute a spherical y y
C1 C2 C3
+Q –Q +Q –Q +Q –Q capacitor. So, the capacitance of the
ab +q2 +q2
system is C = 4 πε0 .
V1 V2 V3 b −a
Q
52 The equivalent circuit is x x
(–)q1 O (–)q1 O (x, 0)
+ – 59 V
S
V –q3 +q3
21 µF 7 µF
1 1 1 1 (a) (b)
= + +
Cs C1 C2 C3 7 µF A

[QC1 = C2 = C3 = C ] 2 Resultant force,


C 28 µF 3 µF F′= F 2 + F 2 + 2FF cos 120° = F
Cs =
3
DAY NINETEEN ELECTROSTATICS 211

5 Gauss’ law states that the net electric 8 In first case, when charge + Q is situated
flux through any closed surface is equal at C.
F
F' +Q
to the net charge inside the surface +q –q
+Q
divided by ε0 .
120° A A C B
q
F i.e. φ total = L L
ε0
2L
F '' Let electric flux linked with surfaces
A, B and C are φA , φ B and φ C , Electric potential energy of system
respectively. 1 (q )(− q ) 1 (− q )Q
U1 = +
B C i.e. φ total = φ A + φ B + φC 4 πε0 2L 4 πε0 L
–Q +Q
[Q φC = φA ] 1 qQ
+
q 4 πε0 L
Now, from figure ∴ 2φ A + φ B = φ total =
ε0 In second case, when charge + Q is
F = F ′2 + F ′′2 + 2F ′ F ′′ cos 90°
1 q  moved from C to D
Now, the force normal to BC or φA =  − φB  +q –q +Q
2  ε0 
at vertex A is
A B D
F ′′ = F 2 − F ′2 = 0 [Q F ′ = F ] [Q φ B = φ]
2L L
1 q 
Hence, φA =  − φ Electric potential energy of system in
3 Electrostatic force between electron and 2  ε0 
proton is given by that case
6 Given that conducting plates have 1 (q )(− q ) 1 qQ
1 q1 q2 U2 = +
Fe = surface charge densities + σ and −σ, 4 πε0 2L 4 πε0 3L
4 πε0 r 2
respectively. Since, the sheet is large, 1 (− q )(Q )
+
1 4 πε0
Substituting = 9 × 109 Nm2C −2 the electric field E at energy point near L
4 πε0 the sheet will be perpendicular to the
∴ Work done = ∆U = U 2 − U 1
sheet.
q1 = 1.6 × 10−19C , q2 = 1.6 × 10−19C ,  1 q2 1 qQ 1 qQ 
= − + − 
r = 5 × 10−11 m, we get + –
 4 πε0 2L 4 πε0 3L 4 πε0 L 
+ –
−8 –
Fe = 9.22 × 10 N +
–  1 q2 1 qQ 1 qQ 
+
– − − − + 
E E
Now, gravitational force between E +
+
E –  4 πε0 2L 4 πε0 L 4 πε0 L 
electron and proton is given by + –
qQ (1 − 3) qQ
= =−
M M 4 πε0 3L 6 πε0 L
Fg = G 1 2 The resultant electric field is given by
r2 E ′ = E + E = 2E 9 When charge q3 is at C, then its potential
Substituting G = 6.67 × 10−11 , If σ is surface charge density, then
σ energy is
M1 = 9.1 × 10−31 k g , electric field E = 1  q1q3 + q2 q3 
2ε0 UC =  
M2 = 1.61 × 10 −27
kg and 4 πε0  0.4 0.5 
2σ σ
∴ E ′ = 2E = = Vm −1 When charge q3 is at D, then
r = 5 × 10 −11
, we get 2ε0 ε0
1  q1q3
+ 2 3 
q q
UD = 
F g = 3.9 × 10−47 N 7 Volume of n mercury droplets 4 πε0  0.4 . 
01
= Volume of bigger drop
So, the ratio of electrostatic and Hence, change in potential energy
4 4
gravitational forces is given by ⇒ n × πr 3 = πR3 ⇒ nr 3 = R3 ∆U = U D − U C
3 3
1  q1q3
+ 2 3 
q q
Fe 9.22 × 10−8 ⇒ R = n1 /3 r …(i) = 
= = 2.36 × 1039 4 πε0  01
. 0.5 
Fg 3.9 × 10−47 Now, charge on bigger drop = n q3
But ∆U = k
(Charge on each mercury drop) 4πε0
4 When a point positive charge is brought ⇒ q ′ = nq …(ii)
near an isolated conducting sphere, q3 1  q2 q3 q2 q3 
Therefore, potential of bigger drop ∴ k =  − 
there developes some negative charge on 4 πε0 4 πε0  01
. 0.5 
becomes,
left side of the sphere and an equal 1 q′ ⇒ k = q2 (10 − 2) = 8q2
V′ =
positive charge on the right side of the 4 πε0 R dV
sphere. Electric lines of force emitting
10 E = − = − 2ar
Using Eqs. (i) and (ii), we get dr
from the point positive charge and 1 nq From Gauss’ theorem,
normally on the left side of the =
4 πε0 n1 /3 r E (4 πr 2 ) = q / ε0 ⇒ q = − 8 π ε0 ar 3
sphere,the electric lines of force dq dq dr
 1 q ρ= = ×
emanate normally from the right ⇒ V ′ = n2 /3   =n V
2 /3
dv dr dv
side.The electric field is best given by  4 πε0 r 
 1
Fig (i). 1 q = (− 24 π ε0 ar 2 ) ×
 here,V = 4 πε r  4 πr 2
 0 
⇒ ρ = − 6 ε0 a
212 40 DAYS ~ NEET PHYSICS DAY NINETEEN

11 Choose the three coordinate axes as x, y Substituting, Given, U Series = U Parallel


1 C1 1
and z and plot the charges with the C = 1.77 µF = 1.77 × 10−6 C, or (4V )2 = (n2C2 ) V 2
given coordinates as shown in figure. 2 n1 2
d = 6.6 × 10−6, K = 200 16C1
O is the origin at which − 2q charge is ⇒ C2 =
placed. The system is equivalent to two ε0 = 8.85 × 10−12 , we get A = 103 m2 n2 n1
dipoles along x and y-directions,
respectively. The dipole moments of two
14 When a parallel plate air capacitor 17 In given figure, C2 and C3 are in
connected to a cell of emf V , then charge
dipoles are shown in figure. parallel.
stored will be
y ∴ C ′ = C2 + C3 = 4 µF
q
q = CV ⇒ V =
C C2 = 1 µF
1 q2 C1 = 4 µF
(0, a, 0)
P (a, a, 0) Also, energy stored is U = CV 2 =
q 2 2C
As the battery is disconnected from the
a p capacitor the charge will not be C3 = 3 µF
destroyed, i.e. q ′ = q with the A C4 = 6 µF
introduction of dielectric in the gap of B
–2q x
O p q (a, 0, 0) capacitor the new capacitance will be
(0, 0, 0)
q q C6 = 8 µF
a C ′ = CK ⇒ V ′ = =
C ′ CK C5 = 2 µF
z The new energy stored will be
q2 As C ′ and C1 are in series,
The resultant dipole moment will be U′ = ⇒ ∆U = U ′ − U 1 1 1 1 1
2CK = + = +
directed along OP, where P ≡ (a, a, 0). C ′′ C′ C1 4 4
The magnitude of resultant dipole q2  1  1 2 1 
=  − 1 = CV  − 1 ⇒ C ′′ = 2 µF
moment is 2C  K  2 K 
Similarly, C 4 and C 5 are in parallel
P′ = p + p
2 2 Hence, option (c) is incorrect.
C ′′′ = 6 + 2 = 8 µF
= (qa) + (qa) = 2 qa
2 2 15 As shown in figure below, the two C ′′′ and C 6 are in series
capacitors are connected in parallel. 1 1 1 1 1
= + = +
12 Let R and r be the radii of bigger and Initially, the capacitance of capacitor C ′′′ C ′′′ C6 8 8
each smaller drop, respectively. In ε0 A ⇒ C ′′′ = 4 µF
coalescence into a single drop, charge C=
d Now, C ′′′ and C ′′ are in parallel
remains conserved. Hence, charge on
bigger drop ∴ C = 4 µF + 2 µ F = 6 µF
= 27 × charge on smaller drop 18. On differentiating force between two
K1 = 8 K2 = 4 d
charges q and θ − q is
i.e. q ′ = 27q q(θ − q )
F =k w.r.t. q, we get
Now, before and after coalescing, r2
d  Kq
(Q − q )
volume remains same. dF
=
4 4 After filling with dielectrics, we have dq dq  r 2 
i.e. πR 3 = 27 × πr 3
3 3 two capacitors of capacitance. dF K d
or = [Qq − q 2 ]
∴ R = 3r K ε ( A /2) 8 ε0 A 4ε0 A dq r 2 dq
C1 = 1 0 = = = 4C
Hence, capacitance of bigger drop d 2 d d dF K
or = [Q − 2q] …(i)
C ′ = 4 πε0 R = 4 πε0 (3 r ) K ε ( A/2) dq r2
and C2 = 2 0
= 3(4 πε0 r ) = 3C d But we know that, when force is
4 ε0 A 2ε0 A dF
13 We know, the electric field is given by = = = 2C maximum, then =0
V V 2 d d dq
E = or E max =
d d min Hence, their equivalent capacitance, Then, from Eq. (i), we have
where, E max = dielectric strength Ceq = C1 + C2 = 4C + 2C = 6C K
[Q − 2q] = 0
Substituting V = 20 V i.e. new capacitance will be six times of r2
the original. or Q − 2q = 0
and E max = 3 × 106 V,
16 Case I When the capacitors are joined or Q = 2q
we get d min = 6.6 × 10−6 m in series or q =
Q
2
We know that capacitance is given by 1 C1
U Series = (4V )2 Q Q
Kε 0 A So, the other part = Q − =
C = 2 n1 2 2
d Hence, the each part have the same
Case II When the capacitors are joined
Cd in parallel Q
A= charge .
Kε 0 1 2
U Parallel = (n2C2 ) V 2
2
DAY NINETEEN ELECTROSTATICS 213

19 Energy stored in a system of capacitors 22 QTorque on an electric dipole in an 26 Force between plates of parallel
1 electric field, capacitor,
= Σ CV 2
2 τ = p × E ⇒ |τ| = pE sin θ  σ 
where, θ is angle between E and p F =q E =q  
⇒ 4 = p × 2 × 105 × sin 30  2ε0 
q
⇒ p = 4 × 10−5 cm QSurface charge density σ =
A
∴q 2 l = 4 × 10−5 [Q p = q 2 l ]
 q  q2
+ – where, 2l = 2 cm = 2 × 10−4 m ∴ F =q   ⇒ F =
 2 Aε0  2 Aε0
4 × 10−5
V ∴ q = ⇒ 2 × 10−3 C = 2 mC So, net charge across a capacitor, q = CV
2 × 10−2
Also, potential drop remains same in
F =
C 2V 2 C = Aε0 
parallel across both capacitors. 23 Given capacitor is equivalent to 2 Aε0  d 
Initially stored energy, capacitors K 1 , K 2 and K 3 in parallel and  Aε0  × CV 2
part of K 4 in series with them   2
1
U 1 = CV 2  d  CV
⇒ F = =
2 2 A ε0 2d
Finally, potential drop across each C1 C2 C3
K1 K2 K3 27 As we know that relation between
capacitor will be still V. So, finally
potential difference and electric field E
K4 K4 K4
stored energy, C4/3 C4/3 C4/3 in a particular region is given by,
1 1 1 dV
U 2 = CV 2 + CV 2 = (2C ) V 2 E= −
2 2 2 dr
1 3 3d 3d As V = 6 x − 8 xy − 8 y + 6 yz
= 2  CV 2  = 2U 1 + = +
1
2  C1 C 4 2K 1 ε0 A 2K 4 ε0 A So, E = −
dV
3d  1 1  dr
= +
2ε0 A  K 1 K 4  = − [(6 − 8y )$i + (−8 x − 8 + 6z )$j
20 Net charge on one H-atom
q = − e + e + ∆e = ∆e + 6yk$ ]
Kε 0 A
Net electrostatic repulsive force between ⇒ Ceq = The value of E at coordinate (1, 1, 1)
two H-atoms d
2ε0 A  K 1 K 4 K2K 4 K3K 4  E = − [−2i$ − 10$j + 6k$ ]
Kq 2 K (∆e )2 = + +
Fr =
d2
=
d2 3d  K 1 + K 4 K 2 + K 4 K 3 + K 4  So, Enet = (−2)2 + (−10)2 + 62
Similarly, net gravitational attractive 2  K1K 4 K2K 4 K3K 4  = 2 35 N/C
K = + +
Gm2r 3  K 1 + K 4 K 2 + K 4 K 3 + K 4  and force on charge q due to Enet is
force between two H-atoms, FG =
d2 So, none of these option is correct. given by
It is given that, F = q Enet = 2 × 2 35 = 4 35 N
24 Given, E = Ar …(i)
F r − FG = 0
K (∆e )2 Gm2r 28 Case I Fe cos θ = mg sin θ
⇒ − =0 Fe
d2 d2 ⇒ = tanθ
mg
Gm2r
⇒ (∆e )2 = F
K ⇒ mg = e …(i)
−11 a tanθ
(6.67 × 10 ) (1.67 × 10−27 ) 2 q
(∆e )2 = Fe cosθ
9 × 10 9

⇒ ∆e = 1.437 × 10−37 C θ
y

21 When the switch S is connected to point 1 q


Here, r = a ⇒ E = ⋅ Fe r
1, then initial energy stored in the 4 πε0 a2 (90 – θ)
θ
1
capacitor can be given as = (2µF ) × V 2 . From Eq. (i), we get
2 1 q
⋅ = Aa ⇒ q = 4 πε0 Aa3 s
When the switch S is connected to point 4 πε0 a2
2, energy dissipated on connection mg mg sinθ
across 8 µF will be 25 Given, potential in a region, Case II Fe ′ cos θ1 = mg sin θ1
1  C1C2  V = 6 xy − y + 2 yz
=   ⋅V
2 Fe ′
2  C1 + C2  Electric field in a region, ⇒ = tanθ1
∂V $ ∂V $ ∂V $ mg
1 2µF × 8 µF E =− i − j − k
= × × V2 ∂x ∂y ∂z Fe ′
⇒ mg = …(ii)
2 10 µF tanθ1
⇒ E = − 6 y$i − (6 x − 1) $j − 2 y k$
1
= × (1.6 µF ) × V 2 At, (1, 1, 0), electric field can be From Eqs. (i) and (ii), we get
2 expressed, Fe Fe ′ Fe tan θ
= ⇒ =
Therefore, % loss of energy E = − (6 × 1 i$ ) − (6 × 1 − 1) $j − 2 × 1 ⋅ k$ tan θ tan θ1 Fe ′ tan θ1
1.6
= × 100 = 80% = − (6i$ + 5$j + 2 k$ ) N /C
2
214 40 DAYS ~ NEET PHYSICS DAY NINETEEN

P (perpendicular ) r 30 Force on a charged particle in the ε0 A


As, tan θ = = C =
b (base ) 2y presence of an electric field is given as d
r r F = qE …(i) Substituting the value of C in
tan θ1 = P / b = ×2= where, q is the charge on the charged Eq. (iii), we get
2× y y
particle and E is the electric field. Q 2d Q2
F = =
From Newton’s second law of motion, 2 ε0 Ad 2 ε0 A
Fe cosθ1 force on a particle with mass m is given This means, electrostatic force is
as independent of the distance between the
y F = ma …(ii) plates.
θ1 where, a is the acceleration.
Fe y/ 2
From Eqs. (i) and (ii), we get 32 According to the question,
Fe
(90 – θ1) r qE For the time duration 0 < t < 1s,
F = ma = qE ⇒ a = …(iii)
θ1 m the velocity increase from 0 to 6 ms −1
mg Now, consider that a particle falls from As the direction of field has been
X rest through a vertical distance h. reversed for 1 < t < 2s, the velocity firstly
mg sinθ1 Therefore, u = 0 and the second
decreases from 6 ms −1 to 0.
equation of motion becomes
Fe r Then, for 2 < t < 3s; as the field strength
= × y ⇒ Fe ′ = 2Fe 1
s = ut + at 2
Fe ′ 2 y × r 2 is same; the magnitude of acceleration
1 would be same, but velocity increases
kq 2 2kq 2 or h = 0 × t + at 2
So, = 2 from 0 to − 6 ms −1 .
r ′2 r2
1 qE 2
= × t [from Eq. (iii)] 0<t<1s B 1<t<2s
1 2 2 m
= A
a –1 –a C
r′ r 2hm 2hm v=0 v = 6 ms v=0
r ⇒ t2 = or t = D
∴ r′ = qE qE –1
2 v = – 6 ms – a
Since, the particles given in the question 2 < t < 3s
29 According to question, two identical is electron and proton; and the quantity
charged spheres suspended from a 2h Acceleration of the car
(here, q p = qe = e ) for both of them v −u 6− 0
common point by two massless strings qE | a| = = = 6 ms −2
of length L. t 1
is constant. Thus, we can write
t =k m The displacement of the particle is given
θ as
l2 2h
where, k = 1
s = ut + at 2
qE 2
A q
F q or t ∝ m For t = 0 to t = 1 s
B
x/2 As, mass of proton (m p ) >> mass of u = 0, a = + 6 m / s 2
θ electron (me ).
x 1
C mg Thus, the time of fall of an electrons ⇒ s1 = 0 + × 6 × (1)2 = 3 m
2
would be smaller than the time of fall of
Q In ∆ABC For t = 1 s to t = 2 s
a protons.
F F
tan θ = or = tan θ …(i) u = 6 ms −1 , a = − 6 ms −2
mg mg 31 As we know that, the total work done in
transferring a charge to a parallel plate 1
⇒ s2 = 6 × 1 − × 6 × (1)2
Since, the charges begins to leak from capacitor is given as 2
both the spheres at a con stant rate. As a = 6 − 3 = 3m
Q2
result, the spheres approach each other W = …(i)
with velocity v. 2C For t = 2 s to t = 3 s

Therefore, Eq. (i) can be rewritten as


where, C is the capacitance of the u = 0, a = − 6 ms −1
capacitor. 1
Kq 2 x /2 ⇒ s3 = 0 − × 6 × (1)2 = − 3 m
= We can also wirte a relation for work 2
x2 mg x2 done as,
l2 − ∴Net displacement, s = s1 + s2 + s3
4 W = F ⋅d …(ii)
where, F is the electrostatic force =3m + 3m−3m=3m
Kq 2 x
⇒ = or q 2 ∝ x3 ⇒ q ∝ x3 /2 between the plates of capacitor and d is Hence, average velocity
x2 mg 2l
the distance between the plates. Net displacement 3
dq d ( x3 /2 ) dx = = = 1 ms −1
⇒ ∝ ⋅ From Eqs. (i) and (ii), we get Total time 3
dt dx dt Q2 Q2
W = = Fd ⇒ F = …(iii) Total distance travelled, d = 9 m
dq
⇒ ∝ x ⋅v
1 /2
2C 2Cd
dt Hence, average speed
As, the capacitance of a parallel plate is Total distance 9
⇒ v ∝
1
or v ∝ x −1 /2 given as = = = 3 ms −1
x1 /2 Total time 3
EXAM BITES

This Pdf Is
Downloaded From
www.exambites.in

Visit www.exambites.in for


More Premium Stuffs,Latest
Books,Test Papers,Lectures etc.
jeeneetadda
jeeneetadda_official
jeeneetadda

VISIT NOW !!
DAY TWENTY

Current
Electricity
Learning & Revision for the Day
u Electric Current u Temperature Dependence u Meter Bridge
u Ohm’s Law of Resistance u Potentiometer
u Resistance of u Electric Energy and Power u Galvanometer
Different Materials u Electric Cell u Ammeter
u Series and Parallel u Potential Difference and emf of a Cell u Voltmeter
Combinations u Kirchhoff’s Laws
of Resistors u Wheatstone’s Bridge

Electric Current
Electric current is defined as the amount of charge flowing across any section of wire per
unit time. If charge ∆q passes through the area in time interval ∆t at uniform rate, then
∆q
current i is defined as i=
∆t
SI unit of electric current is ampere (A).
l
Conventional direction of flow of current is taken to be the direction of flow of positive
charge or opposite to the direction of flow of negative charge.
Electric current is a scalar as it does not follow the vector law of addition.
PREP
l

Current Density MIRROR


Your Personal Preparation Indicator
Current per unit area is termed as current density.
I No. of Questions in Exercises (x)—
J = (Am −2 )
u

A u No. of Questions Attempted (y)—


It is a vector quantity. u No. of Correct Questions (z)—
(Without referring Explanations)

Drift Velocity u Accuracy Level (z / y × 100)—


l
Drift velocity is the average uniform velocity acquired by conduction electrons inside u Prep Level (z / x × 100)—
a metallic conductor on application of an external electric field.
The drift velocity is given by the relation In order to expect good rank in NEET, your
Accuracy Level should be above 85 & Prep
eE
vd = − τ Level should be above 75.
m
216 40 DAYS ~ NEET PHYSICS DAY TWENTY

where, τ known as relaxation time.


Resistance of Different Materials
l
Drift velocity per unit electric field is called the mobility of
the electrons. Thus, mobility, A perfect conductor would have zero resistivity and a perfect
insulator would have infinite resistivity. Though these are ideal
v e
µ = d  = τ limits, the electrical resistivity of substances has a very wide
E m range. Metals have low resistivity of 10 −8 Ωm to 10 −6 Ωm,
l
In terms of drift speed, electric current flowing through a while insulators like glass or rubber have resistivity, some 10 18
conductor is expressed as I = nAevd times (or even more) greater, Generally, good electrical
where, A = cross-section area of conductor, conductors like metals are also good conductors of heat, while
n = number of conduction electrons per unit volume, insulators like ceramic or plastic materials are also poor
vd = drift velocity of electrons thermal conductors.
and e = charge of one electron.
V-I Characteristics of Ohmic and
Ohm’s Law Non-ohmic Conductors
Substances obeying Ohm’s law are called Ohmic resistors, e.g.
Ohm’s law states that the physical conditions such as metals and their alloys. Substances which do not obey Ohm’s
temperature, mechanical strain, etc., are kept constant, the law are called non-ohmic resistors, e.g. electrolytes, gases,
current (i) flowing through a conductor is directly thermionic tubes, transistors, rectifiers, etc.
proportional to the potential difference across its two ends.
i.e. i ∝ V or V ∝ i or V = Ri
V
or = R = a constant, I I
i
where R depends on the nature of material and it given
dimension. V V
Non-ohmic resistors Ohmic resistors

Electrical Resistance
Electrical resistance is defined as the ratio in the potential
Colour Code for Resistors
difference (v) across the ends of the conductor to the current (i) The electronic colour code is
flowing through it, used to indicate the values or
V ratings of electronic components.
i. e., R= The resistance value and
i A B C R
tolerance can be determined Colour coded resistor
The SI unit of electrical restristance is Ω (ohm) and its from the standard resistor colour
dimension is [ML2 T −3 A−2 ]. code. The following diagram shows a carbon resistor. A
variation on the colour code is used for precision resistors
which may have five colour bands.
Electrical Resistivity
In that case, the first three bands indicate the first three digits
The resistance of a resistor (an element in a circuit with some of the resistance. Value and the fourth band indicates the
resistance R) depends on its geometrical factors (length, number of zeros. In the five band code, the fifth band is gold
cross-sectional area) as also on the nature of the substance of for 1% resistors and silver for 2%.
which the resistor is made. Electrical resistance of a
rectangular slab depends on its length (l ) and its Resistor Code Colour
cross-sectional area ( A). Resistance value 0 Black (B)
i. e., R∝l First three bands 1 Brown(B)
1
and R∝ 1st band-1st digit 2 Red (R)
A 2nd band-2nd digit 3 Orange (O)
Combining the two dependences, we get 3rd band-number of zeros 4 Yellow (Y)
l
R∝ 5 Green (G)
A
6 Blue (B)
ρl
or R= 7 Violet (V)
A
8 Grey (G)
where, ρ is a constant of proportionality called resistivity.
m 9 White (W)
ρ= 2
ne τ Shortcut to learn the series
B B R O Y Great Britain Very Good Wife.
DAY TWENTY CURRENT ELECTRICITY 217

Other expressions for power,


Series and Parallel Combinations
V2
of Resistors P = I 2 R watt ⇒ P =
R

Series Combination
A series circuit is a circuit in
Electric Cell
R1 R2 R3
which resistors are arranged in a V An electric cell is a device which maintains a continuous flow
chain, so the current has only one of charge (or electric current) in a circuit by a chemical
path to take. The current is the reaction. In an electric cell, there are two rods of different
I
same through each resistor. The Three resistor in series metals called electrodes.
total resistance of the circuit is
found by simply adding up the resistance values of the
individual resistors. Equivalent resistance of resistors in series
Internal Resistance of a Cell
Thus, when a current is drawn through a source, the potential
R = R1 + R2 + R3 + K
difference between the terminal of the source is
V = E − ir
Parallel Combination
This can also be shown as below
A parallel circuit is a circuit in I1 R1
E r
which the resistors are arranged I I2 I
A B
with their heads connected R1 R2 I
together and their tails connected V I3 R3 V A − E + Ir = VB or V A − VB = E − Ir
together. The current in a parallel
circuit breaks up, with some Three resistor in parallel Following three special cases are possible
flowing along each parallel branch (i) If the current flows in opposite direction (as in case of
and recombining, when the branches meet again. The voltage charging of battery), then V = E + Ir
across each resistor is parallel is the same.
(ii) V = E , if the current through the cell is zero.
The total resistance of a set of resistors in parallel is found by
(iii) V = 0, if the cell is short circuited.
adding up the reciprocals of the resistance values, and then,
taking the reciprocal of the total. This is because current in the circuit,
E
The equivalent resistance of resistors in parallel, I = or E = Ir
1 1 1 1 r
= + + +K
R R1 R2 R3 ∴ E − Ir = 0 or V = 0

Temperature Dependence Short circuited


of Resistance
E r
Resistance and resistivity of metallic conductors increases
with increase in temperature. The relation is written as Thus, we can summarise, it was follows
Rθ = R0 (1 + αθ + βθ ) and ρ θ = ρ 0 (1 + αθ + βθ )
2 2 E r

I
where, R0 and ρ 0 are values of resistance and resistivity at 0° C
and Rθ and ρ θ at θ° C. α and β are two constants whose value V = E − ir or V < E
vary from metal to metal. E r

I
Electric Energy and Power V = E + Ir or V > E
E r
Whenever the electric current is passed through a conductor, ⇒
it becomes hot after short time. This effect is known as
heating effect of current or Joule heating effect. V = E , if I = 0
2
I Rt
H = W = I 2 Rt joule = cal E
418
. i
r
The rate at which work is done by the source of emf in
maintaining the effect of current in a circuit is called electric r
power of the circuit, E
V = 0 is short circuited
P = VI watt
218 40 DAYS ~ NEET PHYSICS DAY TWENTY

The equivalent emf of the parallel grouping is given by


Potential Difference and E eq E 1 E2 E
emf of a Cell = + + 3 +…
req r1 r2 r3
Electromotive force (emf) of a cell is the terminal potential
If n cells, each of emf E and internal resistance r, all joined
difference of cell when it is in an open circuit, i.e. it is not r
supplying any current to the external circuit. However, when in parallel, then req =
n
it is supplying a current to an external resistance, the voltage
across the terminals of cell is called the terminal voltage or But E eq = E
terminal potential difference.
If E be the emf and r the internal resistance of a cell and a Kirchhoff’s Laws
resistance R is joined with it, then current in the circuit, Many electric circuits cannot be reduced to simple series
E parallel combinations. For example, two circuits that cannot
I = and terminal potential difference,
R+r be so broken down are shown in figure
ER
V = IR = or V = E − Ir A
B
(R + r ) C D
R1
E −V E 
Internal resistance of cell, r =   R = R  − 1 R2 R3 R4
 V  V 
Terminal voltage is more than emf of cell when cell is charged A B
R1 E1
and it is given by V = E + Ir .
D C E1 E2 E3
R2 E2 F
Combination of Cells in Series E
H G

and in Parallel F
R3
E
R5
A group of cells is called a battery. Two common grouping of (a) (b)
cells are
However, it is always possible to analyze such circuits by
applying two rules, devised by Kirchhoff.
1. Series Grouping
In series grouping, if all the cells are joined so as to supply
current in the same direction, then resultant Junction Rule
emf, Eeq = E 1 + E2 + E3 + … The algebraic sum of the currents at any junction is zero.
However, if one or more cells are joined so as to supply
current in reverse direction, then emf of that/those cells is i2
i1
taken as negative, while calculating the equivalent emf.
E1 E2 E3 i3
+ve –ve i4
r1 r2 r3
Three cell in series Illustration of Junction Rule

The equivalent internal resistance of the cell, i.e. Σ i =0


junction
req = r1 + r2 + r3 + …
This law can also be written as, ‘‘the sum of all the currents
If n cells, each of emf E and internal resistance r, are joined directed towards a point in circuit is equal to the sum of all
in series, then the currents directed away from that point.’’
E eq = nE and req = nr Thus, in figure, i 1 + i2 = i3 + i 4
The junction rule is bases on conservation of electric charge.
2. Parallel Grouping
E1
In parallel grouping, if positive +
terminals of all cells have been r1 Loop Rule
joined at one point and all E The algebraic sum of the potential difference in any loop
+ve + 2 –ve
including those associated emf’s and those of resistive
negative terminals at another r2
elements, must be equal to zero. That is, Σ ∆V = 0
point, then E3 closed loop

1 1 1 1 + This law represent conservation of energy.


= + + +… r3
Applying Kirchhoff’s law for the following circuit, we have
req r1 r2 r3 Three cell in parallel Resulting equation is Vr1 + Vr2 + Vr3 − 10 = 0.
DAY TWENTY CURRENT ELECTRICITY 219

E1
Wheatstone’s Bridge
For measuring accurately any resistance Wheatstone bridge is
widely used. There are two known resistors, are variable i i
resistor and one unknown resistors, one variable resistor and
L
one unknown resistor connected in bridge form as shown. A B
B i C
i2 =0
P Q
G
P G E2 , r
A C
Potentiometer
S2 Potential gradient,
R S
Potential difference across AB
k =
E
D Total length
V AB iR AB
S1 = = = iλ
L L
Wheatstone’s Bridge
R
where, λ = AB = resistance per unit length of
L
Meter Bridge potentiometer wire.
(Special Case of Wheatstone Bridge) The emf of source balanced between points B and C
R
This is the simplest form of Wheastone bridge and is specially E2 = kl = i CB × l = iRCB
useful for comparing resistances more accurately. The l
construction of the meter bridge is shown in the figure. It Here, AB is a long uniform resistance wire (length AB may be
consists of one metre resistance wire clamped between two ranging from 1 m to 10 m). E 0 is a battery whose emf is known
metallic strips bent at right angles and it has two points for supplying a constant current I for flow through the
connection. potentiometer wire. If R be the total resistance of
There are two gaps; in one of whose value is to be determined potentiometer wire and L its total length, then potential
is connected. The galvanometer is connected with the help of gradient, i.e. fall in potential per unit length along the
jockey across BD and the cell is connected across AC. After potentiometer will be
making connections, the jockey is moved along the wire and V IR E 0R
k = = =
the null point is found. Wheatstone bridge, wire used is of L L (R0 + R) L
uniform material and cross-section. the resistance can be
found with the help of the following relation where, E 0 = emf of battery,

S
R0 = resistance inserted by means of rheostat Rh
R
k = potential gradient.
B
L → balancing length
J → jockey.
G

A D C Applications of Potentiometer
I1 100 I1
(i) To find emf of an unknown battery
Metre sale E1 E1

V K1 i i i i
Meter Bridge
l1 l1
R l1 l1 A B A B
= or R =S i C1 i C2
S (100 − l 1) 100 − l 1
i2 =0 i2 =0
G G
Potentiometer Ek EU

Principle Potentiometer is an ideal device to measure the We calibrate the device by replacing E2 by a source of known
potential difference between two points. It consists of a long emf E k and then by unknown emf Eu . Let the null points are
resistance wire AB of uniform cross-section in which a steady obtained at lengths l 1 and l2 . Then,
direct current is set up by means of a battery.
220 40 DAYS ~ NEET PHYSICS DAY TWENTY

E K = i( ρl 1) and EU = i ( ρl2 )
Ammeter
Here, ρ = resistance of wire AB per unit length
An ammeter is a device used to measure current directly in
E K l1 l  ampere or its submultiples.
∴ = or EU =  2  E K
EU l2  l1 
I – Ig S
So, by measuring the lengths l 1 and l2 , we can find the emf of +ve –ve
G
an unknown battery. I Ig

(ii) To find the internal resistance of a cell Ammeter


Firstly, the emf E of the cell is balanced against a length A galvanometer may be converted into an ammeter of rating I
AD = l 1. For this, the switch S′ is left opened and S is closed. A by connecting a suitable low resistance (known as shunt S) in
known resistance R is then connected to the cell as shown. parallel with the galvanometer. Value of shunt resistance,
The terminal voltage V is now balanced against a smaller GI g
length AD′ = l2 . Here, now switch is opened and S′ is closed. S =
I − Ig
l1
l2 where, I g = maximum safe current (full scale deflection
S current) which can be passed through galvanometer,
E I = range of ammeter, G = resistance of galvanometer.
G
If I = nI g , then shunt S =
(n − 1)
D′ D
A B GS
The equivalent resistance of ammeter = .
(E, r) G+S
G
S′

R Voltmeter
E l1 A voltmeter is a device used to measure potential difference
Then, = across a circuit element in volts.
V l2
E R+r
Since, = {Q E = i(R + r ) and V = iR} +ve –ve
V R G
Ig R Ig
R + r l1 l 
or = ⇒ r =  1 − 1 R
R l2  l2  Voltmeter
A galvanometer may be converted into a voltmeter by
connecting a suitable high resistance R in series with
Galvanometer galvanometer. Value of series resistance,
V
It is a sensitive instrument used to detect and measure very R= −G
small currents even of the order of few micro ampere. Ig
Figure of merit of a galvanometer is defined as the current where, V = range of voltmeter.
which gives one division deflection in galvanometer. The equivalent resistance of voltmeter = G + R.
DAY PRACTICE SESSION 1

FOUNDATION QUESTIONS EXERCISE


1 The plot represents the flow of 2 8 When a wire of uniform cross-section a ,length l and
current through a wire at three resistance R is bent into a complete circle, resistance
1
different times. The ratio of between two of diametrically opposite points will be
0
charges flowing through the 1 2 3 4 5 6 7 8 R R R
1 (a) (b) (c) 4R (d)
wire at different times is (see Time in seconds 4 8 2
2
figure) 9 A wire of resistance 4Ω is stretched to twice its
(a) 2 : 1 : 2 (b) 1 : 3 : 3 (c) 1 : 1 : 1 (d) 2 : 3 : 4 original length. The resistance of stretched wire
2 Consider a current carrying wire (current I ) in the would be j NEET 2013

shape of a circle. Note that as the current progresses (a) 2 Ω (b) 4 Ω (c) 8 Ω (d) 16Ω
along the wire, the direction of J (current density)
10 6 Ω and 12 Ω resistors are connected in parallel. This
changes in an exact manner, while the current I
remains unaffected. The agent that is essentially combination is connected in series with a 10 V battery
responsible for is and 6 Ω resistor. What is the potential difference
(a) source of emf between the terminals of the12 Ω resistor?
(b) electric field produced by charges accumulated on the (a) 4 V (b) 16 V (c) 2 V (d) 8 V
surface of wire
11 The resistance R t of a conductor
(c) the charges just behind a given segment of wire which
push them just the right way by repulsion varies with temperature t as shown
(d) the charges ahead in figure. If the variation is
represented by R
3 Across a metallic conductor of non-uniform cross-section,
R t = R 0 (1 + α t + β t ). Then,
2
a constant potential difference is applied. The quantity
(a) α and β both negative
which remains constant along the conductor is
(b) α is positive and β is negative t
(a) current density (b) current j CBSE AIPMT 2015 (c) α and β both are positive
(c) drift velocity (d) electric field (d) α is negative and β is negative
4 Which of the following characteristics of electrons 12 An electric kettle has two heating coils. When one of the
determines the current in a conductor? coils is connected to an AC source, the water in the
(a) Drift velocity alone (b) Thermal velocity alone kettle boils in 10 min. When the other coil is used the
(c) Both drift velocity and thermal velocity water boils in 40 min. If both the coils are connected in
(d) Neither drift velocity nor thermal velocity parallel, the time taken by the same quantity of water to
boil will be
5 Charge passing through a conductor of cross-section
(a) 8 min (b) 4 min (c) 25 min (d) 15 min
area A = 0.3 m 2 is given by q = 3 t 2 + 5 t + 2 in coulomb,
where t is in second. What is the value of drift velocity at 13 Three equal resistors connected in series across a
t = 2s? (Take, n = 2 × 1025 /m 3 ) source of emf together dissipate 10 W power. If the same
resistors are connected in parallel across the same
(a) 0.77 × 10−5 m/s (b) 1.77 × 10−5 m/s
source, the power dissipated will be
(c) 2.08 × 105 m/s (d) 0.57 × 105 m/s
10
(a) 90 W (b) W (c) 30 W (d) 10 W
6 A resistor of 6 kΩ with tolerance 10% and another of 3
4 kΩ with tolerance 10% are connected in series. The
14 Two cities are 150 km apart. Electric power is sent from
tolerance of combination is about
one city to another city through copper wires. The fall of
(a) 5% (b) 10% (c) 12% (d) 15% potential per km is 8 V and the average resistance per km
7 A carbon resistor of (47 ± 4.7) k Ω is to be marked with is 0.5 Ω. The power loss in the wire is j CBSE AIPMT 2014
rings of different colours for its identification. The colour (a) 19.2 W (b) 19.2 kW
code sequence will be j
NEET 2018 (c) 19.2 J (d) 12.2 kW
(a) Yellow - Green - Violet - Gold 15 If voltage across a bulb rated 220 V-100 W drops by
(b) Yellow - Violet - Orange - Silver 2.5% of its rated value, the percentage of the rated value
(c) Violet - Yellow - Orange - Silver
by which the power would decrease is j CBSE AIPMT 2012
(d) Green - Orange - Violet - Gold
(a) 20% (b) 2.5% (c) 5% (d) 10%
222 40 DAYS ~ NEET PHYSICS DAY TWENTY

16 If power dissipated in the 9 Ω resistor in the circuit shown 22 When the resistance of 9 Ω is connected at the ends of a
is 36W, the potential difference across the 2 Ω resistor is battery, its potential difference decreases from 40 V to
j CBSE AIPMT 2011 30 V. The internal resistance of the battery is
9Ω (a) 6 Ω (b) 3 Ω (c) 9 Ω (d) 15 Ω
23 A 50 V battery is connected across 10 Ω resistor. The
6Ω current is 4.5 A. The internal resistance of the battery is
(a) zero (b) 0.5 Ω (c) 1.1 Ω (d) 5.0 Ω
24 The internal resistance of a 2.1 V cell which gives a current
+ – of 0.2 A through a resistance of10 Ω is j NEET 2013
V 2Ω (a) 0.2 Ω (b) 0.5 Ω (c) 0.8 Ω (d) 1.0 Ω
(a) 8 V (b) 10 V (c) 2 V (d) 4 V 25 A current of 2 A flows through a 2 Ω resistor when
17 A resistance coil and a battery are given. In which of the connected across a battery. The same battery supplies a
following cases, the heat generated is maximum? current of 0.5 A when connected across a 9 Ω resistor. The
internal resistance of the battery is j CBSE AIPMT 2011

(a) When the coil is directly connected to the battery as 1 1


(a) Ω (b) Ω (c) 1 Ω . Ω
(d) 05
such 3 4
(b) When the coil is divided into two equal parts and both
26 A cell of emf 1.5 V having a finite internal resistance is
parts are connected to the battery in parallel
connected to a load resistance of 2 Ω. For maximum
(c) When the coil is divided into four equal parts which are power transfer, the internal resistance of the cell should be
connected to the battery in parallel (a) 4 Ω . Ω
(b) 05
(d) When only half the coil is connected to the battery (c) 2 Ω (d) None of these
18 In a copper voltmeter, the mass deposited in 30 s is 27 In figure, values of I x and I y are respectively
m gram. If the current-time graph as shown in the figure,
0.3A
the ECE of copper, in gC −1, will be 0.7A A B 0.4 A 0.4 A
Ix 2.2A C Iy
1.0A
I (mA) (a) 1 A, 1A (b) 1.2 A, 1.2 A
(c) 0.8 A, 0.8 A (d) 1 A, 1.2 A
28 In figure, value of current I is
100 mA
1.2A 0.3A
t (s)
10 20 30 1.8A I
m
(a) m (b) (c) 0.6 m (d) 0.1 m
2 (a) 1.5 A (b) 0.4 A (c) 0.9 A (d) 0.7 A
19 A 5.0 A current is set up in an external circuit by a 6.0 V 29 In figure E is equal to
storage battery for 60 min. The chemical energy of the 3V 1.5Ω 1Ω
C
battery is reduced by A B
. × 104 J
(a)108 . × 10−4 J
(b) 108 2A 3Ω
. × 104 J
(c)18 . × 10−4 J
(d) 18 R
10V E
20 Two heater coils separately take 10 min and 5 min to boil
a certain amount of water. If both the coils are connected D 1A
8V
in series, the time taken will be
(a) 5 V (b) 4 V (c) 3 V (d) 2 V
(a) 2.5 min (b) 3.33 min
(c) 7.5 min (d) 15 min 30 The potential difference (VA − VB ) between the points A
and B in the given figure is j
NEET 2016
21 The number of dry cells each of emf 1.5 V and internal
3V
resistance 0.5 Ω that must be joined in series with a VA 2W + – 1W VB
resistance of 20 Ω, so as to send a current of 0.6 A
A I=2A B
through the circuit is (a) – 3 V (b) + 3 V
(a) 2 (b) 8 (c) + 6 V (d) + 9 V
(c) 10 (d) 12
DAY TWENTY CURRENT ELECTRICITY 223

31 In the circuit shown, the cells A and B have negligible 36 The resistances of the four arms P , Q , R and S in a
resistances. For VA = 12 V, R1 = 500 Ω and R = 100 Ω, the Wheatstone bridge are10 Ω, 30 Ω, 30 Ω and 90 Ω,
galvanometer (G) shows no deflection. The value ofVB is respectively. The emf and internal resistance of the cell
j CBSE AIPMT 2012 are 7 V and 5 Ω, respectively. If the galvanometer
R1 resistance is 50 Ω, the current drawn from the cell will be
G j NEET 2013
(a) 1.0 A (b) 0.2 A (c) 0.1 A (d) 2.0 A
VA R VB 37 In the Wheatstone’s bridge shown in figure, where
P = 2 Ω, Q = 3 Ω, R = 6 Ω and r = 8 Ω. In order to obtain
balance, shunt resistance across S must be
(a) 4 V (b) 2 V (c) 12 V (d) 6 V
32 See the electrical circuit shown in this figure. Which of P
the following equations is a correct equation for it ? Q
j CBSE AIPMT 2009
R

ε1 r1
i1
S R

i2

r2 ε2
(a) ε1 − (i1 + i 2 )R − i1r1 = 0 (b) ε2 − i 2r2 − ε1 − i1r1 = 0
(c) − ε2 − (i1 + i 2 ) R + i 2 r2 = 0 (d) ε1 − (i1 + i 2 )R + i1r1 = 0 (a) 2 Ω (b) 3 Ω
(c) 6 Ω (d) 8 Ω
33 Consider the following two statements:
38 In a meter bridge, the balancing length from the left end
I. Kirchhoff’s junction law follows from the conservation
of charge. (standard resistance of one ohm is in the right gap) is
II. Kirchhoff’s loop law follows from the conservation of found to be 20 cm. The value of the unknown resistance is
energy. (a) 0.8 Ω (b) 0.5 Ω
(c) 0.4 Ω (d) 0.25 Ω
Which of the following is correct? j
CBSE AIPMT 2010
(a) Both I and II are wrong (b) I is correct and II is wrong
39 For a cell of emf 2V, a balance is obtained for 50 cm of
(c) I is wrong and II is correct(d) Both I and II are correct the potentiometer wire. If the cell is shunted by a 2 Ω
34 The resistance of each arm of the Wheatstone’s bridge is resistor and the balance is obtained across 40 cm of the
10 Ω. A resistance of 10 Ω is connected in series with wire, then the internal resistance of the cell is
galvanometer, then the equivalent resistance across the (a) 0.025 Ω (b) 0.50 Ω
(c) 0.80 Ω (d) 1.00 Ω
battery will be
(a) 10 Ω (b) 15 Ω (c) 20 Ω (d) 40 Ω 40 Figure shows a simple potentiometer circuit for
measuring a small emf produced by a thermocouple.
35 Thirteen resistances each of resistance R ohm are
connected in the circuit as shown in the figure below. 2.00 Driver cell R
The effective resistance between A and B is
R R
0.600 m
P Q
R R Thermocouple

R R Galvanometer
6.00 mV
A R B
The meter wire PQ has a resistance of 5 Ω and the driver
R R cell an emf of 2.00 V. If a balance point is obtained
R 0.600 m along PQ when measuring an emf of 6.00 mV,
R
what is the value of resistance R ?
(a) 95 Ω (b) 995 Ω
R R (c) 195 Ω (d) 1995 Ω
2R 4R
(a) R Ω (b) Ω (c) Ω (d) 2R Ω
3 3
224 40 DAYS ~ NEET PHYSICS DAY TWENTY

41 A potentiometer wire has length 4 m and resistance 8 Ω. 45 Two cells of emfs approximately 5 V and 10 V are to be
The resistance that must be connected in series with the accurately compared using a potentiometer of length
wire and an accumulator of emf 2 V, so as to get a 400 cm.
potential gradient 1 mV per cm on the wire is (a) The battery that runs the potentiometer should have
j CBSE AIPMT 2015 voltage of 8 V
(a) 32 Ω (b) 40 Ω (b) The battery of potentiometer can have a voltage of
(c) 44 Ω (d) 48 Ω 15 V and R adjusted, so that the potential drop across
the wire slightly exceeds 10 V
42 A potentiometer circuit has been set up for finding the
(c) The first portion of 50 cm of wire itself should have a
internal resistance of a given cell. The main battery, used
potential drop of 10 V
across the potentiometer wire, has an emf of 2.0 V and a
(d) Potentiometer is usually used for comparing resistances
negligible internal resistance. The potentiometer wire and not voltages
itself is 4 m long. When the resistance R connected
across the given cell, has values of 46 A galvanometer of resistance 100 Ω gives full scale
deflection for 10 mA current. What should be the value of
(i) infinity
shunt, so that it can measure a current of 100 mA?
(ii) 9.5 Ω
The ‘balancing length’, on the potentiometer wire are (a) 11.11 Ω (b) 9.9 Ω
(c) 1.1 Ω (d) 4.4 Ω
found to be 3 m and 2.85 m, respectively.
47 A circuit contains an ammeter, a battery of 30 V and a
The value of internal resistance of the cell is
resistance 40.8 Ω all connected in series. If the ammeter
j CBSE AIPMT 2014
(a) 0.25 Ω
has a coil of resistance 480 Ω and a shunt of 20 Ω, then
(b) 0.95 Ω reading in the ammeter will be j CBSE AIPMT 2015

(c) 0.5 Ω (a) 0.5 A (b) 0.25 A


(d) 0.75 Ω (c) 2 A (d) 1 A
43 A potentiometer is an accurate and versatile device to 48 A millivoltmeter of 25 mV range is to be converted into an
make electrical measurement of emf because the method ammeter of 25 A range. The value (in ohm) of necessary
involves shunt will be j
CBSE AIPMT 2012
(a) cells j
NEET 2017 (a) 0.001 (b) 0.01 (c) 1 (d) 0.05
(b) potential gradients 49 The range of a voltmeter of resistance G Ω is V volt. The
(c) a condition of no current flow through the galvanometer resistance required to be connected in series with it in
(d) a combination of cells, galvanometer and resistances order to convert it into a voltmeter of range nW volt, will
44 A potentiometer wire of length L and a resistance r are be
connected in series with a battery of emf E 0 and a (a) (n − 1)G (b) G / n
resistance r1. An unknown emf is balanced at a length l of (c) nG (d) G/(n − 1)
the potentiometer wire. The emf E will be given by 50 A 100 V voltmeter of internal resistance 20 kΩ in series
j
CBSE AIPMT 2015 with a high resistance R is connected to 110 V line. The
LE 0r voltmeter reads 5V, the value of R is
(a)
l r1 (a) 210 kΩ (b) 315 kΩ
E r l (c) 420 kΩ (d) 4440 kΩ
(b) 0 .
(r + r1) L
E l
51 A galvanometer has a coil of resistance 100 Ω and gives a
(c) 0 full scale deflection for 30 mA current. If it is to work as a
L
LE 0 r voltmeter of 30 V range, the resistance required to be
(d)
(r + r1) l added will be j
CBSE AIPMT 2010
(a) 900 Ω (b) 1800 Ω (c) 500 Ω (d) 1000 Ω
DAY TWENTY CURRENT ELECTRICITY 225

DAY PRACTICE SESSION 2

PROGRESSIVE QUESTIONS EXERCISE


1 A cylindrical conductor AB of non-uniform area of 6 Two cells having the same emf, are connected in series
cross-section carries a current of 5 A. The radius of the through an external resistance R. Cells have internal
conductor at one end A is 0.5 cm. The current density at resistances r1 and r2 (r1 > r2 ), respectively. When the
the other end of the conductor is half of the value at A. circuit is closed, the potential difference across the first
The radius of the conductor at the end B is nearly cell is zero. The value of R is
(a) 1.4 cm (b) 0.7 cm r1 + r2 r1 − r2
(a) r1 − r2 (b) (c) (d) r1 + r2
(c) 0.6 cm (d) None of these 2 2

2 A metal rod of the length 10 cm and a rectangular 7 Two batteries of emf E 1 and E 2 E1
(E 2 > E 1) and internal resistances r1 r1
cross-section of 1 cm × 1/2 cm is connected to a battery
across opposite faces. The resistance will be and r2 respectively are connected
A B
in parallel as shown in figure.
(a) maximum when the battery is connected across
1 cm × 1/2 cm faces (a) The equivalent emf Eeq of the two r2
E2
(b) maximum when the battery is connected across cells is between E1 and E 2 , i.e.
10 cm × 1 cm faces E1 < Eeq < E 2
(c) maximum when the battery is connected across (b) The equivalent emf Eeq is smaller than E1
10 cm × 1/2 cm faces (c) The Eeq is given by Eeq = E1 + E 2 always
(d) same irrespective of the three faces (d) Eeq is independent of internal resistances r1 and r2

3 Each of the resistors showing in figure has resistance R. 8 A, B and C are voltmeters of
resistance R ,1.5R and 3R B
Find the equivalent resistance between A and B.
respectively as shown in the A
X Y
a b figure. When some potential C
difference is applied between X
f and Y, the voltmeter readings are VA , VB and VC ,
e h
A B g respectively. Then, j
CBSE AIPMT 2015
(a)VA = VB = VC (b)VA ≠ VB = VC
d c (c)VA = VB ≠ VC (d)VA ≠ VB ≠ VC

7R 5R 9 A filament bulb (500 W, 100 V) is to be used in a 230 V


(a) (b)
4 4 main supply. When a resistance R is connected in series,
9R 11R it works perfectly and the bulb consumes 500 W. The
(c) (d)
4 4 value of R is j
NEET 2016
4 The effective resistance between A and B in figure is (a) 230 Ω (b) 46 Ω (c) 26 Ω (d) 13 Ω

6Ω 8Ω 10 The resistance in the two arms of


the meter bridge are 5 Ω and 5Ω RΩ
3Ω 8Ω B R Ω, respectively.
A When the resistance R is shunted
10Ω 10Ω 5Ω
4Ω with an equal resistance, the new G
20Ω
balance point is at1.6l1. The
A B
4Ω resistance R, is j CBSE AIPMT 2014 l1 100 – l1
(a) 10 Ω (b) 15 Ω
(a) 2 Ω (b) 3 Ω (c) 5 Ω (d) 6 Ω (c) 20 Ω (d) 25 Ω
5 Two lamps P and Q are connected in parallel in an 11 A ring is made of a wire having a resistance R 0 = 12Ω.
electric circuit. Lamp P glows brighter than lamp Q. If RP Find the points A and B, as shown in the figure, at which
and R Q are their respective resistances, then a current carrying conductor should be connected, so
(a)RP > RQ (b) RP < RQ that the resistance R of the sub circuit between these
(c)RP = RQ 8
(d) None of these points is equal to Ω.
3 j
CBSE AIPMT 2012
226 40 DAYS ~ NEET PHYSICS DAY TWENTY

l1 17 Two metal wires of identical dimensions are connected in


series. If σ1 and σ 2 are the conductivities of the metal
A B wires respectively, the effective conductivity of the
combination is j CBSE AIPMT 2015

l2 2 σ1 σ 2 σ1 + σ 2
(a) (b)
l 5 l 1 l 3 l 1 σ1 + σ 2 2 σ1 σ 2
(a) 1 = (b) 1 = (c) 1 = (d) 1 =
l2 8 l2 3 l2 8 l2 2 σ + σ2 σ σ
(c) 1 (d) 1 2
σ1 σ 2 σ1 + σ 2
12 When a copper voltmeter is connected with a battery of
emf 12 V. 2g of copper is deposited in 30 min. If the 18 In an ammeter 0.2% of main current passes through the
same voltmeter is connected across a 6 V battery, then galvanometer. If resistance of galvanometer is G, the
the mass of copper deposited in 45 min would be resistance of ammeter will be j CBSE AIPMT 2014

(a) 1 g (b) 1.5 g (c) 2 g (d) 2.5 g 1 499 1 500


(a) G (b) G (c) G (d) G
499 500 500 499
13 The resistance of a wire is R ohm. If it is melted and
stretched to n times its original length, its new resistance 19 A potentiometer circuit is + –
will be j NEET 2017 set up as shown. The
R R potential gradient across
(a) nR (b) (c) n 2R (d)
n n2 the potentiometer wire, is
k volt/cm and the ammeter
14 A potentiometer wire is 100 cm long and a constant A B
present in the circuit, 1 G
potential difference is maintained across it. Two cells are 2
reads 1.0 A when two way
connected in series first to support one another and then 3
key is switched OFF. The
in opposite direction. The balance points are obtained at
balance points, when the R X
50 cm and 10 cm from the positive end of the wire in the
key between the terminals –
two cases. The ratio of emf is j
NEET 2016 A
(a) 1 and 2 (b) 1 and 3, is +
(a) 5 : 4 (b) 3 : 4 (c) 3 : 2 (d) 5 : 1 plugged in, are found to
15 Two batteries, one of emf 18 V and internal resistance be at lengths l1cm and l2 cm, respectively. The
2 Ω and the other of emf 12 V and internal resistance magnitudes, of the resistors R and X in ohm are then
1 Ω, are connected as shown in figure. The voltmeterV equal respectively to j
CBSE AIPMT 2010
will record a reading of (a) k (l 2 − l1) and kl 2 (b) kl1 and k (l 2 − l1)
V (c) k (l 2 − l1) and kl1 (d) kl1 and kl 2
20 A student measures the terminal potential difference (V )
2Ω
of a cell (of emf ε and internal resistance r) as a function
18V of the current (I ) flowing through it. The slope and
1Ω intercept of the graph between V and I respectively,
12V equal to j
CBSE AIPMT 2009
(a) 15 V (b) 30 V (c) 14 V (d) 18 V (a) ε and − r (b) − r and ε (c) r and − ε (d) −ε and r
−1
16 In the given circuit, find the potential difference across 21 A wire of resistance 12 Ω m is bent to form a complete
6 µF capacitor in steady state. circle of radius 10 cm. The resistance between its two
12 V diametrically opposite points A and B as shown in the
1Ω
A B figure is j
CBSE AIPMT 2009
4V
1.5Ω 1.5Ω
6Ω
1.5Ω
C D A B
8µF

1.5Ω 1.5Ω
(a) 0.6 π Ω (b) 3 Ω (c) 6 π Ω (d) 6 Ω
2µF
22 The charge following through a resistance R varies with
E F time t as Q = at − bt 2 , where a and b are positive
constants. The total heat produced in R is j
NEET 2016
3µF 6µF
a 3R a 3R a 3R a 3R
(a) (b) (c) (d)
(a) 4 V (b) 2 V (c) 6 V (d) None of above 3b 2b b 6b
DAY TWENTY CURRENT ELECTRICITY 227

23 In producing chlorine by electrolysis 100 kW power at 25 A battery consists of a variable number n of identical
125 V is being consumed. How much chlorine per minute cells (having internal resistance r each) which are
is liberated (Take, ECE of chlorine is 0.367 ×10−6 kg C−1 ) connected in series. The terminals of the battery are
j CBSE AIPMT 2010 short-circuited and the current I is measured. Which of
−3
. × 10 kg
(a) 176 the graphs shows the correct relationship between I
(b) 9.67 × 10− 3 kg and n ? j NEET 2018

(c) 17.61 × 10−3 kg


I I
(d) 3.67 × 10−3 kg
24 A set of n equal resistors, of value R each , are (a) (b)
connected in series to a battery of emf E and internal
resistance R . The current drawn is I. Now, the n resistors O O
n n
are connected in parallel to the same battery. Then, the
current drawn from battery becomes 10I. The value of n I I
is j NEET 2018
(c) (d)
(a) 20 (b) 11
(c) 10 (d) 9 O
O n n

ANSWERS
SESSION 1 1 (c) 2 (b) 3 (b) 4 (a) 5 (b) 6 (b) 7 (b) 8 (a) 9 (d) 10 (a)
11 (c) 12 (a) 13 (a) 14 (b) 15 (c) 16 (b) 17 (c) 18 (b) 19 (a) 20 (d)
21 (c) 22 (b) 23 (c) 24 (b) 25 (a) 26 (c) 27 (b) 28 (c) 29 (d) 30 (d)
31 (b) 32 (a) 33 (d) 34 (a) 35 (b) 36 (b) 37 (d) 38 (d) 39 (b) 40 (b)
41 (a) 42 (c) 43 (c) 44 (b) 45 (b) 46 (a) 47 (a) 48 (a) 49 (a) 50 (c)
51 (a)
1 (b) 2 (a) 3 (d) 4 (b) 5 (b) 6 (a) 7 (a) 8 (a) 9 (c) 10 (b)
SESSION 2 11 (d) 12 (b) 13 (c) 14 (c) 15 (c) 16 (b) 17 (a) 18 (c) 19 (c) 20 (b)
21 (a) 22 (d) 23 (c) 24 (c) 25 (c)

Hints and Explanations


SESSION 1 4 Current, I = n A e v d , i.e. I ∝ v d . Error in combination,
10 10
1 Charge = Area under the current-time Therefore, current in a conductor is ∆R = ∆R1 + ∆R2 = × 6+ ×4
graph determined by drift velocity alone. 100 100
= 0.6 + 0.4 = 1
q1 = 2 × 1 = 2, q2 = 1 × 2 = 2
5 Given, A = 03
. m2 , n = 2 × 1025/m3 , ∆R 1
1 ∴ = = 10%
and q3 = ×2×2=2 q = 3t 2 + 5t = + 2 C R 10
2
∴ q1 : q2 : q3 = 2 : 2 : 2 = 1 : 1 : 1 dq d 7 Given, R = (47 ± 4.7) kΩ
∴I = = (3t 2 + 5 t + 2) = 6t + 5
dt dt
2 The current density is a vector = 47 × 103 ± 10% Ω
At t = 25, I = 6 × 2 + 5 = 17
quantity. Its direction is given by the I As per the colour code for carbon
direction of flow of positive charge in ∴Drift velocity, v d = resistors, the colour assigned to
neA
the circuit. The same is possible due to numbers
17
electric field produced by charges = 4 – Yellow
accumulated on the surface of wire. 2 × 1025 × 1.6 × 10−19 × 03 .
7 – Violet
17 −5
3 The area of cross-section of conductor = = 177
. × 10 m/s 3 – Orange
0.96 × 10−6 For ± 10% accuracy, the colour is
is non-uniform, so current density will
be different, but the numbers of flow of silver. Hence, the bands of colours on
6 In series combination,
electron will be same, so current will carbon resistor in sequence are yellow,
R = R1 + R2 = 6 + 4 = 10 kΩ
be constant. violet, orange and silver.
228 40 DAYS ~ NEET PHYSICS DAY TWENTY

2
V2
8 When wire is bent to form a complete 13 In series, 10 = V ⇒ = 30 21 For n identical cells (series grouping),
circle, then 3R R nE n × 1.5
I = ⇒ 0.6 =
R/ 2 nr + R n × 0.5 + 20
V2
In parallel, P = = 3 × 30 = 90 W This gives, n = 10
 R
A B  
 3 22 The internal resistance of battery is
given by
14 Resistance = 150 × 0.5 = 75 Ω
r =  − 1 R =  − 1 × 9
R/ 2 E 40
∆V 8  V   30 
∴ I = = = 16 A
R ∆R 0.5
2 πr = R ⇒ r = 9 × 10
2π Therefore, power, = =3Ω
30
Resistance of each semi-circle P = I 2 R = (16)2 × 75 W
πR R 23 By applying kirchhoff’s loop law, we get
= πr = = = 19200 = 19.2 kW
2π 2
2
E = I(R + r )
Thus, net resistance in parallel 15 Power, P = V
R So, 50 = 4.5 (10 + r )
combination of two semi-circular
∆P 4.5 r = 50 − 45 = 5
R R R2 For small variation, × 100 % 5
× P ⇒ r = = 11
. Ω
resistances, R ′ = 2 2 = 4 = R 2∆V 4.5
R R R 4 = × 100 % = 2 × 2.5 = 5%
+ V 24 Current in the circuit,
2 2
Therefore, power would decrease E
9 When a wire is stretched, both its area by 5%. I = or E = I (R + r )
R+ r
and length changes. So, the new
resistance of wire, 16 Electric power, P = i 2 R ⇒ . = 0.2(10 + r )
21
R ′ = n2 R = 22 × 4 = 16 Ω P ⇒ r = 10.5 − 10 = 0.5Ω
∴ Current, i =
R
10 6Ω 25 Current, I = E
For resistance of 9 Ω, R+ r
I1 6Ω 36 E
i1 = = 4 = 2A 2= …(i)
12Ω
9 2+ r
I2
i × R 2× 9 E
and i2 = 1 = =3A and 0.5 = …(ii)
6 6 9+ r
Q I = i1 + i2 = 2 + 3 = 5A
On dividing Eq. (i) by Eq. (ii), we get
So, V2 = IR2 = 5 × 2 = 10 V 2 9+ r 9+ r
= ⇒4 = ⇒3r = 1
10 V 17 Since, battery supplies constant emf. 0.5 2 + r 2+ r
6 × 12 6 × 12 V2 1 1
∴ R= = = 4Ω So, power, P = or Power ∝ ⇒ r = Ω
6 + 12 18 R R 3
Total resistance, So, R should be minimum to generate
26 For maximum power, external
Req = 6 + 4 = 10 Ω maximum heat. In option (c), resistance
resistance = internal resistance
V 10 would be minimum. So, heat generated
∴ Current, I = = = 1A would be maximum. 2Ω = 2Ω
R 10
The current in 12 Ω resistor is 18 ∴ Average current, 27 At the node A, I x + 07
. + 03
. = 2.2
 R1   6  50 + 100 + 50 200 i.e. I x = 1.2 A
I2 = I   =1×   I = = mA
 R1 + R2   6 + 12  3 3
m 3m m At the node B, 2.2 A enters the node,
1 z= = =
⇒ I2 = Q while the other three currents leave the
3 I t 200 × 10−3 × 30 2
node. The unknown current at B is
The potential difference in 12 Ω 2.2 − (1.0 + 0.4) = 0.8 A
19 ∴Chemical energy reduced = VI t
resistor, Then, at the node C, 0.8 + 0.4 = I y
1 = 6 × 5 × 6 × 60 = 10800 = 1.08 × 104 J
V = I2 × R = × 12 = 4 V i.e. I y = 1.2 A
3
20 H is same, therefore t ∝ R 28 At the node A,1.8 = 1.2 + I x
11 Graph indicates that resistance 2
V i.e. I x = 0.6 A
increases with increase in temperature, Q H = t
so α and β both are positive. R Then, at the node B,
V2 H t R1 10 l x + 03
. = I = 0.6 + 03. = 0.9 A
12 H = t⇒ = = constant ∴ = =2
R V2 R R2 5
∴ t∝R 0.3 A
When the coils are connected in series, 1.2 A
R1 R2 Reff = R1 + R2 = 3R2 Ix B
Q Rp = A
R1 + R2 t 3 R2
=
t t 10 × 40 5 R2 1.8 A I
∴ tp = 1 2 = = 8 min
t 1 + t 2 10 + 40
∴ t = 15min
DAY TWENTY CURRENT ELECTRICITY 229

29 By Kirchhoff’s first law, current in 36 Effective resistance, 41 L=4m


branch CD is 3A. Q 8Ω
Applying KCL to the loop ACDA,
2 × 1.5 + 3 × 2 = 10 −`3 + E Since, 1 cm → 1 mV
10 Ω 30 Ω
9=7+ E ∴ 100 cm → 100 mV
or E =2V ⇒ 400 cm → 400 mV = 0.4 V
P R
G 50 Ω Change in potential,
30 VA 2A 1 Ω + – 1Ω 2
VB ∆V = 0.4 = ×8
3V 8+ R
90 Ω 30 Ω
16 160
Applying KVL, ⇒8+ R = = = 40 ⇒ R = 32 Ω
V A + ΣV = V B + 2 × 2 + 2 × 1 0.4 4
V A − V B − 3 = 4 + 2; V A − V B = 9 V S
42 Internal resistance of a cell,
31 Concept If potential difference across l 
r =  1 − 1 R
R Ω resistor is equal to potential  l2 
7 V, 5 Ω
difference of cell B, galvanometer shows
1 1 1 
∴
3 
− 1 9.5 Ω =
0.15
× 9.5 Ω= 0.5 Ω
no deflection. = + …(i)  2.85  2.85
Reff R1 R2
Applying Kirchhoff’s law,
Then, R1 = 10 + 30 43 When a cell is balanced against
500I + 100I = 12
−2
⇒ R1 = 40 potential drop across a certain length of
12 × 10 R2 = 90 + 30 = 120
So, I = Now, potentiometer wire, no current flows
6 R2 = 120 through the cell.
= 2 × 10−2 A By Eq. (i), we get ∴emf of cell = potential drop across
1 1 1 balance length of potentiometer wire.
= +
Hence, V B = 100 (2 × 10−2 ) Reff 40 120 So, potentiometer is a more accurate
= 2V 40 × 120 4800 device for measuring emf of a cell or no
⇒ Reff = = current flows through the cell during
32 The algebraic sum of the changes in 120 + 40 160
measurement of emf.
potential in complete transversal of a = 30 Ω
mesh (closed loop) is zero, i.e. ΣV = 0 In the balancing condition, 44 Consider a potentiometer wire of length
So, ε1 − (i1 + i2 )R − i1 r1 = 0 L and a resistance r are connected in
7  E 
∴Current, I = Q I =  series with a battery of emf E 0 and a
33 Kirchhoff’s junction law follows from (30 + 5)  R + r resistance r1 as shown in figure. Current
the conservation of charge. 7 E0
= = 0.2 A in wire AB =
Kirchhoff’s loop law follows from the 35 r1 + r
r1
conservation of energy.
37 For a balanced wheat stone’s bridge,
34 To keep a resistance in series with a P x E0
= ⇒ x= 4Ω
balanced galvanometer is not Q R l
meaningful. The bridge would stay P r
1 1 1 1 1 1 A
balanced. Therefore, net resistance and + = ⇒ + = , gives L
B
S r x 8 r 4
across the battery
−1 r = 8Ω
 1 1  E
= +  38 For a meter bridge,
 10 + 10 10 + 10  P l 20
Potential gradient,
= 1 ⇒ P = × 1 = 0.25 Ω Ir  E 0  r
= 10 Ω Q l2 80 x= = 
L  r1 + r  L
35 Equivalent circuit will be 39 Internal resistance of the cell, emf produced across E will be given by
P  l − l2  50 − 40  E r l
r = 1  R= × 2 = 0.50 Ω E = x⋅l =  0 
2
R1 = R 2  l2  40  r1 + r  L
3 R1 = R
3
40 The voltage per unit length on the meter 45 In a potentiometer experiment, the emf of
wire PQ is a cell can be measured, if the potential
R
A B 6.00 mV
or 10 mVm −1 drop along the potentiometer wire is more
0.60 m than the emf of the cell to be determined.
2 2
R1 = R R1 = R Hence, potential across the meter wires As values of emfs of two cells are
3 3
PQ is 10 mV −1 (1 m) = 10 mV. Current approximately 5 V and 10 V, therefore the
Q potential drop along the potentiometer
drawn from the driver cell is
10 mV wire must be more than 10 V. Hence,
Now, above circuit is a Wheatstone’s I = = 2 mA option (b) is correct.
bridge. By solving, we get 5Ω
2 Resistance of the resistor R is 46 ∴ Shunt resistance,
R AB = R Ω GI g 100 × 10 100
3 2V − 10 mV 1990 mV S = = = = 1111
. Ω
R= = = 995 Ω
2 mA 2 mV I − Ig 100 − 10 9
230 40 DAYS ~ NEET PHYSICS DAY TWENTY

47 Effective resistance of a circuit, SESSION 2 4 Figure is equivalent to the one shown


RS=20 Ω JA
below. It is a Wheatstone’s bridge in
1 Given, J B = which
2
C
40.8 Ω Q
6Ω 8Ω
RA=480 Ω P
JA JB 3Ω 8Ω
B
A 10Ω 10Ω
30 V I B A
480 × 20 20Ω
Reff = 40.8 + 4Ω S
480 + 20 i.e. I ( πr 2A ) = I ( πr B2 ) / 2 R
4Ω 5Ω
= 40.8 + 192. = 60 Ω r B = (r A ) 2 = 0.5 × 1.414 = 0.7 cm
So, current flowing across ammeter, D
I =
V
=
30 1
= = 0.5 A 2 We know that, R = ρl 6×3 8
R 60 2 A P = = 2 Ω, Q = = 4 Ω,
6+ 3 2
Hence, reading of ammeter = 0.5 A 4 20 × 5
R = = 2 Ω and S = = 4Ω
48 The full scale deflection current, 1 cm 2 20 + 5
25 mV 2 1cm P 2 R
ig = A We find that = =
G 10 cm Q 4 S
where, G is the resistance of the
i.e. The bridge is balanced. Effective
meter. (a) When the battery is connected
resistance between A and B is
The value of shunt required for across 1 cm × 1/2 cm faces, then (P + Q ) (R + S ) (2 + 4) (2 + 4)
converting it into ammeter of range 25 l = 10 cm; A = 1 × 1 / 2 cm2 , R AB = =
igG P + Q + R + S 2+ 4+ 2+ 4
A is, S = ρ × 10
i − ig R1 = = 20ρΩ 6× 6
1 × 1/2 = =3Ω
G 12
⇒ S = ig (as i >> i g ) (b) When the battery is connected
i 2

25 mV G 25 mV
across 10 cm × 1 cm faces, then 5 Power, Q = V t or Q = P ∝ 1
So that, S ≈ ⋅ = l = 1 / 2 cm, A = 10 × 1 cm2 , R t R
G i 25 PP RQ
= 0.001 Ω ρ × 1/2 ⇒ = >1
R2 = PQ RP
10 × 1
49 Potential, V = I g G ⇒ nV = I g (G + R ) ρ
= Ω ∴ R P < RQ
On dividing, R = (n − 1) G 20
6 Net resistance of the circuit
50 The circuit is as shown in figure. (c) When the battery is connected = r1 + r2 + R
20 kΩ across 10 cm × 1/2 cm faces, then Net emf in series = E + E = 2E
V
R l = 1 cm, A = 10 × 1 / 2 cm2 , E
100 V E
ρ× 1 ρ
R3 = = Ω
(10 × 1 / 2) 5
r2 r1 > r2
r1
3 The figure can be redrawn as follows I
110 V
Potential difference across voltmeter = 5R R/2
a b
∴ Current in the circuit
5 R
= = 0.25 × 10−3 A R/2 R/2
20 × 103 R A B R Therefore, from Ohm’s law, current in
e, f g, h the circuit,
Voltage across R1 , net emf
V1 = 110 − 5 = 105 V R/2 R/2 I=
net resistance
V 105
Hence, R = 1 = 2E
I 0.25 × 10−3 d c ⇒ I= …(i)
R/2 r1 + r2 + R
= 420 × 103 = 420 k Ω 3R/2
e,f g, h It is given that, as circuit is closed.
51 Required resistance to convert a ⇒ A B For first call, i.e. E − Ir1 = 0
R R
galvanometer into voltmeter of 30 V is E
3R/2 ⇒ I= …(ii)
V r1
given by, R = −G 3R
ig R AB =R+ + R
4 Equating Eqs. (i) and (ii), we get
Symbols have their usual meaning 11R E 2E
= = = r1 − r2
30 4 r1 r1 + r2 + R
= − 100 = 900 Ω
30 × 10−3
DAY TWENTY CURRENT ELECTRICITY 231

7 Refer figure the equivalent internal 230


⇒ R + 20 = = 46 14 According to question, emf of the cell is
resistance of two cells between A and B is 5
directly proportional to the balancing
E1 ∴ R = 26 Ω length i.e.
r1
5 R E ∝l …(i)
10 Initially, = ...(i)
A B l1 100 − l1 Now, in the first case, cells are
5 R connected in series to support one
Finally, = ...(ii)
r2 1.6 l1 2(100 − 1.6 l1 ) another i.e.
E2
R R Net emf = E1 + E2
1 1 1 r + r2 ⇒ = From Eq. (i), E1 + E2 = 50 cm (given)
= + = 1 1.6 (100 − l1 ) 2(100 − 1.6 l1 )
req r1 r2 r1 r2 …(ii)
⇒ 160 − 1.6l1 = 200 − 3.2 l1
r1 r2 Again cells are connected in series in
or req = ...(i) ⇒ 1.6l1 = 40 ⇒ l1 = 25
r1 + r2 opposite direction i.e.
From Eq. (i), we get
5
=
R Net emf = E1 − E2
If Eeq is the equivalent emf of the two
cells in parallel between A and B, then
25 75 From Eq. (i), E1 − E2 = 10 …(iii)
⇒ R = 15 Ω From Eqs. (ii) and (iii)
Eeq E E E r + E2 r1
= 1 + 2 = 1 2 E1 + E2 50
req r1 r2 r1 r2 11 We know, R ∝ l =
E1 − E2 10
E r + E2 r1
∴ Eeq = 1 2 × req Here, R1 + R2 = 12 Ω E1 5+ 1 6 3
r1 r2 ⇒ = = =
R1 × R2 8 E2 5− 1 4 2
(E r + E2 r1 ) r1 r2 and = Ω
= 1 2 × R1 + R2 3
r1 r2 (r1 + r2 ) 15 It is clear that the two cells oppose each
E r + E2 r1 other, hence the effective emf in closed
= 1 2 ⇒ R1 R2 = 32 Ω
circuit is 18 − 12 = 6 V and net
(r1 + r2 ) We get, R1 = 8 Ω and R2 = 4 Ω resistance is 1 + 2 = 3 Ω (because in the
This shows that whatever may be the 12 l1 closed circuit, the internal resistances of
values of r1 and r2 , the value of Eeq is Again, R1 =
l1 + l2 two cells are in series).
between E1 and E2 . As E2 > E1 , so
E1 < Eeq < E2 . The current in circuit will be in
12 l2
and R2 = direction of arrow shown in figure.
8 1 = 1 + 1 l1 − l2 effective emf 6
R ′ RB RC I = = = 2A
l1 1 total resistance 3
Hence, = .
RB= 1.5R l2 2 The potential difference across V will be
B same as the terminal voltage of either
X P Q 12 Mass, m = ZIt ⇒ m = ZVt cell.
A R
RA=R C or m ∝ Vt
V
m1 V t
RC= 3R ∴ ∝ 1 1
m2 V2 t 2 18V
V
Here, m1 = 2g, V1 = 12 V,
2Ω
t 1 = 30 min, V2 = 6 V
1
=
1
+
1 and t 2 = 45 min
R ′ 1.5R 3R 2 12 × 30
= 1Ω
1 2+ 1 m2 6 × 45 12V
= ⇒R′= R
R′ 3R ⇒ m2 = 1.5 g
∴ V XP = V A = iR The mass of copper deposited = 1.5 g.
and V PQ = V B = VC = iR Since, current is drawn from the cell of
V A = V B = VC 13 Volume of material remains same in 18 V, hence
stretching.
V1 = E1 − Ir1
9 If a rated voltage and power are given, As volume remains same, A1 l 1 = A2 l 2
= 18 − (2 × 2) = 18 − 4 = 14 V
2
V rated Now, given l 2 = nl 1
then Prated =
A l A 16 The given circuit in steady state reduces
R ∴New area, A2 = 1 1 = 1
∴ Current in the bulb, l2 n to
P 12 V
i = (Q P = Vi ) Resistance of wire after stretching, 1Ω I
V l nl 1 A B
500 R2 = ρ 2 = ρ ⋅
i = = 5A A2 A1 / n
100 4V
∴ Resistance of bulb,  l 
=  ρ 1  ⋅ n2 = n2 ⋅ R 1.5Ω 1.5Ω
100 × 100  A1  C D
Rb = = 20 Ω 0
500
  l1   12 − 4
Q Resistance R is connected in series. Q R =  ρ  I = =2A
E 230   A1   4
∴ Current, i = =
R net R + Rb VCD = 3 × 2 = 6 V
232 40 DAYS ~ NEET PHYSICS DAY TWENTY

Now, change on 6 µF capacitor is 20 According to Ohm’s law 100 × 103


6V = 0367
. × 10−6 × × 60
= 12 µC dV 125
= − r and V = ε
1 + 1 1 dI = 17.61 × 10−3 kg
 
 3 6  µF if I =0 [as V + Ir = ε]
Potential difference across 6 µF So, slope of the graph = − r and 24 When n equal resistors of resistance R
capacitor is intercept = ε are connected in series, then the current
12 µC drawn is given as
=2V 21 E
6 µF I =
nR + r
17 Net resistance of a metal wire having A B where, nR = equivalent resistance of n
resistivity ρ, we have resistors in series and r = internal
12 π Ω resistance of battery.
L L 10
A Given, r = R
ρ1 ρ2 E E
⇒ I = = …(i)
L ⇒A B nR + R R(n + 1)
R1 = ρ1
A Similarly, when n equal resistors are
L 12 π connected in parallel, then the current
Similarly, R2 = ρ2 Ω
A 10 drawn is given as
E
Then, net effective resistance of two Circumference of circle I′ =
R
metal wires, 10 2 π π + R
= 2 πr = 2 × π × = =
Req = R1 + R2 100 10 5
n
2L L L R
⇒ ρ = ρ1 + ρ2 π 12 π where, = equivalent resistance of n
A A A Resistance of wire = 12 × = n
5 5 resistors in parallel.
⇒ 2ρ = ρ1 + ρ2
12 π
1 Resistance of each section = Ω Given, I ′ = 10 I
As, conductivity σ = , we have 10
ρ E nE
∴ Equivalent resistance ⇒ 10I = = …(ii)
2 1 1 2 σ + σ2 R ( n + 1)R
= + ⇒ = 1 12 π 12 π + R
σ σ1 σ2 σ σ1 σ2 × n
= 10 10 = 6 π = 0.6 π Ω
⇒ Net effective conductivity of 12 π 12 π 10 Substituting the value of I from
+
2σ 1 σ 2 10 10 Eq. (i) in Eq. (ii), we get
combined wires, σ =
σ1 + σ2  E  nE
22 Given, charge, Q = at − bt 2 …(i) 10   =
 R(n + 1) R(n + 1)
18 For ammeter, Q We know that, current, I =
dQ
dt ⇒ n = 10
a
0.002 I G So, Eq. (i) can be written as 25 If n identical cells are connected in
A d series, then
I = (at − bt 2 ) ⇒ I = a − 2bt …(ii)
0.9988 I dt Equivalent emf of the combination,
For maximum value of t, till the current E eq = nE
rs exist is given by ⇒ a − 2bt = 0 Equivalent internal resistance,
0.002I × G = 0.998I × rs ∴ t =
a
…(iii) r eq = nr
0.002 2b
rs = G E eq nE
0.998 Q The total heat produced (H ) can be ∴ Current, I = =
1 r eq nr
⇒ rs = 0.002004G = ×G given as
499 t E
I= = constant
Equivalent resistance of ammeter,
H = ∫0 I 2 R dt or
r
1 1 1 a /2b Q t = a 
R G
= +
rs
= ∫0 (a − 2bt )2 R ⋅ dt 


2b 
Thus, current (I ) is independent of the
number of cells (n ) present in the
1 1 1 G a /2b circuit.

R G
= +
G / 499
⇒R =
500
= ∫0 (a2 + 4b 2t 2 − 4abt ) Rdt
Therefore, the graph showing the
a /2b
 t3 4abt 2  relationship between I and n would be
19 The balancing length for R (when 1, 2 H =  a2t + 4b 2 −  R as shown below.
are connected) be l 1 and balancing  3 2 0
length for R + X (when 1, 3 is Solving above equation, we get
connected) is l 2 . a3 R I
Then, iR = kl 1 and i (R + X ) = kl 2 ⇒ H =
6b
Given, i = 1A
∴ R = kl 1 …(i) 23 Mass of the substance deposited at the
cathode is given by m = Z it
R + X = kl 2 …(ii)
(Z = electrochemical equivalent)
Also, subtracting Eq. (i) from Eq. (ii), we O n
= Z   t
get X = k (l 2 − l 1 ) W
V 
DAY TWENTY ONE

Unit Test 4
(Electrostatics and Current Electricity)

1 In given figure, E is equal to 4 A conducting loop has a charge of q = 4 µC. The


A thickness of the loop is negligible and its radius is
2A r = 6 cm. What is the electric field (E ) on the axis of
1V
symmetry of the loop at a distance of x = 8 cm from the
10 V centre of the loop?
R 1Ω E
1Ω (a) 288 × 102 Vm−1 (b) 288 × 103 Vm−1
1A (c) 288 × 104 Vm−1 (d) 288 × 105 Vm−1

B D
5 Eight dipoles of charges of magnitude e are placed
C
3Ω 5Ω inside a cube. The total electric flux coming out of the
(a) 1 V (b) 3 V cube will be
(c) 2.5 V (d) None of these
8e 16e
(a) (b)
2 In given figure, potential difference across 5 Ω resistor is ε0 ε0
nearly 3 V, 1 Ω e
(c) (d) zero
ε0

6 Consider a current carrying wire ( current I ) in the shape


2 V, 2 Ω
of a circle. Note that as the current progresses along the
5Ω
wire, the direction of J (current density) changes in an
(a) 2.4 V (b) 5 V exact manner, while the current l remains unaffected.
(c) 4 V (d) 6 V The agent that is essentially responsible for is
3 In the bridge as shown in figure, current through the (a) source of emf
galvanometer will be zero when the value of x is (b) electric field produced by charges accumulated on the
B surface of wire
5Ω 2Ω (c) the charges just behind a given segment of wire which
push them just the right way by repulsion
5Ω 3Ω
(d) the charge ahead
G
A C 7 Two conductors A and B of the same length and radius
x
are connected across the same battery. Resistivity of A is
10 Ω 40 Ω twice that of B. If PA and PB be the powers dissipated
across A and B respectively, then
D
(a) PA = PB (b) PA > PB
(a) 10 Ω (b) 20 Ω
(c) 40 Ω (d) 5 Ω (c) PA < PB (d) None of these
234 40 DAYS ~ NEET PHYSICS DAY TWENTY ONE

8 Electric potential at any pointV = − 5x + 3y + 15 z , then 15 In the given circuit diagram, E = 5 V, r = 1 Ω,


the magnitude of the electric field is R 2 = 4 Ω, R1 = R 3 = 1 Ω and C = 3 µF.
(a) 3 2 (b) 4 2 (c) 5 2 (d) 7
9 Eight mercury droplets having a radius of 1 mm and a C R1 C
charge of 0.066 pC each merge to have one droplet. Its
potential is R2
(a) 2.4 V (b) 1.2 V (c) 3.6 V (d) 4.8 V
10 A charged sphere of diameter 4 cm has a charge density C R3 C
−4 − 3
of 10 C cm . The work done in joule when a charge of
40 nC is moved from infinity to a point which is at a E,r
distance of 2 cm from the surface of the sphere is
Then, what is the numerical value of the charge on each
(a) 14.4 π (b) 28.8 π plate of the capacitor?
(c) 144 π (d) 288 π
(a) 24 µC (b) 12 µC (c) 6 µC (d) 3 µC
11 A current of 1.5 A flows through a copper voltmeter. The
thickness of copper deposited on the electrode surface 16 Two cells of emfs approximately 5 V and 10 V are to be
of area 50 cm 2 in 20 min will be accurately compared using a potentiometer of length
(Density of copper = 9000 kgm −3 and 400 cm.
ECE of copper = 0.00033 g C −1) (a) The battery that runs the potentiometer should have
−5 −4 voltage of 8 V
(a) 2.6 × 10 m (b) 2.6 × 10 m
(c) 1.3 × 10−5 m (d) 1. 3 × 10−4 m (b) The battery of potentiometer can have a voltage of 15 V
and R adjusted, so that the potential drop across the
12 A charge Q is distributed over two concentric hollow wire slightly exceeds 10 V
spheres of radii r and R (R > r ) such that, their surface (c) The first portion of 50 cm of wire itself should have a
densities are equal. Find the potential at the common potential drop of 10 V
centre k = ( 4πε 0 )−1. (d) Potentiometer is usually used for comparing resistances
kQ kQ (R + r ) and not voltages
(a) (b)
R+r (R 2 + r 2 ) 17 A 500 W heating unit is designed to operate on a 115 V
kQ kQ
(c) (d) line. If the line voltage drops to 110 V line, the
r R
percentage drop in heat output will be
13 A charged particle q is shot towards another charged (a) 7.6% (b) 8.5% (c) 8.1% (d) 10.2%
particle Q, which is fixed, with a speed v. It approaches Q
up to a closest distance r and then returns. If q were 18 A parallel plate capacitor is connected to a battery. The
given a speed of 2v, then closest distance of approach plates are pulled apart with a uniform speed. If x is the
would be separation between the plates, the time rate of change of
r electrostatic energy of capacitor is proportional to
(a) r (b) 2r (c) r /2 (d)
4 (a) x −2 (b) x
(c) x −1 (d) x 2
14 Figure below shows six equal capacitors each of
capacitance C connected to each other. 19 A 120 W, 60 V bulb is to be operated on a 120 V DC
supply. For this a resistor of resistance R is connected in
C1
A series with the bulb. The value of R is
C2 C3 (a) 60 Ω
a (b) 30 Ω
b
(c) 90 Ω
(d) 100 Ω

C4 C5 C6 20 Twenty identical bulbs are connected in series across a


220 V supply. Two of the bulbs are found to burn out.
B The remaining 18 bulbs are connected again in series
across the same supply. The illumination will be
What will be the value of equivalent capacitance
between A and B ? (a) the same in both the cases
(b) more with 20 bulbs than with 18 bulbs
(a) 6 C (b) C (c) more with 18 bulbs than with 20 bulbs
(c) 2 C (d) C/2
(d) unpredictable as the given data is not sufficient
DAY TWENTY ONE UNIT TEST 4 (ELECTROSTATICS AND CURRENT ELECTRICITY) 235

21 Five identical plates, each of area A are joined as shown 27 Assertion (A) If the distance between parallel plates of a
in figure. The distance between the plates is d. The capacitor is halved and dielectric constant becomes
plates are connected to a potential differenceV volt. The three times, then the capacitance becomes 6 times.
charge on plates 1 and 4 will be
Reason (R) Capacity of the capacitor does not depend
upon the nature of the material.

– 28 Assertion (A) When an insulated wire is bent, its


1234 5 V
+ resistivity increases.
Reason (R) On bending, the drift velocity of electron
decreases.
(a) q and q (b) q and + 2q
29 Assertion (A) An electric bulb becomes dim, when an
(c) q and − 2q (d) q and 3q
electric heater in parallel circuit is switched on.
22 In a potentiometer experiment the balancing with a cell
is at length 240 cm. On shunting the cell with a Reason (R) Dimness decreases after some time.
resistance of 2 Ω, the balancing length becomes 30 Assertion (A) The power dissipated in a conductor of
120 cm. The internal resistance of the cell is resistivity ρ is proportional to ρ −1.
(a) 4 Ω (b) 2 Ω
(c) 1 Ω (d) 0.5 Ω Reason (R) The expression for resistance of a conductor
ρl
23 Electric bulbs 50 W-100 V glowing at full power are to be is given by R = .
A
used in parallel with battery 120 V, 10 Ω. Maximum
number of bulbs that can be connected, so that they 31 Assertion (A) A thermoelectric refrigerator is based on
glow in full power is the Peltier effect.
(a) 2 (b) 8 Reason (R) A thermocouple may be used as a radiation
(c) 4 (d) 6 detector.
24 Masses of 3 wires of same metal are in the ratio 1 : 2 : 3
32 A spherical drop of capacitance 1µF is broken into eight
and their lengths are in the ratio 3 : 2 : 1. The electrical
drops of equal radius. Then, the capacitance of each
resistances are in ratio
small drop is
(a) 1 : 4 : 9 (b) 9 : 4 : 1
1 1 1
(c) 1 : 2 : 3 (d) 27 : 6 : 1 (a) µF (b) µF (c) µF (d) 8 µF
2 4 8
25 A uniform resistance wire of length L and diameter d has
33 A parallel plate capacitor filled with a material of
a resistance R. Another wire of same material has length
dielectric constant K is charged to a certain voltage. The
4L and diameter 2 d, the resistance will be
dielectric material is removed. Then,
(a) 2R (b) R
(c) R /2 (d) R /4 1. the capacitance decreases by a factor K
2. the electric field reduces by a factor K
Direction (Q. Nos. 26-31) In each of the following questions, 3. the voltage across the capacitor increases by a factor K
a statement of Assertion is given followed by a corresponding 4. the charge stored in the capacitor increases by a factor K
statement of Reason just below it. Of the statements, mark the (a) 1 and 2 are true (b) 1 and 3 are true
correct answer as (c) 2 and 3 are true (d) 2 and 4 are true
(a) If both Assertion and Reason are true and Reason is the
34 The capacity of a condenser is 4 × 10−6 F and its
correct explanation of Assertion
potential is 100 V. The energy released on discharging it
(b) If both Assertion and Reason are true but Reason is not
fully will be
the correct explanation of Assertion
(a) 0.04 J (b) 0.02 J (c) 0.025 J (d) 0.05 J
(c) If Assertion true but Reason is false
(d) If both Assertion and Reason are false 35 A parallel plate capacitor having a plate separation of
26 Assertion (A) A parallel plate capacitor is connected 2 mm is charged by connecting it to a 300 V supply. The
across battery through a key. A dielectric slab of energy density is
dielectric constant K is introduced between the plates. (a) 0.01 Jm−3 (b) 0.1 Jm−3 (c)1 .0 Jm−3 (d) 10 Jm−3
The energy which is stored becomes K times. 36 The capacity of an air condenser is 2.0 µF. If a medium is
Reason (R) The surface density of charge on the plate placed between its plates. The capacity becomes 12 µF.
remains constant or unchanged. The dielectric constant of the medium will be
(a) 5 (b) 4 (c) 3 (d) 6
236 40 DAYS ~ NEET PHYSICS DAY TWENTY ONE

37 The capacitors A and B have 10 V


42 In a practical Wheatstone bridge circuit, when one more
identical geometry. A material with resistance of100 Ω is connected in parallel with unknown
a dielectric constant 3 is present resistance x, then the ratio l1/l2 becomes 2. l1 is the balance
between the plates of B. The length. AB is a uniform wire. Then, the value of x must be
potential difference across A and B A B Copper strip Copper strip
are respectively
100 W x
(a) 2.5 V, 7.5 V (b) 2 V, 8 V (c) 8 V, 2 V (d) 7.5 V, 2.5 V
38 Between the plates of parallel plate capacitor of capacity G
C, two parallel plates of the same material and area A B
same as the plate of the original capacitor, are placed. If
the thickness of these plates is equal to 1/5th of the l1 l2
distance between the plates of the original capacitor,
then the capacity of the new capacitor is E r

(a)
5
C (b)
3
C (c)
3C
(d)
10 C (a) 50 Ω (b) 100 Ω (c) 200 Ω (d) 400 Ω
3 5 10 3 43 In the circuit given. the points A, B
−1
39 A wire has linear resistance ρ (in Ωm ). Find the B and C are 70V, zero and 10V 20 Ω
resistance R between points A and B, if the side of the respectively. Then, 10 Ω
big square is d A
(a) The point D will be at a D
A potential of 60V 30 Ω
(b) The point D will be at a
C
potential of 20V
(c) Currents in the paths AD, DB and DC are in the ratio of
1:2:3
(d) Current in the paths AD, DB and DC are in the ratio of
B 3:2:1
ρd
(a) (b) 2 ρd
2 44 For the circuit shown, a shorting wire of negligible
(c) 2ρd (d) None of these resistance is added to the circuit between points A and
40 The equivalent resistance between the points P and Q in B. When this shorting wire is added, bulb 3 goes out.
the network given here is equal to R. Which bulbs (all identical) in the circuit are brighten?
1
A
R R
R
3
P Q E 2 B
R R
4
R
R R

2R R (a) Only bulb 2 (b) Only bulb 4


(a) R (b) (c) 2R (d)
3 2 (c) Bulbs 1 and 4 (d) Bulbs 2 and 4
41 For the potentiometer arrangement as shown in the 45 A charged ball of mass 9 kg is
figure, length of wire AB is 100 cm and its resistance is suspended from a string in a
y
9 Ω. Find the length AC for which the galvanometer G will uniform electric field
θ =37°
show zero deflection. E = ( 3i + 5j) × 105 N/C. The ball is in x
E = 10 V equilibrium with θ = 37°. If direction
r=1W of electric field is reversed, find the
new equilibrium position of the ball.
Give your answer in terms of angle
made by string with vertical
C
A G B (Take, g = 10 ms −2 ).
(a) tan−1   (b) cot −1  
3 3
E=5V  4  14 
(c) cot −1   (d) tan−1  
r=1W 3 3
(a) 66.7 cm (b) 60 cm (c) 50 cm (d) 33.3 cm  4  14 
DAY TWENTY ONE UNIT TEST 4 (ELECTROSTATICS AND CURRENT ELECTRICITY) 237

46 Three infinite planes have a uniform surface charge 48 One-fourth of a sphere of radius R is removed as shown
distribution σ on its surface. All charges are fixed. On in figure. An electric field E exists parallel to the xy-plane.
each of the three infinite planes, parallel to the yz-plane Find the flux through the remaining curved part.
placed at x = − a, x = 0, and x = a, there is a uniform y
surface charge of the same density, σ. The potential
difference between A and C is E
y
A B C
+ +
45°
+
+ + + x
+ + +
+ + +
+ + + + + +
+ + +
+ + + + + +
x
+ + + + + + + + +
+ + +
+ + + + + σ+ +
+ σ +
+ +
σ + +

Z
2σ σ (a) πR 2E
(a) a (b) a
ε0 ε0 (b) 2 πR 2E
σa (c) πR 2E / 2
(c) (d) None of these
2 ε0 (d) None of the above
47 In the given arrangement of capacitors, 6 µF charge is 49 A hollow cylinder has a charge q coulomb within it. If φ
added to point a. Find the charge on upper capacitor be the electric flux in units of voltmeter associated with
the curved surface B, the flux linked with the plane
surface A in units of voltmeter will be
3 µF
B
A
2 µF 1 µF
C A

1 q  q φ q
(a) 3 µC (b) 1µC (a)  – φ (b) (c) (d) –φ
2  ε0  2 ε0 3 ε0
(c) 2 µC (d) 6 µC

ANSWERS
1. (b) 2. (a) 3. (c) 4. (c) 5. (d) 6. (b) 7. (c) 8. (d) 9. (a) 10. (a)
11. (c) 12. (b) 13. (d) 14. (c) 15. (c) 16. (b) 17. (b) 18. (a) 19. (b) 20. (c)
21. (c) 22. (b) 23. (c) 24. (d) 25. (b) 26. (c) 27. (b) 28. (d) 29. (b) 30. (a)
31. (b) 32. (a) 33. (b) 34. (b) 35. (b) 36. (d) 37. (d) 38. (a) 39. (a) 40. (b)
41. (a) 42. (b) 43. (d) 44. (c) 45. (d) 46. (d) 47. (a) 48. (c) 49. (a)
238 40 DAYS ~ NEET PHYSICS DAY TWENTY ONE

Hints and Explanations


1 Current through the branch CDA = 1 A 5 Dipole consists of equal and opposite q1 = 4 π r 2 σ 
⇒ and  …(i)
towards A. charges. Here, the net charge enclosed q2 = 4 π R 2 σ 
Applying KVL to the mesh ACDA , is zero.
Now, total charge on the spheres is
1 × 5− 1 × R = E + 1 6 The current density is a vector quantity. given by
5− 1 × 1 = E + 1 Its direction is given by the direction of Q = q1 + q2 = 4 π r 2 σ + 4 π R 2 σ
∴ E = 3V flow of positive charge in the circuit. [using Eq. (i)]
The same is possible due to electric or Q = 4 π (r 2 + R2 ) σ …(ii)
2 When two cells of emf’s E1 and E2 and field produced by charges accumulated Q
internal resistances r1 and r2 , on the surface of wire. ⇒ σ =
respectively are connected in parallel 4 π (r 2 + R2 )
and across an external resistance R as 7 R = ρl = ρl2 . Given, ρ A = 2ρB , while l Now, total potential due to both
shown in the figure, it can be proved
A πr
concentric spheres is given by
that and r have the same value.
1 q1 1 q2
∴ R A = 2R B , i.e. R A > R B V= +
E1, r1 4 πε 0 r 4 πε 0 R
V2 1  q1 q 
∴ P = ⇒ P A < PB or V = + 2
E2, r2 R 4 πε0  r R 
8 Magnitude of electric field is given by 1  4 πr σ2
4 πR 2 σ 
I or V =  +
E=−
dV 4 πε 0  r R 
R dr [using Eq. (i)]
Substituting, V = − 5x + 3 y + 15 z, we σ
E1 /r1 + E2 /r2 or V = (R + r )
I= get ε0
1 1
1 + R +  E x = − 5; E y = 3 ; E z = 15 1 Q (R + r )
 1 or V = [using Eq. (ii)]
r r2 4 πε 0 R 2 + r 2
Now, E = (E 2x + E 2y + E 2z )1 /2 = 7
Now, E1 = 3 V, r1 = 1 Ω, E2 = 2 V, kQ (R + r )
⇒ V= [using Eq. (ii)]
r2 = 2 Ω, R = 5 Ω 9 Here, radius of drops = 1 mm = 10−3 m R2 + r 2
Charge = 0.066 pC = 0.066 × 10−12 C 13 From principle of conservation of
3 2
+ energy, we have
1 2 4 8 Potential,
∴I= = = A
 1 1  1 + 15/ 2 17 1 q 0.066 × 10−12 Kinetic energy = Electrostatic potential
1 + 5 +  V= = 9 × 109 ×
 1 2 4 πε 0 r 10−3 energy, i.e.
Potential difference across 5 Ω resistor 1 KqQ
Potential of bigger drop = n2 /3 × mv 2 = …(i)
8 40 2 r
= × 5= = 2.4 V potential of small drops
17 17 When the speed of charge q is doubled,
0.066 × 10−12 1 KqQ
= (8)2 /3 × 9 × 109 × then we have m(2v )2 = …(ii)
3 P = 2.5 Ω , Q = 5 Ω , R = 10 Ω, 10−3 2 r′
S = 40 x / (40 + x ) = 2.4 V On dividing Eq. (i) by Eq. (ii), we get
P R
For balance, = 10 We know that, ∆W = 1 2
kq q 1 r′ r
Q S = ⇒ r′=
r 4 r 4
2.5 10 Here, q1 = 40 × 10−9C 14 C 4 is in parallel to a balanced
i.e. = (40 + x )
5 40 x 4 Wheatstone bridge made from the rest
Charge on sphere, q2 = πr 3ρ
1 40 + x 3 five capacitors as shown in the figure.
i.e. = , 2 x = 40 + x, x = 40 Ω
2 4x r = 2 cm, ρ = 10−4 C cm −3 Therefore, equivalent capacitance
=C + C = 2C
4 Electric field at point on the axis of Putting the values, we get
symmetry of a circular loop at a Work done = 14.4 π
C2 C5
distance x from the centre of loop is
11 Thickness = V = m = ZIt
kqx A dA dA
E= A B
(r 2 + x2 )3 /2 3.3 × 10−7 × 1.5 × 20 × 60 C3
=
Substituting, k = 9 × 109 N -m2C −2 ; 50 × 10−4 × 9000

q = 4 µC = 4 × 10−6C; = 1.3 × 10−5 m C1 C6


−2
r = 6 cm = 6 × 10 m 12 Since, the surface charge densities of
x = 8 cm = 8 × 10−2 m, we get both the concentric spheres are equal
−1 q1 q2
∴ E = 288 × 10 Vm 4 ⇒ σ = = C4
4πr 2 4 πR 2
DAY TWENTY ONE UNIT TEST 4 (ELECTROSTATICS AND CURRENT ELECTRICITY) 239

15 Current will flow only through the 20 The bulbs are in series. When two of the 24 Resistance of wire in terms of length
branch containing resistance R2 . bulbs burn out and the remaining 18 are and area of cross-section is given by
E 5 connected in series, the effective l
∴ I= = = 1A R=ρ
R2 + r 4+ 1 resistance will be reduced. As a result, a A
Potential difference across higher current flows through the mass
Since, volume = and
R2 = 1 × 4 = 4 V combination. Hence, the illumination density
If q be the charge on each plate of the will be more with 18 bulbs than with 20
Volume
capacitor, then bulbs. Area =
q q 2q Length
+ = 4 or =4 21 The equivalent circuit of given network
C C C l2
2q is given as, Thus, R=ρ d
or = 4 or q = 6 µC M
3 × 10−6 l12 l2 l2
1 + –– 2 ⇒ R1 : R2 : R3 = : 2 : 3
+
+– M1 M2 M3
16 In a potentiometer experiment, the emf
of a cell can be measured, if the 3+ – 2 Here, M1 : M2 : M3 = 1 : 2 : 3
potential drop along the potentiometer +–
and l1 : l2 : l3 = 3 : 2 : 1
wire is more than the emf of the cell to + – ⇒ R1 : R2 : R3 = 27 : 6 : 1
be determined. As values of emf’s of 3 + –– 4
two cells are approximately 5 V and 10 +
+– 25 Resistance of wire of length L and
V, therefore the potential drop along the diameter d is given by
5+ – 4
potentiometer wire must be more than +– L 4ρ L
10 V. Hence, option (b) is correct. R =ρ or R =
Each capacitor has a value π (d /2) 2 π d2
2 2
17 R = V = (115) Ω ε A
i.e. C= 0 Now, resistance when length is 4 L and
P 500 d
diameter 2d will be
Percentage drop in heat output when The equivalent capacitance of the ρ4L
voltage drops 110 V circuit is R′ =
2
π 
110 × 110 4C = 4ε0 A / d 2d 
500 − × 500 
115 × 115  2 
= × 100 = 8.5% Thus, the charge on the combination
500 ρ (4 L )
will be or R′ =
4q = 4ε0 AV /d π d2
18 Energy of a parallel plate capacitor is
4ρ L
given by In parallel connection total charge is ⇒ R′ = R =
1 1  ε A divided in the ratio of capacitances. π d2
U= CV 2 =  0  V 2
2 2 d  Therefore, charge on first plate is q and
26 If a dielectric slab of dielectric constant
Given that, d = x fourth plate is − 2q.
K is filled in between the plates of a
1  ε A l −l
Therefore, we have U =  0  V 2 22 Here, r = 1 2 × 2 Ω condenser while charging it, the
2 x  l2 potential difference between the plates
dU 1 1 dx does not change, but the capacity
Therefore, = ε0 AV 2 (−1) where, l1 = 240 cm, l2 = 120 cm
dt 2 x2 dt becomes K times. Therefore,
240 − 120 V ′ = V , C ′ = KC
dU 1 ε0 AV 2 So, r = ×2
or =− (v ) 120 ∴ Energy stored in the capacitor,
dt 2 x2
120 1 1
Potential energy decreases as or r= ×2=2Ω U ′ = C ′ V ′2 = (KC ) ( V 2 )
120 2 2
1 ε0 AV 2
=− (v )  1 2
2 x2 =  CV  K = KU
23 Potential drop across battery, V = 20 V 2 
dU
⇒ ∝ x −2 Thus, energy stored becomes K times.
dt Thus, current from the battery,
V 20 Surface charge density,
I= = =2A
19 Resistance of the bulb is R 10 q ′ C ′ V ′ KCV q
σ ′= = = =K =K σ
R1 = V 2 /P = 60 × 60/120 = 30 Ω A A A A
E = 120 V, r = 10 W
R1 = 30 Ω R=? 27 The capacitance of a capacitor,
Kε 0 A K
C = ∝
d d
I
L C1 K d K d /2 1
Hence, = 1 × 2 = 1 × =
50 W, 100 V C2 d1 K2 d 3K 6
120 V
Current through each lamp is given by or C2 = 6 C1
Current through the bulb is P 50
IL = = ⇒ I L = 0.5 A Q
I = P / V = 120 / 60 = 2 A V1 100 Again, capacity of a capacitor C =
V
120
I= = 2, R + R1 = 60 Since, total current from battery is 2 A
R + R1 Therefore, capacity of a capacitor does
and each lamp requires 0.5 A, therefore not depend upon the nature of the
R = 60 − R1 = 60 − 30 = 30 Ω maximum 4 lamps can be connected. material.
240 40 DAYS ~ NEET PHYSICS DAY TWENTY ONE

28 Resistivity or specific resistance is a will decrease by factor K and since, 1


×d =
Q 5
material property. So, it does not V = .
change on bending the insulated wire. C The capacitance of capacitor is given by
On bending, the cross-sectional area of Hence, if C decreases by factor K, then V ε A
C1 = 0 …(i)
wire changes but drift velocity of will increase by factor K. d −t
electron does not depend on area of
34 Energy Thickness of two conductor is given by
cross-section, so it does not change.
t = 2   =
1 1 d 2d
29 The electric power of a heater is more E = CV 2 = × 4 × 10−6 × (100)2  5 5
2 2
1
than that of a bulb. As P ∝ , the = 2 × 10−2 = 0.02 J On putting the values of t in Eq. (i), we
R get
resistance of heater is less than that of 35 The energy per unit volume or the ε0 A 5 ε0 A
the electric bulb. When a heater C1 = =
energy density is given by 2d 3 d
connected in parallel to the bulb is d −
1 5
switched on, it draws more current due U = ε0 E 2 …(i)
2 5  ε A
to its lesser resistance, consequently, = C where, C = 0
where, ε0 is permittivity of free space 3  d 
the current through the bulb decreases
and E is electric field.
and so it becomes dim. 39 The circuit is equivalent to
Also,
When the heater coil becomes Potential difference V A
E = = r r
sufficiently hot, its resistance becomes Distance between the plates d
more and then it draws a little lesser
…(ii) r r
current. Consequently, the current r √2
through the electric bulb recovers. From Eqs. (i) and (ii), we have
2 r √2
1 V 
Hence, dimness of the bulb decreases. U = ε0   r r
2 d 
30 From the formula, r r
Given, V = 300 V, d = 2 mm B
V2 = 2 × 10−3 m,
P= …(i) d
R Let each half side has resistance r = ρ .
ρl ε0 = 8.85 × 10−12 C2 N −1 m −2 2
But R= …(ii) 2 2r
A 1  300  r r
Now, from Eqs. (i) and (ii), we have U = × 8.85 × 10−12 ×  
−3
2  2 × 10 
V2 V2 A 1
P= = ⇒P ∝ . Jm −3
= 01 r √2
ρl ρl ρ A B
A 36 Dielectric constant of medium 2r
⇒ P ∝ ρ −1 Capacity of condenser with medium r r
K =
Capacity of condenser
31 A thermocouple, which is also called 12
thermoelectric detector can be used to = =6 r √2
detect heat radiations. It is also true that 2.0
On solving, we get
working of thermoelectric refrigerator is 37 Let q be charge in the series circuit and 1 (2r ) (r 2 ) 
R = 2r + =r 2
C be the capacitance across A and C ′ (2 + 2 ) r 
based on the Peltier effect.
2
32 Let R and r be the radii of bigger and across B. Then,
ε A Kε 0 A ρd
each smaller drop, respectively. C = 0 ,C′ = =
d d 2
4 4
∴ π R3 = 8 × π r 3
3 3 Also, q = CV 40 The given circuit can be simplified as
⇒ R = 2r …(i) Let V1 , V2 be potential across A and B,
The capacitance of a smaller spherical respectively. Then,
R R
drop is V = V1 + V2 = 10 V R
C = 4 π ε0 r q q q
…(ii) where, V1 = , V2 = = P Q
The capacitance of bigger drop is C KC 3C R R
R
C ′ = 4 π ε0 R = 2 × 4 π ε0 r ∴
q
+
q
= 10 R R
[Q R = 2 r ] C 3C
= 2C [from Eq. (ii)] q
1 +  = 10
1
C′ 1 R R
∴ C = = µF [QC ′ = 1µF] C  3 
2 2 q 4 R R
× = 10 P Q
33 As capacitance of capacitor after C 3 R R
introducing dielectric is given by q 30
⇒ = = 7.5 V
Kε 0 A C 4 1 1 1 1
C = ⇒ = + +
d and V2 = 10 − 7.5 = 2.5 V Req 2R 2R 2R
Here, if dielectric is introduced the
capacitance increases by K, but when 38 Thickness of each plates = 1 distance ⇒ Req =
2R
5 3
dielectric is removed, then capacitance between plates
DAY TWENTY ONE UNIT TEST 4 (ELECTROSTATICS AND CURRENT ELECTRICITY) 241

41 Potential difference across AC is zero. VD V – 10 T ′ sin α = 3q × 105


⇒ 70 – V D = + D
5 − 2I = 0 2 3 or T ′ cos α = mg + 5q × 105
or I = 2.5 A ⇒ V D = 40V
3q × 105
Let the resistance of part BC be r. V – VD
I1 = A =
70 – 40
= 3A tan α =
Applying KVL, we get 10 10 mg + 5q × 105
V – VB 40 – 0 3 × 10−4 × 105 3
E = 10 V I2 = D = = 2A = =
r=1Ω 20 20 9 × 10 + 5 × 10−4 × 105 14
V – VC 40 – 10
I3 = D = = 1A α = tan −1  
3
30 30 or
 14 
44 Initially, Req = 5R / 3. Finally, 46 Potential difference
C
G Req = 3R / 2 B
A B ∆V = − ∫A E ⋅ dl = zero
Equivalent resistance decreases, so
current increases in circuit and in 1 q1 q2 q3
E=5V 47 = = =K
also. Hence, brightness of 1 increases. It 1 2 3
r=1Ω
means potential difference across 1 q1 + q2 + q3 = 6
10 + 5 − 2I − Ir − I = 0 increases, so across 2 potential Solving, we get [Q K = 1]
or 2.5r = 7.5 or r = 3 Ω difference decreases, hence brightness or q3 = 3 µC
As resistance of part AB = 9 Ω, length of 2 decreases.
AC = 66 .7 cm Initially, potential difference across 4 is
48 φ plane + φ curve = 0 or φ plane = − φ curve
πR 2 $ πR 2 $
42 The Wheatstone bridge is in balanced 1  (2R / 3) E  E A1 = − i, A 2 = − i
V4 i = =
2  2R / 3 + R 
conditions, so 2 2
5
E = E cos 45° $i + E sin 45° $j
100 Ω (R / 2) E E
Finally, V 4 f = = E $ E $
= i + j
100 Ω R /2 + R 3 2 2
x
Since, V 4 f > V 4 i ; hence, brightness of 4 φ = E ⋅ (A 1 + A 2 )
increases. − E πR 2 E πR 2 − πR 2 E
= − =
45 2 2 2 2 2
T T cos θ 5 This is the flux entering. So, flux is
q (5 × 10 )
πR2 E
l1 l2 θ
2
100 x T sin θ θ θ (3 × 105) y
100 100 + x
=
l1 l2 E
mg
l1 A2
∴ = 2 or x = 100 Ω
l2 T sin θ = 3 q × 105 ...(i) 45°
x
43 At point D apply KCL, T cos θ = mg − 5q × 10 5
...(ii)
On solving, we get q = 100 µC, T = 50 N
0V
B After the reversal of direction of electric
field,
A1
I 20 Ω
70V 10 Ω I1 2
T ′ cos q
A D
I3 30 Ω α
q
α 49 φ total = φA + φB + φC =
ε0
10V C T ′ sin q As, φB = φ
3q (5 × 105)
I1 = I2 + I3 Let φA = φC = φ′, then

V A – VD V – VB
= D
V – VC
+ D 5q × 105 q 1q 
2φ′ + φ = ⇒ φ′ =  – φ
10 20 30 ε0 2  ε0 
DAY TWENTY TWO

Magnetic Effects
of Current
Learning & Revision for the Day
u Concept of Magnetic Field u Cyclotron
u Biot-Savart’s Law u Magnetic Force on a Current Carrying Conductor
u Ampere’s Circuital Law u Current loop as a Magnetic Dipole
u Force on a Moving Charge in Uniform u Torque
Magnetic Field and Electric Field u Moving Coil Galvanometer

Concept of Magnetic Field


A compass needle suffers a deflection when brought near a current carrying wire. This
means that electric current, i.e. electric charge in motion, gives rise to magnetism.

Oersted’s Experiment
A magnetic field is established around a current carrying conductor just as it occurs
around a magnet.
+– K +– K – + K

Conducting
wire
A B A B A S B
S N
N
PREP
No current, key is open When key is pressed,
needle suffers deflection
When key is pressed, it suffers
deflection in opposite direction
MIRROR
Your Personal Preparation Indicator

u No. of Questions in Exercises (x)—


Biot-Savart Law I u No. of Questions Attempted (y)—
u No. of Correct Questions (z)—
The magnetic field d B at a point P, due to a current element I dl
θ (Without referring Explanations)
is given by dl
µ I (d l × r)
dB = 0 u Accuracy Level (z / y × 100)—
4π r3 r
u Prep Level (z / x × 100)—
where, θ is the angle between d l and r. P
X In order to expect good rank in NEET, your
Direction of magnetic field produced due to a current carrying inside Accuracy Level should be above 85 & Prep
straight wire can be obtained by the right hand thumb rule. Illustration of the Level should be above 75.
Biot-Savart Law
DAY TWENTY TWO MAGNETIC EFFECTS OF CURRENT 243

For a wire of infinite length φ1 = φ2 = 90 ° and hence


Magnetic Field due to Circular l

µ I
B= 0
Current Loop 2 πr
l
If there is a circular coil of radius R and N number of turns, l
When the wire XY is of infinite length, but the point P lies
carrying a current I through the turns, then magnetic field near the end X or Y, then φ1 = 0 ° and φ2 = 90 ° and hence,
at the centre of coil is given by µ I
B= 0
µ NI 4πr
B= 0
2R l
When point P lies on axial position of current carrying
l
If there is a circular arc of wire R conductor, then magnetic field at P,
subtending an angle θ at the centre B = 0.
of arc, then the magnetic field at O θ I l
When wire is of infinite length, then magnetic field near
the centre point the end will be half, that of at the perpendicular bisector.
R
µ I  θ
B= 0   2. Magnetic Field due to a Thick (Cylindrical) Wire
2R  2π 
l
At a point O situated at a distance Magnetic field at a point
r from centre of a current outside the wire
carrying circular coil along its
I R µ I
B = 0 , where r is the B
axial line. O P 2 πr r P
I
The magnetic field is distance of given point
from centre of wire and I
µ 0 NIR2
B = r r > R.
2(R2 + r 2 )3 /2 R
Circular current l
Magnetic field at a point Thick cylindrical wire
If r >> R, then at a point along the carrying loop
inside the wire at a
µ NIR2 distance r from centre of wire (r < R) is
axial line, B = 0 3
2r µ I r
B= 0 ⋅ 2
2π R
Ampere’s Circuital Law l
Magnetic field inside a hollow current carrying
conductor is zero.
The line integral of the magnetic
in
field B around any closed path is
equal to µ 0 times the net current I
3. Magnetic Field due to a Solenoid
B
threading through the area dI
A current carrying
i2
enclosed by the closed path. solenoid behaves as a bar
Mathematically, ∫ B ⋅ dl = µ 0 ΣI i1
i3 magnet. The face, where
in current is flowing
Now, consider the diagram above. clockwise behaves as
Illustration of the
Here, ∑ I = i 1 + i2 − i3 Ampere’s circuital law South pole and the face,
Hence, ∫ B ⋅ dl = µ 0 ⋅ (i 1 + i2 − i3 ) where current is seen (a) (b)
flowing anti-clockwise,
behaves as North pole.
Applications of Ampere’s law For such a solenoid, the magnetic field inside it is uniform
and directed axially.
1. Magnetic field due to Straight Current l
For a solenoid coil of infinite length at a point on its
Carrying Wire axial line, the magnetic field, B = µ 0nI
X
The magnetic field due to a current where, n is number of turns per unit length.
carrying wire of finite length at a I
1
point P situated at a normal distance φ2
l
At the end of solenoid, B = µ 0nI
2
r is r φ1
P l
At the end field is half of at the centre, this is called
µ I end effect.
B = 0 (sin φ1 + sin φ2 )
4πr
Y
l
If point P lies symmetrically on the Straight current
4. Toroidal Solenoids
perpendicular bisector of wire XY, carrying wire For a toroid (i.e. a ring shaped closed solenoid) magnetic
then φ1 = φ2 = φ (say) and hence field at any point within the core of toroid B = µ 0nI ,
µ I
B = 0 ⋅ 2 sin φ = µ 0I sin φ where n =
N
, R = radius of toroid.
4πr 2 πr 2πR
244 40 DAYS ~ NEET PHYSICS DAY TWENTY TWO

If the direction of a v is parallel or anti-parallel to B, θ = 0


Force on a Moving Charge in
l

or θ = 180 ° and therefore F = 0. Hence, the trajectory of the


Uniform Magnetic Field and particle is a straight line.
Electric Field If the velocity of the charged particle is not perpendicular to
the field, we will break the velocity in parallel ( v|| ) and
l
If a charge q is moving with velocity v enters in a region in perpendicular (v⊥ ) components.
which electric field E and magnetic field B both are mv⊥
present, it experiences force due to both fields r=
qB
simultaneously. The force experienced by the charged
particle is given by the expression Pitch, p = (v||)T
F = q (v × B) + qE
Here, magnetic force Fm = q (v × B) = Bqv sin θ and electric
Cyclotron
force Fe = qE. It is a device used to accelerate positively charged particles,
l
The direction of magnetic force is same as v × B if charge is e.g. proton, deuteron, α-particle and other heavy ions to
positive and opposite to v × B, if charge q is negative. high energy of 100 MeV or more.
Bq
Cyclotron frequency, ν = .
Motion of a Charged Particle in a 2 πm
Uniform Magnetic Field Maximum energy gained by the charged particle
l
(i) If a charge particle enters a uniform magnetic field B q 2 B2  2
E max =  r
with a velocity v in a direction perpendicular to that of
 2m 
B (i.e.θ = 90 ° ), then the charged particle experiences a
force Fm = qvB. Under its influence, the particle where, r = maximum radius of the circular path followed by
describes a circular path, such that the positive ion.
mv
Radius of circular path, r = Maximum energy obtained by the particle is in the form of
qB kinetic energy.
In general,
mv p 2mK
r=
qB qB
= =
qB
Magnetic Force on a Current
2 mqV 1 2 mV Carrying Conductor
= =
qB B q If a current carrying conductor is placed in a magnetic field B,
then a small current element I dl experiences a force given by
where, p = mv = momentum of charged
particle, K = kinetic energy of charged particle dFm = Idl × B
and V = accelerating potential difference. and the total force experienced by whole current carrying
2πm conductor will be
(ii) The period of revolution of charged particle T = , Fm = ∫ dFm =
qB ∫ I(dl × B)
qB The direction of force can also be determined by applying
the frequency of revolution ν =
2 πm Fleming’s left hand rule or right hand thumb rule.
qB
or angular frequency ω = ⋅ Force between Two Parallel Current
m
Carrying Conductors
l
(i) If a charged particle is moving at an angle θ, to the
magnetic field (where θ, is other than 0°, 90° or 180°), it
l
Two parallel current 1 2 1 2
describes a helical path, where carrying conductors
radius of helical path, exert magnetic force on
one another. I2 I2
mv sin θ I1 I1
r= . l
Magnetic force
qB
experienced by length l
2πm
(ii) Revolution period, T = of any one conductor F12 F21 F12 F21
qB due to the other
qB
or Frequency, ν = current carrying r r
2 πm conductor is
(iii) Moreover, pitch (the linear distance travelled during µ 0 2I1 I2 (a) (b)
F = ⋅ l
one complete revolution) of helical path is given by 4π r
2 πmv cos θ F µ 2I I
p = v cos θ ⋅ T = Force per unit length, = F0 = 0 ⋅ 1 2
qB l 4π r
DAY TWENTY TWO MAGNETIC EFFECTS OF CURRENT 245

NOTE — If the conductors carries current in same direction, then


l
Conversion of Galvanometer into Ammeter An ammeter is
force between them will be attractive. made by connecting a low resistance S in parallel with a
— If the conductor carries current in opposite direction, then pivoted type moving coil galvanometer G. S is known as
force will be repulsive. shunt. Then, from circuit,
I g × G = (I − I g ) × S
 Ig 
Current Loop as ⇒ S =   G
I − I g 
a Magnetic Dipole
A current carrying loop (of any shape) behaves as a magnetic
dipole whose magnetic moment is given by Ig G
I I
M = IA
If we have a current carrying coil having N turns, then I – Ig
magnetic moment M of dipole will be
S
M = NIA
Magnetic moment of a current carrying coil is a vector and its Ammeter
direction is given by right hand thumb rule. So, S << G, only a small fraction of current goes through the
galvanometer.
Torque l
Conversion of Galvanometer into Voltmeter
A voltmeter is made by connecting a resistor of high
When a current carrying loop placed in a uniform magnetic
resistance R in series with a pivoted type moving coil
field, it experience torque,
galvanometer G.
τ = NIAB sin θ
V
where, NiA is defined as the magnitude of the dipole moment From the circuit, Ig =
of the coil G+ R
V
(pm ) ⋅ τ = pm B sin θ ⇒ R= −G
Ig
⇒ τ = pm × B
V
a b
Moving Coil
Galvanometer (MCG)
Ig
MCG is used to measure the current upto nanoampere. The
deflecting torque of MCG,
τdef = NBIA G R

A restoring torque is set up in the suspension fibre. If α is the Voltmeter


angle of trust, the restoring torque is l
Current Sensitivity The current sensitivity of a
τ restoring = KI galvanometer is defined as the deflection produced in the
where, K is galvanometer constant. galvanometer per unit current flowing through it.
α NBA
Sl = =
I C
Some Important Concepts Related to l
Voltage Sensitivity Voltage sensitivity of a galvanometer is
Moving Coil Galvanometer defined as the deflection produced in the galvanometer per
Some of the important concepts related to galvanometer, i.e. unit voltage applied to it.
current sensitivity, voltage sensitivity and some of α α S I NBA
SV = = = =
conversions used in galvanometer are given below. V IR R RC
246 40 DAYS ~ NEET PHYSICS DAY TWENTY TWO

DAY PRACTICE SESSION 1

FOUNDATION QUESTIONS EXERCISE


1 A coil of 100 turns is lying in a magnetic field of 1 T as 7 Biot-Savart law indicates that the moving electrons
shown in the figure. A current of 1A is flowing in this coil. (velocity v) produce a magnetic field B, such that
The torque acting on the coil will be (a) B ⊥ v (b) B || v
(c) it obey inverse cube law
(d) it is along the line joining the electron and point of
0.2m

N S observation

8 Magnetic field B on the axis of a circular coil far away at


0.1m distance x from the centre of the coil are related as
(a) 1 N-m (b) 2 N-m (a) B ∝ x −3 (b) B ∝ x −2
(c) 3 N-m (d) 4 N-m (c) B ∝ x −1 (d) None of these
2 A current loop in a magnetic field j NEET 2013 9 A current carrying coil is bent sharply, so as to convert it
(a) experiences a torque, whether the field is uniform or into a double loop both carrying current in the same
non-uniform in all orientations direction. If B be the initial magnetic field at the centre,
(b) can be in equilibrium in one orientation what will be the final concentric magnetic field ?
(c) can be equilibrium in two orientations, both the (a) 2B (b) 4B (c) 8B (d) Zero
equilibrium states are unstable
(d) can be in equilibrium in two orientations, one stable, 10 Two similar coils of radius R are lying concentrically with
while the other is unstable their planes at right angles to each other. The currents
flowing in them are I and 2I, respectively. The resultant
3 A beam of α-particles having specific charge
magnetic field induction at the centre will be
2.5 × 107 C kg −1 move with a speed of 2 × 10 5 ms −1 in a
j
CBSE AIPMT 2012
magnetic field of 0.05 T. What is the radius of the circular
5 µ 0I 3 µ 0I µ I µ I
path described by the α-particles? (a) (b) (c) 0 (d) 0
2R 2R 2R R
(a) 4 cm (b) 8 cm
(c) 16 cm (d) 32 cm 11 A current of 4 × 10−3 A is flowing in a long straight
4 Current of 5 A and 2 A are passed through two parallel conductor. The value of line integral of magnetic field
wires A and B respectively, in opposite directions. If the around the closed path enclosing the straight conductor
wire A is infinitely long and the length of the wire B is 5 m, will be
the force on the conductor B, which is situated at 20 cm (a) 1.6 π × 10−9 Wbm−2 (b) 1.6 × 10−9 Wbm−2
distance from A will be (c) 1.6 × 10−8 Wbm−2 (d) 1.6 π × 10−7 Wbm− 2
(a) 5 × 10−5 N (b) 2 × 10−5 N 12 The magnetic force acting on a charged particle of
(c) 5 π × 10−7 N (d) 2 π × 10−7 N charge − 2 µC in a magnetic field of 2 T acting in
5 A cyclotron is used to obtain a 2 MeV beam of protons. y-direction, when the particle velocity is
The alternating potential difference applied between the ( 2 $i + 3 $j ) × 106 ms −1 is j
CBSE AIPMT 2009
dees has a peak value of 20 kV and its frequency is (a) 8 N in z-direction (b) 4 N in z-direction
(c) 8 N in y-direction (d) 8 N in z-direction
5 MHz. The intensity of magnetic field to be applied for
resonance will be 13 An electron is projected with uniform velocity along the
(a) 0.0327 T (b) 0.327 T axis of a current carrying long solenoid. Which of the
(c) 3.27 T (d) None of these following is true?
(a) The electron will be accelerated along the axis
6 A current I ampere flows along an infinitely long straight
(b) The electron path will be circular about the axis
thin walled tube. The magnetic induction at any point
(c) The electron will experience a force at 45° to the axis
inside the tube at a distance r metre from axis, is
and hence execute a helical path
(a) 0 (b) ∞
µ I µ I (d) The electron will continue to move with uniform velocity
(c) 0 (d) 0 along the axis of the solenoid
2r 2 πr
DAY TWENTY TWO MAGNETIC EFFECTS OF CURRENT 247

µ 0 2I µ 0 2I  π
14 A uniform electric field and a uniform magnetic field are (a) [ 2 + π] (b) 2 + 
acting along the same direction in a certain region. If an 4π r 4 π r  4 
µ I µ I π
electron is projected in the region such that its velocity is (c) 0 [ 2 + π] (d) 0  2 + 
4π r 4 π r  4 
pointed along the direction of fields, then the electron
(a) speed will decrease j CBSE AIPMT 2011
21 The magnetic field due to a straight conductor of uniform
(b) speed will increase cross-section of radius a and carrying a steady current is
(c) will turn towards left of direction of motion
(d) will turn towards right of direction of motion
represented by

15 The radius of the earth is 6.4 × 10 6 m. The specific charge


of proton is 9.6 × 107 Ckg −1. What should be the value of
minimum magnetic field at the equator of earth, such that (a) B (b) B
a proton moving with velocity 1.92 × 107 ms −1 may
revolve round the earth ?
O a O a
(a) 1.57 × 10−8 T (b) 3.12 × 10−8 T r r
(c) 5.36 × 10−6 T (d) 7.83 × 10−6 T
16 When a charged particle moving with velocity v is
subjected to a magnetic field of induction B, the force on
it is non-zero. This implies that (c) B (d) B
(a) angle between v and B is necessarily 90°
(b) angle between v and B can have any value other than 90°
O a O a
(c) angle between v and B can have any value other than r r
zero and 180° z
22 A wire of arbitrary shape
(d) angle between v and B is either zero or 180° I = 2A
carries a current I = 2A,
17 A square wire of each side l carries a current I. What is consider the portion of wire
the magnetic field at the mid-point of the square ? between (0, 0, 0) and (4, 4,
(4, 4, 4)
I 4). A magnetic field given by
B = (1. 2 × 10−4 $i + 2 × 10−4 $j) y
exists in the space. The force x
O
I I acting on the given portion, is
(a) in calculatable as length of wire is not known
(b) F = [($i + $j + k$ ) × (12
. $i + 12
. $j)] N
−4 $
I (c) F = 8 × 10 [( i + j + k) × (12
$ $ . $i + 2 $j)] N
µ I µ I µ0 I µ I (d) zero
(a) 4 2 0 (b) 8 2 0 (c) 16 2 (d) 32 2 0
4π l 4π l 4π l 4π l 23 A current carrying closed loop in the form of a right
18 An electron moving in a circular orbit of radius r, makes n angled isosceles ∆ ABC is placed in a uniform magnetic
rotations per second. The magnetic field produced at the field acting along AB. If the magnetic force on the arm
centre has magnitude j
CBSE AIPMT 2015 BC is F, the force on the arm AC is j
CBSE AIPMT 2011
µ 0ne µ 0n 2e µ 0ne A
(a) (b) zero (c) (d)
2 πr r 2r
19 A regular hexagon carries current I. The hexagon can be
inscribed inside a circle of radius R. What is the magnetic
field at the centre of the circle ? B C
µ 2I µ 12I µ I µ 12I (a) −F (b) F (c) 2 F (d) − 2 F
(a) 0 (b) 0 (c) 0 3 (d) 0 3
4π R 4π R 4π R 4π R
24 Two charged particles traverse identical helical paths in
20 What is the magnetic field at the centre of arc in the figure a completely opposite sense in a uniform magnetic field
below? B = B0 k.
I
(a) They have equal z-components of momenta
I I (b) They must have equal charges
To ∞
(c) They necessarily represent a particle-anti-particle pair
From ∞
(d) The charge to mass ratio satisfy :   +   = 0
90° e e
r
 m 1  m  2
O
248 40 DAYS ~ NEET PHYSICS DAY TWENTY TWO

25 An irregular loop carrying current 5 A is placed in a 31 A long wire carrying a steady current is bent into a
uniform magnetic field B = 0.5 T, as such straight circular loop of one turn. The magnetic field at the centre
segment AB of length 10 cm is out of magnetic field. The of the loop is B. It is then bent into a circular coil of n
magnitude and the direction of the magnetic force acting turns. The magnetic field at the centre of this coil of n
on the loop are turns will be j NEET 2016

× × × × × × × × × (a) nB (b) n 2B
A B
× × × × × × × × × (c) 2nB (d) 2n 2B
× × × × × × C× × ×
32 A rectangular loop carrying a current I is situated near a
× × × × × × × × ×
long straight wire, such that the wire is parallel to one of
× × × × × × × × ×
the sides of the loop and is in the plane of the loop. If a
× × × × × × × × ×
B steady current I is established in the wire as shown in the
× × × × × × × × ×
figure, the loop will j WB JEE 2013

× × × × × × × × × I
(a) rotate about an axis parallel to the wire
(a) ON and unlike parallel to BC
(b) move away from the wire
(b) 0.25 N and unlike parallel to BC
(c) 0.25 N and like parallel to BC (c) move towards the wire I
(d) None of the above (d) remain stationary

26 An electron moves in a circular orbit with a uniform 33 An α-particle describes a circular path of radius r in a
speed v. It produces a magnetic field B at the centre of magnetic field B. What will be the radius of the circular
the circle. The radius of the circle is proportional to path described by the proton of same energy in the
B v v B same magnetic field ?
(a) (b) (c) (d) r
v B B v (a) (b) r (c) 2 (d) 2r
2
27 An electron is revolving around a proton in a circular
path of diameter 0.1 nm. It produces a magnetic field
34 Under the influence of a uniform magnetic field, a
charged particle is moving in a circle of radius R with
4 W m −2 on the proton. What is the angular speed of the
constant speed v. The time period of the motion
electron?
(a) depends on v are not on R
(a) 1.4 × 1016 rad s −1 (b) 2.2 × 106 rad s −1
(b) depends on both R and v
(c) 6.4 × 1016 rad s −1 (d) 8.8 × 1016 rad s −1
(c) is independent on both R and v
28 A beam of electrons passes undeflected through (d) depends on R and not on v
mutually perpendicular electric and magnetic fields. If 35 A 250 turn rectangular coil of length 2.1 cm and width
the electric field is switched OFF and the same magnetic 1.25 cm carries a current of 85 µA and subjected to a
field is maintained, the electrons move magnetic field of strength 0.85 T. Work done for rotating
(a) in an elliptical orbit (b) in a circular orbit the coil by 180° against the torque is j
NEET 2017
(c) along a parabolic path (d) along a straight line (a) 9.1 µJ (b) 4.55 µJ (c) 2.3 µJ (d) 1.5 µJ
29 A square conducting loop of side length L carries a 1
36 If a shunt of th of the coil resistance is applied to a
current I. The magnetic field at the centre of the loop is 10
(a) independent of L (b) proportional to L2 moving coil galvanometer, its sensitivity becomes
(c) inversely proportional to L (d) linearly proportional to L 1
(a) 10 fold (b) fold
30 Two identical long conducting wires AOB and COD are 10
1
placed at right angle to each other, with one above other (c) 11 fold (d) fold
such that O is their common point for the two. The wires 11
carry I1 and I 2 currents, respectively. Point P is lying at 37 A thin ring of radius R metre has charge q coulomb
distance d from O along a direction perpendicular to the uniformly spread on it. The ring rotates about its axis with
plane containing the wires. The magnetic field at the a constant frequency of f revolution/s. The value of
point P will be j
CBSE AIPMT 2014 magnetic induction in Wb m −2 at the centre of the ring is
µ0  I1  µ0 j
CBSE AIPMT 2010
(a)   (b) (I1 + I 2 )
2 πd  I2  2 πd µ qf µ q µ q µ qf
(a) 0 (b) 0 (c) 0 (d) 0
µ µ 2 πR 2 π fR 2f R 2R
(c) 0 (I12 − I 22 ) (d) 0 (I12 + I 22 )1/ 2
2 πd 2 πd
DAY TWENTY TWO MAGNETIC EFFECTS OF CURRENT 249

DAY PRACTICE SESSION 2

PROGRESSIVE QUESTIONS EXERCISE


1 Torques τ1 and τ 2 are required for magnetic needle to 5 The magnetic moment of the current carrying loop as
remain perpendicular to the magnetic field B1 and B2 at shown in figure, is equal to
B
two different places. The ratio 1 is O
B2
τ2 τ1 τ1 + τ 2 τ1 − τ 2 θ
(a) (b) (c) (d)
τ1 τ2 τ1 − τ 2 τ1 + τ 2 a

2 Infinite number of straight Q


P
wires, each carrying current
I are equally placed as b
a
shown in the figure.
30°
Adjacent wires have current P S R
30°
in opposite direction. Net 1
2
magnetic field at point P, is I (b 2 + 2ab)θ
µ I ln 2 $ 3 (a) (b) Iab θ
(a) 0 k 4 2
8 π 3a I (a 2 + ba) θ
µ I ln 4 $ (c) (d) None of these
(b) 0 k 2
4 π 3a
6 A coil in the shape of an equilateral triangle of side l is
µ I ln 4
(c) 0 (−k$ ) suspended between the pole pieces of a permanent
4 π 3a
magnet, such that B is in plane of the coil. If due to a
(d) zero current I in the triangle, a torque τ acts on it. The side l of
3 A charged particle q enters a region of uniform magnetic the triangle is
1/ 2 1/ 2
τ   τ τ 1 τ
(a) 2  (c) 2  
field B (out of page) and is deflected a distance d after 2
 (b)   (d)
travelling a horizontal distance a. The magnitude of the  3BI  3  BI   BI  3 BI
momentum of the particle is
B
7 A wire carrying current I has the shape as shown in
adjoining figure. Linear parts of the wire are very long
d
and parallel to X-axis, while semi-circular portion of
radius R is lying in YZ -plane. Magnetic field at point O is
j
CBSE AIPMT 2015
a
Z
qB  a 2  qB
(a)  + d  (b) (c) zero (d) 2qB I
2 d  2
R
4 A particle of mass 1.6 × 10 −27 kg and charge O
Y
1.6 × 10 −19 C enters a uniform magnetic field of 1 T as I I
shown in the figure. The speed of the particle is 107 ms −1. X
The distance PQ will be
µ0 I µ0 I
(a) B = (π $i + 2k$ ) (b) B = − (π $i − 2k$ )
× × × × × × ×
θ
× × × × × × × 4π R 4π R
Q µ I µ I
× × × × × × × (c) B = − 0 (π $i + 2k$ ) (d) B = 0 (π $i − 2k$ )
4π R 4π R
× × B× × × × ×
× × × × × × × 8 When a proton is released from rest in a room, it starts
P × × × × × × ×
with an initial acceleration a 0 towards West. When it is
projected towards North with a speed v 0, it moves with
× × × × × × ×
45° an initial acceleration 3a 0 towards West. The electric and
× × × × × × ×
magnetic fields in the room are j
NEET 2013
(a) 0.141 m (b) 0.28 m (c) 0.4 m (d) 0.5 m
250 40 DAYS ~ NEET PHYSICS DAY TWENTY TWO

ma0 2ma0 ma0 2ma0 13 An electron is moving in a circular path under the influence
(a) West, up (b) West, down
e ev 0 e ev 0 of a transverse magnetic field of 3. 57 × 10−2 T. If the value
(c)
ma0
East,
3ma0
up (d)
ma0
East,
3ma0
down . × 1011 C/kg, the frequency of revolution of the
of e/m is176
e ev 0 e ev 0 electron is j NEET 2016

9 Two circular coils 1 and 2 are made from the same wire, (a) 1 GHz (b) 100 MHz (c) 62.8 MHz (d) 6.28 MHz
but the radius of the 1st coil is twice that of the 2nd coil. 14 An alternating electric field of frequency ν, is applied
What is the ratio of potential difference applied across across the dees (radius = R) of a cyclotron, i.e. being
them, so that the magnetic field at their centres is the used to accelerate protons (mass = m). The operating
same? magnetic field (B) used in the cyclotron and the kinetic
(a) 3 (b) 4 (c) 6 (d) 2 energy (K ) of the proton beam, produced by it, are given
10 Current sensitivity of a moving coil galvanometer is by j CBSE AIPMT 2012


5 div/mA and its voltage sensitivity (angular deflection (a) B = and K = 2mπ 2 ν2R 2
e
per unit voltage applied) is 20 div/V. The resistance of 2 π mν
the galvanometer is j NEET 2018 (b) B = and K = m 2 πνR 2
e
(a) 250 Ω (b) 25 Ω 2 π mν
(c) B = and K = 2mπ 2 ν2R 2
(c) 40 Ω (d) 500 Ω e

(d) B = and K = m 2 πνR 2
11 An arrangement of three parallel d C e
B
straight wires placed perpendicular 15 A beam of cathode rays is subjected to crossed electric (E)
90°
to plane of paper carrying same and magnetic fields (B). The fields are adjusted such that
current I along the same direction is d the beam is not deflected. The specific charge of the
shown in figure. Magnitude of force cathode rays is given by
per unit length on the middle wire B j
CBSE AIPMT 2010
A
is given by j
NEET 2017 B2 2VB 2 2VE 2 E2
µ0I2 2µ 0 I 2 (a) (b) (c) (d)
(a) (b) 2VE 2 E2 B2 2VB 2
2 πd πd
16 A closed loop PQRS carrying a Q
2µ 0 I 2 µ0I2
(c) (d)
πd 2 πd current is placed in a uniform
magnetic field. If the magnetic P
12 A long straight wire of radius a carries a steady current I. forces on segments PS , SR and F3
The current is uniformly distributed over its cross-section. RQ are F1, F2 and F3 respectively F1
The ratio of the magnetic fields B and B′ at radial and are in the plane of the paper
a
distances and 2 a respectively, from the axis of the and along the directions shown S R
2 in figure, the force on the F2
wire is j
NEET 2016 segment QP is
1
(a) (b) 1 (a) F3 − F1 − F2 (b) (F3 − F1)2 + F22
2
(c) 4 (d)
1 (c) (F3 − F1)2 − F22 (d) F3 − F1 + F2
4

ANSWERS
SESSION 1 1 (b) 2 (d) 3 (c) 4 (a) 5 (b) 6 (a) 7 (a) 8 (a) 9 (b) 10 (a)
11 (a) 12 (a) 13 (d) 14 (a) 15 (b) 16 (c) 17 (b) 18 (d) 19 (d) 20 (b)
21 (a) 22 (c) 23 (a) 24 (d) 25 (b) 26 (c) 27 (b) 28 (b) 29 (c) 30 (d)
31 (b) 32 (c) 33 (a) 34 (c) 35 (a) 36 (d) 37 (d)
SESSION 2 1 (b) 2 (a) 3 (a) 4 (a) 5 (a) 6 (a) 7 (c) 8 (b) 9 (b) 10 (a)
11 (d) 12 (b) 13 (a) 14 (c) 15 (d) 16 (b)
DAY TWENTY TWO MAGNETIC EFFECTS OF CURRENT 251

Hints and Explanations


1 ∴Torque, τ = NBIA 10 The magnetic field (B ) at the centre of Since, force on charged particle is
non-zero, so angle between v and B can
= 100 × 1 × 1 × 0.2 × 0.1 = 2 N -m circular current carrying coil of radius R
µ I have any value other than zero and 180°.
2 For parallel magnetic field is stable and and current I is B = 0
2R 17 ∴ Magnetic field,
for anti-parallel is unstable.
Similarly, if current is 2 I, then µ I 
B = 4  0 (sin φ1 + sin φ2 )
3 ∴Radius of circular path, µ 2I  4π a 
mv v 1 Magnetic field = 0 = 2B
R= = 2R µ0 I  1 1 
Bq 0 B q0 /M = 4× +
So, resultant magnetic field 4 π (l / 2)  2 2 
= 2 × 10 / 0.05 × 2.5 × 10
5 7
µ0 I [Here, φ1 = φ2 = 45°]
= 0.16 m = 16 cm = B 2 + (2 B )2 = 5B 2 = 5B = 5
2R µ I 
µ 2I I =8 2 0 
4 Force, F = 0 1 2 l 11 ∴Ampere’s law, ∫ B ⋅ dl = µ 0 I 4 π l 
4π r
18 Current, I = e
−7
Here, I1 = 5 A, I2 = 2A,l = 5m, or ∫ B ⋅ dl = 4 π × 10 × 4 × 10−3
r = 20 cm = 0.2 m T r
= 1.6 π × 10−9 Wbm −2 e ωe e
Hence, I = =
2 × 5× 2 × 5 (2 π / ω ) 2 π
F = 10−7 × = 5 × 10−5 N 12 Magnetic Lorentz force, F = q( v × B ) B
(2 πn ) e
0.2
= −2 × 10−6 {(2$i + 3$j) × 106 × 2$j} = = ne I
Bq 2π
5 Frequency, n = = − 2 × 10−6 [2 × 2 × 106 k$ ] Magnetic field,
2πm µ I µ ne
2πn 2 × 3.14 × 5 × 106 = 8 N along negative z-direction B = 0 = 0
or B = = 2r 2r
q 96 × 106 13 There is a uniform magnetic field B
  19 Each arm acts as a chord. Its distance
 m inside the current carrying long π R
= 0.327 Wbm −2
= 0.327 T solenoid acting along the axis of from the centre is d = R cos = ⋅
3 2
6 Inside the tube, the magnetic field is solenoid. The magnitude of force on the Magnetic field due to each chord
electron of charge (− e ) moving with µ I  π π
+ sin 
zero.
= 0 sin
7 According to Biot Savart’s law, the velocity v in a magnetic field B is 4 π d  3 3 
magnetic field B at a point distance r |F |= − e |v × B| = − e v B sinθ µ 2I
= 0 × 3
from a charge q moving with a velocity Here, angle θ between v and B is zero, 4π R
v is given by i.e. θ = 0° and sin θ = 0.  µ 2I 
µ q (v × r ) µ v sin θ Total magnetic field = 6  0 3
B= 0 or B = 0 . Therefore, F = 0.  4 π R 
4π r3 4π r2
µ 0 12I
The direct B is along (v × r ), i.e. It means, the electron will continue to = 3
move with uniform velocity along the 4π R
perpendicular to the plane containing v
and r. B at a point obeys inverse square axis of the solenoid. 20 Distance of straight part from
law and not inverse cube law. r
14 Magnetic field (B ) will not apply any O =l= ⋅
8 Magnetic field on the axis of circular 2
force. Only electric field E will apply a
µ 2 πIR2 force opposite to velocity of the Hence,
coil is = 0 B = B1 + B2
4 π (R + x2 ) 3 /2
2 electron, hence speed decreases.
µ 2I µ 2 πI  1 
2 = 0 + 0  
where, R is the radius of the coil and x
15 ∴Force, F = qvB = mv 4π r/ 2 4 π r  4
is the distance of the observation point. R
µ 0 2I  π
At the far away point x >> R, so R2 may 1.92 × 107 = 2+ 
4 π r  4 
v
2 or B = =
be neglected as compared to x . Hence, q 9.6 × 107 × 6.4 × 106
magnetic field is related to x as,  R
 m 21 The magnetic field at a point outside
B ∝ x −3 . −8 the straight conductor is given by
= 3.12 × 10 T µ I
9 Radius of the double loop r = R , where B = 0
2 16 When a charged particle q is moving in 2 πr
R is the radius of single loop B (centre) a uniform magnetic field B with velocity 1
µ 2πI v such that angle between v and B be θ, It means B ∝ [if, r > a] [hyperbola]
= 0 ⋅ r
4π R the charge q experiences a force which The magnetic field at a point inside the
is given by conductor is
So, magnetic field due to each coil is
F = qvB sinθ µ I ×r
double of that due to single coil. And B = 0
total magnetic field is 2B + 2B = 4B . If θ = 0° or 180°, then sin θ = 0 2 πa2
∴ F = qvB sin θ = 0 or B ∝ r [if, r < a] [straight line]
252 40 DAYS ~ NEET PHYSICS DAY TWENTY TWO

22 The magnetic force in a uniform field is 28 If both electric and magnetic fields are 32 Wires placed close to each other carry
F = I ( l × B) present and perpendicular to each current in the same direction and hence
I = 4 i$ + 4 $j + 4 k$ other; and the particle is moving attract. Wires placed far apart carry
perpendicular to both of them with current in opposite direction and hence
B = (1.2 $i + 2 $j ) × 10−4 T repel each other. But here attraction is
Fe = F m . In this situation, E ≠ 0 and
F = 8 × 10−4 [( i$ + $j + k$ ) B ≠ 0. strong and repulsion is weak. So, loop
× (1 .2 i$ + 2 $j )] N Fe moves towards the wire.

23 Force on AB is given by 33 Centripetal force = Magnetic force


i.e.
FAB = 0
mv 2 mv 2mE
According to the question, E q v = q 0 vB , hence r = =
FAB = 0 r q0B q0 B
FAB + FBC + FCA = 0 where, E is the energy of the particle.
FBC + FCA = 0 Mass of α-particle is 4 times that of
FCA = − FBC = − F proton and charge is 2 times that of
Fm
proton.
24 In a uniform magnetic field, the two But, if electric field becomes zero, then
charged particles will traverse identical only force due to magnetic field exists. 34 When magnetic field is perpendicular to
helical paths in a completely opposite Under this force, the charge moves motion of charged particle, then
sense, if the charge/mass ratio of these along a circle. centripetal force = magnetic force
two particles is same and charges on them mv 2 mv
29 Magnetic field at the centre due to i.e. = Bqv or R =
are of opposite character. In this situation, R Bq
(e / m )1 + (e / m )2 = 0, holds good. either arm,
µ I Further, time period of the motion,
B1 = 0 × [sin 45° + sin 45° ]  mv 
25 If the loop is placed in a uniform 4π  L  2π 
  
magnetic field, then net magnetic force  2 2 πR  Bq 
on the loop is zero. T = =
µ 0 2 2I v v
∴ Magnetic force = Magnetic force due = × 2 πm
4π L or T =
to loop – Bq
Field at centre due to the four arms of
Magnetic force on the line segment So, the time period of the motion is
the square,
= 0 − IlB = − 5 × 0.5 × 10 × 10−2 independent on both R and v.
µ 2 2I
= − 0.25 N B = 4B1 = 0 ×
π L 35 Work done for rotating the coil,
Since, magnetic force on the line 1
i.e. B ∝ . W = MB (cos θ1 − cos θ2 )
segment is parallel to BC. So, magnetic L where, M = magnetic moment
force on the curve section is unlike
parallel to BC. 30 Magnetic field, and B = magnetic field.
Given, θ1 = 0° , θ2 = 180°
26 The time period of electron moving in a B = B12 + B22
∴ W = MB (cos 0° − cos 180° )
circular orbit, µ0 2 = 2MB = 2 × NIA × B
= (I1 + I22 )1 /2
Circumference of circular path 2 πd = 2 × 250 × 85 × 10−6
T =
Speed × (2.1 × 1.25 × 10−4 ) × 0.85
=
2πr I1 = 9.48 × 10−6 J
B2
v −6
d ≈ 9.5 × 10 J = 9.5 µ J
and equivalent current due to electron I2 B1
flow, Q P The closest option is (a).
e e ev 36 Sensitivity of a galvanometer is directly
I = = =
T (2 πr / v ) 2 πr proportional to the current through it.
n⋅µ 0 i When a shunt resistance, S =   is
Magnetic field at centre of circle, G
µ I µ ev 31 Bcentre = (for a circular coil)
B = 0 = 0 2R  10 
2r 4 πr 2 applied to galvanometer of resistance G,
v then current through galvanometer I g
⇒ r ∝ R
B can be written as
i (I − I g ) G
27 Magnetic field, I g G = (I − I g )S =
10
µ 0 2π µ e 1
B = I = 0 ×2 π × where, n = number of turns in circular (I − I g )
4π R 4π t R coil or Ig =
µ 10
= 0 ×2 π ×
e
×
1 µ i µ πi
B = 0 = 0
4π 2π R / v R 2R l or 10 I g = I − I g
µ ev µ eω µ 0 ni n2µ 0 πi I
= 0 × 2π × = 0 = = = n2 B 11 I g = I , I g =
4π 2π R 2 4 π R2  l  l 11
2 
∴ ω = 2.2 × 106 rads −1  2nπ 
DAY TWENTY TWO MAGNETIC EFFECTS OF CURRENT 253

37 The magnetic field at the centre of the a2 d Net magnetic field,


⇒ R= +
circle is 2d 2 B = B I + B II + B III
µ 0 2 πI ∴ Momentum, µ I
B = × …(i) B = 0 (−2k$ − π$i )
4π R qB  a2  4Rπ
p = mv = qBR =  + d 
q = It µ I
Since, 2 d  B = − 0 (2k$ + π$i )
On multiplying and dividing by t, we 4 πR
get 4 Magnetic force = Centripetal force,
µ 2 πIt µ 2π × q 8 Initial acceleration, a0 = eE …(i)
B = 0 × ⇒B = 0 × mv 2 mv m
4π Rt 4π Rt i.e. qvB = or r =
r qB a m
1 ∴ Electric field, E = 0 West
Also, f = frequency = × × × ×
e
t 45° ev 0 B + eE
µ qf × × × × ∴ = 3 a0
∴ B = 0 ⋅ Q m
2 R × × × × or ev 0 B + eE = 3 a0 m
r
45° × ×B× × ∴ ev 0 B = 3 ma0 − eE
SESSION 2 Q
⇒ ev 0 B = 3 ma0 − ma0 [from Eq. (i)]
r 45° × × × ×
1 Torque, τ = MB sin θ P ⇒ ev 0 B = 2 ma0
P × × × × 2ma0
∴ τ1 = MB1 sin 90° = MB1 ∴Magnetic field, B = down
and τ2 = MB2 sin 90° = MB2 45° × × × × ev 0
MB1 τ × × × × 9 Magnetic field at the centre of circular
or = 1
MB2 τ2 mv coil is
PQ = 2r cos 45° = 2 × r = 2
τ1 qB µ 0 2 πI

B1
= B = ×
τ2 4π r
B2 1.41 × 1.6 × 10−27 × 107
= = 0.141 m where, I is current flowing in the coil
2 Magnetic field due to straight wire is 1.6 × 10−19 × 1
and r is radius of coil.
µ 0I At the centre of coil1,
B = [sin θ1 + sin θ2 ] 5 Magnetic moment, M = IA = I( A 2 − A1 )
4 πd µ 2 π I1
 (a + b )2 a2  B1 = 0 × …(i)
⇒ M =I  θ− θ 4π r1
θ1 = 30°  2 2 
At the centre of coil 2,
θ1  (b 2 + 2ab )  µ 2 π I2
= I θ B2 = 0 × …(ii)
d θ2 2 4π r2
 
θ2 = 30° But B1 = B2
6 Torque acting on equilateral triangle in µ 0 2 π I1 µ 0 2 π I2 I I
a magnetic field B is τ = IAB sin θ ∴ = or 1 = 2
∴ B = B1 + B2 + K 4 π r1 4 π r2 r1 r2
B = B1 k$ + B2 (− k$ ) + B3 k$ + .. 3 2 As, r1 = 2r2
Area of ∆ LMN, A = l and θ = 90°
B = B1 − B2 + B3 − B4 + K 4 I1 I
∴ = 2 or I1 = 2I2 …(iii)
µ 0I  Substituting the given values in the 2r2 r2
1 − + − + ...
1 1 1
B= expression for torque, we have
16 πd  2 3 4  Now, ratio of potential differences,
L V2 I × r2 I × r2 1
cos 30° =
d
⇒ d = a cos 30° =
3
a = 2 = 2 =
a 2 V1 I1 × r1 2I2 × 2r2 4
S N
µ 0I × 2 µ I ln 2 $ B V1 4
B= ln 2k$ ⇒ B = 0 k l ∴ =
16 π 3 a 8 π 3a 90°
V2 1

3 Hypotenuse, AB = a2 + d 2 = h 10 Current sensitivity of a moving coil


M O N galvanometer is the deflection (θ) per unit
h
In ∆OAD, cos θ = Normal current (I ) flowing through it, i.e.
2R θ NAB
IS = = …(i)
π
ln ∆ACB , sin  − θ = ⇒
d h d 3 2
= τ =I × l B sin 90° I k
2  h 2R h 4 where,
⇒ h2 = 2Rd ⇒ a2 + d 2 = 2Rd 3 2 N = number of turns in the coil,
= Il B [Qsin 90° = 1]
4 A = area of each turn of coil,
R B
O 1 /2 B = magnetic field
 τ 
Hence, l=2  and k = restoring torque per unit
 3 BI  twist of the fibre strip.
µ0 I Similarly, voltage sensitivity is the
R d 7 ∴B I = (− k$ ), deflection per unit voltage, i.e.
D 4π R
µ I θ  NAB  I NAB
VS = =  = …(ii)
θ B III = 0 (− k$ ), V  k  V kRG
π –θ 4π R
2 µ I where, RG is the resistance of the
A a C B II = 0 (− i$ ) galvanometer.
4R
254 40 DAYS ~ NEET PHYSICS DAY TWENTY TWO

From Eqs. (i) and (ii), we get For the smaller loop, m(2πRν)
∴ Radius, R =
I 2 eB
RG = S × π  
…(iii) a I a
VS B × 2π = µ 0 × 2πmν
2 πa2  2 Magnetic field, B =
Here, IS = 5div/mA = 5 × 10−3 div/A 1 µ 0I e
= µ 0I × = 1
and VS = 20 div/V 4 4 Kinetic energy, K = m(2 πRν)2
µ I a 2
Substituting the given values in Eq. (iii), ⇒ B I = 0 , at distance from the
5 × 103 4 πa 2 = 2 mπ2 ν2 R2
we get RG = = 250
20 axis of the wire.
15 As the electron beam is not deflected,
Similarly, for bigger amperian loop,
∴ The resistance of the galvanometer is then
B ′ × 2 π (2a) = µ 0 I [total current
250 Ω. F m = Fe or Bev = Ee
enclosed by Amperian loop is 2]
11 As force on wire B due to A and C are µ I E
⇒ B′ = 0 , or v = …(i)
attractive, so we have following 4 πa B
condition At distance 2a from the axis of the wire. As the electron moves from cathode to
F B µ I 4 πa anode, its potential energy at the cathode
B So, ratio of, = 0 × =1
B ′ 4 πa µ 0 I appears as its kinetic energy at the
anode. If V is the potential difference
13 As we know that, radius of a charged between the anode and cathode, then
particle in a magnetic field B is given by potential energy of the electron at
mv cathode = eV . Also, kinetic energy of the
r = K (i)
qB 1
electron at anode = mv 2 .
F Resultant where, r = charge on the particle 2
µ I2 and v = speed of the particle. According to law of conservation of
F = 0 energy,
2 πd ∴The time taken to complete the circle,
1
Resultant force on T =
2πr mv 2 = eV
v 2
B= F12 + F22 2eV

T
=
m
[from Eq. (i)] or v = …(ii)
µ 0 I2 2π qB m
= 2F = 2×
2 πd 2 π qB From Eqs. (i) and (ii), we have
∴ ω= = 2eV E
µ 0 I2 T m =
=
2 πd e m B
Q q = e and = 1.76 × 1011 C/kg
m e E2
12 Consider two amperian loops of radius or =
a B = 3.57 × 10−2 T m 2VB 2
and 2a as shown in the figure.
2 16 As the net force on closed loop is equal

2π eB
= f =
1 e Q 1 = f  to zero. So, force on QP will be equal
B  
T m 2π m  T  and opposite to sum of forces on other
1 −2 three sides.
a/2 = × 1.76 × 10 × 3.57 × 10
11
2π F1 F3
= 1.0 × 109 Hz = 1 GHz
2a
14 Frequency, ν = eB
2 πm
1 mν
KE = mν2 and radius R =
2 eB F2
Applying Ampere’s circuital law for
πR 2 πR So, from vector laws,
these loops, we get Here, velocity, v = = = 2 πRν
∫ B ⋅ dL = µ 0 Ienclosed T /2 T FQP = (F3 − F1 )2 + F22
DAY TWENTY THREE

Magnetism
Learning & Revision for the Day

u Magnetic Dipole Moment u Magnetisation of Materials u Electromagnet


u Magnetic Field Lines u Magnetic Materials u Permanent Magnet
u The Earth’s Magnetism u Hysteresis Curve

A naturally occurring ore of iron, magnet attracts small piece of iron towards it. The
phenomenon of attraction of small bits of iron, steel, cobalt, nickel, etc., towards the ore
is called magnetism.

Magnetic Dipole Moment


The simplest magnetic structure is the magnetic dipole characterised by a magnetic
dipole moment M .
A bar magnet exhibits two important properties, namely
(i) the attractive property (ii) the directive property
If m is the pole strength and 2l is the magnetic length of the bar magnet, then its
magnetic moment is
M = m(2l)
l
Magnetic moment is a vector whose direction is from S-pole towards N-pole.
l
The magnetic moment produced due to motion of electron,

M = iA = −
evr
2
 eh 
= − n 
 2πm 
 nh 
Q mvr = 2π 
PREP
MIRROR
Your Personal Preparation Indicator
Magnetic Field due to a Bar Magnet u No. of Questions in Exercises (x)—
The magnetic field in free space, at a point having distance r from the given bar magnet u No. of Questions Attempted (y)—
(or magnetic dipole) is calculated in two conditions, along axial line and along u No. of Correct Questions (z)—
equatorial line. (Without referring Explanations)
µ 2 Mr
l
Along axial line B = 0 2
4π (r − l 2 )2 u Accuracy Level (z / y × 100)—
u Prep Level (z / x × 100)—
and the direction of B is the same as the direction of M. For a short dipole (or for a far
away point on the axis) when r > > l , the above relation is simplified as In order to expect good rank in NEET, your
µ 2M Accuracy Level should be above 85 & Prep
B= 0 3 Level should be above 75.
4π r
256 40 DAYS ~ NEET PHYSICS DAY TWENTY THREE

l
Along the equatorial line l
Potential energy of a magnetic dipole placed in a uniform
µ M magnetic field, is given by UB = − M ⋅ B = − MB cos θ
B= 0 2
4π (r + l 2 )3 / 2 l
The magnetic compass (needle) of magnetic moment M
µ M and moment of inertia I oscillate in the magnetic field B.
If r > > l , the relation is modified as, B = 0 3 Then, its time-period is
4π r
T = 2 π I / MB
However, along the equatorial line, the direction of B is
opposite to that of M. l
Behaviour of a magnetic dipole in a magnetic field, is
In general, in a direction making an angle θ with the similar to the behaviour of an electric dipole in an electric
magnetic axis, the magnetic field is given by 1 µ
field. However, the constant is replaced by 0 .
µ M 4πε 0 4π
B = 0 3 (3 cos2 θ + 1)
4π r l
If a magnetic dipole is in the form of a wire or a thin rod,
In these relations, µ 0 is a constant having a value of when bent, its magnetic dipole moment M changes because
4π × 10 −7 T mA −1 and it is known as the magnetic the separation between its poles has changed.
permeability of free space.
For solenoid B = µ 0ni The Earth’s Magnetism
where, n is number of turns per unit length of solenoid and i The earth is a natural source of magnetic field, a magnetic
the current through it. field is always present everywhere near the surface of the
earth.
A freely suspended magnet always points in the north-south
Magnetic Field Lines direction even in the absence of any other magnet. This
The magnetic field lines is defined as the path along which suggests that the earth itself behaves as a magnet which
the compass needles are aligned. They are used to represent causes a freely suspended magnet (or magnetic needle) to
magnetic field in a region. point always in a particular direction : north and south. The
l
Magnetic field lines are closed continuous curves. shape of earth’s magnetic field resembles that of a bar magnet
of length one-fifth of earth’s diameter buried at its centre.
l
Tangent drawn at any point on magnetic field lines gives
the direction of magnetic field at that point. Magnetic field of earth is shown in the figure given below.
l
Two magnetic field lines cannot intersect each other. Geographical L
North
l
Outside a magnet, they are directed from north to south Magnetic P α O
pole and inside a magnet they are directed from south to North H δ
north.
Geographical S
B
meridian
Magnetic N
meridian M V
S N BE
Q R

Magnetic Elements of Earth


B l
Angle of Declination (α) At a given place, the acute angle
between the magnetic meridian and the geographical
Torque on a Magnetic Dipole in a meridian is called the angle of declination (or magnetic
declination) α at that place.
Magnetic Field
l
Angle of Inclination or Dip (δ ) The angle of dip δ at a place
A magnetic dipole when placed in an uniform magnetic field,
does not experience any net force. However, it experiences a is the angle which the direction of the earth’s total
torque given by magnetic field BE subtends with the horizontal direction.
τ = M × B or τ = MB sin θ l
Horizontal Component of the Earth’s Magnetic Field
where, θ is the angle from the magnetic field, along which the ( B H ) As earth’s magnetic field, in general, is inclined at an
dipole has been placed. angle δ with the horizontal direction, it may be resolved
l
Work done in rotating a magnetic dipole in a uniform into horizontal component BH and a vertical component BV ,
magnetic field from an initial orientation θ 1 to the final where BH = BE cos δ and BV = BE sin δ
orientation θ2 , is given by ⇒ BE = B2H + BV2
W = MB(cos θ 1 − cos θ2 ). BV
and tan δ =
BH
DAY TWENTY THREE MAGNETISM 257

Magnetisation of Materials Magnetic Materials


There are some substances/materials which acquire magnetic According to behaviour of magnetic substances, they are
properties on placing them in magnetic field, the phenomena classified into three cases:
is called magnetisation of materials. To describe the magnetic
properties of material, we have to understand the following Diamagnetic Materials
terms:
These are materials which show a very small decrease in
(i) Magnetic Induction or Magnetic Flux Density (B) magnetic flux, when placed in a strong magnetising field.
Whenever a piece of magnetic substance is placed in an Hydrogen, water, copper, zinc, antimony, bismuth, etc., are
external magnetising field, the substance becomes examples of diamagnetic materials.
magnetised. If B0 is the magnetic field in free space,
then B = µ r B0.
l
In a diamagnetic material, the net magnetic moment (sum
of that due to orbital motion and spin motion of electrons)
∫ B ⋅ dS is magnetic flux which is equal to µ 0 m inside , where of an atom is zero. The external magnetic field B distorts
m inside is the net pole strength inside a close surface. the electron orbit and thus induces a small magnetic
moment in the opposite direction.
(ii) Magnetic Permeability (µ ) It is the degree or extent to
which the magnetic lines of induction may pass through
l
Diamagnetic materials are feebly repelled in an external
a given distance. magnetic field and thus have a tendency to shift from the
Magnetic permeability of free space µ 0 has a value of stronger to weaker regions of the magnetic field.
4π × 10 −7 TmA −1. However, for a material substance, l
The relative permeability of any diamagnetic substance is
absolute permeability ( µ) has a value, different than µ 0. slightly less than 1 (i.e. µ r < 1) and susceptibility has a
small negative value.
µ B
For any magnetic substance, = = µ r = relative l
Diamagnetism is an intrinsic property and does not vary
µ 0 B0
with magnetic field B or temperature.
magnetic permeability of that substance. Relative
magnetic permeability µ r is a unitless and
dimensionless term.
Paramagnetic Materials
These are the materials which show a small increase in the
(iii) Intensity of Magnetisation (I) Intensity of magnetisation
magnetic flux when placed in a magnetising field. Oxygen,
of a substance is defined as the magnetic moment
air, platinum, aluminium, etc., are examples of paramagnetic
induced in the substance per unit volume, when placed
materials.
in the magnetising field.
M l
In a paramagnetic material, the net magnetic moment of
Thus, I= every atom is non-zero.
V
It is a vector quantity and its SI unit is Am−1. l
Paramagnetic materials are feebly attracted in an external
magnetic field and thus, have a tendency to shift from the
(iv) Intensity of Magnetising Field or Magnetic Intensity (H)
weaker to the stronger regions of magnetic field.
It is a measure of the capability of external magnetising
field to magnetise the given substance and is l
The relative permeability µ r of a paramagnetic material is
mathematically defined as slightly greater than one (µ r > 1). Magnetic susceptibility χ m
B B B of paramagnetic materials is positive.
H = 0 or H = or H = −I
µ0 µ µ0 l
Paramagnetism is temperature dependent. According to the
Magnetic intensity H is a vector quantity and its SI unit Curie’s law, the magnetic susceptibility of a paramagnetic
is Am−1. substance is inversely proportional to its temperature T.
C
(v) Magnetic Susceptibility ( χ m ) Magnetic susceptibility of Mathematically, χ m = , where C is the Curie constant.
T
a substance is the ratio of the intensity of magnetisation
I induced in the substance to the magnetic intensity H.
I Ferromagnetic Materials
Thus, χ m = . It is a scalar quantity and it has no units
H These are the materials which are strongly attracted by a
or dimensions. magnetic field and can themselves be magnetised even in a
Relation between µ r and χ m we have, B = µ 0(I + H ) weak magnetising field. Iron, steel,nickel and cobalt are
ferromagnetic.
 I  B
or B = µ 0H  + 1 or B = B0 (χ m + 1) or = χm + 1 l
These materials show a large increase in the magnetic flux,
H  B0
when placed in a magnetic field. Thus, for them µ r > > 1.
B µ Accordingly, χ m is positive and large.
But = = µ r = relative permeability
B0 µ 0 l
Ferromagnetic materials exhibit all properties exhibited by
∴ µr = χm + 1 paramagnetic substances and by a much larger measure.
258 40 DAYS ~ NEET PHYSICS DAY TWENTY THREE

Magnetic susceptibility of ferromagnetic materials


Electromagnet
l

decreases steadily with a rise in temperature. Above a


certain temperature Tc (known as Curie temperature), the Electromagnets are usually in the form of iron core solenoids.
The ferromagnetic property of the iron core causes the
substance loses its ferromagnetic character and begins to internal magnetic domains of the iron to line up with the
behave as a paramagnetic substance. smaller driving magnetic field produced by the current in the
l
Above the Curie temperature Tc , the magnetic susceptibility of solenoid.
a ferromagnetic material varies as The effect is the multiplication of the magnetic field by
1 C factors of ten to eleven thousands. The solenoid field
χm ∝ or χ m = relationship is B = k µ 0nI , where µ = k µ 0 and k is the
(T − Tc ) (T − Tc )
relative permeability of the iron, the figure shows the
where, C is a constant. It is known as the Curie-Weiss law. magnetic effect of the iron core.

Hysteresis Curve N = North pole


The lag of intensity of magnetisation behind the magnetising N
field during the process of magnetisation and
Iron core
demagnetisation of a ferromagnetic material is called
hysteresis.
S
I
S = South pole
A
B
Retentivity
C
O F
H Permanent Magnet
Substances which at room temperature retain their
E
D ferromagnetic property for a long period of time are called
permanent magnets. Permanent magnets can be made in a
Coercivity variety of ways.
An efficient way to make a permanent magnet is to place a
The whole graph ABCDEFA is a closed loop and known as ferromagnetic rod in a solenoid and pass a current. The
hysteresis loop. magnetic field of the solenoid magnetise the rod.

DAY PRACTICE SESSION 1

FOUNDATION QUESTIONS EXERCISE


1 Which of the following is most suitable as the core of 4 At a certain place, the angle of dip is 60° and the
transformers? horizontal component of earth’s magnetic field is
(a)Steel (b) Alnico 0.4 × 10 −4 T . The earth’s total magnetic field is
(c)Soft iron (d) None of these (a) 0.02 mT
2 What is the angle of dip at a place, where horizontal (b) 0.04 mT
(c) 0.08 mT
component of earth’s magnetic field is equal to the
(d) 0.16 mT
vertical component?
(a) 0° (b) 30° (c) 45° (d) 90°
5 At a place, the value of BH and BV are 0.4 × 10−4 T and
0.3 × 10−4 T. The angle of dip is
3 In a certain place, the horizontal component of magnetic
(a) tan−1  
1
1 (b) tan−1 (075
. )
field is times the vertical component. The angle of  2
3
(c) tan−1  
1
dip at this place is (d) tan−1 (0.09)
 3
(a) zero (b) π / 3 (c) π / 2 (d) π / 6
DAY TWENTY THREE MAGNETISM 259

6 The correct value of the dip angle at a place is 45°. The 14 The permeability of the paramagnetic substance is
dip circle is rotated through 45° out of the magnetic (a) very large (b) small, but more than 1
meridian. The apparent angle of dip will be (c) less than 1 (d) negative
(a) tan−1 ( 2 ) (b) tan−1 
1 
(d) tan−1  
−1 1 15 What is the relation between permeability µ, permeability
 (c) tan (2 )
 2  2
of free space µ 0 and susceptibility χ m ?
7 A uniform magnetic field parallel to the plane of paper, (a) µ = µ 0 (1 + χm )
existed in space initially directed from left to right. When (b) µ = µ 0 (1 − χm )
a bar of soft iron is placed in the field parallel to it, the (c) µ 0 = µ (1 + χm )
lines of force passing through it will be represented by (d) µ 0 = µ (1 − χm )
figure 16 Hysteresis cycle for the material of permanent magnet
should be
(a)high and high (b) high and thin
(a) (b) (c)low and high (d) low and thin
17 Electromagnets are made of soft iron, because soft iron
has j CBSE AIPMT 2010
(a) low retentivity and high coercive force
(c) (d) (b) high retentivity and high coercive force
(c) low retentivity and low coercive force
(d) high retentivity and low coercive force
8 If a diamagnetic substance is brought near the North or 18 There are four light weight rod samples A, B, C and D
the South pole of a bar magnet, it is j
CBSE AIPMT 2009 separately suspended by thread. A bar magnet is slowly
(a) repelled by both the poles brought near each sample and the following observations
(b) repelled by the North pole and attracted by the South are noted
pole (i) A is feebly repelled
(c) attracted by the North pole and repelled by the South (ii) B is feebly attracted
pole (iii) C is strongly attracted
(d) attracted by both the poles (iv) D remains unaffected
9 Nickel shows ferromagnetic property at room Which one of the following is true? j
CBSE AIPMT 2011
temperature. If the temperature is increased beyond (a) C is of a diamagnetic material
Curie temperature, then it will show (b) D is of a ferromagnetic material
(a) paramagnetism (b) anti-ferromagnetism (c) A is of a non-magnetic material
(c) no magnetic property (d) diamagnetism (d) B is of a paramagnetic material

10 Above Curie temperature, 19 The magnetic susceptibility of a material of a rod is 499.


Permeability of vacuum is 4π × 10−7 Hm −1. Absolute
(a) a ferromagnetic substance becomes paramagnetic
(b) a paramagnetic substance becomes diamagnetic permeability of the material of the rod (in Hm −1) is
(c) a diamagnetic substance becomes paramagnetic (a) π × 10−4 (b) 2 π × 10−4
(d) a paramagnetic substance becomes ferromagnetic (c) 3 π × 10−4 (d) 4 π × 10−4
11 The susceptibility of the diamagnetic substance is 20 The variation of magnetic susceptibility ( χ ) with absolute
(a) very large (b) positive and small temperature T for a ferromagnetic is given in figure, by
(c) zero (d) negative +ve +ve
12 Which one of the following is not made of soft iron?
χ χ
(a) Electromagnet j
AIIMS 2011 (a) (b)
(b) Core of transformer
(c) Core of dynamo T T
(0, 0) (0, 0)
(d) Magnet of loudspeaker
+ve
13 The magnetic susceptibility is negative for j
NEET 2016
(a) paramagnetic material only χ χ T
(c) (d) (0, 0)
(b) ferromagnetic material only
(c) paramagnetic and ferromagnetic materials
T +ve
(d) diamagnetic material only (0, 0)
260 40 DAYS ~ NEET PHYSICS DAY TWENTY THREE

21 A curve between magnetic moment and temperature of 27 The period of oscillation of a magnet in a vibration
magnet is magnetometer is 8s. The period of oscillation of magnet
whose magnetic moment is four times that of the first
M M
magnet is
(a) (a) 1 s (b) 4 s (c) 8 s (d) 16 s
(b)
28 Two identiucal bar magnets each of dipole
moment pm and length l are perpendicular N S
T T N
to each other as shown in the figure. The
M M dipole moment of the combination is
(c) (d) (a) 2 pm (b) 2 pm
pm pm S
(c) (d)
2 2
T T 29 A thin bar magnet vibrates in the horizontal plane with a
period of 8 s. The magnet is cut into two halves
22 A tangent galvanometer of reduction factor 1A is placed
perpendicular to magnetic axis. Then, the period of
with the plane of its coil parallel to the magnetic meridian,
vibration of each half is approximately
when a current of 1A is passed through it, the deflection
(a) 4 s (b) 2 s
produced is
(c) 1 s (d) 0.5 s
(a) 45° (b) zero
(c) 30° (d) 60° 30 If the magnet is suspended at an angle 30° to the
magnetic meridian, the dip needle makes an angle of 60°
23 How much work is done to rotate a magnet of dipole
with the horizontal. What is the true dip?
moment pm in a magnetic field B through 180°?
(a) tan−1 (3) (b) tan−1 (3/2)
B
(a) pm (b) pm B (c) tan−1 (2/3) (d) tan−1 (1/3)
2
B 31 A coil in the shape of an equilateral triangle of side
(c) 3 pm (d) 2pm B
2 0.02 m is suspended from its vertex, such that it is
24 A current loop in a magnetic field j
NEET 2013 hanging in a vertical plane between the pole pieces of
permanent magnet producing a uniform field of
(a) experiences a torque whether the field is uniform or
non-uniform in all orientations 5 × 10−2 T. If a current of 0.1 A is passed through the coil,
(b) can be in equilibrium in one orientation what is the torque acting on it?
(c) can be equilibrium in two orientations, both the (a) 5 3 × 10−7 N-m (b) 5 3 × 10−10 N-m
3
equilibrium states are unstable (c) × 10−7 N-m (d) None of these
(d) can be in equilibrium in two orientations, one stable, 5
while the other is unstable 32 A North pole of 40 A-m is placed 20 cm apart from a
25 A compass needle which is allowed to move in a South pole of 80 A-m. Calculate the distance of a point
horizontal plane is taken to a geomagnetic pole. It form the South pole on the line joining, the two poles
j
CBSE AIPMT 2012 where the resultant field due to these poles is zero
(a) will become rigid showing no movement (a) 8.2 cm towards North pole
(b) will stay in any position (b) 8.2 cm away from North pole
(c) will stay in North-South direction only (c) 48.2 cm towards North pole
(d) will stay in East-West direction only (d) 48.2 cm away from North pole
26 Two bar magnets of the same mass, same length and 33 A circular current loop of magnetic moment M is in an
breadth, but having magnetic moments M and 2M are arbitrary orientation in an external magnetic field B. The
joined together pole to pole and suspended by a string. work done to rotate the loop by 30° about an axis
The time period of assembly in a magnetic field of perpendicular to its plane, is
strength B is 3s. If now the polarity of one of the magnet MB
(a) MB (b) 3
is reversed and the combination is again made to 2
MB
oscillate in the same field, then the time of oscillation is (c) (d) zero
2
(a) 3 s (b) 3 3s (c) 3 s (d) 6 s
DAY TWENTY THREE MAGNETISM 261

DAY PRACTICE SESSION 2

PROGRESSIVE QUESTIONS EXERCISE


1 At the magnetic pole of the earth, the value of angle of the magnets is removed and if the other magnet
dip is oscillates in the same field, then the time period
(a) 0° (b) 30° (in second) is
(c) 45° (d) 90° (a) 21/ 4 (b) 21/ 2 (c) 2 (d) 2 3 / 4
2 A small magnet of dipole moment M is kept on the arm of 7 A vibration magnetometer placed in magnetic meridian
a deflection magnetometer set in tanA position at a has a small bar magnet. The magnet executes
distance of 0.2 m. lf the deflection is 60°, the value of M oscillations with the time period of 2 s in earth’s horizontal
is (Take, BH = 0.4 × 10−4 T) magnetic field of 24 µT. When a horizontal field of 18 µT is
(a)2.77 A-m 2 (b) zero produced opposite to the earth’s field by placing a
(c)10.82 A-m 2 (d) None of these current carrying wire, the new time period of magnet will
3 Two short magnets of magnetic moment 2 A-m 2 and be
5 A-m 2 are placed along two lines drawn at right angle to (a) 1 s (b) 2 s (c) 3 s (d) 4 s
each other on the sheet of paper as shown in the figure. 8 Following figures show the arrangement of bar magnets
What is magnetic field at the point of intersection of their in different configurations. Each magnet has magnets in
axis? different configurations. Each magnet has magnetic
dipole moment m. Which configuration has highest net
S N magnetic dipole moment? j
CBSE AIPMT 2014
P
1 N
r1 = 0.3 m
N S
r2 = 0.4 m
S S N S N
N
1 2

N
2 N

S 30° 60°
S N S N
(a)0 (b) 215 × 10−5 T 3 4
(c)2.15 × 10−5 T (d) 0.215 T (a) 1 (b) 2 (c) 3 (d) 4
4 Two tangent galvanometer having coils of the same 9 A bar magnet of length l and magnetic dipole moment M
radius are connected in series. A current flowing in is bent in the form of an arc as shown in figure. The new
them, produces deflection of 60° and 45°, respectively. magnetic dipole moment will be j
NEET 2013
The ratio of the number of turns in the coils is
4 ( 3 + 1) 3 ( 3 + 1)
(a) (b) (c) (d)
3 1 1 ( 3 − 1)

5 In tanA position, two short bar magnets of moments in r r


60°
the ratio 1:1.7321 are placed at the same distance
separately. If the deflection produced for the first magnet
is 30°, then the deflection produced for the second 3 2 M
(a) M (b) M (c) M (d)
magnet will be π π 2
(a) 60° (b) 30° 10 A thin magnetic needle vibrates in the horizontal plane
(c) 45° (d) 42° with a period of 4s. The needle is cut into two halves by a
6 A vibration magnetometer consists of two identical bar plane normal to magnetic axis of the needle. Then, the
magnets placed one over the other, such that they are period of vibration of each half needle is approximately
perpendicular and bisect each other. The time period of (a) 4 s (b) 2 s
oscillation in a horizontal magnetic field is 25 / 4 s. One of (c) 8 s (d) 1 s
262 40 DAYS ~ NEET PHYSICS DAY TWENTY THREE

11 A thin diamagnetic rod is placed vertically between the 15 Two small magnets A and B of dipole BH
poles of an electromagnet. When the current in the moments M 0 and 2M 0 respectively, are
electromagnet is switched ON, then the diamagnetic rod fixed perpendicular to each other with
is pushed up, out of the horizontal magnetic field. Hence, their North poles in contact. The B
A α
the rod gains gravitational potential energy. The work combination is placed on a floating β
required to do this comes from j NEET 2018
body, so as to move freely in earth’s S1 S2
(a) the lattice structure of the material of the rod magnetic field, the value of α is
(b) the magnetic field
(b) sin−1  
1
(c) the current source (a) tan−1 (2)
 2
(d) the induced electric field due to the changing magnetic
(c) cos−1  
1
field (d) 20°
 2
12 Two magnets placed one above the other oscillate with a
16 A current carrying circular loop of radius R is placed in
period of 17 s. If one of them is reversed the time period
the XY -plane with centre at the origin. Half of the loop
becomes 8 s. The ratio of their magnetic moment is
with x > 0 is now bent, so that it now lies in theYZ -plane.
nearest to
8 9 1 1 (a)The magnitude of magnetic moment now diminishes
(a) (b) (c) (d) (b)The magnetic moment does not change
17 14 4 2
(c)The magnitude of B at (0, 0, z), z >> R increases
13 A bar magnet is hung by a thin cotton thread in a uniform (d)The magnitude of B at (0, 0, z), z >> R unchanged
horizontal magnetic field and is in equilibrium state. The
17 A bar magnet is suspended with unspun thread. It lies in
energy required to rotate it by 60° is W . Now the torque
the magnetic meridian. The upper end of the thread is
required to keep the magnet in this new position is
turned through 150°. It turns the magnet by 30° with the
j
NEET 2016
magnetic meridian. Through what angle should the upper
W 3W 2W end of the thread be turned to rotate the magnet through
(a) (b) 3W (c) (d)
3 2 3 90°?
14 A bar magnet having a magnetic moment of 2 × 104 JT −1 (a) 360° (b) 330° (c) 300° (d) 270°
is free to rotate in a horizontal plane. A horizontal
18 If θ1 and θ 2 be the apparent angles of dip observed in
magnetic field B = 6 × 10−4 T exists in the space. The work
two vertical planes at right angles to each other, then the
done in taking the magnet slowly from a direction parallel
true angle of dip θ is given by j
NEET 2017
to the field to a direction 60° from the field is
(a) cot 2 θ = cot 2 θ1 + cot 2 θ2 (b) tan2 θ = tan2 θ1 + tan2 θ2
j
CBSE AIPMT 2009 (c) cot 2 θ = cot 2 θ1 − cot 2 θ2 (d) tan2 θ = tan2 θ1 − tan2 θ2
(a) 0.6 J (b) 12 J (c) 6 J (d) 2 J

ANSWERS
SESSION 1 1 (c) 2 (c) 3 (b) 4 (c) 5 (b) 6 (a) 7 (b) 8 (a) 9 (a) 10 (a)
11 (d) 12 (d) 13 (d) 14 (b) 15 (a) 16 (a) 17 (d) 18 (d) 19 (b) 20 (a)
21 (c) 22 (a) 23 (d) 24 (d) 25 (c) 26 (b) 27 (b) 28 (b) 29 (a) 30 (b)
31 (a) 32 (c) 33 (d)
SESSION 2 1 (d) 2 (a) 3 (c) 4 (c) 5 (c) 6 (c) 7 (b) 8 (c) 9 (b) 10 (b)
11 (c) 12 (b) 13 (b) 14 (c) 15 (a) 16 (a) 17 (b) 18 (a)
DAY TWENTY THREE MAGNETISM 263

Hints and Explanations


1 The core of the transformer should have become simply paramagnetic, i.e. the 21 With rise in temperature, the magnetism
high permeability and low coercivity. dipoles still tend to align with an of magnet falls and at critical
The soft iron have high permeability external field, but much more weakly temperature, it becomes zero.
and lens coercivity. So, the core of and thermal agitation can now more
easily disrupt the alignment. 22 I = k tanθ
transformers are made of soft iron.
I 1
10 Ferromagnetism decreases with rise in ⇒ tanθ = = = 1
2 The dip angle is 45° at a place, where k 1
horizontal component of earth’s temperature. So, the temperature above
∴ θ = 45°
magnetic field is equal to the vertical which a ferromagnetic substance
component. becomes paramagnetic is called the θ θ

Curie temperature of the substance.


23 W = ∫0 τ ⋅ dθ = ∫0 p m B H sin θ dθ
3 From tan δ = V = V = 3 = p m B H [− cos
θ
θ] 0
H V / 3 11 The susceptibility of the diamagnetic
∴ δ = π /3 substance is less than zero or negative. = p m B H [1 − cos θ]
When θ = π, then cos θ = − 1
B 12 Temporary magnet is made from soft
4 B H = B 0 cos δ. Hence, B 0 = H 1 /2
cos δ 2 T  8
iron, while magnet of loudspeaker is T′=   T = =   s = 4s
made from permanent magnet.  8 2  2
Here, δ = 60° and B H = 0.4 × 10−4 T
Hence, W = 2 p m B H = 2 p m B
0.4 ×10−4 13 As we know the relation between the
⇒ B0 = = 0.8 × 10−4 magnetic permeability and 24 For like parallel magnetic field is stable
cos60°
susceptibility of material, i.e. and for anti-parallel is unstable.
⇒ B 0 = 0.08 mT
µ r = 1 + χm …(i)
BV 25 It will stay in any position at
5 Qtanδ = Q For diamagnetic substances, µ r < 1
BH geomagnetic North and South poles.
So, according to Eq. (i), the magnetic
susceptibility (χ m ) of diamagnetic T2 (3M )1 /2 3
. × 10−4
03 3 26 Q = =
tan δ = = substance will be negative. T1 (M )1 /2 1
0.4 × 10−4 4
14 The permeability of the paramagnetic Given, T1 = 3s, then we get T2 = 3 3 s
 3
⇒ δ = tan −1   = tan −1 (075
. ) substance is small, but more than 1.
 4
27 T = 2 π [I / p m B]1 /2 . So, when p m
15 The relation between permeability µ,
6 tan δ = V = 45°. Hence, V = H permeability of free space µ 0 and
becomes 4 times, T becomes half.
H 8
susceptibility χ m is, µ = µ 0 (1 + χ m ) ∴ T = = 4s
When rotated through 45° out of 2
magnetic meridian, the vertical 16 The material of permanent magnet
component remains unchanged, but the 28 Distance between the free North and
should have high retentivity as well as South poles is 2 L . Hence, dipole
horizontal component changes to
H cos 45° = H / 2. high coercivity. moment of the combination is 2 p m .
V 17 The material suitable for making
Hence, tan δ′ = = 2 29 T = 2 π [I / p m B]1 /2⋅ p m = q m l. On cutting
H / 2 electromagnets should have high l is reduced to half, so p m also becomes
⇒ δ ′ = tan −1 2 retentivity and low coercivity. half. For a thin bar, the moment of
M 2 M 2
18 Paramagnetic material will be feebly inertia, I = [l + b2 ] ≈ l . Because
7 Because of large permeability of soft 12 12
iron magnetic lines of force prefer to attracted, diamagnetic material will be for thin bar, b is negligible. On cutting l
pass through it. Concentration of lines feebly repelled and ferromagnetic becomes half. Also, M becomes half. So,
in soft iron bar increases as shown in the moment of inertia becomes
figure (b). material will be strongly attracted.
one-eighth. Hence, new time period is
8 Diamagnetic substances are weakly 19 Magnetic susceptibility, χ = µ r − 1  2
1 /2
T  8
T′=   T = =   s = 4s
magnetised in a direction opposite to µ  8 2  2
or χ= −1
that of applied magnetic field. These are µ0
repelled in an external magnetic field, µ 30 tan δ = V and tan δ ′ = V
i.e. have a tendency to move from high or 499 = −1 H H cos θ
to low field region, i.e. it is repelled by 4 π × 10−7 tan δ
or tan δ ′ =
both North and South poles of a bar or µ = 500 × 4 π × 10−7 cos θ
magnet.
= 2 π × 10−4 Hm −1 ⇒ tan δ = tan δ ′ cos θ
9 If the temperature of a ferromagnetic = tan (60° ) cos (30° )
material is raised above a certain critical 20 As temperature of a ferromagnetic 3 3
material is raised, its susceptibility = 3× =
value, called the Curie temperature, the 2 2
χ remains constant first and then
exchange coupling ceases to be −1  3 
decreases as shown in figure (a). ⇒ δ = tan  
effective. Most such materials, then  2
264 40 DAYS ~ NEET PHYSICS DAY TWENTY THREE

31 Torque, τ = IAB sin θ, I = 01


. A, θ = 90° Magnetic field due to magnet 2, (4)
1 µ 2M2 10−7 × 2 × 5
A = × base × height B2 = 0 =
2 4 π r23 (0.4)3
60°
1
or A = a ×
a 3 = 1.56 × 10−5 T
2 2 M 4 = 2m cos 30° = m 3
Net field at P,
3 a2 3 × (0.02)2
= = B = B12 + B22 = 215
. × 10 −5
T
4 4 9 The magnetic moment,
= 3 × 10−4 m2 M = ml
4 For tangent galvanometer, π 3l
τ = 01 −4
. × 3 × 10 × 5 × 10 sin 90° −2 µ 0 nI From figure, l = × r or r =
= H tan θ 3 π
= 5 3 × 10−7 N-m 2r
∴ New magnetic moment,
n ∝ H tanθ 3l
32 The situation is shown in figure. n1 tan θ1 M′ = m × r = m ×
Q = π
Let at point P, magnetic field is zero. n2 tan θ2 3 3M
µ 0 m1 µ 0 m2 = ⋅ ml =
∴ = n1 tan 60° 3 π π
4 π r12 4 π r22 ⇒ = =
n2 tan 45° 1 10 The moment of inertia of needle, when
40 A-m 80 A-m it cuts into two halves by a plane normal
µ 2M
N S 5 B = BE tan θ or 0 = B E tan θ to magnetic axis of the needle.
4 π r2 2
P 20 cm 1  m  I  I
x or M ∝ tanθ I′ =
    or I ′ =
12  2   2  8
40
=
80 M1 tan 30° 1
∴ = = M
x2 (20 + x )2 M2 tan q 1.7321 Also, M ′ =
2
(20 + x )2 = 2 x2 or tan θ = 1 or θ = 45° I′
T ′ = 2π
⇒ 20 + x = 2 x I I M′B
6 T = 2π ∝
20 MB M The time period of vibration
∴ x=
2−1 I ×2 1 1
T′ I′ M I 2M 1 = 2π = T = 4s = 2s
∴ = = = 8 × MB 2 2
= 48.2 cm (towards North pole) T I M′ 2I M 21/4

33 Let magnetic moment M of current loop 25/ 4 11 As the source of current is switched ON,
or T ′ = = 2s
be making an angle α with the direction 21 / 4 a magnetic field sets up in between the
of B. When a current circular loop is poles of the electromagnet.
rotated in a magnetic field by 30° about 7 Time period in vibration magnetometer, As we know that a diamagnetic
an axis perpendicular to its plane, there I 1 substance when placed in a magnetic
T = 2π ⇒T ∝
is no change in the angle α between M × BH BH field acquires a feeble magnetism
magnetic moment M and magnetic field opposite to the direction of magnetic
T1 (B H )2 2 18
B. Therefore, θ1 = α, θ2 = α. ⇒ = ⇒ = field.
T2 (B H )1 T2 24
Also, in the presences of the field
Work done, W = MB (cos θ1 − cos θ2 ) T = 2.3s ≈ 2s (non-uniform), these substances are
= MB(cos α − cos α) = 0 attracted towards the weaker field, i.e.
8 (1) M1 = m √2 they move from stronger to weaker
SESSION 2 magnetic field.
1 At magnetic poles, the angle of dip Due to these reasons, the rod is repelled
m by the field produced to the current
is 90°.
source. Hence, it is pushed up, out of
2 B = B H tanθ horizontal field and gains gravitational
m
µ 0 2M potential energy.
= 0.4 × 10−4 tan 60°
4π r3 (2) m 1 /2
⇒ M2 = 0  I1 + I2 
µ 0M 12 T1 = 2 π  
= 0.4 × 10−4 tan 60° m −
2 π (0.2)3  m1
( p p m2 ) B 
(3) 1 /2
0.4 × 2 π × (0.2)3 × (10−4 ) × 3  l1 + l2 
⇒M = m and T2 = 2 π  
µ0  (p m1 + p m2 ) B 
= 277
. A-m2 30° T2 p m1 + p m2
m Therefore, 1 = .
3 Magnetic field due to magnet 1, T22 p m1 − p m2
µ 0 2M1 M3 = m (1 + cos 30° ) 2
B1 = This gives,
4 π r13  T12 − T22 (17)2 − (8)2
3 =
p m2
=
−7 =m 1 + 2
10 ×2×2  2  T12 + T22 p m1 (17)2 + (8)2
=
3
(03
. ) 225 9
= m 2+ 3 = ≡
B1 = 1.48 × 10−5 T 353 14
DAY TWENTY THREE MAGNETISM 265

13 QWork done in rotating the magnet, M1 sin α = M2 sin(90 − α ) 18 Let B H and B V be the horizontal and
W = MB (cos θ 0 − cos θ) [Qα + β = 90°] vertical component of earth’s magnetic
where, M = magnetic moment of M1 sin α = M2 cos α field B.
the magnet M 0 sin α = 2M 0 cos α
Plane 1 Magnetic meridian
and B = magnetic field. ⇒ tanα = 2
W = MB (cos 0° − cos 60° ) α = tan −1 (2)
 1  MB
= MB  1 −  = 16 The magnetic moment, M = I × πR2 . It
 2 2
acts perpendicular to the loop along BH sin θ
∴ MB = 2W …(i)

θ′
z-direction. When half of the current

°–
Torque on a magnet in this position is

90
loop is bent in YZ-plane, then magnetic
given by θ1
moment due to half current loop in
τ = M×B
XY-plane, M1 = I ( πR2 / 2) acting along BH cos θ Plane 2
= MB ⋅ sin θ = 2W ⋅ sin 60°
z-direction. B B
[from Eq. (i)] tan θ = V ⇒ cot θ = H ...(i)
3 Magnetic moment due to half current BH BV
= 2W = 3W loop in YZ-plane, M2 = I ( πR2 / 2) along
2 Let planes 1 and 2 are mutually
x-direction. perpendicular planes making angle θ and
14 When magnetic dipole is rotated from (90° − θ) with magnetic meridian. The
initial position θ1 to final position θ2 , Effective magnetic moment due to entire
vertical component of earth’s magnetic
then work done bent current loop,
field remain same in two plane, but
= MB(cos θ1 − cos θ2 ) M′ = M12 + M22 effective horizontal components in the
Given, θ1 = 0°, θ2 = 60° two planes is given by
= (IπR2 / 2)2 + (IπR2 / 2)2
B1 = B H cos θ′ ...(ii)
Magnetic moment, M = 2 × 10 J/T 4
IπR2 and B2 = B H sin θ′ ...(iii)
−4 = 2< M
Magnetic field, B = 6 × 10 T 2 B BV
Then, tan θ1 = V =
 1 i.e. Magnetic moment diminishes. B1 B H cos θ′
So, W = MB  1 − 
 2 17 The twist in the thread is B cos θ′
cot θ1 = H ...(iv)
150° − 30° = 120°. BV
Qcos 0° = 1 
and cos 60° = 1 / 2 Restoring torque, B BV
Similarly, ⇒ tan θ2 = V =
2 × 10 × 6 × 10
4 −4 B B2 B H sin θ′
= = 6J p m B sin 30° = p m ⋅
2 B sin θ′
2 ⇒ cot θ2 = H ...(v)
p mB BV
15 For equilibrium, Hence, ∝ 120° …(i)
2 From Eqs. (iv) and Eq. (v), we get
M1 sin α = M2 sin β
[∴ α + β = 90° ]
In the second case, let the thread be ⇒ cot2 θ1 + cot2 θ2
turned through angle θ. Then, twist in B 2H cos 2 θ′
B 2 sin2 θ′
M2
BH
M1 the thread (θ − 90°). And restoring = + H
2
BV B 2V
torque is
β α ⇒ cot θ1 + cot θ2
2 2
p m B sin 90° = p m B .
Therefore, p m B ∝ (θ − 90° ) …(ii) B2
A α B = H (cos 2 θ′ + sin2 θ′ )
β From Eqs. (i) and (ii), we get B 2V
θ = 330° ⇒ cot θ1 + cot2 θ2 = cot2 θ
2
DAY TWENTY FOUR

Electromagnetic
Induction
Learning & Revision for the Day
u Magnetic Flux ( φB ) u Motional Emf u Mutual Induction
u Faraday’s Law of u Rotational Emf u Combination of Inductors
Electromagnetic Induction u Self-Induction u Eddy Currents
u Lenz’s Law

Magnetic Flux ( φB )
ds
The flux associated with a magnetic field is defined in a similar B
manner to that used to define electric flux. Consider an element
θ
of area ds on an arbitrary shaped surface as shown in figure. If
the magnetic field at this element is B, the magnetic flux
through the element is,
dφB = B ⋅ ds = Bds cos θ
Here, ds is a vector that is perpendicular to the surface and has a
magnitude equal to the area ds and θ is the angle between B and ds at that element.
Magnetic flux is a scalar quantity. Outward magnetic flux is taken as positive
(i.e. θ < 90 °) and inward flux is taken as negative (i.e. θ > 90 °).
SI unit of magnetic flux is 1 weber (1 Wb).
where, 1 Wb = 1 T × 1 m2 = 1 T-m2
Dimensional formula of magnetic flux is [ML2 T−2 A−1 ] . PREP
MIRROR
Your Personal Preparation Indicator
Faraday’s Law of Electromagnetic Induction
u No. of Questions in Exercises (x)—
This law states that, the induced emf in a closed loop equals the negative of the time rate
of change of magnetic flux through the loop. u No. of Questions Attempted (y)—
No. of Correct Questions (z)—
dφ B
u

Induced emf, |e | = (Without referring Explanations)


dt
dφ B Accuracy Level (z / y × 100)—
|e | = N
u
l
For N turns,
dt u Prep Level (z / x × 100)—
However, if we consider the direction of induced emf, then
In order to expect good rank in NEET, your
dφ B Nd(BA cos θ) − NBA(cos θ2 − cos θ 1)
e=−N =− = Accuracy Level should be above 85 & Prep
dt dt ∆t Level should be above 75.
DAY TWENTY FOUR ELECTROMAGNETIC INDUCTION 267

If the given electric circuit is a closed circuit having a total


Rotational Emf
l

resistance R, then the induced current,


e N dφ B Let a conducting rod of length l rotate about an axis passing
I = =− through one of its ends (that end may be fixed), with an angular
R R dt
velocity ω in a plane perpendicular to the magnetic field B, then
N
Induced charge, dq = Idt = − dφ B an induced emf is set up between the ends of the rod, whose
R magnitude is given by
2
e2 N 2  dφ B 
and induced power, P = =   |e | =
1 2
Bl ω
R R  dt  2

Lenz’s Law Self-Induction


Self-induction is the phenomenon due to which an induced
The negative sign in Faraday’s equations of electromagnetic emf is set up in a coil or a circuit whenever the current passing
induction describes the direction in which the induced emf through it changes. The induced emf opposes the change that
drives current around a circuit. However, that direction is causes it and is thus known as back emf.
most easily determined with the help of Lenz’s law. This law
Induced current
states that the direction of any magnetic induction effect is (make)
such as to oppose the cause of the effect.
Later, we will see that Lenz’s law is directly related to energy Induced current +
conservation. A
(break) –

Motional Emf + –
Let a conducting rod of length l be moving with a uniform K Rh
velocity v perpendicular to a uniform magnetic field B, an l
Inductance is the inherent property of electrical circuits and
induced emf is set up.
is known as the electrical inertia.
X X X X + X X X X

l
l
An inductor is said to be an ideal inductor if its resistance is
X X X X v X X X X
zero.
X X X X – X X X X l
An inductor does not oppose current but opposes changes
The magnitude of the induced emf will be (growth or decay of current) in the circuit.
|e | = B l v
l
If the rod is moving such that it makes an angle θ with the Self-Inductance
direction of the magnetic field, then Flux linked with the coil is
| e| = B l v sin θ Nφ B ∝ I or Nφ B = LI,
Hence, for the motion parallel to B, the induced emf where the constant L is known as the coefficient of
is zero. self-induction or self-inductance of the given coil.
l
When a conducting rod moves horizontally, then an It may be defined as the magnetic flux linked with the coil,
induced emf is set up between its ends due to the vertical when a constant current of 1 A is passed through it.
component of the earth’s magnetic field. However, at the
Induced emf due to self-induction,
magnetic equator, induced emf will be zero, because
B V = 0. dφ dI
e=–N =–L
dt dt
l
If during landing or taking off, the wings of an aeroplane
are along the East-West direction, an induced emf is set SI unit of inductance is henry.
up across the wings (due to the effect of BH ).
Magnetic Potential Energy of an Inductor
In building, a steady current in an electric circuit, some
Motional Emf in a Loop
l

work is done by the emf of the source, against the


If a conducting rod moves on two parallel conducting rails, self-inductance of the coil.
then an emf is induced whose magnitude is|e | = B l v 1
The work done, W = LI 2
and the direction is given by the Fleming’s right hand rule. 2
|e | B l v The work done is stored as the magnetic potential energy of
Induced current,| I | = =
l
l

R R that inductor.
B2 l2 v 1
U = L I2
l
Magnetic force, Fm = BIl = Thus,
R 2
268 40 DAYS ~ NEET PHYSICS DAY TWENTY FOUR

Formulae for Self-Inductance For a pair of two magnetically coupled coils of self-inductances
L1 and L2 respectively, the mutual inductance,
l
For a circular coil of radius R and N turns, the
self-inductance, M 12 = M21 = M = k L1L2
1 where, k is the coupling coefficient.
L = µ0 π N2 R
2
l
For a solenoid coil having length l, total number of turns N Formulae for Mutual Inductance
and cross-sectional area A, l
Assuming the coupling coefficient k = 1 and medium to be
µ0 N2 A  N a free space or air. Mutual inductance of a pair of concentric
L= = µ 0 n Al
2
where, n =
l  l  circular coils is
For a toroid of radius R and number of turns N, µ 0N 1N2 πr 2
M=
l

1 2R
L = µ0 N2 R
2 where, r = radius of the coil (of small radius)
l
For a square coil of side a and number of turns N, and R = radius of the coil (of larger radius).
2 2 l
For a pair of two solenoid coils, wound one over the
L= µ0 N2 a µ N N A
π other, M= 0 1 2
l
For a pair of concentric coplanar square coils,
Mutual Induction 2 2 µ 0N 1N2 a2
Mutual induction is the phenomenon due to which an emf is M=
πb
induced in a coil when the current flowing through a
neighbouring coil changes. where, a = side of the smaller coil and b = side of the larger
coil.
For a given pair of coils, mutually coupled, then according
Mutual Inductance
l

to theorem of reciprocity,
Mutual inductance of a pair of coils is defined as the magnetic M 12 = M21 = M
flux linked with one coil, when a constant current of unit
magnitude, flows through the other coil.
Mathematically, NφB2 = MI 1 Combination of Inductors
where, M is known as the mutual inductance for the given l
If two coils of self-inductances L 1 and L 2 are placed quite
pair of coils. far apart and are arranged in series, then their equivalent
Induced emf due to mutual inductance, inductance,
dφB2 dI L s = L1 + L 2
e2 = – N =–M 1
dt dt l
If the coils are placed quite close to each other, so as to
Hence, mutual inductance for a pair of coils is numerically mutually affect each other, then their equivalent inductance,
equal to the magnitude of induced emf in one coil when L s = L1 + L2 ± 2 M
current in the other coil changes at a rate of 1 As −1. Here, M has been written with ± sign depending on the fact
SI unit of mutual inductance M, is henry. whether currents in the two coils are flowing in same sense
or opposite sense.
Mutual inductance of a pair of coils is maximum, when the
two coils are wound on the same frame. However, mutual
l
If two coils of self-inductances L1 and L 2 are connected in
inductance is negligible when the two coils are oriented parallel, then equivalent inductance L p is given by
mutually perpendicular to each other (see figure). In this 1 1 1 L1L 2
= + ⇒ Lp =
context, we define a term coupling coefficient k. L p L1 L 2 L1 + L 2

S
S
Eddy Currents
P S Currents induced in the body of bulk of the conductors due to
P
charge in magnetic flux linked to them, are called the eddy
P currents. The production of eddy currents in a metallic
(a) (b) (c)
conductor leads to a loss of electric energy in the form of
Coupling coefficient is given by heat energy.
Magnetic flux linked with secondary coil Eddy currents can be minimised by taking the metal
k =
Magnetic flux developed in primary coil (generally soft iron) core in the form of a combination of thin
laminated sheets or by slotting process.
It is observed that 0 ≤ k ≤ 1.
DAY TWENTY FOUR ELECTROMAGNETIC INDUCTION 269

DAY PRACTICE SESSION 1

FOUNDATION QUESTIONS EXERCISE


1 A square of side L metres lies in the XY -plane in a region, 8 A rectangular coil of 20 turns and area of cross-section
where the magnetic field is given by B = B0 ( 2$i + 3$j + 4k$ ) T, 25 cm 2 has a resistance of 100 Ω. If a magnetic field
where B0 is constant. The magnitude of flux passing which is perpendicular to the plane of the coil changes
through the square is at a rate of 1000 T/s, the current in the coil is
(a)(2 B 0 L2 ) Wb (b) (3 B 0 L2 ) Wb (a) 1.0 A (b) 50 A (c) 0.5 A (d) 5.0 A
2 2 9 Magnetic flux (weber) in a closed circuit of resistance
(c)(4 B 0 L ) Wb (d) ( 29 B 0 L ) Wb
10 Ω varies with time t (second) as φ = 6 t 2 − 5 t + 1. The
2 As a result of change in the
magnitude of induced current at t = 0 . 25 s is
magnetic flux linked to the closed
loop shown in the figure, an emf (a) 0.2 A (b) 0.6 A (c) 1.2 A (d) 0.8 A
V (volt) is induced in the loop. The 10 A circular ring of diameter 20 cm has a resistance of
work done (joules) in taking a 0.01 Ω. The charge that will flow through the ring, if it is
charge q (coulomb) once along the turned from a position perpendicular to a uniform
loop is magnetic field of 2.0 T to a position parallel to the field is
qV about j CBSE AIPMT 2010

(a) qV (b) zero (c) 2qV (d)


2 (a) 63 C (b) 0.63 C (c) 6.3 C (d) 0.063 C
3 The magnetic flux linked with a coil varies with time as 11 The magnetic flux through a circuit of resistance R
φ = ( 3 t 2 + 4 t + 9) Wb. What is the induced emf at, changes by an amount ∆φ in a time ∆t. Then, the total
t = 2 s? quantity of electric charge q that passes any point in the
(a) 3 V (b) 4 V (c) 9 V (d) 16 V circuit during the time ∆t is presented by
1 ∆φ ∆φ ∆φ ∆φ
4 The flux linked with a coil at any instant t given by (a) q = (b) q = (c) q = (d) q = R
R ∆t R ∆t ∆t
φ = 10 t 2 − 50 t + 250 12 A cylindrical bar magnet is
Then, induced emf at t = 3 s is rotated about its axis shown in Axis
figure. A wire is connected from A
(a) −10 V (b) 10 V (c) 190 V (d) −190 V
the axis and is made to touch the N
5 The magnetic flux through a surface varies with time as
cylindrical surface through a Bar
follows φ = 12 t 2 + 7t − 3 magnet ω
contact. Then,
Here, φ is in milliweber and t is in seconds. What will be (a) a direct current flows in the
the induced emf at t = 5 s? ammeter A
(a)338 mV (b) 127 mV S
(b) no current flows through the
(c)105 mV (d) None of these ammeter A
6 A coil of resistance 400 Ω is placed in a magnetic field. If (c) an alternating sinusoidal current flows through the
the magnetic flux φ (Wb) linked with the coil varies with ammeter A with a time period T = 2π / ω
time t (second) as φ = 50 t 2 + 4. (d) a time varying non-sinusoidal current flows through the
ammeter A
The current in the coil at t = 2 s is j
CBSE AIPMT 2012
(a) 0.5 A (b) 0.1 A 13 A copper ring having a cut such as not to
(c) 2 A (d) 1 A form a complete loop is held horizontally S
and a bar magnet is dropped through the
7 When a coil of cross-sectional area A and number of
ring with its length along the axis of the N
turns N is rotated in a uniform magnetic field B with
ring, as shown in figure. The acceleration
angular velocity ω, then the maximum emf induced in the
of the falling magnet is
coil will be
BAω (a) g (b) less than g
(a) BNA (b) (c) BNAω (d) zero (c) more than g (d) zero
N
270 40 DAYS ~ NEET PHYSICS DAY TWENTY FOUR

a
14 The current from A to B is
increasing in magnitude. What is
the direction of induced current, b d
if any in the loop as shown in A B
figure. c
(a) No current is induced (b) Clockwise current X Y
(c) Anti-clockwise current (d) Alternating current (a) abcd (b) adcb
(c) The current will reverse its direction as the
15 Three identical coils A, B and C are electron goes past the coil
placed with their planes parallel to (d) No current induced
one another. Coils A and C carry
19 A thin semi-circular conducting ring (PQR ) of radius r is
currents as shown in figure. Coils B
falling with its plane vertical in a horizontal magnetic field
and C are fixed in position and coil A B C
B, as shown in figure. The potential difference developed
A is moved towards B. Then,
across the ring when its speed is v, is
current
X X X X X X
induced in B is in
B
(a) clockwise current X X X X X X
Q
(b) anti-clockwise current X X X X X X

(c) no current is induced in B r


X X X X X X
(d) current in induced only when both coils move
X X P X X X R X
16 The magnet in figure rotates a shown on a pivot through
its centre. At the instant shown, what are the directions of (a) zero j CBSE AIPMT 2014
the induced currents (b) Bv πr 2 /2 and P is at higher potential
(c) πrBv and R is at higher potential
s
(d) 2rBv and R is at higher potential
20 A conducting circular loop is placed in a uniform
magnetic field 0.04 T with its plane perpendicular to the
magnetic field. The radius of the loop starts shrinking at
A B C D
n 2 mm s −1. The induced emf in the loop when the radius is
(a) A to B and C to D (b) B to A and C to D 2 cm is j
CBSE AIPMT 2009
(c) A to B and D to C (d) B to A and D to C (a) 3.2 π µV (b) 4.8 π µV (c) 0.8 π µV (d) 1.6 π µV
17 The variation of induced emf (E ) with time (t ) in a coil, if a 1
21 A fan blade of length metre rotates with frequency
short bar magnet is moved along its axis with a constant π
velocity is best represented as 5 cycle per second perpendicular to a magnetic field
10 T. The potential difference between the centre and the
end of the blade is
(a) − 50 V (b) + 50 V (c) − 2 V (d) + 0.02 V
E E 22 A wire loop is rotated in a magnetic field. The frequency
of change of direction of the induced emf is j
NEET 2013

(a) (b) (a) once per revolution (b) twice per revolution
t (c) four times per revolution (d) six times per revolution
t
23 A disc of radius 0.1 m is rotating with a frequency
E E 10 rev/s in a normal magnetic field of strength 0.1 T. Net
induced emf is
(a) 2 π × 10−2 V (b) π × 10−2 V
(c) (d) π
(c) × 10−2 V (d) None of these
t t 2
24 The coefficients of self-induction of two coils are L1 and
L 2. To induce an emf of 25 V in the coils change of current
18 An electron moves on a straight line path XY as shown.
of 1A has to be produced in 5 s and 50 ms, respectively.
The abcd is a coil adjacent in the path of electron. What
The ratio of their self-inductances L1 : L 2 will be
will be the direction of current, if any induced in the coil?
(a) 1 : 5 (b) 200 : 1 (c) 100 : 1 (d) 50 : 1
j
CBSE AIPMT 2015
DAY TWENTY FOUR ELECTROMAGNETIC INDUCTION 271

25 When the current changes from + 2 A to − 2 A in 0.05 s, (a) 3 H (b) 2 H


an emf of 8 V is induced in a coil. The coefficient of (c) 1 H (d) 4 H
self-induction of the coil is 30 Two coils of self-inductances 2 mH and 8 mH are placed
(a) 0.1 H (b) 0.2 H (c) 0.4 H (d) 0.8 H so close together that the effective flux in one coil is
26 In a solenoid, if number of turns is doubled, then completely linked with the other. The mutual inductance
self-inductance will become between these coils is
(a) half (b) double (c) 1/4 times (d) quadruple (a) 10 mH (b) 6 mH
(c) 4 mH (d) 16 mH
27 The current in a coil changes from + 10A to − 2A in 3 ms.
What is the induced emf in the coil? The self-inductance 31 Two coils X and Y are placed in a circuit such that a
of the coil is 2 mH. current changes by 3 A in coil X and the magnetic flux
changes of 1.2 Wb occurs inY . The value of mutual
(a) 8 V (b) 4 V (c) 0.8 V (d) 0.4 V
inductance of the coils is
28 The inductance of a coil is proportional to
(a) 0.2 H (b) 0.4 H
(a) its length (b) the number of turns (c) 0.6 H (d) 3.6 H
(c) the resistance of coil
(d) the square of the number of turns 32 The cause of production of eddy currents is
(a) the motion of a conductor in a varying magnetic field
29 A long solenoid has 1000 turns. When a current of 4A
(b) the motion of a insulator in a varying magnetic field
flows through it, the magnetic flux linked with each turn of (c) current flowing in a conductor
the solenoid is 4 × 10−3 Wb. The self-inductance of the (d) current flowing in an insulator
solenoid is j NEET 2016

DAY PRACTICE SESSION 2

PROGRESSIVE QUESTIONS EXERCISE


1 A square loop of wire of side 5 cm is lying on a horizontal 4 A rectangular, a square, a circular and an elliptical loop,
table. An electromagnet above and to one side of the all in the XY -plane, are moving out of a uniform magnetic
loop is turned on, causing a uniform magnetic field field with a constant velocity, v = v $i . The magnetic field is
downward at an angle of 60° to the vertical as shown in directed along the negative Z -axis direction. The induced
figure. The magnetic induction is 0.50 T. The average emf, during the passage of these loops, out of the field
induced emf in the loop, if the field increases from zero to region, will not remain constant for j
CBSE AIPMT 2009
its final value in 0.2 s is (a) the rectangular, circular and elliptical loops
(a) 5.4 × 10−3 V (b) 3.12 × 10−3 V θ = 60° (b) the circular and the elliptical loops
(c) 0 (d) 0.25 × 10−3 V (c) only the elliptical loop
(d) any of the four loops
2 A coil having n turns and resistance R Ω is connected with
a galvanometer of resistance 4R Ω. This combination is 5 A bicycle wheel of radius 0.5 m has 32 spokes. It is
moved in time t second. From a magnetic field (W1 ) Wb to rotating at the rate of 120 revolutions per minute,
(W2 ) Wb. The induced current in the circuit is perpendicular to the horizontal component of the earth’s
W2 − W1 − n (W2 − W1) magnetic field BH = 4 × 10−5 T. The emf induced
(a) (b)
5 Rnt 5 Rt between the rim and the centre of the wheel will be
− (W2 − W1) n (W2 − W1)
(c) (d) − × × × × × × Rim
Rnt Rt
× × × × × × ×
3 A conducting circular loop is placed in a uniform × × × × × × ×
magnetic field B = 0.025 T with its plane perpendicular to × × × × × × ×
the loop. The radius of the loop is made to shrink at a
× × × × × × ×Spokes
constant rate of 1 mm s –1. The induced emf when the
× × × × × × ×
radius is 2 cm, is j
CBSE AIPMT 2010
π (a) 6.28 × 10−5 V (b) 4.8 × 10−5 V
(a) 2 π µV (b) πµV (c) µV (d) 2 µV (c) 6 × 10−5 V (d) 1.6 × 10−5 V
2
272 40 DAYS ~ NEET PHYSICS DAY TWENTY FOUR

6 A square metal wire loop PQRS of side 10 cm and 10 The current i in a coil varies with time as shown in the
resistance 1 Ω is moved with a constant velocity vc in a figure. The variation of induced emf with time would be
uniform magnetic field of induction B = 2 Wbm −2 , as j CBSE AIPMT 2011
shown in figure. The magnetic field lines are i
perpendicular to the plane of the loop (directed into the
paper). The loop is connected to net work ABCD of
resistors each of value 3 Ω. The resistance of the lead
0 t
wires SB and RD are negligible. The speed of the loop so T/4 T/2 3T/4 T
as to have a steady current of 1 mA in the loop is
j CBSE AIPMT 2010
B emf
emf
× × × ×
× ×P × × S 3Ω 3Ω
T/4 T/2 3T/2 T
3Ω
(b) 0 t
× × × × (a) 0 t
A C T/4 T/2 3T/2 T
× × × ×
× ×Q × × R 3Ω 3Ω emf emf
× × × ×
D
(a) 2 ms −1 (b) 2 × 10−2 ms −1 (d)
(c) T/4
(c) 20 ms −1 (d) 200 ms −1 0
T/2 3T/4 T
t 0 t
T/4 T/2 3T/4 T
7 A long solenoid of diameter 0.1 m has 2 × 104 turns per
metre. At the centre of the solenoid, a coil of 100 turns
and radius 0.01 m is placed with its axis coinciding with
the solenoid axis. The current in the solenoid reduces at a
11 In a uniform magnetic field of induction B, a wire in the
constant rate to 0 A from 4 A in 0.05 s. If the resistance of form of semi-circle of radius r rotates about the diameter of
the coil is10π 2 Ω, the total charge flowing through the coil the circuit with angular frequency ω. If the total resistance
during this time is j
NEET 2017 of the circuit is R, the mean power generated per rotation
(a) 32 π µC (b) 16 µC (c) 32 µC (d) 16π µC of rotation is
8 A uniform but time varying Bπr 2ω (Bπr 2ω)2
××
× ××× × × × r P (a) (b)
magnetic field B(t ) exists in a ×× × 2R 8R
× × × ×
circular region of radius a and is ×× × × × (Bπrω)2
× ×
× × × a
×× (c) (d) None of these
× 2R
directed into the plane of the × × × × ××
× × × × × ×
×
paper as shown in figure. The × × × × × 12 A loop made of straight edges has six corners at
× × × ×
magnitude of induced electric A ( 0, 0, 0), B (L , 0, 0), C (L , L , 0), D ( 0, L , 0), E ( 0, L , L ) and
field at point P at a distance r F ( 0, 0, L ). A magnetic field B = B0( $i + k$ ) T is present in the
from the centre of the circular region region. The flux passing through the loop ABCDEFA (in
(a) is zero (b) decreases as 1/ r that order) is
(c) increases as r (d) decreases as 1 / r 2 (a) B 0 L2 Wb (b) 2 B 0 L2 Wb

9 The current (I ) in the inductance is (c) 29 B 0 L2 Wb (d) 4 B 0 L2 Wb


varying with time according to the plot I 13 A conductor of length l and mass m can slide along a
shown in figure. j
CBSE AIPMT 2012 pair of vertical metal guides connected by a resistor R. A
Which one of the following is the T/2
uniform magnetic field of strength B normal to the plane
correct variation of voltage with time of page is directed outwards. The steady speed of fall of
t T
in the coil? rod is

V V
(a) (b) B
t
T/2 T T/2 T
t v

V V
(c) (d) mgR mg B 2l 2 mgB
(a) 2 2
(b) 2 2
(c) (d)
t B l B l R mgR l 2R
T/2 t T T/2 T
DAY TWENTY FOUR ELECTROMAGNETIC INDUCTION 273

14 A uniform magnetic field is restricted within a region of 15 A conducting square frame of x


radius r. The magnetic field changes with time at a rate side a and a long straight wire
dB I
. Loop 1 of radius R > r encloses the region r and loop carrying current I are located in
v
dt the same plane as shown in the
2 of radius R is outside the region of magnetic field as figure. The frame moves to the
shown in the figure. Then, the emf generated is j NEET 2016 right with a constant velocity v. a
The emf induced in the frame will
r be proportional to j CBSE AIPMT 2015
R R
1 1 1
1 2 (a) (b) (c)
x2 (2 x − a)2 (2 x + a)2
(a) zero in loop 1 and zero in loop 2 1
(d)
dB 2 dB 2 (2 x − a) (2 x + a)
(b) − πr in loop 1 and − πr in loop 2
dt dt 16 The magnetic potential energy stored in a certain
dB
(c) − πR 2 in loop 1 and zero in loop 2 inductor is 25 mJ, when the current in the inductor is
dt
dB 2 60 mA. This inductor is of inductance
(d) − πr in loop 1 and zero in loop 2
dt (a) 1.389 H (b) 138.88 H (c) 0.138 H (d) 13.89 H

ANSWERS
SESSION 1 1 (c) 2 (a) 3 (d) 4 (a) 5 (b) 6 (a) 7 (c) 8 (c) 9 (a) 10 (c)
11 (b) 12 (b) 13 (a) 14 (b) 15 (a) 16 (a) 17 (b) 18 (c) 19 (d) 20 (a)
21 (a) 22 (b) 23 (b) 24 (c) 25 (a) 26 (d) 27 (a) 28 (d) 29 (c) 30 (c)
31 (b) 32 (a)
SESSION 2 1 (b) 2 (b) 3 (b) 4 (b) 5 (a) 6 (b) 7 (c) 8 (b) 9 (d) 10 (d)
11 (b) 12 (b) 13 (a) 14 (c) 15 (d) 16 (d)

Hints and Explanations


− dφ − d
SESSION 1 ∴ e = = (10t 2 − 50t + 250) 8 The emf of coil, e = − dφ
dt dt dt
1 Here, A = L2 k
= − (20t − 50) e
and current, I =
and B = B 0 (2$i + 3$j + 4k$ ) At t = 3s, e = − (20 × 3 − 50) = − 10 V R
φ=? 5 e = − dφ = 24 t + 7 So, the current in the coil,
As, φ = B ⋅ A = B 0 (2$i + 3$j + 4k$ ) ⋅ L2 k$ dt 1 dφ −1 d NA dB
I = = (NBA ) = −
At t = 5s, we get e = 24 × 5 + 7 = 127 mV R dt R dt R dt
∴ φ = (4 B 0 L2 ) Wb
6 Induced emf in a coil is given by 20 × (25 × 10−4 )
=− × 1000
2 ∴ Induced emf  − dφ
E =  100
Work done in taking a charge q  dt = 0.5 A
once along the loop
= Given, φ = 50t 2 + 4 9 Induced emf,
charge (q ) dφ d
W and resistance, R = 400 Ω e =− =− (6t 2 − 5t + 1) = − 12t + 5
i.e. V= ⇒ W = Vq  − dφ
 = |100t|
dt dt
q So, E =  t = 2 = 200 V At t = −025
. s, e = − 12( 0.25) + 5 = 2 V
 dtt = 2
e 2
3 Given, φ = (3t 2 + 4t + 9) Wb Induced current, I = = = 0.2 A
So, current in the coil will be R 10
dφ E 200 1
∴ e = ∴ I = = = = 0.5 A BA(cos 0° − cos 90° )
dt R 400 2 10 ∴ q = dφ =
R R
d (3t 2 + 4t + 9) dφ d
= 7 e =− = (NAB cos ω t ) Bπr 2 (1 − 0) Bπr 2
dt dt dt = =
R R
= 6t + 4 or e = NAB ω sin ω t = e 0 sinω t 2 × 3.143 × (10−1 )2
At t = 2, we get e = 16 V ∴ e 0 = NAB ω =
0.01
4 Here, φ = 10t 2 − 50t + 250 = 6.286 C = 6.3 C
274 40 DAYS ~ NEET PHYSICS DAY TWENTY FOUR

11 From Faraday’s second law, emf Now, if r = 2 cm 30 QM max = L1 L2


induced in the circuit, e = − 0.04 × π × 2 × 2 × 10−2 × 2 × 10−3
∆φ Given, L1 = 2 mH, L2 = 8 mH
e= = 32
. π µV
∆t V a
M max = 2 × 8 = 16 = 4 mH
If R is the resistance of the circuit, then 21 ∫V de = − ∫0 Bx ω dx
e ∆φ
0
31 ∴ φ = MI
I= = V − V0 = −
1 2
Ba ω
R R∆t 2 Hence, dφ = M dI
Thus, charge passes through the circuit, 1 1 Here, dφ = 1.2 Wb, dI = 3 A
∆φ ∆φ = − × 10 × 5 × 2 π × = − 50 V
q = I × ∆t ⇒ q = × ∆t ⇒ q = 2 π Hence, M = 0.4 H
R∆t R
22 Flux will change two times per 32 The cause of production of eddy currents
12 As there is no change in magnetic flux revolution. is the motion of a conductor in a varying
associated with the circuit, no current is
magnetic field.
induced in the circuit. The ammeter A 23 Net induced emf due to disc is
shows no deflection. 1
e = Bω l 2 SESSION 2
13 Though emf is induced in the copper 2
(NBA cos θ − 0)
ring, but there is no induced current, 1
= × 01 . × 2 π × 10 × 01
. × 01
. 1 ∴ e = dφ =
because of cut in the ring. Hence, 2 dt t
nothing opposes the free fall of the = π × 10−2 V 1 × 50 × 25 × 10−4 cos 60° − 0
magnet. Therefore, a = g . =
0.2
dI
14 Magnetic flux through the loop is 24 e =L or L ∝ dt e = 3.12 × 10−3 V
upwards and it is increasing due to dt
increasing current along AB. Current L1 dt 5 2 The rate of change of flux or emf
∴ = 1 = = 100 : 1
induced in the loop should have L2 dt 2 50 × 10−3 induced in the coil is
magnetic flux in the downward dφ
e = −n
direction, so that to oppose the increase 25 Use, e = L dI dt
in flux. Therefore, current induced in dt Induced current,
the loop is clockwise. Here, e = 8 V, dI = 2 A − (−2 A ) e n dφ
I = =−
15 As coil A is moved closer to B, field due = 4 A, dt = 0.05 s R′ R ′ dt
to A intercepting B is increasing. Hence, L = 01 Given, R ′ = R + 4R = 5R,
. H
Induced current in B must oppose this
increase. Hence, the current in B must 26 The self-inductance of the solenoid is dφ = W2 − W1 , dt = t
n (W2 − W1 )
be clockwise. µ 0N2 A ∴ I =−
L = 5R t
16 In the rotation of magnet, N pole moves l
closer to coil CD and S pole moves When N ′ = 2N, then 3 Magnetic flux φ linked with magnetic
closer to coil AB. As per Lenz’s law, N µ (2N ) 2 A 4µ 0 N 2 A field B and area A is given by
pole should develop at the end L′ = 0 = = 4L
l l φ = B ⋅ A = |B||A|cos θ
corresponding to C. Induced current
flows from C to D. Again, S pole should Here, θ = 0°
Hence, when number of turns is
develop at the end corresponding to B. doubled, then self-inductance becomes So, φ = BA = Bπr 2
Therefore, induced current in the coil quadruple. Now, induced emf, |e|
flows from A to B. − dφ dr
17 Polarity of emf will be opposite in the 27 ∴ e = L dI = 2 × 10−3 × 10 − (−−23) = 8 V =
dt
= Bπ (2 r )
dt
dt 3 × 10
two cases, while entering and while
= 0.025 × π × 2 × 2 × 10−2 × 1 × 10− 3
leaving the coil. Only in option (b) 28 From the relation, L = µ 0 n2 lA
polarity is changing. = π µV
N2 Q n = N 
18 First current develops in direction of ⇒ L = µ0 A  4 Area coming out per second from the
abcd, but when electron moves away, l l 
magnetic field is not constant for
then magnetic field inside loop ⇒ L ∝ N2 elliptical and circular loops, so induced
decreases and current changes its Thus, inductance of a coil is directly emf, during the passage out of these
direction. proportional to square of the number of loops, from the field region will not
19 For emf, e = Bv (Leff ) = Bv × (2 r ) = 2Bvr turns. remain constant.
5 Induced emf is e 0 = Ba ω
2
[QLeff = diameter = 2r ] 29 Given, number of turns of solenoid,
R will be at higher potential, we can find N = 1000 2
it by using right hand rule. Current, I = 4 A where, a is length of each spoke.
120
20 Magnetic field, B = 0.04 T and rate of Magnetic flux, φ B = 4 × 10−3 Wb n= =2
60
change of radius of coil due to shrinkage, QSelf-inductance of solenoid is given by rad
− dr φ ⋅N ∴ ω = 2 πn = 4 π
= 2 mm s −1 L = B …(i) s
dt I ⇒ a = 0.5m
− dφ dA
Induced emf, e = =−B Substitute the given values in Eq. (i), we Ba2ω
dt dt e =
get 2
d ( πr 2 ) dr 4 × 10−3 × 1000
=−B = − Bπ2 r L = =1H = 6.28 × 10−5 V
dt dt 4
DAY TWENTY FOUR ELECTROMAGNETIC INDUCTION 275

6 Wheatstone bridge is balanced. Current T 3T di where, φ = BA = πR2 B


For to , = constant
through AC is zero. Effective resistance 2 4 dt dφ dB
So, e = positive ⇒ = − πR 2
R of bridge is dt dt
1 1 1 1
= + = ⇒R =3Ω 11 Qφ = BA cos θ Rate of change of magnetic flux
R 6 6 3 associated with loop 1,
Total resistance = 1 + 3 = 4 Ω
1
=Bπr 2 cos ω t Q A = 1 πr 2 
  dφ dB
Induced emf, e = IR = Blv 2 2 e 1 = − 1 = − πR2
dt dt
IR 1 × 10−3 × 4 dφ d 1 
∴ v = = e induced = − =−  Bπr cos ω t 
2
Similarly, e 2 = emf associated with loop 2
2 × 0.1 dt dt  2 
Bl dφ
1 =− 2=0 [Q φ 2 = 0]
= 2 × 10−2 ms −1 = Bπr 2ω sin ωt dt
2
7 Given, resistance of the solenoid, e 2induced B 2 π2 r 4ω2 sin2 ωt 15 x
∴ ρ= =
R = 10 π2 Ω R 4R P S
Radius of second coil, r = 10−2 m B 2 π2 r 4ω2 1
Hence, ρmean = < p > = x
∆t = 0.05 s, ∆i = 4 − 0 = 4A 4R 2 I B v
Charge flowing through the coil is given (Bπr 2ω)2
=
by 8R Q R
∆φ 1
∆q =   ( ∆t ) 12 Here, B = B 0 ( $i + k$ ) a
 ∆t  R
Area of vector of ABCD = L2 k$ Potential difference across PQ is
∆i 1
= µ 0 N 1 N 2 πr 2   ∆t V P − VQ = B1 (a)v =
µ 0I
 ∆t  R Area of vector of DEFA = L2 $i av

2 π  x − 
a
= 4 π × 10−7 × 2 × 104 × 100 × π Total area of vector, A = L2 (i$ + k$ )  2
Y
× (10−2 )2 × 
4  1 Potential difference across side RS of
× × 0.05 frame is
 0.05 10 π2
µ 0I
= 32 × 10−6 C = 32 µC VS − V R = B2 (a)v = av
2 π  x + 
 a
8 For r ≥ a, ∫ E ⋅ dl D (0, L, 0) C (L, L, 0)  2
∧ Hence, the net potential difference in
k the loop will be
(0, L, L) ∧
E E i V net = (V P − VQ ) − (VS − V R )
X  
a dB
A (0, 0, 0) B (L, 0, 0) µ 0 Iav  1 1 
=  − 
2π  
x −   x + a 
2 dt a

   
r=B r F (0, 0, L) 2  2  
 dφ 
⇒ = A   ⇒ E (2 πr ) = πa2 
 dB   dB 
 Z  
 dt   dt   dt   
Total magnetic flux, µ 0 Iav  a 
=
a2 
 dB 
 φ = B ⋅ A = B 0 ( $i + k$ ) ⋅ L2 ($i + k$ ) 2π   a  a 
E =   x −  x + 
2r  dt   2  2 
= B 0 L2 (1 + 1) = 2 B 0 L2 Wb
1 1
∴ Induced electric field, E ∝ . Thus, V net ∝
r 13 Induced emf, e = − dφ, I = e = Blv (2 x − a)(2 x + a)
9 For inductor, as we know induced dt R R
voltage for t = 0 to t = T /2, 16 Given, magnetic potential energy stored
dI d  2I0 t  in an inductor,
V =L =L R
  = constant U = 25 mJ = 25 × 10−3 J
dt dt  T  B
For t = T /2 to t = T , F Current in an inductor, I 0 = 60 mA
b = 60 × 10−3 A
dI  −2 I 0 t  I θ
V =L =  = − constant As, the expression for energy stored in
dt  T 
mg an inductor is given as
So, answer can be represented with 1
graph (d). Resultant magnetic force, U = LI 20
2
B 2 l2 v
10 As we know that, F m = IlB . ⇒ F m = where, L is the inductance of the
di R inductor.
emf induced, e = − L
dt and is directed upwards, Substituting the given values in above
T di F m = mg equation., we get
During 0 to , = constant 1
4 dt B 2 l2 v mgR (25 × 10−3 ) = × L × (60 × 10−3 )2
⇒ Fm = = mg ⇒ v = 2
So, e = negative R B 2 l2
T T di 2 × 25 × 10−3 500
For to , =0 ⇒ L = =
4 2 dt 14 Induced emf in the region is given by 3600 × 10−6 36

So, e =0 |e | = or L = 13.89 H
dt
DAY TWENTY FIVE

Alternating
Current
Learning & Revision for the Day
u Peak and RMS Values of u Quality Factor u Choke Coil
Alternating Current/Voltage u L-C Oscillatons u AC Generator
u Different Types of AC Circuits u Power in an AC Circuit u Transformer
u Series AC Circuits u Wattless Current

An alternating current is the current (or voltage) whose magnitude keeps on changing
continuously with time, between zero and a maximum value and its direction also
reverses periodically.
I0/V0
VI

t
0
T T 3T T
4 2 4

Peak and RMS Values of


Alternating Current/ Voltage PREP
Mean Value or Average Value or Peak Value MIRROR
Your Personal Preparation Indicator
The steady current, which when passes through a circuit for half the time period of
alternating current, sends the same amount of charge as done by the alternating current u No. of Questions in Exercises (x)—
in the same time through the same circuit, is called mean or average value of alternating u No. of Questions Attempted (y)—
current. It is denoted by i m or i av u No. of Correct Questions (z)—
(Without referring Explanations)
2i
Thus, i m or i av = 0 = 0.636 i 0
π u Accuracy Level (z / y × 100)—
Thus, mean or average value of alternating current during a half cycle is 0.636 times (or u Prep Level (z / x × 100)—
63.6% of) its peak value (i 0). Similarly, mean or average value of alternating emf
In order to expect good rank in NEET, your
2V
Vm or Vav = 0 = 0.636 v0 Accuracy Level should be above 85 & Prep
π Level should be above 75.
DAY TWENTY FIVE ALTERNATING CURRENT 277

RMS Value 2. Pure Inductive Circuit


The steady current, which when passes through a resistance Let an alternating voltage V = V0 sin ωt be applied across a
for a given time will produce the same amount of heat as the pure inductance L.
alternating current does in the same resistance and in the π
same time, is called rms value of alternating current. It is Then, the average power = Vrms I rms cos = 0
i 2
denoted by i rms or i v = 0 = 0.707 i 0
2 The inductance offers some opposition to the flow of AC,
where, i 0 = peak value of alternating current known as inductive reactance X L = 2πνL = Lω.
Similarly, rms value of alternating emf Thus, a pure inductance does not oppose the flow of DC
V (ω = 0) but opposes the flow of AC.
V rms = 0 = 0.707 V0 V
2 Current flowing, I =
XL

Reactance and Impedance V


I
XL
l
The opposition offered by a pure inductor or capacitor or
both to the flow of AC, through it, is called reactance (X ).
(a) L (b) (c)
Its unit is ohm (Ω) and dimensional formula is
π
[ML 2 T−3 A −2 ]. 2
V = V0 sin ωt 0 I 0 V
l
Reactance is of two types
(i) Inductive reactance, X L = Lω and
1 In pure inductive circuit, current decreases with an
(ii) Capacitive reactance, X C = π
Cω increase in frequency, it lags behind the voltage by
2
l
Reciprocal of reactance is known as susceptance. π
1 (or voltage leads the current by ) and is thus given by
Thus, S = 2
X  π
I = I 0 sin ωt − 
l
Total opposition offered by an AC circuit to the flow of  2
current through it, is called its impedance (Z). Its unit is
ohm and dimensional formula is [ML 2 T −3 A −2 ]. 3. Pure Capacitive Circuit
For an AC circuit, Z = X + R = (X L − X C ) + R
2 2 2 2
Let an alternating voltage V = V0 sin ωt be applied across a
pure capacitance C. Then, the capacitance offers some
l
Reciprocal of impedance is known as admittance. opposition to the flow of current, but allows AC to pass
1
Thus, Y = . Its unit is Siemens (S). through it. The opposition offered is known as the
Z capacitive reactance.
1
XC = Ω

Different Types of AC Circuits 1
= Ω
The circuit consists of resistor, capacitor and inductor are called C × 2 πν
pure resistive, pure inductive and pure capacitive circuit.
V
Current flowing, I =
XC
1. Pure Resistive Circuit XC
I
Let an alternating voltage V = V0 sin ωt be applied across a π
pure resistance R. Then, 2
V V (a) C (b) (c)
Current, I = or I rms = rms
R R
V = V0 sin ωt V
V
Irms

R I
0 V In pure capacitive circuit, current increases with an
π
increase in frequency and leads the voltage by (or
V = V0 sin wt 2
V π
(a) (b) (c) voltage lags behind the current by ) and is thus, given by
2
Current and voltage are in the same phase, i.e. current is π

given by I = I 0 sin ωt . I = I 0 sin ωt + 
 2
278 40 DAYS ~ NEET PHYSICS DAY TWENTY FIVE

Series AC Circuits 3. Series L-C Circuit


Some of the series AC circuits are given below The potential difference across a capacitor in AC lags in
phase by 90° and leads in phase by 90° by inductance
with the current in the circuit.
1. Series L-R Circuit
E0
The potential difference across the resistance in an AC E = E 0 sin ωt , I = sin(ωt − φ)
circuit is in phase with current and it leads in phase by Z
90° with current across the inductor. X L − XC
where, Z = X L − X C and tan φ = =∞
L R 0
VL V L C VL
VL VR
φ VL – VC
VR I
I
E = E0 sin ωt
E = E0 sin ωt
E VC
E = E 0 sin ωt and I = 0 sin(ωt − φ)
Z π π
For X L > X C , φ = and for X L < X C , φ = −
2 2
where, Z = R2 + (ωL)2 1
If X L = X C i.e. at ω = , Z = 0 and I 0 becomes infinity.
Current lags behind the voltage by φ. LC
ωL This condition is termed as the resonant condition and
and tan φ = this frequency is termed as natural frequency of the
R circuit.
∴ V = VR2 + VL2 I Z

where, VR = voltage across resistor R


and VL = voltage across inductor.

ω ω
2. Series R-C Circuit ω0 ω0
Variation of I with Variation of Z with
The potential difference across a resistance in AC circuit
ω in L-C circuit ω in L-C circuit
is in phase with current and it lags in phase by 90° with
the current in the capacitor.
E = E 0 sin ωt 4. Series L-C-R Circuit
E E0
and I = 0 sin (ωt + φ) For L-C-R circuit E = E 0 sin ωt , I = sin (ωt − φ)
Z Z
2 2
 1   1 
where, Z = R2 +   where, Z = R2 + ωL − 
 ωC   ωC 
X X − XC
Current leads the voltage by φ. and tan φ = = L
R R
−1 / ωC
and tan φ = VL
R
C R L C R VL – VC
VR
I V
VC VR φ
φ
I
V VR
VC
E = E0 sin ωt E = E0 sin ωt
VC
Phasor diagram for L-C-R
∴ V = VR2 + VC2 series circuit for XL>XC

where, VR = voltage across resistor R For X L > X C , current lags voltage.


and VC = voltage across capacitor. X L < X C , current leads voltage.
X L = X C , current and voltage are in phase.
DAY TWENTY FIVE ALTERNATING CURRENT 279

1 For a pure resistive circuit, V and I are in phase


X L = XC ⇒ ω 0 =
l
If , i.e. the natural frequency of the
LC (φ = 0 °), hence cos φ = 1 and average power = Vrms I rms
circuit is equal to the applied frequency, then the circuit is For a pure inductive or a pure capacitive circuit, current
said to be in resonance. π π
and voltage differ in phase by  i.e. φ =  .
At resonance, the current in the circuit is maximum and 2  2
the impedance is minimum. V2
1 l
Power loss = I 2 R =
Resonance frequency, ν = R
2 π LC
Wattless Current
Quality Factor Average power is given by Pav = E rms I rms cos φ
The phase difference between E rms and I rms is φ. We can
The Q-factor or quality factor of a resonant L-C-R circuit is
defined as ratio of the voltage drop across inductor resolve I rms into two components
(or capacitor) to applied voltage. Thus, I rms cos φ and I rms sin φ
voltage across L (or C) Here, the component I rms cos φ contributes towards power
Q= dissipation and the component I rms sin φ does not contribute
applied voltage
towards power dissipation. Therefore, it is called wattless
1 L current.
Q=
R C
Choke Coil
A low resistance inductor coil used to suppress or limit the
L-C Oscillations flow of alternating current without affecting the flow of direct
I
An L-C circuit also called a resonant circuit, current is called choke coil.
tank circuit or tuned circuit. When Let us consider a choke coil (used in tube lights) of large
connected together, they can act as an L C inductance L and low resistance R. The power factor for such
electrical resonator, storing energy a coil is given by
oscillating at the circuits resonant R R
frequency. cos φ = ≈ [as, R << ωL]
R2 + ω2 L2 ωL
The energy oscillates back and forth between the capacitor
and inductor until internal resistance makes the oscillations As R << ωL, cos φ is very small. Thus, the power absorbed by
die out. The oscillation frequency is determined by the the coil V rms I rms cos φ is very small. On account of its large
capacitance and inductance values,
ω 1 impedance Z = R2 + ω2 L2 , the current passing through the
f = 0 = coil is very small. Such a coil is used in AC circuits for the
2 π 2 π LC
purpose of adjusting current to any required value without
wastage of energy.
Power in an AC Circuit
The only loss of energy is due to hysteresis in the iron core,
Let a voltage V = V0 sin ωt be applied across an AC and which is much less than the loss of energy in the resistance
consequently a current I = I 0 sin(ωt − φ) flows through the that can also reduce the current, if placed instead of the
circuit. Then,
choke coil.
l
Instantaneous power = V I = V0I 0 sin ωt sin(ωt − φ)
and its value varies with time. Here, φ is known as phase AC Generator
difference between V and I.
An electric generator or dynamo is a device used to produce
l
Average power over a full cycle of AC is electrical energy at the expense of mechanical/thermal energy.
1
Pav = Vrms I rms cos φ = V0I 0 cos φ It works on the principle of electromagnetic induction, when
2 a coil is rotated in a uniform magnetic field, an induced emf
The term Vrms I rms is known as the apparent or virtual is set up between its ends. The induced emf is given by
power, but Vrms I rms cos φ is called the true power. e = e 0 sin ωt = NBAω sin ωt .
l
The term cos φ is known as the power factor of the given The direction of the induced emf is alternating in nature.
circuit. Thus,
R
cos φ = = power factor
Z Transformer
true power
= It is a device which works in AC circuits only and is based on
apparent power the principle of mutual induction.
280 40 DAYS ~ NEET PHYSICS DAY TWENTY FIVE

Transformer is used to suitably increase or decrease the voltage The efficiency of a transformer is given by
in an AC circuit. Transformer which transforms strong AC at output power Vs I s
low voltage into a weaker current at high alternating voltage is η= =
input power V p I p
called a step-up transformer. A step-down transformer
transforms weak current at a higher alternating voltage into a For an ideal transformer, η = 100% or 1. However, for practical
strong current at a lower alternating voltage. transformers, η ≈ 85 - 90%.
e V N Ip Possible causes of energy loss in transformer are
For an ideal transformer s = s = s = =k
ep Vp Np Is l
Heating due to winding resistance
where, k is known as the transformation ratio. l
Eddy current losses
For a step-up transformer, k > 1 but for a step-down l
Magnetic flux leakage and
transformer, k < 1. l
Hysteresis loss. To minimise these losses, the transformer
In a transformer, the input emf and the output emf differ in core is made up of a laminated soft iron strips.
phase by π radians.

DAY PRACTICE SESSION 1

FOUNDATION QUESTIONS EXERCISE


1 The electric current in a circuit is given by I = 3 t π π π
(a) rad (b) rad (c) zero (d) rad
4 2 6
Here, t is in second, I is in ampere. The rms current for
the period t = 0 to t = 1 s is 8 A 100 Ω resistance and a capacitor of 100 Ω reactance
(a) 3 A (b) 9 A (c) 3 A (d) 3 3 A are connected in series across a 220 V source. When the
capacitor is 50% charged, the peak value of the
2 If the rms current in a 50 Hz AC circuit is 5 A, the value of
displacement current is j
NEET 2016
the current 1/300 s after its value becomes zero is
(a) 2.2 A (b) 11 A (c) 4.4 A (d) 11 2 A
(a) 5 2 A (b) 5 3 / 2 A (c) 5 / 6 A (d) 5 / 2 A
9 In an L-C-R series circuit, the potential difference
3 Through which of the AC circuit elements, both the emf
between the terminals of the inductance is 60 V, between
and current are in phase?
the terminals of the capacitor is 30 V and that across the
(a)Impedance (b) Inductive reactance
(c)Capacitive reactance (d) Resistance resistance is 40 V. Then, supply voltage will be equal to
(a) 50 V (b) 70 V (c) 130 V (d) 10 V
4 The electric mains in the house is marked 220 V, 50 Hz.
Write down the equation for instantaneous voltage. 10 In the given circuit, the reading of voltmeterV1 and V2 are
(a) 3.1V sin (100π) t (b) 31.1V cos (100π) t
300 V each. The reading to the voltmeterV3 and ammeter
(c) 311.1V sin (100π) t (d) 311.1V cos (100π) t A are respectively j
CBSE AIPMT 2010
L C R = 100 Ω
5 In an AC circuit, an alternating voltage e = 200 2 sin 100 t
volt is connected to a capacitor of capacity 1 µF. The rms
value of the current in the circuit is j
CBSE AIPMT 2011 V1 V2 V3
(a) 100 mA (b) 200 mA (c) 20 mA (d) 10 mA A

6 If emf E = 4 cos 1000 t volt is applied to an L-R circuit


of inductance 3 mH and resistance 4 Ω, the amplitude 220 V, 50 Hz
of current in the circuit is
(a) 150 V, 2.2 A (b) 220 V, 2.2 A
4
(a) A (b) 10
. A (c) 220 V, 2.0 A (d) 100 V, 2.0 A
7
4
(c) A (d) 0.8 A
11 What is the value of inductance L for which the current is
7 maximum in a series L - C - R circuit with C = 10 µF and
7 An AC voltage is applied to a resistance R and an ω = 1000 s −1 ?
inductor L in series. If R and the inductive reactance are (a) 100 mH
(b) 1 mH
both equal to 3Ω, the phase difference between the
(c) Cannot be calculated unless R is known
applied voltage and the current in the circuit is (d) 10 mH
j
CBSE AIPMT 2011
DAY TWENTY FIVE ALTERNATING CURRENT 281

12 An AC circuit contains a resistance R, capacitance C and 20 The instantaneous values of alternating current and
inductance L in series with a source of emf voltages in a circuit are given as
E = E 0 sin (ωt + φ ). The current through the circuit is 1
maximum, when I= sin (100 πt ) ampere
2
1
(a) R = L = C (b) ω L = (c) ω2 = LC (d) ω = RLC
ωC e=
1
sin (100πt + π / 3) volt
2
13 The bandwidth of a series resonant circuit is 500 Hz and
the resonant frequency is 5000 Hz. The quality factor of The average power (in watts) consumed in the circuit is
the circuit will be j CBSE AIPMT 2013
(a) 40 (b) 20 (c) 10 (d) 5 1 3
(a) (b)
14 Which of the following combinations should be selected 4 4
1 1
for better tuning of an L-C-R circuit used for (c) (d)
2 8
communication? j NEET 2016
(a) R = 20Ω, L = 15. H, C = 35 µF 21 The potential differences across the resistance,
(b) R = 25 Ω, L = 2.5 H, C = 45 µF capacitance and inductance are 80 V, 40 V and 100 V
(c) R = 15Ω ,L = 3.5 H, C = 30 µF
respectively in an L-C-R circuit. The power factor of this
(d) R = 25 Ω , L = 15
. H, C = 45 µF
circuit is j NEET 2016

15 A condenser of capacitance of 2.4 µF is used in (a) 0.4 (b) 0.5


transmitter to transmit at λ wavelength. If the inductor of (c) 0.8 (d) 1.0
10−8 H is used for resonant circuit, then value of λ is
22 An AC ammeter is used to measure current in a circuit.
(a) 292 m (b) 400 m (c) 334 m (d) 446 m
When a given direct current passes through the circuit,
16 A transmitter transmits at a wavelength of 300 m. A the AC ammeter reads 3 A. When another alternating
condenser of capacitance 2.4 µF is being used. current passes through the circuit, the AC ammeter reads
The value of the inductance for the resonant circuit is 4A. Then, the reading of this ammeter, if DC and AC flow
approximately through the circuit simultaneously, is
(a) 10−4 H (b) 10−6 H (c) 10−8 H (d) 10−10 H (a) 3 A (b) 4 A
(c) 7 A (d) 5 A
17 By what percentage, the impedance in AC series circuit
should be increased, so that the power factor changes 23 An inductor of reactance 1 Ω and a resistor of 2 Ω are
from (1 / 2) to (1 / 4) (when R is constant)? connected in series to the terminals of a 6 V (rms) AC
(a)200% (b) 100% (c) 50% (d) 400% source. The power dissipated in the circuit is
18 A small signal voltage V (t ) = V0 sin ωt is applied across (a) 8 W (b) 12 W
(c) 14.4 W (d) 18 W
an ideal capacitor C j
NEET 2016
(a) over a full cycle the capacitor C does not consume any 24 A transformer having efficiency of 90% is working on
energy from the voltage source 200 V and 3 kW powers supply. If the current in the
(b) current I(t) is in phase with voltage V(t) secondary coil is 6 A, the voltage across the secondary
(c) current I(t), Ieads voltage V(t) by 180° coil and the current in the primary coil respectively are
(d) current I(t), lags voltage V(t) by 90° CBSE AIPMT 2014
j

(a) 300 V, 15 A (b) 450 V, 15 A


19 Power dissipated in an L-C-R series circuit connected to
(c) 450 V, 13.5 A (d) 600 V, 15 A
an AC source of emf ε is j
CBSE AIPMT 2009
εR
2
εR
2 25 A 220 V input is supplied to a transformer. The output
(a) (b) circuit draws a current of 2.0 A at 440 V. If the efficiency of
 2  Lω − 1  
2 2
R +  Lω −
1 
R +
2
    the transformer is 80%, the current drawn by the primary
  Cω    Cω  windings of the transformer is j
CBSE AIPMT 2010
 1  
2
+  Lω −
2
ε2 R 2 +  Lω −
ε2 R 2   1  (a) 3.6 A (b) 2.8 A
 Cω   
  Cω  (c) 2.5 A (d) 5.0 A
(c)  (d)
R R
282 40 DAYS ~ NEET PHYSICS DAY TWENTY FIVE

DAY PRACTICE SESSION 2

PROGRESSIVE QUESTIONS EXERCISE


1 An alternating voltageV = 30 sin 50 t + 40 cos 50 t is 8 In an electrical circuit R , L , C and an AC voltage source
applied to a resistor of resistance10 Ω. The rms value of are all connected in series. When L is removed from the
current through resistor is circuit, the phase difference between the voltage and the
5 10 7 π
(a) A (b) A (c) A (d) 7A current in the circuit is . If instead, C is removed from
2 2 2 3
π
2 One 10 V, bulb of 60 W is to be connected to 100 V line. the circuit, the phase difference is again . The power
3
The required self-inductance of induction coil will be
factor of the circuit is j CBSE AIPMT 2012

(Take, f = 50 Hz) 1 1 3
(a) 0.052 H (b) 2.42 H (c) 16.2 H (d) 16.2 mH (a) (b) (c) 1 (d)
2 2 2
3 An AC source is connected with a resistance R and an
uncharged capacitance C in series. 9 A coil of self-inductance L is connected in series with a
bulb B and an AC source. Brightness of the bulb
The potential difference across the resistor is in phase decreases when j NEET 2013

with the initial potential difference across the capacitor (a) frequency of the AC source is decreased
for the first time at the instant (assume that, at t = 0, emf (b) number of turns in the coil is reduced
is zero) (c) a capacitance of reactance XC = XL is included in the
π 2π π 3π same circuit
(a) (b) (c) (d)
4ω ω 2ω 2ω (d) an iron rod is inserted in the coil
10 An inductor 20 mH, a capacitor 50 µF and a resistor 40 Ω
4 An AC circuit having supply voltage E consists of a
are connected in series across a source of emf
resistor of resistance 3 Ω and an inductor of reactance
V = 10 sin 340 t. The power loss in AC circuit is
4 Ω as shown in the figure. The voltage across the
j
NEET 2016
resistance at t = π / ω is
(a) 0.67 W (b) 0.76 W (c) 0.89 W (d) 0.51 W
3Ω 4Ω
11 In a circuit, L , C and R are connected in series with an
alternating voltage source of frequency f . The current
leads the voltage by 45°. The value of C is
E = 10 sin ω t 1 1
(a) (b)
(a) 6.4 V (b) 10 V (c) zero (d) 4.8 V 2 πf (2 πfL + R) πf (2 πfL + R)
5 A coil of inductive reactance 31 Ω has a resistance of 1 1
(c) (d)
8 Ω. It is placed in series with a condenser of 2 πf (2 πfL − R) πf (2 πfL − R)
capacitative reactance 25 Ω. The combination is 12 In the given AC circuit, when switch S is at position 1,
connected to an AC source of 110 V. The power factor π
the source emf leads current by . Now, if the switch is
of the circuit is 6
(a) 0.56 (b) 0.64 (c) 0.80 (d) 0.33 at position 2, then
6 The maximum current in the circuit, if a capacitor of L = √ 3 mH
1
capacitance 1 µF is charged to a potential of 2 V and is S
connected in parallel to an inductor of inductance10−3 H is R

(a) 4000 mA (b) 2000 mA


(c) 1000 mA (d) 5000 mA 2
1000 µF
C=
7 A resistance R draws power P when connected to an AC 3
source. If an inductance is now placed in series with the
V = V0 sin 1000 t
resistance, such that the impedance of the circuit
becomes Z, the power drawn will be j CBSE AIPMT 2015 π
(a) current leads source emf by
2 3
(a) P0   (c) P0  
R R R π
(b) P0 (d) P0 (b) current leads source emf by
Z Z Z
4
DAY TWENTY FIVE ALTERNATING CURRENT 283

π battery with emf ε = 18 V. The current i through the battery


(c) source emf lead current by
4 just after the switch closed is j NEET 2017

(d) None of the above


13 As given in the figure, a series circuit connected across L R R
a 200 V, 60 Hz line consists of a capacitor of capacitive +
ε –
reactance 30 Ω, a non-inductive resistor of 44Ω, and a
R L
coil of inductive reactance 90Ω and resistance 36 Ω. The C
power dissipated in the coil is
XC= 30 Ω (a) 2 mA (b) 0.2 A (c) 2 A (d) 0 A
15 A series R-C circuit is connected to an alternating voltage
200 V
source. Consider two situations : j CBSE AIPMT 2015

R1= 44 Ω 1. When capacitor is air filled.


60 Hz 2. When capacitor is mica filled.
R2= 36 Ω Current through resistor is i and voltage across
capacitor is V , then
XL= 90 Ω (a)Va < Vb (b)Va > Vb (c) ia > ib (d)Va = Vb

(a) 320 W (b) 176 W 16 An inductor 20 mH, a capacitor 100 µF and a resistor
(c) 144 W (d) 0 W 50 Ω are connected in series across a source of emf,
V = 10 sin 314 t . The power loss in the circuit is
14 Figure shows a circuit that contains three identical
j
NEET 2018
resistors with resistance R = 9.0 Ω each, two identical
(a) 2.74 W (b) 0.43 W (c) 0.79 W (d) 1.13 W
inductors with inductance L = 2 .0 mH each, and an ideal

ANSWERS
SESSION 1 1 (c) 2 (b) 3 (d) 4 (a) 5 (c) 6 (d) 7 (a) 8 (a) 9 (a) 10 (b)
11 (a) 12 (b) 13 (c) 14 (c) 15 (a) 16 (c) 17 (b) 18 (a) 19 (a) 20 (d)
21 (c) 22 (d) 23 (c) 24 (b) 25 (d)

SESSION 2 1 (a) 2 (a) 3 (d) 4 (d) 5 (c) 6 (a) 7 (a) 8 (c) 9 (d) 10 (d)
11 (c) 12 (b) 13 (a) 14 (*) 15 (b) 16 (c)

Hints and Explanations


SESSION 1 3 Through the resistance in the AC circuit, and C = 1 µF = 1 × 10−6 F
1 Given, I = 3t ⇒ I = 9t 2 2 both the emf and current are in phase. As e rms = 200V and ω = 100 s −1
1 1
1 4 We are given that, ∴ XC = = = 104 Ω
∫0 9t dt
2
εrms = 220 V, ν = 50 Hz ω C 100 × 10−6
(I 2 )0 −1 = =3
1
ε0 e 200
= 2 × 10−2 A
∫0 dt As, εrms = , ε0 = εrms 2 ∴ i rms = rms =
XC 104
2
∴ I rms = 3 A = 20 mA
(220 V) (1.414) = 311.1 V
2 Here, ν = 50 Hz, I V = 5A, I = ?, t = 1 s Further, 6 Impedance, Z = R2 + X 2L
300 ω = 2 πν = 2 π × 50 = 100 π rad/s
Here, R = 4 Ω, X L = Lω
I 0 = 2I V = 2 × 5A Thus, the equation for the instantaneous
voltage is given as = 3 × 10−3 × 1000 Ω = 3 Ω
From I = I 0 sin ωt ε = ε0 sin ωt = 311.1 V sin (100 π ) t Then, Z = (4)2 + (3)2
1
= 5 2 sin 100 π × 5 We know that, e = e m sinω t = 16 + 9 = 25 = 5 Ω
300
3 where, e m = e rms E0 4
= 5 2× = 5 3/2 A Hence, current, I 0 = = = 0.8 A
2 Given, emf, e = 200 2 sin 100 t Z 5
284 40 DAYS ~ NEET PHYSICS DAY TWENTY FIVE

XL Lω 3 Ω 4 π2 × (3 × 108 )2 × 2.4 21 Power factor of an L-C-R circuit


7 tan φ = = = ⇒ λ=
R R 3Ω × 10−6 × 10−8 R
= cos φ =
−1 π Z
tan φ = 1 ⇒ φ = tan (1) = 45° = rad ⇒ λ = 292 m
4 IR 80
= =
16 Given, λ = 300 m, c = 3 × 108 m/s IZ
8 Impedence of the R-C circuit, I ( X L − X C )2 + R2
c 3 × 108
Z = R2 + X C2 ∴Frequency, n = = = 106 Hz =
80
λ 300
(IX L − IX C )2 + (IR )2
where, R = 100 Ω and X C = 100 Ω Resonance frequency , n =
1
80
⇒ Z = (100)2 + (100)2 = 100 2 Ω 2π LC =
1 (100 − 40)2 + (80) 2
Peak value of the current, or LC =
2πn 80 80
V
I max = max =
220 2
= 2.2 A = = = 0.8
1 100
Z 100 2 or LC = (60) + (80)
2 2
4 π2 n2
9 In an L-C-R series circuit, Here, C = 2.4 × 10−6 F 22 Quantity of heat liberated in the
1 ammeter of resistance R
V = V R2 + (V L − VC ) = 10−8 H
2
∴L =
4 π2 (106 )2 × 2.4 × 10−6 (i) due to direct current of 3 A
= (40)2 + (60 − 30)2  R
=  (3)2 
17 Power factor = R  J
= 1600 + 900 = 2500 = 50 V Z
1 (ii) due to alternating current of 4 A
10 For series L-C-R circuit, Pi = (∴ Z = 2R )
2  R
= (4)2
Voltage, V = V R2 + (V L − VC )2 1  J 
Pf = (∴ Z = 4 R )
Since, V L = VC 4 Total heat produced per second
Hence, V = V R = 220 V The percentage of the impedance will be (3)2 R (4)2 R 25R
100%. = + =
Also, current, J J J
V 220 18 For an AC circuit containing capacitor
i = = = 2.2 A Let the equivalent alternating current
R 100 only, the phase difference between be I virtual ampere, then
π
11 If X L = XC current and voltage will be (i.e. 90°). I 2 R 25R
2 = or I = 5A
J J
This happens in resonance state of the Hence, power in this case is given by
circuit. P = VI cos φ 23 Here, X L = 1 Ω, R = 2 Ω, E V = 6 V, P = ?
i.e. ωL =
1
or L =
1
…(i) (where, φ = phase difference between
ωC ∴ Z = X 2L + R2 = 12 + 22 = 5Ω
ω2C voltage and current)
Given, P = VI cos 90° = 0 EV 6
∴ lV = = A
ω = 1000s −1 , C = 10 µF = 10 × 10−6 F 19 Power dissipated in series L-C-R, Z 5

Hence, L =
1 P = I rms
2
R ∴ P = E V l V cos θ
(1000)2 × 10 × 10−6 ε2rms R ε R
2 6 2
= = = 6× ×
= 0.1 H = 100 mH  2  1  
2 2 5 5
|Z |
 R +  ωL −  
12 When ω L = 1 /ωC . The circuit is in   ωC   = 14.4 W
resonance. Impedance is equal to
 As, I V rms  24 Power output = 3k W × 90 = 2.7 kW
resistance alone. rms =
 Z  100
fr 5000 Ib = 6 A
13 Q = = = 10
20 Given equation,
f2 − f1 500 2.7 kW
1 VS = = 450 V
I = sin(100 πt ) ampere 6A
14 For better tuning, peak of current growth 2
must be sharp. This is ensured by a high 3 kW
1 IP = = 15 A
value of quality factor Q. and e = sin (100 πt + π / 3) volt 200 V
2
Now, quality factor is given by 1 1
∴ I0 = and e 0 = 25 Efficiency is defined as the ratio of
1 L
Q = 2 2 output power and input power,
R C P
We know that, average power, i.e. η % = out × 100
From the given options, highest value of Pinput
Pav = V rms × I rms cos φ
Q is associated with R = 15 Ω, L = 3.5 H
1 1 Vs is
and C = 30 µF = × × cos 60° = × 100
2 2 Vp i p
1
15 Resonant frequency, n =  I V 
2π LC Q I rms = 0 and V rms = 0 2 × 440
 2 2  80 = × 100
1 λ2 220 × i p
LC = ⇒C = 1 1 1 1
= × × = W
4 π2 n2 4 π2c 2 L 2 2 2 8 ⇒ i p = 5A
λ= 4 π2 c2 LC
DAY TWENTY FIVE ALTERNATING CURRENT 285

8
SESSION 2 ∴ cos φ = 9 Current in the circuit is given by
30 40 (8) + (31 − 25)2
2
E
1 I =
V
= sin 50 t + cos 50t I = , where E is the voltage
8 ω2 L2 + R2
R 10 10 =
64 + 36 of an AC source
I = 3 sin 50 t + 4 cos 50t
cos φ = 0.80 µ 0µ r N 2 A
⇒ I = 5  cos 50 t + sin 50 t 
4 3 Hence,
As L = ⇒ L ∝µr
 5 5  6 Charge on capacitor, q 0 = CV l
= 5 [cos 37° cos 50t + sin 37° sin 50t ] q 0 = 2 × 10−6 C When iron rod is inserted,
= 5cos[50 t − 37° ] L increases, therefore current
q = q 0 sinω t
I 5 I decreases.
Now, I rms = 0 = A For maximum current,
2 2 dq
I0 = = ωq 0 10 Given, inductance, L = 20 mH
2 I = P = 60 = 6 A dt
Capacitance, C = 50 µF
V 10 1
Also, ω = = (109 )1 /2 Resistance, R = 40 Ω
LC
⇒ V = V R2 + V L2 emf, V = 10 sin 340 t
I 0 = (109 )1 /2 (2 × 10−6 )
(100)2 = (10)2 + V L2 = 10 × 104 × 2 × 10−6 Q Power loss in AC circuit will be given
as
⇒ V L = 99.5 V = 2 10 × 10−2 A 2
V L = IX L = I (2 π νL )  EV   E 
= 4000 mA Pav = I 2V R = ⋅R QI = V
 Z   V 2 
99.5 = 6 × 2 × 314
. × 50 L
⇒ L = 0.052 H
7 R  10 
2
=  ⋅ 40
 2
3 V = V 0 sinω t [as, V = 0 at t = 0]
1
V R = V 0 sinω t 2
 340 × 20 × 103 
π
VC = V 0 sin  ω t −  402 +  − 1 
  
2 P0 = V rms I rms = V rms
V rms
 340 × 50 × 10
−6

R
V and V R are in same phase. While VC 100 1
QV = × 40 ×
= I rms R ⇒ V rms = I rms  2 1600 + (6.8 − 58.8)2
lags V (or V R ) by 90°. Now, V R is in same  rms R 
phase with initial potential difference 2000
V 2
= ≈ 0.46W ≈ 0.51 W
P0 = rms 1600 + 2704
across the capacitor for the first time R
when ⇒ V rms
2
= P0 R 1
ωL −
π 3π 3π 11 tan φ = ωC
ωt = − + 2 π = ⇒t = L R R
2 2 2ω
φ being the angle by which the current
4 Z = (3) + (4) = 5 Ω
2 2
leads the voltage.
10
I0 = =2A Given, φ = 45°
Z
X  P = V rms I rms cos φ 1 1
φ = tan −1  L  = tan −1   = 53°
4 ωL − ωL −
 R   3 = V rms
2 R
= P0 R
R ωC ωC
∴ tan 45° = ⇒1 =
Z2 Z2 R R
∴ I = 2 sin (ω t − 53° )
π R2 1 1
At t = or ω t = π, P = P0 ⇒ R = ωL − ⇒ ωC =
ω Z2 ωC (ω L − R )
I = 2 sin ( π − 53° ) 8 Here, phase difference, ⇒ C =
1
=
1
4
= 2 × = 1.6 A XL − X C ω (ω L − R ) 2 πf (2 πfL − R )
5 tan φ =
V R = IR = 4.8 V
R 12 In position 1,
π XL − X C π ωL (1000) ( 3 × 10−3 )
⇒ tan = tan = =
5 Power factor of AC circuit is given by 3 R 6 R R
R
cos φ = …(i) When L is removed, then 1 3
Z XC ⇒ = ⇒ R = 3Ω
3= ⇒ X C = 3R 3 R
where, R is resistance employed and Z R  1 
the impedance of the circuit.  
When C is removed, then XC  ωC 
X In position 2, tan φ = =
Z= R2 + ( X L − X C )2 …(ii) 3 = L ⇒ XL = 3R R R
R 1
From Eqs. (i) and (ii), we get
Z = R2 + ( X L − X C )2 = R 1000
R 1000 × × 10−6
cos φ = …(iii) 3
[Q X L = X C ] tan φ = =1
R + ( X L − X C )2
2
3
cos φ =
Z
=
R
=1 π
Given, R = 8 Ω , X L = 31 Ω , X C = 25 Ω ⇒ φ=
R R 4
286 40 DAYS ~ NEET PHYSICS DAY TWENTY FIVE

13 So, equivalent resistor =


R 9
= Ω = 3Ω Resistance, R = 50 Ω
XC = 30 Ω 3 3 emf, V = 10 sin 314t
Battery emf, V = 18 V QThe general equation of emf is given as
∴ Current in circuit =
V
=
18 V = V 0 sin ωt
200 V R1 = 44 Ω
60 Hz R2 =36Ω R 3 ∴ Comparing Eqs. (i) and (ii), we get
= 6A V 0 = 10 V, ω = 314 rad s−1
XL = 90 Ω The power loss associated with the
15 Net reactive capacitance,
X L = 90 Ω, R2 = 36 Ω, X C = 30 Ω 1 given AC circuit is given as
XC =
2πfC P = V rms I rms cos φ
and total resistance,
= V rms  rms   
R C V R
R = R1 + R2 = 44 + 36 = 80 Ω
 Z  Z
Z = R + ( X L − X C ) = (80) + (60)
2 2 2 2
2 2
 V0 
=  rms  R = 
V
Z = 6400 + 3600 = 100  R …(i)
 Z   2 ⋅ Z
V 200 V=V0 sin ωt
Current, I = = = 2A ∴ Impedance, Z = R2 + ( X L − X C )2
Z 100
2
R2 +  ωL −
Power dissipated in the coil, So, current in circuit, 1 
= 
I =
V
=
V  ωC 
Pav = I 2 R = (2)2 × 80 = 320 W
Z 2
∴Substituting the given values in the
 1 
14 (*) No option is matching. R2 +   above equation, we get
 2πfC 
2
Thinking Process Just after switch is (314 × 20 × 10−3 ) 
2 πfC
closed, inductor acts like an closed ⇒ I = ×V = (50) + 
2
1 
 −
switch (closed path) and capacitor acts 4 π2 f 2C 2 R2 + 1 −4 
 314 × 10 
like a open switch (open path), because
Voltage drop across capacitor,
in DC circuit, inductive resistance
becomes zero. VC = I × X C = 2500 + [6280 × 10−3 − 0.00318 × 10 4 ] 2
2 πfC × V 1
= × = 2500 + (25.56) 2
R 4π f C R + 1
2 2 2 2 2 π fC
+ = 56.15 Ω ~
− 56 Ω
– R V
i.e. VC = Now, substituting this values in Eq. (i),
R
4 π2 f 2C 2 R2 + 1 we get
2
When mica is introduced, capacitance  10 
P =  × 50
will increase, hence voltage across  2 × 56 
capacitor get decrease.
100
⇒ + = × 50 = 0.79 W
– R R R 16 (c) Here, inductance, 2 × 3136
L = 20 mH = 20 × 10 −3 H Thus, power loss in the circuit is
Capacitance, C = 100µF = 100 × 10−6 F 0.79 W.
DAY TWENTY SIX

Electromagnetic
Waves
Learning & Revision for the Day
u Electromagnetic Waves and u Properties of Electromagnetic Waves
their Characteristics u Transverse Nature of Electromagnetic Waves
u Maxwell’s Equations u Spectrum of Electromagnetic Radiation

Electromagnetic Waves and


their Characteristics
Electromagnetic waves are those waves, in which electric and magnetic fields vary
sinusoidally in space and with time. The electric and magnetic fields are mutually
perpendicular to each other and each field is perpendicular to the direction of
propagation of the wave.
l
Maxwell’s theory predicted that electromagnetic waves of all frequencies (and hence
all wavelengths) propagate in vacuum, with a speed given by
1
c= .
µ 0ε 0
where, µ 0 is the magnetic permeability and ε 0 is the electric permittivity of vacuum.
Now, for the vacuum, µ 0 = 4π × 10 −7 TmA −1 and ε 0 = 8.85 × 10 −12 C2 N −1m−2 .
Substituting these values in the above relation, we have PREP
c=
1
[(4π × 10 −7 )(8.85 × 10 −12 )]1/ 2
~ 3.0 × 10 8 ms −1 MIRROR
Your Personal Preparation Indicator
l
All the electromagnetic waves are of the transverse nature whose speed depends u No. of Questions in Exercises (x)—
upon the medium, but their frequency does not depend on the medium. u No. of Questions Attempted (y)—
l
Transverse waves can be polarised. u No. of Correct Questions (z)—
l
Energy is being transported with the electromagnetic waves. (Without referring Explanations)

u Accuracy Level (z / y × 100)—


Conduction Current u Prep Level (z / x × 100)—
It is a current in the electric circuit, which arises due to the flow of electrons in the
connecting wires of the circuit, in a definite closed path. In order to expect good rank in NEET, your
Accuracy Level should be above 85 & Prep
Level should be above 75.
288 40 DAYS ~ NEET PHYSICS DAY TWENTY SIX

Energy density of electromagnetic wave,


Maxwell’s Displacement Current
l

1 1 B2
It is that current which comes into play in the region, ue = ε 0Eu2B =
2 2 µ0
whenever the electric field and hence the electric flux is
changing with it. u
l
Momentum delivered, p = (absorbing surface)
dφ E c
id = ε 0
dt 2u
The generalised form of the Ampere’s law is p= (reflecting surface)
c
 dφ E  hc
∫ B ⋅ d l = µ 0(i + id ) = µ 0  i + ε 0 dt  l
Energy of wave =
λ
= hν

Maxwell’s Equations Transverse Nature of


Maxwell in 1862, gave the basic laws of electricity and
magnetism in the form of four fundamental equations, which Electromagnetic Waves
are known as Maxwell’s equations. According to Maxwell, electromagnetic waves consist of time
l
Gauss’s law for electrostatics This law states that the varying electric and magnetic fields, which are perpendicular
electric lines of force start from positive charge and end at to each other, as well as direction of wave propagation.
negative charge i.e. the electric lines of force do not form a y
continuous closed path. Wave Propagation
q E B
Mathematically, ∫ E ⋅ dS =
S ε0
l
Gauss’s law for magnetism This law also predicts that the x
isolated magnetic monopole does not exist.
Mathematically, ∫ B ⋅ dS = 0 z B E
S
l
Faraday’s law of electromagnetic induction This law tells
that the changing magnetic field is the source of electric
dφ Spectrum of Electromagnetic
field. Mathematically, ∫ E ⋅ dl = − B
dt Radiation
l
Ampere-Maxwell’s law At an instant, in a circuit, the The array obtained on arranging all the electromagnetic
conduction current is equal to displacement current. waves in an order on the basis of their wavelength is called
 dφ  the electromagnetic spectrum.
Mathematically, ∫ E ⋅ dl = µ 0  I c + ε 0 E 
 dt  The Electromagnetic Spectrum
These equations are collectively called Maxwell’s Frequency Wavelength
equations. Name Source
Range (Hz) Range (m)
Radio 104 to 108 0.1 to 600 Oscillating electric
waves circuits
Properties of Microwaves 109 to 1012 10−3 to 0.3 Oscillating current in
Electromagnetic Waves special vacuum tubes
l
If the electromagnetic wave is travelling along the positive Infrared 1011 to 10−6 to Outer electrons in atoms
direction of the X -axis, the electric field is oscillating 5 × 1014 5 × 10−3 and molecules
parallel to the Y-axis and the magnetic field is oscillating Visible 4 × 1014 to 4 × 10−7 to Outer electrons in atoms
parallel to the Z-axis. light 7 × 1014
8 × 10−7
E = E 0 sin(k x − ωt ) ⇒ B = B0 sin(k x − ωt )
Ultraviolet 1015 to 1017 1.5 × 10−7 to Outer electrons in atoms
In this, E 0 and B0 are the amplitudes of the fields.
3.5 × 10−7
E 1
Further, c = 0 = = speed of light in vacuum X-rays 1018 to 1020 10−11 to 10−8 Inner electrons in atoms
B0 ε0 µ 0
and sudden
l
The rate of flow of energy in an electromagnetic wave, is deacceleration of high
described by the vector S called the Poynting vector, which energy free electrons
is defined by the expression, Gamma 1019 to 1024 10−16 to 10−13 Nuclei of atoms and
1 rays sudden deacceleration of
S= E×B high energy free electrons
µ0
DAY TWENTY SIX ELECTROMAGNETIC WAVES 289

maximum heating effect. The glass absorbs these


Various Electromagnetic Radiations radiations, therefore for the study of these radiations, rock
l
Gamma rays The main sources of gamma rays are the salt prism is used instead of a glass prism. These waves are
natural and artificial radioactive substances. These rays mainly used for therapeutic purpose by the doctors because
affect the photographic plate and mainly used in the of their heating effect.
treatment of cancer disease. l
Microwaves These waves are produced by the spark
l
X-rays X-rays are produced, when highly energetic discharge or magnetron valve. They are detected by the
cathode rays are stopped by a metal target of high melting crystal or semiconductor detector. These waves are used
point. They affect the photographic plate and can penetrate mainly in radar and long distance communication.
through the transparent materials. They are mainly used in l
Radio waves They can be obtained by the flow of high
detecting the fracture of bones, hidden bullet, needle, frequency alternating current in an electric conductor.
costly material etc. inside the body, and also used in the
These waves are detected by the tank circuit in a radio
study of crystal structure.
receiver or transmitter.
l
Ultraviolet Rays The major part of the radiations received
from sun consists of the ultraviolet radiation. Its other
sources are the electric discharge tube, carbon arc, etc. Applications of
These radiations are mainly used in excitation of Electromagnetic Spectrum
photoelectric effect and to kill the bacteria of many diseases. l
Radio waves are used in radar and radio broadcasting.
l
Visible Light Visible light is obtained from the glowing l
Microwaves are used in long distance wireless
bodies, while they are white hot. The light obtained from communications via satellites.
the electric bulbs, sodium lamp, fluorescent tube is the l
Infrared, visible and ultraviolet radiations are used to know
visible light.
the structure of molecules.
l
Thermal or Infrared Waves A body on being heated, emits l
Diffraction of X-rays by crystals, gives the details of the
out the infrared waves. These radiations have the structure of crystals.

DAY PRACTICE SESSION 1

FOUNDATION QUESTIONS EXERCISE


1 The speed of electromagnetic waves in a vacuum, is 4 The ratio of amplitude of magnetic field to the amplitude
given by of electric field for an electromagnetic wave propagating
1 1 in vacuum is equal to j
CBSE AIIPMT 2013
(a) (b) (c) µ 0 ε0 (d) µ 0 ε0
µ 0 ε0 µ 0 ε0 (a) the speed of light in vacuum
2 An electric field E and magnetic field B exist in a region. (b) reciprocal of speed of light in vacuum
If these fields are not perpendicular to each other, then (c) the ratio of magnetic permeability to the electric
susceptibility of vacuum
the electromagnetic wave
(d) unity
(a) will not pass through the region
(b) will pass through the region 5 Electromagnetic waves are transverse in nature, is
(c) may pass through the region evident by
(d) Nothing is definite (a) polarisation (b) interference
3 Which of the following statement is false for the properties (c) reflection (d) diffraction
of electromagnetic waves ? j
CBSE AIPMT 2010
6 Maxwell’s modified form of Ampere’s circuital law is
(a) Both electric and magnetic field vectors attain the
(a) ∫ B ⋅ d S = 0 (b) ∫ B ⋅ d l = µ 0Ic
maxima and minima at the same place and same time
1 dq d φE
(b) The energy in electromagnetic wave is divided equally (c) ∫ B ⋅ d l = µ 0Ic + (d) ∫ B ⋅ d l = µ 0Ic + µ 0 ε0
between electric and magnetic vectors ε0 dt dt
(c) Both electric and magnetic field vectors are parallel to 7 In an electromagnetic wave, the electric and magnetic
each other and perpendicular to the direction of fields are 100 Vm −1 and 0.265 Am −1. The maximum
propagation of wave energy flow is
(d) These waves do not require any material medium for
(a) 26.5 Wm−2 (b) 36.5 Wm−2 (c) 46.7 Wm−2 (d) 765 Wm−2
propagation
290 40 DAYS ~ NEET PHYSICS DAY TWENTY SIX

8 Which of the following pairs of space and time varying (c) 300 km (d) 400 km
E = ( $i E x + $j E y + k$ E z ) and B = ( $i Bx + $j By + k$ Bz ) would 16 The wave of wavelength 5900Å emitted by any atom or
generate a plane electromagnetic wave travelling in the molecule must have some finite total length which is
z-direction? known as the coherence length. For sodium light, this
(a) E x , B z (b) E y , B z (c) E z , B x (d) E x , B y length is 2.4 cm. The number of oscillations in this length
9 Consider the following two statements, regarding a will be
linearly polarised plane electromagnetic wave. (a) 4.068 × 108 (b) 4.068 × 104
I. Electric field and the magnetic field have equal average (c) 4.068 × 106 (d) 4.068 × 105
values. 17 21 cm radio wave emitted by hydrogen in intersteller
II. Electric energy and the magnetic energy have equal space is due to the interaction called the hyperfine
average values. interaction in atomic hydrogen, the energy of the emitted
(a) I is true (b) II is true
wave is nearly
(c) Both statements are true (d) Both statements are false
(a) 10−17 J (b) 1J
10 In an apparatus, the electric field was found to oscillate (c) 7 × 10−8 J (d) 10−24 J
with an amplitude of 18 Vm −1. The magnitude of the
18 The condition under which a microwave oven heats up a
oscillating magnetic field will be
food item containing water molecules most efficiently is
(a) 4 × 10−6 T (b) 6 × 10−8 T j NEET 2013
(c) 9 × 10−9 T (d) 11 × 10−11 T
(a) the frequency of the microwave must match the
11 An electromagnetic wave going through vacuum is resonant frequency of the water molecules
described by E = E 0 sin(kx − ωt ); B = B0 sin(kx − ωt ). (b) the frequency of the microwave has no relation with
Which of the following equations is true ? natural frequency of water molecules
(a) E 0k = B 0ω (b) E 0ω = B 0k (c) microwave are heat waves, so always produce heating
(c) E 0B 0 = ωk (d) None of these (d) infrared waves produce heating in a microwave oven
12 The electric and the magnetic field associated with an 19 The decreasing order of wavelength of infrared, microwave,
electromagnetic wave, propagating along the +Z-axis, ultraviolet and gamma rays is j
CBSE AIPMT 2011
can be represented by j
CBSE AIPMT 2011
(a) gamma rays, ultraviolet, infrared, microwaves
(a) E = E 0 k$ , B = B 0 $i (b) E = E 0 $j, B = B 0 $j (b) microwaves, gamma rays, infrared, ultraviolet
(c) E = E 0 $j , E = B 0 k$ (d) E = E 0 $i , B = B 0 $j (c) infrared, microwave, ultraviolet, gamma rays
13 A radio can tune into any station in the 7.5 MHz to (d) microwave, infrared, ultraviolet, gamma rays
12 MHz band. The corresponding wavelength band is 20 On required 11eV of energy to dissociate a carbon
(a) 5 m to 15 m (b) 2 m to 16 m monoxide molecule into carbon and oxygen atoms. The
(c) 25 m to 40 m (d) 30 m to 45 m minimum frequency of the appropriate electromagnetic
14 If a source is transmitting electromagnetic wave of radiation to achieve the dissociation lies in
frequency 8.2 × 10 6 Hz, then wavelength of (a) visible region (b) infrared region
electromagnetic waves transmitting from the source will be (c) ultraviolet region (d) microwave region

(a) 36.6 m (b) 40.5 m 21 The energy of an electromagnetic waves is of the order of
(c) 42.3 m (d) 50.9 m 15 keV. To which part of the spectrum does it belong?
j
CBSE AIPMT 2015
15 An electric charge oscillating with a frequency of
1 kilo cycle/s can radiate electromagnetic wave of (a) X-rays (b) Infrared rays
(c) Ultraviolet rays (d) γ-rays
wavelength?
(a) 100 km (b) 200 km
DAY TWENTY SIX ELECTROMAGNETIC WAVES 291

DAY PRACTICE SESSION 2

PROGRESSIVE QUESTIONS EXERCISE


1 Out of the following options which one can be used to 7 In an electromagnetic wave in free space, the root mean
produce a propagating electromagnetic wave? square value of the electric field is E rms = 6 V/m. The
(a) A stationary charge j NEET 2016 peak value of the magnetic field is j NEET 2017

(b) A chargeless particle (c) An accelerating charge (a) 1.41 × 10−8 T (b) 2.83 × 10−8 T
(d) A charge moving at constant velocity (c) 0.70 × 10−8 T (d) 4.23 × 10−8 T
2 An electromagnetic wave going through vacuum is 8 In a plane electromagnetic wave, the electric field
described by E = E 0 sin(kx − ωt ). oscillates sinusoidally at a frequency of 2 × 1010 Hz and
Which of the following is independent of wavelength? amplitude 48 Vm −1. Then, which one of the following
statement is true?
(a) k (b) ω (c) k/ω (d) k ω
2 (a) Wavelength of the wave is 2 × 105 m
3 Light with an energy flux of 18 W/cm falls on a
(b) Amplitude of oscillating magnetic field is 48 T
non-reflecting surface at normal incidence. If the surface
(c) Average energy density of electric field equals the
has an area of 20 cm 2 , the average force exerted on the average energy density of magnetic field
surface during a span 30 min is (d) None of the above
(a) 1.2 × 10−6 N (b) 10−3 N
9 The electric field part of an electromagnetic wave in a
(c) 4 × 10−7 N (d) 5 × 10−4 N
medium is represented by E x = 0;
4 The electric field associated with an electromagnetic
N  rad  − 2 rad 
wave in vacuum, is given by E = $i 40 cos (kz − 6 × 108 t ), E y = 2.5 cos  2π × 10 6  t −  π × 10 x ;
where E , z and t are in Volt/m, metre and second, C   m   s  
respectively. The value of wave vector k is E z = 0. The wave is j
CBSE AIPMT 2009
j
CBSE AIPMT 2012 (a) moving along y-direction with frequency 2 π × 106 Hz
−1 −1 −1 −1 and wavelength 200 m
(a) 2 m (b) 0.5 m (c) 6 m (d) 3 m
(b) moving along x-direction with frequency 106 Hz and
5 An EM wave is propagating in a medium with a velocity
wavelength 100 m
v = v $i . The instantaneous oscillating electric field of this
(c) moving along x-direction with frequency 106 Hz and
EM wave is along +Y -axis. Then, the direction of oscillating wavelength 200 m
magnetic field of EM wave will be along j
NEET 2018 (d) moving along – x-direction with frequency 106 Hz and
(a) −y-direction (b) + z-direction wavelength 200 m
(c) −z-direction (d) −x-direction
10 A uniform electric field and an uniform magnetic field are
6 A linearly polarised electromagnetic wave given as acting along the same direction in a certain region. If an
E = E 0 $i cos (kz − ω t ) is incident normally on a perfectly electron is projected in the region, such that its velocity is
reflecting infinite wall at z = a. Assuming that the material pointed along the direction of fields, then the electron
of the wall is optically inactive, the reflected wave will be (a) speed will decrease j
CBSE AIPMT 2011
given as (b) speed will increase
(a) Er = − E 0 $i cos (kz − ω t ) (b) Er = E 0 $i cos (kz + ω t ) (c) will turn towards left of direction of motion
(d) will turn towards right of direction of motion
(c) Er = − E 0 $i cos (kz + ω t ) (d) Er = E 0 $i sin (kz − ω t )

ANSWERS
SESSION 1 1 (b) 2 (c) 3 (c) 4 (b) 5 (a) 6 (d) 7 (a) 8 (d) 9 (b) 10 (b)
11 (a) 12 (d) 13 (c) 14 (a) 15 (c) 16 (b) 17 (d) 18 (a) 19 (d) 20 (c)
21 (a)
SESSION 2 1 (c) 2 (c) 3 (a) 4 (a) 5 (b) 6 (b) 7 (b) 8 (c) 9 (c) 10 (a)
292 40 DAYS ~ NEET PHYSICS DAY TWENTY SIX

Hints and Explanations


SESSION 1 12 µ = E × B = E 0 $i × B 0 $j = E 0 B 0 k$ SESSION 2
1 Speed of electromagnetic waves in E × B points in the direction of wave 1 A particle is known that an electric
1 propagation. charge at rest has electric field in the
vacuum =
µ 0 ε0 region around it, but no magnetic field.
13 ∴ λ = c
f A moving charge produces both the
2 The electromagnetic wave being packets electric and magnetic fields. If a charge
of energy moving with speed of light When f = 7.5 MHz = 7.5 × 106 Hz,
is moving with a constant velocity, the
may pass through the region. 3 × 108 electric and magnetic fields will not
then λ = = 40 m
3 The time varying electric and magnetic 7.5 × 106 change with time, hence no EM wave
fields are mutually perpendicular to When f = 12 MHz = 12 × 106 Hz, will be produced. But, if the charge is
each other and also perpendicular to moving with a non-zero acceleration,
the direction of propagation of the 3 × 108
then λ= = 25 m both the electric and magnetic field will
wave. 12 × 106 change with space and time, it then
E B
4 As, B 0 = 0 ⇒ 0 = 1 ∴ Wavelength band of stations is 25 m produces EM wave. This shows that
c E0 c to 40 m. accelerated charge emits
electromagnetic waves.
14 ∴ λ = c = 3 × 10 6 = 36.58 ≈ 36.6 m
8
i.e. reciprocal of speed of light in
vacuum. ν . × 10
82 2 Here, kx = θ 0 or k = θ 0 / x and ω t = θ 0
or ω = θ 0 /t
15 ∴ λ = c = 3 × 10 = 3 × 105 m = 300 km
8
5 As electromagnetic waves are transverse k t
in nature, i.e. have oscillating electric ν 1000 ∴ =
and magnetic vector propagating ω x
16 Number of oscillations in coherence 1 1
perpendicular to each other and also to = =
l 0.024 ( x/t ) v
wave propagation. Hence, can be length = =
proved by polarisation. λ 5900 × 10−10 where, v is the velocity of
= 4.068 × 104 electromagnetic wave, which is
6 The modified form of Ampere’s circuital −34 independent of wavelength of wave.
17 ∴E = hc = 6.6 × 10 × −32 × 10
8
law is
 dφ E  λ 21 × 10 3 Total energy falling on the surface,
∫ B ⋅ d l = µ 0  Ic + ε0 dt 

≈ 10−24 J U = (18W/cm2 ) (30 × 60 s) (20 cm2 )

7 Maximum rate of energy flow, 18 It is an electromagnetic wave. The U = 6.48 × 105 J


frequency of the microwave oven must Total momentum,
S = E 0 × B 0 = 100 × 0.265
match the resonant frequency of the U 6.48 × 105
= 26.5 Wm −2 water molecules. p= = kg m/s
c 3 × 108
8 Direction of propagation of 19 Decreasing order of wavelength of ⇒ . × 10−3 kg m/s
p = 216
electromagnetic waves is in the various rays is Microwave > Infrared >
direction of cross product of E and B Ultraviolet > Gamma rays Average force exerted on the surface is
(i.e. E × B). So, if wave is propagating in −19 p 216. × 10−3
20 Given, E = 11 eV = 11 × 1.6 × 10 V, F = =
two direction, then t 30 × 60
E = hν
E = E x $i and B = B y $j. . × 10−6 N
= 12
11 × 1.6 × 10−19
or ν =
Then, E × B point in + z-direction. h 4 Electromagnetic wave equation,
11 × 1.6 × 10−19 E = E 0 cos (kz − ωt ) …(i)
9 Average energy is equally shared by = = 2.65 × 1015 Hz Speed of electromagnetic wave,
electric field and magnetic field. 6.62 × 10−34
ω
However, their average magnitudes are v =
This frequency radiation belongs to k
not equal. ultraviolet region. Given equation,
1
As, |B 0 |= ⋅ |E 0 | 21 Given, energy of an electromagnetic E = $i 40cos (kz − 6 × 108t ) …(ii)
C waves is of the order of 15keV,
c On comparing Eqs. (i) and (ii), we get
E0 18
10. ∴ B 0 = = = 6 × 10−8 T i.e. E = hν = h × ω = 6 × 108 and E 0 = 40 $i
c 3 × 108 λ
h×c 6.624 × 10−34 × 3 × 108 Here, wave factor,
11. We know E 0 = c B 0 , where c is the ⇒λ = =
15 × 103 × 1.6 × 10−19 ω 6 × 108
E k = = = 2 m −1
velocity of light. .
13248 × 10−29 v 3 × 108
ω ω = = 0.828 × 10−10 m
∴ c = νλ = λ= 1.6 × 10−19
2π k 5 Here, velocity of EM wave, v = vi$
−10
ω = 0.828 Å [Q 1Å = 10 m] Instantaneous oscillating electric field,
Thus, E0 = B0
k λ = 0.828 Å E = E$j
or E 0k = B0 ω Thus, this spectrum is a part of X-rays.
DAY TWENTY SIX ELECTROMAGNETIC WAVES 293

As we already know that, during the E rms = 6 V/m  E0 


2

propagation of EM waves through a We know that, peak value of electric  


 c  1 E 20
medium oscillating electric and field, = =
4µ 0 4µ 20 c 2
magnetic field vectors are mutually E 0 = 2 E rms
perpendicular to each other and to the E 20  1 
⇒ E0 = 2 × 6 V/ m = Qc = 
direction of each other and to the 2
µ 0 ε0 
E  1  
direction of propagation of the wave Also, we know that, c = 0 4µ 0  
(E × B ). B0  µ 0 ε0 
i.e. E × B = v ⇒ (E$j) × B = v$i …(i) where, c = speed of light in vacuum µ ε 1
= 0 0 E 20 = ε0 E 20
As we know that from vector algebra, B 0 = peak value of magnetic field 4µ 0 4
$j × k$ = $i …(ii) E
⇒ B0 = 0 Hence, option (c) is true.
Comparing Eqs. (i) and (ii), we get c
2×6 9 Comparing the given equation,
B = Bk$ , ⇒ B0 = N  6 rad 
where, B (say) be the magnitude of 3 × 108 E y = 2.5 cos   2 π × 10 m  t
8.48
C 
magnetic field. ⇒ B0 = × 10−8 rad  
3 −  π × 10−2 x
Thus, we can say that the direction of
⇒ B 0 = 2.83 × 10−8 T
 s  
oscillating magnetic field of the EM
wave will be along + z-direction. With the standard equation
8 ∴ Wavelength, E y = E 0 cos (ωt − kx ), we get
6 When a wave is reflected from denser c 3 × 108 ω = 2 πf = 2 π × 106
medium, the reflected wave is without λ= = = 1.5 × 10−2 m
f 2 × 1010 ∴ f = 106 Hz
change in type of wave, but with a
change in phase by 180° or π radian. Hence, option (a) is false.
Moreover, we know that
E 48
Therefore, for the reflected wave, we ∴ B0 = 0 = 2π
c 3 × 108 = k = π × 10−2 m −1 ⇒ λ = 200m
use z = − z, $i = − $i and additional phase λ
of π in the incident wave. The incident = 16 × 10−8 T
As direction of field E of
of electromagnetic wave is, E = E 0 $i cos = 1.6 × 10−7 T electromagnetic wave is in
(kz − ωt ). The reflected electromagnetic Energy density of electric field is y-direction, so the wave is moving
2
wave is
ε0 E 2 = ε0  0  = ε0 E 20
1 1 E 1 along positive x-direction with
E r = E 0 (− i$ ) cos [k (− z ) − ω t + π] UC =
2 2  2 4 frequency 106 Hz and wavelength 200 m.
= − E 0 i$ cos[−(kz + ω t ) + π] Energy density of magnetic field is 10 Field B will not apply any force. Field
= E 0 $i cos [− (kz + ω t )] 2
E will apply a force opposite to velocity
1 2 1  B0 
[Q cos (θ + π ) = − cos θ] UB = B =   of the electron, hence speed will
= E 0 i$ cos (kz + ω t ) 2µ 0 2µ 0  2 
decreases.
[Q cos (− θ) = cos θ] 1 B2 ∴ B = E 0 
= ⋅ B 20 = 0
4µ 4µ 0  0
c 
7 Given, root mean square value of
electric field,
DAY TWENTY SEVEN

Unit Test 5
(Magnetostatics EMI
and AC, EM Waves)
1 The magnetic field of a given length of wire carrying a 7 A beam of electrons is moving with constant velocity in a
current for a single turn circular coil at centre is B, then region having simultaneous perpendicular electric and
its value for two turns for the same wire, when same magnetic fields of strength 20 Vm −1 and 0.5 T,
current is passing through it, is respectively at right angles to the direction of motion of
B B the electrons. Then, the velocity of electrons must be
(a) (b) (c) 2B (d) 4B
1
4 2 (a) 8 ms −1 (b) 20 ms −1 (c) 40 ms −1 (d) ms −1
40
2 A long solenoid carrying a current, produces a magnetic
field B along its axis. If the current is doubled and the 8 A straight wire of length 0.5 m and carrying a current of
number of turns per cm is halved, the new value of the 1.2 A is placed in uniform magnetic field of induction 2 T.
magnetic field is The magnetic field is perpendicular to the length of the
B wire. The force on the wire is
(a) 2B (b) 4B (c) (d) B
2 (a) 2.4 N (b) 1.2 N (c) 3.0 N (d) 2.0 N
3 Magnetic field due to 0.1A current flowing through a 9 Two wires are held perpendicular to the plane of paper
circular coil of radius 0.1 m and 1000 turns at the centre and are 5 m apart. They carry currents of 2.5 A and 5 A
of the coil is in same direction. Then, the magnetic field strength B at
(a) 0.2 T (b) 2 × 10−4 T a point midway between the wires will be
(c) 6.28 × 10−4 T (d) 9.8 × 10−4 T µ0 µ0 3µ 0 3µ 0
(a) T (b) T (c) T (d) T
4 The magnetic field at a distance r from a long wire 4π 2π 2π 4π
carrying current I is 0.4 T. The magnetic field at a
10 An uniform magnetic field acts right angles to the
distance 2 r, is
direction of motion of electrons. As a result, the electron
(a) 0.2 T (b) 0.8 T
moves in a circular path of radius 2 cm. If the speed of
(c) 0.1 T (d) 1.6 T
electrons is doubled, then the radius of the circular path
5 A charged particle of charge q and mass m enters will be
perpendicularly in a magnetic field B. If kinetic energy of (a) 2.0 cm (b) 0.5 cm
the particle is E , then frequency of rotation is (c) 4.0 cm (d) 1.0 cm
qB qB qBE qB 11 The work done in turning a magnet of magnetic moment
(a) (b) (c) (d)
mπ 2 πm 2 πm 2 πE M by an angle of 90° from the meridian, is n times the
6 A charge q moves in a region, where electric field E and corresponding work done to turn it through an angle of
magnetic field B both exist, then the force on it, is 60°. The value of n is given by
(a) 2 (b) 1
(a) q (v × B) (b) q E + q (v × B) (c) 0.5 (d) 0.25
(c) q B + q (B × v) (d) q B + q (E × v)
DAY TWENTY SEVEN UNIT TEST 5 (MAGNETOSTATICS EMI AND AC, EM WAVES) 295

12 Two bar magnets having same geometry with magnetic A1 C R1


moments M and 2 M, are firstly placed in such a way that
their similar poles are on the same side, then its period of L A2
oscillation isT1. Now, the polarity of one of the magnets is
reversed the time period of oscillations becomesT2 . R2
K
Then,
(a)T1 < T2 (b)T1 > T2 (c)T1 = T2 (d)T2 = ∞ Battery
13 In which type of material, the magnetic susceptibility (a) zero in both A1 and A 2
does not depend on temperature? (b) maximum in both A1 and A 2
(a) Diamagnetic (b) Paramagnetic (c) zero in A1 and maximum in A 2
(c) Ferromagnetic (d) Ferrite (d) maximum in A1 and zero in A 2
14 A diamagentic material in a magnetic field moves 23 In an experiment, 200 V AC is applied at the ends of an
(a)perpendicular to the field L - C - R circuit. The current consists of an inductive
(b)from weaker to the stronger parts of the field reactance ( X L ) = 50 Ω, capacitive reactance ( X C ) = 50 Ω
(c)from stronger to the weaker parts of the field and ohmic resistance (R ) = 10 Ω. The impedance of the
(d)None of the above circuit is
15 A magnetic field of 2 × 10−2 T acts at right angles to a (a) 10 Ω (b) 20 Ω
2 (c) 30 Ω (d) 40 Ω
coil of area 100 cm , with 50 turns. The average emf
induced in the coil is 0.1 V, when it is removed from the 24 The current in self-inductance L = 40 mH is to be
field in t second. The value of t is increased uniformly from 1 A to 11 A in 4 ms. The emf
(a) 10 s (b) 0.1 s (c) 0.01 s (d) 1 s induced in the inductor during the process, is
(a) 100 V (b) 0.4 V
16 A conductor of length 0.4 m is moving with a speed of
−1 (c) 4.0 V (d) 440 V
7 ms perpendicular to a magnetic field of intensity
0.9 Wbm −2 . The induced emf across the conductor is 25 An L - C - R series circuit is connected to a source of
(a) 1.26 V (b) 2.52 V (c) 5.04 V (d) 25.2 V alternating current. At resonance, the applied voltage
and the current flowing through the circuit will have a
17 If the number of turns per unit length of a coil of solenoid
phase difference of
is doubled, the self-inductance of the solenoid will π
(a) π (b)
(a)remain unchanged (b) be halved 2
(c)be doubled (d) become four times π
(c) (d) zero
4
18 A varying current in a coil change from 10 A to zero in
0.5 s. If the average emf induced in the coil is 220 V, the 26 A wire of resistance R is connected in series with an
self-inductance of the coil is inductor of reactance ωL. Then, quality factor of R - L
(a) 5 H (b) 6 H (c) 11 H (d) 12 H circuit is
R ωL
19 In an inductor of self-inductance L = 2mH, current (a) (b)
2 −t ωL R
changes with time according to relation I = t e . At what
R ωL
time emf is zero? (c) (d)
R +ω L
2 2 2
R + ω2 L2
2
(a) 4 s (b) 3 s (c) 2 s (d) 1 s
20 A 100 mH coil carries a current of 1 A. Energy stored in 27 Which of the following combinations should be selected
its magnetic field is for better tuning of an L-C-R circuit used for
(a) 0.5 J (b) 1 A (c) 0.05 J (d) 0.1 J communication?
21 Two coils have a mutual inductance of 0.005 H. The (a) R = 20 Ω, L = 15
. H, C = 35 µF
current changes in the first coil according to equation (b) R = 25 Ω, L = 2.5 H, C = 45µF
I = I 0 sin ωt , I 0 = 10 A and ω = 100 π rads −1. The maximum (c) R = 15 Ω, L = 3.5 H, C = 30 µF
value of emf in the second coil is (d) R = 25 Ω, L = 15
. H, C = 45µF
(a) 2 π (b) 5 π (c) π (d) 4 π 28 For a series L - C - R -circuit, the power loss at resonance is
22 In a circuit inductance L and capacitance C are V2
(a) (b) I 2 Cω
1
connected as shown in figure. A1 and A 2 are ammeters. ωL −
When key K is pressed to complete the circuit, then just ωC
V2
after closing key (K ), the reading of current will be (c) I 2R (d)
ωC
296 40 DAYS ~ NEET PHYSICS DAY TWENTY SEVEN

29 An alternating current generator has an internal resistance Direction (Q. Nos. 35-39) In each of the following
R g and an internal reactance X g . It is used to supply questions, a statement of Assertion is given followed by a
power to a passive load consisting of resistance R g and a corresponding statement of Reason just below it. Of the
statements mark the correct answer as
reactance X L . For maximum power to be delivered from
(a) If both Assertion and Reason are true and the Reason is
the generator to the load, the value of X L is equal to
the correct explanation of the Assertion
(a) zero (b) X g (c) −X g (d) R g
(b) If both Assertion and Reason are true but the Reason is
30 The primary winding of transformer has 500 turns, not correct explanation of the Assertion
whereas its secondary has 5000 turns. The primary is (c) If Assertion is true but Reason is false
connected to an AC supply of 20V-50 Hz. The secondary (d) If both Assertion and Reason are false
will have an output of 35 Assertion (A) In an electromagnetic wave, the average
(a)2 V, 5 Hz (b) 200 V, 500 Hz energy density of electric field is equal to the average
(c)2 V, 50 Hz (d) 200 V, 50 Hz
energy density of the magnetic field.
31 If ε 0 and µ 0 are respectively, the electric permittivity and
Reason (R) Electric and magnetic fields are related
magnetic permeability of free space, ε and µ are the
as E = cB.
corresponding quantities in a medium, the index of
refraction of the medium is 36 Assertion (A) Two parallel wires carrying currents in the
ε0 µ 0 εµ ε0 µ ε same direction, attract each other due to magnetic force
(a) (b) (c) (d)
εµ ε0 µ 0 ε µ0 ε0 between them.
32 A step-up transformer operates on a 230 V line and Reason (R) They attract each other, if the currents
supplies current of 2 A to a load. The ratio of the primary flowing in them are in opposite direction.
and secondary windings is 1 : 25. The current in the
primary coil is 37 Assertion (A) The torque on the coil is maximum, when
(a) 15 A (b) 50 A (c) 25 A (d) 12.5 A coil is suspended in a radial magnetic field.

33 Two coil of self inductance L1 and L2 are placed clear to Reason (R) The torque tends to rotate the coil about its
each other so that focal flux in one CaCl is completely own axis.
linked with other. If M is mutual inductance between 38 Assertion (A) When a magnet is brought near iron nails,
them, then only translatory force act on it.
L1
(a) M = L1L2 (b) M = (c) M = L1L2 (d) M = (L1 L2 )2
L2 Reason (R) The field due to a magnet is generally
uniform.
34 An LCR series circuit with R = 100 Ω is connected to a
200 V, 500 Hz a.c source when only the capacitance is 39 Assertion (A) Susceptibility is defined as the ratio of
removed, the current leads the voltage by 6s. When only intensity of magnetisation I to magnetic intensity H.
the inductance is removed, the current leads the voltage
Reason (R) Greater the value of susceptibility, smaller
by 60°. The current in the circuit is
the value of intensity of magnetisation I.
3 2
(a) 2 A (b) 7 A (c) A (d) A
2 3

ANSWERS
1 (d) 2 (d) 3 (c) 4 (a) 5 (b) 6 (b) 7 (c) 8 (b) 9 (b) 10 (c)
11 (a) 12 (a) 13 (a) 14 (c) 15 (b) 16 (b) 17 (d) 18 (c) 19 (c) 20 (c)
21 (b) 22 (d) 23 (a) 24 (a) 25 (d) 26 (b) 27 (c) 28 (c) 29 (c) 30 (d)
31 (b) 32 (b) 33 (c) 34 (a) 35 (b) 36 (c) 37 (b) 38 (d) 39 (c)
DAY TWENTY SEVEN UNIT TEST 5 (MAGNETOSTATICS EMI AND AC, EM WAVES) 297

Hints and Explanations


1 Magnetic field at the centre of circular 7 When v, E and B are mutually 12 The time period of bar magnet,
µ NI perpendicular to each other and particle I
coil, B = 0 . T = 2π
2r pass undeflected then, MH
L E
Case I N = 1, L = 2 πr ⇒ r = qvB = qE ⇒ v = When same poles of magnets are placed
2π B on same side, then net magnetic moment
µ 0 × 1 × I µ 0I Here, E = 20 Vm −1 and B = 0.5 T
∴ B = = M1 = M + 2M = 3 M
2r 2r 20
Case II N = 2, L = 2 × 2 πr ′ ∴ v = = 40 ms −1 ∴ T1 = 2 π
I
= 2π
I
…(i)
0.5 M1 H 3MH
L r
⇒ r′ = =
4π 2 8 A magnetic force is given by When opposite poles of magnets are
µ × 2 × I µ 0 × 2I 4µ 0 I F = Il B sinθ placed on same side, then net magnetic
∴B ′ = 0 = = = 4B
2r ′ 2 × (r /2) moment
2r For θ = 90°, F max = IlB
M2 = 2M − M = M
Here, I = 1.2 A, l = 0.5 m, B = 2 T
2 For a solenoid, B = µ 0 nI I I
F = 2 × 1.2 × 0.5 = 1.2 N ∴ T2 = 2 π = 2π …(ii)
where, n = number of turns per unit M2 H MH
length 9 According to Maxwell’s right handed
or B ∝ nI screw rule. From Eqs. (i) and (ii), we observe that,
B1 n I T1 < T2 .
∴ = 1 1 1 2
B2 n2 I2 13 For diamagnetic substance, magnetic
n I1 I2 susceptibility is independent of
Here, n1 = n, n2 = , temperature.
2
I1 = I , I2 = 2I , B1 = B 14 Diamagnetic substance moves from
B n I r stronger to weaker field.
Hence, = × = 1 or B2 = B
B2 n/ 2 2I
Therefore, net magnetic field
15 Emf induced in the coil is
dφ (φ − φ1 )
3 At the centre of circular coil carrying µ I µ I e =− =− 2
current, the magnetic field, B = B1 − B2 = 0 1 − 0 2 dt dt
2 πr1 2 πr2
µ NI (0 − NBA )
B = 0 r 5 ∴ e =−
2r At mid point, r1 = r2 = = = 2.5 cm dt
2 2
Given, N = 1000, I = 01. A, r = 01
. m NBA 50 × 2 × 10−2 × 10−2
Hence, or dt = =
Substituting the values, we get e 0.1
µ  I I  µ
B = 0  1 − 2  = 0 
4 × 10−7 × 1000 × 01. 5 2.5 = 0.1 s
B = − 
2 × 01
. 2 π  r /2 r /2  2 π  2.5 2.5
16 Length of conductor (l ) = 0.4 m
= 2π × 10−4 = 6.28 × 10−4 T µ0 µ
= (2 − 1) = 0 T Speed (v ) = 7 ms −1
4 Magnetic field due to a long current 2π 2π
carrying wire at distance r, is given by Magnetic field B = 0.9 Wbm −2
10 The force F on the charged particle due
µ 2I 1 Induced emf, e = Blv sinθ
B = 0 ⋅ or B ∝ to magnetic field provides the required
4π r r centripetal force (= mv 2 /r ) necessary for = 0.9 × 0.4 × 7 × sin90° = 2.52 V
When r is doubled, the magnetic field motion along the circular path of radius 17 Self-inductance of solenoid is given by
becomes halved i.e. now the magnetic r. µ 0N2 A
field will be 0.2 T. L =
mv 2 mv l
∴ qvB = or r =
5 Here, magnetic force = centripetal force r qB ∴ L ∝ N2
mv 2 ∴ r ∝v when N is doubled, L becomes 4 times.
i.e. qvB =
r As, v is doubled, then the radius also 18 Emf induced in the coil is given by
or qvB = mrω2 [Q v = rω] becomes double. dI
e = −L
or ω=
qB Hence, radius = 2 × 2 = 4 cm. dt
m Here, dI = I2 − I1 = 0 − 10 = − 10 A
11 Work done in rotating the dipole from
If ν is the frequency of rotation, then dt = 0.5 s , e = 220V
ω θ = θ1 to θ = θ2 , is (−10) 220
ω = 2 πν ⇒ ν = W = − MB (cos θ2 − cos θ1 ) ∴ 220 = − L or L = = 11 H
2π 0.5 20
qB Case I
∴ ν= W1 = − MB (cos 90° − cos 0° ) = MB 19 It is given that emf is zero i.e.
2 πm dI dI d
Case II W2 = − MB (cos 60° − cos 0° ) e = −L = 0 or L = 0 or (t 2e − t ) = 0
6 When a charged particle moves in dt dt dt
= − MB  − 1 = MB = W1
region of E ∝ B, then the net force will 1 1 1
or 2t × e − t + t 2 × (−1)e − t = 0
be = F F + F B = qE + q(v + B ) 2  2 2
As, W1 = nW2 ⇒ n = 2 or te − t (2 − t ) = 0 ⇒ t = 2s (Qte − t ≠ 0)
298 40 DAYS ~ NEET PHYSICS DAY TWENTY SEVEN

20 Energy stored in coil is E = 1 LI 2 which is given by Q =


1 L
.
di2
e 2 = − L2 ⇒ L2 =
e2
2 R C dt di / dt
where, L is self-inductance of coil and I To make Q high ; R should be low ; L e 1e 2
M2 = = L1 L2 ⇒ M = L1 L2
is current induced. should be high and C should be low.  di   di2 
  
1
∴E = × (100 × 10−3 ) × (1) 2 = 0.05 J
Therefore, choice (c) is the best suited.  dt   dt 
2 28 In series L -C - R circuit at resonance,
34 If the capacitance is removed, it is an
21 The given equation of current changing Capacitive reactance ( X C ) L-R circuit φ = 60°.
in the first coil is I = I 0 sinωt …(i) = Inductive reactance ( X L ) X
Differentiating Eq. (i) with respect to 1 tan φ = L = tan 65° = 3
i.e. = ωL R
time, we have ωC If inductance is removed, it is a
dI d dI
= (I 0 sin ω t ) or = I 0 ω cos ω t Total impedance of the circuit, capacitative circuit or R-C circuit.|φ| is
dt dt dt
dI Z = R2 + ( X L − X C ) 2 the same
For maximum , the value of cos ωt L
dt 2 ∴ Lω = this is resistance circuit.
R2 +  ωL −
1  Cω
should be equal to 1. = 
 ωC  E
 dI  z = R, I rms = rms ⇒ E rms = 200V
So,   = I0 ω i.e. Z =R R
 dt  max
R R 200
The maximum value of emf is given by So, power factor cos φ = = =1 I rms = = 2A
Z R 100
∴ e max = M  
dI
= MI 0ω Thus, power loss at resonance is given
 dt  max 35 Energy density in electric field is
by P = V rms I rms cos φ = V rms I rms × 1 1
Here, M = 0.005 H, UE = ε0 E 2
= (I rms R )I rms = (I rms ) 2 R = I 2 R
I 0 = 10 A, ω = 100 π rads −1 2
Energy density in magnetic field is
∴ e max = 0.005 × 10 × 100 π = 5π 29 For delivering maximum power from
1 2
the generator to the passive load, total UB = B
22 Initially, there is no DC current in reactance must vanish, i.e. or 2µ 0
inductive circuit and maximum DC XL = − Xg . We know that,
current in capacitive circuit. Hence, the
30 The transformation ratio of transformer E = cB
current is zero in A 2 and maximum in
is given by 1
A 1. and c =
Vs N
= s µ 0 ε0
23 Total effective resistance of L-C-R Vp Np 1 1
circuit is ∴ UE = ε0 E 2 = ε0 (cB ) 2
V Making substitution, we obtain 2 2
Z = 0 = R2 + ( X L − X C ) 2 N 5000 1 1 B2
I0 Vs = s V p = × 20 = 200 V = ε0 × × B2 = = UB
Np 500 2 µ 0 ε0 2µ 0
Given, V AC = 200 V, X L = 50 Ω,
X C = 50 Ω, R = 10 Ω, Z = ? Also, frequency of AC remains Therefore, UE = UB
unchanged. Thus, option (d) has voltage
∴Z = (10) 2 + (50 − 50) 2 or Z = 10 Ω 36 We know that, when two parallel wires
200 V and frequency 50 Hz.
carrying currents in same direction attract
24 Emf induced in the coil is given by 31 Refractive index of medium is given by each other due to magnetic interaction
dI n = c /v
|e | = L but, if they carry currents in opposite
dt 1 1 directions, they repel each other.
Here, c = and v =
−3  10  µ 0 ε0 µε
∴|e | = 40 × 10 ×  = 100 V 37 The torque on the coil is given by
−3
 4 × 10  µ ε εµ
∴ n= . = τ = nIBA cos θ
25 The impedance (Z ) of an L-C-R circuit is µ 0 ε0 ε0 µ 0 When the magnetic field is radial, then
given by coil is set with its plane parallel to
VP N
2 32 For a transformer, = P magnetic field i.e. θ = 0, hence cos θ = 1,
Z = R +  ωL −
1  Vs NS
2
 τmax = nIBA × 1 = nIBA
 ωC  Also the power remains constant, if
At resonance, X L = X C η = 100% 38 The field due to a magnet is
1 I V N non-uniform. Therefore, it exerts both, a
i.e. ωL = So, V P I P = VS IS ⇒ P = P = S net force and a torque on the nails
ωC IS VS NP
1 which will translate and also rotate the
ωL − Np 1 nails before striking to North pole of
ωc = 0 ⇒ φ = Zero Here, = , Is = 2 A
As, tan φ = Ns 25 magnet with their induced South poles
R
I p 25 and vice-versa.
So, circuit behaves as, if it contains R ∴ =
only. So, phase difference = 0 2 1 39 From the relation, susceptibility of the
or I p = 25 × 2 = 50 A I
26 Quality factor = Q = L = ωL material is χ m = ⇒ χm ∝ I
X
R R e e H
33 M = − 2 = − 1 Thus, it is obvious that greater the value
di1 / dt di1 / dt
27 The L-C-R circuit used for of susceptibility of a material greater
di e1
communication should posses high Also, e 1 = − L1 1 ⇒ L1 = will be the value of intensity of
quality factor (Q factor) of resonance. dt di / dt magnetisation.
EXAM BITES

This Pdf Is
Downloaded From
www.exambites.in

Visit www.exambites.in for


More Premium Stuffs,Latest
Books,Test Papers,Lectures etc.
jeeneetadda
jeeneetadda_official
jeeneetadda

VISIT NOW !!
DAY TWENTY EIGHT

Ray Optics
Learning & Revision for the Day
u Reflection of Light u Total Internal Reflection u Human Eye
u Mirror formula (TIR) u Optical Instruments
u Refraction of Light u Deviation by a Prism u Resolving Power of an
u Lens u Dispersion by a Prism Optical Instrument

Reflection of Light
The phenomena of bouncing back of light on striking a smooth surface is called
reflection of light.
l
According to the laws of reflection, (i) the incident ray, reflected ray and the normal
drawn on the reflecting surface at the point of incidence lie in the same plane and (ii)
the angle of incidence ∠i = angle of reflection ∠r.
l
Laws of reflection are true for reflection from a polished mirror or from an unpolished
surface or for diffused reflection.
l
Whenever reflection takes place from a denser medium, the reflected rays undergo a
phase change of π.

Reflection from a Plane Mirror


l
If a ray is incident on a plane mirror at an angle of incidence i, then it suffers a
deviation of (π − 2i) and for two inclined plane mirrors deviation is (360 °−2θ).
l
While keeping an object fixed, a plane mirror is rotated in its plane by an angle θ,
then the reflected ray rotates in the same direction by an angle 2θ.
Focal length as well as the radius of curvature of a plane mirror is infinity. Power of a
PREP
l

plane mirror is zero. MIRROR


Your Personal Preparation Indicator
l
If two plane mirrors are inclined to each other at an angle θ, the total number of

or  
2π u No. of Questions in Exercises (x)—
images formed of an object kept between them, is n = − 1 , when it is odd.
θ  θ  u No. of Questions Attempted (y)—
u No. of Correct Questions (z)—
l
The minimum size of a plane mirror fixed on a wall of a room, so that a person at the (Without referring Explanations)
centre of the room may see the full image of the wall behind him, should be 1/3rd the
size of the wall. u Accuracy Level (z / y × 100)—
u Prep Level (z / x × 100)—
Reflection from a Spherical Mirror
In order to expect good rank in NEET, your
l
A spherical mirror is a part of a hollow sphere whose one surface is polished, so that Accuracy Level should be above 85 & Prep
it becomes reflecting. The other surface of the mirror is made opaque. Level should be above 75.
300 40 DAYS ~ NEET PHYSICS DAY TWENTY EIGHT

Images formed by a concave mirror may be real or virtual,


Refractive Index
l

may be inverted or erect, and may be smaller, larger or


equal in size to that of the object. The image is virtual and For a given pair of media, the ratio of the sine of angle of
erect when the object is placed between the pole and the incidence (i) to the sine of angle of refraction (r ) is a constant,
principal focus of concave mirror. In all other cases, the which is called the refractive index of second medium, w.r.t.
image formed is real and inverted one. first medium.
Image formed by a convex mirror is virtual, erect and sin i n
= constant = n21 = 2
l
Thus,
diminished in size irrespective of the position of the object. sin r n1
Moreover, image is formed in between the pole and the This is also called Snell’s law.
principal focus of the mirror.
l
Refractive index is a unitless, dimensionless and a scalar
l
The focal length of a spherical mirror is half of its radius of quantity.
R
curvature, i.e. f = . l
The refractive index of a medium w.r.t. vacuum
2 (or free space) is known as its absolute refractive index. It is
defined as the ratio of the speed of light in vacuum (c) to
Mirror Formula the speed of light in a given medium (v).
Let an object be placed at a distance u from the pole of a c
∴ n=
mirror and its image is formed at a distance v from the pole. v
1 1 1 Value of absolute refractive index of a medium can be 1 or
Then, according to mirror formula, + =
v u f more than 1, but never less than 1.
l
The power of a mirror (in dioptre), is given as l
When light travels from one material medium to another,
1 the ratio of the speed of light in the first medium to that in
P= the second medium is known as the relative refractive
f (in metre)
index of second medium, w.r.t. the first medium. Thus,
l
If a thin object of height h is placed perpendicular to the v c / v2 n2
n21 = 1 = =
principal axis of a mirror and the height of its image be h′, v2 c / v1 n1
then the transverse or lateral magnification produced is
given by
l
When light undergoing refraction through several media
finally enters the first medium itself, then
h′ v f f −v
m= =− = = n31
h u f −u f n21 × n32 × n 13 = 1 or n32 =
n21
Negative sign of magnification means the inverted image
and positive sign means an erect image. l
When the object is in denser medium and the observer is in
l
When a small sized object is placed along the principal rarer medium, then real and apparent depth have the
Real depth
axis, then its longitudinal (or axial) magnification is given relationship, = n21
by Apparent depth
2 2 2  1 
dv  v   f   f − v i.e. real depth > apparent depth shift, y = h − h′ = 1 − h
Axial magnification = − =  =  =   n21 
du  u   f − u  f 
where, h and h′ are real and apparent depths.

Refraction of Light Refraction from a Spherical Surface


When light passes from one medium, say air, to another Let an object be placed in a medium of refractive index n1 at a
medium, say glass, a part is reflected back into the first distance u from the pole of a spherical surface of radius of
medium and the rest passes into the second medium. When it curvature R and after refraction, its image is formed in a
passes into the second medium, it either bends towards the medium of refractive index n2 at a distance v, then
normal or away from the normal. n2 n1 n2 − n1
− =
This phenomenon is known as refraction. v u R
The relation is true for all surfaces, whether the image formed
Rarer Denser is real or virtual.
i i
medium medium

Denser Rarer Lens


medium medium r
r A lens is part of a transparent refracting medium bound by
two surfaces, with atleast one of the two surfaces being a
(a) (b) curved one. The curved surface may be spherical or
cylindrical.
DAY TWENTY EIGHT RAY OPTICS 301

1 1 1
The lens formula is given by − =
v u f
Power of a Lens
The power of a lens is mathematically given by the
For a thin object of height h placed perpendicular to the 1
principal axis at a distance u, if the height of image formed is h′, reciprocal of its focal length, i.e. power P =
f (m)
then lateral or transverse magnification m is given by
h′ v f f −v SI unit of power is dioptre (D). Power of a converging lens is
m= = = = positive and that of a diverging lens is negative.
h u f +u f
For a small sized object placed linearly along the principal axis,
its axial or longitudinal magnification is given by
Lens Maker’s Formula
2 2 2 For a lens having surfaces with radii of curvature R1 and R2
dv  v   f   f − v respectively, its focal length is given by
Axial magnification = − =  =  = 
du  u   f + u  f  1  1 1
P = = (n21 − 1)  − 
f  R1 R2 
Silvering of Lens where, n21 =
n2
= refractive index of the lens material w.r.t.
n1
When one surface of a lens is silvered, it behaves as a mirror.
The focal length of silvered lens is the surroundings.
1 2 1
F
= +
fl fm
Cutting and Combination of a Lens
If a symmetrical convex f 2f 2f f f
In case of plano-convex lens
lens of focal length f is
l
When curved surface is silvered, then focal length of cut into two parts along
silvered lens is its optic axis, then focal
R
F = length of each part
2µ (a plano-convex lens) is
2 f [fig(b)]. However, if the (a) (b) (c) (d)
two parts are joined as
+ shown in the figure (c) and (d),
the focal length of the combination is again f .
If a symmetrical convex lens of focal length f is cut into two
F fl fm parts along the principal axis, then the focal length of each
part remains unchanged, as f [Fig. f].
where, R = 2 fm or R = fl (µ − 1)
If these two parts are joined with the curved ends on one
l
When plane surface is silvered, then
f
R R side, the focal length of the combination is [Fig. g]. But on
F = and fl = , fm = ∞ 2
2(µ − 1) (µ − 1) joining the two parts in opposite sense, the net focal length
becomes ∞ (or net power = 0) (Fig. h).
f f f ∞
+ 2

F fl fm

l
When double convex lens is silvered, f
R R (e) (f) (g) (h)
F = and fl =
2(2µ − 1) 2(µ − 1) l
The equivalent focal length of co-axial combination of two
R 1 1 1 d
⇒ fm = . lenses is given by = + −
2 F f1 f2 f1 f2
f1 f2

+
d < f1 d < f2

O1 O2
F fl fm

d
302 40 DAYS ~ NEET PHYSICS DAY TWENTY EIGHT

l
If a number of lenses are in contact, then Common Examples of Total Internal Reflection
1 1 1
= + + ... l
Looming An optical illusion in cold countries.
F f1 f2 l
Mirage An optical illusion in deserts.
l
If two thin lenses of focal lengths f1 and f2 are in contact, l
Brilliance of diamond Due to repeated internal reflections
then their equivalent focal length diamond sparkles.
1 1 1
= + l
Optical fibre Each fibre consists of core and cladding. The
feq f 1 f2
refractive index of core material is higher than that of
In terms of power, Peq = P1 + P2 cladding. Light entering at small angle on one end
undergoes repeated total internal reflections along the fibre
Important Results and finally comes out.
l
Effective diameter of light transmitting area is called
aperture. Intensity of image ∝ (Aperture)2 Deviation by a Prism
2
 f  A prism is a homogeneous, A
l
Relation between object and image speed, vi =   vo transparent medium
 f + u
bounded by two plane n n′
l
Newton’s formula states, f 2 = x 1 x2 where, x 1 and x2 = surfaces inclined at an δ
distance of object and image from first and second i i′
angle A with each other.
r r′
principlal foci. This formula is also called thin lens These surfaces are called as
formula. refracting surfaces and the
l
If lens immersed in a liquid, then angle between them is
B C
fl (aµ g − 1) (aµ g − 1) called angle of prism A.
= =
fa (l µ g − 1)  aµ g  Deviation produced by a prism is, δ = i + i′ − A ⇒ r + r ′ = A
 − 1
 aµ l  For grazing incidence i = 90 °
l
Displacement method for finding focal length of lens is and grazing emergence i′ = 90°
D2 − x2 For minimum deviation
f = δ + A
sin  m 
4D  2 
(i) i = i′ and r = r ′ (ii) µ =
A
sin
Total Internal Reflection (TIR) 2
In case of minimum deviation, ray is passing through prism
When a ray of light goes from a denser to a rarer medium, it
symmetrically.
bends away from the normal.
For maximum deviation (δ max ), i = 90 ° or i′ = 90 °
For a certain angle of incidence iC , the angle of refraction in
rarer medium becomes 90°. The angle iC is called the critical For thin prism, δ = (µ − 1) A
angle.
Rarer
Dispersion by a Prism
r
90° n1 Dispersion of light is the phenomenon of splitting of white light
into its constituent colours on passing light through a prism.
i ic i r This is because different colours have different wavelength and
hence different refractive indices.
Denser Angular dispersion = δ v − δ r = (nv − nr ) A
n2 where, n v and n r represent refractive index for violet and red
n1 1 lights.
sin iC = = n − nr n + nr
n2 n21 Dispersive power, ω = v , where n = v is the mean
n−1 2
For the angle of incidence greater than the critical angle
refractive index.
(i > iC ) in the denser medium, the light ray is totally internally
reflected back into the denser medium itself. By combining two prisms with angle A and A′ and refractive
index n and n′ respectively, we can create conditions of
Conditions for Total Internal Reflection (n − 1) A
l
Dispersion without deviation when, A′ = −
l
The light ray should travel from the denser medium (n′ − 1)
towards the rarer medium. l
Deviation without dispersion when,
l
The angle of incidence should be the greater than the
 n − nr 
critical angle. A′ = −  v A
 n′v − n′r 
DAY TWENTY EIGHT RAY OPTICS 303

eye near point is at 25 cm, but far point comes closer to eye
Refraction Through a Prism from infinite. Causes of near sightedness are
A ray of light suffers two refractions at the two surfaces on (i) decrease in focal length of eye lens, or
passing through a prism. Angle of deviation through a prism (ii) elongation of eye ball. The defect can be rectified by use
δ = i + e − A. diverging lens.
where, i is the angle of incidence, e is the angle of emergence
If a myopic person cannot see objects situated beyond a
and A is the angle of prism.
distance x, then he should use lens of focal length f = − x.
So, concave lens is used to correct this.
Scattering of Light l
A person suffering from hypermetropia or long sightedness
Molecules of a medium after absorbing incoming light can see distant objects clearly, but cannot see nearby
radiations, emit them in all directions. This phenomenon is objects distinctly. It means that his near point has shifted
called scattering. According to Rayleigh, intensity of scattered away from 25 cm to distance x. Possible causes of
1 hypermetropia are
light ∝ 4 .
λ (i) increase in focal length of eye lens, or
There are some phenomenon based on scattering (ii) shortening of eye ball.
l
Sky looks blue due to scattering. To rectify hypermetropia, the person should use a
l
At the time of sunrise and sunset, sun looks reddish. converging lens of focal length f , so that it forms virtual
image of an object situated at distance (D = 25 cm) at near
l
Danger signals are made of red colour. xD
point x of defective eye, f = .
x−D
Human Eye So, convex lens is used for its correction.
The human eye is one of the most valuable and sensitive
Some Other Defects of Vision
sense organs of human being. The cornea, acqueous humour,
crystalline lens and vitreous humour together form a lens l
Presbyopia is due to weakness of ciliary muscles due to
system which forms an inverted and real image on retina for advancing age. Generally, defective eye suffers from near
the objects situated in front of eye. sightedness as well as long sightedness. To rectify the
The optic nerve transmits this image to the brain which defect bifocal lenses are used.
makes it erect and analyses it. l
Astigmatism is due to non-uniformity of curvature of the
Cornea cornea. The defective eye cannot concentrate
Iris simultaneously along horizontal as well as vertical. To
Retina
Vitreous rectify this defect, cylindrical lenses are used.
Humour
Yellow spot
Pupil Crystalline
lens
Defects of Images
Blind spot The two types of defects which are occurred commonly,
Optic nerve given below
Aqueous
humour Ciliary muscles Spherical Aberration
This defect is present in spherical mirrors as well as in
The eye lens has power of accommodation to adjust its focal lenses, whose aperture is comparatively large.
length, so as to focus objects situated at different distances
In mirrors, the spherical aberration can be almost eliminated
from eye at the retina.
by taking a paraboloidal mirror instead of a spherical mirror.
l
The least distance of distinct vision (D) or near point of an
L
eye is generally 25 cm. Far point of a normal eye is at Marginal rays
infinity. Ciliary muscles play an important role in changing
focal length of eye lens. Paraxial rays
Fp
C Fm
1
Persistence of vision is s, i.e. if time interval between
10
two consecutive light pulses is lesser than 0.1 s, eye cannot In a lens, we can minimise the aberration by
distinguish them separately.
 1 °
(a) using plano-convex lens with plane surface on the side of
Resolving limit for eye is 1′ = more convergent/divergent light beam.
60 
(b) by using a combination of two lenses separated by a
l
A person suffering from myopia can see near objects clearly distance d = f1 − f2 .
but can not see distant objects. For a myopic (near sighted)
304 40 DAYS ~ NEET PHYSICS DAY TWENTY EIGHT

Chromatic Aberration 2. Compound Microscope


L
This defect is present only in White It consists of two lenses of small focal length and small
lenses. apertures. Also, the focal length and aperture of objective
FR
Chromatic aberration is the C lens are smaller than that of eyepiece. The image formed
inability of a lens to focus rays of FV by the objective lens is real, inverted and magnified. This
different colours/ wavelengths at a image acts as the object for the eyepiece and the final
single point. image is highly magnified, virtual and inverted w.r.t. the
Longitudinal chromatic aberration = fR − fV = ωf , where original object.
ω = dispersive power of lens material and f = mean focal vo ue Eyepiece
length of the lens. uo fo

Chromatic aberration can be removed by preparing a


combination of two thin lenses of different materials h F
h' q
(of dispersive powers ω and ω′) and different focal lengths f
and f ′, such that
ω ω′ Objective
+ =0
f f′

Image
Optical Instruments
If m o and m e be the magnifications produced by the
An optical instrument is used to enhance and analyses the objective and the eyepiece respectively, then total
light waves. The light waves are in the form of photons, magnification of microscope m = m o × m e .
hence optical instruments also determine the characteristics
If final image is formed at the near point (D) of the eye,
properties of light waves.
then
v  D
Microscope m = − o 1 + 
uo  fe 
It is an optical instrument which forms a magnified image of
L  D
a small nearby object and thus, increases the visual angle or m=− 1 +  (approx)
fo  fe 
subtended by the image at the eye, so that the object is seen
to be bigger and distinct. If final image is formed at infinity, then
v D L D
m =− o ⋅ = − ⋅ (approx)
1. Simple Microscope (Magnifying Glass) uo fe fo fe
It consists of a single convex lens of small focal length and Length of tube of microscope, L = vo + ue .
forms a magnified image of an object placed between the
optical centre and the principal focus of the lens. • Huygens’ eyepiece is free from chromatic and spherical
NOTE
aberration, but it cannot be used for measurement
A'
purposes.
L
• Ramsden’s eyepiece can be used for precise measurement
as cross wires can be fixed in this eyepiece. It slightly suffers
A
A'' from spherical and chromatic aberrations.
b
a F
B'
F B C Telescope
Telescope is an optical instrument which increases, the
f visual angle at the eye by forming the image of a distant
object at the least distance of distinct vision, so that the object
If the image is formed at the near point of eye, then
is seen distinct and bigger.
 D
m = 1 + 
 f 1. Refracting Telescope
But, if the image is formed at infinity, then It consists of an objective lens of large focal length fo and
D large aperture.
m=
f The eyepiece consists of a convex lens of small aperture and
small focal length fe . Distance between the two lenses is set
where, D = normal viewing distance (25 cm),
f = focal length of magnifying lens. as, L = fo + fe
DAY TWENTY EIGHT RAY OPTICS 305

In normal adjustment, the final image is formed at infinity resolving power. Limit of resolution of a normal human eye
and magnifying power of the telescope is is 1′ .
f The minimum distance (or angular distance) between two
m=− o
fe points of an object whose images can be formed distinctly by
the lens of an optical instrument, is called its limit of
In practical adjustment, the final image is formed at the near
resolution.
point of the observer’s eye. In this arrangement,
fo  f 
m=−
fe
1 + e 
 D
Resolving Power of a Telescope
If the aperture (diameter) of the telescope objective be the D,
2. Reflecting Telescope then the minimum angular separation (dθ) between two
It consists of an objective which is a large paraboloid concave distant objects, whose images are just resolved by the
mirror of maximum possible focal length fo and the eyepiece telescope, is
is a convex lens of small focal length and small aperture, then 1. 22 λ
dθ =
f D
Magnifying power, m=− o
fe and resolving power of the telescope,
1 D
Reflecting type telescope is considered superior as it is free RP = =
from spherical and chromatic aberrations, is easy to install dθ 1. 22 λ
and maintain, and can produce image of greater intensity.
Resolving Power of a Microscope
NOTE
• The large aperture of telescope objective, helps in forming
The least distance (d) between two points, whose images
a brighter image.
are just seen distinctly by a microscope is given by
• If diameter of pupil of human eye is d and that of telescope
2
1. 22 λ
be D, then image formed by telescope will be   times
D
d=
 d 2 nm sin θ
brighter than the image of the same object, seen directly by
where,
the unaided eye.
λ = wavelength of light used to illuminate the object,
nm = refractive index of the medium between the object and
Resolving Power of the objective lens, and
an Optical Instrument θ = semi angle of the cone of light from the point object.
Resolving power of an optical instrument is its ability to The term nm sin θ is generally called the numerical aperture of
produce distinct images of two points of an object (or two the microscope.
nearby objects) very close together. Resolving power of an ∴ Resolving power of the microscope,
optical instrument is inverse of its limit of resolution. 1 2 nm sin θ (NA)
Smaller the limit of resolution of a device, higher is its RP = = =
d 1. 22 λ 0.61 λ

DAY PRACTICE SESSION 1

FOUNDATION QUESTIONS EXERCISE


1 A ray of light is successively deflected from two plane mirrors move through a distance y on the scale. The angle θ is
inclined to each other at a certain angle. If the total deviation given by j
NEET 2017
in the path of the rays reflected from the two mirrors be 300°, y y x x
(a) (b) (c) (d)
then what is the number of images formed? 2x x 2y y
(a) 30 (b) 15 (c) 11 (d) 5 3 A concave mirror forms the real image of an object which
2 A beam of light from a source L is incident normally on a is magnified 4 times. The object is moved 3 cm away, the
plane mirror fixed at a certain distance x from the source. magnification of the image is 3 times. What is the focal
The beam is reflected back as a spot on a scale placed length of the mirror?
just above the source L. When the mirror is rotated (a) 3 cm (b) 4 cm (c) 12 cm (d) 36 cm
through a small angle θ, the spot of the light is found to
306 40 DAYS ~ NEET PHYSICS DAY TWENTY EIGHT

4 The image formed by a convex mirror of focal length 12 A concave mirror of focal length f1 is placed at a distance
30 cm is a quarter of the size of the object. The distance of d from a convex lens of focal length f2 . A beam of light
of the object from the mirror is coming from infinity and falling on this convex lens-
(a) 30 cm (b) 90 cm (c) 120 cm (d) 60 cm concave mirror combination returns to infinity. The
5 The frequency of a light wave in a material is 2 × 1014 Hz distance d must be equal j CBSE AIPMT 2012

(a) f1 + f2 (b) −f1 + f2 (c) 2f1 + f2 (d) −2f1 + f2


and wavelength is 5000 Å. The refractive index of
material will be 13 A convex lens and a concave lens, each having same
(a) 1.40 (b) 1.50 (c) 3.00 (d) 1.33 focal length of 25 cm, are put in contact to form a
6 A glass slab (µ = 1.5) of thickness 6 cm is placed over a combination of lenses. The power in dioptres of the
paper. What is the shift in the letters? combination is
(a) 25 (b) 50 (c) infinite (d) zero
(a) 4 cm (b) 2 cm
(c) 1 cm (d) None of these 14 If in a plano-convex lens, radius of curvature of convex
7 An air bubble in a glass slab with refractive index 1.5 (near surface is 10 cm and the focal length of the lens is 30 cm,
normal incidence) is 5 cm deep when viewed from one the refractive index of the material of the lens will be
surface and 3 cm deep when viewed from the opposite (a) 1.5 (b) 1.66
face. The thickness (in cm) of the slab is j NEET 2016 (c) 1.33 (d) 3
(a) 8 (b) 10 (c) 12 (d) 16 15 When a biconvex lens of glass having refractive index
8 A student measures the focal length of a convex lens by 1.47 is dipped in a liquid, it acts as a plane sheet of
putting an object pin at a distance u from the lens, and glass. This implies that the liquid must have refractive
measuring the distance v of the image pin. The graph index j
CBSE AIPMT 2012
between u and v plotted by the student should look like. (a) equal to that of glass (b) less than one
(c) greater than that of glass (d) less than that of glass
v (cm) v (cm)
16 Which of the following is not due to total internal
(a) (b) reflection? j
CBSE AIPMT 2011
(a) Difference between apparent and real depth of a pond
(b) Mirage on hot summer days
O O (c) Brilliance of diamond
u (cm) u (cm)
(d) Working of optical fibre
v (cm) v (cm)
17 If the critical angle for total internal reflection from a
(c) (d) medium to vacuum is 30°, the velocity of light in the
medium is
(a) 3 × 108 ms −1 (b) 1.5 × 108 ms −1
−1
O
u (cm)
O
u (cm) (c) 6 × 10 ms 8
(d) 3 × 108 ms −1

9 A lens has focal length 10 cm. An object is placed 15 cm 18 A ray of light travelling in a transparent medium of
in front of it. Where should a convex mirror be placed, so refractive index µ falls on a surface separating the
that image is formed at the object itself, when focal medium from air at an angle of incidence of 45°. For
length of convex mirror is 12 cm? which of the following value of µ, the ray can undergo
(a) 6 cm from lens (b) 8 cm from lens total internal reflection? j
CBSE AIPMT 2010
(c) 5 cm from lens (d) 4 cm from lens (a) µ = 1.33 (b) µ = 1.40 (c) µ = 1. 50 (d) µ = 1. 25

10 The distance of the image from the focus of a lens is x 19 A hollow prism is filled with water and placed in air. It will
and that of object is y. What is the nature of the graph y deviate the incident rays
versus x? (a) towards the base
(a) Straight line (b) Ellipse (b) away from base
(c) Parabola (d) Hyperbola (c) parallel to base
(d) towards or away from base depending on the location
11 A lens when placed on a plane mirror,
then object needle and its image 20 A thin prism of angle 7° and refractive index 1.5 is
coincide at 15 cm. The focal length of combined with another prism of angle θ and refractive
the lens is index 1.7. The emergent ray goes undeviated. What is
(a) 15 cm (b) 30 cm the value of θ?
(c) 20 cm (d) ∞ (a) 3° (b) 5° (c) 9° (d) 1°
DAY TWENTY EIGHT RAY OPTICS 307

21 What is the refractive index of a prism whose angle 27 For a normal eye, the cornea of eye provides a
A = 60° and angle of minimum deviation dm = 30°? converging power of 40 D and the least converging
1 1 power of the eye lens behind the cornea is 20 D. Using
(a) 2 (b) (c) 1 (d)
2 3 this information, the distance between the retina and the
22 A beam of light consisting of red, green and blue colours cornea-eye lens can be estimated to be j NEET 2013

is incident on a right angled prism. The refractive index of (a) 5 cm (b) 2.5 m (c) 1.67 cm (d) 1.5 cm
the material of the prism for the above red, green and 28 A person can see clearly objects only when they lie
blue wavelengths are 1.39, 1.44 and 1.47, respectively. between 50 cm and 400 cm from his eyes. ln order to
j CBSE AIPMT 2015 increase, the maximum distance of distinct vision to
A
infinity, the type and power of the correcting lens, the
Blue
person has to use, will be j NEET 2016

Green (a) convex, + 2.25 D (b) concave, – 0.25 D


Red (c) concave – 0.2 D (d) convex, + 0.15 D
45° 29 A microscope is focussed on a mark on a piece of paper
B C and then a slab of glass of thickness 3 cm and refractive
The prism will index 1.5 is placed over the mark. How should the
(a) separate the blue colour part from the red and green microscope be moved to get the mark in focus again?
colours (a) 1 cm upward (b) 4.5 cm downward
(b) separate all the three colours from one another (c) 1 cm downward (d) 2 cm upward
(c) not separate the three colours at all 30 In a compound microscope, the intermediate image is
(d) separate the red colour part from the green and blue (a) virtual, erect and magnified
colours (b) real, erect and magnified
23 The angle of a prism is A. One of its refracting surfaces is (c) real, inverted and magnified
silvered. Light rays falling at an angle of incidence 2A on (d) virtual, erect and reduced
the first surface returns back through the same path after 31 A simple telescope, consisting of an objective of focal
suffering reflection at the silvered surface. The refractive length 60 cm and a single eye lens of focal length 5 cm is
index µ of the prism is j
CBSE AIPMT 2014 focused on a distant object in such a way that parallel rays
1 emerge from the eye lens. If the object subtends an angle
(a) 2 sin A (b) 2 cos A c) cos A (d) tan A
2 of 2° at the objective, the angular width of the image is
24 A ray of light is incident at an angle of incidence i on one (a) 10° (b) 24° (c) 50° (d) (1/6)°
face of a prism of angle A (assumed to be small) and
32 The diameter of the moon is 3.5 × 10 km and its distance
3
emerges normally from the opposite face. If the refractive
from the earth is 3.8 × 10 5 km. Seen by a telescope
index of the prism is µ, the angle of incidence i , is nearly
having focal lengths of the objective and the eyepiece as
equal to j
CBSE AIPMT 2012
µA
40 mm and 1.0 cm respectively, the angular diameter of
(a) µ A (b) (c) A /µ (d) A/ 2µ the image of the moon will be approximately
2
(a) 2° (b) 10°
25 A glass prism ABC (refractive (c) 20° (d) None of these
B θ A
index 1.5), immersed in water
33 In an astronomical telescope in normal adjustment, a
(refractive index 4/3). A ray of straight black line of length L is drawn on inside part of
light is incident normally on face objective lens. The eyepiece forms a real image of this
C
AB. If it is totally reflected at face line. The length of this image is I. The magnification of the
AC, then telescope is j
CBSE AIPMT 2015
(a) sinθ ≥
8
(b) sinθ ≥
2 L L L+1 L
(a) +1 (b) −1 (c) (d)
9 3 I I L −1 I
3 2 8
(c) sinθ > (d) < sinθ <
2 3 9 34 The magnifying power of a telescope is 9. When it is
adjusted for parallel rays, the distance between the
26 To correct myopia, the focal length of the concave lens
objective and eyepiece is 20 cm. The focal length of
should be
lenses are j
CBSE AIPMT 2012
(a) equal to the distance of far point (a) 10 cm, 10 cm (b) 15 cm, 5 cm
(b) less than the distance of far point (c) 18 cm, 2 cm (d) 11 cm, 9 cm
(c) less than the distance of near point
(d) equal to the distance of near point
308 40 DAYS ~ NEET PHYSICS DAY TWENTY EIGHT

35 The focal length of objective lens is increased, then 36 The ratio of resolving powers of an optical microscope for
magnifying power of j CBSE AIPMT 2014 two wavelengths λ 1 = 4000 Å and λ 2 = 6000 Å is
(a) microscope will increase, but that of telescope decrease j NEET 2017
(b) microscope and telescope both will increase (a) 8 : 27 (b) 9 : 4
(c) microscope and telescope both will decrease (c) 3 : 2 (d) 16 : 81
(d) microscope will decrease, but that of telescope will
increase

DAY PRACTICE SESSION 2

PROGRESSIVE QUESTIONS EXERCISE


1 A room (cubical) is made of mirrors. An insect is moving How fast is the light travelling in the liquid?
along the diagonal on the floor, such that the velocity of (a) 1.8 × 108 ms −1 (b) 2.4 × 108 ms −1
image of insect on two adjacent wall mirrors, is 10 cms −1. (c) 3.0 × 108 ms −1 (d) 1.2 × 104 ms −1
The velocity of image of insect in ceiling mirror is
5 For an optical arrangement as shown in the figure. Find
(a) 10 cms −1 (b) 20 cms −1
the position and nature of image.
10
(c) cms −1 (d) 10 2 cms −1
2
µ=1 µ = 1.33
2 In the given figure, the angle of reflection is 1 cm
y O C

C
y = x2 20 cm
1, 1
2 4 40 cm
B (a) 32 cm (b) 0.6 cm (c) 6 cm (d) 0.5 cm
x
6 A thin plano-convex lens of focal length f is split into two
(a) 30° (b) 60°
halves. One of the halves is shifted along the optical axis.
(c) 45° (d) None of these
The separation between object and image plane is 1.8 m.
3 A 4 cm thick layer of water The magnification of the image formed by one of the half
covers a 6 cm thick glass 4 cm h1 Water lens is 2. Find the focal length of the lens and separation
slab. A coin is placed at the between the two halves.
bottom of the slab and is
being observed from the air 6 cm h2 Glass
side along the normal to the
surface. Find the apparent O
Coin
position of the coin from
(a) 7.0 cm (b) 8.0 cm (c) 10 cm (d) 5 cm
1.8m
4 A small coin is resting on the bottom of a beaker filled
with a liquid. A ray of light from the coin travels upto the (a) 0.1 m (b) 0.4 m (c) 0.9 m (d) 1 m
surface of the liquid and moves along its surface as 7 A thin convergent glass lens ( µ = 1.5) has a power of
shown in figure. + 5.0 D. When this lens is immersed in a liquid of
3 cm refractive index µl , it acts as a divergence lens of focal
length 100 cm. The value of µl should be
(a) 3/2 (b) 4/3 (c) 5/3 (d) 2
4cm

8 A thin glass prism (µ = 1.5) is immersed in water (µ = 1.3).


If the angle of deviation in air for a particular ray be D,
then that in water will be
Coin (a) 0.2D (b) 0.3D (c) 0.5 D (d) 0.6 D
DAY TWENTY EIGHT RAY OPTICS 309

9 A ray of light incident at an angle θ on a refracting face of 16 Two identical glass ( µ g = 3 / 2) equi-convex lenses of
a prism emerges from the other face normally. If the focal length f each are kept in contact. The space
angle of the prism is 5° and the prism is made of a between the two lenses is filled with water ( µw = 4 / 3).
material of refractive index 1.5, the angle of incidence is The focal length of the combination is j NEET 2016

4f 3f
(a) 7.5° (b) 5° (a) f / 3 (b) f (c) (d)
(c) 15° (d) 2.5° 3 4

10. The refracting angle of a prism is A and refractive index 17 A plano-convex lens fits exactly into a plano-concave
lens. Their plane surfaces are parallel to each other. If
of the material of the prism is cot(A/2). The angle of
lenses are made of different materials of refractive
minimum deviation is j CBSE AIPMT 2015

indices µ1 and µ 2 ; and R is the radius of curvature of the


(a) 180°− 3 A (b) 180°− 2 A
curved surface of the lenses, then the focal length of the
(c) 90°− A (d) 180° + 2 A
combination is j NEET 2013

11 Two identical thin plano-convex glass lenses (refractive R R R 2R


(a) (b) (c) (d)
index 1.5) each having radius of curvature of 20cm are 2 (µ 1 + µ 2 ) 2 (µ 1 − µ 2 ) (µ 1 − µ 2 ) (µ 2 − µ 1)
placed with their convex surfaces in contact at the
18 A biconvex lens has a radius of curvature of magnitude
centre. The intervening space is filled with oil of refractive
20 cm. Which one of the following options describe best
index 1.7. The focal length of the combination is
the image formed of an object of height 2 cm placed
j CBSE AIPMT 2015
30 cm from the lens? j CBSE AIPMT 2011

(a) –20 cm (b) –25 cm


(a) Virtual, upright, height = 05
. cm
(c) –50 cm (d) 50 cm (b) Real, inverted, height = 4 cm
(c) Real, inverted, height = 1cm
12 The radius of curvature of the curved surface of a (d) Virtual, upright, height = 1cm
plano-convex lens is 20 cm. If the refractive index of the
material of the lens be 1.5, it will 19 A lens having focal length f and aperture of diameter d
d
(a) act as a convex lens only for the objects that lie on its forms an image of intensity I. Aperture of diameter in
2
curved side
(b) act as a concave lens for the objects that lie on its central region of lens is covered by a black paper. Focal
curved side length of lens and intensity of image now will be
(c) act as a convex lens irrespective of the side, on which respectively j
CBSE AIPMT 2010
the object lies I 3f I 3I f I
(a) f and (b) and (c) f and (d) and
(d) act as a concave lens irrespective of side, on which the 4 4 2 4 2 2
object lies 20 An astronomical refracting telescope will have large
13 An thin prism having refracting angle 10° is made of angular magnification and high angular resolution, when
glass of refractive index 1.42. This prism is combined it has an objective lens of j
NEET 2018
with another thin prism of glass of refractive index 1.7. (a) large focal length and large diameter
(b) large focal length and small diameter
This combination produces dispersion without deviation.
(c) small focal length and large diameter
The refracting angle of second prism should be
(d) small focal length and small diameter
j
NEET 2017
(a) 4° (b) 6° 21 The refractive index of the material of a prism is 2 and
(c) 8° (d) 10° the angle of the prism is 30°. One of the two refracting
surfaces of the prism is made a mirror inwards, by silver
14 An astronomical telescope has objective and eyepiece of coating. A beam of monochromatic light entering the
focal lengths 40 cm 4 cm, respectively. To view an object prism from the other face will retrace its path (after
200 cm away from the objective, the lenses must be reflection from the silvered surface), if its angle of
separated by a distance j
NEET 2016 incidence on the prism is j
NEET 2018
(a) 46.0 cm (b) 50.0 cm (a) 30° (b) 45° (c) 60° (d) zero
(c) 54.0 cm (d) 37.3 cm
22 An object is placed at a distance of 40 cm from a
15 The angle of incidence for a ray of light at a refracting concave mirror of focal length 15 cm. If the object is
surface of a prism is 45°. The angle of prism is 60°. If the displaced through a distance of 20 cm towards the
ray suffers minimum deviation through the prism, the
mirror, the displacement of the image will be j NEET 2018
angle of minimum deviation and refractive index of the
(a) 30 cm towards the mirror
material of the prism respectively, are j
NEET 2016
(b) 36 cm away from the mirror
(a) 30°; 2 (b) 45°; 2 (c) 30 cm away from the mirror
1 1
(c) 30°; (d) 45°; (d) 36 cm towards the mirror
2 2
310 40 DAYS ~ NEET PHYSICS DAY TWENTY EIGHT

ANSWERS
SESSION 1 1 (c) 2 (a) 3 (d) 4 (b) 5 (c) 6 (b) 7 (c) 8 (a) 9 (a) 10 (d)
11 (a) 12 (c) 13 (d) 14 (c) 15 (a) 16 (a) 17 (b) 18 (b) 19 (a) 20 (b)
21 (a) 22 (d) 23 (b) 24 (a) 25 (a) 26 (a) 27 (c) 28 (b) 29 (a) 30 (c)
31 (b) 32 (a) 33 (d) 34 (c) 35 (d) 36 (c)
SESSION 2 1 (d) 2 (c) 3 (a) 4 (a) 5 (b) 6 (b) 7 (c) 8 (b) 9 (a) 10 (b)
11 (c) 12 (c) 13 (b) 14 (c) 15 (a) 16 (d) 17 (c) 18 (b) 19 (c) 20 (a)
21 (b) 22 (b)

Hints and Explanations


1 δ = 360° − 2θ. f 1 30 8 From the lens formula,
4 ∴m = ⇒ = ⇒ u = − 90 cm
f −u 4 30 − u 1 1 1
Here, δ = 300° , − = = constant
Hence, θ = 30°. v u f
5 Velocity of light waves in material is
So, the number of images v = νλ …(i) u is always negative, v is positive.
360
n= − 1 = 11. Refractive index of material is 9 1= 1 − 1 = 1
30 c v 10 15 30
µ = …(ii)
v
2 When the plane mirror is rotated by an
where, c is speed of light in vacuum or
angle θ, the reflected ray or beam of light
air.
must rotate by angle 2θ, from refraction
c
at plane surface theory. or µ =
νλ I
From the figure
BS y Given, ν = 2 × 1014 Hz 24 cm
tan 2θ = =
SO x λ = 5000 Å = 5000 × 10−10 m,
c = 3 × 108 ms −1 15 cm 30 cm
If the angle is small tan 2θ ≈ 2θ
y ⇒ v = 30 cm.
So, 2θ = This gives µ = 3
x We have to place the convex mirror in
 
y 6 Shift = x  1 − 1  = 6  1 − 1  such a way that it receives normal
⇒ θ=  µ  3/2 radiation. Hence, we shall place at a
2x
1 distance of radius of curvature (24 cm)
= 6 × = 2 cm from I (position of image) or 6 cm from
B 3
lens.
7 Let thickness of the given slab is t.
y
According to the question, when viewed 10 According to Newton’s formula, xy = f 2 .
q q from both the surfaces Hence, y versus x graph is a rectangular
q hyperbola.
S x O Glass slab
11 When the object is placed at the focus,
the rays are parallel.
L x

f
3 In the first case, m = =4
u− f Bubble
f x t − x t
In the second case, =3 + = 3 + 5 ⇒ = 8 cm
(u + 3) − f µ µ µ The mirror placed normal sends them
3 back. Hence, image is formed at the
On solving, we find f = 36cm ⇒ t = 8 × µ ⇒ t = 8 × = 12 cm
2 object itself.
DAY TWENTY EIGHT RAY OPTICS 311

12 According to question, the ray diagram 21 Refractive index of prism, i


µ =
will be A+δ 60° + 30° A
m
sin sin i = µA
µ = 2 = 2
A 60° 25 For total internal reflection at AC face,
sin sin
2 2 µw 4
sin i ≥ ⇒ sin θ ≥
1 µg 3 × 1.5
f2 2f1 sin 45° 1 2
= = 2 = × = 2 sin θ ≥
8
So, distance d between convex lens and sin 30° 1 2 1 9
concave mirror is given by 2
d = 2 f1 + f2 26 To correct myopia focal length should
22 For refractive index of a prism, be equal to the distance of far point.
13 Focal length of combination of lenses 1 1
µ = = = 2 = 1.414
placed in contact, is sin ic sin 45° 27 Given, P1 = 40 D and P2 = 20 D
1 1 1 We have, Peff = 40 D + 20 D = 60 D
= + A
F f1 f2 100 100
∴ f = = = 1.67 cm
For convex lens, f1 = 25 cm Peff 60
For concave lens, f2 = − 25 cm
1 1 1 i 28 Given, image distance, v = 400 cm = 4 m
Hence, = +
F 25 (−25) ⇒u=∞
1 1 45° Using lens equation,
= − =0
25 25 B C 1 1 1
− =
1 v u f
∴ F = =∞ As, µ red = 139
. , µ green = 1.44 and
0 1 1 1
µ blue = 1.47 ⇒ − =
Hence, power of combination,
∴(µ red = 139
. ) < µ, µ green > µ, µ blue > µ −4 ∞ f
1
P = =0 Thus, only red colour do not suffer total ⇒ f = −4m
F Now, power of the required lens is,
internal reflection.
14 For plano-convex lens, we have 1 1
23 According to question, diagram is shown P = = = − 0.25 D
1  1 f −4
= (µ − 1)   below.
f  R Thus, the person require a concave lens
M of power − 0.25 D.
Here, f = 30 cm, R = 10 cm,
This gives µ = 1.33. 29 Apparent depth of mark as seen through
A
)

a glass slab of thickness x and refractive


–A

15 If biconvex lens behaves like a plane N index µ is


sheet of glass, ray will pass undeviated


(9

through it only when medium has same O 90° Silvered Apparent depth
refractive index as that of biconvex lens. 2A r =A real depth 3
16 Real and apparent depth are explained = = = 2 cm
µ refractive index 1.5
on the basis of refraction only. The
concept of TIR is not involve here. So, ∠MON = 90°− A As image appears to be raised by 1 cm,

17 ∴ µ = 1 = 1 = 2 and ∠r = 90° − (90°− A ) therefore microscope must be moved


sin iC sin 30° ⇒ ∠r = A upward by 1 cm.
c sin i sin(2A )
Hence, v = By Snell’s law, =µ⇒ =µ 30 In compound microscope, intermediate
µ sin r sin( A )
image (image formed by objective lens)
−1 is real, inverted and magnified.
3 × 10 ms
8
= 2sin A cos A
= µ ⇒ µ = 2cos A
2 sin A 31 It is a case of normal adjustment.
= 1.5 × 108 ms −1 Hence, M = fo / fe .
24 Refractive index, Also, M = β/α.
18 For total internal reflection, i > C sin i sin i
µ = , µ = β f
sin r sin A Therefore, = o
⇒ sin i > sin C α fe
1 In case of small angle, sin i ≈ i and
⇒ sin 45° > sin A ≈ A Here, fo = 60 cm, fe = 5 cm, α = 2°
µ
Hence, β = 24°.
⇒ µ > 2 ⇒ µ > 1. 4
32 Q β = o
f
A
19 The prism deviates the light rays α fe
towards its base.
i Here, fo = 40 mm, fe = 1.0 cm = 10 mm
r=A
20 Here, ( µ 1 − 1) A 1 = ( µ 2 − 1)θ α = 3.5 × 103 / 3.8 × 105 rad. This gives
β ≅ 2°.
i.e. (1.5 − 1)7 = (1.7 − 1) θ.
Hence, θ = 5°.
312 40 DAYS ~ NEET PHYSICS DAY TWENTY EIGHT

33 3 Using equation, the total apparent shift 1.33 1 1.33


− =
is v (−40) −20
L
 1   1  After solving, v = − 32 cm
s = h1  1 −  + h2  1 −  h2 µ 1 v
I  µ1   µ2  The magnification is m = =
h1 µ2u
fo+fe  1   1 
or s = 4  1 −  + 6 1 −  h2 1(32)
 4 / 3  3 / 2 ∴ =
We know, magnification of telescope, h1 1.33 (−40)
f = 3.0 cm
we have M = o, or h2 = 0.6 cm
fe Air
The positive sign shows that the image
fe −I
Here, = is erect.
fe + u L 4 cm h1 Water
h 6 This is a modified displacement method
fe −I f I
⇒ = ⇒ e = problem.
fe − ( fo + fe ) L fo L
a+ d 2
L Here, a = 1.8 m and =
i.e. M = 6 cm h2 s Glass a−d 1
I
f Solving, we get d = 0.6 m
34 Given, magnification = o = 9 Coin a2 − d 2
fe ∴ f = = 0.4 m
4a
and fo + fe = 20 ⇒ fo = 9 fe Thus, h = h1 + h2 − s = 4 + 6 − 3
So, 9 fe + fe = 20 ⇒ fe = 2 cm = 7.0 cm 7 When the lens is in air, we have
∴ fo = 9 × 2 ⇒ fo = 18 cm 4 As shown in figure, a light ray from the 1 µg − µa  1 1 
Pa = =  − 
35 Magnifying power of microscope coin will not emerge out of liquid, if f µa R R
 1 2
L  P i > ic .
1 +  = When the lens is in liquid, we have
fo  fe 
1 µg − µl  1 1 
Magnifying power of telescope A R B Pl = =  − 
f  f  fl µl R
 1 R2
= o 1 + e 
fe  D i > ic Here, Pa = 5, Pl = − 1, µ a = 1,µ g = 1.5
So, the focal length of objective lens is h This gives, µ l = 5/ 3.
increased, then magnifying power of
microscope will decrease but that of 8 δ ≅ ( µ − 1)A
telescope will increase. For air, D = (1.5 − 1)A
S Coin
36 As, resolving power of a microscope For water,
1 Therefore, minimum radius R
(RP) ∝ corresponds to i = ic . δ = ( g µ w − 1)A
λ (wavelength)
R
RP1 λ 6000 3 In = tan ic or R = h tan ic  1.5 
∴ = 2 = = h = − 1 A
RP2 λ1 4000 2 h  1.3 
or R =
µ −1
2 0.2 D
Hence, δ = × ≅ 0.3 D
SESSION 2 1.3 0.5
1 Qv cos 45° = 10 ⇒ v = 10 2 cms −1 9 Here, A = 5° , µ = 1.5, i1 = ?
As, the ray emerges from the other face
µ of prism normally,
1
10 cms–1

v i2 = 0 (∴ r2 = 0°)
As, r1 + r2 = A
ic r1 = A − r2 = 5 − 0 = 5°
45°
sin i1
10 cms–1 µ2 –1 From µ = , sin i1 = µ sin r1
sin r1
5
In the ceiling mirror, the original Given, R = 3 cm, h = 4 cm ⇒ µ = = 1.5 × sin 5° = 1.5 × 0.087 = 01305
.
3
velocity will be seen. i1 = sin −1 (01305
. ) = 7.5°
c 3 × 10 8
But v = = = 1.8 × 108 ms −1
2 Given, y = x2 µ 5/ 3 A
dy 1
∴ = 2x = 2 × = 1 5 According to Cartesian sign convention, 5°
dx 2
u = − 40 cm , R = − 20 cm, N2
⇒ tanθ = 1 ⇒ θ = 45°
The slope of normal at B, µ 1 = 1, µ 2 = 1.33 N1 90°
Applying equation for refraction i1
1 1 r1
m2 = − = − = − 1 ⇒ θ = − 45° through spherical surface, we get
m 1 µ2 µ1 µ2 − µ1
− =
From the figure, angle of incidence is 45°. v u R
Hence, angle of reflection is 45°.
DAY TWENTY EIGHT RAY OPTICS 313

 A + δ 1  1 1  From the geometry of the prism and the


sin   = ( µ − 1)  − 
 2  f′  R1 R2  ray diagram, it is clear that
10 µ =
 A angle of incidence, i = 45°
sin    1 1 1
2 = (1.5 − 1)  +  = angle of refraction r = r ′ = 30°
 20 ∞  40
angle of emergence, e = 45°
 A + δ f ′ = 40 cm. The lens behaves as convex.
sin   Therefore, minimum deviation suffered
A  2 
⇒ cot = 13 Let prism angle of the first and second by the ray is
2  A prisms are A1 and A2 , respectively. δ min = i + e − (r + r ′)
sin  
2 Similarly, their refractive indices are = 90° − 60° = 30°
 A  sin  A + δ  µ 1 and µ 2 . Also, we know that,
cos    
2  2   A + δm 
= Condition for dispersion without sin  
 A  A deviation is δ1 − δ2 = 0  2 
sin   sin   µ =
2 2 ⇒ (µ 1 − 1) A1 − (µ 2 − 1) A2 = 0 A
sin
π A  A δ  µ − 1 2
⇒ sin  −  = sin  +   1.42 − 1 
2 2 2 2 ⇒ A2 =  1  ⋅ A1 =   (10° )
 µ2 − 1  1.7 − 1  where, µ = refractive index of the
δ = π − 2 A = 180°−2 A material of the prism.
⇒ A2 = 6°
1 1 A = angle of prism = 60°
11 ∴ = (1.5 − 1)  
f R 14 According to question,  60° + 30° 
sin  
Focal length of objective lens ( fo )  2  sin 45°
∴ µ = =
= + 40 cm 60° sin 30°
sin
Focal length of eyepiece lens ( fe ) 2
= 4 cm 1/ 2 2
= = = 2
Object distance for objective lens 1/2 2
(uo ) = − 200 cm 16 Consider the situation shown in figure.
Applying lens formula for objective lens, Let radius of curvature of lens surface is
1 0.5
= Objective lens Eyepiece lens R. The combination is equivalent to three
f lens R lenses in contact.
1  1 1
= (17
. − 1)  − −  4cm
fconcave  R R
µ=3/2
1 −07. × 2 −1.4
= = 200 cm v
fconcave R R
1 0.5 1.4 0.5
= − + 1 1 1 1 1 1
feq R R R − = ⇒ − =
vo uo f vo (−200) 40
−0.4 −0.4
= = 1 1 1 5− 1 4 f1 f3
R 20 ⇒ = − = =
vo 40 200 200 200 f2
⇒ feq = −50 cm
⇒ v o = 50 cm
12 Here, µ = 1.5 ∴
1
=
1
+
1
+
1
=
2
+
1
Image will be form at first focus of
feq f1 f2 f3 f1 f2
eyepiece lens.
So, for normal adjustment distance (Q f1 = f3 )
O O between objectives and eyepiece lense 1 1   1
2
(length of tube) will be Now, = = (µ − 1)   =
f1 f3  R f
v + fe ⇒ 50 + 4 = 54 cm
1  2 4   −2 
(a) (b) 15 Consider a ray of light PQ incident on = (µ w − 1)  −  =  − 1  
f2  R 3   R
the surface AB and moves along RS,
If object lies on plane side,
after passing through the prism ABC.  1   2   −2   1 
R1 = ∞, R2 = − 20 cm =   −  =    
A  3   R   3   2(µ − 1)
1  1 1 
= ( µ − 1)  −   
 1 2  1  1   1 
f R R 60° 1 2 1
⇒ = −     =− ×
1 1 f2  3   3 − 1   f 
= (1.5 − 1)  +  45° 30° 45° 3 f
 ∞ 20  P 30° 30° S 2 
1 Q R
= 1 2 2 1
40 ∴ = − ×
feq f 3 f
f = + 40 cm. The lens behaves as B C
convex. 6−2 4
It is given that the incident ray suffers = =
3f 3f
If object lies on its curved side, minimum deviation. Therefore, the ray
3f
R1 = 20 cm, R2 = ∞ inside the prism must be parallel to the ⇒ feq =
base BC of the prism. 4
314 40 DAYS ~ NEET PHYSICS DAY TWENTY EIGHT

17 Focal length of the combination, 3 22 Case I When the object distance,


⇒ I2 = I1
1 1 1 4 u1 = − 40 cm
= + …(i)
f f1 f2 and focal length remains unchanged. Focal length of mirror, f = − 15cm
We have,
1  1  1   µ1 − 1 20 Angular magnification of an O
= (µ 1 − 1)  −   = astronomical refracting telescope is
f1  ∞  −R   R f
1  1 1 given as M = o 40 cm
and = (µ 2 − 1)  −  fe
f2  −R ∞ 
where, fo and fe are the focal length of f=–15 cm
(µ − 1)
=− 2 objective and eyepiece, respectively. Using the mirror formula, we get
R
1 1 From the given relation, it is clear that 1 1 1
Putting these values of and in Eq. for large magnification either fo has to = +
f1 f2 f v1 u1
be large or fe has to be small.
(i), we get Substituting the given values, we get
Angular resolution of an astronomical
1 (µ 1 − 1) (µ 2 − 1) 1 1  −1 
= − refracting telescope is given as − = +  
f R R a 15 v 1  40 
[µ − 1 − µ 2 + 1] µ 1 − µ 2 R=
= 1 = 1.22 λ 1 1 1
⇒ = −
R R v1 40 15
R where, a is the diameter of the objective.
∴ f = Thus, to have large resolution, the 3− 8 −5
µ1 − µ2 = =
diameter of the objective should be large. 120 120
18 Given, R = 20 cm, Hence, from the above objective lens −120
⇒ v1 = = − 24 cm
ho = 2 cm and u = − 30 cm should have large focal length ( fo ) and 5
large diameter (a).
In general we have assumed µ = 1.5 Case II When the object distance,
1 2 1 21 According to the question, the figure of u2 = 20 cm
As, = (1.5 − 1) × =
f 20 20 mentioned prism is given as
O
So, f = 20 cm A
1 1 1
Now, as = − 20 cm
f v u 30°
R
1 1 1 r2=0 f=–15 cm
= + O
r1 m=√
20 v 30 i 2
Using the mirror formula, we get
1 1 1 10
= − = O¢ 1
=
1
+
1
v 20 30 600 P f v2 u2
∴ v = 60 cm B C
v 60 Substituting the given values, we get
As, m = = = −2 Here, r1 + r2 = A = 30°
u − 30 1 1  1
But r2 = 0 [reflected normally] − = + − 
h 15 v 2  20 
⇒ m = i = −2 ∴ r1 = 30°
1 1 1
ho From Snell’s law, ⇒ = −
v 2 20 15
∴ hi = − 2 × 2 = − 4 cm sin i1 µ 2 2
= = 3 − 4 −1
Here, image is real, inverted, magnified sin r1 µ 1 1 = =
and height of image is 4 cm. 60 60
⇒ sin i1 = 2 sin r1
⇒ v 2 = − 60 cm
19 As we know that, = 2 sin 30°
∴ The displacement of the image is
(Intensity) I ∝ A (Area exposed) 1
= 2× = v2 − v1
2
1 = − 60 − (−24)
πd 2 πd 2 / 4 =
A  − = − 60 + 24
I 4 4 3 2
⇒ 2 =  2= = = − 36 cm
I1  A1  πd 2 4 1
⇒ sin i1 = = sin 45° or i1 = 45°
2 = 36 cm away from the mirror
4
DAY TWENTY NINE

Wave Optics
Learning & Revision for the Day
u Wavefront u Young’s Double Slit Experiment u Diffraction
u Huygens’ Principle u Coherent Sources u Polarisation of Light
u Interference of Light u Interference in Thin Films u Brewster’s Law
u Polaroids

According to Huygens’, light is a form of energy, which travels in the form of waves
through a hypothetical medium ‘ether’. The medium was supposed to be all pervading,
transparent, extremely light, perfectly elastic and an ideal fluid.
Light waves transmit energy as well as momentum and travel in the free space with a
constant speed of 3 × 10 8 ms −1. However, in a material medium, their speed varies from
medium to medium depending on the refractive index of the medium.

Wavefront
A wavefront is the locus of all those points (either particles) which are vibrating in the
same phase. The shape of the wavefront depends on the nature and dimension of the
source of light.
l
In an isotropic medium, for a point source of light, the wavefront is spherical in nature.
l
For a line (slit) source of light, the wavefront is cylindrical in shape.
l
For a parallel beam of light, the wavefront is a plane wavefront.

Huygens’ Principle
Every point on a given wavefront, acts as secondary source of light and emits secondary PREP
wavelets which travel in all directions with the speed of light in the medium. A surface
touching all these secondary wavelets tangentially in the forward direction, gives the new
MIRROR
Your Personal Preparation Indicator
wavefront at that instant of time.
Laws of reflection and refraction can be determined by using Huygens’ principle. u No. of Questions in Exercises (x)—
u No. of Questions Attempted (y)—
Interference of Light u No. of Correct Questions (z)—
(Without referring Explanations)
Interference of light is the phenomenon of redistribution of light energy in space when
two light waves of same frequency (or same wavelength) emitted by two coherent u Accuracy Level (z / y × 100)—
sources, travelling in a given direction, superimpose on each other. If a1 and a2 be the u Prep Level (z / x × 100)—
amplitudes of two light waves of same frequency and φ be the phase difference
between them, then the amplitude of resultant wave is given by In order to expect good rank in NEET, your
Accuracy Level should be above 85 & Prep
AR = a21 + a22 + 2 a1a2 cos φ Level should be above 75.
316 40 DAYS ~ NEET PHYSICS DAY TWENTY NINE

and in terms of intensity of light, xd λ


Case II If = (2n − 1) , then we get nth dark fringe.
I R = I 1 + I2 + 2 I 1I2 cos φ. D 2
(2n − 1) Dλ
Hence, for nth dark fringe, x =
Condition for Constructive Interference 2d
If at some point in space, the phase difference between where, n = 1, 2, 3, …… .
two waves, φ = 0 ° or 2nπ or path difference between two
waves, ∆ = 0 or nλ, where n is an integer, then AR = a1 + a2 Fringe Width
or I R = I 1 + I2 + 2 I 1I2 is maximum. Such an interference is
The separation between any two consecutive bright or dark
called constructive interference. fringes is called fringe width β.
Condition for Destructive Interference Dλ
Thus, β=
d
If at some point in space, the phase difference between two
λ and for a given arrangement, it is constant, i.e. all fringes are
waves, φ = (2 n − 1)π or path difference, ∆δ = (2 n − 1) , then at uniformly spaced.
2
such points AR = (a1 − a2 ) and I R = I 1 + I2 − 2 I 1I2 is Moreover, fringe width β is
1
minimum leading to a destructive interference. (i) β ∝ D, (ii) β ∝ λ and (iii) β ∝ .
d
Amplitude Ratio Angular fringe width of interference pattern,
2 β λ
I max I 1 + I2 + 2 I 1I2  I 1 + I2  α= =
= =  D d
I min I 1 + I2 − 2 I 1 I2  I 1 − I2 
If in a given field of view n1, fringes of light of
2 2
 a + a2  r + 1 wavelength λ 1 are visible and n2 fringes of wavelength λ 2 are
= 1  =  visible, then n1λ 1 = n2 λ 2
 a1 − a2  r − 1
a
where, r = 1 = amplitude ratio. NOTE • If whole apparatus of Young’s double slit experiment is
a2
immersed in a transparent medium of refractive index nm,
NOTE • For identical sources, I1 = I2 = I0 Dλ
φ then fringe width in the medium, β m = .
• For constructive interference, Imax = 4 I0 and I = 4 I0 cos 2 nm d
2
• For destructive interference, Imin = 0
Coherent Sources
Young’s Double Slit Experiment Two light sources are said to be coherent, if they emit light of
The arrangement of young’s double slit experiment is shown exactly same frequency (or wavelength), such that the
in figure. Here monochromatic light of one wavelength is used. originating phase difference between the waves emitted by
P them is either zero or remains constant. For sustained
interference pattern, the interfering light sources must be
x
coherent one.
There are two possible techniques for obtaining coherent
S1 d light sources.
O
Light S S2 l
In division of wavefront technique, we divide the
source D wavefront emitted by a narrow source in two parts by
reflection, refraction or diffraction.
Bright and dark fringes are formed on the screen with central l
In division of amplitude technique, a single extended light
point O be having as the central bright fringe, because for O, beam of large amplitude is splitted into two or more waves
the path difference ∆ = 0. by making use of partial reflection or refraction.
For light waves reaching a point P, situated at a distance x
from central point ∆, the path difference, NOTE
• Two independent sources of light can never be coherent.
Two light sources can be coherent only, if these have been
xd
∆ = S2 P − S 1 P = derived from a single parental light source.
D
xd
Case I If = nλ, then we get nth bright fringe. Hence,
D Interference in Thin Films
position of bright fringes on the screen are given by the In white light thin films, whose thickness is comparable to
relation, x = nDλ ⋅ wavelength of light, show various colours due to interference
d of light waves reflected from the two surfaces of thin film.
DAY TWENTY NINE WAVE OPTICS 317

For interference in reflected light condition of constructive


interference (maximum intensity),
Diffraction due to Single Slit and
∆ = 2 nm t cos r = (2 n − 1)
λ Width of Central Maximum
2 Fraunhofer’s arrangement for studying diffraction at a single
Condition of destructive interference narrow slit (width = a) is shown in adjoining figure. Lenses L1
(minimum intensity), and L2 are used to render incident light beam parallel and to
λ focus parallel light beam.
∆ = 2 nm t cos r = (2 n)
2
W
For interference in refracted light condition of constructive A θ P
interference (maximum intensity),
λ a θ
O
∆ = 2 nmt cos r = (2 n)
2 S θ
Condition of destructive interference (minimum intensity), L1 B
f1
L2
λ
∆ = 2 nmt cos r = (2 n − 1) , where n = 1, 2, 3,… . Slit
f2
2 Screen
As a result of diffraction, we obtain a broad, bright maxima at
Shift in Interference Pattern symmetrical centre point O and on either side of it, we get
If a transparent thin sheet of thickness t and refractive index secondary diffraction maxima of successively falling intensity
nm is placed in the path of one of the superimposing waves and poor contrast, as shown in figure.
(say in front of slit S2 of Young’s double slit experiment), then I
it causes an additional path difference due to which
interference pattern shifts.
l
Additional path difference due to sheet = (nm − 1) t
D β
l
Fringe shift = (nm − 1)t = (nm − 1)t
d λ –ve O +ve
θ (or x)
l
If due to presence of thin film, the fringe pattern shifts by n
fringes, then l
Condition of diffraction minima is given by
(n − 1) t a sin θ = nλ
n= m
λ where, n = 1, 2, 3, 4,... .

or t = But the condition of secondary diffraction maxima is
(nm − 1) λ
a sin θ = (2 n + 1)
Shift is independent of the order of fringe and wavelength. 2
NOTE • Fresnel’s biprism is a device to produce coherent sources where, n = 1, 2, 3, 4,... .
by division of wavefront, l
Angular position of nth secondary minima is given by
d = 2 a ( n − 1) α λ
The distance between the coherent sources and screen, sin θ = θ = n
a
D=a+ b
Dλ λ ( a + b) nDλ nf2 λ
The fringe width is given by β = = and linear distance, x n = Dθ = =
d 2a ( n − 1) α a a
where, f2 is focal length of lens L2 and D = f2 .
l
Similarly, for nth maxima, we have
Diffraction (2 n + 1) λ (2 n + 1) Dλ (2 n + 1) f2 λ
sin θ = θ = and x n = =
Diffraction of light is the phenomenon of bending of light 2a 2a 2a
around the edges of an aperture or obstacle and entry of light 2λ
even in the region of geometrical shadow, when size of The angular width of central maxima is 2 θ =
a
aperture or obstacle is comparable to wavelength of light
used. 2 Dλ 2 f2 λ
or linear width of central maxima = =
Diffraction is characteristic of all types of waves. Greater the a 2
wavelength, more pronounced is the diffraction effect. It is
due to this reason that diffraction effect is very commonly NOTE • The angular width of central maxima is double as compared
observed in sound. to angular width of secondary diffraction maxima.
318 40 DAYS ~ NEET PHYSICS DAY TWENTY NINE

fully polarised (perpendicular to the plane of incidence),


Polarisation of Light provided
l
Light is an electromagnetic wave in which electric and µ = tan i p
magnetic field vectors very sinusoidally, perpendicular to
This relation represents Brewster’s law. Note that the parallel
each other as well as perpendicular to the direction of
components of incident light do not disappear, but refract into
propagation of wave of light.
the medium, with the perpendicular components.
l
The phenomenon of restricting the vibrations of light
(electric vector) in a particular direction, perpendicular to Law of Malus
the direction of wave motion is called polarisation of light.
When a beam of completely plane polarised light is incident
The tourmaline crystal acts as a polariser.
on an analyser, the resultant intensity of light (I ) transmitted
A from the analyser varies directly as the square of cosine of
D
E angle ( θ) between plane of transmission of analyser and
F polariser.
i.e. I ∝ cos2 θ
H G If intensity of plane polarised light incidenting on analyser is
B C I 0, then intensity of emerging light from analyser is I 0 cos2 θ.
Polarisation of Light
NOTE • We can prove that when unpolarised light of intensity I0
Thus, electromagnetic waves are said to be polarised when gets polarised on passing through a polaroid, its intensity
their electric field vector are all in a single plane, called the 1
becomes half, i.e. I = I0 .
plane of oscillation/vibration. Light waves from common 2
sources are upolarised or randomly polarised.

Plane Polarised Light Polaroids


The plane ABCD in which the vibrations of polarised light are Polaroids are thin and large sheet of crystalline polarising
confined is called the plane of vibration. It is defined as The material (made artifically) which are capable of producing
light, in which vibrations of the light (vibrations of electric plane polarised beams of large cross-section.
vector) when restricted to a particular plane the light itself is The important uses are
called plane polarised light. The vibrations in a plane l
These reduce excess glare and hence sun glasses are fitted
polarised light are perpendicular to the plane of polarisation. with polaroid sheets.
NOTE • Only transverse waves can be polarised. Thus, it proved
l
These are also used to reduce headlight glare of cars.
that light waves are transverse waves. l
They are used to improve colour contrast in old oil
paintings.
Brewster’s Law l
In wind shields of automobiles.
According to this law, when unpolarised light is incident at an l
In window panes.
angle called polarising angle, i p on an interface separating air
from a medium of refractive index µ, then the reflected light is
l
In three dimensional motion pictures.

DAY PRACTICE SESSION 1

FOUNDATION QUESTIONS EXERCISE


1 Two coherent monochromatic light beams of intensities I 3 The ratio of intensity at the centre of a bright fringe to the
and 4I are superposed. The maximum and minimum intensity at a point distance one-fourth of the distance
possible intensities in the resulting beam are between two successive bright fringes will be
(a) 5I and I (b) 5I and 3I (c) 9I and I (d) 9I and 3I (a) 4 (b) 3 (c) 2 (d) 1
2 If the equations of two light waves are y1 = 8 sin ω t and 4 A mixture of light consisting of wavelength 590 nm and
y 2 = 6 sin (ω t + φ ). Then, ratio of maximum and minimum an unknown wavelength illuminates Young’s double slit
intensity will be and gives rise to two overlapping interference patterns
(a) 11 : 49 (b) 49 : 1 on the screen. The central maximum of both light
(c) 7 : 1 (d) 1 : 7 coincide. Further, it is observed that the third bright
DAY TWENTY NINE WAVE OPTICS 319

fringe is known light coincides with the 4th bright fringe of 10 Two slits in Young’s experiment have widths in the ratio
an unknown light. From this idea, the wavelength of an 1 : 25. The ratio of intensity at the maxima and minima in
unknown light is I
the interference pattern max is
(a) 885.0 nm (b) 442.5 nm (c) 776.8 nm (d) 393.4 nm Imin j CBSE AIPMT 2015

9 121
5. In a Young’s experiment, two coherent sources are (a) (b)
4 49
placed 0.90 mm apart and the fringes are observed one 49 4
(c) (d)
metre away. If it produces the second dark fringe at a 121 9
distance of 1 mm from the central fringe, then wavelength
of monochromatic light used would be 11 In Young’s double slit experiment, when wavelength used
−4 is 6000 Å and the screen is 40 cm from the slits, the
(a) 60 × 10 cm 4
(b) 10 × 10 cm
(c)10 × 10−5 cm (d) 6 × 10−5 cm fringes are 0.012 cm wide. What is the distance between
the slits?
6 In the given figure, O ′ is the position of first bright range
(a) 0.024 cm (b) 2.4 cm
towards right from OP is the position of 5th bright fringe (c) 0.24 cm (d) 0.2 cm
on the other side of O with respect to O ′. If wavelength of
used light is 6000 Å, then value of S1B will be 12 In Young’s double slit experiment using sodium light
S1 ( λ = 5898 Å ), 92 fringes are seen. If given colour
B
( λ = 5461 Å ) is used, how many fringes will be seen?
S0 O O′ (a) 62 (b) 67
(c) 85 (d) 99
S2 ∠S1S2 B = 2° P
13 In Young’s experiment, two coherent sources are
(a) 2.4 × 10−4 m (b) 2.4 × 10−2 m 0.90 mm apart and fringes are observed at a distance
(c) 2.4 × 10−3 m (d) 2.4 × 10−6 m of 1 m, if 2nd dark fringe is at 1 mm distance from
7 A narrow slit S transmitting light of wavelength λ is central fringe, then wavelength of the monochromatic
placed a distance d above a large plane mirror as light will be
shown. The light coming directly from the slit and that (a)60 × 10−4 cm (b) 10 × 10−4 cm
after reflection interfere at P on the screen placed at a (c)10 × 10−5 cm (d) 6 × 10−5 cm
distance D from the slit. What will be x, for which first 14 In Young’s double slit experiment, the spacing between
maxima occurs? the slits is d and wavelength of light used is 6000 Å. If the
P angular width of a fringe formed on a distant screen is 1°,
then value of d is
S
x (a) 1 mm (b) 0.05 mm
d (c) 0.03 mm (d) 0.01 mm
O
15 In Young’s double slit experiment, the slits are 2 mm apart
D
and are illuminated by photons of two wavelength
λ 1 = 12000 Å and λ 2 = 10000 Å. At what minimum distance
4d λD 2d D from the common central bright fringe on the screen 2cm
(a) (b) (c) (d)
λD 4d D 2d from the slit will a bright fringe from one interference pattern
coincide with a bright fringe from the other? j
NEET 2013
8 In the Young’s double slit experiment, the intensity of
(a) 8 mm (b) 6 mm
light at a point on the screen, where the path difference (c) 4 mm (d) 3 mm
in λ is K (λ being the wavelength of light used). The
λ 16 Young’s double slit experimental arrangement is as
intensity at a point, where the path difference is , will be
4 shown in figure. If λ is the wavelength of light used and
j
CBSE AIPMT 2014 ∠S1CS 2 = θ, then the fringe width will be
K K S1
(a) K (b) (c) (d) zero
4 2
θ
9 The Young’s double slit experiment is performed with C
blue and with green light of wavelengths 4360 Å and
5460 Å, respectively. If x is the distance of 4th maxima S2
from the central one, then λ λ 2λ
(a) (b) (c) λθ (d)
(a) xblue = xgreen (b) xblue > xgreen θ 2θ θ
(c) xblue < xgreen (d) xblue / xgreen = 5460 / 4360
320 40 DAYS ~ NEET PHYSICS DAY TWENTY NINE

17 In Young’s double slit experiment, the separation d 24 For a parallel beam of monochromatic light of wavelength
between the slits is 2 mm, the wavelength λ of the light λ, diffraction is produced by a single slit whose width a is
used is 5896 Å and distance D between the screen and of the order of the wavelength of the light. If D is the
slits is 100 cm. It is found that the angular width of the distance of the screen from the slit, the width of the
fringes is 0.20°. To increase the fringe angular width to central maxima will be j CBSE AIPMT 2015

0.21° (with same λ and D) the separation between the 2Dλ Dλ


(a) (b)
slits needs to be changed to j NEET 2018
a a
(a) 2.1 mm (b) 1.9 mm (c) 1.8 mm (d) 1.7 mm DA 2Da
(c) (d)
λ λ
18 In Young’s double slit experiment, one of the slit is
covered with a transparent sheet of thickness 25 A beam of light λ = 600 nm from a distant source, falls on
3.6 × 10−3 cm due to which position of central fringe shifts a single slit 1 mm wide and the resulting diffraction
to a position originally occupied by 30th bright fringe. pattern is observed on a screen 2 m away. The distance
The refractive index of the sheet, if λ = 6000 Å, is between the first dark fringes on other side of the central
(a) 1.5 (b) 1.2 (c) 1.3 (d) 1.7 bright fringe, is j CBSE AIPMT 2014

(a) 1.2 cm (b) 1.2 mm


19 In an experiment of double slits, interference fringes are
(c) 2.4 cm (d) 2.4 mm
obtained by using light of wavelength 4800 Å. If the first
slit is covered with a thin sheet of glass having refractive 26 At the first minimum adjacent to the central maximum of a
index 1.4 and the second slit is covered with a sheet of single slit diffraction pattern, the phase difference
same thickness of refractive index 1.7, then the central between the Huygens’ wavelet from the edge of the slit
fringe is displaced to the position of 5th bright fringe. The and the wavelet from the mid-point of the slit is
thickness of glass will be j CBSE AIPMT 2015
−3 −3 π π
(a) 10.5 × 10 mm (b) 8 × 10 mm (a) rad (b) rad
(c) 6.5 × 10−3 mm (d) 2.5 × 10−3 mm 4 2
π
20 Young’s double slit experiment is first performed in air and (c) π rad (d) rad
8
then in a medium other than air. It is found that 8th bright
fringe in the medium lies, where 5th dark fringe lies in air. 27 A linear aperture whose width is 0.02 cm is placed
The refractive index of the medium is nearly j NEET 2017 immediately in front of a lens of focal length 60 cm. The
(a) 1.25 (b) 1.59 (c) 1.69 (d) 1.78 aperture is illuminated normally by a parallel beam of
wavelength 5 × 10 −5 cm. The distance of the first dark
21 What is necessary for easy occurrence of Fresnel’s
band of the diffraction pattern from the centre of the
diffraction?
screen is j
NEET 2016
(a) Obstacle should of the order of wavelength
(a) 0.10 cm (b) 0.25 cm
(b) Narrow opening should be of the order of wavelength
(c) 0.20 cm (d) 0.15 cm
(c) Source and screen should be at finite distance from the
obstacle 28 Unpolarised light is incident from air on a plane surface
(d) All of the above of a material of refractive index µ. At a particular angle of
22 In Fraunhoffer diffraction, the centre of diffraction image is incidence i , it is found that the reflected and refracted
(a) always bright (b) always dark rays are perpendicular to each other. Which of the
(c) sometimes bright and sometimes dark following options is correct for this situation? j NEET 2018
 1
(d) bright for large wavelength and dark for low wavelength (a) i = sin−1  
µ 
23 For Fraunhofer single slit diffraction?
(b) Reflected light is polarised with its electric vector
(a) width of central maxima is proportional to λ
perpendicular to the plane of incidence
(b) on increasing the slit width, the width of central maxima
(c) Reflected light is polarised with its electric vector
decreases
parallel to the plane of incidence
(c) on making the slit width a = λ, central maxima spreads
 1
in the range ± 90° (d) i = tan−1  
µ 
(d) All of the above
DAY TWENTY NINE WAVE OPTICS 321

DAY PRACTICE SESSION 2

PROGRESSIVE QUESTIONS EXERCISE


1 In Young’s double slit experiment, the intensity at a point, 5 In a double slit experiment, the two slits are 1mm apart
λ and the screen is placed 1 m away. A monochromatic light
where the path difference is (λ being the wavelength of
6 of wavelength 50 nm is used. What will be the width of each
I slit for obtaining ten maxima of double slit, within the central
light used) is I. If I 0 denotes the maximum intensity, is
I0 maxima of single slit pattern? j CBSE AIPMT 2015

equal to (a) 0.2 mm (b) 0.1 mm (c) 0.5 mm (d) 0.02 mm


3 1 3 1
(a) (b) (c) (d) 6 Two beams of light of intensity I1 and I 2 interfere to give
4 2 2 2
an interference pattern. If the ratio of maximum intensity
2 In the given figure, C is middle point of line S 1 S 2 . A 25 I
to that of minimum intensity is , then 1 is
monochromatic light of wavelength λ is incident on slits. 9 I2
The ratio of intensity of S 3 and S 4 is 5
(a) (b) 4
3
S4
81
(c) (d) 16
λ D 625
I0 S2
2 d
I0 7 Two polaroids P1 and P2 are placed with their axis
d S3
I0 C perpendicular to each other. Unpolarised light I 0 is
I0 S1 incident on P1. A third polaroid P3 is kept in between
D P1 and P2 such that its axis makes an angle 45° with that
of P1. The intensity of transmitted light through P2 is
(a) 0 (b) ∞ (c) 4 : 1 (d) 1 : 4 j
NEET 2017
I I I I0
3 Two coherent point sources S1 and S 2 , vibrating in phase (a) 0 (b) 0 (c) 0 (d)
2 4 8 16
emit light of wavelength λ. The separation between them is
2λ. The light is collected on a screen placed at a distance 8 The interference pattern is obtained with two coherent
D >> λ from the slit S1 as shown. The minimum distance, light sources of intensity ratio n. In the interference
so that intensity at P is equal to intensity at O I −I
pattern, the ratio max min will be
P Imax + Imin j
NEET 2016
n 2 n n 2 n
x (a) (b) (c) (d)
n+1 n+1 (n + 1)2 (n + 1)2
S1 S2
O 9 The intensity at the maximum in a Young’s double slit
2λ experiment is I 0. Distance between two slits is d = 5λ,
D where λ is the wavelength of light used in the experiment.
Q
What will be the intensity in front of one of the slits on the
(a) 2 D (b) 3 D (c) 8 D (d) 5 D screen placed at a distance D = 10 d ? j
NEET 2016
I0 3
4 Consider the arrangement as shown. The distance D is (a) (b) I0
4 4
large compared to d. Minimum value of d, so that there is I0
(c) (d) I 0
a dark fringe at O, is 2
P
B 10 In a diffraction pattern due to a single slit of width a, the
d first minimum is observed at an angle 30° when light of
A wavelength 5000 Å is incident on the slit. The first
O
x secondary maximum is observed at an angle of
D D j
NEET 2016
(a) sin−1   (b) sin−1  
2 1
λD 2 D 2λ  3  2
(a) (b) (c) (d)
Dλ 2λ (c) sin−1   (d) sin−1  
2 D 3 1
 4  4
322 40 DAYS ~ NEET PHYSICS DAY TWENTY NINE

ANSWERS
SESSION 1 1 (c) 2 (b) 3 (c) 4 (b) 5 (d) 6 (d) 7 (b) 8 (c) 9 (c) 10 (a)
11 (d) 12 (d) 13 (d) 14 (c) 15 (b) 16 (a) 17 (b) 18 (a) 19 (b) 20 (d)
21 (c) 22 (a) 23 (d) 24 (a) 25 (d) 26 (c) 27 (d) 28 (b)
SESSION 2 1 (a) 2 (b) 3 (b) 4 (a) 5 (a) 6 (d) 7 (c) 8 (b) 9 (c) 10 (c)

Hints and Explanations


SESSION 1 (2n − 1) λ D 12 ∴ n1 λ1 = n2 λ2
7 xn =
2d n1 λ
1 I max = ( I1 + I2 ) 2 = ( I + 4I )2 = 9I ⇒ = 2
n2 λ1
and I min = ( I1 − I2 ) 2
n1 5898
=( I − 4I ) 2 = I ⇒ = ⇒ n1 = 99
2d 92 5461
2 Here, a1 = 8, a2 = 6 13 Given, d = 0.90 × 10−3 m , D = 1 m
2 2
I  a + a2   8 + 6 49  3  X = 1 mm = 10−3 m
Q max =  1  =  8 − 6 = Virtual source  
I min a
 1 − a2    1  2
Put n = 1 and d = 2d as image of S will
3 Intensity at the centre of bright fringe, Dλ 2 × 10−3
be 2d apart, we get ⇒ X = ,=
I 0 = I + I + 2 II cos 0° λD λD d 3
∴ x1 = =
I 0 = 2I + 2I ⇒ I 0 = 4I 2(2d ) 4d 2 × 10−3 × 0.9 × 10−3
or λ=
Intensity at a point distant
P 3×1
8 Path difference λ means maxima,
4 = 6 × 10−7 m = 6 × 10−5 cm
2π π I max = K
(with a phase difference = = ), is φ 2π λ 1
4 2 I = K cos 2 = K cos 2  × × 14 sinθ ≈ θ = Y ⇒ ∆θ = ∆Y
π 2  λ 4 2  D D
I ′ = I + I + 2 II cos
2 2 π K Angular fringe width, θ 0 = ∆θ (width
= K cos =
⇒ I ′ = 2I + 2 II × 0 = 2I 4 2 ∆Y = β)
I0 β Dλ 1 λ π
∴ =
4I
=2 9 Distance of nth maxima, x = nλ D ∝ λ θ0 = = × = = 1° = rad
I ′ 2I d D d D d 180
λ = 6 × 10−7 m
4 As 4th bright fringe of an unknown As, λb < λ g ;∴ x blue < x green
λ 180
wavelength λ′ coincides with 3rd bright d = = × 6 × 10−7
fringe of a known wavelength 10 Given, YDSE experiment, having two θ0 π
λ = 590 nm, therefore slits of width are in the ratio of 1 : 25. = 3.44 × 10−5 m = 0.03 mm
4λ ′ D 3λD 3 3 So, ratio of intensity,
= ⇒ λ ′ = λ = × 590
d d 4 4 I1 W1 1 I2 25 15 Given, λ1 = 12000 Å, λ2 = 10000 Å,
= = ⇒ =
= 442.5 nm I2 W2 25 I1 1 D = 2 cm and d = 2 mm = 2 × 10−3 cm
5 Distance of nth dark fringe from central  
2 λ1 n 12000 6
I2 We have, = 1 = =
fringe,  + 1 λ2 n2 10000 5
I max ( I2 +
2
I1 ) I1
(2n − 1) λD ∴ = =  n λ D
xn = I min ( I2 − I1 )2  I2  As, x = 1 1
2d
 − 1 d
(2 × 2 − 1) λD 3λD  I1 
∴ x2 = = 5 × 12000 × 10−10 × 2
2d 2d 2 =
2 × 10−3
2
 5 + 1  6 36 9
3× λ ×1 ⇒   =  4  = 16 = 4
⇒ 1 × 10−3 =  5 − 1 = 5 × 1 . 2 × 104 × 10−10 × 103
2 × 0.9 × 10−3
I
Thus, max =
9 = 6 mm
⇒ λ = 6 × 10−5 cm
I min 4 16 Fringe width, β = Dλ
6 O ′ is the 4th fringe with respect to O. d
i.e. n = 4, d = 6000 × 10−10 m …(i)
11 ∴ β = λD ⇒ d = λD But here, θ =
d
⇒ d = Dθ
d β
D
Path difference, ∆ = S 1 B = nλ …(ii) 6000 × 10−10 × (40 × 10−2 ) Dλ λ
From Eqs. (i) and (ii), we get = = 0.2 cm ∴ β= =
0.012 × 10−2 Dθ θ
∆ = 4 × 6 × 10−7 = 2.4 × 10−6 m
DAY TWENTY NINE WAVE OPTICS 323

17 In a YDSE, angular width of a fringe is 24 sin θ = λ index = µ) and if reflected and refracted
given as a light are perpendicular to each other.
λ 1 θ d Then, the reflected light is completely
θ= ⇒θ∝ or 1 = 2 …(i)
d d θ2 d 1 plane polarised at a certain angle of
a θ 2λ incidence. This means, the reflected
Here, θ1 = 0.20 ° , θ2 = 0.21°, d 1 = 2 mm
light has electric vector perpendicular to
Substituting the given values in incidence plane.
Eq. (i), we get
0.20° d2
= y λ
= ⇒y =
λD SESSION 2
0.21° 2 mm D a a 2π 2π λ π
⇒ d2 = 2 ×
0.20 0.40
= = 1.9 mm 2λD 1∴ φ= ∆x = × =
Width of central maxima is . λ λ 6 3
0.21 0.21 a I = I1 + I2 + 2 I1 I2 cos φ
18 The position of 30th bright fringe, 25 Distance between first order dark fringes π
30 λ D I ′ = I + I + 2I cos = 3I
Y30 = = Width of principal maxima 3
d 2λD 2 × 600 × 10−9 × 2 and I 0 = I + I + 2I cos 0° = 4I
Position shift of central fringe is x= = I′ 3
30 λ D D
d 10−3 ∴ =
Y0 = ⇒ Y0 = ( µ − 1 )t I0 4
d d = 2400 × 10−6
30 λ D D = 2.4 × 10−3 m = 2.4 mm 2 At S3 , ∆ x = S1S3 − S2S3 = 0
⇒ = ( µ − 1 )t
d d 2π
26 For first minima at P, asin θ = nλ ∴ φ= ∆x = 0
30 λ λ
⇒ ( µ − 1) =
t I3 = I 0 + I 0 + 2 I 0 × I 0 cos 0 °
30 × (6000 × 10−10 ) P
(µ − 1) = = 0.5 I3 = 4I 0
3.6 × 10−5 A The path difference at S 4 is
µ = 1. 5 θ ∆x ′ = S 1S 4 − S 2S 4 =
dY

19 Displacement of central fringe a θ D


d λD λ  λD
β β = × = Q Y =
= ( µ 1 − 1) t   − ( µ 2 − 1) t   = ∆x
5
D 2d 2  2d 
 λ  λ β
2π λ
β φ′ = = π
∴ ( µ 1 − µ 2 ) t   = 5β λ 2
 λ
where, n = 1 ⇒ asin θ = λ I 4 = I 0 + I 0 + 2I 0 cos π = 0
 1  So, phase difference, I3 4I
(0.3) t   =5 = 0 =∞
−10
 4800 × 10  ∆x1 (a / 2)sin θ I4 0
∆φ1 = × 2π = × 2π
λ λ
4.8 × 10−7 × 5 3 Path difference = 2λ cos θ
t = λ
0.3 = × 2 π = π rad
2λ P
⇒ t = 8 × 10−6 m = 8 × 10−3 mm
27 Key Idea First minima is formed at a
20 According to question, 5th dark fringe in distance x

air = 8 bright fringe in the medium λD (5 × 10−5 ) (0.6) 2λ co
Y = ⇒Y = θ
λD λD (given)
(2 × 5 − 1) =8 a 0.02 × 10−2 O
2d µd 2λ
⇒ Y = 0.15 cm
λD λD 9 8 8×2
⇒9 =8 ⇒ = ⇒ µ = D a
2d µd 2 µ 9 28 The figure shown below represents the
course of path an unpolarised light 2λ cos θ = nλ
∴ Refractive index of the medium,
follows when it is incident from air on For x to be minimum, n = 1
16
µ = = 1.7777 ≈ 1.78 plane surface of material of refractive cos θ =
1
9 index µ. 2
21 If either source or screen, both are at (Normal) θ = 60°, x = D tan 60° = 3 D
finite distance from the diffracting Incident Plane polarised
device, the diffraction is called Fresnel unpolarised light reflected light
4 Path difference = AB + BO − 2D
diffraction.
λ
⇒ 2 (D 2 + d 2 ) − 2D =
22 In Fraunhofer diffraction, the centre of 2
diffraction image is always bright. λ
Medium ⇒ 2 D2 + d 2 = + 2D
(µ) 2
23 For Fraunhofer diffraction of a single slit,
λ2
we have all the conditions true, i.e. 4( D 2 + d 2 ) = + 4 D 2 + 2λD
central maxima has width ∝ λ, on 4
Partially polarised
increasing the slit width, the width of λ2
refracted light Eliminating as
central maxima decreases. Also, if a ≈ λ, 4
then central maxima has angular When the beam of unpolarised light is λD
separation of ± 90°. λ < < D, d =
reflected from a medium (refractive 2
324 40 DAYS ~ NEET PHYSICS DAY TWENTY NINE

5 Given, d = 1 mm = 1 × 10−3 m, From the above diagram, intensity According to question, the intensity at
transmitted through P3 , maximum in this Young’s double slit
D = 1 m, λ = 50 × 10−9 m I
I2 = 0 cos 2 45° experiment is I 0 .
2 ⇒ I max = I 0
2
⇒ I2 = 0 × 
I 1  I0 Q Path difference
 ⇒ I2 =
2λ 2  2 4 d ×
d
a
a Similarly, intensity transmitted through =
dYn
= 2 = d = λ [Q d = 5λ]
P2 , D 10d 20 4
s 2 A path difference of λ corresponds to
I I  1 
I3 = 0 cos 2 45° ⇒ I3 = 0 ×   phase difference 2π.
4 4  2
Width of central maxima So, for path difference λ / 4, phase
I0 1 I0
= Width of ten maxima ⇒ I3 = × ⇒ I3 = difference,
2λ λD  4 2 8 2π λ
= 10   φ= × = π / 2 = 90°
a  d  8 It is given that, λ 4
2d 2 × 10−3 I2 φ
⇒ a= = = n ⇒ I2 = nI1 As we know, I = I 0 cos 2
10D 10 × 1 I1 2
90°
⇒ a = 2 × 10−4 m ∴ Ratio of intensities is given by ⇒ I = I 0 cos 2
I max − I min 2
a = 0.2 mm 2
I max + I min  1  I0
2 ⇒ I = I0 ×   ⇒I =
 a + a2   2 2
6 Q I max = 25 or  1 25 ( I2 + I1 ) − ( I2 −
2 2
 = =
I1 )
I min 9  a1 − a2  9
( I1 + I2 )2 + ( I2 − I1 )2 ) 10 As the first minimum is observed at an
where, a denotes amplitude. 2 2 angle of 30° in a diffraction pattern due to a
a + a2 5 a  I2   I 
or 1 = or 1 = 4  + 1 −  2 − 1 single slit of width a.
a1 − a2 3 a2  I1   I1 
= i.e. n = 1, θ = 30°
As, (amplitude)2 ∝ intensity  I2 
2
 I 
2
Q According to Bragg’s law of
 + 1 +  2 − 1
I1  a1 
2
 I1   I1  diffraction,
Hence, =   = 16 a sin θ = nλ ⇒ a sin 30° = (1) λ
I2  a2  ( n + 1)2 − ( n − 1)2
= [Q n = 1]
( n + 1)2 + ( n − 1)2
a = 2λ …(i) Q sin 30° = 

7 According to the question, 1

2 n  2 
=
P1 P3 P2 n+1 For first secondary maxima,
3λ 3λ
9 a sin θ1 = ⇒ sin θ1 = …(ii)
2 2a
I0 I1 I2 I3 Screen
Substituting the value of a from
Eq. (i) into Eq. (ii), we get
S1 2.5λ 3λ 3
sin θ1 = ⇒ sin θ1 =
90° 5λ 4λ 4
⇒ θ1 = sin −1  
S2 O 3
45° 50λ  4
DAY THIRTY

Unit Test 6
(Optics)
1 Sun is visible a little before the actual sunrise and until a 7 A beam of monochromatic light is refracted from vacuum
little after the actual sunset. This is due to into a medium of refractive index 1.5. The wavelength of
(a) total internal reflection (b) reflection refracted light will be
(c) refraction (d) polarisation (a) dependent on intensity of refracted light
(b) same
2 The magnification of a compound microscope is 30 and
(c) smaller
the focal length of its eye piece is 5cm. The (d) larger
magnification produced by the objective, when the
8 There is a prism with refractive index equal to 2 and
image is to be formed at least distance of distinct vision
the refracting angle equal to 30°. One of the refracting
(25 cm), is
surface of the prism is polished. A beam of
(a) 5 (b) 6 (c) 8 (d) 10
monochromatic light will retrace its path, its angle of
3 A simple microscope consists of a concave lens of incidence over the refracting surface of the prism is
power −10 D and a convex lens of power +20 D in (a) 0° (b) 30° (c) 45° (d) 60°
contact. If the image is formed at infinity, then the
9 What is Brewster angle for air to glass transition? ( µ of
magnifying power (D = 25 cm ) is
glass is 1.5)
(a) 2.5 (b) 3.5 (c) 2.0 (d) 3.0
(a) 56.3° (b) 57.5° (c) 59.1° (d) 83°
4 Interference is possible in
10 In a double slit experiment 5th dark fringe is formed
(a) light waves only
opposite to one of the slits. The wavelength of light is
(b) sound waves only
(c) both light and sound waves d2 d2 d2 d2
(a) (b) (c) (d)
(d) neither light nor sound waves 6D 5D 15 D 9D
5 The axes of the polariser and analyser are inclined to 11 Angle of deviation (δ ) by a prism (refractive index = µ,
each other at 45°. If the amplitude of the unpolarised and supposing the angle of prism A to be small) can be
light incident on the polariser is a ,then what is the given by
amplitude of the light transmitted through the analyser? (a) δ = (µ − 1) A (b) δ = (µ + 1) A
a a 3 3 A+δ
(a) (b) (c) a (d) a sin
2 2 2 4 2 µ −1
(c) µ = (d) δ = A
6 A ray is incident at an angle of incidence i on one sin
A µ+1
surface of a prism of small angle A and emerges 2
normally from opposite surface. If the refractive index of 12 The illuminance of a surface 2 m away from a point
the material of prism is µ, the angle of incidence i is source is 4 Wm −2 . It will be 2 Wm −2 when the distance of
nearly equal to the point from the source is
A A µA (a) 2 2 m (b) 2 / 2 m
(a) (b) (c) µ A (d)
µ 2µ 2 (c) 2 m (d) 1/ 2 m
326 40 DAYS ~ NEET PHYSICS DAY THIRTY

13 A wavefront is represented by the plane y = 3 − x . The 23 A transparent cube contains small air bubble. Its
propagation wave takes place at apparent distance is 2cm when seen through one face
(a) 45° with the + ve x -direction and 5 cm when seen through other face. If the refractive
(b) 135° with the + ve x-direction index of the material of the cube is 1.5, the real length of
(c) 60° with the + ve x-direction
the edge of cube must be
14
(d) no sufficient data (a) 7 cm (b) 7.5 cm (c) 10.5 cm (d) cm
3
14 In Young’s double slit experiment with monochromatic
24 Transmission of light in optical fibre is due to
light of wavelength 600 nm, the distance between slits is
(a) scattering (b) diffraction
10−3 m . For changing fringe width by 3 × 10−5 m
(c) polarisation
(a)the screen is moved away from the slits by 5 cm (d) multiple total internal reflections
(b)the screen is moved by 5 cm towards the slits
(c)the screen is moved by 3 cm towards the slits 25 Diameter of human eye lens is 2 mm. What will be the
(d)Both (a) and (b) be correct minimum distance between two points to resolve them,
which are situated at a distance of 50 m from eye?
15 A lens is placed between a source of light and wall. It
(The wavelength of light is 500 Å)
forms images of area A1 and A 2 on the wall, for its two
different positions, the area of the source of light is (a) 2.32 m (b) 4.28 mm (c) 1.25 cm (d) 12.48 cm
A1 + A 2 A1 − A 2 1 1 26 Two nicol prisms are placed with their principal planes at
(a) A 1A 2 (b) (c) (d) +
2 2 A1 A 2 60°. What percentage of light incident on the
combination is transmitted?
16 A convex lens of focal length 80 cm and a concave lens
of focal length 50 cm are combined together. What will (a) 75% (b) 60% (c) 25% (d) 12.5%
be their resulting power? 27 In Young’s double slit experiment, if the slit widths are in
(a) + 6.5 D (b) − 6.5 D (c) + 7.5 D (d) − 0.75 D the ratio 1 : 9, the ratio of the intensity at minima to that at
−2 maxima will be
17 Unpolarised light of intensity 32 Wm is incident on the
combination of three polaroids. The first and last (a) 1 (b) 1/9 (c) 1/4 (d) 1/3
polaroids are crossed. If the intensity of the light 28 Two coherent light sources S1 and S 2 (λ = 6000 Å) are
transmitted through the combination be 3 Wm −2 , then 1 mm apart from each other. The screen is placed at a
what is the angle between the transmission axes of first distance of 25 cm from the sources. The width of the
two polaroids? fringes on the screen should be
(a) 0° (b) 30° (c) 45° (d) 60°
(a) 0.015 cm (b) 0.025 cm (c) 0.010 cm (d) 0.030 cm
18 Two lamps of powers P1 and P2 are placed on either side 29 Light from two coherent sources of same amplitude ( A )
of a paper having an oil spot. The lamps are at 1m and and wavelength ( λ ) illuminates the screen. The intensity
2m respectively, on either side of the paper and the oil of central maximum is I 0. If the source were incoherent,
spot is invisible. What is the value of P1/P2 ? the intensity at the same point will be
(a) 0.25 (b) 0.40 (c) 0.50 (d) 0.60
(a) I 0 / 2 (b) I 0 (c) 2I 0 (d) 4I 0
19 An astronomical telescope of ten-fold angular
30 What will be the angle of diffraction for the first minimum
magnification has a length of 44 cm. The focal length of
the objective is due to Fraunhoffer diffraction with sources of light of
wavelength 550 nm and slit of width 0.55 mm?
(a) 440 cm (b) 44 cm (c) 40 cm (d) 4 cm
(a) 1 rad (b) 0.1 rad (c) 0.01 rad (d) 0.001 rad
20 The wavefront due to a source situated at infinity is
(a) spherical (b) cylindrical 31 Young’s double slit experiment is performed with light of
(c) planar (d) None of these wavelength 550 nm. The separation between the slits is
21 The refractive index of the material of an equilateral 1.10 mm and the screen is placed at a distance of 1m.
prism is 3, then the angle of minimum deviation of the What is the distance between the consecutive bright and
prism is dark fringes ?
(a) 30° (b) 45° (c) 60° (d) 75° (a) 0.5 mm (b) 1.0 mm
22 A parallel beam of light of wavelength 500 nm falls on a (c) 1.5 mm (d) None of these
narrow slit and the resulting diffraction pattern is 32 To produce an achromatic combination, a convex lens of
observed on a screen 1m away. It is observed that the focal length 42 cm and dispersive power 0.14 is placed
first minimum is at a distance of 2.5 mm from the centre in contact with a concave lens of dispersive power 0.21.
of the screen . Find the width of the slit. The focal length of the concave lens is
(a) 0.2 mm (b) 1.0 mm (c) 4.1 mm (d) 0.6 mm
(a) 21 cm (b) 42 cm (c) 63 cm (d) 84 cm
DAY THIRTY UNIT TEST 6 (OPTICS) 327

33 The focal length of a lens for red and violet colours are  A
42 If the refractive index of a glass prism is cot   and A is
16.4 cm and 16 cm. What is its dispersive power?  2
angle of prism, then angle of minimum deviation is
(a) 0.050 (b) 0.025 (c) 0.012 (d) 0.006
π π − A
(a)  − A (b)  2 π −  (c) 
A
34 A concave mirror of focal length 20cm produces an  (d) (π − 2A)
2   2  2 
image twice the size of the object. If the image is real,
then the distance of the object from the mirror is 43 If a convex lens of focal length 80 cm and a concave
lens of focal length 50 cm are combined together, what
(a) 10 cm (b) 20 cm (c) –30 cm (d) 60 cm
will be their resultant power?
35 The luminous efficiency of the lamp is 5 lm W −1. If the (a) + 0.65 D (b) − 0.65 D (c) + 075
. D (d) − 075
. D
luminous intensity of the lamp be 35 Cd, then power of
the lamp is 44 A convex lens forms a full image of the object on a screen.
(a) 28 W (b) 88 W (c) 112 W (d) 140 W If half of the lens is covered with an opaque object, then
(a) the image disappears (b) half of the image is seen
36 A ray of light passes through an equilateral prism, such
(c) full image is seen and of same intensity
that an angle of incidence is equal to the angle of (d) full image is seen and of decreased intensity
emergence and the latter is equal to 3/4 th the angle of
prism. The angle of deviation is 45 What is the focal length of a convex lens of focal length
30 cm in contact with a concave lens of focal length
(a) 45° (b) 39° (c) 20° (d) 30°
20 cm. Is the system a converging or a diverging lens?
37 One face AC of the glass prism is A (Ignore thickness of the lenses.)
silvered as shown and the principle
α (a) 60 cm, diverging (b) 40 cm, converging
section of the glass prism is an (c) 50 cm, converging (d) 40 cm, diverging
isosceles triangle ABC with Direction (Q. Nos. 46-50) In each of the following
AB = AC. The ∠BAC, if the ray β β questions a statement of Assertion is given followed by a
incident normally on the face AB and B C corresponding statement of Reason just below it. Of the
after two reflections, it emerges from the base BC, statements mark the correct answer as :
perpendicular to it, is (a) If both Assertion and Reason are true and the Reason is
the correct explanation of the Assertion
(a) 70° (b) 36° (c) 72° (d) 44°
(b) If both Assertion and Reason are true but the Reason is
38 A man having height 6 m, observes image of 2 m height not correct explanation of the Assertion
erect, then mirror used is (c) If Assertion is true but Reason is false
(a) concave (b) convex (d) If both Assertion and Reason are false
(c) plane (d) None of these 46 Assertion (A) The resolving power of a telescope is
39 A vessel consists of two more, if the diameter of the objective lens is more.
plane mirrors at right angle
Reason (R) Objective lens of large diameter collects
(as shown in the figure). The more light.
vessel is filled with water.
The total deviation in incident 47 Assertion (A) The focal length of the objective of the
telescope is larger than that of eyepiece.
ray is 90°
Reason (R) The resolving power of telescope increases
(a) 0° (b) 90° when the aperture of objective is small.
(c) 180° (d) None of these
48 Assertion (A) Convergent lens property of converging
40 An object is placed at a distance equal to focal length of remain same in all mediums.
convex mirror. If the focal length of the mirror be f, then
Reason (R) Property of lens whether the ray is diverging
the distance of the image from the pole of the mirror is or converging is independent of the surrounding
(a) less than f (b) equal to f (c) more than f (d) infinity medium.
41 If a ray light is incident on an P 49 Assertion (A) If Brewster’s angle be θ, then critical angle
equilateral glass prism PQR is sin−1(cot θ ).
placed on a horizontal table, 1
C Reason (R) The refractive index of medium µ = ,
then for the minimum deviation B D sin C
C being critical angle.
which of the following is true? A 50 Assertion (A) In Young’s double slit experiment
(a) AB is horizontal interference pattern disappear when one of the slits is
(b) CD is horizontal Q R closed.
(c) Either AB or CD is horizontal
(d) BC is horizontal Reason (R) Interference occurs due to superimposition of
light waves from two coherent sources.
328 40 DAYS ~ NEET PHYSICS DAY THIRTY

ANSWERS
1 (c) 2 (a) 3 (a) 4 (c) 5 (a) 6 (c) 7 (c) 8 (c) 9 (a) 10 (d)
11 (a) 12 (a) 13 (a) 14 (d) 15 (a) 16 (d) 17 (b) 18 (a) 19 (c) 20 (c)
21 (c) 22 (a) 23 (c) 24 (d) 25 (c) 26 (d) 27 (c) 28 (a) 29 (a) 30 (d)
31 (a) 32 (c) 33 (b) 34 (c) 35 (b) 36 (d) 37 (b) 38 (b) 39 (c) 40 (a)
41 (d) 42 (d) 43 (d) 44 (d) 45 (a) 46 (a) 47 (c) 48 (d) 49 (b) 50 (a)

Hints and Explanations


1 Sun is visible a little before the actual 7 Refractive index, µ = c = vλ v δ = (i1 + i2 ) − A
sunrise and until a little after the actual v vλ m
12 L1 r12 = L2 r22
sunset. This is due to refraction. λ
∴ λ m = v ⇒ λ m < λ v [Qµ = 1, given] L1 r12 4 × (2) 2
2 When final image is formed at least µ r22 = = =8
L2 2
distance of distinct vision, then the Hence, the wavelength decreases in
magnifying power second medium. So, option (c) is correct. or r2 = 8m = 2 2m
 D
= Mo ×  1 +  8 The ray QR will retrace its path,when 13 The propagation of ray takes place
 fe  perpendicular direction of wavefront.
∠ ARQ = 90° , ∠r = 30°
Here, m1 m2 = − 1
30 = Mo  1 +
25 ∴ ∠ AQR = 90° − 30° = 60°

 5 As, from Snell’s law, Here, m1 = slope of wavefront = − 1
Mo = 5 sin i = µ sin r = 2 sin 30° Q y = mx + c
1 1 m2 = slope of ray
3 Power of combination, P = P1 + P2 = 2× = ∴ m1 m2 = − 1
2 2
P = + 20 − 10 = + 10 D or (−1) m2 = − 1
A ∴ m2 = 1 or tanθ = 1 ⇒ θ = 45°
1 1 100
F = = m= cm = 10 cm D
P 10 10 14 β= λ
N' d
For final image at infinity
∆D
D 25 90° ∆β = λ
M = = = 2.5 Q 60° R d
F 10 i
30°=r d∆β 10−3 × 3 × 10−5
4 Interference phenomenon is shown by ∆D = =
P λ 600 × 10−9
both light and sound waves. N
= 5 cm away or towards the slits
5 Amplitude of the light emerging from B C
polariser = a/ 2 and that through 15 In displacement method, total
∴ ∠i = 45° magnification
analyser = (a/ 2 ) cos 45° = a / 2.
9 i p = ?, µ = 1.5
m= m1 m2 . Therefore, area of source
6 Angle of prism is given by tan i p = µ = 1.5
A = r1 + r2 = A1 A2 .
⇒ i p = tan −1 (1.5) = 563
. °
where, r1 is refraction angle on incident 16 Power of a lens is P = 1
face and r2 is angle of incidence on 10 For dark fringe, f
second face of prism. As refracted ray xd λ When f is in cm
= (2m − 1)
emerges normally from opposite D 2 100
P = D
surface, r2 = 0. f
d
∴ A = r1 Here, m = 5, x = Here, f1 = 80 cm and f2 = − 50 cm
2
100 100
µ =
sin i1 d d λ d2 ∴ P = − = − 0.75 D
Now, ∴ = (2 × 5 − 1) or = 9λ 80 50
sin r1 2D 2 D
2 I
If ∠i1 and ∠r1 are very small, then ∴ λ=
d 17 I = i cos 2 θ sin2 θ
9D 2
sin i1 ≈ i1 , sin r1 ≈ r1 2I 2 × 3 sin2 2θ
11 When refracting angle of a prism is Here, = =
i1 i Ii 32 4
∴ µ = = small (∠ 10° ), the deviation δ is
r1 A 3
calculated from the relation Hence, sin2θ =
∴ i = µA δ = (µ − 1) A. For prism with bigger 2
refracting angles, we use the relation i.e. θ = 30°
DAY THIRTY UNIT TEST 6 (OPTICS) 329

P1 P2 24 It consists of very long and thin fibre of 30 Here, d sinθ = nλ . For n = 1


18 Here, =
r12 r22 quartz glass. λ
2
We find, sinθ =
P1  r1  Quartz (µ = 1.7) d
Hence, =   = (1 / 2) 2 = 0.25 550 × 10−9
P2  r2  A = = 10−3 rad = 0.001 rad
0.55 × 10−3
19 For an astronomical telescope. −9
Magnification, m =
fo B 31 ∴ β = λD = 550 × 10 −3× 1
fe Layer (µ = 1.5) d 1.1 × 10
Length of telescope tube L = fo + fe = 500 × 10−6 m ≅ 0.5 mm
Given, m = 10, L = 44 cm When a light ray is incident at one end
f2 ω2
fo A of fibre making a small angle of 32 Here, =
∴ = 10 and fo + fe = 44 incidence. It suffers multiple total f1 ω1
fe
internal reflections and finally, it 0.21
f i.e. f2 = × 42 = 63 cm
⇒ fe = o reaches the point B. 0.14
10
f 25 Angular limit of resolution of eye 33 Dispersive power,
So, fo + o = 44 Wavelength of light
10 = df 16.4 − 16 0.4
ω= = = = 0.025
11 fo Diameter of eye lens f 16 16
or = 44
10 λ
i.e. θ= …(i) 34 Given, f = – 20 cm, m = 2
or fo = 40 cm d
–v
20 The wavefront due to a source situated If y is the minimum resolution between As, m =
two objects at distance D from eye, then u
at infinity is planar.
v
θ=
y
…(ii) ∴ 2= or v = 2u
21 The refractive index of material of u
prism is D
1 1 1
A + δm  From Eqs. (i) and (ii), we have As, + =
sin 
 y λ λD u v f
2  = or y = …(iii)
µ = 1 1 1
A D d d ∴ + = ⇒ u = – 30 cm
sin Given, u 2u –20
2
where, A is the angle of prism and δ m λ = 5000 Å = 5 × 10−7 m, D = 50 m ,
35 φ = 4 π l = 4 π × 35 = 140 π lumen.
the angle of minimum deviation. d = 2 mm = 2 × 10−3 m φ
Also, η =
Given, µ = 3, A = 60° (for prism) On substituting in Eq. (iii), we get P
 60° + δ m  5 × 10−7 × 50 i.e. P =
φ 140 π
= ≈ 88 W
sin   y =
 2  2 × 10−3 η 5
Thus, 3=
sin 30° = 12.5 × 10−3 m = 1.25 cm 36 The following ray diagram shows the
 60° + δ m  1 angle of minimum deviation.
or sin   = × 3 26 Intensity of polarised light from first
 2  2
I
° + δ polariser = 0
sin  m
60 A
or  = sin 60° 2
 2  Intensity of polarised light from second
60° + δ m I I δm
or = 60° polariser = 0 cos 2 60° = 0
2 2 8 i i′
r r′
or δ m = 2 × 60° − 60° = 60° ∴Percentage of transmitted light of
(I / 8)
22 Here, λ = 500 nm = 500 × 10−9 m incident light = 0 × 100 = 12.5%
I0 B C
= 5 × 10−7 m, D = 1 m, n = 1, x = 2.5 mm
= 2.5 × 10−3 m, a = ? 27 Amplitude of the superimposing waves δm = i + i ′ − A
are a1 / a2 = (1 / 9)1 /2 = 1 / 3 3
x Given, i = A and i = i ′
The condition for minima is a = nλ 2 4
D  a1  3 3
 − 1 ∴ δm = A + A − A
n λ D 1 × 5 × 10−7 × 1 I minima (a − a2 ) 2  a2 
1
a= = = 1 = = 4 4
x 2.5 × 10−3 Imaxima (a1 + a2 ) 2 2 4 6A A
 a1  δm = − A ⇒ δm =
 + 1
= 2 × 10−4 m = 0.2 mm  2
a 
4 2
For equilateral prism A = 60°
Real depth 60°
23 Refractive index ( µ ) = 28 β = D λ. Here, D = 25 cm, d = 1 mm, ∴ δm = = 30°
Apparent depth d 2
Refractive index ( µ ) = 1.5 λ = 6000 Å. This gives β = 0.015 cm . 37 From figure,
Apparent depth = 2 + 5 = 7 cm α = 90° – A
29 For coherent sources amplitudes are
Real depth
1.5 = ⇒ A = 90° – α …(i)
So, added. Hence, I 0 ∝ ( A + A )2 = 4 A2 . For
7 and i = 90° – α …(ii)
incoherent sources intensity is added.
∴ Real depth = 1.5 × 7 = 10.5 cm From Eqs. (i) and (ii), We get
i.e. I ∝ ( A2 + A2 ) = 2 A2 . Hence, I 0 = 2I .
i = A …(iii)
330 40 DAYS ~ NEET PHYSICS DAY THIRTY

A
42 The refractive index ( µ ) of a prism of 45 Here, f1 = 30 cm, f2 = − 20 cm, f = ?
angle A, and minimum deviation δ m is 1 1 1
As, = +
given by f f1 f2
α A + δm 
i sin   ∴
1
=
1

1
=
2−3
=−
1
 2 
i µ = f 30 20 60 60
sin A / 2
β f = − 60 cm
r ∴ The combination of lenses behaves as
A a concave lens. The system is diverging.
y y δm
90º
B C 46 Resolving power telescope is = a
1.22 λ
where, a is the diameter of objective
Again, from figure,
lens and λ is the wavelength of light
β = 90° – 2i C
B used. It is obvious that on increasing a,
β = 90° – 2 A [Q from Eq. (iii)] more light is collected by objective lens
or 2 A = 90° – β A and so, the image formed is more bright.
2A = γ …(iv) Given, µ = cot
2 Thus, resolving power telescope
Again, from figure, ( A + δm ) increases.
γ = 90° – β = γ = 2 A sin
A 2
∴ cot = 47 The magnifying power telescope in
∴ γ = 2A …(v) 2 sin( A / 2) fo
∴In ∆ABC, relaxed state is m =
A ( A + δm ) fe
A + B + C = 180° cos sin
⇒ 2 = 2
A + γ + γ = 180° sin( A / 2) sin( A / 2)
So, for high magnification, the focal
A + 2 A + 2 A = 180° ⇒ A = 36° length of objective length should be larger
A + δm 
= sin 
A
⇒ cos  than that of eyepiece.
38 The magnification of image is less than 2  2  d
1, so mirror used is convex. Resolving power of a telescope =
 + δ m 1.22 λ
∴ sin  90° −  = sin 
A A

39 The deviation produced by combination  2  2  For high resolving power, diameter (d )
of two plane mirrors is A A + δm of objective should be higher.
π ⇒ 90° − =
δ = 2 π − 2θ = 2 π − 2 × [θ = 90° ] 2 2 48 In air or water, a convex lens made of
2 ⇒ 180° − A = A + δ m glass behaves as a convergent lens but
⇒ δ = π = 180° ⇒ δ m = 180° − 2 A = π − 2 A when it is placed is carbon disulphide,
Hence, total deviation in incidnet ray is it behaves as a divergent lens.
180° 43 Combined focal length
1 1 1 Therefore, when a convergent lens is
= + placed inside a transparent medium of
f f1 f2
refractive index greater than that of
Given, f1 = 80 cm, f2 = − 50 cm material of lens, it behaves as a
(concave) divergent lens.
1 1 1
∴ = − It simply concludes that property of a
f 80 50
lens whether the ray is diverging or
90° 30
=− converging depends on the surrounding
4000 medium.
40 Image formed by convex mirror as 1 – 30 × 100
∴ P = = 49 According to Brewster’s law
shown in figure. f 4000
µ = tan θ …(i)
−3
= = − 075
. D where, µ is the refractive index of
4
medium and tanθ is the tangent of the
A 44 The various rays from the object, after angle of polarisation
refraction through different parts of the 1
µ = tan θ = …(ii)
A′ lens, will still converges at the point, cot θ
only their number will be less. When 1
Also, we know that, µ = …(iii)
B F C
half the lens is covered with an opaque sinC
B′
f f object. Hence, still the full image of the From Eqs. (ii) and (iii), we get
object is obtained but its intensity will 1 1
be less. = and sin C = cot θ
sin C cot θ
−1
Object (Half lens covered) C = sin (cot θ)
41 For minimum deviation the refracted
ray passes parallel to the base of the 50 Both Assertion and Reason are true and
Image Reason is correct explanation.
prism. Hence, BC is horizontal.
DAY THIRTY ONE

Matter Waves
Learning & Revision for the Day
u de-Broglie Waves u X-Rays
u Davisson-Germer Experiment u Moseley’s Law

de-Broglie Waves
Light is said to have dual character, i.e. it behaves like matter (particle) and wave both.
Some properties like interference, diffraction can be explained on the basis of wave
nature of light, while the phenomena like photoelectric effect, black body radiation, etc.
can be explained on the basis of particle nature of light.
In 1942, Louis de-Broglie explained that like light, matter also show dual behaviour,
there is a wave associated with moving particle, known as matter waves or de-Broglie
waves.

de-Broglie Relation
According to quantum theory, energy of photon
E = hν …(i)
If mass of the photon is taken as m, then as per Einstein’s equation
E = mc2 …(ii)
From Eqs. (i) and (ii), we get, hν = mc2 PREP
c
h = mc2 , MIRROR
λ Your Personal Preparation Indicator
where, λ = wavelength of photon u No. of Questions in Exercises (x)—
h
λ= u No. of Questions Attempted (y)—
mc u No. of Correct Questions (z)—
de-Broglie asserted that the above equation is completely a general function and applies (Without referring Explanations)
to photon as well as all other moving particles.
u Accuracy Level (z / y × 100)—
h h
So, λ= = u Prep Level (z / x × 100)—
mv 2 mE
where, m is mass of particle and v is its velocity. In order to expect good rank in NEET, your
Accuracy Level should be above 85 & Prep
Level should be above 75.
332 40 DAYS ~ NEET PHYSICS DAY THIRTY ONE

l
de-Broglie wavelength associated with charged particle Clearly from figure, we have θ + φ + θ = 180 °
h h h
λ= = =
p 2 mE 2 mqV
l
de-Broglie wavelength of a gas molecule θ φ
D
h θ d
λ=
3 mkT
where, T = absolute temperature
k = Boltzmann’s constant = 1.38 × 10 −23 J / K
l
Ratio of wavelength of photon and electron The X-Rays
wavelength of photon of energy E is given by
X-rays were discovered by Roentgen. X-rays are produced
hc
λp = while the wavelength of an electron of kinetic when fast moving electrons strike a metal of high atomic
E weight and high melting point.
h
energy K is given by λ c = ⋅ Therefore for same The phenomena of thermionic emission is used to produce
2 mK
energy, the ratio electrons in coolidge tube. Intensity of X-rays is directly
proportional to the square of the strength of the current in the
λp c 2 mc2 K filament which heats the cathode.
= 2 mK =
λe E E2 Quality of X-rays is measured in terms of their penetrating
power and depends upon the potential difference applied to
Davisson-Germer Experiment the X-ray tube. X-rays are diffracted by crystals in accordance
with Bragg’s law.
l
The de-Broglie hypothesis was confirmed by
Davisson-Germer experiment. It is used to study the
scattering of electron from a solid or to verify the wave Properties of X-Rays
nature of electron. l
X-rays are electromagnetic waves with wavelength range
l
A beam of electrons emitted by electron gun is made to fall 0.1 Å to 100 Å.
on nickel crystal cut along cubical axis at a particular l
X-rays are invisible.
angle. Ni crystal behaves like a three dimensional l
X-rays carry no charge, so they are not deflected by electric
diffraction grating and it diffracts the electron beam and magnetic fields.
obtained from electron gun.
l
They travel in straight line with speed 3 × 10 8 ms −1 through
l
The diffracted beam of electrons is received by the detector
which can be positioned at any angle by rotating it about vacuum.
the point of incidence. l
They obey phenomenon of interference, diffraction and
l
The energy of the incident beam of electrons can also be polarisation of light.
varied by changing the applied voltage to the electron gun. l
They ionise gases.
l
According to classical physics, the intensity of scattered l
They effect photographic plate.
beam of electrons at all scattering angle will be same but l
They can pass through flesh and blood, but not through
Davisson and Germer found that the intensity of scattered bones.
beam of electrons was not same but different at different l
They produce photoelectric effect and Compton effect.
angles of scattering.
l
They are not used in RADAR as they are not reflected by the
l
It is maximum for diffracting angle
target. X-rays can be used to detect diseases and to cure them.
50° at 54 V potential difference.
54 V
Incident beam

l
If the de-Broglie waves exist for
electrons, then these should be Types of X-Rays
diffracted as X-rays. X-rays are classified into two types on the basis of penetrating
Using the Bragg’s formula 50°
power
2d sin θ = nλ ,
we can determine the wavelength of
1. Soft X-rays
these waves, l
These types of X-rays produced when the potential
where d = distance between diffracting planes, difference across the cathode and target is less than 20000 V.
180 − φ l
They have low penetrating power.
θ=
2 l
These are used in the field of medicine.
= glancing angle for incident beam = Bragg’s angle. l
These are having large wavelengths.
DAY THIRTY ONE MATTER WAVES 333

2. Hard X-rays l
If the electron striking the target ejects an electron from
L-shell of the target atom, an electron from the M, N,…
l
These type of X-rays produced when the potential difference shells jumps to the L-shell, so that X-rays photons of lesser
across the cathode and target is more than 30000 V. energy are emitted. These photons form the L-series of
l
These rays are more penetrating than soft X-rays. spectrum.
l
They have low wavelength of the order of 1Å. l
In similar way, the formation of M-series, N-series, etc. may
l
These are used in the field of science and industry. be explained.
l
Intensity wavelength graph is given by
Typese of X-rays Spectra
X-rays spectra are of two types

1. Continuous X-Ray Spectrum Kβ L β Lα

Intensity

l
X-rays of all wavelengths but having different intensities
K-series
are emitted by the tube.
L-series
l
As incident electron loses its energy continuously, due to
collisions with atoms of the target, the loss of energy is
found as X-rays. Hence, X-rays of all wavelengths are λmin Wavelength
produced.

Moseley’s Law
30kV Moseley studied the characteristic of X-ray spectrum of a
Intensity

number of a heavy elements and concluded that the spectra


20kV of different elements are very similar and with increasing
10kV atomic number, the spectral lines merely shift towards higher
frequency. If ν denotes the emitted frequency of a substance
λmin of atomic number Z, then
Wavelength ν ∝ Z ⇒ ν = a(Z − b )2
l
These wavelength are of different intensity. where, a and b are constants, a = proportionality constant,
l
The maximum frequency or minimum wavelength limit is b = nuclear screening constant, values of (a) and (b) vary from
due to the loss of the total energy of electron during a one series to another series.
single collision. (i) For K-series, b = 1
1 (ii) For L-series, b = 7.4
mv2 = hν max = eV
2 (iii) For M-series, b = 19.2
hc Kγ
⇒ = eV Kβ Lγ Lβ
λ min
Kα Lα
hc 12400 √ν √ν
⇒ λ min = = in Å
eV V

2. Characteristics of X-Rays Spectrum O X O X


b Z b Z
l
Few of fast moving electrons having high velocity penetrate (a) (b)
the surface atoms of the target material and knock out the
tightly bound electrons even from the inner most shells of The intercept along + X -axis denotes the constant b.
the atoms. l
Moseley’s law is in accordance with Bohr’s theory of
l
If the electron striking the target eject an electron from spectral lines of atoms.
K-shell of the atom, a vacancy is created in the K-shell. An Wavelength of characteristic spectrum,
electron from one of the outer shell say L-shell jumps to
1 1 1 
K-shell, emitting an X-ray photon of energy equal to the = R(Z − b )2  2 − 2 
energy difference between the two shells. λ  n1 n2 
l
Similarly, if an electron from the M-shell jumps to the and energy of X-ray radiations,
K-shell, X-ray photon of higher energy is emitted. The 1
hc 1 
X-ray photons emission gives K α , K β , K γ lines of the ∆E = hν = = Rhc (Z − b )2  2 − 2 
K -series of the spectrum. λ  1
n n2 
334 40 DAYS ~ NEET PHYSICS DAY THIRTY ONE

DAY PRACTICE SESSION 1

FOUNDATION QUESTIONS EXERCISE


1 If α, β and γ rays carry same momentum which has the 9 If an electron and a proton have the same de-Broglie
longest wavelength? wavelength, then the kinetic energy of the electron is
(a) α-rays (b) β-rays (a) zero
(c) γ-rays (d) All have same wavelength (b) less than that of proton
(c) more than that of a proton
2 What is de-Broglie wavelength of a dust particle of mass (d) equal to that of proton
1 × 10−9 kg drifting with a speed of 2.2 m/s?
10 The de-Broglie wavelength of a proton (charge
(a) 1.1 × 10−32 m (b) 3.01 × 10−25 m
= 1.6 × 10 − C, m = 1.6 × 10−27 kg) accelerated through a
19
(c) 1.1 × 10−3 m (d) 3.01 × 10−2 m
potential difference of 1 kV is
3 What will be the de-Broglie wavelength of a particle of
(a) 600 Å (b) 0.9 × 10−12 m
rest mass m0 , if it moves with the speed of light? (c) 7 Å (d) 0.9 nm
h 2h
(a) (b) 11 The de-Broglie wavelength of 1 MeV proton is
m0c m0c
(c) ∞ (d) 0 (a) 6.63 × 10−34 m (b) 3.33 × 10−30 m
(c) 2.32 × 10−20 m (d) 2.86 × 10−4 m
4 An α-particle and a deuteron are moving with the same
kinetic energy. What will be the ratio of their de-Broglie 12 The de-Broglie wavelength associated with an electron,
wavelengths? accelerated through a potential difference of 100 V is
1 (a) 0.529 nm (b) 52.9 nm (c) 0.123 nm (d) 1.23 nm
(a) (b) 2
2 13 A particle is dropped from a height H. The de-Broglie
(c) 1 (d) 2
wavelength of the particle as a function of height is
5 Which of the following figures represent the variation of proportional to
particle momentum and the associated de-Broglie (a) H (b) H 1/ 2
wavelength? j
CBSE AIPMT 2015 (c) H 0 (d) H −1/ 2
14 Energy of neutron (in eV) whose de-Broglie wavelength
(a) p (b) p is 1 Å
(a) 1.674 × 10−27eV (b) 8.13 × 10−2 eV
λ λ (c) 6.62 × 10−22 eV (d) 3.23 × 10−2 eV
15 In the Davisson-Germer experiment, if the incident beam
(c) p (d) p consists of electrons, then the diffracted beam consists
of
λ λ (a) protons (b) neutrons (c) α-particles (d) electrons

6 If the kinetic energy of the particle is increased to 16 times 16 The X-ray tube is operated at 50 kV. The minimum
its previous value, the percentage change in the wavelength produced, is
de-Broglie wavelength of the particle is j CBSE AIPMT 2014 (a) 0.5 Å (b) 0.75 Å
(a) 25 (b) 75 (c) 60 (d) 50 (c) 0.25 Å (d) 1.0 Å

7 The following particles are moving with the same velocity, 17 What kV potential is to be applied on X-ray tube, so that
then maximum de-Broglie wavelength will be for minimum wavelength of emitted X-ray may be 1 Å
j
CBSE AIPMT 2002 (Take, h = 6.6 × 10−34 J - s )
(a) proton (b) α-particle (c) neutron (d) β-particle (a) 12.42 kV (b) 12.84 kV
(c) 11.98 kV (d) 10.78 kV
8 Nuclear radii may be measured by scattering high
18 If f1, f2 and f3 are the frequencies of corresponding
energy electrons from nuclei. What is the de-broglie
K α , Kβ and Lα , X-rays of an electron, then
wavelength for 200 MeV electrons?
(a) f1 = f2 = f3 (b) f1 − f2 = f3
(a) 8.28 fm (b) 7.98 fm
(c) 6.45 fm (d) 6.20 fm (c) f2 = f1 + f3 (d) f22 = f1f3
DAY THIRTY ONE MATTER WAVES 335

19 According to the Moseley’s law, the frequency of 20 Two elements A and B with atomic numbers Z A and Z B
characteristic X-rays is related to the atom in number of are used to produce characteristics X-rays with
target element as frequencies ν A and νB , respectively. If Z A : Z B = 1 : 2,
(a) Z (b) Z 2 then ν A : νB will be
(c) Z −1 (d) Z −2 (a) 1 : 2 (b) 1 : 8 (c) 4 : 1 (d) 1 : 4

DAY PRACTICE SESSION 2

PROGRESSIVE QUESTIONS EXERCISE


1 A particle of mass 1 mg has the same wavelength as an The de-Broglie wavelength of the particle in the range
electron moving with a velocity of 3 × 106 ms −1. The 0 ≤ x ≤ 1 is λ 1 and that in the range x > 1 is λ 2 . The total
λ
velocity of the particle is (Take, mass of electron energy of the particle is 2E 0 . Ratio 1 is
= 9.1 × 10−31 kg) j CBSE AIPMT 2008 λ2
1 1
(a) 2.7 × 10−18 ms −1 (b) 9 × 10−2 ms −1 (a) (b) 3 (c) (d) 2
(c) 3 × 10−31 ms −1 (d) 2.7 × 10−21 ms −1 2 3

2 The de-Broglie wavelength of a bus moving with speed v 7 The de-Broglie wavelength of a neutron in thermal
is λ. Some passengers left the bus at a stoppage. Now, equilibrium with heavy water at a temperature T (kelvin)
when the bus moves with twice its initial speed, its kinetic and mass m, is j NEET 2017

energy is found to be twice its initial value. The h h 2h 2h


(a) (b) (c) (d)
de-Broglie wavelength will now mkT 3mkT 3mkT mkT
λ λ 8 Electrons of mass m with de-Broglie wavelength λ fall on
(a) λ (b) 2λ (c) (d)
2 4 the target in an X-ray tube. The cut-off wavelength (λ 0) of
3 X-rays of wavelength λ 0 = 0.200 nm are scattered from a the emitted X-ray is j
NEET 2016
block of material. The scattered X-rays are observed at 2mcλ2 2h
(a) λ 0 = (b) λ 0 =
an angle of 45° to the incident beam. Calculate their h mc
wavelength. 2m 2c 2 λ3
(c) λ 0 = (d) λ 0 = λ
(a) 0.300700 nm (b) 0.100710 nm h2
(c) 0.200710 nm (d) 0.400710 nm 9 The energy that should be added to an electron, to
4 An electron is moving with an initial velocity v = v 0 $i and reduce its de-Broglie wavelength from10−10m to
is in a magnetic field B = B0 $j . Then, it’s de-Broglie 0.5 × 10−10 m, will be
wavelength (a) four times the initial energy
(a) remains constant (b) increases with time (b) thrice the initial energy
(c) decreases with time (c) equal to the initial energy (d) twice the initial energy
(d) increases and decreases periodically 10 An electron of mass m and a photon have same energy
5 A parallel beam of fast moving electrons is incident E.The ratio of de-Broglie wavelengths associated with
normally on a narrow slit. A fluorescent screen is placed them is (c being velocity of time) j
NEET 2016
at a large distance from the slit. If the speed of the 1 1 1 1

(a) 
E 2
(c) 
1 2m  2
(d) 
1 E 2
electrons is increased, then which of the following  (b) c (2mE ) 2  
 2m  c E  c  2m 
statement(s) is/are correct? j
NEET 2013
(a) Diffraction pattern is not observed on the screen in the 11 An electron of mass m with a velocity v = v 0i$ (v 0 > 0)
case of electrons enters an electric field E = − E 0$i (E 0 = constant > 0) at
(b) The angular width of the central maximum of the
t = 0. If λ 0 is its de-Broglie wavelength initially, then its
diffraction pattern will increase
de-Broglie wavelength at time t is
(c) The angular width of the central maximum will be
 eE 0 
decrease (a) λ 0 t (b) λ 0  1 + t
(d) The angular width of the central maximum will be  mv 0 
unaffected λ0
(c) (d) λ 0
 eE 0 
6 The potential energy of a particle of mass m varies as 1 + t
 mv 0 
U(x) = E 0 for 0 ≤ x ≤ 1
0 for x > 1
336 40 DAYS ~ NEET PHYSICS DAY THIRTY ONE

ANSWERS
SESSION 1 1 (d) 2 (b) 3 (d) 4 (a) 5 (b) 6 (b) 7 (d) 8 (d) 9 (c) 10 (b)
11 (d) 12 (c) 13 (d) 14 (b) 15 (d) 16 (c) 17 (a) 18 (c) 19 (b) 20 (d)
SESSION 2 1 (a) 2 (a) 3 (c) 4 (a) 5 (b) 6 (d) 7 (b) 8 (a) 9 (b) 10 (d)
11 (c)

Hints and Explanations


SESSION 1 electron is minimum. Thus, de-Broglie = 13.01 × 10−21 J
wavelength is maximum for β-particle.
1 de-Broglie suggested that the dual ∴ E = 8.13 × 10−2 eV
nature is not only of light, but each 8 The de-Broglie wavelength is given by
moving material particle has the dual hc 1240 15 In Davisson-Germer experiment
λ= = = 6.20 fm working on the scattering basis, if the
h 200 × 106
nature. The wavelength of wave, λ = E
p incident beam consists of electrons,
1
where, p is momentum 9 Given, λ = h = h
⇒ KE = then the diffracted beam consists of
mv 2mKE 2mλ2 electrons.
It is given that, α, β and γ-rays carry
As, λ is same for both electron and 6.6 × 10−34 × 3 × 108
same momentum, so they will have 16 ∴ λ = hc = −19
1 eV 1.6 × 10 3
× 50 × 10
same wavelength. Hence, option (d) is proton, KE ∝ .
m
true. 19.8 × 10−26
Hence, kinetic energy will be maximum ⇒λ = = 0.25Å
2 Given, m = 1 × 10−9 kg, v = 2.2 ms −1 80 × 10−16
for particle with lesser mass, i.e.
∴ p = mv = 1 × 10−9 × 2.2 electron. 17 ∴λ min = 12375 Å = 12375 Å
= 2.2 × 10−9 kgms −1 V 1
10 λ = h = h = h
h 6.62 × 10 −34 p 2mE 2mqV = 12375
. kV = 12.42 kV
∴ λ= =
p 2.2 × 10−9 6.6 × 10−34 18 For K α , L-shell to K -shell, K β ,
⇒ λ=
−25
⇒ λ = 3.01 × 10 m 2 × 1.6 × 10−27 × 1.6 For K β , M-shell to K -shell
3 The mass of particle will be infinite, × 10−19 × 1000 L α , M-shell to L-shell
h
hence λ = = 0. 6.6 × 10−34 −12 E M − E K = (E M − E L ) + (E L − E K )
mc ⇒ λ= = 0.9 × 10 m
. × 10−22
716 ⇒ hf2 = hf3 + hf1
4 λ= h . ⇒ f2 = f1 + f3
0.286 0.286
2mK 11 Q λ = = = 2.86 × 10−4 m 19 According to the Moseley’s law, the
λ md 1 V 106
Hence, α = = frequency of characteristic X-rays is
λd mα 2 12 Accelerating potential, V = 100 V related to the atom in number of target
The de-Broglie wavelength, element as Z 2 .
5 Since, λ = h [Q pλ = h]
1.227
p λ= nm, 20 According to Moseley’s law, ν ∝ Z
Therefore, graph will be hyperbola. V 2 2
νA Z 
=  A  =   =
1 1
λ=
1.227
nm = 0.123 nm ∴
6 From de-Broglie wavelength, νB  ZB   2 4
h h 100
λ= = [Q p = 2mE ]
p 2mE 13 Velocity acquired by a particle, while SESSION 2
h λ falling from a height H is,
∴ λ′ = = = 0.25λ,
2m(16 E ) 4 v = 2g H 1 According to de-Broglie relation,
wavelength of a particle is given by
% change = 75% h h
∴ λ= = or λ ∝ H −1 /2 λ=
h
7 de-Broglie wavelength is given by mv m 2g H p
h
λ= h2 where, h is Planck’s constant and
mv 14 Q λ = h = h
⇒E = wavelength of an electron is given by
1 mv 2mE 2mλ2
For same velocity, λ ∝ −27 λe =
h
m Given, m = 1.674 × 10 kg, λ = 1 Å,
pe
h = 6.62 × 10−34 J-s
Out of the given particles, the mass of But λ = λe , so p = pe
β-particle which is a fast moving (6.62 × 10−34 )2 or me v e = mv
=
2 × 1.674 × 10−27 × (10−10 )2
DAY THIRTY ONE MATTER WAVES 337

me v e h Energy of a photon can be given as


or v = λ2 =
m 2m(2E 0 ) E = hν
Here, me = 91. × 10−31 kg, λ1 hc
∴ = 2 ⇒ E =
v e = 3 × 106 ms −1 λ2 λp
and m = 1 mg = 1 × 10−6 kg 7 de-Broglie wavelength of the neutron, ⇒ λp =
hc
…(iv)
E
9.1 × 10−31 × 3 × 106 λ=
h
=
h
∴ v = p Hence, λ p = de-Broglie’s wavelength of
1 × 10−6 2m(KE )
h h photon.
= 2.7 × 10−18 m / s = =
3 3mkT Now, divide equation (iii) by (iv), we get
1 2m × kT λe h E
mv × v 2 = .
2 KE λp
2 Momentum, p = mv = 2 =
8 Given, mass of electrons = m
2mE hc
1 v
×v λc 1 E
2 de-Broglie wavelength = λ ⇒ = .
2 λp c 2m
If kinetic energy as well as speed are p
doubled, momentum p remains So, kinetic energy of electron =
2m 11 According to the question,
h
unchanged λ = .  h
2
v = v 0 $i, E = − E 0 $i
p  
 λ h2
Hence, de-Broglie wavelength will be = = Thus, magnitude of force on the electron
unchanged.
2m 2mλ2 due to the electric field,| F | = q| E |
Now, maximum energy of photon can be ⇒ F = eE 0
3 The shift in wavelength of the scattered given by
X-rays is given by hc h2 From Newton’s second law of motion,
h E = = F = ma
∆λ = λ ′ − λ = (1 − cos φ) λ 0 2mλ2
m 0c ∴ F = ma = eE 0
hc × 2λ2 . m eE 0
Substituting the values, we have ⇒ λ0 = ⇒ a= …(i)
6.626 × 10−34 h2 m
λ′ − λ = 2mcλ2 (− e ) (− E 0 i ) eE 0 $
$
(9.11 × 10−31 )(3.00 × 108 ) = or a = = i
h m m
(1 − cos 45° )
−13 9 The wavelength of electron of energy E From first equation of motion,
= 7.10 × 10 m = 0.000710 nm
is λ=
h v = u + at
∴λ′ = (0.200) + (0.000710) 2mE Here, u (initial velocity) = v 0
= 0.200710 nm 1 eE 0
⇒ λ∝ ⇒ v = v0 + t …(ii)
E m
4 Here, v = v 0 $i, B = B 0 $j. λ1 E2 (from Eq. (i))
⇒ =
Force on moving electron due to λ2 E1 Initial de-Broglie wavelength of the
magnetic field is
F = − e (v × B) = − e [ v 0 i$ × B 0 $j ] 10−10 E2 electron is given as
⇒ = h
= − ev B k$ 0.5 × 10−10 E1 λ0 = ⇒ h = λmv 0 …(iii)
0 0 mv 0
E2
As this force is perpendicular to v and ⇒ =4
E1 After time t, de-Broglie wavelength is
B, so the magnitude of v will not
⇒ E2 = 4E1 given as
change, i.e. momentum (= mv ) will
h
remain constant in magnitude. Hence, Hence the required energy λ=
de-Broglie wavelength λ = h / mv = E2 − E1 = 3E1 is thrice of initial mv
remains constant. energy. Substituting the value of v from Eq. (ii),
we get
5 As the screen is placed at larger 10 (d) Since, it is given that electron has h
distance and the speed of electron λ=
mass m.  eE 0 
increases, hence the angular width of m v0 + t
de-Broglie’s wavelength for an electron  m 
the central maxima of the diffraction
will be given as h
pattern will increase. =
λe =
h  eE 0 
6 Total energy, …(i) mv 0 1 + t
P  mv 0 
2E 0 = Kinetic energy + Potential energy where, h = Planck’s constant λ mv 0
= K + U ( x) = [from Eq. (iii)]
P = Linear momentum of electron  eE 0 
In the range 0 ≤ x ≤ 1,U ( x ) = E 0 (given) mv 0 1 + t
∴ K = 2E 0 − U ( x ) = 2E 0 − E 0 = E 0 P2  mv 0 
As kinetic energy of electron, E = λ0
de-Broglie wavelength, 2m =
h h  eE 0 
λ= ⇒ λ1 = ⇒ P = 2mE …(ii) 1 + t
2mK 2mE 0  mv 0 
From equation (i) and (ii), we get λ0
In the range x > 1, U ( x ) = 0 (given) ∴ λ=
h eE 0
∴ K = 2E 0 − U ( x ) = 2E 0 λe = …(iii) 1+ t
2mE mv 0
DAY THIRTY TWO

Photoelectric
Effect
Learning & Revision for the Day
u Photon u Laws of Photoelectric Emission Effect
u Photoelectric Effect u Energy and Momentum of Photon

Photon
A particle of light called a photon has energy E that is related to the frequency f and
wavelength λ of light wave.
hc
By the Einstein equation, E = hf = …(i)
λ
where, c is the speed of light (in vacuum) and h is Planck’s constant.
h = 6.626 × 10 −34 J-s = 4136
. × 10 −15 eV-s
Since, energies are often given in electron volt (1eV = 1.6 × 10 −19 J) and wavelengths are
in Å, it is convenient to the combination hc in eV-Å. We have,
hc = 12375 eV-Å
12375
Hence, Eq. (i), in simpler form can be written as, E ( in eV) = …(ii)
λ ( in Å)
PREP
The propagation of light is governed by its wave porperties whereas the exchange of
energy between, light with matter is governed by its particle properties. The MIRROR
Your Personal Preparation Indicator
wave-particle duality is a general property of nature. For example, electrons (and other
so called particles) also propagate as waves and exchange energy as particles. u No. of Questions in Exercises (x)—
u No. of Questions Attempted (y)—
Particle Nature of Light u No. of Correct Questions (z)—
(Without referring Explanations)
Photoelectric effect gave evidence to the strange fact that light in interaction with matter
behaved as if it was made of quanta or packets of energy, each of energy hν. Einstein u Accuracy Level (z / y × 100)—
 hν  Prep Level (z / x × 100)—
stated that the light quantum can also be associated with momentum   ⋅ u

 c 
This particle like behaviour of light was further confirmed, in 1924, by the experiment of In order to expect good rank in NEET, your
Accuracy Level should be above 85 & Prep
A.H. Compton on scattering of X-rays from electrons. Level should be above 75.
DAY THIRTY TWO PHOTOELECTRIC EFFECT 339

Photoelectric Effect
Photoelectric current
l
Photoelectric effect is the phenomenon of emission of
electrons (known as photoelectrons) from the surface of
ν 3 > ν 2 > ν1
metals when light radiation of suitable frequency are
Saturation
incident on them. ν3 current
l
The minimum energy of incident radiation needed to eject ν2
ν1
the electrons from metal surface is known as work function
(φ0) of that surface.
– V03 – V02 – V01 0 Collector plate
l
The frequency or wavelength corresponding to the work Stopping potential potential
function is called threshold frequency or threshold
A photon may collide with a material particle. The total
wavelength. Work function is related to threshold
energy and the total momentum remain conserved in such a
frequency as,
collision. Photoelectric emission is an instantaneous
hc
φ 0 = hν 0 = phenomenon.
λ0
V0
where, λ 0 = threshold wavelength.
hc 12400
l
In electron volt units, φ (eV) = =
eλ 0 λ (Å) O
ν0 ν
l
For photoemission to take place energy of incident light (E )
is related as, E ≥ p0
– φ0
l
According to Einstein’s photoelectric equation,
hν = φ0 + K max
1 Variation of stopping potential V0 with frequency ν of incident
where, K max = mv2max = maximum kinetic energy of radiation is as shown in above figure.
2
h φ
ejected photoelectron. As, eV0 = h(ν − ν 0) = hν − φ0 ⇒ V0 = ν − 0
e e
h
Effect of Intensity on Thus, V0- ν graph is a straight line whose slope is and
e
Photoelectric Emission intercept is − φ0 eV. The graph meets the ν-axis at ν 0.
For a light of given frequency ν > ν 0 (or given wavelength Photocurrent ∝ ∝ λ
1
λ < λ 0), if the intensity of light incident on photosensitive metal ν
surface is increased, the number of photoelectrons and
consequently the photoelectric current I increases. However, the
stopping potential V0 remain constant.
Energy and Momentum of Photon
l
From Einstein’s mass-energy relation E = hν = mc2
Photoelectric


Kinetic mass of photon is m =
current

I 1 > I2 > I3 c2
I1 c
I2 But ν = , where λ is wavelength of the photon.
I3 λ
h c h
Saturation ∴Kinetic mass of photon, m = 2   =
Stopping
current c  λ  cλ
potential
hν h
Kinetic mass of photon, m = 2 =
–V0 O Collector plate c cλ
Stopping potential potential l
Momentum of photon,
p = kinetic mass of photon × velocity of photon
Effect of Frequency on hν
= 2 ×c =

c c
Photoelectric Emission c
Also, ν =
If keeping the intensity of incident light constant, the λ
frequency of incident light is increased, then the stopping hc h
potential V0 (and hence, K max ) increases, but the photoelectric ∴Momentum of photon, p =   =
c  λ λ
current I remains unchanged.
340 40 DAYS ~ NEET PHYSICS DAY THIRTY TWO

Laws of Photoelectric Emission Effect


Lenard and Millikan gave the following laws on the basis of experiments on photoelectric effect.
l
The rate of emission of photoelectrons from the surface of a metal varies directly as the intensity of the incident light falling
on the surface.
l
The maximum kinetic energy of the emitted photoelectrons is independent of the intensity of the incident light.
l
The maximum kinetic energy of the photoelectrons increases linearly with increase in the frequency of the incident light.
l
As soon as, the light is incident on the surface of the metal, the photoelectrons are emitted instantly, i.e. there is no time lag
between incidence of light and emission of electrons (≈ 10 −9 s).

DAY PRACTICE SESSION 1

FOUNDATION QUESTIONS EXERCISE


1 Which of the following characteristics of photoelectric 6 The figure shows a plot of photocurrent versus anode
effect supports the particle nature of radiations? potential for a photosensitive surface for three different
(a) Threshold frequency radiations. Which one of the following is a correct
(b) Dependence of the velocity of photoelectron on statement ? j CBSE AIPMT 2009

frequency Photocurrent
(c) Independence of velocity of photoelectrons of intensity
of radiations b
(d) Instantaneous photoelectric emission c a

2 A photocell employs photoelectric effect to convert


(a) change in the frequency of light into a change in electric Retarding potential Anode potential
voltage (a) Curves a and b represent incident radiations of different
frequencies and different intensities
(b) change in the intensity of illumination into a change in
(b) Curves a and b represent incident radiations of same
photoelectric current
frequency, but of different intensities
(c) change in the intensity of illumination into a change in (c) Curves b and c represent incident radiations of different
the work function of the photocathode frequencies and different intensities
(d) change in the frequency of light into a change in the (d) Curves b and c represent incident radiations of same
electric current frequency having same intensity

3 Photoelectric emission occurs only when the incident 7 Consider a beam of electrons (each electron with energy
light has more than a certain minimum j
CBSE AIPMT 2011 E 0) incident on a metal surface kept in an evacuated
(a) wavelength (b) intensity chamber. Then,
(c) frequency (d) power (a) no electrons will be emitted as only photons can emit
electrons
4 The number of photoelectrons emitted for light of a
(b) electrons can be emitted, but all with an energy E 0
frequency ν (higher than the threshold frequency ν 0 ) is (c) electrons can be emitted with any energy, with a
proportional to j
CBSE AIPMT 2009 maximum of E 0 - φ (φ is the work function)
(a) ν − ν0 (d) electrons can be emitted with any energy, with a
(b) threshold frequency (ν0 ) maximum of E 0
(c) intensity of light
8 From Einstein’s photoelectric equation, the graph of
(d) frequency of light (ν)
kinetic energy of the photoelectron emitted from the
5 What is E in the Einstein’s photoelectric equation metal versus the frequency of the incident radiation gives
E = hν − φ 0 , where ν is the frequency of incident a straight line graph, whose slope
radiations and φ 0 is the work function? (a) depends on the intensity of the incident radiation
(a) Kinetic energy of every photoelectron (b) depends on the nature of the metal and also on the
(b) Mean kinetic energy of photoelectrons intensity of incident radiation
(c) Minimum kinetic energy of photoelectrons (c) is same for all metals and independent of the
(d) Maximum kinetic energy of photoelectrons intensity of the incident radiation
(d) depends on the nature of the metal
DAY THIRTY TWO PHOTOELECTRIC EFFECT 341

9 Light of wavelength λ falls on a metal having work 18 A photoelectric surface is illuminated successively by
hc λ
function . Photoelectric effect will take place only, if monochromatic light of wavelength λ and . If the
λ0 2
(a) λ ≥ λ 0 (b) λ ≥ 2 λ 0 (c) λ ≤ λ 0 (d) λ = 4 λ 0 maximum kinetic energy of the emitted photoelectrons in
the second case is 3 times that in the first case, the work
10 The work functions for metals A , B and C are
function of the surface of the material is j CBSE AIPMT 2015
respectively 1.92 eV, 2.0 eV and 5 eV. According to
Einstein’s equation, the metal (s) which will emit (where, h = Planck’s constant, c = speed of light)
hc hc 2 hc hc
photoelectrons for a radiation of wavelength 4100 Å is/are (a) (b) (c) (d)
2λ λ λ 3λ
(a) Only A (b) A and B
(c) All of these (d) None of these 19 In an experiment, photoelectrons are emitted when light
11 Ultraviolet beam of wavelength 280 nm is incident on of wavelength 4000 Å is incident on it. They can be
lithium surface of work function 2.5eV. The maximum stopped by a retarding potential of 2V. If the wavelength
velocity of electron emitted from metal surface is of the incident light be 3000Å, the stopping potential will
(a)8.2 × 105 m/s (b) 106 m/s be
(c)7 × 105 m/s (d) 3 .8 × 106 m/s (a) 1 V (b) 1.5 V (c) 2 V (d) 3 V
12 A light of wavelength 5000 Å falls on a sensitive plate 20 In photoelectric emission process from a metal of work
with photoelectric work function 1.90 eV. Kinetic energy function 1.8 eV, the kinetic energy of most energetic
of the emitted photoelectrons will be electrons is 0.5 eV. The corresponding stopping
(Take, h = 6.62 × 10−34 Js) potential is j CBSE AIPMT 2011

(a) 0.1 eV (b) 2 eV (c) 0.58 eV (d) 1.581 eV (a) 1.3 V (b) 0.5 V (c) 2.3 V (d) 1.8 V
13 The photoelectric threshold wavelength of silver is 21 Photons with energy 5 eV are incident on a cathode C in
3250 × 10−10 m. The velocity of the electron ejected from a photoelectric cell. The maximum energy of emitted
a silver surface by ultraviolet light of wavelength photoelectrons is 2 eV. When photons of energy 6 eV are
2536 × 10−10 m is (Take, h = 4.14 × 10−15 eVs and incident on C, no photoelectrons will reach the anode A,
c = 3 × 108 ms −1 ) j
NEET 2017 if the stopping potential of A relative to C is j NEET 2016
(a) ≈ 6 × 105 ms −1 (b) ≈ 0.6 × 106 ms −1 (a) + 3 V (b) + 4 V (c) – 1 V (d) – 3 V
(c) ≈ 61 × 103 ms −1 (d) ≈ 0.3 × 106 ms −1
22 A certain metallic surface is illuminated with
14 When the energy of the incident radiation is increased by monochromatic light of wavelength λ. The stopping
20%, the kinetic energy of the photoelectrons emitted potential for photoelectric current for this light is 3V0. If
from a metal surface increased from 0.5 eV to 0.8 eV. the same surface is illuminated with light of wavelength
The work function of the metal is j
CBSE AIPMT 2014 2λ, the stopping potential is V0. The threshold
(a) 0.65 eV (b) 1.0 eV (c) 1.3 eV (d) 1.5 eV wavelength for this surface for photoelectric effect is
15 For photoelectric emission from certain metal, the cut-off j
CBSE AIPMT 2015
frequency is ν. If radiation of frequency 2ν impinges on the λ λ
(a) 6λ (b) 4λ (c) (d)
metal plate, the maximum possible velocity of the emitted 4 6
electron will be (where, m is the electron mass) j NEET 2013 23 Photons absorbed in matter are converted to heat. A
hν hν 2hν hν source emitting n photon/s of frequency ν is used to
(a) (b) (c) (d) 2
(2m) m m m convert 1kg of ice at 0°C to water at 0°C. Then, the
time T taken for the conversion
16 If a surface has a work function 4.0 eV, what is the
(a) increases with increasing n with ν fixed
maximum velocity of electrons liberated from the surface
when it is irradiated with ultraviolet radiation of (b) increases with n fixed ν increasing
wavelength 0.2 µm? (c) remains constant with n and ν changing such that,
nν = constant
(a)4.4 × 105 m/s (b) 8.8 × 107 m/s
(d) increases when the product nν increases
(c)8.8 × 105 m/s (d) 4.4 × 107 m/s
24 When a monochromatic point source of light is at a
17 Light of two different frequencies whose photons have
distance r from a photoelectric cell, the cut-off voltage is
energies 1eV and 2.5 eV respectively, illuminate a
V and the saturation current is I. If the same source is
metallic surface whose work function is 0.5eV
placed at a distance 3 r away from the photoelectric cell,
successively. Ratio of maximum speeds of emitted
then
electrons will be j
CBSE AIPMT 2011
(a) no change in saturation current and stopping potential
(a) 1 : 2 (b) 1 : 1 (c) 1 : 5 (d) 1 : 4
342 40 DAYS ~ NEET PHYSICS DAY THIRTY TWO

(b) saturation current will decrease and stopping potential (a) 9 × 1017 (b) 3 × 1016
will not change (c) 9 × 1015 (d) 3 × 1019
(c) saturation current will increase and stopping potential 28 A radiation of energy E falls normally on a perfectly
will decrease
reflecting surface. The momentum transferred to the
(d) None of the above
surface is (where, c = velocity of light) j CBSE AIPMT 2015
25 A 200 W sodium street lamp emits yellow light of (a)
E
(b)
2E
wavelength 0.6 µm. Assuming it to be 25 % efficient in c c
converting electrical energy to light, the number of 2E E
(c) (d)
photons of yellow light it emits per second is c2 c2
j CBSE AIPMT 2012 29 Photon and electron are given same energy (10−20 J).
Wavelength associated with photon and electron are λ p
(a) 1.5 × 1020 (b) 6 × 1018 (c) 62 × 1020 (d) 3 × 1019
and λ e , the correct statement will be
26 A source S1 is producing 1015 photons/s of wavelength
(a) λ p > λe (b) λ p < λe
5000 Å. Another source S 2 is producing1.02 × 1015 λ
(c) λ p = λe (d) e = c
photon/s of wavelength 5100 Å. Then, λp
(power of S 2 )/ (power of S1 ) is equal to j CBSE AIPMT 2010
30 The wavelength λ e of an electron and λ p of a photon of
(a) 1.00 (b) 1.02 (c) 1.04 (d) 0.98
same energy E are related by j NEET 2013
27 Monochromatic light of wavelength 667 nm is produced
(a) λ p ∝ λ2e (b) λ p ∝ λe
by a helium-neon laser. The power emitted is 9 mW. The
1
number of photons arriving per second on the average at (c) λ p ∝ λe (d) λ p ∝
λe
a target irradiated by this beam is j CBSE AIPMT 2009

DAY PRACTICE SESSION 2

PROGRESSIVE QUESTIONS EXERCISE


1 A photosensitive metallic surface has work function h ν 0. 3 When a piece of metal is illuminated by a monochromatic
If photons of energy 2hν 0 fall on this surface, the light of wavelength λ, then stopping potential is 3Vs .
electrons come out with a maximum velocity of When same surface is illuminated by light of wavelength
4 × 106 ms −1. When the photon energy is increased to 2 λ, then stopping potential becomes Vs . The value of
5 hν 0, then maximum velocity of photoelectrons will be threshold wavelength for photoelectric emission will be
4
(a) 2 × 106 ms−1 (b) 2 × 107 ms−1 (a) 4 λ (b) 8 λ (c) λ (d) 6 λ
(c) 8 × 105 ms−1 (d) 8 × 106 ms−1 3

2 Photoelectric effect experiments are performed using 4 Two identical photocathodes receive light of frequencies f1
and f2 . If the velocities of the photoelectrons (of mass m)
three different metal plates p, q and r having work
coming out are respectively v1 and v 2 , then
functions φ p = 2 eV, φ q = 2.5 eV and φ r = 3 eV,
1/ 2
(b) v1 + v 2 =  (f1 + f2 )
respectively. A light beam containing wavelengths of 2h 2h
(a)v12 − v 22 = (f1 − f2 )
m  m 
550 nm, 450 nm and 350 nm with equal intensities
1/ 2
(d) v1 − v 2 =  (f1 − f2 )
illuminates each of the plates. 2h 2h
(c)v12 + v 22 = (f1 + f2 )
The correct I-V graph for the experiment is m  m 
I I
p 5 In a photoelectric experiment, it was found that the
q
(a) r (b) q stopping potential decreases from 1.85 V to 0.82 V as
p r
the wavelength of incident light is varied from 300 nm to
V V 400 nm. Planck constant from this data is
(a)6.634 × 10−34 eVs (b) 4.12 × 10−15 eVs
I p I
q p (c)2 × 10−30 eVs (d) 6.63 × 10−15 eVs
(c) r (d) q
r
6 A horizontal cesium plate (φ = 1.9 eV) is moved vertically
V V downward at a constant speed v in a room full of
radiation of wavelength 250 nm and above. The minimum
DAY THIRTY TWO PHOTOELECTRIC EFFECT 343

value of v, so that the vertically upward component of 10 What will be the number of photons emitted per second
velocity is non-positive for such a photoelectron. by a 10 W sodium vapour lamp assuming that 90% of the
−1 −1
. × 10 ms
(a)104 6
(b) 3 × 10 ms 4 consumed energy is converted into light?
(c)2 × 103 ms −1 (d) None of these [Take wavelength of sodium light is 590 nm and
7 A totally reflecting mirror placed horizontally h = 6.63 × 10−34 J-s]
faces a parallel beam of light as shown in (a) 0. 267 × 1018 (b) 0. 267 × 1019
figure. The mass of the mirror is 20g. (c) 0. 267 × 1020 (d) 0. 267 × 1017
Assume 30% of the light emitted by the 11 When the light of frequency 2ν 0 (where, ν 0 is threshold
source passes through the lens frequency), is incident on a metal plate, the maximum
unabsorbed. The power of the source velocity of electrons emitted is v1. When the frequency of
needed to support the mirror the incident radiation is increased to 5ν 0, the maximum
(a) 104 W (b) 10 2 W (c) 10 8 W (d) 10 −1 W velocity of electrons emitted from the same plate is v 2 .
8 Ultraviolet light of wavelength 66.26 nm and intensity The ratio of v1 to v 2 is j NEET 2018

2 W/m 2 falls on potassium surface by which (a) 4 : 1 (b) 1 : 4 (c) 1 : 2 (d) 2 : 1


photoelectrons are ejected out. If only 0.1% of the 12 When a metallic surface is illuminated with radiation of
incident photons produce photoelectrons and surface wavelength λ, the stopping potential is V. If the same
area of metal surface is 4 m 2 , then the number of surface is illuminated with radiation of wavelength 2λ, the
electrons are emitted per second V
stopping potential is . The threshold wavelength for the
(a) 2.67 × 1015 (b) 3 × 1015 (c) 3.33 × 1017 (d) 4.17 × 1016 4
metallic surface is j NEET 2016
9 In a photocell, with exciting wavelength λ, the faster
5
electron has speed v. If the exciting wavelength (a) 5λ (b) λ (c) 3λ (d) 4λ
2
is changed to 3λ / 4, the speed of the fastest electron
will be 13 Monochromatic radiation emitted when electron state on
1/ 2 1/ 2 hydrogen atom jumps from first excited state to the
(a) v   (b) v  
3 4
ground state irradiates a photosensitive material. The
 4  3
1/ 2 1/ 2 stopping potential is measured to be 3.57 V. The threshold
(c) less than v   (d) greater than v  
4 4
frequency of the material is j
CBSE AIPMT 2012
 3  3
(a) 4 × 1015 Hz (b) 5 × 1015 Hz
. × 1015 Hz
(c) 16 (d) 2.5 × 1015 Hz

ANSWERS
SESSION 1 1 (a) 2 (b) 3 (c) 4 (c) 5 (d) 6 (b) 7 (d) 8 (c) 9 (c) 10 (b)
11 (a) 12 (c) 13 (a,b) 14 (b) 15 (c) 16 (c) 17 (a) 18 (a) 19 (d) 20 (b)
21 (d) 22 (b) 23 (c) 24 (b) 25 (a) 26 (a) 27 (b) 28 (b) 29 (a) 30 (a)
SESSION 2 1 (d) 2 (c) 3 (a) 4 (a) 5 (b) 6 (a) 7 (c) 8 (a) 9 (b) 10 (c)
11 (c) 12 (c) 13 (c)

Hints and Explanations


SESSION 1 the intensity of illumination into a 3 By the concept of threshold minimum
change in photoelectric current. frequency needed for photoelectric
1 Threshold characteristics of A
photoelectric effect supports the particle emission.
Photoelectric current

nature of radiations. 4 Independent of frequency (ν) of light, it


only depends on the intensity of
2 Using the incident radiations of a fixed incident light. If intensity increases,
frequency, it is found that the
number of photoelectrons increases.
photoelectric current increases linearly
with intensity of incident light as shown 5 In the Einstein’s photoelectric equation
in figure. Hence, a photocell employs (E = hν − ω 0 ), E is maximum kinetic
photoelectric effect to convert change in O Intensity energy of photoelectrons.
344 40 DAYS ~ NEET PHYSICS DAY THIRTY TWO

6 Photocurrent 11 From Einstein’s equation, Maximum Kinetic energy of liberated


hc 1 hc
= φ + mv 2max electron, (KE) max = −W
λ 2 λ
1 hc
2 − φ mv 2max = −W
hc
 λ  2 λ
L1 v max = 1
b a m . × 10−31 v 2max
× 91
2
2  − 2.5 eV 
1242 meV
6.6 × 10−34 × 3 × 108
c
 280 nm  = − 6.4 × 10−19
= 0.2 × 10−6
9.1 × 10−31
Retarding potential Anode potential = 9.9 × 10−19 − 6.4 × 10−19 = 3.5 × 10−19
2 × 1.9 × 1.6 × 10−19
=
Since, in the graph, retarding potential is
9.1 × 10−31 3.5 × 10−19 × 2
same in curves (a) and (b) and ∴ v max =
photocurrent is different, so for curves, = 8.2 × 105 m/s . × 10−31
91
they have same frequency, but different 7
12 From photoelectric equation, E k = E − W = × 1012 = 8.8 × 105m/s
intensity of light. 91
.
where, E k is kinetic energy of emitted
7 When a beam of electrons of energy E 0 photoelectrons, 17 Kinetic energy, KE = φ − φ0
is incident on a metal surface kept in Here, KE1 = 1 − 0.5 = 0.5 eV
W is the work function and E is the
an evacuated chamber, electrons can
energy supplied. KE 2 = 2.5 − 0.5 = 2 eV
be emitted with maximum energy E 0
hc KE 1 0.5 1 v2 1
(due to elastic collision) and with any E = hν = ∴ = = or 1 =
energy less than E 0 , when part of λ KE 2 2 4 v 22 4
incident energy of electron is used in 6.6 × 10−34 × 3 × 108
∴ E = v1 1 1
liberating the electrons from the 5000 × 10−10 or = =
v2 4 2
surface of metal. = 3.96 × 10−19 J
8 From Einstein’s photoelectric equation, Also, 1 eV = 1.6 × 10−19 J 18 According to Einstein’s photoelectric
KE max = hν − φ 0 equation, K max = E − φ = hν − φ
−19
3.96 × 10 hc
Comparing with the equation of straight ∴ E = = 2.48 eV K max = −φ …(i)
1.6 × 10−19 λ
line y = mx + c
Similarly, in second case, maximum
Hence, E k = 2.48 − 1.90 = 0.58 eV
KEmax kinetic energy of emitted electron is 3
13 Applying Einstein’s photoelectric times that in first case, we get
equation, we have hc
1 3K max = −φ …(ii)
E = K + φ 0 ⇒ hν = me v 2 + hν 0 λ
2 2
ν 1 1 1  Solving Eqs. (i) and (ii), we get work
⇒ me v 2 = hν − hν 0 = hc  − 
⇒ Slope is h which is same for all 2  λ λ 0
function of an emitted electron from a
metals and independent of the intensity metal surface.
∴Velocity of electron hc
of the incident radiation. φ=
2hc  1 1  2λ
9 The energy of photon, E = hc v =  − 
λ me  λ λ 0  19 hc − ω 0 = eV1 , hc − ω 0 = eV2
hc λ1 λ2
Work function of metal, W =
λ0 2 × 4.14 × 10−15 × 1.6 × 10−19 × 3 × 108 1 1 
Hence, e ( V2 − V1 ) = hc  − 
= 9.1 × 10−31 λ
 2 λ
For photoelectric effect, 1
hc hc  3250 − 2536 
E>W ⇒ > ⇒ λ ≤ λ0    λ − λ2 
λ λ0  3250 × 2536  = hc  1 
≈ 0.6 × 106 ms −1 ≈ 6 × 105 ms −1  λ1 λ2 
10 Work function for wavelength of 4100 Å This gives, V2 = 3 V
is 14 KE = hν − φ ⇒ 0.5 = hν − φ ...(i)
hc 6.62 × 10−34 × 3 × 108 Again, 0.8 = 1.2 = hν − φ ...(ii) 20 Stopping potential
W = = = Maximum kinetic energy
λ 4100 × 10−10 Solving Eqs. (i) and (ii), we get
φ = 1 eV KE max
= 4.8 × 10 −19
J eV = KE max or V =
15 As, mv max = hν ⇒ v 2 max = 2hν ⇒
1 2 e
4.8 × 10−19 Here, KE max = 0.5 eV
= eV = 3 eV 2 m
1.6 × 10−19 2hν 0.5eV
v max = ∴ V = = 0.5V
Now, we have W A = 1.92 eV, m e
W B = 2.0 eV, WC = 5 eV
16 Work function, W = 4 eV 21 Using Einstein’s photoelectric equation,
Since, WA < W.
= 4 × 1.6 × 10 −19
J = 6.4 × 10 −19
J We know that, E = (KE) max +
and W B < W , hence A and B will emit
Work function (φ)
photoelectrons. Wavelength of incident radiation, 1
λ = 0.2µ m = 0.2 × 10−6m (KE ) max = mv 02
2
DAY THIRTY TWO PHOTOELECTRIC EFFECT 345

⇒ ( KE ) max = hν − φ 27 Here, λ = 667 × 10−9 m, 2 K p = E p − φp = 1240 − 2 = 0.2545 eV


⇒ 2 eV = 5eV − φ (given) −3 550
P = 9 × 10 W
⇒ φ = 3 eV 1240
∴ Power =
energy (E ) nhc Nhc
= = Kq = Eq − φq = − 2.5 = 0.255 eV
Thus, Vcathode − Vanode = 3 V 450
time (t ) λt λ
⇒ Vanode − Vcathode = − 3 V
 E = hc ,  K r = E r − φr =
1240
− 3 = 0.543 eV
22 From photoelectric equation,  λ  350
hν = W + eV 0  n = total number of photons,
 N = number of photons  K has maximum kinetic energy of
(where, W = work function)  n 
hc emitted per second = photoelectrons and E is the energy of
So, = W + 3eV 0 …(i)  t 
λ P ×λ incident radiation.
hc So, N =
Also, = W + eV 0 hc From above equation, it is clear that,
2λ 9 × 10−3 × 667 × 10−9
= |V r| > |Vq | > |V p|
hc
⇒ = 2W + 2eV 0 …(ii) 6.6 × 10−34 × 3 × 108 their intensities are equal, but energy of
λ
= 3 × 1016 individual photon of r is maximum.
Subtracting Eq. (i) from Eq. (ii),
we get 0 = W − eV 0 ⇒ W = eV 0 28 The radiation energy is given by Hence, number of photons incident on r
hc can be assumed to be least. Hence,
From Eq. (i), we get E =
hc λ saturation current of r is minimum.
= eV 0 + 3eV 0 = 4eV 0 Initial momentum of the radiation is
λ Hence, option (c) is true.
h E
The threshold wavelength is given by pi = =
4eV 0 λ λ c 3 According to Einstein’s photoelectric
hc
λ th = = = 4λ The reflected momentum is 1 1 
W eV 0 h E equation, eV = hc  − 
pr = − = −  λ λ0 
λ c
23 Energy spent to convert ice into water So, the change in momentum of light is 1 1 
= m L = (1000 g) × (40 cal / g) = 80000 cal Case I 3 eVs = hc  −  ...(i)
2E
∆p light = p r − p i = −  λ λ0 
Energy of photons used c
1 1 
Thus, the momentum transferred to the Case II eVs = hc  −  …(ii)
= n T × E = n T × hν [QE = hν] 2λ λ
surface is  0
mL
∴ n T hν = m L or T = ∆p light =
2E From Eqs. (i) and (ii), we get λ 0 = 4 λ
n hν c
∴ T ∝ 1/n when ν is constant; T ∝ 1/ ν 4 Photon energy, hf = hf 0 + 1 mv 2
29 Wavelength of photon will be greater 2
when n fixed; T ∝ 1/nν. Thus, T is
than that of electron, because mass of 2hf1 2hf 0
constant, if nν is constant.
photon is less than that of electron.
Hence, v 12= − ,
m m
24 When the distance is increased, ⇒ λ p > λe 2hf2 2hf 0
frequency of incident light and hence v 22 = −
the stopping potential does not change, 30 Wavelength of electron, λe = h m m
2mE 2h
but the intensity and hence saturation ∴ v 12 − v 22 = ( f1 − f2 )
and proton, m
current decreases nine times.
h h2 h
λ p = c ⇒ λ2e = or E = c 5 Change in stopping potential,
25 Efficient power, E 2mE λp hν1 φ  hν φ
N hc Vs 1 − Vs 2 = − − 2 − 
P = × = 200 × 0.25 h2 h2 e e  3 e
∴ λ2e = , λ2e = λp
t λ λc 2mλc
2m hc  1 1 
λp = −
N
= 50 ×
λ e  λ1 λ2 
t hc ⇒ λ2e ∝ λ p e ( V s 1 − Vs 2 )
h=
50 × 0.6 × 10−6  1 1 
= = 1. 5 × 1020 SESSION 2 c − 
6.6 × 10−34 × 3 × 108  λ1 λ2 
1 Einstein’s photoelectric equation can be 1.03 eV
26 Number of photons emitted per second is written as =
3 × 10 ×  × 107 
8 1
given by 1
mv 2 = hν − W 0  12 
P  where, P = Power  2
n=  = 4.12 × 10−15 eVs
= Energy 
hc 1
 hc    ⇒ m × (4 × 106 ) 2 = 2hν 0 − hν 0 …(i)
   λ 2
 λ 
1
6 Energy, E (N ) = 1242 = 4.97 eV,
nhc and m × v 2 = 5h ν 0 − hν 0 …(ii) 50
So, P = 2 (KE) max = 4.97 − 1.9 = 3.07 eV
λ
On dividing Eq. (ii) by Eq. (i), we get 1
So, for two different situations, mv2 = 3.07 × 1.6 × 10−19
n λ v2 4 hν 0 2
P2
= 2 1 =
P1 n1 λ2 (4 × 10 )
6 2 hν 0 3.07 × 1.6 × 10−19 × 2
⇒ v =
1.02 × 1015 × 5000 or v = 4 × 16 × 10
2 12
= 64 × 10
12
9.1 × 10−31
= =1 −1
1015 × 5100 ∴ v = 8 × 10 ms
6
= 1.04 × 106 ms −1
346 40 DAYS ~ NEET PHYSICS DAY THIRTY TWO

7 Weight, F = 2P = mg 10 Energy of photon, 12 In first case, when a metallic surface is


c hc 6.63 × 10−34 × 3 × 108 illuminated with radiation of
2(03
. P) E = = wavelength λ, the stopping potential is
⇒ = 20 × 10−3 × 10 λ 590 × 10−9
3 × 108 V.
6.63 × 3
= × 10−18 So, photoelectric equation can be
3 × 108 × (0.2) 59
⇒ P = = 108 W written as
0.6 Light energy produced per second hc hc
90 eV = − …(i)
8 Number of photons falling on metal = × 10 = 9 W λ λ0
100
surface, In second case, when the same surface
Intensity × Area Number of photons emitted per second
np = 9 × 59 is illuminated with radiation of
Energy per quanta = wavelength 2λ, the stopping potential is
6.63 × 3 × 10−18 V
(2 Wm −2 ) (4 m2 ) . So, photoelectric equation can be
np = = 2.67 × 1019 = 0. 267 × 1020 4
 (6.62 × 10−34 ) (3 × 108 )
  written as
 66.26 × 10−9  11 According to the Einstein’s eV hc hc
photoelectric equation, = −
n p = 2.67 × 10 per second
18 4 2λ λ0
1
K max = mv 2max = hν − φ 0 4hc 4hc
ne = 01
. % of n p 2 ⇒ eV = − …(ii)
2λ λ0
=
01
.
× 2.67 × 1018 = hν − hν 0 …(i)
100 When incident frequency of light, From Eqs. (i) and (ii), we get
= 2.67 × 1015 per second ν = 2ν 0 ⇒
hc

hc
=
4hc

4hc
λ λ0 2λ λ0
9 According to Einstein’s photoelectric Substituting the value of ν in Eq. (i), we
1 1 2 4
equation, get ⇒ − = −
1 λ λ0 λ λ0
hc 1 m v 12 = h(2 ν 0 ) − hν 0
= W 0 + mv 12 2 ⇒ λ 0 = 3λ
λ1 2
= 2hν 0 − hν 0 = hν 0 …(ii)
hc 1
13 Concept When an electron in hydrogen
and = W 0 + mv 22 If incident frequency of radiation, atom jumps from first excited state
λ2 2 ν = 5ν 0 (n = 2) to ground state (n = 1), energy is
These expression show that, Substituting the value of ν in Eq. (i), we released and is given by
v 2 ∝  
1 get E = E( n = 2 ) − E( n = 1 )
 λ 1 13.6
mv 22 = h(5ν 0 ) − hν 0 where, E n = − eV
2 n2
 1 
  = 5hν 0 − hν 0 = 4hν 0 …(iii)
v1  λ1  Energy released from emission of
∴ = On dividing Eq. (ii) by Eq (iii), we get electron is given by
v2  1  1
  mv 12 E = − 3.4 − (−13.6 )
 λ2  2 hν 0
= = 102. eV
λ2 1
mv 22 4hν 0 Now, from photoelectric equation,
= 2
λ1 work function,
v 12 1 φ = E − eV = hν
3λ / 4 ⇒ =
= v 22 4 E − eV (102 . − 3.57 )e
λ ν= =
1 /2
or
v1 1
=
h 6.67 × 10−34
v 2 = v 1  
4
∴ v2 2 6.63 × 1.6 × 10−19
 3 ⇒ν = = 1.6 × 1015Hz
∴ v1 : v2 = 1 : 2 6.67 × 10−34
DAY THIRTY THREE

Atoms and
Nuclei
Learning & Revision for the Day
u Scattering of α-particles u Ionisation Energy and u Concept of Nucleus
u Rutherford‘s Model of the Atom Potential u Isotopes, Isobars, and
u Bohr’s Model u Excitation Energy and Isotones
Potential
u Hydrogen Spectrum

Atom is the smallest particle of an element which contains all properties of element.
Nuclei refer to a nucleus of an atom, having a given number of nucleons.

Scattering of α-particles
In 1911, Rutherford successfully explained the scattering of α-particles on the basis of
nuclear model of the atom.
Number of α-particles scattered through angle θ is given by
Z2
N (θ) ∝
sin (θ /2) K 2
4

where, K is the kinetic energy of α-particle and Z is the atomic number of the metal.
At distance of closest approach the entire initial kinetic energy of α-particles is PREP
converted into potential energy, so MIRROR
1 1 Ze(e) 1 2 ze2 ze2 Your Personal Preparation Indicator
mv2 = ⇒ r0 = × = K ×
2 4πε 0 r0 4πε 0 mv2 mv2 u No. of Questions in Exercises (x)—
u No. of Questions Attempted (y)—
u No. of Correct Questions (z)—
Rutherford’s Model of an Atom (Without referring Explanations)
On the basis of scattering of α-particles, Rutherford postulated the following model of
the atom
u Accuracy Level (z / y × 100)—
−10
u Prep Level (z / x × 100)—
l
Atom is a sphere of diameter about 10 m. Whole of its positive charge and most of
its mass is concentrated in the central part called the nucleus. In order to expect good rank in NEET, your
l
The diameter of the nucleus is of the order of 10 − 15 m. Accuracy Level should be above 85 & Prep
Level should be above 75.
348 40 DAYS ~ NEET PHYSICS DAY THIRTY THREE

l
The space around the nucleus is virtually empty with 1 v me 4
l
Orbital frequency is given by f = = = 2 3 3
electrons revolving around the nucleus in the same way as T 2πr 4 ε 0n h
the planets revolve around the sun.
l
The total energy of the orbital electron is
l
The electrostatic attraction of the nucleus provides  me 4Z 2 
centripetal force to the orbiting electrons. E = − 2 2 2 
 8ε 0h n 
l
Total positive charge in the nucleus is equal to the total
negative charge of the orbiting electrons.  me 4  Z 2
= −  2 3  ch 2
Rutherford’s model suffers from the following drawbacks  8ε 0ch  n
(a) stability of the atomic model. Z2 Z2
= − Rch 2
= −13.6 2 eV
(b) nature of energy spectrum. n n
me 4Z 2 me 4Z 2
KE = , PE = −
8n h ε 0
2 2 2
4 n2 h2 ε20
Bohr’s Model l
The kinetic, potential and total energies of the electron
Bohr’s added the following postulates to the Rutherford’s with r as the radius of the orbit are as follows
model of the atom
1  1 Ze2 
KE =  ,
The electrons revolve around the nucleus only in certain 2  4πε 0 r 
l

permitted orbits, in which the angular momentum of the


electron is an integral multiple of h /2 π , where h is the 1 Ze2
PE = −
 nh 4πε 0 r
Planck constant  L = mvn rn = .
 2π  1  1 Ze2 
and E =− 
l
The electrons do not radiate energy, while revolving in the 2  4πε 0 r 
permitted orbits, i.e. the permitted orbits are stationary, Therefore, they are related to each other as follow
non-radiating orbits. KE = − E and PE = 2 E
l
The energy is radiated only when the electron jumps from 1 1
l
For a hydrogen atom rn ∝ n2 , vn ∝ and | E | ∝ 2
an outer permitted orbit to some inner permitted orbit. n n
(Absorption of energy makes the electron jump from inner l
The difference in angular momentum associated with the
orbit to outer orbit)
electron in the two successive orbits of hydrogen atom is
l
If the energy of the electron in nth and mth orbits be E n and h nh h
E m respectively, then while the electron jumps from nth to ∆L = (n + 1) − =
2π 2π 2π
mth orbit, the radiation frequency ν is emitted, such that
E n − E m = hν.
This is called the Bohr’s frequency equation. Hydrogen Spectrum
Hydrogen spectrum consists of spectral lines classified as five
NOTE • Radius of the orbit of electron in a hydrogen atom in its spectral series of hydrogen atom.
stable state, corresponding to n = 1, is called Bohr‘s radius. Out of these five, Lyman series lies in the ultraviolet region of
Value of Bohr‘s radius is r0 = 0 . 529 Å ≈ 0 . 53 Å. spectrum, Balmer series lies in the visible region and the
• The time period of an electron in orbital motion in the Bohr’s remaining three series, lie in the infrared region of spectrum.
2π r 2 π × 0 .53
orbit is T = = Å = 1.52 × 10 − 6 s
v c n=∞
0
137
– 0.28 n=7
1 n=6
and the frequency of revolution is, f = = 6.5757 × 10 15 cps – 0.38
Pfund
T
– 0.54 n=5
E (eV)

Brackett n=4
– 0.85
n=3
Some Characteristics of an Atom –1.51
Paschen
Infrared
l
The orbital radius of an electron is Balmer n=2
–3.40
2 2
n h n 2 Visible light
rn = 4πε 0 = 0.53 Å Lyman
4 π Zme
2 2
Z
series
l
The orbital velocity of an electron is n=1
–13.60
1 2 Zπ e2  c  Z 6 Z
Ultraviolet
vn = =  = 2.2 × 10   m/s
4πε 0 nh  137 n  n Hydrogen spectrum
DAY THIRTY THREE ATOMS AND NUCLEI 349

Total number of emission spectral lines from some excited


state n1 to another energy state n2 (< n1) is given by
Ionisation Energy and Potential
(n1 − n2 )(n1 − n2 + 1) Ionisation energy of an atom is defined as the energy required
.
2 to ionise it, i.e. to make the electron jump from its present
n (n − 1) orbit to infinity. Thus, ionisation energy of hydrogen atom in
e.g. Total number of lines from n1 = n to n2 = 1 is . the ground state = E ∞ − E 1 = 0 − (−13.6 eV) = + 13.6 eV
2
The five spectral series of hydrogen atom are given below The potential through which an electron is to be accelerated,
so that it acquires energy equal to the ionisation energy is
1. Lyman Series called the ionisation potential.
Spectral lines of Lyman series correspond to the transition of Therefore, ionisation potential of hydrogen atom in its ground
electron from higher energy levels (orbits) ni = 2, 3, 4, … to state is 13 . 6 V.
ground energy level (1st orbit) nf = 1. Z2 E
In general, E ion = 13.6 eV or Vion = ion
1  1 1  n2 e
For Lyman series, = ν = R  2 − 2 ,
λ (1) n 
where n = 2, 3, 4, … .
Excitation Energy and Potential
It is found that a term Rch = 13.6 eV = 2.17 × 10 −18 J. The
Excitation energy is the energy required to excite an electron
term Rch is known as Rydberg’s energy.
from a lower energy level to a higher energy level. The
potential through which an electron is accelerated, so as to
2. Balmer Series
gain requisite ionisation energy is called the ionisation potential.
Electronic transitions from ni = 3, 4, 5, … to nf = 2, give rise to
Thus, first excitation energy of hydrogen atom
spectral lines of Balmer series.
 1 = E2 − E 1 = − 3.4 − (− 13.6) eV = + 10.2 eV
1 1 
Thus, for a Balmer series line, = ν = R  2 − 2  Similarly, second excitation energy of hydrogen atom
λ (2) n 
= E3 − E 1 = − 1.51 − (−13.6) = 12.09 eV
where, n = 3, 4, 5, … .

3. Paschen Series Concept of Nucleus


Lines of this series lie in the infrared region and In every atom, the positive charge and mass is densely
correspond to electronic transition from ni = 4, 5, 6,… to concentrated at the centre of the atom forming its nucleus. In
nf = 3. nucleus, the number of protons is equal to the atomic number
1  1 1  of that element and the remaining particles to fulfil the mass
Thus, = ν = R  2 − 2 , where n = 4, 5, 6,... number are the neutrons.
λ (3) n 

4. Brackett Series Composition of Nucleus


It too lies in the infrared region and corresponds to Nucleus consists of protons and neutrons. Electrons cannot
transition from ni = 5, 6, 7, … to nf = 4. exist inside the nucleus. A proton is a positively charged
particle having mass (m p) of 1.007276 u and charge
Thus, for Brackett series,
(+ e) = +1.602 × 10 – 19 C.
1  1 1 
= ν = R  2 − 2 , where n = 5, 6, 7, … For a neutral atom,
λ (4) n 
Number of proton ( Z ) = Number of electron
5. Pfund Series This number is called the atomic number. A neutron is a
It lies in the far infrared region of spectrum and neutral particle having mass mn = 1.008665 u. The number of
corresponds to electronic transitions from higher orbits neutrons in the nucleus of an atom is called the neutron
ni = 6, 7, 8,... to orbit having nf = 5. Thus, we have number N. The sum of the number of protons and neutrons is
1  1 1  called the mass number A. Thus, A = N + Z .
= ν = R  2 − 2 , where n = 6, 7, 8, ...
λ  (5) n 
Properties of Nucleus
NOTE • Energy of emitted radiation,
 1 1 Nuclear size
∆E = E 2 − E 1 = ± RchZ 2  2 − 2 
(a) Size of the nucleus is of the order of fermi
 1
n n2 
(1 fermi = 10 −15 m ).
 1 1
= 136
. Z2 2 − 2  (b) The radius of the nucleus is given by R = R0 A 1/ 3 ,
 n1 n2  where, R0 = 1.3 fermi and A is the mass number.
350 40 DAYS ~ NEET PHYSICS DAY THIRTY THREE

Volume Isotopes, Isobars and Isotones


The volume of nucleus is
4 Isotopes
V = π (R0 A 1/ 3 )3
3 Isotopes of an element are nuclides having same atomic
where, R0 = radius of the nucleus. number Z, but different mass number A (or different neutron
number N) is called isotopes. 11 H, 12 H, 13 H and 11 12 14
6 C, 6 C, 6 C, etc.,
Density are isotopes.
Mass of nucleus
(a) Density = Isobars
Volume of the nucleus
Am p Nuclides having same mass number A, but different atomic
= number Z are called isobars. In isobars number of protons Z as
4
π (R0 A 1/ 3 )3 well as number of neutrons N differ but total nucleon (or
3
mp mass) number A = N + Z is the same. 13 H, 32 He and 14 14
6 C, 7 N are
=
4 isobars.
πR 3
3 0
where, m p = 1.6 × 10 −27 kg = mass of proton and Isotones
R0 = 1.3 fermi. Nuclides with different atomic number Z and different mass
(b) Density of nuclear matter is of the order number A, but same neutron number are called isotones.
of 10 17 kg/m3 . 3 4 198 197
1 H, 2 He and 80 Hg, 79 Au are examples of isotones.
(c) Density of nuclear matter is independent of the mass
number.

DAY PRACTICE SESSION 1

FOUNDATION QUESTIONS EXERCISE


1 In Rutherford scattering experiment, the number of 4 The simple Bohr’s model cannot be directly applied to
α-particles scattered at 60° is 5 × 106. The number of calculate the energy levels of an atom with many
α-particles scattered at 120° will be electrons. This is because
3 (a) of the electrons not being subject to a central force
(a)15 × 106 (b) × 106
5 (b) of the electrons colliding with each other
5
(c) × 106 (d) None of these (c) of screening effects
9
(d) the force between the nucleus and an electron will no
2 In a Rutherford scattering experiment, when a projectile longer be given by Coulomb’s law
of charge Z 1 and mass M1 approaches a target nucleus 5 For the ground state, the electron in the H-atom has an
of charge Z 2 and mass M 2 , the distance of closest angular momentum = h, according to the simple Bohr’s
approach is r0. The energy of the projectile is model. Angular momentum is a vector and hence there
(a) directly proportional to M1 × M 2 j
CBSE AIPMT 2009 will be infinitely many orbits with the vector pointing in all
(b) directly proportional to Z1Z 2 possible directions. In actual, this is not true,
(c) inversely proportional to Z1
(d) directly proportional to mass M1 (a) because Bohr’s model gives incorrect values of angular
momentum
3 In Rutherford experiment, a 5.3 MeV α-particle moves
(b) because only one of these would have a minimum
towards the gold nucleus ( Z = 79). How close does the energy
alpha particle to get the centre of the nucleus, before it (c) angular momentum must be in the direction of spin of
comes momentarily to rest and reverses its motion? electron
(Take, ε 0 = 8.8 × 10−12 F / m) (d) because electrons go around only in horizontal orbits
(a) 3.4 × 10−15 m (b) 8.6 × 10−14 m
(c) 4.5 × 10−13 m (d) 1.6 × 10−14 m
DAY THIRTY THREE ATOMS AND NUCLEI 351

6 In the lowest energy level of hydrogen atom, the electron 15 If an electron in a hydrogen atom jumps from the 3rd
has the angular momentum orbit to the 2nd orbit, it emits a photon of wavelength λ.
π h h 2π When it jumps from the 4th orbit to the 3rd orbit, the
(a) (b) (c) (d)
h π 2π h corresponding wavelength of the photon will be
7 Taking the Bohr radius as a 0 = 53 pm, the radius of Li + + j NEET 2016
16 9 20 20
ion in its ground state, on the basis of Bohr’s model, will (a) λ (b) λ (c) λ (d) λ
be about 25 16 7 13
(a) 53 pm (b) 27 pm (c) 18 pm (d) 13 pm 16 Hydrogen atom in ground state is excited by a
8 How many revolutions does an electron complete in one monochromatic radiation of λ = 975 Å. Number of
second in the first orbit of hydrogen atom? spectral lines in the resulting spectrum emitted will be
. × 1015 rev/s
(a) 657 (b) 100 rev/s
j CBSE AIPMT 2014
(c) 1000 rev/s (d) 1 rev/s (a) 3 (b) 2 (c) 6 (d) 10
9 The ratio of kinetic energy to the total energy of an 17 The ionisation energy of the electron in the hydrogen
electron in a Bohr orbit of the hydrogen atom, is atom in its ground state is 13.6 eV. The atoms are excited
(a) 2 : − 1 (b) 1 : − 1 (c) 1 : 1 (d) 1 : − 2 to higher energy levels to emit radiations of 6 wavelengths.
Maximum wavelength of emitted radiation corresponds to
10 Which of the following transition in hydrogen atoms limit
the transition between j CBSE AIPMT 2009
photons of highest frequency?
(a) n = 3 to n = 2 states (b) n = 3 to n = 1 states
(a) n = 1to n = 2 (b) n = 2 to n = 6 (c) n = 2 to n = 1 states (d) n = 4 to n = 3 states
(c) n = 6 to n = 2 (d) n = 2 to n = 1
18 Monochromatic radiation of wavelength λ is incident on a
11 Given that, R is Rydberg’s constant. When an electron in
hydrogen sample. In ground state, hydrogen atom
an atom of hydrogen jumps from an outer orbit n = 3 to
absorbs a fraction of light and subsequently emits
an inner orbit n = 2, the wavelength of emitted radiations
radiation of three different wavelengths. The wavelength
will be equal to
λ is
R 62
(a) (b) (a) 102.73 nm (b) 121.6 nm
62 R (c)110.3 nm (d) 45.2 nm
5R 36
(c) (d)
36 5R 19 The limit of Balmer series is 3646 Å. The wavelength of
first member of this series will be
12 An excited hydrogen atom returns to the ground state.
(a) 6563 Å (b) 3646 Å
The wavelength of emitted photon is λ. The principal (c) 7200 Å (d) 1000 Å
quantum number of the excited state will be
1/ 2 1/ 2
20 According to Bohr’s theory (assuming infinite mass of the
 λR  λR − 1
(a)   (b)   nucleus), the frequency of the second line of the Balmer
 λR − 1  λR 
series is
1/ 2
 1  (a) 6.16 × 1014 Hz (b) 6.16 × 1010 Hz
(c) [λR (λR − 1)]1/ 2 (d)  
 λ R (λ R − 1)  (c) 6.16 × 1013 Hz (d) 6.16 × 1016 Hz

13 In an inelastic collision, an electron excites a hydrogen 21 In the spectrum of hydrogen, the ratio of the longest
atom from its ground state to a M-shell state. A second wavelength in the Lyman series to the longest
electron collides instantaneously with the excited wavelength in the Balmer series is j
CBSE AIPMT 2015
4 9 27 5
hydrogen atom in the M-state and ionises it. At least how (a) (b) (c) (d)
9 4 5 27
much energy the second electron transfers to the atom in
the M-state? 22 The ratio of wavelengths of the last line of Balmer series
(a) + 3.4 eV (b) + 1.51 eV and the last line of Lyman series is j
NEET 2017
(c) – 3.4 eV (d) – 1.51 eV (a) 2 (b) 1
14 A hydrogen like atom emits radiations of frequency (c) 4 (d) 0.5

2.7 × 10 15 Hz when it makes a transition from n = 2 to n = 1. 23 The transition from the state n = 4 to n = 3 in a hydrogen
The frequency emitted in a transition from n = 3 to n = 1 like atom results in ultraviolet radiation. Infrared radiation
will be will be obtained in the transition from
(a) 1.6 × 1015 Hz (b) 3.2 × 1015 Hz (a) 2 → 1 (b) 3 → 2
(c) 4.8 × 1015 Hz (d) 6.4 × 1015 Hz (c) 4 → 2 (d) 5 → 3
352 40 DAYS ~ NEET PHYSICS DAY THIRTY THREE

24 The energy of electron in the nth orbit of hydrogen atom (c) the series limit of Lyman series, second member of
−13.6 Balmer series and second member of Paschen series
is expressed as E n = eV. The shortest and longest
n2 (d) the series limit of Lyman series, third member of Balmer
wavelength of Lyman series will be series and second member of Paschen series
(a) 910 Å , 1213 Å (b) 5463 Å , 7858 Å 27 The ground state energy of hydrogen atom is −13.6 eV.
(c) 1315 Å , 1530 Å (d) None of these When its electron is in the first excited state, its excitation
25 ν1 is the frequency of the series limit of Lyman series, ν 2 energy is
is the frequency of the first line of Lyman series and ν 3 is (a) 3.4 eV (b) 6.8 eV (c) 10.2 eV (d) zero
the frequency of the series limit of the Balmer series.
28 The energy of a hydrogen atom in the ground state is
Then,
– 13.6 eV. The energy of a He+ ion in the first excited
(a) ν1 − ν2 = ν3 (b) ν1 = ν2 − ν3 state will be j CBSE AIPMT 2010

1 1 1 1 1 1
(c) = + (d) = + (a) –13.6 eV (b) –27.2 eV (c) – 54.4 eV (d) – 6.8 eV
ν2 ν1 ν3 ν1 ν2 ν3
29 The ionisation potential of hydrogen atom is 13.6 eV. The
26. In figure, the energy levels of the hydrogen atom have energy required to remove an electron from the second
been shown along with some transitions marked A, B and orbit of hydrogen will be
C. The transitions A, B and C respectively, represents (a)27.4 eV (b) 13.6 eV
(c)3.4 eV (d) None of these
Continuum 0 eV
30 The total energy of the electron orbiting around the
n=5 – 0.54 eV nucleus in the ground state of the atom is
n=4 – 0.85 eV (a) less than zero
C
n=3 –1.51 eV (b) zero
B
n=2 – 3.40 eV (c) more than zero
A (d) sometimes less and sometimes more than zero
n=1 –13.60 eV
(a) the first member of the Lyman series, third member of 31 The ratio of nuclear radii of the gold isotope 79 Au 197 and
107
Balmer series and second member of Paschen series the silver isotope 47 Ag is
(b) the ionisation potential of H, second member of Balmer (a) 0.233 (b) 2.33 (c) 1.225 (d) 12.25
series and third member of Paschen series

DAY PRACTICE SESSION 2

PROGRESSIVE QUESTIONS EXERCISE


1 The total energy of electron in the ground state of 4 In a hydrogen like atom electron make transition from an
hydrogen atom is −13.6 eV. The kinetic energy of an energy level with quantum number n to another with
electron in the first excited state is quantum number (n −1). If n >> 1, the frequency of
(a) 3.4 eV (b) 6.8 eV radiation emitted is proportional to
(c) 13.6 eV (d) 1.7 eV 1 1 1 1
(a) (b) (c) (d)
2 An α-particle after passing through a potential difference n n2 n3 / 2 n3
of V volt collides with a nucleus. If the atomic number of 5 In the Bohr’s model of a hydrogen atom, the centripetal
the nucleus is Z , then the distance of closest approach is force is furnished by the Coulomb attraction between the
Z Z proton and the electron. If a 0 is the radius of the ground
(a) 14.4 Å (b) 14.4 m
V V state orbit, m is the mass and e is the charge on the
V V
(c) 14.4 m (d) 14.4 Å electron, ε 0 is the vacuum permittivity, the speed of the
Z Z
electron is
3 When an electron jumps from a level n = 4 to n = 1, (a) zero (b)
e
momentum of the recoiled hydrogen atom will be ε0 a0 m
(a) 6.8 × 10−27 kg ms −1 (b) 12.75 × 10−19 kg ms −1 e 4 π ε0 a0 m
(c) (d)
(c) 136 × 10−19 kg ms −1 (d) zero 4 π ε0 a0 m e
DAY THIRTY THREE ATOMS AND NUCLEI 353

6 The binding energy of a H-atom, considering an electron K = 9 × 109 constant, Z = 2 and h (Planck constant)
me 4
= 6.6 × 10 −34 J-s] j CBSE AIPMT 2015
moving around a fixed nuclei (proton), is B = −
8 n 2 ε 20 h 2 (a)2.92 × 106 m/s (b) 1.46 × 106 m/s
(where, m = electron mass) (c)0.73 × 106 m/s (d) 3.0 × 108 m/s
If one decides to work in a frame of reference, where the 11 Electron in hydrogen atom first jumps from third excited
electron is at rest, the proton would be moving around it. state to second excited state and then from second
By similar arguments, the binding energy would be excited to the first excited state. The ratio of the
Me 4 wavelengths λ 1 : λ 2 emitted in the two cases is
B=−
8 n 2 ε 20 h 2 j CBSE AIPMT 2012

(where, M = proton mass) (a) 7/5 (b) 27/ 20 (c) 27/5 (d) 20/ 7
12 The wavelength of the first line of Lyman series for
This last expression is not correct, because
hydrogen atom is equal to that of the second line of
(a) n would not be integral
Balmer series for a hydrogen like ion. The atomic number
(b) Bohr quantisation applies only to electron
Z of hydrogen like ion is j CBSE AIPMT 2011

(c) the frame in which the electron is at rest is not inertial


(a) 2 (b) 3 (c) 4 (d) 5
(d) the motion of the proton would not be in circular orbits,
even approximately 13 An electron of a stationary hydrogen atom passes from
7 The recoil speed of hydrogen atom after it emits a the fifth energy level to the ground level. The velocity that
photon in going from n = 5 state to n = 1 state is the atom acquired as a result of photon emission will be
CBSE AIPMT 2012
(Take, R ∞ = 1. 097 × 107 m −1, h = 6 . 63 × 10−34 J-s,
j

24 hR 25 hR 24 m 25 m
MH = 1.67 × 10 −27 kg) (a) (b) (c) (d)
25 m 24 m 25 hR 24 hR
(a)2.2 ms −1 (b) 4.18 ms −1
(c)6.2 ms −1 (d) 1 ms −1 14 Hydrogen (1H1 ), deuterium (1H2 ), singly ionised helium
8 A hydrogen atom moves with velocity u and makes head ( 2 He4 )+ and doubly ionised lithium ( 3 Li8 )+ + all have one
on inelastic collision with another stationary H-atom. Both electron around the nucleus. Consider an electron
atoms are in ground state before collision. The minimum transition from n = 2 to n =1 . If the wavelengths of emitted
value of u, if one of the them is to be given a minimum radiation are λ 1, λ 2 , λ 3 and λ 4 respectively, for four
excitation energy is elements, then approximately which one of the following
(a)2.64 × 104 ms −1 (b) 6.24 × 104 ms −1 is correct?
(c)2.64 × 108 ms −1 (d) 6.24 × 108 ms −1 (a) 4 λ1 = 2 λ 2 = 2 λ 3 = λ 4 (b) λ1 = 2 λ 2 = 2 λ 3 = λ 4
9 Ionisation potential of hydrogen atom is 13.6 eV. (c) λ1 = λ 2 = 4 λ 3 = 9λ 4 (d) λ1 = 2 λ 2 = 3 λ 3 = 4 λ 4
Hydrogen atoms in the ground state are excited by 15 The radius of the orbit of an electron in a hydrogen like
monochromatic radiation of photon energy 12.1 eV. The atom is 4.5 a 0, where a 0 is the Bohr radius. Its orbital
spectral lines emitted by hydrogen atoms according to 3h
angular momentum is . It is given that h is Planck’s
Bohr’s theory will be 2π
(a) one (b) two constant and R is Rydberg’s constant. The possible
(c) three (d) four wavelength(λ ), when the atom de-excites, is (are)
10 Consider 3rd orbit of He + (helium), using non-relativistic (a)
9
(b)
9
(c)
9
(d)
4
approach, the speed of electron in this orbit will be [Take, 32R 16R 10R 3R

ANSWERS
SESSION 1 1 (c) 2 (b) 3 (c) 4 (a) 5 (a) 6 (c) 7 (c) 8 (a) 9 (b) 10 (a)
11 (d) 12 (a) 13 (d) 14 (b) 15 (c) 16 (c) 17 (d) 18 (a) 19 (a) 20 (a)
21 (d) 22 (c) 23 (d) 24 (a) 25 (a) 26 (d) 27 (c) 28 (a) 29 (c) 30 (a)
31 (c)

SESSION 2 1 (a) 2 (a) 3 (a) 4 (d) 5 (c) 6 (c) 7 ( b) 8 (b) 9 (c) 10 (b)
11 (c) 12 (a) 13 (a) 14 (c) 15 (a)
354 40 DAYS ~ NEET PHYSICS DAY THIRTY THREE

Hints and Explanations


SESSION 1 In the lowest energy level, n = 1 . 1 1 1 
12 ∴ = R −
1 Number of α-particles scattered at angle Then,  h 
mvr1 = 1   λ  1 n2 
 2π 
θ, 1 1 λR − 1
1 =
h or =1− =
N ∝ n2 λR λR
θ 2π
sin 4 λR
2 7 On the basis of Bohr’s model, or n=
4 λR − 1
 θ  n h 2 2
n2
sin 1 r = = a0
N2  2

4 π2 m KZe 2
∴ =  Z 13 We know that, E m = − 13.62 eV
N 1  sin θ2  ++ (n )
  For Li ion, Z = 3, n = 1 for ground
2 13.6
4 state. Em = − = − 1.51
 sin 60°  Given, a0 = 53 pm
(3)2
 
or N 2 = 5 × 106 ×  2  = 5 × 106 53 × 12 Minimum energy required by electron
 sin 120°  9 ∴ r = = 18 pm
should be – 1.51 eV.
 3
2 
1 ν 1 1 
8 By using, f n = 4 πK 3Z 3e m
2 2 4
2 At the distance of closest approach, the 14 Q = = RZ 2  − 
n h λ c 2
 n1 n22 
kinetic energy of particle is completely
converted to potential energy.  vn  1 1 
 1 − 4 
1
Q f n = =  ν1 2.7 × 1015 3 × 9
Mathematically,
 T n 2πr n  ⇒ =  or =
1
M1u2 =
1 Z1 Z2 ν2 1 1  ν2 4× 8
2 4 πε 0 r 0 12.56 × 81 × 1018 × (1.6 × 10−19 )4  1 − 9 
. × 10−31
× 91 or ν 2 = 3.2 × 1015 Hz
So, the energy of the particle is directly ∴ f1 =
proportional to Z1 Z2 . (6.62 × 10−34 )3
15 From Rydberg’s formula,
= 6.57 × 10 rev/s
15
3. The kinetic energy of α-particles is 1  1 1 
= R  − 
n22 
completely converted into potential 9 Kinetic energy of an electron in a Bohr λ 2
 n1
energy is given by orbit of a hydrogen atom is given as
1 1 (2e )(Ze ) Rhc 1 1  5R
mu2 = KE n = …(i) =R −  =
2 4 πε0  22 32  36
r0 n2
1 1 1 7R
∴ Distance of closest approach, Total energy of an electron in a Bohr =R −  =
λ′  32 42  144
1 2Ze 2 orbit of a hydrogen atom is given as
r0 = − Rhc 1 1 5R 7R
4 πε0 1 mu2 TE n = …(ii) ∴ / = ÷
n2 λ λ′ 36 144
2 λ′ 5R 144 20
9 × 109 × 2 × 79 (1.6 × 10 −19 ) 2 Dividing Eq. (i) by Eq. (ii), we get ⇒ = × =
= λ 36 7R 7
0.5 × 1.6 × 10 −19 × 106  Rhc  20
 2  ⇒ λ′ = λ
KE n  n 
9 × 2 × 79 × 1.6 × 1.6 × 10 9 × 10−38 = 7
= TE n  Rhc 
0.5 × 1.6 × 10−19 × 106 − 
 n2  16 Energy incident = hc
= 4.5 × 10 −13 m λ
⇒ KE n : TE n = 1 : − 1 6.63 × 10−34 × 3 × 108
= eV
4 The simple Bohr’s model cannot be 10 Energy emitted 975 × 10−10 × 1.6 × 10−19
directly applied to calculate energy  1 1  = 12.75 eV
levels of an atom with many electrons. = E2 − E1 = R  − 
2 2 The hydrogen atom will be excited to
This is because, all the electrons in the  2
n n1
n=4
atom are not being subjected to one which is highest for n2 = 1 to n1 = 2. 4(4 − 1)
single central force. ∴Number of spectral lines = =6
  2
5 Bohr’s model does not give correct 11 As, 1 = R  12 − 12 
λ  n1 n2  17 Number of spectral lines,
values of angular momentum of
Here, n2 = 3 and n1 = 2 n(n − 1) n(n − 1)
revolving electron. It gives only the N = ⇒ =6
magnitude of angular momentum, 1  1 1  2 2
⇒ = R − 
which is a vector. So, the given λ  (2)2 (3)2  or n − n − 12 = 0
2

statement in the question is not true. or (n − 4)(n + 3) = 0 or n = 4


 32 − 22   5
6 Angular momentum of electron in nth = R  = R 
 32 × 22   36  Now, as the first line of the series has
orbit is given by the maximum wavelength, therefore
36
mvr n =
nh ⇒ λ= electron jumps from the fourth orbit to
2π 5R the third orbit.
DAY THIRTY THREE ATOMS AND NUCLEI 355

18 As H-atom emits three spectral lines, For last line of Balmer series, 28 Energy E of an atom with principal
n(n − 1) n1 = 2 and n2 = ∞ quantum number n is given by
=3 1 1 1  R −13.6 2
2 ⇒ = Z2 R  − = E = Z
λB  22 ∞2  4 n2
Q n=3
[QZ = 1] For first excited state n = 2 and for He + ,
∴ ∆E = E 4 − E1 = 13.6 − 1.51
Similarly, for last line of Lyman series, Z =2
= 12.09 eV n1 = 1 and n2 = ∞ −13.6 × (2)2
1 1 1  So, E = = − 13.6 eV
∴ λ=
1242
= 102.73 nm ⇒ = Z2 R  −  =R (2)2
12.09 λ2  12 ∞2 
1 R 29 The potential energy of hydrogen atom,
= R  − 
1 1 1
19 ∴ λB 1 En = −
13.6
eV
λ limit 4 ∞  ∴ = 4 =
1 R 4 n2
1 1 1 λL So, the potential energy in second orbit
⇒ = R − 
λ first  4 9 is
λL 1
⇒ = 13.6
36 λB 4 E2 = − eV = − 3.4 eV
∴ λ first = 3646 × = 6563 Å ( 2 )2
4× 5 λΒ
⇒ =4 Now, the energy required to remove an
λΛ
20 Wavelength of Balmer series is given by electron from second orbit to infinity, is
1  1 1  23 Infrared radiation corresponds to least U = E ∞ − E2
= R  − 
λ 2
 n1 n22   1
value of  −
1 
 , i.e. from Paschen,
[from work-energy
theorem and E ∞ = 0]
ν  1 1 
2
 n1 n22 
= R  −  [Qc = νλ] ⇒ U = 0 − (−3.4) eV
c 2
 n1 n22  Brackett and Pfund series. Thus, the or U = 3.4 eV
transition corresponds to 5 → 3.
 1 1  Hence, the required energy is 3.4 eV.
ν = cR  −  1  1 1 
2
 n1 n22  24 ∴ =R −  30 The total energy of the electron orbiting
λ max  (1 )2
(2 )2  around the nucleus in the ground state
For second line of Balmer series, 4
⇒ λ max = ≈ 1213 Å of the atom is less than zero.
n1 = 2; n2 = 4
3R
1 1 31 Q R = R 0 A1 /3
ν = 3 × 108 × 10967800  −  1  1 1
 22 42  and =R − 
λ min  (1)
2 ∞ R (Au197 )  197 
1 /3
= 6 .16 × 1014 Hz ∴ =  = (1.841)1 /3
⇒ λ min =
1
≈ 910 Å R (Ag 107
)  107 
21 In hydrogen atom, wavelength of R
1 
characteristic spectrum, = antilog  log(1.841)
25 We know that, frequency, 3 
1 1 1 
= RZ 2  −  1 1 
λ 2
n22  ν = RC  −  1 
 n1 2
n22  = antilog  × 0.2650
 n1 3 
For maximum wavelength in Lyman
ν1 = RC 1 −  = RC
1
series, n1 = 1, n2 = 2 …(i) = antilog(0.08833) = 1.225
 1  ∞
1 1 
= RZ 2  −  …(i)  1 3
λ1  (1)2
(2 )2  ν2 = RC 1 −  = RC …(ii) SESSION 2
 4 4
For maximum wavelength in Balmer 1 The energy of hydrogen atom when the
ν3 = RC  −  =
1 1 RC
series, n1 = 2, n2 = 3 …(iii) electron revolves in nth orbit, is
4 ∞  4 −13.6
1  1 1  E = eV
= RZ 2  −  …(ii) On comparing Eqs. (i), (ii) and (iii), we n2
λ2  (2)
2
(3)2  get In the ground state, n = 1
Dividing Eq. (ii) by Eq. (i) we get ⇒ ν1 − ν2 = ν3 −13.6
E = = −13.6 eV
12
RZ 2  − 
1 1 5 26 A represents series limit of Lyman
λ1  4 9  36 series, B represents third member of −13.6
= = For n = 2, E = = −3.4 eV
λ2 22
RZ 2 1 − 
1 3 Balmer series and C represents second
 4 4 member of Paschen series. So, kinetic energy of electron in the first
27 In the ground state, n = 1 excited state (i.e. for n = 2), is
λ1 5 4 5
∴ = × = KE = −E = −(−3.4) = 3.4 eV
λ2 36 3 27 E1 = −
13.6
= −13.6 eV
12 2 KE of a particle = 2 eV
22 Wavelength of spectral lines are given For the first excited state (i.e. for n = 2), 2(Ze )(e ) 2Ze 2 × 9 × 109
by −13.6 r = =
 1 E2 = = −3.4 eV 4 πε0 (KE ) 2eV
1 1  22
= Z2 R  − 
λ 2
 n1 n22  ∴ ∆ E = E2 − E1 = − 3.4 + 13.6 ⇒ r =
2 × Ze × 9 × 109
2V
= 10.2 eV
356 40 DAYS ~ NEET PHYSICS DAY THIRTY THREE

2 × Z × 1.6 × 10−19 × 9 × 109 Momentum of photon, Similarly, for λ2 ,


⇒ r=
2V h 6.63 × 10−34 hc  1 1 
p= = = 6.98 × 10−27 = −13.6  −
2
Z
= 14.4 Å λ 0.95 × 10−7 λ2 2
 (2) (1) 
V From conservation of linear momentum, hc 3
= 13.6   ...(ii)
13.6  13.6  Momentum of recoiled H-atom = λ2  4
3∴ E = E 4 − E1 = − − − 
42  12  Momentum of photon Hence, from Eqs. (i) and (ii), we get
= − 0.85 + 13.6 = 12.75 eV h λ1 27
mH × v = =
= 12.75 × 1.6 = 10−14 J λ λ2 5
(1.67 × 10−27 ) × v = 6.98 × 10−27
E 12.75 × 1.6 × 10−19
p= = 6.98 12 Lyman series for H-ion,
c 3 × 108 ∴ v= = 4.18 ms −1 hc 1 1
1.67 = Rhc  − 
= 6.8 × 10−27 kg ms −1 λ  12 22 
u
This must be the momentum of recoiled 8 mu = 2mv or v = and for H-like ion,
2
hydrogen atom hc 1 1
2 = Z 2 Rhc  − 
(in opposite direction). 1 1 u mu2 λ  22 42 
∆E = mu2 − ( 2m)   =
2 2  2  4 1 1
4 ∴∆ E = hν 1 1
∴  −  = Z2  − 
1 1 1  12 22   4 16 
∆E  1 1  mu2 = 13.6  − 
⇒ ν= = K −  4  12 22   1 21 1
h  (n − 1)
3
n2  1 −  = Z  − 
1  4  4 16 
K 2n 2K (1.0078) (1.66 × 10−27 )u2
1 ∴ Z =2
= = ∝ 4
n2 (n − 1)2 n3 n3 = 10.2 × 1.6 × 10−19 hc
13 Here, E 5 − E1 =
5 From Coulomb’s attraction between the ⇒ u = 6.24 × 104 ms −1 λ
positive proton and negative electron Rhc hc 24 1
9 Energy in excited state and − Rhc = ⇒ R=
1 e2 25 λ 25 λ
= [for neutral atom] = −13.6 + 12.1 = −1.5 eV
h h 24 Rh
4 πε 0 r 2 −13.6 But p = and ν = =
∴ = −1.5 λ mλ 25 m
Centripetal force has magnitude, n2
mv 2 13.6 14 For hydrogen, we get
F= ∴ n= =3 1 1 1 1  3
r 1.5 = RZ 2  − ⇒ = R(1)2  
λ  12 22  λ1  4
So, for the revolving electrons, Number of spectral lines
n(n − 1) 3(3 − 1) 1 2 3 1 2 3
mv 2 1 e2 = = =3 = R(1)   ⇒ = R(2)  
= 2 2 λ2  4 λ3  4
r 4 πε 0 r 2
10 Energy of electron in He + 3rd orbit 1  3
= R(3)2  
1 e2 2 λ4  4
⇒v = 2
= −13.6 ×eV
Z
4 πε0 mr 1 1 1 1
n2 ⇒ = = =
e λ1 4λ3 9λ 4 λ2
or v = 4
E3 = −13.6 × eV
4 πε0 mr 9 ⇒ λ1 = λ2 = 4λ3 = 9 λ 4
For ground state of H-atom, r = a0 4
= −13.6 × × 1.6 × 10−19 J 15 Orbital angular momentum is given by
e 9  h 
∴ v = L = 3
4 πε0 ma0 In Bohr’s model, E3 = − KE3 
 2π 
1
∴ 9.7 × 10−19 J = me v 2
n = 3, as L = n 
h 
6 In a hydrogen atom, electron 2 ∴ 
revolving around a fixed proton  2π 
2 × 9.7 × 10−19 n2
nucleus has some centripetal v= rn ∝ ⇒ r3 = 4.5 a0
acceleration. Therefore, its frame of 9.1 × 10−31 Z
reference is non-inertial. In the frame = 1.46 × 10 m/s
6
∴ Z =2
of reference, where the electron is at 1 1 1  1 1
rest, the given expression cannot be 11 Here, for wavelength λ1 , = RZ 2  −  = 4R  − 
λ1  22 32   4 9
true for binding energy as the frame n 1 = 3 and n 2 = 2 9
in which electron is at rest would not And for λ2 , n 1 = 2 and n 2 = 1. ⇒ λ1 =
5R
be inertial. hc 1 1 
= −13.6  − 1 1 1  1
2 = RZ 2  −  = 4R  1 − 
We have,
  λ 2
 2 1 λ2  12 32   9
7 Q 1 = R  12 − 12  n n 
λ  n1 n2  So, for λ1 , 9
⇒ λ2 =
hc  1 1  32R
1 1 = −13.6  −
= (1.097 × 107 )  −  λ1 2 2 1 1 1  1
 12 52   (3) (2)  = RZ 2  −  = 4R  1 − 
λ3  12 22   4
hc  5
= 1.053 × 107 ms −1 = 13.6   ...(i) 1
λ1  36  ⇒ λ3 =
∴ λ = 0.95 × 10−7 m 3R
DAY THIRTY FOUR

Radioactivity
Learning & Revision for the Day
u Law of Radioactive Decay u Nuclear Fusion
u Nuclear Fission u Mass Defect and Binding Energy

Radioactivity is the phenomenon of spontaneous emission of radiations by heavier nucleus.


Some naturally occurring radioactive substances are uranium, thorium, polonium,
radium, neptunium, etc. In fact, all elements having atomic number Z > 82 are
radioactive in nature.
Radiations emitted by radioactive substances are of three types, namely (i) α-particles,
(ii) β-particles and (iii) γ- rays.
l
α-particles are positively charged particles with charge q α = + 2 e and mass mα = 4m p.
Thus, α-particles may be considered as helium nuclei (or doubly charged helium ions).
Ionising power of α-particles is maximum, but their penetrating power is minimum.
l
β-particles are negatively charged particles with rest mass as well as charge same as
that of electrons. But origin of β-particles is from the nucleus. Their ionising power is
lesser than that of α-particles, but speed as well as penetrating power is much greater
than that of α-particles. Generally, β-decay means β − - decay.
l
γ-rays are electromagnetic radiations of extremely short wavelengths. Thus, γ-rays
travel with the speed of light. Their ionising power is least, but penetrating power is
extremely high. These are not deflected either in an electric or a magnetic field.
PREP
Law of Radioactive Decay MIRROR
Your Personal Preparation Indicator
According to Rutherford-Soddy’s law for radioactive decay, ‘The rate of decay of a
radioactive material at any instant is proportional to the quantity of that material actually u No. of Questions in Exercises (x)—
present at that time.’ u No. of Questions Attempted (y)—
dN dN No. of Correct Questions (z)—
∝ N or = − λN
u
Mathematically,
dt dt (Without referring Explanations)

Here, λ is a proportionality constant, known as the decay constant (or disintegration


u Accuracy Level (z / y × 100)—
constant). Unit of λ is s–1 or day –1 or year –1 , etc. u Prep Level (z / x × 100)—
It can be shown that number of nuclei present after time t is given by
In order to expect good rank in NEET, your
N = N 0 e − λt Accuracy Level should be above 85 & Prep
Level should be above 75.
where, N 0 = number of nuclei present at time t = 0.
358 40 DAYS ~ NEET PHYSICS DAY THIRTY FOUR

Again, number of nuclei decayed in time t will be If the principle of conservation of energy is to hold for nuclear
− λt reactions it is clear that mass and energy must be regarded as
N − N 0 = N 0 [e − 1]
equivalent. The implication of E = mc2 is that any reaction
= number of daughter nuclei produced at time t .
producing an appreciable mass decrease is a possible source
of energy.
Half-Life Period (T1/2 ) l
At the rest, mass energy of each of electron and positron, is
It is the time in which, activity of the sample falls to one-half E 0 = m0 c2 = 9.1 × 10 −31 × (3 × 10 8)2 J
of its initial value.
= 0.51 MeV
T N R
Thus, for t = , N = 0 and R = 0 Therefore, an energy of atleast 1.02 MeV is needed for pair
2 2 2
production.
l
The half-life period is related to decay constant λ as
0.693
T1/ 2 =
λ Mass Defect and Binding Energy
l
After n half-lives, the quantity of a radioactive substance l
The difference in mass of a nucleus and its constituent
left intact (undecayed) is given by nucleons is called the mass defect of that nucleus. Thus,
n
t Mass defect, ∆M = Zm p + ( A − Z )mn − M
 1  1 T
N = N 0   = N 0   1 /2 where, M is the mass of a given nucleus.
 2  2
l
Packing fraction of an atom is the difference between mass
of nucleus and its mass number per nucleon. Thus,
Mean Life Period ( τ ) Packing fraction =
M−A
.
l
Mean life of a radioactive sample is the time, at which both A
1 The energy equivalent of the mass defect of a nucleus is
N and R have been reduced to or e −1 or 36.8% of their
l

e called its binding energy.


1 Thus, binding energy, ∆Eb = ∆M c2
initial values. It is found that τ = .
λ = [Zm p + ( A − Z )mn − M ] c2
l
Half-life T1/ 2 and mean life τ of a radioactive sample are
If masses are expressed in atomic mass units, then
correlated as, T1/2 = 0.693 τ or τ = 1.44T1/2 .
∆Eb = ∆M × 931.5 MeV
= [Zm p + ( A − Z )mn − M ] × 931.5 MeV
Activity l
Binding energy per nucleon (∆E bn ) is the average energy
The activity of a radioactive substance is defined as the rate of
needed to separate a nucleus into its individual nucleons.
disintegration (or the count rate) of that substance.
∆Eb ∆M × 931 MeV
Mathematically, activity is defined as Thus, ∆E bn = =
A A Nucleon
dN
R=− = λN = λN 0 e − λt = R0 e − λt l
The figure show binding energy per nucleon versus mass
dt number. The nuclides showing binding energy per nucleon
where, R0 = λN 0 = initial value of activity. greater than 7.5 MeV/nucleon are stable.
Units of activity are 10
Binding energy per nucleon (MeV)

32 56 100
S Fe Mo
1 becquerel = Bq = 1 disintegration per second (SI unit)
16 12
l O I
12
8 C
l
1 curie = 1 Ci = 3 . 7 × 10 10 Bq 4
18
O 184
W 197
He 14 An 238
U
l
1 rutherford = 1 Rd = 10 6 Bq 6
N

6
Li

Mass Energy Relation 4


3
H
In nuclear physics, mass is measured in unified atomic mass 2
units (u), 1 u being one-twelfth of the mass of carbon-12 atom 2
H
and equals 1.66 × 10–27 kg. It can readily be shown using 0
0 50 100 150 200 250
E = mc2 that, 1 u mass has energy 931.5 MeV Mass number (A)
Binding energy per nucleon versus mass number variation
Thus, 1 u = 931.5 MeV or c 2 = 931.5 MeV/u
A unit of energy may therefore be considered to be a unit of
NOTE • Nucleons attract each other when they are separated by a
mass. For example, the electron has a rest mass of about
distance of 10 –14 m.
0.5 MeV.
• The density of nucleus is of the order of 10 17 .
DAY THIRTY FOUR RADIOACTIVITY 359

Nuclear Fission Reproduction Factor


Nuclear fission is the process of splitting of a heavy nucleus Reproduction factor (k ) of a nuclear chain reaction is defined
(235 239
92 U or 94 Pu) into two lighter nuclei of comparable as
masses along with the release of a large amount of energy Rate of production of neutrons
k =
− 200 MeV ) after bombardment by slow neutrons.
(~ Rate of loss / Absorption of neutrons
A characteristic nuclear fission reaction equation for 235
92 U is l
If k = 1, then the chain reaction will be steady and the
1
+ 235
→ 236
→ 144
+ 89
+3 1
+Q reactor is said to be critical.
0 n(slow ) 92 U 92 U 56 Ba 36 Kr 0n
l
If k > 1, then the chain reaction is accelerated and it may
In the fission of uranium, the percentage of mass converted
cause explosion in the reactor. Such a reactor is called
into energy is about 0.1% .
super-critical.
If k < 1, then chain reaction gradually slows down and
Controlled Chain Reaction l

comes to a halt. Such a reactor is called sub-critical.


and Nuclear Reactor The reactors giving fresh nuclear fuel which often exceeds the
In the fission of one nucleus of 235 nuclear fuel used is known as breeder reactor.
92 U, on an average,
l

1
2 neutrons are released. These released neutrons may
2
further, trigger more fissions causing more neutrons being
Nuclear Fusion
formed, which in turn may cause more fission. Thus, a self Nuclear fusion is the process, in which two or more light
sustained nuclear chain reaction is formed. To maintain the nuclei combine to form a single large nucleus.
nuclear chain reaction at a steady (sustained) level, the extra The mass of the single nucleus, so formed is less than the sum
neutrons produced, are absorbed by suitable neutron of the masses of parent nuclei and this difference in mass,
absorbents like cadmium or boron. results in the release of tremendously large amount of energy.
l
Neutrons formed as a result of fission have an energy of The fusion reaction going on in the central core of sun is a
about 2 MeV, whereas for causing further fission, we need multistep process, but the net reaction is
slow thermal neutrons having an energy of about 0.3 eV.
4 11 H + 2 ε – → 4
2 He + 2 ν + 6γ + 26.7 MeV
For this purpose, suitable material called a moderator is
used, which slow down the neutrons. Water, heavy water When two positively charged particles (protons or deuterons)
and graphite are commonly used as moderators. combine to form a larger nucleus, the process is hindered by
l
A nuclear reactor is an arrangement in which nuclear the Coulombian repulsion between them.
fission can be carried out through a sustained and a To overcome the Coulombian repulsion, the charged particles
controlled chain reaction and can be employed for are to be given an energy of atleast 400 keV.
producing electrical power, for producing different isotopes For this, proton/deuterons must be heated to a temperature of
and for various other uses. about 3 × 10 9 K.
nE
l
Power of a reactor, P = , where n = number of atoms Nuclear fusion reaction is therefore, known as thermo nuclear
t fusion reaction.
undergoes fission in time t seconds and E = energy released
in each fission.

DAY PRACTICE SESSION 1

FOUNDATION QUESTIONS EXERCISE


1 An electron of 1.02 MeV and a positron of 1.02 MeV 2 When a nucleus in an atom undergoes a radioactive
collide and annihilate into energy producing two decay, the electronic energy levels of the atom
γ-photons. The energy of each γ-photon will be (a) do not change for any type of radioactivity
(a) 1.02 MeV (b) 2.04 MeV (b) change for α and β-radioactivity, but not for γ-radioactivity
(c) 0.51 MeV (d) 1.53 MeV (c) change for α-radioactivity, but not for others
(d) change for β-radioactivity, but not for others
360 40 DAYS ~ NEET PHYSICS DAY THIRTY FOUR

3 A radon nucleus of mass 3.6 × 10 −25 kg


86Rn
222
12 A radio isotope X with a half-life 1.4 × 109 yr decays to Y
undergoes α-decay. α-particle has mass 6.7 × 10 −27 kg which is stable. A sample of the rock from a cave was
and energy 8.8 × 10 −13 J. The resulting nucleus is found to contain X and Y in the ratio 1 : 7. The age of the
(a) 84 Sr
220
(b) 84 Po 218 rock is j CBSE AIPMT 2014

220
(c) 84 Sn (d) None of these . × 109 yr
(a) 196 (b) 3.92 × 109 yr
4 A nucleus X emits one α-particle and two β-particles.
m (c) 4.20 × 109 yr (d) 8.40 × 109 yr
n
The resulting nucleus is j CBSE AIPMT 2011
13 The half-life of a radioactive isotope X is 20 yr. It decays to
(a) m −n6 Z (b) m −n4 X (c) m −4
n −2
Y (d) m −6
n −4
Z another element Y which is stable. The two elements X and
5 The number of beta particles emitted by a radioactive Y were found to be in the ratio 1 : 7 in a sample of a given
substance is twice the number of alpha particles emitted rock. The age of the rock is estimated to be j NEET 2013

by it. The resulting daughter is an j CBSE AIPMT 2009 (a) 40 yr (b) 60 yr (c) 80 yr (d) 100 yr
(a) isobar of parent (b) isomer of parent 14 If the half-life of any sample of radioactive substance is
(c) isotone of parent (d) isotope of parent 4 days, then the fraction of sample will remain
6 Radioactive material A has decay constant 8λ and undecayed after 2 days, will be
material B has decay constant λ. Initially, they have 1 2 −1 1
(a) 2 (b) (c) (d)
same number of nuclei. After what time, the ratio of 2 2 2
1
number of nuclei of material B to that A will be ? 15 The half-life of a radioactive substance is 30 min. The
e
time (in minutes) taken between 40% decay and 85%
j NEET 2017
decay of the same radioactive substance is j NEET 2016
1 1 1 1
(a) (b) (c) (d) (a) 15 (b) 30 (c) 45 (d) 60
λ 7λ 8λ 9λ
16 For a radioactive material, half-life is 10 min. If initially
7 Samples of two radioactive nuclides A and B are taken.
there are 600 number of nuclei, the time taken (in min) for
λ A and λ B are the disintegration constants of A and B,
the disintegration of 450 nuclei is j
NEET 2018
respectively. In which of the following cases, the two
samples can simultaneously have the same decay rate at (a) 30 (b) 10 (c) 20 (d) 15
any time? 17 If a radioactive substance decays for time interval equal
(a) Initial rate of decay of A is twice the initial rate of decay to its mean life, then the fraction of the substance
of B and λ A = λB remaining undecayed, will be
(b) Initial rate of decay of A is twice the initial rate of decay 1 1
(a) (b) (c) e 2 (d) e
of B and λ A > λB e e2
(c) Initial rate of decay of B is twice the initial rate of decay 18 A neutron causes fission in 235
producing 40 Zr 97 and
92 U
of A and λ A > λB 134
Te ; and some neutrons. The atomic number of Te will
(d) Initial rate of decay of B is always same
be
8 The half-life of a radioactive material is 3 h. If the initial (a) 50 (b) 51 (c) 52 (d) 53
amount is 300 g, then after 18 h, it will remain 235
19 If 200 MeV energy is obtained per fission of , then
92 U
(a) 4.68 g (b) 46.8 g the number of fissions per second to produce 1kW
(c) 9.375 g (d) 93.75 g
power will be
9 The half-life of a radioactive isotope X is 50 yr. It decays (a) 1.25 × 1018 (b) 3.2 × 10−8
to another element Y which is stable. The two elements X (c) 3.125 × 1013 (d) 0.125 × 1013
and Y were found to be in the ratio of 1 : 15 in a sample
20 On fission of one nucleus of U 235 , the amount of energy
of a given rock. The age of the rock was estimated to be
obtained is 200 MeV. The power obtained in a reactor is
j
CBSE AIPMT 2011
1000 kW. Number of nuclei fissioned per second in the
(a) 200 yr (b) 250 yr (c) 100 yr (d) 150 yr reactor, is
10 The half-life of radioactive element is 600 yr. The fraction (a) 9.4 × 1016 s −1 (b) 2.3 × 108 s −1
of sample that would remain after 3000 yr, is (c) 3.125 × 1016 s −1 (d) 4.2 × 108 s −1
(a) 1 / 2 (b) 1 / 16 (c) 1/8 (d) 1/32
21 1014 fissions per second are taking place in a nuclear
11 A radioactive sample has an initial activity of 50 dpm, reactor having efficiency 40%. The energy released per
20 min later, the activity is 25 dpm. How many atoms of fission is 250 MeV. The power output of the reactor is
the radioactive nuclide were there originally? (a) 2000 W (b) 4000 W
(c) 1600 W (d) 3200 W
(a) 20 (b) 80 (c) 1443 (d) 5441
DAY THIRTY FOUR RADIOACTIVITY 361

22 Fusion reaction takes place at high temperature, (a) neutrons are heavier than protons
because j CBSE AIPMT 2011 (b) electrostatic force between protons is repulsive
(a) atoms get ionised at high temperature (c) neutrons decay into protons through β -decay
(b) kinetic energy is high enough to overcome the (d) nuclear forces between neutrons is weaker than that
Coulomb repulsion between nuclei between protons
(c) molecules break up at high temperature 29 If all the atoms of 1kg of deuterium undergo nuclear
(d) nuclei break up at high temperature fusion, then the amount of energy released will be
23 If mass of proton = 1.008 amu and mass of (a) 6 × 10 27 cal (b) 2 × 107 kWh
neutron = 1.009 amu, then the binding energy per (c) 56.9 × 1013 J (d) 8 × 1023 MeV
nucleon for 4 Be9 (mass = 9.012 amu) will be 30 The mass of uranium required per day to generate 1MW
235
(a) 0.0672 MeV (b) 0.672 MeV power from the fission of 92 U , will be
(c) 6.72 MeV (d) 67.2 MeV (a) 1.05 g (b) 2.05 g (c) 3.05 g (d) 4.05 g
40 7
24 The mass of 18 Ar is 39.9480 amu. Its mass defect will 31 The binding energy per nucleon of and 42 He nuclei
3 Li
be (Take, mp =1.0078 amu and mn =1.0087 amu) are 5.60 MeV and 7.06 MeV, respectively. In the nuclear
(a) 0.3694 amu (b) 0.3318 amu reaction 73 Li + 11H → 42He + 42He + Q, the value of energy
(c) 0.3480 amu (d) 0.3838 amu Q released is j CBSE AIPMT 2014

25 The binding energy of neutron in deuterium 1H2 will be (a) 19.6 MeV (b) −2.4 MeV (c) 8.4 MeV (d) 17.3 MeV
(Take, mp = 1.0078 amu, mn = 1.0087 amu and 32 A certain mass of hydrogen is changed to helium by the
md = 2.0141 amu) process of fusion. The mass defect in fusion reaction is
(a) 2.2344 MeV (b) 4.4688 MeV 0.02866 u. The energy liberated per u, is
(c) 1.1172 MeV (d) 7.8 MeV
(Take, 1u = 931 MeV) j
NEET 2013
26 The energy released in the following β-decay process (a) 2.67 MeV (b) 26.7 MeV
will be 0n
1
→ 1p1 + −1q
0
+ν (c) 6.675 MeV (d) 13.35 MeV
−27
Take, mn = 1.6747 × 10 kg, 33 Consider the nuclear reaction X 200 → A110 + B 80. If
the binding energy per nucleon for X, A and B are
mp = 1.6725 × 10 −27 kg and me = 0.00091 × 10 −27 kg
7.4 MeV, 8.2 MeV and 8.1 MeV, respectively. Then, the
(a) 0.931 MeV (b) 0.731 MeV
energy released in the reaction is
(c) 0.511 MeV (d) 0.271 MeV
(a) 70 MeV (b) 200 MeV (c) 190 MeV (d) 10 MeV
27 If the mass defect in a fusion process is 0.3%, then the 7
energy released in the fusion of 1kg of material will be
34 The mass of a 3 Li
nucleus is 0.042u less than the sum of
−14 the masses of all its nucleons. The binding energy per
(a) 2.7 × 10 J
14
(b) 2.7 × 10 J
nucleon of 73 Li nucleus is nearly j
CBSE AIPMT 2010
(c) 2.7 × 1014 erg (d) 2.7 × 1014 eV
(a) 46 MeV (b) 5.6 MeV
28 Heavy stable nuclei have more neutrons than protons. (c) 3.9 MeV (d) 23 MeV
This is because of the fact, that

DAY PRACTICE SESSION 2

PROGRESSIVE QUESTIONS EXERCISE


1 The activity of a radioactive sample is measured as 3 If 10% of a radioactive substance decays in every 5 yr,
9750 counts/min at t = 0 and as 975 counts/min at then the percentage of the substance that will have
t = 5 min. The decay constant is approximately decayed in 20 yr, will be
(a) 0.922/min (b) 0.691/min (c) 0.461/min (d) 0.230/min (a) 40% (b) 50% (c) 65.6% (d) 34.4%
2 A mixture consists of two radioactive materials A 1 and 4 The activity of a radioactive sample is measured as Ne
A 2 with half-lives of 20 s and 10 s, respectively. Initially, counts per minute at t = 0 and N0 / e counts per minute at
the mixture has 40 g of A 1 and 160 g of A 2 . The amount t = 5 min. The time (in minute) at which the activity
of the two in the mixture will become equal after reduces to half its value, is j
CBSE AIPMT 2010
j
CBSE AIPMT 2012 5
(a) loge 2 / 5 (b) (c) 5 log10 2 (d) 5 loge 2
(a) 60 s (b) 80 s (c) 20 s (d) 40 s loge 2
362 40 DAYS ~ NEET PHYSICS DAY THIRTY FOUR

5 Half-life of a radioactive substance is 20 min. The time 11 In the nuclear decay given below
between 20% and 80% decay will be A−4 * A−4
A
Z X → A
Z + 1Y → Z − 1B → Z − 1 B,
(a) 20 min (b) 40 min (c) 80 min (d) 60 min
the particles emitted in the sequence are
6 The number N of nuclei of a radioactive element X , at j CBSE AIPMT 2009
time t, if at time t = 0 , the element has N0 number of
(a) β, α, γ (b) γ, β, α
nuclei. Nuclei of the element X is being produced at a
(c) β, γ, α (d) α, β, γ
constant rate α and the element has a decay constant λ, is
α λ 12 In the reaction 21H + 31 H → 42 He+ 01n, if the binding
(a) N = (1 − e − λ t ) + N 0 e − λ t (b) N = (1 − e − λ t ) + N 0 e − λ t
λ α energies of 21H, 31 H and 42 He are respectively a, b and c (in
α λ
(c) N = ( e − λ t − e) + N 0 (d) N = (e − λ t − e) + N 0 MeV), then the energy (in MeV) released in this reaction is
λ α j CBSE AIPMT 2005
7 The binding energy of deuteron is 2.2 MeV and that of (a) c + a − b (b) c − a − b
4
is 28 MeV. If two deuterons are fused to form one (c) a + b + c (d) a + b − c
2 He,
4 13 The mass of proton is 1.0073 u and that of neutron is
2 He, then the energy released is
(a) 25.8 MeV (b) 23.6 MeV (c) 19.2 MeV (d) 30.2 MeV 1.0087 u (u = atomic mass unit). The binding energy of
2 He is (mass of helium nucleus = 4.0015 u)
4
8 A nuclear explosive is designed to deliver 1 MW power in
the form of heat energy. If the explosion is designed with a
j CBSE AIPMT 2003
nuclear fuel consisting of U235 to run a reactor at this (a) 28.4 MeV (b) 0.061 u
power level for one year, then the amount of fuel needed (c) 0.0305 J (d) 0.0305 erg
is (Take, energy per fission is 200 MeV) 14 The variation of decay rate of two radioactive samples A
(a) 1 kg (b) 0.01 kg
and B with time is shown in figure.
(c) 3.84 kg (d) 0.384 kg
235
9 A uranium reactor 92 U takes 30 days to use up 2 kg of
fuel, each fission gives 185 MeV of usable energy, then dN
dt
power output is
(a) 32.23 MW B
(b) 22.28 MW
(c) 58.46 MW A
(d) None of these t

10 A nucleus of uranium decays at rest into nuclei of thorium Which of the following statement(s) is/are true?
and helium. Then, j
CBSE AIPMT 2015
(a) Decay constant of A is greater than that of B, hence A
(a) the helium nucleus has more kinetic energy than the
always decays faster than B
thorium nucleus
(b) Decay constant of B is greater than that of A, but its
(b) the helium nucleus has less momentum than the thorium
decay rate is always smaller than that of A
nucleus
(c) Decay constant of A is greater than that of B, but it
(c) the helium nucleus has more momentum than the
does not always decay faster than B
thorium nucleus
(d) Decay constant of B is same as that of A, but still its
(d) the helium nucleus has less kinetic energy than the
thorium nucleus decay rate becomes equal to that of A at a later instant

ANSWERS
SESSION 1 1 (d) 2 (b) 3 (b) 4 (b) 5 (d) 6 (b) 7 (b) 8 (a) 9 (a) 10 (d)
11 (c) 12 (c) 13 (b) 14 (b) 15 (d) 16 (c) 17 (a) 18 (c) 19 (c) 20 (c)
21 (c) 22 (b) 23 (c) 24 (d) 25 (c) 26 (b) 27 (a) 28 (b) 29 (c) 30 (a)
31 (d) 32 (c) 33 (a) 34 (b)
SESSION 2 1 (c) 2 (d) 3 (d) 4 (d) 5 (b) 6 (a) 7 (b) 8 (d) 9 (c) 10 (a)
11 (a) 12 (b) 13 (a) 14 (c)
DAY THIRTY FOUR RADIOACTIVITY 363

Hints and Explanations


SESSION 1 n
N 1 = 0.6 N 0
N = N 0 =  
1 15 Given, (Q40% decay)
1 Total energy produced in the  2 N 2 = 0.15 N 0 (Q85% decay)
interaction, Given, N 0 = 300 g Putting these in the formula,
7E = 1.02 + 1.02 + 0.51 × 2 N2 0.15 N 0 1  1
2
6
= = =  
∴ N = 300   = 300 ×
= 3.06 MeV 1 1
= 4.68 g N1 0.6 N 0 4  2
∴Energy of each photon  2 64
3.06 So, two half-life periods has passed.
= = 1.53 MeV T1 / 2
Thus, time taken = 2 × T1 /2 = 2 × 30
2 9 We know that, N =  1 
N 0  2 = 60 min
2 As an α-particle carries two units of t/ 50
16 ∴ Number of nuclei left undecayed,
=  
positive charge, a β -particle carries one 1 1
⇒ ⇒ t = 4 × 50
unit of negative charge and γ -particle 16  2  N = N 0 − N ′ = 600 − 450 = 150
Half-life, T1 /2 = 10 min
carries no charge, therefore electronic t = 200 yr T1 / 2
energy levels of the atom change for α
=  
N 1
As,
and β-decay, but not for γ-decay. 10 We know that, n = 1 N 0  2
T
3 A = 222 − 4 = 218 and Z = 86 − 2 = 84 Substituting the given values, we get
Given, t = 3000 yr, t/10 t/10
At Z = 84 there is polonium. 150  1  1  1
T = 600 yr ⇒ n =
3000
=5 =  or = 
α m−4 2β m−4 600 600  2  4  2
4 m
n X → n −2X → nX 2 t/10
or   =  
n 1 1 t
=   =2
A N 1 or
5 Let the radioactive substance be X . Then,  2  2 10
Z
N 0  2
Radioactive transition is given by
5
⇒ t = 20 min
=   =
−α A −4 − 2β A −4 N 1 1
X → → 17 N = N 0 e − λt , if t = 1 = T m ,
A
X X
Z Z −2 Z N 0  2 32 λ
The atoms of element having same − λ⋅
1
atomic number, but different mass 11 Rate decreases from 50 to 25 dpm in then
N
=e λ =
1
numbers are called isotopes. 20 min, hence half-life is 20 min. N0 e
0.693
So, A
X and A −4
X are isotopes. r 0 = λN 0 = × N0 18 235
→ 40 Zr 97 + Te134 + x 0 n1
Z Z t 1 /2 92 U
According to the law of conservation of
6 Let initial number of nuclei in A and B r 0 × t 1 /2 50 × 20
⇒ N0 = = = 1443 charge,
is N 0 . 0.693 0.693 92 = 40 + Z or Z = 52
Number of nuclei of A after time t is
N A = N 0e −8λt ...(i)
12 X : Y = 1 : 7 19 Number of fissions per second
X :( X + Y ) = 1 : 8 = 1 : 23 ⇒ 3 half-life Required rate of emission of energy
Similarly, number of nuclei of B after =
Energy emitted per fission
time t is ∴ ∆T = 3 × 1.4 × 10 yr 9

N B = N 0e − λt 103
...(ii) = 4.20 × 109 yr =
200 × 10 × 1.6 × 10−19
6
It is given that, n
=  
N 1 = 3.125 × 1013 fission/s
NA
=
1
[QN B > N A ]
13 As,
N 0  2
NB e 20 Power received = 1000 kW
3
=   =
N 1 1
Now, from Eqs. (i) and (ii), we get As, = 1000 × 1000 W = 106 Js −1
e −8λt 1 N 0  2 8
= 1 MeV = 1.6 × 10−13 J
e − λt e Number of half-lives = 3 ⇒ T = 20 yr
106
Rearranging ∴ T =
t ∴P = = 6.25 × 1018 MeVs −1
1.6 × 10−13
⇒ e −1 = e −7λt ⇒ 7λt = 1 n
1 or t = T × n = 20 × 3 = 60 yr Number of nuclei fissioned (per second)
⇒ Time, t = 6.25 × 1018
7λ 14 From Rutherford-Soddy’s law, the is = = 3.125 ×1016s −1
fraction left after n half-lives is 200
7 The two samples of two radioactive n
N = N 0  
nuclides A and B can simultaneously 1 21 Total energy released per second
have the same decay rate at any time, if  2
= 250 × 1.6 × 10−13 × 1014 = 4000 J
initial rate of decay of A is twice the where, n is number of half-lives.
initial rate of decay of B and λ A > λ B . Time ( t ) 2 1 40
n= = = Power output = 4000 × = 1600 W
Half - life (T1 /2 ) 4 2 100
8 Number of half-lives, n = t = 18 = 6
T 3 1 /2 22 Fusion reaction takes place at high
=  
N 1 N 1
Amount remained after n half-lives, ∴ ⇒ = temperature, because kinetic energy is
N 0  2 N0 2
high enough to overcome the Coulomb
repulsion between nuclei.
364 40 DAYS ~ NEET PHYSICS DAY THIRTY FOUR

23 Mass defect, 33 For X, energy = 200 × 7.4 = 1480 MeV The fraction of material decayed
∆m = (4 × 1.008 + 5 × 1.009) − 9.012 For A, energy = 110 × 8.2 = 902 MeV = 1 − 0.656 = 0.344 = 34.4%
= 4.032 + 5.045 − 9.012 For B, energy = 80 × 81
. = 648 MeV
= 9.077 − 9.012 = 0.065amu 4 Fraction remains after n half-lives,
Therefore, energy released n t/T
BE 0.065 × 931
=   =  
= (902 + 648) − 1480 N 1 1
∴ = = 6.72 MeV
A 9 = 1550 − 1480 = 70 MeV N 0  2  2
24 ∆m = Zm p + ( A − Z )m n − M (Z , A ) N
34 If m = 1 u, c = 3 × 108 ms −1 , then Given, N = 0
e
= 18 × 1.0078 + (40 − 18) E = 931 MeV, i.e. 1 u = 931 MeV 5/ T
× 1.0087 − 39.9480 =  
N0 1
Binding energy = 0.042 × 931 ⇒
= 18.1404 + 22.1914 − 39.9480 eN 0  2 
= 3910
. MeV
= 0.3838 amu 5/ T
∴ Binding energy per nucleon 1  1
25 BE = {(m p + m n ) − md } × 931 MeV ⇒ = 
=
3910
.
= 558
. ≈ 56. MeV e  2
= {(1.0078 + 1.0087) − 2.014} × 931 7 Taking log on both sides, we get
= 2.2344 MeV 5 1
SESSION 2 log 1 − log e = log
BE T 2
∴ = 1.1172 MeV
A 1 According to law of radioactivity, 5
⇒ − 1 = (− log 2) ⇒ T = 5log e 2
N T
26 Mass defect, = e − λt …(i)
N0 Now, let t be the time after which acitvity
∆m = (1.6747 −1.6725 − 0.0091) × 10−27
N0 reduces to half
= 0.0012 × 10−27 kg ⇒ = e λt t ′ /5 log e 2
N  1 =  1 ⇒ t ′ = 5 log e 2
0.0012 × 10−27 × (3 × 108 )2    
∴ ∆E = On taking logarithm both sides of Eq.  2  2
−13
1.6 × 10 (i), we get
5 Half-life of a radioactive substance is 20
= 0.731 MeV N 
log e  0  = log e (e λt ) min, i.e. t 1 /2 = 20 min
 N 
27 ∴∆ m = 0.3 × 1 = 0.003 kg For 20% decay, we have 80% of the
100 = λ t log e e = λ t substance left,
∆E = ∆mc 2 As we know that, log e x = 2.3026 80N 0
log 10 x = N 0e −λt20 ...(i)
⇒ = 0.003 × (3 × 108 ) 2 100
Making substitution, we get
= 2.7 × 1014 J where, N 0 = initial undecayed
2.3026 log 10 
9750 
 substance and t 20 is the time taken for
28 Heavy nuclei, which are stable contain  975 
λ= 20% of the substance left, hence
more neutrons than protons in their 5 20N 0
nuclei. This is because, electrostatic 2.3026 2.3026 = N 0 e −λ t 80 ...(ii)
= log 10 10 = min −1 100
force between protons is repulsive, 5 5
which may reduce stability. On dividing Eq. (i) by Eq. (ii), we get
= 0.461 min −1
29 Fusion reaction of deuterium, 4 = e λ( t 80 − t20 )

2 For 40 g amount,
1H
2
+ 1 H2 → 2 He 4 + 23.6 MeV ln 4 = λ(t 80 − t 20 )
20 s 20 s
Energy released in 1 kg of deuterium 40 g → 20 g → 10 g 2 ln 2 =
0.693
( t 80 − t 20 )
half - life
6.02 × 1023 × 103 × 23.6 × 1.6 × 10−13 t 1 /2
= For 160 g amont,
2× 2 ⇒ t 80 − t 20 = 2 × T1 /2 = 40 min
10 s 10 s
= 56.9 × 1013 J 160 g → 80 g → 40 g
dN
30 Energy per day = 106 × 24 × 60 × 60 J 6∴ = α − λN
10s
→ 20 g → 10 g
10s dt
Energy received per fission = 200 MeV
= 3.2 × 10−11 J ⇒
dN
= dt
Mass of U235 per day So, after 40 s, A 1 and A 2 remain same. α − λN
235 × 1.67 × 10−27 × 106 × 24 N  On integrating both sides,
× 60 × 60 T log 10  0 
=  N  log e (α − λN )
3.2 × 10−11 3∴ t = =t + A
log 10 2 −λ
= 1.05 × 10−3 kg = 1.05 g
N  where, A is integration constant.
or t ∝ log 10  0 
31 Q = 2 (BE of He) − (BE of Li)  N  At t = 0, N = N 0
= 2 × (4 × 7.06) − (7 × 5.60) log e (α − λN 0 )
log 
100  = A
= 56.48 − 39.2 = 17.3 MeV  −λ
5  90 
∴ =
32 Here, ∆m = 0.02866 u 20 N  Equation becomes,
log  0 
∴ Energy liberated  N  log e (α − λN ) log e (α − λN 0 )
=t +
0.02866 × 931 26 .7 −λ −λ
= = MeV 4
=   = 0.656
N 9
4 4 or ⇒ log e (α − λN ) − log(α − λN 0 ) = − λt
N 0  10 
= 6 . 675 MeV
DAY THIRTY FOUR RADIOACTIVITY 365

A −4
 α − λN  9 Number of atoms in 2 kg fuel (U235 ) is A
Y → β + α24
log e   = − λt Z +1 Z −1
 α − λN 0  6.02 × 1026 A−4
β → A −4
β + γ 00
= × 2 = 5.12 × 1024 Z +1 Z −1
 α − λN  − λt 235
⇒  =e The nuclei of these atoms are fissioned
 α − λN 0  12 Given, binding energy of
− λt − λt
in 30 days. (21 H + 31 H) = a + b
λN = α(1 − e ) + λN 0 e
Therefore, fission rate is
Binding energy of 42 He = c
α
N = (1 − e − λt ) + N 0 e − λt 5.12 × 1024
λ = = 1.975 × 1018s −1 In a nuclear reaction, the resultant
(30 × 24 × 60 × 60)
7 The reaction can be written as nucleus is more stable than the
Each fission gives 185 MeV of energy.
reactants. Hence, binding energy of 42 He
1H
2
+ 1 H2 → 2 He 4 + energy Hence, energy obtained in 1,
The energy released in the reaction, is i.e. power output is will be more than that of (21 H + 31 H) .
difference of binding energies of P = 185MeV × (1.975 × 1018 ) Thus, energy released per nucleon
daughter and parent nuclei. −1 = resultant binding energy
= 365.4 × 10 MeVs
18
Hence, energy released = binding energy
Binding energy of product − Binding
of 2 He 4 − 2 × binding energy of 1 H2 But 1MeV = 1.6 × 10−13 J
energy of reactants
= 28 − 2 × 2.2 = 23.6 MeV ∴ P = (365.4 × 1018 MeVs −1 ) = c − (a + b ) = c − a − b
8 Energy released per fission is × (1.6 × 10−13 J / MeV) 13 2 He 4 contains 2 neutrons and
E = 200 MeV = 58.46 × 106 Js −1 2 protons.
−13 −11
So, mass of 2 protons
= 200 × 1.6 × 10 = 3.2 × 10 J = 58.46 × 10 W = 58.46 MW
6
= 2 × 1.0073 = 2 .0146 u
Number of fissions to produce 1MW 10 92 U
238
→ 92 Th
238
+ 2 He 4 So, mass of 2 neutrons
(= 106 W = 106Js −1 ) power is = 2 × 1.0087
According to law of conservation of
106 Js −1 linear momentum, we have = 2 .0174 u
=
3.2 × 10−11 J |PTh | = |PHe| = P Total mass of 2 protons and 2 neutrons
= 3.125 × 1016s −1 ⇒ As, kinetic energy of an element, = (2 .0146 + 2 .0174) u = 4.032 u
P2 Mass of helium nucleus = 4.0015 u
Total energy required to run a 1 MW KE =
reactor for one year, is 106Js −1 2m Thus, mass defect is lacking of mass in
× (365 × 24 × 60 × 60) = 315
. × 1013 J where, m is mass of an element. forming the helium nucleus from 2
1 protons and 2 neutrons.
Since, 1 fission (1 atom of U 235) Thus, KE ∝
M ∴ ∆m = mass defect
produces 3 . 2 × 10−11 J of energy, total
So, M He < M Th = (4.032 − 4.0015) u
number of U 235 atoms required is ⇒ K He > K Th = 0.0305 u
3.15 × 1013 So, helium nucleus has more kinetic
= = 9.84 × 1023 As we know that, 1u = 931 MeV
3 . 2 × 10−11 energy than the thorium nucleus.
Hence, binding energy
Now, 6. 02 × 1023 atoms of U 235 are 11 Alpha particles are positively charged ∆E = (∆m ) × 931 = 0.0305 × 931
particles with charge +2e and mass 4
contained in 235 g of U 235 = 28.4 MeV
m. Emission of an α-particle reduces the
∴ Mass of U 235
containing mass of the radionuclide by 4 and its 14 From the given figure, we find that rate
9.84 × 1023 atoms is atomic number by 2. β-particles are of decay of A is faster than that of B. It
235 negatively charged particles with rest means, decay constant of A is greater
M = mass as well as charge same as that of than that of B. However, the two curves
6.02 × 10 23 atoms electrons. γ -particles carry no charge intersect at P, beyond P, B decays faster
× 9.84 × 1023 atoms and mass. than A. And at P, decay rate of both A
= 384 g = 0.384 kg Radioactive transition will be as follows and B is the same.
Z X → Z + 1 Y + β −1
A A 0
DAY THIRTY FIVE

Unit Test 7
(Dual Nature of Matter,
Atoms and Nuclei)
1 The de-Broglie wavelength associated with a proton 8 The electric potential at the surface of an atomic nucleus
1 ( Z = 50) of radius 9.0 × 10−13 cm, is
moving with a velocity, equal to th of the velocity of
20 (a) 9 × 105 V (b) 8 × 106 V (c) 80 V (d) 9 V
light 9 K α wavelength emitted by an atom of atomic number
(a) 2.75 × 10−14 m (b) 1.5 × 10−7 m Z = 11 is λ. Find the atomic number for an atom that
(c) 1.5 × 107 m (d) 2.634 × 1014 m emits K α radiation with wavelength 4λ, is
2 An electron has been accelerated from rest through a (a) Z = 6 (b) Z = 4 (c) Z = 11 (d) Z = 44
potential difference of 100 V, its de-Broglie wavelength is 10 The ratio of minimum wavelengths of Lyman and Balmer
(a) 225 Å (b) 1.225 Å (c) 225 m (d) 1.225 cm series will be
3 The experiments of Frank and Hertz showed that (a) 1.25 (b) 0.25 (c) 5 (d) 10
(a) an atom has energy states having a continuous
11 What will be the amount of energy absorbed when an
distribution
electron jumps from first orbit to second orbits, if the
(b) nothing can be send a bond energy states of atom
value of energy in nth orbit of H-atom is expressed as
(c) at atom has energy states, having discrete values
 13.6 
(d) atom is an indivisible particle E n = − 2 eV ?
 n 
4 A photon and an electron have same energy, the ratio of
(a) − 3.4 eV (b) −6.6 eV
their wavelengths is (c) −8.1 eV (d) −10. 2 eV
m E 2mc 2 mc
(a) (b) (c) (d) 12 An electron changes its position from orbit n = 2 to the
E m E E2
orbit n = 4 of an atom. The wavelength of the emitted
5 If the wavelength of incident light changes from 400 nm radiations is (R = Rydberg’s constant)
to 300 nm, the stopping potential for photoelectrons 16 16
(a) (b)
emitted from a surface becomes approximately R 3R
(a) 1.0 V greater (b) 1.0 V smaller 16 16
(c) (d)
(c) 0.5 V greater (d) 0.5 V smaller 5R 7R
6 The photosensitive surface is receiving light of 13 Imagine an atom made of a proton and a hypothetical
wavelength 5000 Å at the rate of 10 −8 Js −1. The number particle of double the mass of the electron but having same
of photons received per second, is charge as the electron. Apply Bohr atom model and
(a) 2 .5 × 1010 (b) 2 .5 × 1011 consider all possible transitions of the hypothetical particle
(c) 2 .5 × 1012 (d) 2 .5 × 109 to the first excited level. The longest wavelength photon that
7 The wavelength of quantum of radiant energy emitted, if will be emitted has wavelength λ equal to
an electron transmitted into radiation and converted into 9 36
(a) (b)
one quantum, is 5R 5R
18 4
(a) 0.0242 Å (b) 2.42 Å (c) 532 Å (d) 0.532 Å (c) (d)
5R R
DAY THIRTY FIVE UNIT TEST 7 (DUAL NATURE OF MATTER, ATOMS AND NUCLEI) 367

14 lonisation potential of hydrogen atom is 13.6 eV. 24 C14 has half-life 5700 yr. At the end of 11400 yr, actual
Hydrogen atoms in ground state are excited by amount left is
monochromatic radiation of photon energy 12.1 eV. (a) 0.5 of original amount (b) 0.25 of original amount
According to Bohr’s theory, the spectral lines emitted by (c) 0.125 of original amount (d) 0.0625 of original amount
hydrogen will be 208 208
25 Ra has half-life 120 days. The amount of Ra, if the
(a) 2 (b) 3 (c) 4 (d) 1 activity level is one mCi
15 In Bohr’s model the atomic radius of the first orbit is r0, (a) 0.184 µg (b) 0.184 mg (c) 0.134 µg (d) 0.314 mg
the radius of the third orbit will be
26 The counting rate observed from a radioactive source at
r0
(a) 9r0 (b) 3 r0 (c) r0 (d) t = 0 s was 1600 count per second, and at t = 8 s, it was
3
100 count per second. The counting rate observed at
16 Taking Rydberg’s constant RH = 1.097 × 107 m first and t = 6 s was
second wavelength of Balmer series in hydrogen
(a) 400 (b) 300 (c) 200 (d) 150
spectrum, are
27 Mean life of a radioactive sample is 100 s. Then, its
(a) 2000 Å, 3000 Å (b) 1575 Å, 2960 Å
half-life (in minute) is
(c) 6529 Å, 4280 Å (d) 6562 Å, 4863 Å
(a) 0.693 (b) 1 (c) 10−4 (d) 1.155
17 Which of the following lines of the H-atom spectrum
28 If parent decays to daughter nucleus with a rate r and
belongs to Balmer series?
daughter nucleus has average life τ and number of
(a) 1025 Å (b) 12184 Å (c) 4861 Å (d) 18751 Å nuclei of daughter nucleus at any instant is N, then for
18 The density of uranium is of the order of radioactive equilibrium to be achieved, we have
(a) 1020 kgm−3 (b) 1017 kgm−3 (c) 1014 kgm−3 (d) 1011 kgm−3 (a) r = Nτ (b) r = τN (c) r τ = N (d) τ = r
19 Two radioactive materials A and B have decay constants 29 In a specific case, angular momentum is an even integral
10λ and λ, respectively. If initially they have the same h
multiple of . The longest possible wavelength emitted
number of nuclei, then the ratio of the number of nuclei of 2π
A to that of B will be 1/e after a time. by hydrogen in visible region in such a world, according
1 1 11 1 to Bohr’s model is
(a) (b) (c) (d)
10λ 11λ 10λ 9λ (a) 273 nm (b) 470 nm (c) 523 nm (d) zero
20 In an experiment, a radioactive isotope is being 30 If the binding energies of a deuteron and an α-particle
dN are 1.125 MeV and 7.2 MeV, respectively. Then, the
produced at a constant rate =R. Its half life is t1/ 2 .
dt more stable of the two is
After t > > t1/ 2 , the number of active nuclei will become (a) deuteron
constant. The value of this constant is (b) α-particle
Rt1/ 2 t1/ 2 R 0.693 (c) Both (a) and (b)
(a) (b) (c) (d)
0.693 R t1/ 2 Rt1/ 2 (d) sometimes deuteron and sometimes alpha particle
31 The nuclei of which one of the following pairs of nuclei
21 If R is the radius and A is the mass number, then log R
are isotones?
versus log A graph will be 74 71 92 92
(a) 34 Se , 31Ga (b) 42 Mo , 40 Zr
(a) a straight line (b) a parabola 84 86 40 32
(c) 38 Sr , 38Sr (d) 20 Ca , 16S
(c) an ellipse (d) a hyperbola
22 Two radioactive materials X and Y have 3000 and 6000 32 In the nuclear reaction,
number of nuclei respectively at initial. If decay constant −α −β −α −γ
72 X
180
→ Y  
→ Z  → A  
→ P
of material X and Y are 12 minute −1 and 10 minute −1,
then the time, in which both elements have same number the atomic mass and atomic number of P are,
of nuceli: respectively
(a) loge  
1 1 2
(b) loge (a) 170, 69 (b) 172, 69 (c) 172, 70 (d) 170, 70
 2  2
1 1 33 A nuclei X with mass number A and charge number Z,
(c) loge (d) loge [2]
2 2 disintegrates into one α-particle and one β-particle. The
resulting nuclide R has atomic mass and atomic number,
23 A radioactive substance has a half-life of four months.
equal to
Three-fourth of the substance will decay in
(a) (A − Z ) and (Z − 1) (b) (A − Z ) and (Z − 2)
(a) 3 months (b) 4 months
(c) (A − 4) and (A − 2) (d) (A − 4) and (Z − 1)
(c) 8 months (d) 12 months
368 40 DAYS ~ NEET PHYSICS DAY THIRTY FIVE

+
6 → B 5 + β + X
34 In a nuclear reaction, C 11 11
40 Assertion (A) If high pressure is applied on a radioactive
What does X stand for? substance, rate of radioactivity will increase.
(a) An electron (b) A proton
Reason (R) Radioactivity is a nuclear process.
(c) A neutron (d) A neutrino
41 Assertion (A) At time t = 0, activity of a radioactive
35 The particle A is converted into C via following reaction, substance is 10 units. At t = 1 s, it remains 90 units. Then,
A → B + 2He , B → C + 2 e
4 − at t = 2 s it should remain 80 units.
Reason (R) In equal interval of time, percentage change
Then,
is same in a radioactive substance.
(a) A and C are isobars (b) A and C are isotopes
(c) A and B are isobars (d) A and B are isotopes 42 Assertion (A) If light continuously falls on a metal
36 A nucleus with mass number 220 initially emits α-particle. surface, then emission of electrons will stop after some
If the Q value of the reaction is 5.5 MeV. Then, the kinetic time.
energy of the α-particle is Reason (R) We cannot extract all the electrons of a
(a) 4.4 MeV (b) 5.4 MeV metal.
(c) 5.6 eV (d) 6.5 eV
43 The energy spectrum of β-particles number N(E ) as a
Direction (Q. Nos. 37-42) These questions consist of two function of β energy E emitted from a radioactive
statements, each linked as Assertion and Reason. While source, is
answering of these questions, you are required to choose any
one of the following.
N(E)
(a) If both Assertion and Reason are true and Reason is N(E)
the correct explanation of the Assertion (a) (b)
(b) If both Assertion and Reason are true but Reason is not
correct explanation of the Assertion
E E
(c) If Assertion is true but Reason is false
(d) If Assertion is false but Reason is true
37 Assertion (A) If wavelength of incident photons is N(E) N(E)
halved, then maximum kinetic energy of photoelectron
will become two times. (c) (d)
1
Reason (R) Energy of photon ∝ .
λ E E
38 Assertion (A) Second orbit circumference of hydrogen
44 If a star can convert all the He nuclei completely into
atom is two times the de-Broglie wavelength of electrons
oxygen nuclei, the energy released per oxygen nucleus
in that orbit.
is [Mass of He nucleus is 4.0026 amu and mass of
Reason (R) de-Broglie wavelength of electron in ground oxygen nucleus is 15.9994 amu]
state is minimum. (a) 7.6 MeV (b) 56.12 MeV
(c) 10.24 MeV (d) 23.9 MeV
39 Assertion (A) Energy E 1 is required to remove first
electron from helium atom and energy E 2 is to required 45 A nuclear transformation is denoted by X (n, α ) Li. 7
3
to remove the second electron. Then, E 1 < E 2 . Which of the following is the nucleus of element X ?
Reason (R) Ionisation energy of single electron of He + is (a) 11
4 Be (b) 95 B (c) 10
5 B (d) 12
6 C
54.4 eV.

ANSWERS
1 (a) 2 (b) 3 (c) 4 (c) 5 (a) 6 (a) 7 (a) 8 (b) 9 (a) 10 (b)
11 (d) 12 (b) 13 (b) 14 (b) 15 (a) 16 (d) 17 (c) 18 (a) 19 (d) 20 (a)
21 (a) 22 (c) 23 (c) 24 (b) 25 (a) 26 (c) 27 (d) 28 (c) 29 (b) 30 (b)
31 (a) 32 (b) 33 (d) 34 (d) 35 (b) 36 (b) 37 (d) 38 (b) 39 (b) 40 (d)
41 (d) 42 (c) 43 (d) 44 (c) 45 (c)
DAY THIRTY FIVE UNIT TEST 7 (DUAL NATURE OF MATTER, ATOMS AND NUCLEI) 369

Hints and Explanations


1 Velocity of proton, Energy received per second 12 Wavelength of lines emitted is given by
=
3 × 108 Energy of one photon 1  1 1 
v = = 1.5 × 107 m/s = R  − 
20 10−8 λ 2
 n1 n22 
= = 2.5 × 1010
m = 1.67 × 10−27 kg 3.98 × 10−19 where, R = Rydberg’s constant
λ=
h
7 When an energy of an electron is Given, n1 = 2, n2 = 4
= R 
mv transmitted into radiation, we have 1 1 1  3R 16
∴ −  = ⇒λ =
6.6 × 10−34 hc λ  22 42  16 3R
= E = mc 2 , E = hν =
1.6 × 10−27 × 1.5 × 107 λ − Rhc
hc
13 En = ⇒
= E3 − E2
= 2.75 × 10−14 m h 6.6 × 10−34 n2 λ
λ= =
mc 9.1 × 10−31 × 3 × 108
= Rhc  −  ⇒ λ m =
12.25 hc 1 1 36
2 λ= Å ⇒
V = 0.0242 Å λ  22 32  5R
V = 100 V 8 Electric potential at the surface of an 14 Ionisation energy corresponding to
12.25 atomic nucleus is ionisation potential = − 13.6 eV
λ= = 1.225 Å
100 1 Ze Photon energy incident = 12.1 eV
V =
4 π ε0 r So, the energy of electron in excited
3 The series of experiments by Frank and
Hertz in 1914 produced direct evidence 9 × 109 × 50 × 1.6 × 10−19 state = – 13.6 + 12.1 = – 1.5 eV
= 13.6 −13.6
for energy levels in atoms. In studying 9 × 10−15 En = − eV ⇒ −1.5 =
the motion of electron through mercury n2 n2
= 8 × 10 V 6
−13.6
vapour under the action of an electric ⇒ n2 = ≈ 9⇒n = 3
field, they observed that the spectrum 1 −1.5
9 ∝ (Z − 1)2
line at 254 nm was emitted by the λ ∴Energy of electron in excited state
vapour when the electrokinetic energy 2 2
λ  Z − 1 1  Z − 1 corresponds to third orbit.
was greater than 4.9 eV. But there was ⇒ 1 = 2  ⇒ = 2  The possible spectral lines are when
no spectrum line when kinetic energy λ 2  Z1 − 1  4  11 − 1 
electron jumps from orbit 3rd to 2nd,
was found less than 4.9 eV. Taking square root on both sides, we get 3rd to 1st and 2nd to 1st. Thus,
1 Z2 − 1
4 The de-Broglie wavelength of electron is = ⇒ Z2 = 6 3 spectral lines are emitted.
given by 2 11 − 1
15 Here, radius of first orbit = r0
h
λe = 10 The series end of Lyman series The radius of nth orbit is given by,
mv corresponds to transitions from n i = ∞
1 ε h2
and mv 2 = E to n f = 1, corresponding to wavelength rn = 0 × n2 ∝ n2
2 πmZe 2
= R  −  = R
1 1 1
2
or mv = 2mE (λ min )L 1 ∞  r1  n1  1
Hence, =  =
h r3  n3  9
∴ λe = ⇒
1
(λ min )L = = 912 Å
2mE R or r3 = 9 r1 = 9 r 0
The de-Broglie wavelength of photon Two rows line of Balmer series, [where, r3 is the radius of 3rd orbit]
hc
= R 
λ ph = 1 1 1  R
−  = 16 The wavelength of the lines in Balmer
E (λ min )B  22 ∞2  4
series is represented by,
λ ph hc 2mE 2m 2mc 2 4
= =c = ⇒ (λ min )B = = 3636 Å 1 1 1 
λe E h E E R = RH  − 
λ 2
2
n22 
(λ )
5 E = hc ⇒ E ∝ 1 ⇒ E ′ ∝ 400 = 1.33 Ratio is given by min L = 0.25
(λ min )B For first wavelength,
λ λ E 300
= 1.097 × 107  − 
1 1 1
But E = eVs , where Vs is stopping 11 Energy of nth orbit of hydrogen atom, λ1  22 32 
potential. 13.6
Thus, stopping potential for En = − eV = 1.524 × 106 m
n2 1
photoelectrons from a surface becomes
The energy absorbed, when an electron or λ1 = = 6.562 × 10−7
approximately 1.0 V greater. 1.524 × 106
jumps from the first energy level (n = 1)
λ1 = 6562 Å
6 E = hc , λ = 5000 Å = 5 × 10−7 m to second level (n = 2)
λ  1 1 
For second wavelength,
6.6 × 10−34 × 3 × 108 = − 13.6 ×  −  1
= 1.097 × 107  −
1 1
E = 2
 n1 n22   22
5 × 10−7
λ1 42 
 1 1  = 2.056 × 106
E = 3.96 × 10−19 J = − 13.6  − 
 (1)
2
(2)2  1
Energy received per second = 10−8 Js −1 or λ2 =
3 2.056 × 106
∴ Number of photons received per second = − 13.6 × = − 10.2eV
4 or λ2 = 4.863 or λ2 = 4863 Å
370 40 DAYS ~ NEET PHYSICS DAY THIRTY FIVE

17 The wavelength of different members of 21 R = R 0 A1 /3 28 r = n1 λ1 , τ = 1


Balmer series are 1 λ
1 log R = log R 0 + log A ,
1 1  3 For radioactive equilibrium,
= R H  −  where, n = 3, 4, 5, …
λ 2
2
n12  which is a straight line. n1 λ1 = n2 λ2
r = Nλ
The first member of Balmer series (H α ) NX N e − λ X t 3000e −12 t
corresponds to n i = 3. It has maximum 22 = 0X = 1
NY N 0Y e −λ Y t 6000e −10 t r =N⋅
energy and hence the longest τ
N X 1 −2 t 1 rτ = N
wavelength. = e ⇒ 1 = e −2 t (N X = N Y ) i.e.
= R H  − 
1 1 1 NY 2 2

λ1  22 32  29 Since, mvr = 2nh
1
e = ⇒ log e e 2 t = log e
2t 1 2π
= 1.097 × 107  
5 2 2 1
 36  ∴ In 2nd state KE becomes th of the
2t = log e   ⇒ t = log e 
 1 1  1 4
⇒ λ1 =
36  2 2  2 
initial KE i.e. 3.4 eV in n = 1.
5 × 1.097 × 107
23 From Rutherford-Soddy’s law, Binding energy BE = − 27.2 + 3.4
= 6.563 × 10−7 m n
= 23.8 eV
N = N 0  
1
⇒ n = 6563 Å  2 23.8
E1 = 23.8 eV, E2 = = 5.95 eV,
The wavelength of the Balmer series 3 1 t t 22
limit corresponds to n i = ∞ and has Given, N =1− = N 0, n = =
E3 = 2.64 eV = 
4 4 T 4 23.8 
shortest wavelength. 
1  1
t/ 4 2 t/ 4  32 
=   ⇒   =  
∴ Wavelength of Balmer series limit is 1 1

given by 4  2  2  2 ∴ Minimum visible light energy is

= R H  −  = 1.097 × 107 ×
1 1 1 1 t 2.64 eV.
⇒ 2 = ⇒ t = 8 months
λ∞  22 ∞  4 4 1242 Q λ ∝ 1 
λ= = 470 nm
4 −7 n 2.64  E 
λ∞ =
1.097 × 107
= 3.646 × 10 m 24 From Rutherford law, N = N 0  1 
 2 30 In order to compare the stability of the
= 3646 Å where, n is number of half-lives. nuclei of different atoms, binding
Only 4861Å is between the first and last 11400 energy per nucleon is determined.
n= =2
line of the Balmer series. 5700 Higher the binding energy per nucleon
2 more stable is the nucleus.
=  
18 Mass of uranium nucleus = Mass of N 1
∴ ∴ BE per nucleon of deuteron
proton + Mass of neutron N 0  2 1125
.
= 92 × 1.6725 × 10−27 + 143 N 1 = = 0.5625 MeV
⇒ = = 0. 25 ⇒ N = 0. 25 N 0 2
× 1.6747 × 10−27 N0 4
BE per nucleon of α-particle =
72
.
−27
= (153.87 × 10 + 239.48 × 10−27 ) N = 0.25 of original amount 4
= 393.35 × 10 −27 = 1.8 MeV
kg 25 λN = 3.7 × 107
Since, binding energy per nucleon of
Radius of nucleus is of the order of λ=
0.693 α-particle is more. Hence, it is more
10−15m, hence volume is t 1 /2 stable.
V ∝ (10−15 )3 m3 ∝ 10−45 m3 .
. × 107 × t 1 /2
37 31 The nuclei which have same number of
Mass ∴N = = 533
. × 1014
∴ Density = 0.693 neutrons but different atomic number
Volume and mass number are known as
6.023 × 1023 nuclides = 208 g
. × 10−27
39335 isotones. In choice (a) nuclei of 34 Se74
= = 1020 kgm −3 ∴ 5.3 × 1014 nuclides
10−45 and 31 Ga 71 are isotones, as
208 × 53
. × 1014
19 Number of nuclei remain undecayed = A − Z = 74 − 34
6.023 × 1023 = 71 − 31
N A = N 0 e −10 λ t
= 1.837 × 10−7 g = 0.184 µg = 40
and N B = N 0 e − λt
NA 1 26 Number of half-lives, n = 100 = 1600 32 When an atom emits an α-particle, its
⇒ = = e ( −10 λt + λt )
n mass number decreases by 4 and atomic
NB e 2
or n = 4 or 4t 1 /2 = 8 s number by 2, when it emits a β-particle
or 9 λt = 1 its atomic number increases by 1, while
1 ∴ t 1 /2 = 4 s
∴ t = N 1600 there is no change in mass number and
9λ N = 0 = = 200 atomic number in γ-ray emission
23 8
remains same.
20 After t > > t 1 /2 , the equilibrium will be 27 Mean life τ = 1.44 T1 /2
72 X
180
 −
α
→ 70 Y 176  −
β

achieved, so that 100
dN ∴ T1 /2 = = 69.44 s 71 Z
176
 −
α
→ 69 A
172
 −
γ
→ 69P
172
= λN 1.44
dt 69.44 33 When an α-particle is emitted, mass
R R t 1 /2 = ≈ 1155
. min
⇒ R = λN ⇒ N = = 60 number of nuclide X is reduced by 4
λ 0.693 and its charge number reduced by 2.
DAY THIRTY FIVE UNIT TEST 7 (DUAL NATURE OF MATTER, ATOMS AND NUCLEI) 371

But when a β-particle is emitted, mass 37 From Einstein’s photoelectric equation 41 In 1s only 90% remains.
number of nuclide remains the same hc
K = −W At t = 2 s, activity will remain 90% of 90,
and its charge number is increased by 1. λ i.e. 81 units.
Hence, the resulting nuclide has atomic
− W = 2  − W  + W
hc hc
K′ =
mass (A − 4) and atomic number (Z − 1 ). λ /2  λ  42 Already emitted electrons will repel the
new electrons, hence assertion is true
34 X is neutrino because mass number and =2K + W
1 but reason is false.
atomic number of neutrino is zero. Energy ∝
λ 43 In the process of β-decay, either electron
35 Let x be the mass number of A and y be
the atomic number. Then, since atomic 38 From Bohr’s orbit concept, or positron is emitted because either a
neutrino or an antineutrino is emitted.
mv 2 r2 = 2 
number and mass number remain h 
 Thus, β-rays have a continuous energy
conserved, we have  2π 
y A 
x
→ y − 2 B x − 4 + 2 He 4  h 
spectrum. The maximum kinetic energy
2 πr2 = 2   = 2 λ2 or end point energy (E ) must be equal to
x −4 x −4
y −2B 
→ yC + 2 −1 e 0
 mv 2  disintegration energy. When electron (or
h positron) has maximum energy, the
Hence, we observe that A and C are Further λ = (de-Broglie concept)
isotopes as their atomic numbers are p energy carried by the daughter nucleus
same but mass numbers are different. ∴ Speed of momentum is maximum in and neutrino is nearly zero.
36 220
X → 216
+ 4
He ground state. 44 When four helium nuclei are fused
z −2 y 2
Hence, λ is minimum. together, one oxygen nucleus is formed.
0 = 216 v 1 + 4 v 2
−4 v 2 v 39 E H = 13.6 eV [ionisation energy] The reaction is
⇒ v1 = =− 2 4 × 2 He 4 → 8O 16 + Q
216 54 E = 13.6 (z )2
He +
1 1 Mass defect, ∆m = 4 × m He − mO2
m1 v 12 + m2 v 22 = Q = 13.6 (2)2 = 54.4 eV
2 2 = 4 × 4.0026 − 15.9994 = 0.011 amu
⇒ E1 < E2
108v 12 + 2v 22 = 55
. MeV Energy produced,
Also, ionisation energy of single electron of
E = 0.011 × 931 = 10.24 MeV
He + is 54.4 eV.
2
 v 
108  − 2  + 2v 22 = 55
. MeV
 54  40 Rate of nuclear process cannot be altered 45 Z X A
+ 0n
1
→ 3 Li7 + 2 He 4
v 22 by altering pressure or temperature. A + 1 = 7 + 4; A = 11 − 1 = 10
+ 2v 22 = 55
. MeV Because any nuclear process involves
27 Z + 0 = 3 + 2; Z = 5
huge amount of energy, also radioactivity
⇒ 2 v 22 = 5.4 MeV The element X is 5 B10 .
is a nuclear phenomena.
DAY THIRTY SIX

Electronic
Devices
Learning & Revision for the Day
u Energy Bands in Solids u Diode as a Rectifier u Transistor
u Semiconductors u Special Purpose Diodes u Logic Gates
u Semiconductor Diode

Energy Bands in Solids


According to band theory of solids, in a crystalline solid due to mutual interaction
among valence electrons of neighbouring atoms, instead of sharp energy levels, energy
bands are formed. Energy bands are of the following three types
(a) Valence band It is the energy band formed by a series of energy levels of valence
electrons actually present. Ordinarily, valence band is completely filled and
electrons in this band are unable to gain energy from external electric field. The
highest energy level in a valence band at 0 K is called fermi energy level.
(b) Conduction band The energy band having just higher energy than the valence band
is called conduction band. Electrons in conduction band are commonly called the
free electrons.
(c) Forbidden band The energy gap between the valence band and the conduction band
of a solid is called the forbidden energy gap E g or forbidden band. Width of
forbidden energy gap depends upon the nature of substance.
PREP
Semiconductors MIRROR
Your Personal Preparation Indicator
l
In semiconducting solids, the valence band is completely filled but conduction band
is completely empty and the energy gap between them is small enough (E g < 3 eV). At u No. of Questions in Exercises (x)—
absolute zero temperature, it behaves as an insulator. u No. of Questions Attempted (y)—
u No. of Correct Questions (z)—
l
A pure semiconductor, in which no impurity of any sort has been mixed, is called (Without referring Explanations)
intrinsic semiconductor. Germanium (E g = 0.72 eV) and silicon (E g = 1.1 eV) are
examples of intrinsic semiconductors. u Accuracy Level (z / y × 100)—
u Prep Level (z / x × 100)—
l
Electrical conductivity of pure semiconductor is very small. To increase the
conductivity of a pure semiconducting material, it is doped with a controlled quantity
In order to expect good rank in NEET, your
(1 in 10 5 or 10 6) of suitable impurity. Such a doped semiconductor is called an Accuracy Level should be above 85 & Prep
extrinsic semiconductor. Level should be above 75.
DAY THIRTY SIX ELECTRONIC DEVICES 373

l
The number of electrons reaching from valence band to This layer containing immobile ions is called depletion layer.
− E / 2 kT
conduction band, n = AT 3 / 2e g The thickness of depletion layer is approximately of the order
where, k = Boltzmann’s constant, T = absolute temperature of 10 −6 m.
and A = atomic weight. The potential difference developed across the p-n junction
due to diffusion of electrons and holes is called the potential
barrier Vb (or emf of fictitious battery).
Superconductors
When few metals are cooled, then below a certain critical For germanium diode barrier potential is 0.3 V, but for Si
temperature, their electrical resistance suddenly becomes diode, its value is 0.7 V. The barrier electric field developed
zero. In this state, these substances are called due to it, is of the order of 10 5 Vm–1.
superconductors and this phenomena is called
superconductivity. Mercury become superconductor at 4.2 K, Mobility of Charge Carriers
lead at 7.25 K and niobium at 9.2 K.
The mobility of a charge carrier is defined as the velocity
gained by its per unit electric field, i.e. µ = Vd / E .
Types of Extrinsic Semiconductor
NOTE Current in semiconductor is, i = i e + i h = eA( neVe + nhVh )
1. n-type Semiconductor Conductivity, σ =
J
=
i
= e( neµ e + nhµ h )
To prepare an n-type semiconductor, a pentavalent E AE
impurity, e.g. P, As, Sb is used as a dopant with Si or Ge. (where, J = current density = nqV)
Such an impurity is called donor impurity, because each
dopant atom provides one free electron. I-V Characteristics in Forward
In n-type semiconductor ne > > nh , i.e. electrons are and Reverse Bias
majority charge carriers and the holes are minority charge
carriers, such that ne ⋅ nh = n2i . An n-type semiconductor is When we join an external potential source, such that p-side of
p-n junction is joined to positve of voltage source and n-side
electrically neutral and is not negatively charged. to negative of voltage source, the junction is said to be
Conductivity, σ ≈ ne µe e forward biased and applied electric field E opposes the
barrier electric field Eb .
2. p-type Semiconductor As a result, width of depletion layer is reduced and on
To prepare a p-type semiconductor, a trivalent impurity, applying a voltage V > Vb , a forward current begins to flow.
e.g. B, Al, In, Ga, etc., is used as a dopant with Si or Ge. Resistance offered by p-n junction in forward bias is small
Such an impurity is called acceptor impurity as each (about 10-50 Ω).
impurity atom wants to accept an electron from the crystal E IF
E Eb (mA)
lattice. Thus, effectively each dopant atom provides a hole. Eb
In p-type semiconductor nh > > ne , i.e. holes are majority r sr s r ss rr s
r s s ss rr r VR VF
s sr r r s
charge carriers and electrons minority charge carriers, p r ss rr s n
p r sr s n
such that nh ⋅ ne = n2i . A p-type semiconductor is + – – +
electrically neutral and is not positively charged. IR (µA)
(a) FB (b) RB Variation of forward and
The number of free electrons in a semiconductor varies reverse current with forward
with temperature as T 3 / 2 . and reverse voltage.
(c)
Conductivity, σ ≈ nhµ he
If connections of potential source are reversed [Fig. (b)], i.e.
p -side is connected to negative terminal of battery and n-side
Semiconductor Diode to positive terminal, the junction is said to be reverse biased
and in this case E and Eb , being in same direction, are added
A p-n junction is obtained by joining a small p-type crystal
with a small n-type crystal without employing any other up. So, the depletion layer broadens and potential barrier is
binding material in between them. Whenever a p-n junction fortified.
is formed, electrons from n-region diffuse through the Consequently, an extremely small leakage current flows
junction into p-region and the holes from p-region diffuse across the junction due to minority charge carriers and
into n-region. − 10 5 Ω).
junction resistance is extremely high (~
As a result of which neutrality of both n and p-regions is For a sufficiently high reverse bias voltage (25 V or even
disturbed, and a thin layer of immobile negative charged ions more), the reverse current suddenly increases. This voltage is
appear near the junction in the p-crystal and a layer of called Zener voltage or breakdown voltage or avalanche
positive ions appear near the junction in n-crystal. voltage.
374 40 DAYS ~ NEET PHYSICS DAY THIRTY SIX

Diode as a Rectifier Light Emitting Diode (LED)


Junction diode allows current to pass only when it is forward It is a specially designed diode made of GaAsP, GaP, etc.
biased. So, if an alternating voltage is applied across a diode, When used in forward biased, it emits characteristic, almost
the current flows only in that part of the cycle, when the monochromatic light. In reverse biased, it works like a normal
diode is forward biased. diode.
This property is used to rectify alternating voltages and the l
I-V Characteristics of LED LEDs are current dependent
circuit used for this purpose is called a rectifier, and the devices with its forward voltage drop (VF ) depending on the
process is known as rectification. forward biased LED current. Light emitting diode I-V
There are two types of rectifier diode are given below characteristics as given below

Infrared

Amber
1. Half Wave Rectifier A rectifier, which rectifies only

Yellow

Green
Blue
I (mA)

Red
one-half of each AC input supply cycle, is called a half
wave rectifier.
A half wave rectifier gives discontinuous and pulsating

Forward current
DC output. As no output is obtained corresponding to
alternate half cycles of the AC input supply, its efficiency
is quite low.
2. Full Wave Rectifier A rectifier, which rectifies both
halves of each AC input cycle is called a full wave
VF
rectifier.
The output of a full wave rectifier is continuous, but
pulsating in nature. However, it can be made smooth by Photodiode
using a filter circuit. It is a special diode used in reverse bias which conducts only
As output is obtained corresponding to both the half when light of suitable wavelengths is incident on the junction
cycles of the AC input supply, its efficiency is more than of diode. The energy of incident light photon must be greater
that of half wave rectifier. than the band gap of semiconductor (i.e. hν > E g ). Materials
used are Cds, Se, Zns, etc.
NOTE • The ripple factor is defined as the ratio of rms value of AC
component in the output of the rectifier to the DC
component in the input. Solar Cell
It is a special p-n junction, in which one of the
semiconductors is made extremely thin, so that solar
Special Purpose Diodes radiation falling on it reaches junction of diode without any
absorption. A solar cell directly converts, solar energy into
Zener Diode electrical energy. Popularly used solar cells, Ni-cd, PbS cell,
etc.
It is a highly doped p-n junction diode which is not
n
damaged by high reverse current. It is always used
p
in reverse bias in breakdown voltage region and is
chiefly used as a voltage regulator.
Transistor
A transistor is a combination of two p-n junctions joined in
l
Zener Diode as Voltage Regulator The following circuit is
series. A junction transistor is known as Bipolar Junction
used for stabilising voltage across a load RL . The circuit
Transistor (BJT). It is a three terminal device.
consists of a series voltage-dropping resistance R and a
Zener diode in parallel with the load RL . Transistors are of two types
+ I R (i) n-p-n transistor,
+
(ii) p-n-p transistor
A transistor has three regions
Fluctuating n Constant
p n Zener VZ RL DC output
(i) An emitter (E), which is most heavily doped, and is of
DC input
voltage diode voltage moderate size. It supplies large number of charge carriers,
p
which are free electrons in an n-p-n transistor and holes
in a p-n-p transistor.

– (ii) A base (B), which is very lightly doped and is very thin
The Zener diode is selected with Zener voltage Vz equal (thickness ~ 10 −5m).
to the voltage desired across the load. (iii) A collector (C), which is moderately doped and is
thickest.
DAY THIRTY SIX ELECTRONIC DEVICES 375

A transistor is symbolically represented as shown in figures. where, RC and RB are net resistances in collector and base
n-p-n p-n-p circuits, respectively.
E E C
l
In common base configuration, AC current gain is defined
C ∆I
as α = C
∆I V = constant
CE
B B
l
Value of α is slightly less than 1. In fact, 0.95 ≤ α ≤ 1.
(a) (b)
∆P
l
Power gain = o = β2AC × Resistance gain
∆Pi
Transistor Action
For proper functioning of a transistor, the emitter-base NOTE • Current gains α and β are correlated as
α β
junction is forward biased, but the collector-base junction is β= or α =
reverse biased. In an n-p-n transistor, electrons flow from 1−α 1+ β
emitter towards the base and constitute a current I E .
Due to larger reverse bias at base-collector junction, most of Transistor as an Amplifier
these electrons further pass into the collector, constituting a A transistor consisting of two p-n junctions, one forward
collector current IC . But a small percentage of electrons (less biased and the other reverse biased can be used to amplify a
than 5%) may combine with holes present in base. These weak signal. The forward biased junction has a low resistance
electrons constitute a base current I B . It is self evident, that path, whereas the reverse biased junction has a high
I E = IC + I B . resistance path. The weak input signal is applied across the
Action of p-n-p transistor is also same, but with one forward biased junction, and the output signal is taken across
difference that holes are moving from emitter to base and the reverse biased junction.
then to collector. A transistor can be connected in either of Since, the input and output currents are almost equal, the
the following three configurations output signal appears with a much higher voltage. The
(i) Common Emitter (CE) configuration transistor, thus acts as an amplifier. Common-emitter
configuration of transistor amplifier is given below
(ii) Common Base (CB) configuration n-p -n C
(iii) Common Collector (CC) configuration.
B
Generally, we prefer common emitter configuration, because
power gain is maximum in this configuration.
E Output
Input
Characteristics of a Transistor
In common emitter configuration, variation of current on the – +
+ –
input side with input voltage (I E versus VBE ) is known as the
input characteristics, and the variation in the output current
with output voltage (IC versus VCE ) is known as output
characteristics. From these characteristics, we obtain the Transistor as an Oscillator
values of following parameters An electronic oscillator is a device that generates electrical
∆VBE oscillations of constant amplitude and of a desired frequency,
l
Input resistance, ri =
∆I B V = constant without any external input.
CE
The circuit providing such oscillation, is known as a tank
∆VCE
l
Output resistance, ro = oscillator, is using positive feedback.
∆ IC I B = constant

∆ IC
l
AC current gain, β= L′
∆I B
Inducting coupled

VCE = constant
C
The current gain for common-emitter configuration
B
β ranges from 20 to 200. n- p- n
B2
∆ IC β
l
Transconductance, g m = = E
∆VBE ri
C
l
A transistor can be used as an amplifier. The voltage gain L
of an amplifier will be given by
V R
A V = o = β⋅ C K
Vi RB B1
376 40 DAYS ~ NEET PHYSICS DAY THIRTY SIX

Some of the properties of the oscillator are The Boolean expression for OR gate is Y = A + B
l
Oscillator is using positive feedback. This indicates Y equals A OR B.
l
To work as an oscillator, Truth table for OR gate ( Y = A + B )
|Aβ| = 1; β → feedback factor
A B Y
1 1
l
f = frequency of oscillation = × . 0 0 0
2π LC
0 1 1

Transistor as a Switch 1 0 1

The circuit resembles that of the Common-Emitter circuit. 1 1 1


The difference this time is that to operate the transistor as a
switch the transistor needs to be turned either fully ‘‘OFF’’ The output of an OR gate assumes 1 if one of more inputs
(Cut-off) fully ‘‘ON’’ (Saturated). An ideal transistor switch assume 1. The output is high when either of inputs A or B is
would have an infinite resistance when turned ‘OFF’ high, but not if both A and B are high.
resulting in zero current flow and zero resistance, when
turned ‘‘ON’’, resulting in maximum current flow.
The AND Gate
In practice, when turned ‘‘OFF’, small leakage currents flow The AND gate a device has also two or more inputs and one
through the transistor and when fully “ON” the device has a output. The logic symbol of AND gate is
A
low resistance value causing a small saturation voltage (Vce ) Y
B
across it. In both the cut-off and saturation regions, the power
dissipated by the transistor is at its minimum. The logic symbol of AND gate is given as under. The Boolean
expression for AND gate is Y = A ⋅ B, this indicates Y equals to
Vcc A AND B.

Load
Truth table for AND gate ( Y = A ⋅ B )
Flywheel Relay
Diode Output A B Y
Ic 0 0 0
RB ia Vce 0 1 0
+ β
Vin Ω R 1 0 0
– 1 1 1

The output of an AND gate is 1 only, when all the inputs


assume 1.
Logic Gates The NOT Gate
A logic gate is an elementary building block of a digital
The NOT gate is a device which has only one input and only
circuit. Most logic gates have two inputs and one output.
one output. The logic symbol of NOT gate is as shown in
At any given moment, every terminal is in one of the two figure.
binary conditions low (0) or high (1), represented by different
voltage levels.
A Y
The logic state of a terminal can, and generally does, change
often, as the circuit processes data. In most logic gates, the
low state in approximately zero volts (0V), while the high The Boolean expression for NOT gate is
state is approximately five volts positive (+ 5V).
Y = A,
There are seven basic logic gates. AND, OR, NOT, XOR,
which indicates Y equals NOT A.
NAND, NOR and XNOR.
The basic logic gates are of three types Truth Table for NOT gate ( Y = A )
A Y
The OR Gate 0 1
1 0
The OR gate is a davice has two or more inputs and one
output. this devices combines two inputs to give one output. The output of a NOT gate assumes 1, if input is 0 and
The logic symbol of OR gate is vice-versa.
A These basic gates (OR, AND and NOT) can be combined in
Y various ways to provide large number of complicated digital
B
circuits.
DAY THIRTY SIX ELECTRONIC DEVICES 377

y′
Combination of Logic Gates A
y
B
1. NAND Gate
The logic symbol of NOR gate is shown as
In this type of gate, the output of AND gate is fed to input of a
NOT gate and final output is obtained at output of NOT gate. A
y=A+B
A B
Y′
Y
B
The Boolean expression for NOR gate is Y = A + B, which
indicates that ‘A OR B are negated’
The logic symbol of NAND gate is shown as Truth table for NOR gate
A A B Y′ Y
Y
B 0 0 0 1
1 0 1 0
The Boolean expression of NAND gate is Y = A ⋅ B, which 0 1 1 0
indicates A and B are negated. 1 1 1 0
Truth table for NAND gate
NOTE • NAND and NOR gates are known as universal gate.
A B Y′ Y
• The Boolean expressions obey the commutative law,
0 0 0 1 associative law as well as distributive law.
1 0 0 1 Commutative law
0 1 0 1 (i) A + B = B + A (ii) A ⋅ B = B ⋅ A
1 1 1 0 Associative law
(iii) A + (B + C ) = ( A + B ) + C (iv) ( A ⋅ B ) ⋅ C = A ⋅ (B ⋅ C )
2. NOR Gate Distributive law
In this type of gate, the output of OR gate is fed to input of (v) A ⋅ (B + C ) = A ⋅ B + A ⋅ C (vi) A + A ⋅ B = A + B
the NOT gate and final output is obtained at output of the (vii) A + A ⋅ B = A (viii) A ⋅ ( A + B ) = A
NOT gate. (ix) A ⋅ ( A + B ) = A ⋅ B (x) A ⋅ B = A + B

DAY PRACTICE SESSION 1

FOUNDATION QUESTIONS EXERCISE


1 The device that can act as a complete electronic circuit 3 Which one of the following statement is false?
is j
CBSE AIPMT 2010 j
CBSE AIPMT 2011
(a) junction diode (b) integrated circuit (a) Pure Si doped with trivalent impurities gives a p-type
(c) junction transistor (d) Zener diode semiconductor
2 The temperature (T) dependence of resistivity ( ρ ) of a (b) Majority carriers in a n-type semiconductor are holes
semiconductor is represented by (c) Minority carriers in a p-type semiconductor are electrons
(d) The resistance of intrinsic semiconductor decreases
r r
with increase of temperature
(a) (b) 4 If a small amount of antimony is added to germanium
crystal, j
CBSE AIPMT 2011
(a) the antimony becomes an acceptor atom
O T O T (b) there will be more free electrons than holes in the
r r semiconductor
(c) its resistance is increased
(c) (d) (d) its capacitance is increased

O T O T
378 40 DAYS ~ NEET PHYSICS DAY THIRTY SIX

5 The conductivity of a semiconductor increases with 12 The contribution in the total current flowing through a
increase in temperature because 3 1
semiconductor due to electrons and holes are and . If
(a) number density of free current carriers increase 4 4
(b) relaxation time increases
5
the drift velocity of the electron is times that of holes at
(c) both number density of carriers and relaxation time 2
increase this temperature, then the ratio of concentration of
(d) number density of current carriers increase, relaxation time electrons and holes is
decreases but effect of decrease in relaxation time is (a) 6 : 5 (b) 5 : 6 (c) 3 : 2 (d) 2 : 3
much less than increase in number density
13 Depletion layer contains
6 In a n-type semiconductor, which of the following (a) Only immobile negative and positive ions
statement is true? ª NEET 2013 (b) Only free charge carrier
(c) Both free carrier and immobile ions
(a) Electrons are majority carriers and trivalent atoms are
(d) None of the above
dopants
(b) Electrons are minority carriers and pentavalent atoms are 14 In an unbiased p-n junction, holes diffuse from the
dopants p-region to n-region because
(c) Holes are minority carriers and pentavalent atoms are (a) free electrons in the n-region attract them
dopants (b) they move across the junction by the potential difference
(d) Holes are majority carriers and trivalent atoms are dopants (c) hole concentration in p-region is more as compared to
(d) it becomes a p-type semiconductor n-region
7 A specimen of silicon is to be made p-type (d) All of the above
semiconductor. For this one atom of indium, on an 15 The barrier potential of a p-n junction depends on
average, is doped in 5 × 107 silicon atoms. If the number I. Type of semiconductor material
density of silicon is 5 × 1028 atoms/m 2 , then the number
II. Amount of doping
of acceptor atoms per cm 3 will be
III. Temperature
(a) 2.5 × 1030 (b) 1.0 × 1013
(c) 1.0 × 1015 (d) 2.5 × 1036 Which one of the following is correct? ª CBSE AIPMT 2014
8 The resistivity of an n-type extrinsic semiconductor is (a) I and II (b) Only II (c) II and III (d) I, II and III
0.25 Ω-m. If the electron mobility is 8.25 m / V-s, then the
2 16 In forward biasing of the p-n junction ª CBSE AIPMT 2011
concentration of donor atoms will be (in m −3 ) (a) the positive terminal of the battery is connected to n-side
(a) 3.0 × 1016 (b) 3.0 × 1017 (c) 3.0 × 1018 (d) 3.0 × 1019 and the depletion region becomes thin
(b) the positive terminal of the battery is connected to n-side
9 The number of densities of electrons and holes in pure
and the depletion region becomes thick
silicon at 27°C are equal and its value is 1.5 × 10 16 m −3 .
(c) the positive terminal of the battery is connected to p-side
On doping with indium, the hole density increases to
and the deplection region become thin
4.5 × 10 27 m −3 . The electron density in doped silicon will be
(d) the positive terminal of the battery is connected to p-side
(a) 50 × 109 m−3 (b) 5 × 109 m−3 and the depletion region becomes thick
(c) 108 m−3 (d) 107 m−3
17 Which one of the following represents forward bias
10 In n-type germanium, the mobility of electrons is diode? ª NEET 2017
3900 cm 2 / Vs and their conductivity is 5 mho / cm. If the R –2 V
0V
cotter contribution is negligible, then impurity (a)
concentration will be –4V R –3 V
(b)
(a) 8 × 1015 per cm3 (b) 9.25 × 1014 per cm3
(c) 6 × 1013 per cm3 (d) 9 × 1013 per cm3 –2V R +2 V
(c)
11 The ratio of electron and hole currents in a
3V R 5V
5 (d)
semiconductor is and the ratio of drift velocities of
4
7 18 If in a p-n junction, a square input signal of 10 V is
electrons and holes is , then the ratio of concentrations
4 applied, as shown, ª CBSE AIPMT 2015
of electrons and holes will be 100 W
+5V
25 49 7 5
(a) (b) (c) (d)
49 25 5 7 RL

-5V
DAY THIRTY SIX ELECTRONIC DEVICES 379

then the output across RL will be 27 In a common emitter amplifier, the output resistance is
5000 Ω and the input resistance is 2000 Ω. If the peak
10V
value of the signal voltage is 10 mV and β = 50, then the
(a) (b) peak value of the output voltage is
–10V (a) 1. 25 V (b) 125 V
(c) 5 × 10−6 V (d) 2.5 × 10−4 V
5V 28 The input and output resistances in a common base
(c) (d)
–5V amplifier circuit are 400 Ω and 400 k Ω. If the current gain
α is 0.98 and emitter current is 2 mA, then the base
19 Current through the ideal diode as shown in figure, is current is
100 W
(a) 0.02 mA (b) 0.08 mA (c) 0.05 mA (d) 0.04 mA
29 A p-n-p transistor is used in common emitter mode in an
2V 5V amplifier circuit. A charge of 40 µA in the base current
brings a charge of 2 mA in collector current and of 0.04 V
in base emitter voltage. The base current amplification
factor is
(a) zero (b) 20 A (c) (1/20) A (d) (1/50 )A (a) 5 (b) 50 (c) 500 (d) 0.5
20 Consider the junction diode as ideal. The value of current 30 The input resistance of a CE amplifier is 333 Ω and the
flowing through AB is ª NEET 2016 load resistance is 5 kΩ. A change of base current by
15 µA a result in the change of collector current by 1 mA.
A 1KΩ B The voltage gain of the amplifier is
+4V – 6V
(a) 550 (b) 51 (c) 101 (d) 1001
(a) 10−2 A (b) 10−1 A (c) 10−3 A (d) 0 A 31 A common emitter amplifier has a voltage gain of 50, an
21 LED has a voltage drop of 2 V across it, and passes a input impedance of 100 Ω and an output impedance of
current of 10 mA. When it operates with a 6 V battery 200 Ω. The power gain of the amplifier is ª CBSE AIPMT 2010
through a limiting resistor R. The value of R is (a) 500 (b) 1000 (c) 1250 (d) 50
(a) 40 kΩ (b) 4 kΩ (c) 200 Ω (d) 400 Ω 32 For CE transistor amplifier, the audio signal voltage
22 The electrical conductivity of semiconductor increases across the collector resistance of 2 kΩ is 4 V. If the
when electromagnetic radiation of wavelength shorter current amplification factor of the transistor is 100 and
than 2480 nm is incident on it. The band gap (in eV) for the base resistance is l kΩ, then the input signal voltage
the semiconductor is is ª NEET 2016
(a) 0.9 (b) 0.7 (c) 0.5 (d) 1.1 (a) 10 mV (b) 20 mV (c) 30 mV (d) 15 mV
23 A p-n photodiode is fabricated from a semiconductor 33 A transistor is operated in common-emitter configuration
with a band gap of 2.5 eV. It can detect a signal of at Vc = 2 volt such that a change in the base current from
wavelength ª CBSE AIPMT 2009 100 µA to 200 µA produces a change in the collector
(a) 6000 Å (b) 4000 nm (c) 6000 nm (d) 4000 Å current from 5 mA to 10 mA. The current gain is
24 In a transistor, the base is ª AFMC 2012 ª CBSE AIPMT 2009

(a) an insulator (a) 75 (b) 100 (c) 150 (d) 50


(b) a conductor of low resistance 34 Symbolic representation of four logic gates are shown as
(c) a conductor of high resistance ª CBSE AIPMT 2011
(d) an extrinsic semiconductor
(i) (ii)
25 The minimum potential difference between the base and
emitter required to switch a silicon transistor ON, is
approximately (iii) (iv)
(a) 1 V (b) 3 V (c) 5 V (d) 4.2 V
26 In a common base transistor circuit Pick out which ones are for AND, NAND and NOT gates,
IC = 9.7 µA , IB = 0.03 µA, then current gain, α = respectively.
(a) 0.97 (b) 0.097 (a) (iii), (ii) and (i) (b) (iii), (ii) and (iv)
(c) 95 (d) 500 (c) (ii), (iv) and (iii) (d) (ii), (iii) and (iv)
380 40 DAYS ~ NEET PHYSICS DAY THIRTY SIX

35 The combination of the gates shown in the figure below (a) A = 1, B = 0, C = 0 (b) A = 1, B = 1, C = 0
produces (c) A = 1, B = 0, C = 1 (d) A = 0, B = 1, C = 0

A
42 What is the output Y in the following circuit, when all the
three inputs A, B, C are first 0 and then 1? ª NEET 2016
Y
B A
P
B Q
(a) NOR gate (b) OR gate Y
(c) AND gate (d) XOR gate C

36 Which logic gate is represented by the following (a) 0, 1 (b) 0, 0


(c) 1, 0 (d) 1, 1
combination of logic gates? ª AIPMT 2015
43 The figure shows a logic circuit with two inputs A and B,
Y1
A and the output C. The voltage waveforms across A, B
and C are as given. The logic circuit gate is
ª CBSE AIPMT 2012
Y

B
Y2
A
(a) OR (b) NAND (c) AND (d) NOR
37 A proper combination of 3 NOT and 1 AND gates is
shown. If A = 0 , B = 1 and C = 1, then the output of this B
combination is
A
C
t1 t2 t3 t4 t5 t6
B Y
(a) OR gate (b) NOR gate
(c) AND gate (d) NAND gate
C
44 To get an output Y = 1 from the circuit shown below, the
(a) 1 (b) zero
input must be ª CBSE AIPMT 2010
(c) not predictable (d) None of these
A
38 The output ( X ) of the logic circuit shown in figure will be B Y
ª NEET 2013 C

A A B C A B C
X
B (a) 0 1 0 (b) 0 0 1
(c) 1 0 1 (d) 1 0 0
(a) X = A ⋅ B (b) X = A ⋅ B
(c) X = A ⋅ B (d) X = A + B 45 In the following circuit, the outputY for all possible inputs
A and B is expressed by the truth table
39 Digital circuit can be made by repetitive use of this gate
A
(a) AND (b) OR Y
(c) NOT (d) NAND B
40 The given electrical network is equivalent to ª NEET 2017 (a) A B Y (b) A B Y
0 0 0 0 0 1
A Y
B 0 1 0 0 1 1
1 0 0 1 0 1
(a) AND gate (b) OR gate
1 1 1 1 1 0
(c) NOR gate (d) NOT gate
(c) A B Y (d) A B Y
41 To get output 1 for the following circuit, the correct 0 0 1 0 0 0
choice for the input is ª NEET 2016 0 1 0 0 1 1
A 1 0 0 1 0 1
B Y
1 1 0 1 1 1
C
DAY THIRTY SIX ELECTRONIC DEVICES 381

DAY PRACTICE SESSION 2

PROGRESSIVE QUESTIONS EXERCISE


1 The following figure shows a logic gate circuit with two 6 Assume that the number of hole-electron pairs in an
inputs A and B, and the output C. The voltage waveform intrinsic semiconductor is proportional to e − ∆E / 2kT . Here,
of A, B and C are as shown below ∆E = energy gap and k = 8.62 × 10−5 eV/K. The energy
A Logic gate gap for silicon is 1.1 eV. The ratio of electron-hole pairs at
C
B circuit 300 K and 400 K, is
(a) e − 5.31 (b) e +5
1 (c) e (d) e +3

A t
7 A potential difference of 2.5 V is applied across the
1 faces of a germanium crystal plate. The face area of the
crystal is 1cm 2 and its thickness is 1.0 mm. The free
B t
electron concentration in germanium is
1
2 × 1019 m −3 , and the electron and hole mobilities are
C t 0.33 m 2 / Vs and 0.17 m 2 / Vs, respectively. The current
across the plate will be
The logic circuit gate is
(a) 0.2 A (b) 0.4 A
(a) AND gate (b) NAND gate (c) 0.6 A (d) 0.8 A
(c) NOR gate (d) OR gate
8 The given graph represents V-I characteristic for a
2 In the combination of the following gates the outputY can semiconductor device. Which of the following statement is
be written in terms of inputs A and B as ª NEET 2018 correct? ª CBSE AIPMT 2014
A
B
I
Y
A
V
(a) A ⋅ B + A ⋅ B (b) A ⋅ B + A ⋅ B B

(c) A ⋅ B (d) A + B

3 In a p-n junction diode, change in temperature due to (a) It is V-I characteristic for solar cell, where point A
heating represents open circuit voltage and point B short circuit
(a) does not affect resistance of p-n junction current.
(b) affects only forward resistance (b) It is for a solar cell and points A and B, represent open
(c) affects only reverse resistance circuit voltage and current, respectively.
(d) affects the overall V-I characteristics of p-n junction (c) It is for a photodiode and points A and B, represent open
4 In the given circuit, all diodes are ideal. The current circuit voltage and current, respectively.
through battery is (d) It is for a LED and points A and B, represent open circuit
voltage and short circuit current respectively.
D1
5Ω
9 C and Si both have same lattice structure, having 4 bonding
electrons in each. However, C is insulator, whereas Si is
intrinsic semiconductor. This is because ª CBSE AIPMT 2012
3Ω (a) in case of C, the valence band is not completely filled at
3Ω
D2
absolute zero temperature
(b) in case of C, the conduction band is partly filled even at
2Ω absolute zero temperature
(c) the four bonding electrons in the case of C lie in the
20 V second orbit, whereas in the case of Si, they lie in the third
(a) 2 A (b) 1 A (c) 3 A (d) 4 A (d) the four bonding electrons in the case of C lie in the third
5 How many gates are required to design P = X + XY ? orbit, whereas for Si, they lie in the fourth orbit
(a) 1 (b) 2 (c) 3 (d) 4
382 40 DAYS ~ NEET PHYSICS DAY THIRTY SIX

10 The given circuit has two ideal diodes connected as 15 In a common emitter transistor amplifier, the audio signal
shown in the figure below. The current flowing through voltage across the collector is 3 V. The resistance of
the resistance R1 will be ª NEET 2016 collector is 3 kΩ. If current gain is 100 and the base
2Ω resistance is 2 kΩ, the voltage and power gain of the
amplifier is ª NEET 2017
R1 D2 (a) 200 and 1000 (b) 15 and 200
D1
(c) 150 and 15000 (d) 20 and 2000
10 V
R2 3Ω R3 2Ω 16 A n-p-n transistor is connected in common emitter
configuration in a given amplifier. A load resistance of
(a) 2.5 A (b) 10.0 A (c) 1.43 A (d) 3.13 A 800 Ω is connected in the collector circuit and the
voltage drop across it is 0.8V. If the current amplification
11 The input signal given to a CE amplifier having a voltage
factor is 0.96 and the input resistance of the circuits is
 π
gain of 150 is Vi = 2 cos 15 t +  . The corresponding 192 Ω, the voltage gain and the power gain of the
 3
amplifier will respectively be ª NEET 2016
output signal will be ª CBSE AIPMT 2015
(a) 3.69, 3.84 (b) 4, 4
π 2π
(a) 300 cos  15 t +  
(b) 75 cos  15 t +  (c) 4, 3.69 (d) 4, 3.84
 3  3 
5π 4π 
(c) 2 cos  15 t + (d) 300 cos  15 t +
17 In the circuit shown in the figure, the input voltageVi is 20
 
 3   3 V, VBE = 0 and VCE = 0. The values of IB , IC and β are
given by ª NEET 2018
12 The base resistance RB in the circuit, is (given hFE = 90)
20 V

RC 2 kW RC 4 kW

C
RB + RB
4V 9V Vi
– 500 kW B
E
3V

(a) IB = 20 µA, IC = 5 mA, β = 250


(a) 10 k Ω (b) 82 k Ω (b) IB = 25 µA, IC = 5 mA, β = 200
(c) 100 k Ω (d) 0.82 k Ω (c) IB = 40 µA, IC = 10 mA,β = 250
13 In a Common Emitter (CE) amplifier having a voltage gain (d) IB = 40 µA, IC = 5 mA, β = 125
G, the transistor used has transconductance 0.03 mho 18 Transfer characteristic [output voltage (Vo ) vs input
and current gain 25. If the above transistor is replaced voltage (Vi )] for a base biased transistor in CE
with another one with transconductance 0.02 mho and configuration is as shown in the figure. For using
current gain 20, the voltage gain will transistor as a switch, it is used ª CBSE AIPMT 2012
ª NEET 2013
2 I II III
(a) G (b) 1.5 G
3 Vo
1 5
(c) G (d) G
3 4
14 In a CE transistor amplifier, the audio signal voltage
across the collector resistance of 2 k Ω is 2 V. If the base
resistance is 1k Ω and the current amplification of the Vi
transistor is 100, the input signal voltage is
(a) in region III
ª CBSE AIPMT 2012
(b) both in region (I) and (III)
(a) 0.1 V (b) 1.0 V
(c) in region II
(c) 1 mV (d) 10 mV
(d) in region I
DAY THIRTY SIX ELECTRONIC DEVICES 383

ANSWERS
SESSION 1 1 (b) 2 (c) 3 (b) 4 (b) 5 (d) 6 (c) 7 (c) 8 (c) 9 (b) 10 (a)
11 (d) 12 (a) 13 (a) 14 (c) 15 (d) 16 (c) 17 (a) 18 (d) 19 (a) 20 (a)
21 (d) 22 (c) 23 (d) 24 (b) 25 (a) 26 (a) 27 (a) 28 (d) 29 (b) 30 (d)
31 (c) 32 (b) 33 (d) 34 (c) 35 (b) 36 (c) 37 (a) 38 (c) 39 (d) 40 (c)
41 (c) 42 (c) 43 (a) 44 (c) 45 (d)
SESSION 2 1 (a) 2 (b) 3 (d) 4 (a) 5 (a) 6 (a) 7 (b) 8 (a) 9 (c) 10 (a)
11 (d) 12 (b) 13 (a) 14 (d) 15 (c) 16 (d) 17 (d) 18 (b)

Hints and Explanations


SESSION 1 7 Number of acceptor atoms/cm3 14 In an unbiased p-n junction, the
1 Integrated circuits are miniature 5 × 1022 −3 diffusion of charge carriers across the
= = 1 × 10 cm15

electronic circuit produced within a 5 × 10 7 junction takes place from higher


single crystal of a semiconductor such concentration to lower concentration.
1 1
as silicon. They contain a million or so 8 ρ= or n = 15 Barrier potential depends on all the
transistors and resistors or capacitors. ne µ e ρ eµ e
three factor.
They are widely used in memory 1
=
circuits, micro computers, pocket 0.25 × 1.6 × 10−19 × 8.25 16 In forward biasing of p -n junction, the
calculators and electronic watches on = 3.0 × 10 m 18 −3 positive terminal of the battery is
account of their low cost and bulk, connected to p -side and the deplection
reliability into specific regions of the 9 n2i = ne n h region becomes thin.
semiconductor crystals. n2i 2.25 × 1032
∴ ne = = = 5 × 109 m −3 17 In the forward biasing of P-N junction, p
2 The resistivity of a semiconductor nh 4.5 × 1022 side of junction diode is connected to
decreases with increase in temperature higher potential and n side of junction
exponentially. 10 σ n = nd eµ n diode is connected to lower potential.
3 p-type semiconductor are obtained by ⇒ 5 = nd × 1.6 × 10−19 × 3900 Hence, the option (a) is correct answer.
adding a small amount of trivalent 5
∴ nd = 5V
impurity to a pure sample of 1.6 × 39 × 10−17
semiconductor (Ge). 18
= 8 × 1015 per cm3
Majority charge carriers — holes As it is forward-bias takes +ve value.
Minority charge carriers — electrons 11 I = neAv or I ∝ nv 19 Here, p-n junction is reverse biased.
In n-type semiconductor, Ie n v Therefore, the current flowing through
∴ = e e
Majority charge carriers—electrons Ih nh v h p-n junction is zero.
Minority charge carriers —holes 5 ne 7
⇒ = ⋅ 20 Let us assume that current through the
The resistance of intrinsic 4 nh 4 diode is I.
semiconductors decreases with increase
n 5
of temperature. or e = From the given condition
nh 7 V − VB 4 − (−6)
4 When a small amount of antimony is Q I = A =
added to germanium crystal, the crystal R 1 KΩ
12 Current density
10
becomes n-type semiconductor because J = nqv ⇒ Je = ne q v e = = 10−2 A
antimony is a pentavalent substrate. It
J h = nh q v h 1 × 103
excess free electrons.
Je n v 21 The term LED is abbreviated as Light
5 As temperature increases, increase in ⇒ = e × e
Jh nh vh Emitting Diode. It is forward biased p - n
number density of current carriers is
3 / 4 ne 5 n 6 junction which emits spontaneous
dominant. Hence, conductivity = × ⇒ e = radiation. Current in the circuit = 10 mA
increases. 1 / 4 nh 2 nh 5
= 10 × 10−3 A and voltage in the circuit
6 The n-type semiconductor can be 13 Depletion layer is formed due to = 6 − 2 = 4V.
produced by doping an impurity atom of production of positive ions on n-side From Ohm’s law, V = IR
valence 5, i.e. pentavalent atoms, i.e. and negative ions on p-side of the V 4
phosphorus. junction. ⇒ R= = = 400 Ω
I 10 × 10−3
384 40 DAYS ~ NEET PHYSICS DAY THIRTY SIX

∆ Ic × R L
22 Band gap, E = hc 30 Voltage gain, A V = 38 X = AB = A ⋅ B (i.e. AND gate)
λ ∆ Ib × R i
If the output X of NAND gate is
6.6 × 10−34 × 3 × 108 19.8 × 10−17 (1 × 10−3 ) × (5 × 10−3 )
= J= = ≈ 1001
connected to the input of NOT gate
2480 × 10−9 2480 (15 × 10−6 ) × 333 (made from NAND gate by joining two
−20
8 × 10 inputs) from the given figure, then we
= 8 × 10−20 J = eV 31 Voltage gain = β × impedance gain get back an AND gate.
1.6 × 10−19
−1 200
= 5 × 10 = 0.5 eV ⇒ 50 = β × ⇒ β = 25 39 We know that, the repetitive use of
100 NAND and NOR gate gives digital
c Also, power gain = β2 × impedance gain
23 Energy, E = hν = h circuits.
λ 200
hc = (25)2 × = 1250 40 Truth table for given network is
⇒ λ= 100
E
32 Given, collector resistance = Rout = 2 kΩ NOR NOR NOT
Substituting the values of h, c and E in A Y
Current amplification factor, β = 100 B Y1 Y2
the above equation
Base resistance, R in = 1 k Ω
6.6 × 10−34 × 3 × 108 Output signal voltage = 4 V
λ= = 5000 Å A B Y1 Y2 Y
2.5 × 1.6 × 10−19 Putting all the values in given equation,
we get 0 0 1 0 1
As, 4000 Å < 5000 Å
R 2 kΩ
A V = β out = 100 × ⇒A V = 200 1 0 0 1 0
Signal of wavelength 4000 Å can be R in 1 kΩ
detected by the photodiode. (V out )AC 0 1 0 1 0
Now, AV = = 200
24 In a transistor, the base is a conductor of (V in )AC 1 1 0 1 0
low resistance. 4
⇒ (V in )AC = = 20 mV Output Y of network matches with that
25 To switch ON the transistor, the 200 of NOR gate.
emitter-base junction of a transistor is 33 Here, ∆IC = 10 × 10−3 − 5 × 10−3 41 Consider the given figure,
forward biased while collector-base
= 5 × 10−3 A The resultant boolean expression of the
junction is reverse biased. The cut-off
voltage for silicon is ~1 V, so to switch ∆I B = 200 × 10−6 − 100 × 10−6 above logic circuit will be
ON a silicon transistor a potential = 100 × 10−6 A Y = ( A + B )⋅ C
Only option (c) is the correct choice, i.e.
difference of 1 V approximately, is ∴Current gain,
required between the base and emitter. ∆I 5 output Y = 1 only when inputs
β= C = × 1000 = 50 A = 1, B = 0 and C = 1.
IC IC ∆I B 100
26 ∴ α = =
42 Output of the given circuit is given by
IE I B + IC 34 The symbols given in problem are
0.97 0.97 y = ( AB ) (C )
= = = 0.97 (i) OR (ii) AND
0.03 + 0.97 1.00 When A = B = C = 0,
(iii) NOT (iv) NAND
27 We have, R L = 5000 Ω, R i = 2000 Ω, Y1 = (0) (0) (0) = 0 = 1
35 Y = A ⋅ B = A + B
B = 50 Thus, the combination of the gates When A = B = C = 1,
The AC voltage gain is given by produces OR gate. Y2 = (1) (1) = 0
R 50 × 5000
β× L = = 125
Ri 2000 36 43 From the given waveforms, the
Thus, the peak output voltage = voltage A B Y1 Y2 Y following truth table can be made
gain × signal voltage 0 0 1 1 0 Input Output
= 125 × 10 mv 0 1 1 0 0
= 1250 mV = 1.25V 1 0 0 1 0
A B C

28 Voltage gain in common base amplifier 0 0 0


1 1 0 0 1
is 1 0 1
Above truth table is described AND gate.
R 400 × 103
Av = α × out = 0.98 × 1 1 1
R in 400 37 The simplified circuit is shown in figure
below. 0 1 1
= 980
I A This truth table is required to OR gate.
⇒ α = C A
So, logic circuit gate is OR gate.
IE
∴ IC = α I E = 0.98 × 2 mA = 1.96 mA B
B 44 Gate I is OR gate, Y ′ = A + B
I E = I B + IC ⇒ I B = I E − IC = 2 mA Y=A+B+C A Y′
− 1.96 mA = 0.04 mA C
C
1
B 2 Y
29 Base current amplification factor, So, output Y = A ⋅ B ⋅ C C
−3
∆ IC 2 mA 2 × 10 If A = 0, B = 1, C = 1
β= = = = 50 Gate II is AND gate, Y = Y ′ ⋅ C
∆I B 40µA 40 × 10−6 ∴ A = 1, B = 0, C = 1 will give Y = 1
∴ Y = 0⋅ 1 ⋅ 1 = 0 + 1 + 1 = 1
DAY THIRTY SIX ELECTRONIC DEVICES 385

45 We can simplify the gate circuit as 7 I = neA (v e + v h ) = neA (µ e + µ h ) E RL


13 As Av =β
V Ri
A A+B = neA (µ e + µ h )
t  ∆l C ∆l C 
Q g m = ∆V = ∆l R 
I II Y
B = 2 × 1019 × 1.6 × 10−19 × 10−4  B B i
2.5  β  
Here, gates I and II are NOR gates. The × (0.33 + 0.17) × β 
10−3 or G =   RL Qgm = 
output ( A + B ) of gate I will appear as = 0.4 A  i
R  R i
input of gate II. The final output is 8 Solar cell → Open circuit I = 0, Potential ⇒ G = g mR L
Y = A+ B = A+ B V = emf ⇒ G ∝ gm
→ Short circuit I = i , Potential V = 0 G2 g m1
SESSION 2 ∴ =
G1 g m2
9 The four bonding electrons in the case
1 The Boolean expression which satisfies of C lie in the second orbit, whereas in 0.02
the output of this logic gate is C = A ⋅ B , case of Si they lies in the third orbit, so ⇒ G2 = ×G
which is for AND gate. 0.03
loosely bounded valency electron in Si 2
2 According to the question, the figure of as compared to C. ∴ Voltage gain, G2 = G
3
combination of gates in terms of inputs Ie = Ib + Ic
∆ IC
and outputs can be given as 10 We know that a diode only conducts in 14 Current amplification factor, β =
∆I B
A C= A ⋅ B
forward biased condition. In the given
A Collector resistance,
B C
circuit, the diode D1 will be in reverse
B 2V
B
A
Y=C+D bias, so it will block the current and ∆ IC =
D diode D2 will be in forward bias, so it 2 × 103 Ω
A
D= A ⋅ B will pass the current = 1 × 10−3 A
Thus, Y = A⋅B + A⋅B V
i = Base current,
R1 + R3
V VB
3 Due to increase in temperature because 10 ∆I B = B = = V B × 10−3
of heating, thermal collision between = = 2.5 A RB 1 × 103
2+ 2
the electron and holes increases. Thus,
net electron-hole pairs increase. This 11 Input signal of a CE amplifer, Given, β = 100
π 10−3
V in = 2cos  15 t + 
leads to increase in the current in diode
Now, 100 =
and overall resistance of the diode  3 V B × 10−3
changes. This in turn changes both the Voltage gain A v = 150 1
forward biasing and the reverse biasing. VB = V
As CE amplifier gives phase difference 100
Thus, the overall I-V characteristics of of π between input and output signals.
p-n junction diode gets affected. = 10 mV
V
So, A v = 0
4 Since, D1 is in reverse biased, so current V in 15 Collector current iC = V
R
through 5 Ω resistance is zero but D2 is ⇒ V 0 = A vV in 3
π =
forward biased, so D2 behaves as a V 0 = 150 × 2cos  15 t + + π  3 × 103
resistor of zero resistance. The  3 
= 10−3 A
equivalent circuit is  4π 
V = 300cos  15 t +  Now base current,
I 2Ω 3Ω  3 
i 10−3
iB = C = = 10−5 A
12 Applying loop law at output port, B 100
9 − 4 = IC RC As, voltage V in = i B R B
3Ω ∴ V in = 10−5 × 2 × 103
or IC = 2.5 mA
= 2 × 10−2 V
2Ω I I IB =
IC
=
2.5
β 90 So, voltage gain
20 V −5 V 3
= 278
. × 10 A A V = out = = 150
I =
20
⇒I = 2A V in 2 × 10−2
10 = 27.8µA
Since, the transistor operates in active Power gain = A V × β
5 P = X + XY = X (1 + Y ) + XY
region, therefore V BE = 07
. V. = 150 × 100 = 15000
= X + ( X + X )Y
Applying loop law at input port, 16 Given, resistance across load,
= X + 1⋅ Y = X + Y
3 − 07
. R L = 800 Ω
Hence, one gate OR gate. IB =
RB Voltage drop across load, V L = 0.8 V
∆E  1 1 

N − ∆E /2 kT1   Input resistance of circuit, R i = 192 Ω.
6 ∴ 1 = e − ∆E /2 kT = e 2 k  T2 T1 
RB =
. × 10
23 5
Collector current is given by,
N2 e 2
278
. V 0.8 8
1.1eV  1 − 1 
  = 82 k Ω IC = L = = = 1 mA
−5  400 300  RL 800 8000
= e 2 × 8.62 × 10 = e − 5.31
386 40 DAYS ~ NEET PHYSICS DAY THIRTY SIX

Q Current amplification 17 Given, V BE = 0 V, VCE = 0 V 20 = I B × (500 × 103 ) + 0


Output current
= and V i = 20 V ⇒ IB =
20
= 0.04 × 10 −3
Input current 500 × 10 3
IC VCC=20 V
= = 0.96 = 40 × 10−6 = 40 µA …(i)
IB RC=4 kW Similarly, VCC = IC RC + VCE
1 mA =4×103W
⇒ IB = Substituting the given values, we get
0.96 IC 20 = IC × (4 × 103 ) + 0
Q Voltage gain, C
20
V VL IB ⇒ IC = = 5 × 10−3 = 5mA
AV = L = Vi VB 4 × 103
V in I B Ri B
RB=500 kW …(ii)
0.8 × 0.96
= =4 = 500×103W E IC
10−3 × 192 Current gain is given as β =
IB
⇒ AV = 4 Substituting the value of I B and IC from
and Power gain, Eqs. (i) and (ii), we get
2
IC2 R L I  R 5 × 10−3
AP = = C . L ⇒β= = 0.125 × 103 = 125
I 2B R i  I B  Ri Applying Kirchhoff’s law to the 40 × 10−6
800 base-emitter loop, we get
= (0.96) ×
2
18 For using transistor as a switch, it is
192 V i = I B R B + V BE
used in cut-off state and saturation state
AP = 3.84 Substituting the values, we get
only.
DAY THIRTY SEVEN

Unit Test 8
(Electronic Devices)
1 In the figure shown below, which of the diodes are 4 The graph given below represents the I-V characteristics
forward biased? of a zener diode. Which part of the characteristics curve
+5V +10V is most relevant for its operation as a voltage regulator?

R R R I (mA)
Forward bias
+5V –10V Reverse bias
a
(1) (2) (3) VZ
d c b V (volt)
e
–10V R R
I (mA)
(4) –5V (5) –10V
(a) ab (b) bc
(a) 1, 2, 3 (b) 2, 4, 5 (c) 1, 3, 4 (d) 2, 3, 4 (c) cd (d) de
2 The resistance of a germanium junction diode whoseV -I 5 Truth table for system of four NAND gates as shown in
graph is shown in the figure, is (Vk = 0.3 V). figure is

I 10 mA A

Y
Vk 2.3V V

(d) 
10 
(a) 5 kΩ (b) 0.2 kΩ (c) 2.3 kΩ  kΩ
 2.3  B

3 In the figure, the input is across the terminals A and C


A B Y A B Y A B Y A B Y
and the output is across B and D. Then, the output is 0 0 0 0 0 0 0 0 1 0 0 1
B (a) 0 1 1 (b) 0 1 0 (c) 0 1 1 (d) 0 1 0
1 0 1 1 0 1 1 0 0 1 0 0
1 1 0 1 1 1 1 1 0 1 1 1

6 In common-emitter amplifier, the current gain is 62. The


A C
collector resistance and input resistance are 5 k Ω and
500 Ω, respectively. If the input voltage is 0.01 V, the
output voltage is
D (a) 0.62 V (b) 6.2 V
(a) zero (b) same as the input (c) 62 V (d) 620 V
(c) full wave rectified (d) half wave rectified
388 40 DAYS ~ NEET PHYSICS DAY THIRTY SEVEN

7 The reverse saturation of p-n diode 14 The following configuration of gates is equivalent to
(a) depends on doping concentrations A
(b) depends on diffusion length of carriers
B AND
(c) depends on the doping concentrations and diffusion G1
length Y
(d) depends on the doping concentrations, diffusion length G3
and device temperature
8 A p-n-p transistor is said to be in active region of G2

operation, when (a) NAND (b) XOR


(a) both emitter junction and collector junction are forward (c) OR (d) None of these
biased 15 The following circuit represents
(b) both emitter junction and collector junction are reverse
biased A
(c) emitter junction is forward biased and collector junction
is reverse biased Y
(d) emitter junction is reverse biased and collector junction
is forward biased
9 A n -p-n transistor conducts, when B
(a) collector is positive and emitter is at same potential as (a) XNOR gate (b) XOR gate
the base (c) AND gate (d) NAND gate
(b) both collector and emitter are negative with respect to 16 The truth table is given below
the base
(c) both collector and emitter are positive with respect to Inputs Output
the base A B Y
(d) collector is positive and emitter is negative with respect
0 0 0
to the base
1 0 0
10 To get an output Y = 1 from the circuit shown, the inputs
0 1 0
A, B and C must be respectively
1 1 1
A
represents
B Y (a) AND gate
C
(b) NOR gate
(a) 0, 1, 0 (b) 1, 0, 0 (c) OR gate
(d) NAND gate
(c) 1, 0, 1 (d) 1, 1, 0
11 An AND gate is followed by a NOT gate in series. With 17 If internal resistance of cell is negligible, then current
two inputs A and B, the Boolean expression for the flowing through the circuit is
output Y will be 30Ω D2

(a) A + B (b) A ⋅ B
(c) A ⋅ B (d) A + B D1 30Ω
12 In the CB mode of a transistor, when the collector voltage
is changed by 0.5 V. The collector current changes by
20Ω
0.05 mA. The output resistance will be + –
(a) 10 k Ω (b) 20 k Ω 5V
(c) 5 k Ω (d) 2.5 k Ω (a)
3
A (b)
5
A
50 50
13 The current gain of a transistor in common emitter mode 4 2
(c) A (d) A
is 49. The change in collector current and emitter current 50 50
corresponding to the change in base current by 5.0 µA, Ic
will be 18 For a transistor = 0.96, the current gain in
Ie
(a) 245 µA, 250 µA (b) 240 µA, 235 µA
common-emitter configuration is
(c) 260 µA, 255 µA (d) None of the above
(a) 6 (b) 12 (c) 24 (d) 48
DAY THIRTY SEVEN UNIT TEST 8 (ELECTRONIC DEVICES) 389

19 For a given circuit of ideal p-n junction diode, which of 24 The diode used in the circuit shown in the figure, has a
the following is correct ? constant voltage drop of 0.5 V at all currents and a
Diode maximum power rating of 100 mW. What should be the
R
value of the resistor R, connected in series with the diode
for obtaining maximum current?
0.5 V
R
V
(a)In forward biasing the voltage across R is V I
(b)In reverse biasing the voltage across R is V 1.5 V
(c)In forward biasing the voltage across R is 2V
(d)In reverse biasing the voltage across R is 2V
(a) 1.5 Ω (b) 5 Ω
20 Identify the operation performed by the circuit given (c) 6.67 Ω (d) 200 Ω
below 25 In a forward biased p-n junction diode, the potential

A
barrier in the depletion region is of the form
A
V V
Y

B
B (a) (b)

(a) NOT (b) AND (c) OR (d) NAND


p n p n
21 In the following circuit, the outputY becomes zero for the
V V
inputs
A
1 (c) (d)
B
3 Y
p n p n
C 2

(a) A = 1, B = 0, C = 0 (b) A = 0, B = 1, C = 1 26 An n-p-n transistor circuit is arranged as shown in figure.


(c) A = 0, B = 0, C = 0 (d) A = 1, B = 1, C = 1 It is

22 The inputs and outputs for different time intervals are


n
given below the NAND gate. p RL
n Vout
Time Input A Input B Output Y Vin
t 1 to t 2 0 1 P
t 2 to t 3 0 0 Q
t 3 to t 4 1 0 R
(a) a common base amplifier circuit
t 4 to t 5 1 1 S
(b) a common emitter amplifier circuit
The values taken by P , Q , R , S are respectively (c) a common collector amplifier circuit
(d) None of the above
(a) 1, 1, 1, 0 (b) 0, 1, 0, 1
(c) 0, 1, 0, 0 (d) 1, 0, 1, 1 27 The junction diode in the R 0.7 V
(e) 1, 0, 1, 0 following circuit requires a
23 In the figure, potential difference between A and B is minimum current of 1 mA to be
above the knee point (0.7 V) of its
10 kΩ A
I-V characteristic curve. The
voltage across the junction diode VB
30 V 10 kΩ 10 kΩ is independent of current above
the knee point, if VB = 4 V, then the maximum value of R,
B so that the voltage is above knee point will be
(a) zero (b) 5 V (c) 10 V (d) 15 V
(a) 3.3 kΩ (b) 4.0 kΩ (c) 4.7 kΩ (d) 6.6 kΩ
390 40 DAYS ~ NEET PHYSICS DAY THIRTY SEVEN

28 The circuit shown in the figure 70 Ω 33 The ratio of electron and hole current in a semiconductor
contains two diodes, each with a is 7/4 and the ratio of drift velocities of electrons and
forward resistance of 30 Ω and 70 Ω holes is 5/4, then ratio of concentrations of electrons and
with infinite backward resistance. holes will be
If the battery is 3 V, the current + 50 Ω
(a) 5/7 (b) 7/5 (c) 25/49 (d) 49/25
(in ampere) through the 50 Ω 3V 34 In a junction diode, the direction of diffusion current is
resistance is
(a) from n-region to p-region
(a) zero (b) 0.01 (c) 0.02 (d) 0.03
(b) from p-region to n-region
29 Which of the following is not equal to 1 in Boolean (c) from n-region to p-region, if the junction is forward
algebra? biased and vice versa, if it is reverse biased
(a) A ⋅ A (b) A ⋅ A (d) from p-region to n-region if the junction is forward
biased and vice versa, if it is reverse biased
(c) A + A (d) A + 1
30 The output Y of the logic circuit shown in figure, is best 35 The direction of flow of current in the given diagram is
represented as +5 V 200Ω +3 V
A
B Y (a) zero (b) 10−2 A (c) 10 A (d) 0.025 A
III
C II
I Direction (Q. Nos. 36-41) In each of the following questions
(a) A + B ⋅ C (b) A + B ⋅ C a statement of Assertion is given followed by a corresponding
(c) A + B ⋅ C (d) A + B ⋅ C statement of Reason just below it. Of the statements mark the
correct answer as
31 A p-n junction diode shown in the figure can act as a rectifier. (a) If both Assertion and Reason are true and the Reason is
An alternating current source (V ) is connected in the circuit. the correct explanation of the Assertion
The current (I ) in the resistor (R ) can be shown by (b) If both Assertion and Reason are true but the Reason is
not correct explanation of the Assertion
(c) If Assertion is true but Reason is false
(d) If both Assertion and Reason are false
V R
36 Assertion (A) In a common emitter transistor amplifier,
the input current is much less than the output current.
Reason (R) The common emitter transistor amplifier has
I I
very high input impedance.
(a) (b)
37 Assertion (A) The logic gate NOT can be built using
t t diode.
Reason (R) The output voltage and the input voltage of
I I
the diode have 180° phase difference.
(c) (d)
38 Assertion (A) A transistor amplifier in common emitter
t t configuration, has a low input impedance.
Reason (R) The base to emitter region is forward biased.

32 In the circuit as shown in figure, A and B represent two 39 Assertion (A) NAND or NOR gates are called digital
inputs and C represents the building blocks.
A Reason (R) The repeated use of NAND or NOR gate can
produce all the basic or completed gates.
C
R
40 Assertion (A) The resistivity of a semiconductor
B
increases with temperature.
Reason (R) The atoms of a semiconductor vibrate with
(a) OR gate (b) NOR gate larger amplitudes at higher temperatures thereby
(c) AND gate (d) NAND gate increasing its resistivity.
DAY THIRTY SEVEN UNIT TEST 8 (ELECTRONIC DEVICES) 391

ANSWERS
1. (b) 2. (b) 3. (c) 4. (d) 5. (a) 6. (b) 7. (d) 8. (c) 9. (d) 10. (c)
11. (b) 12. (a) 13. (a) 14. (b) 15. (a) 16. (a) 17. (b) 18. (c) 19. (a) 20. (b)
21. (d) 22. (a) 23. (c) 24. (b) 25. (d) 26. (b) 27. (a) 28. (c) 29. (b) 30. (d)
31. (b) 32. (a) 33. (b) 34. (b) 35. (b) 36. (c) 37. (d) 38. (a) 39. (a) 40. (d)

Hints and Explanations


1 A p-n junction diode is said to be Y ′ is the input for NOT gate
forward biased, if the positive terminal A B A B A⋅B B⋅ A Y ∴ Y = Y′
of the external battery is connected to
p-side and the negative terminal to the ⇒ Y = A⋅B
0 0 1 1 0 0 0
n-side of the junction.
0 1 1 0 0 1 1 12 Here, ∆VC = 0.5 V
Thus, 2, 4 and 5 are forward biased.
∆IC = 0.05 mA = 0.05 × 10−3 A
2 R = ∆V = 2.3 − 0.3 =
2
× 103 1 0 0 1 1 0 1
∆I 10 × 10−3 10 Output resistance is given by
1 1 0 0 0 0 0 ∆VC
= 0.2 × 103 Ω = 0.2 kΩ Rout =
It is the truth table for XOR gate. ∆ IC
3 The given figure represents a full wave
6 Voltage gain = resistance gain × current 0.5
rectifier, which is also known as bridge = = 104 Ω = 10 kΩ
rectifier, which uses four diodes. gain 0.05 × 10−3
V out R
4 When reverse bias is increased, then the or = out × β ∆ IC
V in R in 13 Current gain, β = , where ∆IC is
electric field across the junction also ∆I B
increases. At some stages, the electric R out
⇒ V out = × β × V in change in collector current and ∆I B is
field becomes so high that it breaks the R in
covalent bonds, creating electron-hole 5000 change in base current.
∴ V out = × 62 × 0.01 = 6.2 V ∴∆IC = β∆I B = 49 × 5 = 245 µ A
pairs. This mechanism is known as 500
Zener breakdown. Also, ∆ I E = ∆ I B + ∆ IC
7. The reverse saturation of p-n diode
In breakdown region for a long range of
depends on the doping concentrations, = (245 + 5)µA
load (R L ), the voltage remains the same = 250 µA
diffusion length and device
though the current may be large. So,
temperature. 14 The given output equation can be
option (d) is correct.
8 For amplification of a p-n-p transistor, written as
5 Output, Y = ( A ⋅ ( A ⋅ B )) ⋅ (B ⋅( A ⋅ B ))
emitter junction is forward biased and
collector junction is reverse biased. In A (A + B)
this state, p-n-p transistor is said to be B
A(AB) in active region of operation. G1
A Y
9 In an n-p-n transistor, the emitter-base A
G3
junction is forward biased. However, A .B
AB B G
y collector-base junction is reverse biased. 2

10 For the given combination,


Y = ( A + B )⋅ C Y = ( A + B ) ⋅ AB
B Y =1
B(AB) Y = ( A + B )⋅ ( A + B ) [QUsing
If A =1
B =0 de Morgan’s theorem]
= ( A + A ⋅ B ) ⋅ (B + A ⋅ B ) C =1 = AA + AB + BA + BB

= ( A + A ⋅ B ) + (B + A ⋅ B ) 11 The inputs for AND gate give output = 0 + AB + AB + 0


= A ⋅ (A ⋅ B) + B ⋅ (A ⋅ B) Y′ = A⋅ B = AB + AB
= A ⋅ (A + B) + B⋅ (A + B) This boolean expression for XOR gate.
A Y' = A.B
= A⋅B + B⋅ A Y = Y' = A.B
B
392 40 DAYS ~ NEET PHYSICS DAY THIRTY SEVEN

15 The given output equation can be 20 The output of the circuit is consequence of this is the effective
written as barrier potential reduces. Hence, the
Y = A + B = A⋅B
graph (d) is correctly shown.
A
A A·B = A ⋅ B [Q A = A and B = B ]
26 Here, emitter is forward biased, and is
which is the output of an AND gate. common between input and output
21 Output of gate-1, Y1 = A ⋅ B circuit. Thus, the circuit is of n-p-n
Y
transistor with a common emitter
A amplifier mode.
1
B
A·B B 27 As, V B = V knee + IR
B
3 Y or 4 = 07. + 10−3 R
Y = A ⋅B ⋅ A⋅B 3.3
C 2 or R = = 3.3kΩ
= (A + B)⋅ (A + B) 10−3
(Using de Morgan's theorem)
Output of gate 2, Y2 = C 28 In the circuit, the upper diode D1 is
= A A + B B + BA + A B
Output of gate 3, Y = Y1 ⋅ Y2 = ( A ⋅ B ) ⋅ C reverse biased and the lower diode D2 is
= AB + A B forward biased. Thus, there will be no
Thus, from the given options output
This Boolean expression for XNOR gate. will be zero, if A = 1, B = 1 and C = 1 current across upper diode junction.
The effective circuit will be as shown in
16 We can see from the truth table that 22 NAND gate is obtained when the output
output is 1, only when its both inputs figure.
of AND gate is made as the input of
are 1. This is possible only for AND NOT gate. Boolean expression for 70Ω
gate. The Boolean expression for AND NAND gate is
gate is Y = A ⋅ B , which satisfies the
truth table as given below. A
0⋅ 0 = 0 Y
B 3V
1⋅ 0 = 0 + – 50Ω
Y = A⋅B
0⋅1 = 0
Total resistance of circuit,
1⋅1 = 1 Input (A) Input (B) Output (Y) R = 50 + 70 + 30 = 150 Ω
Here, symbol (⋅) represents AND 0 0 1 Current in circuit,
operation. V
1 0 1 I =
17 In the circuit, diode D1 is forward R
0 1 1 3
biased and diode D2 is reverse biased. = = 0.02 A
1 1 0 150
Therefore, no current flows in the arm
containing D2 and all of the current 23 Here, p-n junction is forward biased. If 29 (a) A ⋅ A = A + A = A + A = 1
flows through arm containing D1 . p-n junction is ideal, its resistance is (b) A ⋅ A = 0
zero. The effective resistance across A (c) A + A =1
Thus, current flowing through the
and B
circuit =
5
=
5
A 10 × 10 (d) A + 1 = 1
20 + 30 50 = = 5 kΩ Hence, all the options give the output 1
10 + 10
except option (b).
18 Given, Ic = Current gain (α ) = 0.96 Current in the circuit,
Ie V 30 2 30
I = = = A
So, current gain in common emitter R 15 × 103 103
A Y3
configuration is 2
α Current in arm, AB = I = B
B 2 Y2
3
β= 10 1
1−α Potential difference across A and B
C Y1
0.96 0.96 2
= = = 24 = × 5 × 103 = 10 V The output at gate 1, Y1 = B ⋅ C
1 − 0.96 0.04 103
Output at gate 2, Y2 = A + B ⋅ C
19 In forward biasing, the diode conducts. 24 Current through circuit, Final output, Y3 = Y 2 = A + B ⋅ C
For ideal junction diode, the forward P 100 × 10−3
resistance is zero. Therefore, entire I = = = 0.2 V
V 0.5 31 The given circuit represents a circuit of
applied voltage occurs across resistance
Voltage drop across, half wave rectifier. In this, during the
R, i.e. there occurs no voltage drop.
R = 1.5 − 0.5 = 1.0 V positive half cycle of input, p-n junction
Thus, voltage across R is V in forward
1 is forward biased. Hence, current flows
biasing. Hence, R = = 5Ω
0.2 in the load. But in the negative half
While in reverse biasing, the diode does
cycle of input, p-n junction is reverse
not conduct, so it has infinite 25 In forward biased p -n junction, the
biased. Thus, no current flows. So,
resistance. Thus, voltage across R is applied potential is opposite to the
zero in reverse biasing. option (b) is correct.
junction barrier potential V B . The
DAY THIRTY SEVEN UNIT TEST 8 (ELECTRONIC DEVICES) 393

32 The circuit represents OR gate, as the 35 Here, p-n junction as forward biased with 38 As base to emitter region is forward
output at C is 1, when either A or B or voltage = 5 − 3 = 2 V biased. Hence, it will work as simple
both A and B have input at level 1. But 2 forward biased diode and resistance is
∴ Current I =
output at C is zero, when both A and B 200 low.
are at zero level, since for OR gate, the 1
= = 10−2 A 39 NAND or NOR gates are called
Boolean expression is C = ( A + B ). 100 universal (digital) building blocks
33 As, I = Anev d or I ∝ nv d 36 The common emitter transistor amplifier because using repeated order of these
has input resistance equal to 1 k Ω two types of gates we can produce all
Ie n v
∴ = e e (approx.), and output resistance equal to 10 the basic gates namely OR, AND or
Ih nh v h complex gates.
kΩ (approx.). The output current in CE
ne I v 7 4 7
or = e × h = × = amplifier is much larger than the input 40 Resistivity of a semiconductor
nh Ih ve 4 5 5 current. Hence, option (c) is correct. decreases with the temperature. The
37 NOT gate inverts the signal applied to it. atoms of a semiconductor vibrate with
34 In a p-n junction, the direction of
But in diode, the input and output are in larger amplitudes at higher
diffusion current is from p-region to
same phase. Thus, NOT gate cannot be temperature, thereby increasing its
n-region only.
built by a diode. Hence, option (d) is conductivity, not resistivity.
correct.
DAY THIRTY EIGHT

Mock Test 1
Instruction
l This question paper contains of 50 Multiple Choice Questions of Physics, divided into two Sections;
section A and section B.
l Section A contains 35 questions and all questions are compulsory.
l Section B contains 15 questions out of which only 10 questions are to be attempted.
l Each question carries 4 marks.

Section-A The coefficient of friction between the block and the


incline in same and equal to 0.1. Find the normal contact
1 The method of dimensional analysis can be used to
force exerted by 2 kg block on 3 kg block.
derive, which of the following relations?
F
(a) N = N 0e − λt (b) y = A sin (ωt + k x)
1 1
(c) E = mv 2 + Iω2 (d) None of these
2 2 2 kg
3 kg
2 The angle between A = $i + $j and B = $i − $j is
Fixed
(a) 45° (b) 90° incline
37°
(c) − 45 ° (d) 180°
3 A body starts from rest and moves with constant (a) 18 N (b) 30 N
acceleration. The ratio of distance covered by the body (c) 12 N (d) 27.6 N
in n th second to that covered in n second, is
6 Figure shows the orientations of two vectors u and v in
1 2n − 1
(a) (b) the XY plane. If u = a $i + b $j and v = p $i + q $j
n n2
Which of the following is correct?
n2 2n − 1
(c) (d)
2n − 1 2n2 Y

4 A passenger in a train drops a ball from the window of a


train running at an acceleration a. A pedestrian on the v
u
ground by the side of the rails observes the ball falling
along
X
(a) a vertical path with an acceleration g 2 + a 2 O

(b) a vertical path with an acceleration g 2 − a 2 (a) a and p are positive while b and q are negative
(b) a, p and b are positive while q is negative
(c) a parabola with an acceleration g 2 + a 2
(c) a, q and b are positive while p is negative
(d) a parabola with an acceleration g
(d) All a, b, p and q are positive
5 Two blocks of masses 3 kg and 2 kg are placed
side by side on an incline as shown in the figure. A 7 A body is projected from A at an angle of 90° with the
force F = 20 N is acting on 2 kg block along the plane AB. It again touches the plane at B after time T .
incline. Then, what is the length AB?
DAY THIRTY EIGHT MOCK TEST 1 395

u 14 Match the Column I (planar loops of different shapes)


with Column II (direction of induced current) and select
the correct answer from the codes given below.
90° A
Column I Column II
θ
B A. d 1. b a c b
× × × × ×
(a)Tu sin θ (b)Tu cosθ a × × × × ×
(c)Tu tanθ (d)Tu cotθ × × c × ×
× × × ×
8 A rigid body rotates about a fixed axis with variable b
× × × ×
angular velocity equal to α − βt at time t, where α and β × × × × ×
are constants. Find the angle through which it rotates B. × × × × 2. c d a b c
× × a ×
before it comes to stop. × × × ×
c
α2 α2 − β2 × × × ×
(a) (b) × × b ×
2β 2α × × × ×
α −β2 2
(α − β) α C. 3. b c d a b
(c) (d) × × × × ×
2β 2 a
× × × × ×
× × × b ×
9 Satellites orbiting the earth have finite life and sometimes × × × ×
c
debris of satellites fall to the earth. This is because d × × × × ×
× × × × ×
(a) the solar cells and batteries in satellites out
(b) the laws of gravitation predict a trajectory spiralling Codes
inwards A B C A B C
(a) 3 1 2 (b) 2 1 3
(c) of various forces causing the speed of satellite and (c) 1 2 3 (d) 1 3 2
hence height to gradually decrease
(d) of collisions with other satellites 15 A point source emits sound equally in all directions, in a
non-absorbing medium. Two points P and Q are at a
10 A particle performs simple harmonic motion with distance of 9 m and 25 m, respectively from the source.
amplitude A and period T . The mean velocity of the The ratio of the amplitude of the waves at P and Q is
particle over the time interval during which it travels a 3 5 9 25
A (a) (b) (c) (d)
distance of starting from extreme position is 5 3 25 9
2
A 2A 3A A 16 If the critical angle for the medium of a prism is C and
(a) (b) (c) (d)
T T T 2T the angle of prism is A, then there will be no emergent
ray, when
11 The total kinetic energy of all the molecules of helium (a) A < 2 C (b) A = 2 C (c) A > 2 C (d) A ≥ 2 C
having a volume V exerting a pressure p is 1500 J. The
total KE (in joule) of all the molecules of N 2 having the 17 Two points separated by a distance of 0.1 mm can just
same volume V and exerting a pressure 2 p, is be inspected in a microscope, when a light of
wavelength 6000 Å is used. If light of wavelength 4800 Å
(a) 3000 (b) 4000 (c) 5000 (d) 6000
is used, this limit of resolution will become
12 500g of water and 100g of ice at 0°C are in a calorimeter (a) 0.8 mm (b) 0.12 mm (c) 0.10 mm (d) 0.08 mm
whose water equivalent is 40 g. 10g of steam at 100°C is
added to it. Then, water in the calorimeter is
18 A small metallic ball is charged positively and negatively
in a sinusoidal manner at a frequency of106 cps. The
(Latent heat of ice = 80 cal/g, Latent heat of steam maximum change on the ball is10− 6C. What is the
= 540 cal/g) maximum value of displacement current due to this
(a) 578 g (b) 590 g alternating current?
(c) 600 g (d) 610 g (a) 6.28 A (b) 3.4 A (c) 3.75 × 10− 4 A (d) 12.56 A
13 A metal ball of surface area 200 cm 2 and at temperature 19 A capacitor of capacitance C1 = 1.0 µF withstands a
527°C, is surrounded by a vessel at 27°C. If the maximum voltage V1 = 6.0 kV while a capacitor of
emissivity of the metal is 0.4, then the rate of loss of heat capacitance C2 = 2.0 µF withstands the maximum
 J  voltage V2 = 4.0 kV. What maximum voltage will the
from the ball is approximately  σ = 5 .67 ×10 − 8 2 4 
 m sK  system of these capacitors withstand, if they are
connected in series?
(a) 108 Js −1 (b) 168 Js −1
(c) 182 Js −1 (d) 192 Js −1 (a) 10 kV (b) 9 kV (c) 12 kV (d) 15 kV
396 40 DAYS ~ NEET PHYSICS DAY THIRTY EIGHT

20 In the shown circuit the reading of ammeter is same 26 In an L-R circuit, the AC source has voltage 220 V. The
when both the switches are open, and when both the potential difference across the inductance is 176 V. The
switches are closed. Assume the ammeter to be ideal, potential difference across the resistance will be
the value of R satisfying above condition is (a) (220 − 176) V
100Ω (b) (220 + 176) V
A
(c) 220 × 176 V
R (d) (220) 2 − (176) 2 V
50Ω
27 A copper wire of length 1.0 m and a steel wire of length
0.5 m having equal cross-sectional areas are joined end
to end. The composite wire is stretched by a certain load
300Ω 1.5V
which stretches the copper wire by 1 mm. If the Young’s
(a) 100 Ω (b) 400 Ω (c) 600 Ω (d) 440 Ω modulus of copper and steel are respectively
21 The power and voltage rating of a heater is 1000 W and 1. 0 × 1011Nm –2 and 2 . 0 × 1011Nm –2 , the total extension of
100 V, respectively. What should be the value of R in the composite wire is
the given circuit, so that heater operates with power of (a) 1.75 mm (b) 2.0 mm
62.5 W? (c) 1.50 mm (d) 1.25 mm

10Ω IH
28 The earth’s magnetic field lines resemble that of a dipole
I
Heater at the centre of the earth. If the magnetic moment of this
R dipole is close to 8 × 1022 A -m 2 , the value of earth’s
IR magnetic field near the equator is close to (radius of the
100V earth = 6 . 4 × 106 m)
(a) 10 Ω (b) 7 .5 Ω (c) 5 Ω (d) 2.5 Ω (a) 0.6 gauss (b) 1.2 gauss
(c) 1.8 gauss (d) 0.32 gauss
22 It takes 16 min to boil some water in an electric kettle.
Due to some defect, it becomes necessary to remove 29 The radioactivity of a sample is A1 at a time t1 and A 2 at
10% turns of the heating coil of the kettle. After repairs, time t 2 . If mean life of specimen is T, the number of
how much time will it take to boil the same mass of atoms that have disintegrated in the time interval of
water? t 2 − t1 is
(a) 17.7 min (b) 14.4 min A1 − A2
(a) A1 − A2 (b)
(c) 20.9 min (d) 13.7 min T
(c) (A1 − A2 ) T (d) A1t1 − A2 t 2
23 A long wire carries a current of 20 A along the axis of
solenoid. The field due to the solenoid is 4 mT. The 30 The human eye can barely detect a yellow light 6000 Å
resultant field at a point 3 mm from the solenoid axis . × 10− 18 W to the retina. Nearly, how many
that delivers 17
inside solenoid is photons per second does the retina receive?
(a) 1.33 mT (b) 4.2 mT (a) 50 (b) 5
(c) 5.33 mT (d) 2.87 mT (c) 500 (d) More than 5 million
24 A mass of 50g of water in a closed vessel, with 31 According to Bohr’s theory (assuming infinite mass of in
surroundings at a constant temperature takes 2 min to
nucleus), the frequency of the second line of the Balmer
cool from 30°C to 25°C. A mass of 100g of another liquid
series is
in an identical vessel with identical surroundings takes
the same time to cool from 30°C to 25°C. The specific (a) 6.16 × 1014 Hz
heat of the liquid is (The water equivalent of the vessel is (b) 6.16 × 2010 Hz
30 g) (c) 6.16 × 1013 Hz
(a) 2.0 kcal/kg (b) 4 kcal/kg (d) 6.16 × 1016 Hz
(c) 3 kcal/kg (d) 0.5 kcal/kg 32 Select the correct statement from the following.
25 A 50 Hz, AC current of crest value 2 A flows through the (a) Electromagnetic waves cannot travel in vacuum
primary of a transformer. If the mutual inductance (b) Electromagnetic waves are longitudinal waves
between the primary and secondary is 0.25 H, the crest (c) Electromagnetic waves are produced by charges
voltage induced in the secondary is moving with uniform velocity
(a) 50 V (b) 100 V (c) 200 V (d) 300 V (d) Electromagnetic waves carry both energy and
momentum as they propagate through space
DAY THIRTY EIGHT MOCK TEST 1 397

33 In the network shown, 40 The velocity of a particle in SHM varies with time as
D1 v (teq ) = (sin 6 t − cos 6 t ) ms − 1
4Ω
The maximum acceleration of the particle in ms − 2 is
(a) 12 2 (b) 6 2
D2 (c) 6 (d) 12
41 One mole of an ideal diatomic gas is heated from 20°C
+ – to 60°C at a constant pressure of 1 atm. The
5V
change in internal energy of the gas is nearby (R = 8 . 31
(a) the potential difference across D2 is 5 V
(b) current through resistor equals 2.25 A Jmol −1K −1)
(c) current through diode D1 and resistance is 1.25 A (a) 561 J (b) 712 J
(d) current through diode D2 and resistance is 1.25 A (c) 831 J (d) 1013 J

34 The work done in blowing a soap bubble of surface 42 The frequency of a note emitted by a source changes by
tension 0.06 Nm − 1
from 2 cm radius to 5 cm radius is 20% as it approaches an observer. As it recedes away
(a) 0.004168 J (b) 0.003168 J from him, the apparent frequency will differ from the
(c) 0.003158 J (d) 0.004568 J actual frequency by
35 A body is dropped from a height h. If t1 and t 2 be the (a) 20% (b) 16.67%
times in covering first half and the next half distances (c) 14. 3% (d) 19.4%
respectively, then the relation between t1 and t 2 is 43 A spherical droplet having a potential of 2.5 V is obtained
t2 as a result of merging of 125 identical droplets. Find the
(a) t1 = t 2 (b) t1 = 2 t 2 (c) t1 = 3 t 2 (d) t1 =
( 2 − 1) potential of constituent droplet.
(a) 0.4 V (b) 0.5 V (c) 0.2 V (d) 0.1 V
Section-B
44 Charge q of mass m is placed in a uniform electric field
36 Eight equal drops of water are falling through air with a E , then kinetic energy after time t
steady velocity of 10 cms − 1. If the drops combine to form qEt qE 2t 2 2m q 2E 2t 2
a single drop big in size, then the terminal velocity of this (a) (b) (c) (d)
2m 2m q 2E 2t 2 2m
big drop is
(a) 80 cms −1 (b) 30 cms −1 45 A system of four gates is set up as shown. The ‘truth
(c) 10 cms −1 (d) 40 cms −1 table’ corresponding to this system is
37 Two rings of the same radius and mass are placed, such A
that their centres are at a common point and their planes Y1
II
are perpendicular to each other. The moment of inertia of
the system about an axis passing through the centre and
Y′
perpendicular to the plane of one of the rings is I IV Y
(mass of ring = m and radius = r )
1 Y2
(a) mr 2 (b) mr 2 III
2
3 B
(c) mr 2 (d) 2 mr 2
2
(a) Input Output
38 Statement I Kinetic friction is used in stopping relative
motion by using brakes in machines and automobiles. A B Y
Statement II We are able to walk because of static 0 0 1
friction. 0 1 0
(a) Both statement I and statement II are true. 1 0 0
(b) Both statement I and statement II are false. 1 1 1
(c) Statement I is true but statement II is false. (b)
Input Output
(d) Statement I is false but statement II is true.
A B Y
39. The ratio of the kinetic energy required to be given to the 0 0 0
satellite to escape the earth’s gravitational field to the
0 1 0
kinetic energy required to be given, so that the satellite
1 0 1
moves in circular orbit just above earth’s atmosphere, is
(a) one (b) two (c) half (d) infinity 1 1 1
398 40 DAYS ~ NEET PHYSICS DAY THIRTY EIGHT

(c) 48 Match the following columns and choose the correct


Input Output code.
A B Y
0 0 1 Column II
Column I
0 1 1 (Their moment of
(Rotation of different bodies)
inertia)
1 0 1
A. Thin circular ring of radius R 1. MR 2 /2
1 1 0
having axis perpendicular to the
(d) plane and passing through units
Input Output B. Thin circular ring of radius R 2. ML2 /12
A B Y having axis passing through its
0 0 0 diameter
0 1 1 C. Thin rod of length L about an 3. MR 2
1 0 1 axis perpendicular to the rod
and passing through mid-point
1 1 0
D. Circular disc of radius R about 4. MR 2 / 4
46 Statement I For higher temperatures, the peak emission an axis perpendicular to the
disc and passing through the
wavelength of a black body shifts to lower wavelengths.
centre.
Statement II Peak emission wavelength of a black body
is proportional to the four power of temperature. Codes
(a) Both statement I and statement II are true. A B C D A B C D
(a) 3 2 2 1 (b) 2 3 1 2
(b) Both statement I and statement II are false.
(c) 3 1 2 1 (d) 3 1 2 4
(c) Statement I is true but statement II is false.
(d) Statement I is false but statement II is true.
49 Two magnets, the magnetic moment of one being twice
that of other, oscillate in a magnetic field first with like
47 Assertion For insulators and semiconductors, number of poles tied together and then with unlike poles tied
electrons increases with increasing temperature. together. The ratio of the time periods in the two cases, is
Reason This increase of number of electrons is more (a) 1 : 2 (b) 1 : 3
effective than any decrease in τ, so that for such (c) 1 : 2 (d) 1 : 1
materials ρ decreases with temperature.
50 When an electron in hydrogen atom is excited, from its
(a) Both Assertion and Reason are true and Reason is the
4th to 5th stationary orbit, the change in angular
correct explanation of Assertion.
momentum of electron is (Planck’s constant
(b) Both Assertion and Reason are true but Reason is not
h = 6.6 × 10 −34 J-s)
the correct explanation of Assertion.
(a) 4.16 × 10−34 J-s (b) 3.32 × 10−34 J-s
(c) Assertion is true but Reason is false.
(c) 1.05 × 10−34 J-s (d) 2.08 × 10−34 J-s
(d) Assertion is false but Reason is true.

Answers
1 (d) 2 (b) 3 (b) 4 (d) 5 (c) 6 (b) 7 (c) 8 (a) 9 (c) 10 (c)
11 (a) 12 (a) 13 (c) 14 (a) 15 (d) 16 (c) 17 (d) 18 (a) 19 (b) 20 (c)
21 (c) 22 (b) 23 (b) 24 (d) 25 (b) 26 (d) 27 (d) 28 (d) 29 (c) 30 (b)
31 (a) 32 (d) 33 (c) 34 (b) 35 (d) 36 (d) 37 (c) 38 (a) 39 (a) 40 (b)
41 (c) 42 (c) 43 (d) 44 (d) 45 (a) 46 (c) 47 (b) 48 (c) 49 (b) 50 (c)
DAY THIRTY EIGHT MOCK TEST 1 399

Hints and Explanations


1 Out of given choices, (a) is exponential where, f1 = µ × 3g cos 37 ° The viscous force acting on satellite
46 decreases the energy of satellite. As a
equation and (b) is trigonometric and f2 = µ × 2g cos 37 °⇒ a = ms −2
function and we know that, 5 result of it, the value of speed gradually
dimensional analysis fails to derive For 3 kg block, decreases, consequently the height of
physical relations involving N + 3 g sin 37 ° − f1 = 3 a ⇒ N = 12 N satellite gradually decreases.
logarithmic, trigonometric and 10 Let the equation of simple harmonic
exponential functions. Now, choice (c) 6 As per figure in u = a $i + b $j , both a and
motion be
is the equation having two terms added b are positive. In v = p i$ + q $j
x = A cos ωt , v = − Aω sin ωt ,
by an operation ‘+’, it also cannot be = pi$ + ( −q )( − $j ) . So, p is positive and q 2π
derived by dimensional analysis, so where ω =
is negative. Thus, a , b and p are positive T
choice (d) is the answer.
and q is negative. T
And time interval =
2 ∴ cos θ = A ⋅ B 6
AB 7 Time of flight, T = 2 u
g cos θ Mean velocity over the required interval
( i$ + $j ) ⋅ ( i$ − $j ) is
= We know that, range T/6
1 + 12 × 12 + ( −1 )2
∫0
2
2 u 2 sin (α − β ) cos α vdt
1 −1 = vmean =
= = 0 = cos 90 ° ⇒ θ = 90 ° g cos2 β T /6
2 T/6

3 The distance covered by the body in nth


u u cos θ
vmean =
∫0 − Aω sin ωt dt
θ T /6
second, θ
1 6A 3A
sn = u + a (2 n − 1 ) = × [cos ω t ] T/6
0
=
2 T T
g g cos θ
Here, the body starts from rest, so u = 0.
1 11 Kinetic energy per unit volume for a gas
∴ s n = a (2 n − 1 ) ...(i) θ E 3
2 = p
B V 2
Distance covered by the body in n
second, Here, α − β = 90 ° {β = θ} where, p is the pressure applied by the
1 2 u 2 sin 90 ° cos (90 ° + θ) gas.
s = un + an 2 ⇒ AB =
2 g cos2 θ Here, both the gases have same volume, so
1
Again, u = 0, s = an 2 ...(ii) 2 u 2 cos (90 ° + θ) E∝p
2 or AB =
1 g cos2 θ E He p
= He
a (2 n − 1 )
s 2u sin θ E N2 p N2
Required ratio = n = 2 or AB = ⋅u = Tu tan θ
s 1
an 2 g cos θ cos θ 1500 p
⇒ =
2 E N2 2p
2n − 1 8 ∴ Angular velocity, ω = α − βt
= ⇒ E N2 = 3000 J
n2 dθ
∴ = ( α − βt )
dt 12 As latent heat of steam goes to melt the
4 When a ball is dropped from the
dθ = (α − βt ) dt ice, 540 × 10 = m × 80
window of a train, then acceleration due
α 540
to gravity g (in vertically downward When ω = 0, then t = ∴ m = = 67. 5 = 68 g
direction) acts on it, so it moves on a β 8
θ α/ β Now, amount of water
parabola with acceleration g (projectile
situation).
∴ ∫ 0 dθ = ∫ 0 (α − βt ) dt
= 500 + 68 + 10 = 578 g
α/ β
5 The free body diagram of two blocks are  t2 
θ = α [ t ] 0α / β − β   13 The rate of loss of heat from the ball,
shown in figure.  2 0 E = Aeσ ( T 4 − T04 )
a  α  β  α2  α2 α2 α2 Here, A = 200 cm 2 = 200 × 10 − 4 m 2 ,
F =α   −  = − =
 β  2  β2  β 2β 2β J
N 2 kg
f2 e = 0 .4, σ = 5 .67 × 10 − 8
a m 2sK4
N 9 Orbital velocity of satellite at distance r
2g sin 37° T = 527 ° C = 527 + 273 = 800 K
3 kg GM
f1 from the centre of earth is, v = . T0 = 27 ° C = 27 + 273 = 300 K
r ∴ E = 200 × 10 − 4 × 0.4 × 5 . 67 × 10 − 8
3g sin 37° 37°
Total energy of satellite, × {(800 )4 − (300 )4 }
GM m 1
Under the assumption that both blocks TE = PE + KE = − + mv2 = 200 × 10 −4 × 0.4 × 5 . 67 {4096 − 81}
is moving together, r 2
=−
GM m
+ m
1 GM
=−
GMm = 0.8 × 5 . 67 × 4015 × 10 − 2
F + 2g sin 37 ° + 3 g sin 37 ° − f1 − f2
r 2 r 2r = 182 Js −1
= 5a
400 40 DAYS ~ NEET PHYSICS DAY THIRTY EIGHT

d1 λ Ammeter reading is
14 A. The magnetic flux through the = 1
rectangular loop abcd increases, d2 λ2 I ×R 1 .5 R
I2 = 0 =
due to the motion of the loop into 0.1 6000 0.1 5 100 + R 300 (100 + R ) + 100 R
= ⇒ =
the region of magnetic field. Thus, d2 4800 d2 4 As, I1 = I2 which gives R = 600 Ω.
the induced current must flow
0.4
along the path bcdab, so that it ⇒ d2 = = 0.08 mm 21 Resistance of heater is given by
opposes the increasing flux. 5
V2 100 × 100
B. Due to the outward motion, 18 Here, the ball is charged in sinusoidal RH = = = 10 Ω
magnetic flux through the triangular P 1000
manner, so charge on the ball at any
loop abc decreases. Thus, the instant, q = q 0 sin ω t If heater dissipates 62.5 W, then voltage
induced current must flow along where, q 0 = maximum charge. across, it is given by
bacb, so as to oppose the change in
flux. Displacement current, ( V ′ ) 2 = 62 .5 × R H = 62 .5 × 10
dq d
C. As the magnetic flux decreases due Id = = [q 0 sin ωt ] ⇒ V′ = 25 V
to the motion of the irregular shaped dt dt
The remaining 75 V should drop across
loop abcd out of the region of = q 0 cos ω t . ω = q 0 ω cos ωt
10 Ω resistance. In other words, current
magnetic field, the induced current For maximum displacement current, I in the circuit should be 7.5 A. It is
flows along cdabc, so as to oppose
cos ωt = 1 clear from the circuit that
the change in flux.
∴ ( Id ) max = q 0 ω V
I = I H + I R or I = H + R
V
Hence, A → 3, B → 1 and C → 2.
= 10 − 6 × 2 π × 106 RH R
15 ∴ Intensity of sound, I ∝ 12 = 2 π = 6.28 A Substituting,
r
I = 7 .5 A , VH = VR = 25 V and
IP (25 ) 2 19 Maximum amount of charge hold by R H = 10 Ω, we get R = 5 Ω
∴ = and amplitude2 ∝ intensity
IQ (9 ) 2 first capacitor, 2
q 1 = C 1 V1 = 1 × 10 −6 × 6 × 103 22 Heat produced by kettle, H = V t .
A 2P IP R
= = 6000 µC
A 2Q IQ Here, heat taken by the water in both
Similarly, q 2 = C 2 V2
conditions, is same and voltage across
AP IP A (25 ) 2 = 2 × 10 − 6 × 4 × 103 = 8000 µC
⇒ = ⇒ P = the kettle is also same. So,
AQ IQ AQ (9 ) 2 In series order, charge on both the t R
t ∝ R or 1 = 1
AP 25 capacitors is same and is the least value. t2 R2
∴ =
AQ 9 So, in series combination of above two
capacitors, the charge should be Resistance of the kettle is directly
16 From law of refraction, 6000 µC. proportional to the number of turns in
q the coil.
A ∴Voltage on first capacitor V1 = t1 n
C1 So, = 1
6000 t2 n2
= = 6000 V = 6 kV
1 16 100
i1 i2 q ⇒ =
r2 6000
r1 Similarly, V2 = = = 3000 V t2 90
C2 2 16 × 90
⇒ t2 = = 14.4 min
O = 3 kV 100
Voltage on the system,
23 Magnetic field due to solenoid,
B C V = V1 + V2 = 6 + 3 = 9 kV
B1 = 4 mT (along the axis)
sin i 1 20 When both switches are open,
µ = Magnetic field due to a long straight
sin r1 ammeters reading wire,
1 .5
Here, µ = refractive index of material of I1 = µ I 4 π × 10 − 7 × 20
300 + 100 + 50 B2 = 0 =
prism. 2 πr 2 π × 3 × 10 − 3
So, i 1 ∝ r1 When both switches are closed, circuit 4
would be as shown = × 10 − 3 T
Maximum value of i 1 is 90°. If i 1 is 90°, 3
then r1 = C , critical angle at the other 100Ω 4
= mT (perpendicular to the axis)
surface of the prism. Also, there will be A
I2 3
no emergent rays, if the angle of So, resultant magnetic field,
incidence at surface AC is more than the 2
R
critical angle, i.e. r2 > C . B21 + B22 = 42 +  
4
50Ω B= = 4 . 2 mT
Angle of prism, A = r1 + r2 3
A = C + r2 I0 24 As, ∆Q = ms∆θ [for water]
Here, r2 > C
So, A > 2C = 50 × s × 5
300Ω 1.5V
⇒ 
dQ 
17 For microscope, limit of resolution is  = rate of cooling
1 .5  dt  l
proportional to the wavelength of light I0 =
100 R 250 25
used, 300 + = =
i.e. d ∝λ 100 + R 2 × 60 12
DAY THIRTY EIGHT MOCK TEST 1 401

[Qs w = 1 cal/g] which gives BH at equator ≈ 0 . 32 gauss r2 = 5 cm = 0.05 m


 dQ 
Now, other liquid   = rate of 29 A 1 = N1 λ and A 2 = N2 λ Since, bubble has two surface.
 dt  s
where, λ is disintegration constant. Initial surface area of the bubble
100 × s × 5 50s
cooling = = For time ( t 2 − t 1 ), = 2 × 4 πr 21
2 × 60 12
A 1 − A 2 = ( N1 − N2 ) λ = 2 × 4 π × (0.02)2 = 32 π × 10 − 4 m 2
Now, 
dQ   dQ 
 =  
 dt  l  dt  s Number of atoms disintegrated in time Final surface area of the bubble
⇒ s = 0 . 5 cal/g = 0 . 5 kcal/kg ( t 2 − t 1 ) s, = 2 × 4 πr22 = 2 × 4 × π × (0.05) 2
(A 1 − A 2)
25 Frequency = 50 Hz ( N1 − N2 ) = = 200 × π × 10 − 4 m 2
λ
1
But λ = , where T = mean life. So, work done = s × increase in surface
T area
2A
∴ ( N1 − N2 ) = ( A 1 − A 2 ) T = 0.06 × (200 × π × 10 − 4 − 32 π × 10 − 4 )
= 0.06 × 168p × 10 − 4 = 0.003168 J
30 Let retina receives n photons per
2A second. 35 Time taken by the body in covering first
So, nhν = 1 . 7 × 10 − 18 half is t 1 , so
1 c
∴Time period T = s or nh = 1 . 7 × 10 − 18 h 1 2
= gt 1 ⇒ t 21 =
h
⇒ t1 =
h
50 λ 2 2 g g
Rate of change of current λ
∴ n = 1 . 7 × 10 − 18 ×
dI I2 − I1 2 − ( − 2 ) hc For total journey,
= =
dt T/ 2 1 1 . 7 × 10 − 18 × 6000 × 10 − 10 ~ 1
h = gt 2 ⇒ t 2 =
2h
n = — 5
100 6. 6 × 10 − 34 × 3 × 108 2 g
= 4 × 100 = 400 A s −1 t 2 = 2 t 21 ⇒ t = 2 t 1
Crest value of voltage across secondary, 31 Wavelength of Balmer series is given by
dI Time taken in covering next half,
es = M 1  1 1 
dt = R  −  t2 = t − t1 = 2 t1 − t1
λ 2
 n1 n 22 
= 0. 25 × 400 = t1 ( 2 − 1 )
= 100 V  1
ν 1  ⇒ t1 =
t2
= R  −  [Qc = νλ ]
26 Given, V = 220 V and VL = 176 V c 2
 n1 n 22  ( 2 − 1)
For L-R circuit, V2 = VL2 + VR2
 1 1  36 We know that,
VR = V2 − VL2 ν = cR  − 
and 8  πr13  = πr23
2
n 22  4 4
 n1 v ∝ r2
3  3
= (220 ) − (176 ) V
2 2
For second line of Balmer series,
n 1 = 2; n 2 = 4 v1 r2 10 r2 1
∴ = 1 ⇒ = =
27 Here, Yc = 1 × 10 11 2
N / m and r22 8 / 3 r2 4
2
ν = 3 × 108 × 10967800 
1 1  v2 v2
− 
Ys = 2 × 1011 N / m 2  22 42  ⇒ v2 = 40 cm/s
lc = 1 . 0 m, ls = 0 . 5 m = 6 .16 × 1014 Hz
37 Moment of inertia of ring about the axis
and ∆lc = 1 × 10 –3 m passing through its centre and in its
32
Stress 1
As, (Strain)c = B E Electromagnetic wave plane I1 = mr 2
Yc 2
Stress Moment of inertia of ring about the axis
⇒ 1 × 10 =–3 Propagation
1 × 1011 passing through its centre and
⇒ Stress = 108 N / m 2 B E perpendicular to its plane
Stress As electromagnetic wave contains both I2 = mr 2
Now, Ys =
Strain electric field and magnetic field. It carry ∴Moment of inertia of system,
108 both energy and momentum according 1 3
⇒ Stress = = 0 . 5 × 10 –3 I = I1 + I2 = mr 2 + mr 2 = mr 2
to de-Broglie, wave particle duality of 2 2
2 × 1011
radiations.
∆ls
or = 0 . 5 × 10 –3 38 Both the statements are true as kinetic
1/2 33 D1 is forward biased and D2 reverse friction is used to stop relative motion
⇒ ∆ls = 0 . 25 × 10 –3 biased. by using brakes in machines and
∴ ∆l = ∆lc + ∆ls automobiles. Also, walking is possible
Therefore, current through the
because of static friction.
= 1 + 0 . 25 = 1 . 25 mm resistance and D1 will be equal and
5
28 As we know that, M ⋅ 4 π ( r2 + l2 )3 / 2 which is equal to = 1.25 A . 39 Kinetic energy of the satellite moving
BH µ 0 4 round the earth,
1
µ
Q l << r, so BH = 0 =
M 34 As given, s = 0.06 Nm − 1 K1 = mv2o
2
4 π r3 r1 = 2 cm = 0.02 m,
where, vo = orbital velocity of satellite.
402 40 DAYS ~ NEET PHYSICS DAY THIRTY EIGHT

Kinetic energy required to escape from Now, the source recedes away from 47 Resistivity of a material is given by
the earth’s gravitational field observer,
1 m
1
K2 = mv2e v v 6 ρ= =
so, n ′′ = n= n= n σ ne 2 τ
2 v + vs v + v/ 6 7
where, ve = escape velocity. n − n ′′ ρ thus depends inversely both on the
Per cent change = × 100
We know that, ve = 2 vo n number n of free electrons per unit
volume and on the average time τ
∴ K2 = m ( 2 vo )2 = 2  mv2o  n − 6 n
1 1
2    between collisions. For insulators and
2  7 
= × 100 = 14 .3% semiconductors, n increases with
= 2 K1 n temperature.
∴ Kinetic energy required to escape
= K2 − K1 = 2 K1 − K1 = K1 43 Let the radii of bigger and smaller drops This increase of n is more effective than
K − K1 be R and r. any decrease in τ, so that for such
Required ratio = 2 =1 materials ρ decreases with temperature.
K1 Volume of 125 small drops
= Volume of big drop Therefore, Assertion and Reason are
40 v ( t ) = (sin 6 t − cos 6 t ) 4
∴ 125 × πr 3 = πR 3
4 correct but Reason is not the correct
Acceleration, 3 3 explanation of Assertion.
dv ( t ) d ⇒ 5r = R
a (t) = = [sin 6 t − cos 6 t ] nq 48 A → 3, B → 1, C → 2, D → 1.
dt dt Potential on big drop V = k
= cos 6 t × 6 − ( − sin 6 t ) × 6 R
Moment
= 6 [cos 6 t + sin 6 t ] q Body
Potential on small drop V ′ = k of inertia
 1 1  r
=6 × 2  cos 6 t + sin 6 t  A. Thin circular ring of radius R 1. MR 2
 2  kq  5q 
2 V′ = =k   raving anis perpendicular to
π π R /5  R 

= 6 2  sin cos 6 t + cos sin 6 t 
the plane and passing
 4 4  V′ = 5
V
=5 ×
2 .5 though units
π n 125 2. MR 2 /2
= 6 2 sin  + 6 t 
B. Thin circular ring of radius R
4  2 .5 having axis passing throyh
= = 0 .1 V
25 its diameter
For maximum acceleration,
π
sin  + 6 t  = 1. 44 When charge q is placed in uniform C. Thin rod of length L about 3.
4  an axis perpendicular to the ML2 /12
electric field E, then its acceleration,
rod and passing through
∴ [a ( t )] max = 6 2 ms −2 qE
a = mid point
m
41 Change in internal energy, D. Circular disc of radius R 4. MR 2 /2
So, its motion will be uniformly
about an axis perpendicular
dU = dQ − dW . accelerated motion and its velocity after
to the disc and passing
time t is given by,
dQ = amount of heat given at constant qE
through the untie.
pressure v = at = t
7 m
= nC p ∆t = 1 × R × (60 − 20 ) = 140 R 1 49 In sum position,
2 KE = mv2 I1 + I2
2 T1 = 2 π
dQ = work done at constant pressure 2 ( M1 + M2 ) H
R ∆t 1  qEt  q 2 E2 t 2
= pdV = p = m   =
P 2  m  2m In difference position,
I1 + I2
= R ∆T = R (60 − 20 ) = 40 R T2 = 2 π
45 Output of gate I is Y’ = A + B ( M1 − M2 ) H
∴ dU = 140 R − 40 R
= 100 R = 100 × 8.31 = 831J Output of gate II is Y1 = A + Y ′ T1 1
Therefore, =
T2 3
42 When source approaches the observer, Output of gate III is Y2 = B + Y ′
then apparent frequency 50 Change in angular momentum of
Output of gate IV is Y = Y1 + Y2
v
n′ = n electron,
v − vs The truth table of the combination is in
∆L = L2 − L1
where, v = velocity of sound, option (a).
n h n h
vs = velocity of source = 2 − 1
46 According to Wien’s law, the peak 2π 2π
120 n
Here, n ′ = emission wavelength of a body is h
100 inversely proportional to its absolute or ∆L = ( n2 − n1 )
120 n v 2π
∴ = ×n temperature.
100 v − vs 6.6 × 10 −34
= (5 − 4)
6 v λ m T = constant 2 × 3.14
⇒ =
5 v − vs ⇒ λm =
constant = 1.05 × 10 −34 J-s
v T
6 v − 6 vs = 5 v ⇒ vs =
6 Higher T implies lower λ m .
DAY THIRTY NINE

Mock Test 2
Instruction
l This question paper contains of 50 Multiple Choice Questions of Physics, divided into two Sections;
section A and section B.
l Section A contains 35 questions and all questions are compulsory.
l Section B contains 15 questions out of which only 10 questions are to be attempted.
l Each question carries 4 marks.

Section-A 4 The potential energyU between two atoms in a


diatomic molecule as a function of the distance x
1 An electric fan is placed on a stationary boat and air is between atoms has been shown in the adjoining
blown with it on the sail of the boat. Which of the figure. The atoms are
following statements is correct?
Y
(a) The boat will be uniformly accelerated in the direction of
the flow of the air
U
(b) The boat will start moving with uniform speed
(c) The boat will be uniformly accelerated opposite to the
direction of flow of air
A B C
(d) The boat will remain stationary as before X

2 Correct set up to verify Ohm’s law is


X
A (a) attracted when x lies between A and B and are repelled
V A V when x lies between B and C
(a) (b) (b) attracted when x lies between B and C and are repelled
when x lies between A and B
(c) attracted when they reach B
(d) repelled when they reach B
A V
5 Electrons are accelerated through a potential
(c) (d) difference V0 and protons are accelerated through a
potential difference 4 V. The de-Broglie wavelength
V A are λ e and λ p for electrons and protons respectively.
λ
The ratio of e is given by
3 If p represents radiation pressure, c represents speed of λp
light and Q represents radiation energy striking a unit (given, me is mass of electrons and mp is mass of
area per second, then non-zero integers x , y and z;
proton)
such that p x Q y c z is dimension less, are
λe mp λe me
(a) x = 1, y = 1, z = − 1 (a) = (b) =
λp me λp mp
(b) x = 1, y = − 1, z = 1
(c) x = − 1, y = 1, z = 1 λe 1 me λe mp
(c) = (d) =2
(d) x = 1, y = 1, z = 1 λp 2 mp λp me
404 40 DAYS ~ NEET PHYSICS DAY THIRTY NINE

6 The speed of sound in hydrogen at NTP is 1270 ms −1. The corresponding works involved areW1 = 2200 J,
Then, the speed in a mixture of hydrogen and oxygen in W2 = − 825 J, W3 = − 1100 J and W4 , respectively. The
the ratio 4:1 by volume will be value of W4 is
(a) 317 ms −1 (b) 635 ms −1 (c) 830 ms −1 (d) 950 ms −1 (a) 1315 J (b) 275 J (c) 765 J (d) 675 J

7 A large number of water drops each of the radius r 14 A beam of light consisting of red, green and blue
combine to have a drop of radius R. If the surface colours, is incident on a right-angled prism. The
tension is T and the mechanical equivalent of heat is J, refractive indices of the material of the prism for the
then the rise in temperature will be above red, green and blue wavelengths are 1.39, 1.44
2T 3T 3T  1 1  2T  1 1  and 1.47, respectively. The prism will
(a) (b) (c)  −  (d)  − 
rJ RJ J r R  J r R  (a) separate part of the red colour from the green and blue
colours
8 A train of 150 m length is going towards North direction (b) separate part of the blue colour from the red and green
at a speed of 10 ms –1. A parrot flies at a speed of 5 ms –1 colours
towards south direction parallel to the railway track. The (c) separate all the three colours from one another
time taken by the parrot to cross the train is equal to (d) not separate even partially any colour from the other two
(a) 12 s (b) 8 s (c) 15 s (d) 10 s colours
9 Which of the four resistances P, Q, R and S generate the 15 The kinetic energy K of a particle moving along a circle
greatest amount of heat, when a current flows from A to B ? of radius R depends on the distance covered s, as
P=2Ω Q=4Ω K = as 2 , where, a is a constant. The force acting on the
particle is
1/ 2
A B s2  s2 
(a) 2a (b) 2as  1 + 2 
R=1Ω S=2Ω
R  R 
2
R
(a) Q (b) S (c) P (d) R (c) 2as (d) 2a
s
10 A charge Q is uniformly distributed over a large square 16 A car is moving in a circular horizontal track of radius
plate of copper. The electric field at a point very 10 m with a constant speed of 10 ms −1. A plumb bob is
close to the centre of the plate is 10 Vm −1. If the suspended from the roof of the car by a light rigid rod of
copper plate is replaced by a plastic plate of the same length 1.00 m. The angle made by the rod with track is
geometrical dimensions and carrying the same charge Q (a) zero (b) 30° (c) 45° (d) 60°

uniformly distributed, then the electric field at the point P 17 A thin semi-circular ring of radius r j
will be
has a positive charge q distributed
(a) 5 Vm −1 (b) zero (c) 10 Vm −1 (d) 20 Vm −1 ∧
uniformly over it. The net field E at the i
O
11 The speed of electromagnetic waves in vacuum centre O is
depends upon the source of radiation. It q $i q $j
(a) (b) −
(a) increases as we move from γ-rays to radio waves 4 π 2 ε0r 2 4 π 2 ε0r 2
(b) decreases as we move from γ-rays to radio waves q $j q $j
(c) − (d)
(c) is same for all of them 2 π 2 ε0r 2 2 π 2 ε0r 2
(d) None of the above

12 Ice starts freezing in a lake with water at 0°C, when the 18 The plot given below is of the average power delivered to
atmospheric temperature is −10°C. If the time taken for an L-R-C circuit versus frequency. The quality factor of
1 cm of ice to be formed is 12 minutes, the time taken the circuit is
Average power (microwatts)

for the thickness of the ice to change from 1 cm to 2 cm


will be
(a) 12 min 1.0
(b) less than 12 min 1.5
(c) more than 12 min but less than 24 min
(d) more than 24 min 0.0
13 An ideal gas is taken through a cyclic thermodynamical 3 4 5 6 7
process through four steps. The amounts of heat Frequency (kHz)

involved in these steps are Q1 = 5960 J, Q 2 = − 5585 J, (a) 5.0 (b) 2.0 (c) 2.5 (d) 0.4
Q 3 = − 2980 J and Q 4 = 3645 J, respectively.
DAY THIRTY NINE MOCK TEST 2 405

19 Two identical wires A and B have the same length L and 26 Rain is falling vertically downwards with a velocity of
carry the same current I. Wire A is bent into a circle of 4 kmh −1. A man walks in the rain with a velocity of 3 kmh −1.
radius R and wire B is bent to form a square of side a. If The raindrops will fall on the man with a velocity of
B1 and B2 are the values of magnetic induction at the (a) 1 kmh −1 (b) 3 kmh −1 (c) 4 kmh −1 (d) 5 kmh −1
centre of the circle and the centre of the square
respectively, then the ratio B1/B2 is 27 A machine gun is mounted on a 200 kg vehicle on a
(a) (π / 8)
2
(b) (π / 8 2 ) (c) (π / 16)
2 2
(d) (π / 16 2 )
2 horizontal smooth road (friction is negligible). The gun
fires 10 bullets per sec with a velocity of 500 ms −1. If the
20 In non-resonant circuit, what will be the nature of the mass of each bullet be 10 g, what is the acceleration
circuit for frequencies higher than the resonant frequency?
produced in the vehicle?
(a) Resistive (b) Capacitive (a) 25 cm s −2 (b) 35 cm s −2 (c) 50 cm s −2 (d) 50 ms −2
(c) Inductive (d) None of these
21 A particle of mass m is moving in a circular path of 28 The displacement of a particle executing periodic motion
is given by y = 4 cos 2 (t / 2) sin (1000 t )
constant radius r, such that centripetal acceleration ac
varying with time is ac = k 2r t 2 , where k is a constant. This expression may be considered to be a result of
What is the power delivered to the particle by the force superposition of
acting on it? (a) two waves (b) three waves
(c) four waves (d) five waves
(a) 2mkr 2t (b) mkr 2t 2 (c) mk 2r 2t (d) mk 2r t 2
29 If the series limit wavelength of the Lyman series for
22 The maximum height attained by a projectile is increased
hydrogen atom is 912Å, then the series limit wavelength
by 5%. Keeping the angle of projection constant, what is
for the Balmer series for the hydrogen atom is
the percentage increase in horizontal range?
912
(a) 5% (b) 10% (c) 15% (d) 20% (a) 912 Å (b) 912 × 2 Å (c) 912 × 4 Å (d) Å
2
23 A spot of light S rotates in a horizontal plane with a
30 Match the examples given in Column I with the type of
constant angular velocity of 0.1 rads −1. The spot of light
motion they are executing in Column II. There is no
P moves along the wall at a distance of 3 m from S. The
information about nature of surfaces of bodies is given.
velocity of spot P, where θ = 45°, is
(a) 0.5 ms −1 (b) 0.6 ms −1 (c) 0.7 ms −1 (d) 0.8 ms −1 Column I Column II
24 White light may be considered to be a mixture of wave
with λ ranging between 3000 Å to 7800 Å. An oil film of
thickness 10000 Å is examined normally by the reflected
A. 1. Rolling
light. If µ = 1.4, then the film appears bright for
(a) 4308 Å, 5091 Å, 6222 Å A block over an
(b) 4000 Å, 5091 Å, 5600 Å inclined plane
(c) 4667 Å, 6222 Å, 7000 Å
(d) 4000 Å, 4667 Å, 5600 Å, 7000 Å
25 A rectangular loop of B. 2. Translation
wire, supporting a mass A cylinder over an
m, hangs with one end P Q inclined plane
in uniform magnetic y
field B pointing out of i
the plane of the paper. a x C. 3. Rotation
A clockwise current is S R
mg
setup, such that A spinning top
i > mg / Ba, where a is the width of the loop. Then,
(a) the weight rises due to vertical force caused by the S E
magnetic field but work is done on the system D. 4. Precession
(b) the weight rises due to vertical force by the no work field
Earth-Sun system
and work is extracted shown on the electric field
(c) the weight rises due to vertical force caused by the A B C D
magnetic field but no work is done on the system (a) 2 1 3 4
(d) the weight rises due to vertical force caused by the no (b) 2 3 1 2
work field and work is extracted shown on the magnetic (c) 3 1 2 1
field (d) 3 1 2 4
406 40 DAYS ~ NEET PHYSICS DAY THIRTY NINE

31 Assuming that about 20 MeV of energy is released per 38 A mixture of n1 moles of monoatomic gas and n 2 moles
fusion reaction 1H 2 + 1H 3 → 0 n 1 +2 He 4 , then the mass of Cp
2 of diatomic gas has = γ = 1.5. Then,
1H consumed per day in a fusion reactor of power 1 MW CV
will approximately be (a) n1 = n2 (b) 2n1 = n2 (c) n1 = 2n2 (d) 2n1 = 3n2
(a) 0.001 g (b) 0.1 g (c) 10.0 g (d) 1000 g
39 A charged particle with some initial velocity is projected
32 A light ray falls on a square glass in a region where non-zero electric and/or magnetic
slab as shown in the figure. The 45° Incident ray fields are present. In Column I, information about the
index of refraction of the glass, if existence of electric and/or magnetic field and direction
total internal reflection is to occur of initial velocity of charged particle are given, while in
at the vertical face, is equal to Column II the probable path of the charged particle is
( 2 + 1) 5 mentioned. Match the entries of Column I with the entries
(a) (b)
2 2 of Column II and select the correct answer from the
3 3 Diagram codes given below.
(c) (d)
2 2
Column I Column II
33 A coil has an inductance of 0.7 H and is joined in series A. E = 0, B ≠ 0, E ≠ B and initial 1. Straight line
with a resistance of 220 Ω. When an alternating emf of velocity is at any angle with B
220 V at 50 cps is applied to it, then the wattless (θ ≠ 90° )
component of the current in the circuit, is B. E = 0, B ≠ 0 and initial velocity 2. Helical path with
(a) 5 A (b) 0.5 A (c) 0.7 A (d) 7 A is perpendicular to B uniform pitch
C. E ≠ 0, B ≠ 0, E|| B and initial 3. Circular
34 Which of the following statements is/are true?
velocity is ⊥ to both
(a) A clock, when taken on a mountain can be made to give
correct time, if we change the length of pendulum D. E ≠ 0, B ≠ 0, E perpendicular 4. Helical path with
to B and v perpendicular to non-uniform pitch
suitably
both E & B
(b) An increase in value of g makes a clock go slow
(c) If the length of a pendulum is increased, the clock A B C D
becomes fast (a) 2 3 4 1
(d) A clock, when taken to a deep mine or carried to the top (b) 1 3 4 2
of a mountain becomes slow (c) 2 3 1 4
(d) 4 1 2 3
35 I. Electric field, electric displacement and electric
polarisation are related by the relation as D = ε 0E − P . 40 A train standing at the outer signal of a railway station
II. Electric field vector (E), electric displacement vector blows a whistle of frequency 400 Hz in still air. The train
(D) and electric polarisation vector (P) are mutually begins to move with a speed of 10 ms −1 towards the
perpendicular to each other. platform. What is the frequency of the sound for an
III. Electric field vector (E), electric displacement observer standing on the platform? (sound velocity in air
vector (D) and electric polarisation vector (P) are = 330 ms −1)
mutually parallel to each other. (a) 412.5 Hz (b) 312.6 Hz (c) 500 Hz (d) 616.6 Hz
Which of the following statement(s) is/are correct?
(a) Only I (b) Both II and III
41 A dip needle arranged to move freely in the magnetic
(c) Both I and III (d) Only III
meridian dips by an angle θ. If the vertical plane in which
the needle moves is rotated through an angle α to the
Section-B magnetic meridian, then the needle will dip by an angle
(a) θ (b) α (c) more than θ (d) less than θ
36 Electric potential is given by
V = 6x − 8xy 2 − 8y + 6yz − 4z 2 42 The coming rays are forming rainbows
Then, electric force acting on 2 C on point charge placed
on origin will be
(a) 2 N (b) 6N (c) 8 N (d) 20 N Red Violet Violet Red Yellow Red
−6 2
37 There is a current of 40 A in a wire of10 m area of
cross-section. If the number of free electrons per cubic
metre is 1029, then the drift velocity is
Human eye Human eye
(a) 250 × 10−3 ms −1 (b) 25 . 0 × 10−3 ms −1 Human eye
(1) (2) (3)
(c) 2 .50 × 10−3 ms −1 (d) 1. 25 × 103 ms −1
DAY THIRTY NINE MOCK TEST 2 407

(a) Figure (1) forms primary rainbow 46 A body is projected with a velocity of 2 × 11.2 km/s from
(b) Figure (1) and (3) form primary rainbow
(c) Figure (2) forms secondary rainbow the surface of earth. The velocity of the body when it
(d) Figure (1) forms secondary rainbow escapes the gravitational pull of the earth is
(a) 3 × 11.2 km/s (b) 11.2 km/s
43 I. The phenomenon of splitting of light into different
colours is known as dispersion. (c) 2 × 11.2 km/s (d) 0.5 × 11.2 km/s
II. Dispersion takes place because the refractive index 47 Assertion The detection of neutrino particles is
of medium for different frequencies (colours) is extremely difficult.
different.
Reason Neutrinos interact very weakly with matter.
III. Thin lenses show chromatic aberration due to
(a) Both Assertion and Reason are correct and Reason is
dispersion of light.
the correct explanation of Assertion.
Which of the following statement is incorrect? (b) Both Assertion and Reason are correct but Reason is
(a) Only I (b) Only II not the correct explanation of Assertion.
(c) Only III (d) I, II and III (c) Assertion is correct but Reason is incorrect.
(d) Assertion is incorrect but Reason is correct.
44 A table tennis ball which has been covered with a
conducting paint is suspended by a silk thread, so that it 48 In an adiabatic process, wherein pressure is increased
hangs between two metal plates. One plate is earthed. 2 Cp 3
When the other plate is connected to a high voltage by %. If = , then the volume decreases by about
3 CV 2
generator, the ball 4 2 9
(a) % (b) % (c) 4% (d) %
(a) is attracted to the high voltage plate and stays there 9 3 4
(b) hangs without moving
(c) swings backward and forward hitting each plate in turn 49 The wavelength of the K α line for an element of atomic
(d) is repelled to the earthed plate and stays there number 43 is λ. Then, the wavelength of the K α line for
an element of atomic number 29, is
45 Two blocks of masses m and M are
(a)   λ (b)   λ (c)   λ (d)   λ
43 42 9 4
connected by means of a metal wire of  29   28   4  9
cross-sectional area A passing over a
frictionless fixed pulley as shown in the 50 For a transistor amplifier, the voltage gain
T (a) remains constant for all frequencies
figure. The system is then released. If T
m (b) is high at high and low frequencies and constant in the
M = 2m, then the stress produced in the wire
is M middle frequency range
2mg 4gm (c) is low at high and low frequencies and constant at
(a) (b)
3A 3A mid-frequencies
gm 3mg (d) None of the above
(c) (d)
A 4A

Answers
1. (d) 2. (a) 3. (b) 4. (b) 5. (d) 6. (b) 7. (c) 8. (d) 9. (b) 10. (c)
11. (c) 12. (d) 13. (c) 14. (a) 15. (b) 16. (c) 17. (c) 18. (d) 19. (b) 20. (c)
21. (c) 22. (a) 23. (b) 24. (a) 25. (c) 26. (d) 27. (a) 28. (b) 29. (c) 30. (a)
31. (b) 32. (d) 33. (b) 34. (d) 35. (d) 36. (d) 37. (c) 38. (a) 39. (a) 40. (a)
41. (c) 42. (d) 43. (c) 44. (c) 45. (b) 46. (a) 47. (a) 48. (a) 49. (c) 50. (c)
408 40 DAYS ~ NEET PHYSICS DAY THIRTY NINE

Hints and Explanations


1 The air blown at the sail will apply 6 As given, the volumes of hydrogen and Therefore, speed of electromagnetic
forward force but equal and opposite oxygen in a mixture is 4 : 1, so let V be waves is same for all of them.
force will be experienced by the fan. As the volume of Oxygen. The volume of
12 Time taken by ice to grow a thickness y,
fan is also a part of the boat, hence net hydrogen will be 4 V. If ρm be the
ρL 2
force on the boat will be zero, i.e. the density of mixture, then t = y
boat will remain stationary as before. 4 V × 1 + V × 16 2 Kθ
ρm = =4
5V Hence, time intervals to change
2 Ohm’s law states that the current ( I )
1/2 thickness from 0 to y, from y to 2y and
flowing through a conductor is directly 1
As, v∝  so on will be in the ratio
proportional to the potential difference  ρ
( V ) across the ends of the conductor, ∆t 1 : ∆t 2 : ∆t 3 : : ( 12 − 02 ) : ( 22 − 12 )
1270
provided that physical conditions of the ∴Velocity in mixture = = 635 ms
(4 )1 / 2 : ( 32 − 22 )
conductor such as temperature, −1 or ∆t 1 : ∆t 2 : ∆t 3 : : 1 : 3 : 5
mechanical strain etc., kept constant,
3 T 1 1 According to question, ∆t 1 = 12 min
i.e. I ∝ V or V ∝ I
7 Rise in temperature, ∆θ =  −  Hence, ∆t 2 = 3 ∆t 1 = 3 × 12 min
or V = RI [where, R is constant] JSd  r R 
= 36 min
[F] 3T  1 1
3 [p] = = [ML–1 T –2 ] , [c] = [LT –1 ] ∴ ∆θ =  − 
[A] J r R 13 ∆Q = Q 1 + Q 2 + Q 3 + Q 4
[Q] =
[E]
= [MT –3 ] [for water, S = 1 and d = 1] = 5960 − 5585 − 2980 + 3645
[A][T] = 1040 J
8 Relative velocity of the parrot w.r.t. the
As given, p xQ yc z = [M0 L0 T 0 ] ∆W = W1 + W2 + W3 + W4
train
= 2200 − 825 − 1100 + W4
[ML–1 T –2 ] x [MT –3 ] y [LT −1 ] z = [M0 L0 T 0 ] = [10 − ( −5 )] ms −1 = 15 ms −1
x + y −x + z −2 x − z −3 y
= 275 + W4
M L T = [M L T ]
0 0 0
For a cyclic process, ∆U = 0, i.e.
Time taken by the parrot to cross the
∴ x + y =0 U f − U i = 0,
150
−x + z = 0 train = = 10 s From first law of thermodynamics,
15
−2 x − z − 3 y = 0
∆Q = ∆U + ∆W
On solving , we get x = 1, y = − 1, 9 We know that, i ∝ 1 , i 2 = 6 i = 2 i 1040 = 0 + 275 + W4
z=1 R 3+6 3
or W4 = 765 J
4 Negative value of potential energy or i 1 =
3 1
i = i 14 The situation is shown in figure
decreases from C to B and there after 6+3 3
increases. It shows that kinetic energy A
Power rate in 2 Ω of upper series
increases from C to B and there after it
2
decreases. So, there is attraction from C 1  2
= 2 ×  i  = i2 Blue
to B and after which repulsion results. 3  3 45°

5 We have, E = qV, Power rate in 4 Ω of upper series


Red
2 45°
1 2E 1 
we know that E = mv2 . ⇒ v = =4 × 
4
i  = i2
2 m 3  9 Green
h h 45°
As, λ= = Power rate of 1 Ω in lower series 45°
mv 2E
m 2 B C
m 2  4
=1 ×  i  = i2
h 3  9
⇒ λ= ...(i) At face AB, i = 0. So, r = 0, i.e., light
2 mqV Power rate of 2 Ω in lower series will be incident on face AC at an angle
2
For electron, λe =
h 2  8 of incidence 45°.
...(ii) =2 ×  i  = i2
2 m eqV 3  9 The face AC will not transmit the light
h for which i > C .
For proton, λ p = ∴ P → 2/3, Q = 4/9, R → 4/9, S → 8/9
2 m p qV i.e. sin i > sin C
10 Electric field will remain same as charge 1
∴ λp =
h ⇒ sin45 ° >
density σ remain same ( E = σ / ε0 ). µ
2 m pq ⋅ 4 V
11 Speed of electromagnetic waves in 1 1
or >
[QV = 4 V] ...(iii) vacuum 2 µ
On dividing Eq. (ii) by Eq. (iii), we get 1
= = constant ⇒ µ > 2
µ 0 ε0
λe mp
=2 As refractive index for red colour is 1.39
λp me which is less than 2, so red will be
DAY THIRTY NINE MOCK TEST 2 409

transmitted through face AC while q 2 sin θ cos θ


⇒ E= ( − $j) = = 4 cot θ
green and blue will be reflected. 2 π2 ε0 r 2 (sin 2 θ) / 2
ω0 ∆R ∆h
15 According to given problem, Therefore, =
1 18 As quality factor Q = R h
mv2 = as2 B
2 Hence, percentage increase in R
where, ω0 = resonant frequency
= percentage increase in height h
⇒ v=s
2a B = Bandwidth
=5%
m From the graph, B = 2.5 kHz
v2 2 as2 Q = 0.4 23 The situation is shown in figure.
So, aR = = …(i)
R mR [by observing the curve] S
Further more as µ0 2 πI
dv dv ds dv 19 B1 = × θ
at = = ⋅ =v …(ii) 4π R
dt ds dt ds
µ0 2 πI × 2 π r=3m
[by chain rule] = × … (i)
2a 4π L
Which in light of Eq. (i), i.e. v = s [Q L = 2 πR for circular loop] P φ A
m x
µ0 I
yields B2 = × [sin 45 ° + sin 45 ° ] × 4
 2a   2 a  2as 4 π ( a /2 )
a t = s  = …(iii) where, a = L /4
 m m  m Wall
µ0 I
× 8 × 4 × 
1 1 
∴B2 = + From figure, x = r tan θ
2 
So that,
4πL  2
2 2 ∴ Velocity of P is
 2as2   2 as  µ0 I dx  dθ 
a = a 2R + a 2t =   +  = ×
64
v= = r sec 2 θ  
 m 
… (ii)
 mR  4πL 2 dt  dt 
Hence, dθ
where, = angular velocity of rotation
B1  µ 0  4 π2 I µ0 I
64 dt
θ = × ×
B2  4 π  L 4πL 2 of spot = ω
∴ v = ω r sec 2 θ
B1 π2
or = At φ = 45 °, so θ = 45 °
B2 8 2
v2 Hence, v = 0.1 × 3 × sec 2 45 °
θ r 20 At resonant frequency, = 0.1 × 3 × 2 = 0.6 ms −1
a QωL = 1 
X L = XC
2as  ωC  24 The film appears bright, if the path
Hence, a = 1 + [s / R ]2
m At frequencies higher than resonance difference is where,
frequencies, λ
∴ F = ma 2 µ t cos r = (2 n − 1 ) (n = 1, 2, 3 K)
X L > XC 2
= 2as 1 + [s / R ] 2
4 µ t cos r
i.e. behaviour is inductive .
∴ λ=
v /r 2
v2 2 (2 n − 1 )
16 tan θ = = 21 a c = v = k 2 r t 2 ⇒ v2 = k 2 r2 t 2
g rg r 4 × 1 .4 × 10000 × 10 −10 cos 0 °
λ=
 v2  1
KE = mv2 (2 n − 1 )
∴ θ = tan −1   2
 rg  56000
1 = Å
= mk 2 r 2 t 2 (2 n − 1 )
 10 × 10 
= tan −1   2
∴ λ = 56000 Å, 18666 Å, 11200 Å,
 10 × 10  According to work-energy theorem, 8000 Å, 6222Å, 5091 Å, 4308 Å, 3733 Å
∴ θ = tan −1 (1 ) = 45 ° change in kinetic energy is equal to
The wavelengths which are not within
work done.
17 Linear charge density, λ = 
q  1 specified ranges, are to be rejected.
 ∴ W = mk 2 r 2 t 2
 πr 
2 v + rω
y
Thus, power delivered to the particle, Vertical plane
dθ dW O
P= = mk 2 r 2 t
dt v – rω Us
θ
Cu

x ua
l pa
θ
rve

22 Let h be the maximum height attained th


dp

by the projectile. Then,


E = ∫ dE sin θ( − $j)
ath

u 2 sin 2 θ
K⋅dq h =
= ∫ r2
sin θ ( − $j) 2g A B
u sin 2θ
2
K qr R = 25 The net force on the RS is in downward
∫ πr d θ sin θ ( − j)
E= $
2 g direction which increase the weight
r
K q π R u 2 sin 2θ u 2 sin 2 θ attached to it but there is no change in
π ∫0
= ⋅ sin θ ( − $j) ∴ = / magnetic flux. Hence, there, no induced
r2 h g 2g
410 40 DAYS ~ NEET PHYSICS DAY THIRTY NINE

emf which implies there is no work 24 × 36 × 108 From Eq. (i), it is clear that D, E and P
=
done on the system. 32 × 10 −13 are in same direction, i.e. they are
26 Relative velocity of man w.r.t. rain, = 27 × 1021 mutually parallel to each other.
v rm = v r − v m = 4 j − 3i = − 3i + 4 j Mass of 6 × 1023 atoms = 2 g
36 E x = − ∂V = − (6 − 8 y2 )
∴ Mass of 27 × 1021 atoms ∂x
or |v rm |= ( −3 ) + (4 ) = 9 + 16
2 2
∂V
=
2
× 27 × 1021 = 0.1 g Ey = − = − ( − 16 xy − 8 + 6 z )
−1 ∂y
= 25 = 5 kmh 6 × 1023
∂V
Ez = − = − (6 y − 8 z )
27 Momentum carried by each bullet = mv 32 From figure, we get r = sin −1  1  ∂z
= 0.010 × 500 = 5 kg ms –1  3
At origin x = y = z = 0, so
Now, force = change in momentum in 1 s 45°
E x = − 6, E y = 8 and E z = 0
= 5 × 10 = 50 N
50 r ⇒ E= E2x + E2y = 10 NC −1
Hence, acceleration, a = ms −2
200 Hence, force F = QE = 2 × 10 = 20 N.
−2 r
= 25 cm s 90–r 37 Drift velocity,
90–r I
28 y = 4 cos ( t / 2 ) sin (1000 t )
2 r vd =
neA
= 2 [2 cos2 ( t / 2 ) sin (1000 t )] 1 40
For critical angle, sin C = =
= 2 (1 + cos t ) sin (1000 t ) µ 1029 × 1 .6 × 10 −19 × 10 −6
= 2 sin (1000 t ) + 2 sin (1000 t ) cos t Now, by Snell’s law, we have = 2 .5 × 10 −3 ms −1
= 2 sin (1000 t ) + sin (1001 t ) µ sin i
=
+ sin (999 t ) 1 sin r 38 fav
Thus, given wave represents the sin 45 ° 1/ 2 total number of degrees of freedom
superposition of three waves. = = =
  1  1/ 3 total number of molecules
sin  sin −1  
29 For series limit of Balmer series,   3  n 1 N A f1 + n 2 N A f2
=
n2 = 2 , n1 = ∞ 3 n1 N A + n2 N A
µ =
1 1 1  2
=R  − n 1 f1 + n 2 f2 3 n 1 + 5 n 2
λ 2 2 = =
n
 2 n 1
33 Wattless component of current n1 + n2 n1 + n2
 1 1  R Ev 2
= R − = = I v sin θ = sin θ Also, γ = 1 + =1.5 or fav = 4
 (2 )
2
( ∞ )2  4 Z fav
4 4 220 ωL 3 n1 + 5 n2
∴ λ= = m = × ∴ = 4 or n 1 = n 2
R 10967800 R +ω L
2 2 2
R + ω2 L2
2
n1 + n2
= 4 × 912 × 10 −10 m 220 × ωL
= 39
= 4 × 912 Å ( R 2 + ω2 L2 )
A. Since, E = 0 and B ≠ 0 and E ≠ B,
30 A — 2; Block is in translation. 220 × (2 π × 50 × 0.7 ) the magnetic force act on it and the
=
B — 1; Cylinder is rolling. (220 )2 + (2 π × 50 × 0.7 ) 2 path will be helical (with uniform
C — 3; Spinning top is in rotational 220 × 220 1
= = = 0.5 A pitch) if v is at any angle to B (
motion
(220 )2 + (220 )2 2 θ ≠ 90 °).
D — 4; Precession
34 Above or below the earth’s surface, B. Since, E = 0, B ≠ 0 and v ⊥ B, then a
31 Energy produced, force called Lorentz force,
value of acceleration due to gravity
U = Pt  F = q ( v × B), acts on charged
l
= 106 × 24 × 36 × 102 decreases, so time period  T = 2 π 
 g  particle, which provide centripetal
= 24 × 36 × 108 J force to it and hence the path will
increases i.e. clock will become slow.
Energy released per fusion reaction   be circular.
= 20 MeV Q T ∝ 1  C. Since, E ≠ 0, B ≠ 0, E || B, so path
 
 g 
= 20 × 106 × 1 .6 × 10 −19 will be helical with non-uniform
= 32 × 10 −13 J pitch as both electric and magnetic
35 The statement III is correct but rest are forces act on it.
Energy released per atom of 1H 2 incorrect and these can be corrected as,
D. Since, E ≠ 0, B ≠ 0, E ⊥B and v is
= 32 × 10 −13
J Electric field vector ( E), electric
displacement vector (D) and electric perpendicular to both E and B,
Number of 1H 2 atoms used polarisation vector ( P ) are related as then the particle will move in a
D = ε0 E + P …(i) straight line due to the resultant
where, ε0 is permittivity of free space.
DAY THIRTY NINE MOCK TEST 2 411

force of magnetic and electric  2m × 2m  ∆V  2   ∆p 


= g × 100 = −    × 100 
fields.  m + 2m  V  
3  p 
Hence, A → 2, B → 3, C → 4 where, m 1 = m and m 2 = 2 m =
4
mg
2 2
=− × % =− %
4
and D → 1. 3 3 3 9
Force (Tension) 4
Q Stress = Thus, volume decreases by %,
40 Here, ν = 400 Hz, vs = 10 m/s, Area 9
v = 330 m/s. 4
mg 49 Moseley’s law is given by
As the source is moving towards 4 mg
= 3 = ν = a ( Z − b)
stationary observer, therefore A 3A
1 1 For K α line, b=1
v× v 330 × 400
46 KE = mv20 − m (11.2) 2 ν = a ( Z − 1)
ν′ = = = 412.5 Hz 2 2
v − vs (330 − 10 ) [ v0 = 2 × 11.2 km/s]
On squaring both sides, we get
1 1
= m ( 2 × 11.2) 2 − m (11.2) 2 ν = a 2( Z − 1 ) 2
41 In this question, cos α < 1 i.e. 1 > 1 2 2
cos α 1 1 or
c
= a 2( Z − 1 ) 2
mv = 3 × m × (11.2) 2
2
tan θ ′ 2 2 λ
or > 1 or tan θ ′ > tan θ c
tan θ v = 3 × 11.2 km/s ⇒ λ=
or θ′ > θ a 2( Z − 1 ) 2
47 Neutrinos are neutral particles with For Z = 43, wavelength = λ
i.e. angle of apparent dip is more than
very small mass compared to electrons. c
angle of actual dip θ. ∴ λ=
They interact very weakly with matter.
a 2 (43 − 1 ) 2
42 In secondary rainbow, violet is above Therefore, they are very difficult to c
and red is below. detect, since they can even penetrate or λ= … (i)
the large quantity of matter (earth) a 2 (42 ) 2
43 The statement III is incorrect and it can For Z = 29, wavelength,
without any interaction.
be corrected as, c
Therefore, Assertion and Reason are λ′ =
Thick lenses show chromatic aberration a 2 (29 − 1 ) 2
correct and Reason is the correct
due to dispersion of light.
explanation of Assertion. c
= … (ii)
Rest statements are correct. a 2 (28 ) 2
48 For an adiabatic process,
44 When the other plate is connected to pV γ = constant (say C ) On dividing Eq. (ii) by Eq. (i), we get
the high voltage generator, the negative 2
Cp 3 λ ′  42 
charge induced on the ball cause Here, γ = = = 
CV 2 λ  28 
attraction. When it strikes the positive 2
plate charge distribution again takes ∴ pV3 / 2 = C 3
= 
place. This causes repulsion. Hence, the ⇒
3
log p + log V = log C 2
ball swings backward and forward 2 9
hitting each plate in turn. ∆p 3 ∆V ∴ λ′ =   λ
⇒ + =0 4
 2 m1 m2  p 2 V
45 Tension, T =   g, ∆V −2 ∆p
 m1 + m2  ∴ = 50 The voltage gain is low at high and low
V 3 p
frequencies and constant at
mid-frequency.
DAY FOURTY

Mock Test 3
Instruction
l This question paper contains of 50 Multiple Choice Questions of Physics, divided into two Sections;
section A and section B.
l Section A contains 35 questions and all questions are compulsory.
l Section B contains 15 questions out of which only 10 questions are to be attempted.
l Each question carries 4 marks.

Section A
(a) at B, force is zero
1 The magnetic field in a travelling electromagnetic wave
(b) at B, there is a force but towards motion
has a peak value of 20 nT. The peak value of electric
(c) at B, there is a force which opposes the motion
field strength is
(a) 3 V/m (b) 6 V/m (c) 9 V/m (d) 12 V/m (d) None of the above

2 The value of resistance is 10.85452 Ω and the value of 6 Block A of weight 100 kg rests on a block B and is tied
current is 3.23 A. The potential difference is 35.02935 V. with a horizontal string to the wall at C. Block B weighs
Its value in significant number would be 200 kg. The coefficients of friction between A and B is
(a) 3.8 V (b) 35.0 V (c) 35.03 V (d) 35.029 V 0.25, and between B and surface is 1/3. The horizontal
force F necessary to move the block B should be
3 A body initially at rest is accelerate at the rate of 2 ms −2 ( g = 10 ms −2 )
for 5s. If the body continues with uniform velocity for next
10 s, the total distance covered by the body is
(a) 50 m (b) 125 m (c) 165 m (d) 225 m
4 Assertion Angle between i + j and i is 45°. C A

Reason i + j is equally include to both i and j and the


B F
angle between i and j is 90°.
(a) Both Assertion and Reason are true and Reason is the
correct explanation of Assertion. (a) 1150 N (b) 1250 N
(b) Both Assertion and Reason are true but Reason is not (c) 1300 N (d) 1420 N
the correct explanation of Assertion. 7 A stone tied to a string is rotated in a vertical plane. If
(c) Assertion is true but Reason is false. mass of the stone is m, the length of the string is r
(d) Assertion is false but Reason is true. and the linear speed of the stone is v when the stone
5 Velocity-time graph of motion of a body is shown below is at its lowest point, then the tension in the string will
be (g = acceleration due to gravity)
mv 2 mv 2
A (a) + mg (b) − mg
C r r
v
mv 2
B (c) (d) mg
r

t
DAY FOURTY MOCK TEST 3 413

8 The force acting on a body moving along X -axis varies 13 A metal disc of radius r floats on the surface of water.
with the position of the particle as shown in the figure. The water layer goes down and makes an angle θ with
The body is in stable equilibrium at the vertical edge of the disc. If it displaces a weight w of
F water and the surface tension of water is T, then weight
of the disc is
(a) 2 πr T cosθ (b) 2 πr T
(c) 2 πr T cosθ + w (d) 2 πr T cosθ − w
14 If at the same temperature and pressure, the densities
x2 X of two diatomic gases are d1 and d 2 , respectively. The
x1
ratio of mean kinetic energy per molecule of gases will
(a) x = x1 (b) x = x 2 be
(c) Both x1 and x 2 (d) Neither x1 nor x 2 (a) 1 : 1 (b) d1 : d2
(c) d1 : d2 (d) d2 : d1
9 Statement I Velocity-time graph for an object in a uniform
motion along a straight line is, parallel to the time axis. 15 If the refractive index of water is 4/3 and that of given
slab of glass immersed in it is 5/3, what is the critical
Statement II In uniform motion of an object, velocity
angle for a ray of light tending to go from glass to water?
increases as the square of time elapsed.
(a) sin−1   (b) sin−1   (c) sin−1  
(a) Both statement I and statement II are true. 3 3 4
 4 5 5
(b) Both statement I and statement II are false.
(c) Statement I is true but statement II is false. (d) Data given is insufficient to make any calculation
(d) Statement I is false but statement II is true. 16 A block of ice at − 10° C is slowly heated and converted
to steam at 100°C. Which of the following curves
10 The I −V characteristic of an LED is represent the phenomenon qualitatively?
R Y G B R

Temperature
G
Temperature

I
Y
(a) I (b) (b)
R (a)

O V O V
Heat supplied Heat supplied

V O
-Red Temperature
Temperature

R -Yellow
(c) I (d) I -Green (c) (d)
Y
G -Blue
B
O V Heat supplied
Heat supplied
11 Which one of the following represents simple harmonic
17 In a compound microscope, magnifying power is 95 and
motion ? 1
the distance of object from objective lens is cm and
(a) x 2 = a + bv (b) x = a + bv 2 3.8
(c) x = a − bv (d) x = a − bv 2
1
the focal length of objective lens is cm, the
4
12 A graph is shown between stress and strain for metal. The magnification of eye-piece is
part in which Hooke’s law holds good is (a) 5 (b) 10
(c) 100 (d) insufficient data

18 Two polaroids are crossed. If now one of them is rotated


through 30° and unpolarised light of intensity I 0 is
Stress

A C
incident on the first polaroid, then the intensity of
D
transmitted light will be
B I0 3 I0
(a) (b)
4 4
O Strain 3 I0 I0
(c) (d)
(a) OA (b) AB (c) BC (d) CD 8 8
414 40 DAYS ~ NEET PHYSICS DAY FOURTY

19 In Young’s experiment, light of wavelength 4000 Å is 27 A series L-R circuit is connected to an AC source of
used to produce bright fringes of width 0.6 mm, at a frequency ω and the inductive reactance is equal to 2R.
distance of 2 m. If the whole apparatus is dipped in a A capacitance of capacitive reactance equal to R is
liquid of refractive index 1.5, then the fringe width will be added in series with L and R. The ratio of the new power
(a) 0.2 mm (b) 0.3 mm factor to the old one is
(c) 0.4 mm (d) 1.2 mm 2 2 3 5
(a) (b) (c) (d)
20 A ring of mass M and radius R is rotating about its axis 3 5 2 2
with angular velocity ω. Two identical bodies each of 28 The total binding energy of a nucleus is defined as the
mass m, are now gently attached at the two ends of a product of mass defect (in kg) and a constant. The value
diameter of the ring. Because of this, the kinetic energy of constant is
loss will be (a) 8 . 854 × 10−12 Fm−2 (b) 3 × 108 ms −1
m (M + 2m) 2 2 Mm (c) 9 × 1016 m2s −2 (d) 931 MeV
(a) ωR (b) ω2R 2
M (M + 2m)
29 A radioactive element X with a half-life of 2 h decays
Mm (M + m) M 2 2
(c) ω2R 2 (d) ωR giving a stable element Y. After a time of t hours, the ratio
(M − 2m) (M + 2m)
of X to Y atoms is 1 : 7. Then, its time period is
21 White light is used to illuminate the two slits in a (a) 4 (b) 6
(c) in between 4 and 6 (d) 14
Young's double slit experiment. The separation between
slits is b and the screen is at a distance d ( > > b ) from the 30 Generally, semiconductor can be used safely between
slits. At a point on the screen directly infront of one of the the temperatures
slits, certain wavelengths are missing. One of the missing (a) − 75°C and 200°C (b) 0°C and 75°C
wavelengths is (c) − 25°C and 300°C (d) − 40°C and 1000°C
(a) λ = b 2 /d (b) λ = 2 b 2 /d 31 Assuming that the silicon diode (having negligible
(c) λ = 3 b 2 /d (d) λ = 2 b 2 /3 d
resistance), the current through the diode is (knee
22 A charge q is distributed over two concentric hollow voltage of silicon diode is 0.7 V)
conducting sphere of radii r and R ( > r ), such that their R = 100 Ω
surface charge densities are equal. The potential at their
common centre is 3V 0V
q (r + R ) (a) zero (b) 7 mA (c) 2.3 mA (d) 23 mA
(a) zero (b)
4 πε0 (r 2 + R 2 ) 2 32 A ray falls on a prism ABC ( AB = BC ) and travels as
q 1 1  q (r + R ) shown in figure. The minimum refractive index of the
(c) + (d)
4 πε0  r R  4 πε0 (r 2 + R 2 ) prism material should be
23 When resonance is produced in a series L-C-R circuit,
then which of the following is not correct? A
(a) Current in the circuit is in phase with the applied voltage 90°
(b) Inductive and capacitive reactances are equal
(c) If R is reduced, the voltage across capacitor will increase
(d) Impedance of the circuit is maximum
90°
24 In a meter bridge with a standard resistance of 5 Ω in the B C
left gap, the ratio of balancing lengths on the meter (a) 4/3 (b) 2 (c) 15
. (d) 3
bridge wire is 2 : 3. The unknown resistance is
(a) 3.3 Ω (b) 7.5 Ω (c) 10 Ω (d) 15 Ω 33 In an experiment on photoelectric Metal A
effect, a student plots stopping Metal B
25 The inductance of a coil, in which a current of 0.2 A is potential V0 against reciprocal of Vo
increasing at the rate of 0.5 As −1 represents a power flow the wavelength λ of the incident
of 0.5 W, is light for two different metals A and 1/l
(a) 2 H (b) 5 H (c) 10 H (d) 20 H
B. These are shown in the figure.
26 An ideal choke of 10 H is joined in series with resistance Looking at the graphs, you can most appropriately say
of 5 Ω and a battery of 5V. The current in the circuit in 2 s that
after joining in ampere, will be (a) work function of metal B is greater than that of metal A
(a) e −1 (b) 1 − e −1 (b) work function of metal A is greater than that of metal B
(c) 1 − e (d) e (c) data is not sufficient
(d) None of the above
DAY FOURTY MOCK TEST 3 415

34 A gas undergoes a cyclic process ABCDA as shown in 39 Two coherent sources with intensity ratio α interfere.
figure. The part ABC of process is semi-circular. The Imax − Imin
Then, the ratio is
work done by the gas is Imax + Imin
2 α 2 4 α
(a) 2 α (b) (c) (d)
B 1+ α 1− α 1+ α
50
p 40 A body of mass 100 g is sliding from an inclined plane of
(in Nm–2) 30 A
C inclination 30°. What is the frictional force, if µ = 0.7 ?
D
2 0.7 3
(a) 0.7 N (b) N
3 2
0.7 2
20 60 (c) 0.7 3N (d) N
V(m3) 3
(a) 400 π J (b) 2456 J 41 A cell of emf E and internal resistance r is connected with
(c) 200 π J (d) 1828 J external resistance R. The graph between terminal
35 In a given process on an ideal gas, dW = 0 and dQ < 0, voltage and current is
then for the gas E Elr E
(a) the temperature will decrease Elr
(b) the volume will increase
(a) (b)
(c) the pressure will remain constant
(d) the temperature will increase V = E –ir

I
Section-B I
E E
36 The diode used in the circuit shown in the figure has a Elr
constant voltage drop of 0.5 V at all currents and a
(c) (d)
maximum power rating of 100 mW. What should be the
Elr
value of the resistor R, connected in series with the
diode, for obtaining maximum current I ? I I
R
42 Statement I To obtain same change in the value of g,
D depth d below the surface of earth must be equal to
I
twice the height h above the surface of the earth.
Statement II g changes less with depth than with height.
1.5V (a) Both statement I and statement II are true.
(a) 200 Ω (b) 6.67 Ω (b) Both statement I and statement II are false.
(c) 5 Ω (d) 1.5 Ω (c) Statement I is true but statement II is false.
37 Statement I Inertia and moment of inertia are same d) Statement I is false but statement II is true.
quantities.
43 In an n-p-n transistor circuit, the collector current is
Statement II Moment of inertia represents the capacity
10 mA. If 90% of the electrons reach emitted the
of a body to oppose its state of motion.
collector, then
(a) Both statement I and statement II are true.
(a) Ie = 1 mA, Ib = 11 mA
(b) Both statement I and statement II are false.
(b) Ie = 11 mA, Ib = 1 mA
(c) Statement I is true but statement II is false. (c) Ie = − 1 mA, Ib = 9 mA
(d) Statement I is false but statement II is true. (d) Ie = 9 mA, Ib = 1 mA
38 Two blocks A and B of masses 3m and m 44 A capacitor is charged by a battery. The battery is
respectively are connected by a massless removed and another identical uncharged capacitor is
and inextensible string. The whole system is connected in parallel. The total electrostatic energy of
A 3m
suspended by a massless spring as shown resulting system
in figure. The magnitudes of acceleration of B m
(a) increases by a factor of 4
A and B immediately after the string is cut, (b) decreases by a factor of 2
are respectively (c) remains the same
g g g g (d) increases by a factor of 2
(a) g, (b) ,g (c) g, g (d) ,
3 3 3 3
416 40 DAYS ~ NEET PHYSICS DAY FOURTY

45 Assertion For projection angle tan−1(4), the horizontal 49 Thermodynamic processes are indicated in the following
and maximum height of a projectile are equal. diagram
Reason The maximum range of projectile is directly P
proportional to square of velocity and inversely
proportional to acceleration due to gravity. IV
f
(a) Assertion and Reason are true and Reason is the I III
correct explanation for Assertion II f
f 700 K
(b) Assertion and Reason are true but Reason is not a
f 500 K
correct explanation for Assertion 300 K
(c) Assertion is true but Reason is false V
(d) Assertion is false but Reason is true
Match the following :
46 The output of a two input OR gate is 1, if Column-I Column-II
(a) both inputs are zero P. Process I a. Adiabatic
(b) either or both inputs are 1
Q. Process II b. Isobaric
(c) only both inputs are 1
(d) either input is zero R. Process III c. Isochoric
S. Process IV d. Isothermal
47 If A and B are unit vectors, then ( A × B) ⋅ ( A × B) is equal
to (a) P → a, Q → c, R → d, S → b
(a) 1 (b) zero (b) P → c, Q → a, R → d, S → b
1 (c) P → c, Q → d, R → b, S → a
(c) (d) −1 (d) P → d, Q → b, R → a, S → c
2
48 A long solenoid of diameter 0.1 m has 2 × 104 turns per 50 One end of the string of length l is connected to a
metre. At the centre of the solenoid, a coil of 100 turns particle of mass m and the other end is connected to a
and radius 0.01 m is placed with its axis coinciding with small peg on a smooth horizontal table. If the particle
the solenoid axis. The current in the solenoid reduces at moves in circle with speed v, the net force on the particle
a constant rate to 0 A from 4 A in 0.05 s. If the resistance (directed towards center) will be (T represents the
of the coil is 10π 2 Ω, the total charge flowing through the tension in the string)
mv 2
coil during this time is (a)T (b)T +
(a) 32 π µC (b) 16 µC l
(c) 32 µC (d) 16π µC mv 2
(c)T − (d) Zero
l

Answers
1 (b) 2 (b) 3 (b) 4 (a) 5 (c) 6 (b) 7 (a) 8 (b) 9 (c) 10 (a)
11 (d) 12 (a) 13 (c) 14 (a) 15 (c) 16 (a) 17 (a) 18 (c) 19 (c) 20 (b)
21 (a) 22 (d) 23 (d) 24 (b) 25 (b) 26 (b) 27 (d) 28 (c) 29 (b) 30 (b)
31 (d) 32 (b) 33 (c) 34 (c) 35 (a) 36 (c) 37 (d) 38 (b) 39 (b) 40 (b)
41 (b) 42 (b) 43 (b) 44 (d) 45 (b) 46 (b) 47 (a) 48 (c) 49 (b) 50 (a)
DAY FOURTY MOCK TEST 3 417

Hints and Explanations


E0 Therefore, intensity of light transmitted
1 We know that, =c 10 LEDs are current dependent devices
B0 with its forward voltage drop V. from second polaroid
⇒ E0 = cB0 Forward voltage drop for Red, Yellow, I2 = I1 cos2 θ
= 3 × 108 × 20 × 10 −9 = 6 V/m Green and Blue are related as I
= 0 cos2 (90 ° − 30 ° )
VBlue > VGreen > VYellow > VRed 2
2 Number of significant figures in R = 5 2
cos2 60 ° = 0 ×   = 0
11 Relation (d) which is x = a − bv2 I0 I 1 I
Number of significant figures in I = 3 =
2 2 2 8
Hence, potential difference V ( = RI ) correctly represent the SHM because,
should contain 3 significant figures. velocity 19 Fringe width, β ∝ λ
i.e. V = 35 V v = ω a 2 − x2 β1 λ
∴ = 1
3 Distance covered in first 5 s, or x = a 2 − v2 / ω2 β2 λ2
1 2 Q b = 1  β1 λ1
s1 = ut + at or x = a 2 − bv2 or =
2  ω2  β2 λ1 /µ
1
= 0 + × 2 × (5 ) 2 = 25 m β 0.6
12 In the region OA, stress ∝ strain, i.e. ∴ β2 = 1 = = 0.4 mm
2 µ 1.5
Hooke’s law hold good.
Velocity after 5 s,
v = u + at
20 By conservation of angular momentum,
13 When water is depressed downwards at
I1 ω1 = I2 ω2 ...(i)
= 0 + 2 × 5 = 10 ms −1 point of contact, then the force due to
surface tension acts upwards.
Distance covered in next 10 s, where, I1 = mR 2 ...(ii)
s2 = 10 × 10 = 100 m Therefore, apparent weight
and I2 = 2 mR 2 + MR 2 ...(iii)
Total distance = F T + w = 2 πr T cos θ + w
Now, change in kinetic energy,
= s1 + s2 = 25 + 100 = 125 m
14 Mean kinetic energy of gas depends 1 1
KE = L1 I12 − L2 I22 ...(iv)
( i$ + $j ) ⋅ i$ 1 only on the temperature. Here, 2 2
4 cos θ = = = cos 45 °
( i$ + $j ) i$ 2 temperature is given same, so ratio of On substituting the values from Eqs. (i),
kinetic energies will be 1:1. (ii) and (iii) into Eq. (iv), we get
So, θ = 45 °  Mm  2 2
15 Critical angle Change in KE =  ω R
5 At B, velocity is decreasing, so there is a µ  4 /3   M + 2m 
force which opposes the motion. C = sin −1  w  = sin −1  
 µg   5/3  21 Positions of minima are given by
1 Dλ
y n =  n − 
6 Required force, 4
= sin −1  
F = µ AB R A + µ BS R B 5  2 d
1 2 yn d
= 0 . 25 × 100 kg + (100 + 200 ) kg λ=
3 16 Temperature first increases from − 10 ° C (2 n − 1 ) D
= 25 kg + 100 kg to 0°C. Then, remains constant at 0°C
Here, y n = b/2, d = b, D = d
= 125 kg = 125 × 10 = 1250 N till whole ice is melted. Then, it
b2
increases from 0°C to 100°C. Again, it ∴ λ=
7 At point B, remains constant at 100°C till whole (2 n − 1 ) d
2
mv water converts into steam.
T B − mg = where, n = 1, 2, 3,...
r b2 b2
mv2 17 Magnifying power of compound Hence, λ= , , ...
⇒ TB = + mg microscope, M = mo × me d 3d
r
1 1 1
For objective, = − 22 q = 4 π ( r2 + R 2 ) ⋅ σ
8 When displaced from x2 in negative fo vo uo
q
direction, force is positive. So, this force 1 1 1 1 σ =
= − ⇒4 = + 3 .8 4 π ( r + R2 )
2
is of restoring nature or bringing the 1/4 vo −1 / 3.8 vo
body back. Hence, at x2 , body is in Potential at centre,
⇒ vo = 5 cm
stable equilibrium position. σr σR σ
v 5 V= + = (r + R)
∴ mo = o = = 19 ε0 ε0 ε0
9 When the object moves with uniform uo 1/3 .8
q (r + R)
velocity the magnitude of its velocity at m 95 =
Hence, m e = = =5 4 π ε0 ( r 2 + R 2 )
t = 0, t = 1 s and t = 2 s will be m o 19
constant.
23 In series resonant L-C-R circuit,
Hence, velocity-time graph for an object 18 Intensity of transmit light from one
I0 X L = XC
in uniform motion is a straight line polaroid, I1 = ∴Impedance,
parallel to time axis. 2
418 40 DAYS ~ NEET PHYSICS DAY FOURTEEN

3 n
Z = ( X L − XC )2 + R 2 = 0 + R 2 ⇒ 1 = 1
    or n = 3
Zmin = R 2 2 kx
R R Hence, required time period, (3m+m)
Power factor, cos φ = = =1
Z R t = n T1 / 2 = 3 × 2 = 6 h A 3m
∴ φ = 0° Cutting
Hence, current and voltage are in same 30 Semiconductors can be used safely 4 mg plane
B m
phase and impedance of the circuit is between temperatures 0°C and 75°C.
minimum. Hence, option (d) is not ∴ kx = 4 mg
31 Current through diode, When string is cut at the location as
correct.
∆V 3 − 0 . 7 2.3 shown above.
I= = = A = 23 mA
24 For meter bridge, R 100 100 Free body diagram for m is
unknown resistance,
l 3 32 ∠ i at both faces will be 45°. For TIR to m
R = 2 × X = × 5 = 7.5 Ω
l1 2 take place,
i > θC
25 Energy stored in inductance, mg
1 or sin i > sin θC
U = L I2 So,force on mass m = mg
2 1 1
∴ > or µ > 2 ∴ Acceleration of mass, m = g
dU d  1 2 dI 2 µ
Power, P = =  LI  = L I For mass 3m; free body diagram is
dt dt  2  dt
33 We have, eV0 = hc − φ kx=4mg
∴ L=
P
=
0.5
=5 H λ
dI 0 . 2 × 0 . 5 hc φ
I× ⇒ V0 = − a 3m
dt eλ e
V0 = mx + c
26 Time constant τ = L = 10 = 2 s 3 mg
R 5 Q Data is not sufficient.
I0 =
e 5
= = 1A If a = acceleration of block of mass 3m,
34 Work done, W = Area ABCDA then
R 5
∴ Current in 2 s, π R2 π × (20 ) 2 Fnet = 4 mg − 3 mg
= = ⇒ 3m ⋅ a A = mg
2 2
I = I0 (1 − e − t / τ ) = 1(1 − e −2 / 2 ) = 1 − e −1 g
= 200 π J or aA =
3
27 For L-R circuit, So, accelerations for blocks A and B are
35 From first law of thermodynamics,
Z1 = R 2 + (2 R )2 = R 5 g
dQ = dU + dW aA = and a B = g
∴ cos φ1 = power factor 3
R R 1 we have, dQ = dU [as, dW = 0]
= = = But dQ < 0 I max – I min
z1 R 5 5
∴ dU < 0
39 R =
I max + I min
Now, for L-C-R circuit, n C V ∆T < 0
Z2 = R 2 + (2 R − R )2 = R 2 (a 1 + a 2 ) 2 − (a 1 − a 2 ) 2
or ∆ T<0 =
R R 1 Hence, the temperature will decrease. (a 1 + a 2 ) 2 + (a 1 − a 2 ) 2
∴ cos φ2 = = = 4a 1 a 2 2a 1 a 2 2 ( a 2 /a 1 )
Z2 R 2 2
36 Current in the circuit, = = =
1 2 (a 21 + a 22 ) a 21 + a 22 a 
2
P 100 × 10 −3 1 +  2
cos φ2 5 I= = = 200 × 10 −3 A  a1 
∴ = 2 = V 0.5
cos φ1 1 2 2
Voltage across resistance R, a  a
5
V′ = 1.5 − 0.5 = 1.0 V Given,  2  = α and 2 = α
 a1  a1
28 Total binding energy, E = ( ∆m ) c 2 , ∴ Resistance,
V′ 2 α
R = =
1
=5 Ω ∴ R =
where, ∆m = mass defect. 1+α
I 200 × 10 −3
Therefore, the value of constant in the
40 Friction force, F = µ mg cos θ
above product is given by 37 The inertia of a body depends only
= 0 . 7 × 100 × 10 −3 × 10 cos 30 °
c 2 = 9 × 1016 m 2s −2 . upon the mass of the body but the
3 0.7 3
moment of inertia of a body about an = 0.7 × = N
29 Initial amount of radioactive element 2 2
= X+ Y =1 + 7 =8 axis not only depends upon the mass of
n the body but also on the distribution of 41 Terminal voltage decreases as current
Now, N = N0  
1
2 mass about the axis of rotation. increases.
n −2
1 = 8   = g  1 + 
1 g h
∴ 38 Initially system, is in equilibrium with a 42 g h =
2 1 + h 
2  R
total weight of 4mg over spring.  
 R
DAY FOURTY MOCK TEST 3 419

or g h ≈ g  1 −
2h  2gh So, finally stored energy ∆t = 0.05 s, ∆i = 4 − 0 = 4 A
 or g − g h =
 R  R 1 1 Charge flowing through the coil is given
U2 = CV2 + CV2 by
...(i) 2 2
∆φ 1
and g d = g  1 − 
d 1
= (2C ) V2 ∆q =   ( ∆t )
 R 2  ∆t  R
∆i 1
= 2  CV2  = 2 U1 = µ 0 N1 N2 πr 2  
gd 1
or g − g d = ...(ii) ∆t
R 2   ∆t  R
For same change in value of g, = 4 π × 10 −7 × 2 × 104 × 100 × π
45 Horizontal range of projectile,
g − g h = g − gd
× (10 −2 ) 2 × 
4  1
u 2 sin 2θ u2  × × 0.05
2gh gd R = , R max = [at θ = 45 ° ]  0.05  10 π2
or = g g
R R = 32 × 10 −6 C = 32 µC
The maximum height attained by
or d = 2h
projectile 49 In isochoric process, the curve is
From Eqs. (i) and (ii), it is obvious that g u 2 sin 2 θ
H= parallel to y-axis because volume is
changes less with depth than with height.
2g constant.
u 2 sin 2 θ u 2 sin 2θ Isobaric is parallel to x-axis because
43 Given, Ic = 10 mA If H = R , then = , pressure is constant.
90 2g g
Also, 90% of Ie = 10 ⇒ Ie = 10 Along the curve, it will be isothermal
100 sin 2 θ because temperature is constant.
100 = 2 sin θ cos θ
∴ Ie = mA ≈ 11 mA 2 So, P→ c
9 Q→ a
So, tan θ = 4
Again, I b = Ie − Ic = 11 − 10 = 1 mA R→ d
∴ θ = tan − 1 (4 )
44 Thinking Process Energy stored in a S→ b
system of capacitors 46 Output of two input OR gate is
50 Consider the string of length l
1
= Σ CV2 Y = A + B. connected to a particle as shown in the
2 i.e. Y = 1, if either or both inputs are 1. figure.
Also, potential drop remains same in
parallel across both capacitors.
47 As ( A × B) is parallel to A × B, hence peg particle
dot product will be unity. v
l
48 Thinking Process Current induced in Speed of the particle is v. As the particle
the coil is given by
is in uniform circular motion, net force
1  dφ 
=   on the particle must be equal to
+ – R  dt  centripetal force which is provided by
V ∆q 1  ∆φ  the tension ( T ).
⇒ =  
Initially stored energy ∆t R  ∆t  ∴ Net force = Centripetal force
1 mv2
U1 = CV2 Given, resistance of the solenoid,
⇒ =T
2 R = 10 π2 Ω l
Finally, potential drop across each
capacitor will be still V. Radius of second and coil r = 10 −2
EXAM BITES

This Pdf Is
Downloaded From
www.exambites.in

Visit www.exambites.in for


More Premium Stuffs,Latest
Books,Test Papers,Lectures etc.
jeeneetadda
jeeneetadda_official
jeeneetadda

VISIT NOW !!
NEET Solved Paper 2019 40 DAYS ~ NEET PHYSICS 01

NEET (National)
SOLVED PAPER 2019
Held on : 5 May 2019
Duration : 3 Hours Max. Marks : 180

Important Instructions
Test Booklet contains 45 questions.
Each question carries 4 marks. For each correct response, the candidate will get 4 marks.
For each incorrect response, one mark will be deducted from the total scores.
The maximum marks are 180.

1. For a p-type semiconductor, which of the following 5. A small hole of area of cross-section 2 mm 2 is present
statements is true ? near the bottom of a fully filled open tank of height
(a) Holes are the majority carriers and trivalent atoms are the 2 m.
dopants. Taking g = 10 m/s 2, the rate of flow of water through
(b) Holes are the majority carriers and pentavalent atoms are the open hole would be nearly
the dopants.
(a) 8.9 ´ 10-6 m 3 /s (b) 2. 23 ´ 10-6 m 3 /s
(c) Electrons are the majority carriers and pentavalent atoms
are the dopants. (c) 6.4 ´ 10-6 m 3 /s (d) 12.6 ´ 10-6 m 3 /s
(d) Electrons are the majority carriers and trivalent atoms are the 6. A force F = 20 + 10 y acts on a particle in y-direction,
dopants.
where F is in newton and y in meter.
2. A body weighs 200 N on the surface of the earth. How Work done by this force to move the particle from
much will it weigh half way down to the centre of the y = 0 to y = 1 m is
earth ? (a) 5 J (b) 25 J (c) 20 J (d) 30 J
(a) 200 N (b) 250 N (c) 100 N (d) 150 N
7. Two point charges A and B, having charges +Q and -Q
3. In which of the following processes, heat is neither respectively, are placed at certain distance apart and
absorbed nor released by a system? force acting between them is F.
(a) Adiabatic (b) Isobaric (c) Isochoric (d) Isothermal If 25% charge of A is transferred to B, then force
between the charges becomes
4. Six similar bulbs are connected as 9F 16F 4F
shown in the figure with a DC A B (a) (b) (c) (d) F
16 9 3
source of emf E and zero internal
resistance. 8. When an object is shot from the bottom of a long
The ratio of power consumption by smooth inclined plane kept at an angle 60º with
the bulbs when (i) all are glowing horizontal, it can travel a distance x1 along the plane.
and (ii) in the situation when two But when the inclination is decreased to 30º and the
from section A and one from section same object is shot with the same velocity, it can travel
B are glowing, will be x 2 distance. Then x1 : x 2 will be
(a) 9 : 4 (b) 1 : 2 E (a) 2:1 (b) 1: 3
(c) 2 : 1 (d) 4 : 9 (c) 1:2 3 (d) 1: 2
02 40 DAYS ~ NEET PHYSICS NEET Solved Paper 2019

9. Pick the wrong answer in the context with rainbow. 17. +6 V


(a) The order of colours is reversed in the secondary rainbow R
0
(b) An observer can see a rainbow when his front is towards the
LED
sun A 1
(Y)
(c) Rainbow is a combined effect of dispersion refraction and
reflection of sunlight R
(d) When the light rays undergo two internal reflections in a 0
water drop, a secondary rainbow is formed B 1
10. Average velocity of a particle executing SHM in one The correct Boolean operation represented by the
complete vibration is circuit diagram drawn is
Aw2 Aw
(a)Aw (b) (c) zero (d) (a) OR (b) NAND (c) NOR (d) AND
2 2
18. A soap bubble, having radius of 1 mm, is blown from
11. The total energy of an electron in an atom in an orbit a detergent solution having a surface tension of
is -3.4 eV. 2.5 ´ 10 -2 N/m. The pressure inside the bubble equals at
Its kinetic and potential energies are, respectively:
a point Z 0below the free surface of water in a container.
(a) -3.4 eV, -6.8 eV (b) 3.4 eV, -6.8 eV
(c) 3.4 eV, 3.4 eV (d) -3.4 eV, -3.4 eV
Taking g = 10 m/s 2, density of water = 10 3kg/m 3, the
value of Z 0 is
12. At a point A on the earth’s surface the angle of dip, (a) 10 cm (b) 1 cm (c) 0.5 cm (d) 100 cm
d = +25º.
At a point B on the earth’s surface the angle of dip, 19. The displacement of a particle executing simple
d = -25º. harmonic motion is given by y = A0 + A sin wt + Bcos wt
We can interpret that Then the amplitude of its oscillation is given by
(a) A is located in the southern hemisphere and B is located in
(a) A 2 + B2 (b) A02 + ( A + B)2
the northern hemisphere.
(b) A is located in the northern hemisphere and B is located in (c) A + B (d) A0 + A 2 + B2
the southern hemisphere.
(c) A and B are both located in the southern hemisphere. 20. The work done to raise a mass m from the surface of
(d) A and B are both located in the northern hemisphere. the earth to a height h, which is equal to the radius of
the earth, is
13. In which of the following devices, the eddy current 1 3
effect is not used? (a) 2mgR (b) mgR (c) mgR (d) mgR
2 2
(a) Magnetic braking in train (b) Electromagnet
(c) Electric heater (d) Induction furnace 21. The speed of a swimmer in still water is 20 m/s. The
speed of river water is 10 m/s and is flowing due east.
14. Two particles A and B are moving in uniform circular If he is standing on the south bank and wishes to cross
motion in concentric circles of radii rA and rB with the river along the shortest path the angle at which he
speed v A and vB respectively. Their time period of should make his strokes w.r.t. north is given by
rotation is the same. The ratio of angular speed of A to (a) 0º (b) 60° west (c) 45º west (d) 30º west
that of B will be
(a) v A : v B (b) rB : rA (c) 1 : 1 (d) rA: rB 22. Two parallel infinite line charges with linear charge
densities +l C/m and -l C/m are placed at a distance
15. Two similar thin equi-convex lenses, of focal length f of 2R in free space. What is the electric field mid-way
each, are kept co-axially in contact with each other between the two line charges?
such that the focal length of the combination is F1 . 2l l l
(a) N/C (b) N/C (c) N/C (d) Zero
When the space between the two lenses is filled with pe0 R pe0 R 2pe0 R
glycerine (which has the same refractive index (m = 15 .)
as that of glass) then the equivalent focal length is F2. 23. Which of the following acts as a circuit protection
The ratio F1 : F2 will be device?
(a) 1 : 2 (b) 2 : 3 (c) 3 : 4 (d) 2 : 1 (a) Inductor (b) Switch (c) Fuse (d) Conductor

16. An electron is accelerated through a potential 24. A parallel plate capacitor of capacitance 20mF is being
charged by a voltage source whose potential is changing
difference of 10,000 V. at the rate of 3 V/s. The conduction current through the
Its de-Broglie wavelength is, (nearly) : connecting wires and the displacement current through
(me = 9 ´ 10 -31 kg) the plates of the capacitor, would be, respectively.
(a) 12.2 ´ 10-12 m (b) 12.2 ´ 10-14 m (a) 60 mA, 60 mA (b) 60 mA, zero
(c) 12.2 nm (d) 12.2 ´ 10-13 m (c) zero, zero (d) zero, 60 mA
NEET Solved Paper 2019 40 DAYS ~ NEET PHYSICS 03

25. When a block of mass M is suspended by a long wire 33. A disc of radius 2 m and mass 100 kg rolls on a
of length L, the length of the wire becomes (L + l). horizontal floor. Its centre of mass has speed of 20
The elastic potential energy stored in the extended cm/s. How much work is needed to stop it?
wire is
(a) 30 kJ (b) 2 J
1 1
(a) MgL (b) Mgl (c) MgL (d) Mgl (c) 1 J (d) 3 J
2 2
34. In an experiment, the percentage of error occurred in
26. A hollow metal sphere of radius R is uniformly the measurement of physical quantities A , B, C and D
charged. are 1%, 2%, 3% and 4% respectively. Then the maximum
The electric field due to the sphere at a distance r from percentage of error in the measurement X , where,
the centre A 2B1 / 2
(a) zero as r increases for r < R, decreases as r increases for X = 1 / 3 3 will be
r>R C D
3
(b) zero as r increases for r < R, increases as r increases for (a) 16% (b) -10% (c) 10% (d) æç ö÷ %
r>R è 13 ø
(c) decreases as r increases for r < R and for r > R
(d) increases as r increases for r < R and for r > R 35. Body A of mass 4m moving with speed u collides with
another body B of mass 2m, at rest. The collision is
27. A mass m is attached to a thin wire and whirled in a head on and elastic in nature. After the collision the
vertical circle. fraction of energy lost by the colliding body A is
The wire is most likely to break when: (a)
8
(b)
4
(c)
5
(d)
1
(a) the wire is horizontal 9 9 9 9
(b) the mass is at the lowest point
36. The radius of circle, the period of y
(c) inclined at an angle of 60° from vertical P(t=0)
(d) the mass is at the highest point revolution, initial position and
sense of revolution are indicated in T=4 s
28. Which colour of the light has the longest wavelength? the alongside figure. x
(a) Blue (b) Green (c) Violet (d) Red y-projection of the radius vector of 3m
rotating particle P is
29. A particle moving with velocity v is acted by three pt
forces shown by the vector triangle PQR. (a) y(t ) = 4sinæç ö÷, where y in m
è2 ø
The velocity of the particle will 3pt ö
(b) y(t ) = 3cosæç ÷, where y in m
P è 2 ø
pt
(c) y(t ) = 3cosæç ö÷, where y in m
è2 ø
(d) y(t ) = -3cos 2pt , where y in m

R Q 37. In total internal reflection when the angle of incidence


(a) decrease is equal to the critical angle for the pair of media in
(b) remain constant contact, what will be angle of refraction?
(c) change according to the smallest force QR (a) 0º (b) Equal to angle of incidence
(d) increase (c) 90º (d) 180º

30. Ionised hydrogen atoms and a -particles with same 38. A solid cylinder of mass 2 kg and radius 4 cm is
momenta enters perpendicular to a constant magnetic rotating about its axis at the rate of 3 rpm. The torque
field, B. required to stop after 2p revolutions is
The ratio of their radii of their paths rH : ra will be (a) 2 ´ 10-3 N-m (b) 12 ´ 10-4 N-m
(a) 1 : 2 (b) 4 : 1 (c) 1 : 4 (d) 2 : 1 (c) 2 ´ 106 N-m (d) 2 ´ 10-6 N-m

31. The unit of thermal conductivity is : 39. A block of mass 10 kg is in contact against the inner
(a) J m -1
K -1
(b) W m K -1
(c) W m -1
K -1
(d) J m K -1 wall of a hollow cylindrical drum of radius 1 m. The
coefficient of friction between the block and the inner
32. A coil of 800 turns effective area 0.05 m 2 is kept wall of the cylinder is 0.1. The minimum angular
perpendicular to a magnetic field 5 ´ 10 -5 T. When the velocity needed for the cylinder to keep the block
plane of the coil is rotated by 90 º around any of its stationary when the cylinder is vertical and rotating
co-planar axis in 0.1 s, the emf induced in the coil will about its axis, will be (g = 10 m/s 2 )
be 10
(a) rad/s (b) 10 rad/s (c) 10p rad/s (d) 10 rad/s
(a) 0.2 V (b) 2 ´ 10-3 V (c) 0.02 V (d) 2 V 2p
04 40 DAYS ~ NEET PHYSICS NEET Solved Paper 2019

40. A cylindrical conductor of radius R is carrying a 42. Increase in temperature of a gas filled in a container
constant current. The plot of the magnitude of the would lead to
magnetic field B with the distance d from the centre of (a) increase in its kinetic energy
the conductor, is correctly represented by the figure (b) decrease in its pressure
B
(c) decrease in intermolecular distance
B
(d) increase in its mass
(a) (b) 43. A copper rod of 88 cm and an aluminium rod of
unknown length have their increase in length
R d R d independent of increase in temperature. The length of
B B aluminium rod is
(a) 113.9 cm (b) 88 cm
(c) (d)
(c) 68 cm (d) 6.8 cm

44. a-particle consists of


R d R d
(a) 2 electrons, 2 protons and 2 neutrons
(b) 2 electrons and 4 protons only
41. In the circuits shown below, the readings of voltmeters (c) 2 protons only
and the ammeters will be (d) 2 protons and 2 neutrons only
10 Ω i1 10 Ω i2
45. In a double slit experiment, when light of wavelength
400 nm was used, the angular width of the first
10 Ω minima formed on a screen placed 1 m away, was
V1 V2 A2
A1
found to be 0.2º. What will be the angular width of the
first minima, if the entire experimental apparatus is
10 V 10 V immersed in water? (m water = 4/3)
(a) V1 = V2 and i1 > i 2 (b) V1 = V2 and i i = i 2 (a) 0.15º (b) 0.051°
(c) V2 > V1 and i1 > i 2 (d) V2 > V1 and i1 = i 2 (c) 0.1º (d) 0.266º

Answers
1. (a) 2. (c) 3. (a) 4. (a) 5. (d) 6. (b) 7. (a) 8. (b) 9. (b) 10. (c)
11. (b) 12. (b) 13. (c) 14. (c) 15. (a) 16. (a) 17. (b) 18. (b) 19. (a) 20. (b)
21. (d) 22. (b) 23. (c) 24. (a) 25. (b) 26. (a) 27. (b) 28. (d) 29. (b) 30. (d)
31. (c) 32. (c) 33. (d) 34. (a) 35. (a) 36. (c) 37. (c) 38. (d) 39. (b) 40. (b)
41. (b) 42. (a) 43. (c) 44. (d) 45. (a)
NEET Solved Paper 2019 40 DAYS ~ NEET PHYSICS 05

Answer with Explanations


1. (a) p-type semiconductors are obtained when a trivalent \ The equivalent resistance of this circuit is
impurity (e.g. boron, aluminium, gallium or indium) is Req = R A + RB
added to a intrinsic semiconductor. (e.g. germanium or As, section A has three parallel resistance, so equivalent
silicon). resistance,
In other words, the dopants in p-type semiconductor is R
RA =
trivalent atom. 3
Thus, this addition creates deficiencies of valence electron Similarly, for section B, equivalent resistance,
which are most commonly known as holes. These are the R
RB =
majority charge carriers in this type of semiconductor. However, 3
in n-type semiconductors, the dopants are pentavalent R R 2R
impurities. Also, the majority charge carriers are electrons. \ Req = + = … (i)
3 3 3
2. (c) Given, weight of the body, w = 200 N Thus, power consumed in this circuit,
As we know, w = mg, where m is the mass of the body and g V 2 E 2 3E 2
P1 = = = [using Eq. (i)] …(ii)
(» 10 m/s 2 ) is acceleration due to gravity of the body at the R Req 2R
surface of the earth.
Case II When two from section A and one from section B
Since, mass m remains constant irrespective of the position glow, the circuit can be realised as shown in the figure
of the body on the earth. However, g is not constant and its below.
value at a depth d below the earth’s surface is given as
A B
g¢ = gæç1 - ö÷
d
… (i) R
è Rø R
R
where, R is the radius of the earth.
Multiplying m on the both sides of Eq. (i), we get
mg¢ = mgæç1 - ö÷
d
è Rø

Thus, the weight of the body at half way down æç i.e. d = ö÷ to


R E
è 2ø \ Equivalent resistance of section A,
the centre of the earth is R
R / 2ö RA =
mg¢ = 200 ´ æç1 - æ 1ö 1
2
÷ = 200 ç1 - ÷ = 200 ´ = 100
è R ø è 2ø 2 and of section B, RB = R
\ The body will weigh 100 N half way down to the centre of Thus, equivalent resistance of the entire circuit becomes
the earth.
Req = R A + RB
3. (a) In an adiabatic process, the system is completely insulated R 3R
= + R= … (iii)
from the surroundings. Thus, heat is neither absorbed nor 2 2
released by the system to the surroundings. So, DQ = 0. \ Power consumed by this circuit,
Sudden processes are adiabatic like bursting of cycle tyre, etc. V2 E2 2E 2
P2 = = = [using Eq. (iii)]… (iv)
If the pressure of gas is kept constant, then the process is R Req 3R
called isobaric, i.e. Dp = 0.
So, ratio of power of two cases is obtained from Eqs. (ii) and
If the temperature of the system remains constant, then it is
(iv), we get
called isothermal process, i.e. DT = 0.
P1 3E 2 3R
If the volume of gas is constant in a system, then it is called = ´
isochoric process, i.e. DV = 0. P2 2R 2E 2
9
4. (a) Case I When all bulbs are glowing, then the circuit can = or 9 : 4
4
be realised as shown in the figure below.
5. (d) The rate of liquid flow moving with velocity v through an
A B
area a is given by
R R
Rate (R ) = Area (a) ´ Velocity (v)
R R Given, area of hole,
a = 2 mm 2 = 2 ´ 10-6 m 2
R R
height of tank, h = 2 m.

E
06 40 DAYS ~ NEET PHYSICS NEET Solved Paper 2019

The given situation can also be depicted as shown in the 8. (b) The motion of object shot in two cases can be depicted as
figure below. below

2m u u
60° 30°

60° g cos 60° 30° g cos 30°


g sin 60° g sin 30°
2 g g
a=2mm Case I Case II
Using third equation of motion, v2 = u2 − 2gh … (i)
As the velocity of liquid flow is given as v = 2gh As the object stops finally, so v = 0
∴R = av = a 2gh For inclined motion, g = gsinθ and h = x
Substituting the given values, we get Substituting these values in Eq. (i), we get
R = 2 × 10−6 × 2 × 10 × 2 u2
⇒ u2 = 2gsinθ x ⇒ x =
2gsinθ
= 2 × 10−6 × 6.32 = 12.64 × 10−6 m 3/s ~
− 12.6 × 10−6 m 3/s
u2
For case (I), x1 =
6. (b) Work done by a force F, which is variable in nature in 2gsin 60°
moving a particle from y1 to y2 is given by
u2
y2 For case (II), x 2 =
2gsin 30°
W= ∫ F ⋅ dy … (i)
x1 u2 2gsin 30° 1 2 1
⇒ = × = × =
y1
or 1 : 3
Here, force, F = 20 + 10 y, y1 = 0 and y2 = 1 m x 2 2gsin 60° u2 2 3 3
Substituting the given values in Eq. (i), we get
1 9. (b) The necessary conditions for the rainbow to take place is
1
 10 y 2 
⇒ W = ∫ (20 + 10 y)dy =  20 y + 
(i) Sun should be shining in part of the sky while it is
0  2 0 raining in opposite part of the sky.
(ii) The observer must stand with his back towards the sun.
= 20 (1 − 0) + 51
( − 0)2 = 25 J
∴Statement in option (b) is wrong.
∴Work done will be 25 J.
However, rest statements regarding the rainbow are correct.
7. (a) The force between two point charges A and B having
charge + Q and − Q respectively is given by 10. (c) The average velocity of a particle executing simple harmonic
Total displacement x f − x i
+Q –Q motion (SHM) is vav = =
Time interval T
A B
where, x f and x i are the initial and final position of the
r
particle executing SHM.
K Q AQB KQ(− Q) KQ2 As, in vibrational motion, the particle executes SHM about
F= 2
= 2
=− 2 …(i)
r r r its mean position. So, after one complete vibration of the
1 particle, it will reaches its initial position, i.e.
where, K = constant =
4 πε0 Displacement, x f − x i = 0
0
and r = distance between two charges A and B. ∴ vav =
T
When 25% charge of A is transferred to B, then new amount
of charge on A and B respectively become Hence, the average velocity is zero.

Q′ A =
75 3
(Q A) = Q 11. (b) According to Bohr’s model, the kinetic energy of electron
100 4 in term of Rydberg constant R is given by
−3
Rhc
KE = 2
Q′B =  Q + QB  =  Q − Q =
25 1 … (i)
Q
 100 A  4  4 n
where, h = Planck’s constant, c = speed of light
So, the force between the two charges now becomes and n = principal quantum number.
K    − Q
3Q 3 Similarly, potential energy is given by,
K Q′ A Q′B  4  4  2Rhc
F′ = = PE = − 2 … (ii)
r2 r2 n
−9KQ 2
9 Rhc
∴Total energy, E = PE + KE = − 2 [from Eqs. (i) and (ii)]
= = F [from Eq. (i)]
16r 2 16 n
⇒ KE = − E and PE = 2E
Thus, the new force between the charges is 9/16 time the
Given, E = − 34
. eV
initial force between the charges.
∴ KE = − (− 34
. ) = 34
. e V and PE = 2 (− 34. ) = − 6.8 eV
NEET Solved Paper 2019 40 DAYS ~ NEET PHYSICS 07

12. (b) The angle of dip (δ) is the angle BV Case II When glycerine of same
Glass
between the horizontal refractive index at that of the glass Glass
component of earth’s magnetic –ve is filled in the space between two
field and the total magnetic field lens, then the combination will
of the earth. Its value is different δ now comprises of three lenses; first
BH
at different places. It is zero at δ bi-convex, second bi-concave and Glycerine (µ=1.5)
equator, as the dip neddle third is bi-convex. So, the focal
becomes parallel to horizontal +ve length of the combination now is given as
component. It varies from − 90° in 1 1 1 1 1
= + + = ⇒ F2 = f … (ii)
South pole to + 90° in the North pole. F2 f (− f) f f
This means the values δ is positive in northern hemisphere
From Eqs. (i) and (ii), we get F1 : F2 = 1 : 2
and is negative in southern hemisphere.
∴For point A, δ = + 25°, 16. (a) Given, potential difference, V = 10000 V
So, A lies in the northern hemisphere. If electron is accelerated through a potential of V volt, then
Similarly, for B, δ = − 25°, so the wavelength associated with it is given by
h
B lies in the southern hemisphere. λ= … (i)
2eVme
13. (c) Electric heaters are not based on the eddy current effect.
Rather their working is based on Joule’s heating effect of where, h = Planck’s constant = 6.63 × 10−34 J-s,
current. According to this effect, the passage of an electric e = electronic charge = 1.6 × 10−19C
current through a resistor produces heat. and me = mass of electron = 9 × 10−31 kg
However, when a changing magnetic flux is applied to a bulk Substituting these values in Eq. (i), we get
piece of conducting material, then circulating current is
12.27 12.27 12.27 × 10−10
induced in the body of this conductor, which is usually λ= Å= × 10−10 = = 12.27 × 10−12 m
known as eddy currents. This current shows both heating V 10000 100
and magnetic effects. 17. (b) The LED will glow when the current flows through it, i.e.
Thus, it is the basic principle behind the working of when the voltage across it is high. The truth table can be
magnetic braking in train, electromagnet and induction formed from this
furnace. +6V

14. (c) The angular speed of a particle in a uniform circular R


angle of circle 0
motion is given by ω =
Time A 1
2π Y
ω= , where T is the time period of rotation
T
2π R
For particle A, ω A =
TA 0

For particle B,ωB = B 1
TB
ω A 2π TB T 1 A B Y
∴ = × = B = or 1 : 1 [Q TA = TB (given)]
ωB TA 2π TA 1 0 0 1
0 1 1
15. (a) Key Idea When the space between two convex lenses is
filled with a liquid of refractive index same as that of glass of 1 0 1
lens, then it behaves like a diverging lens, i.e. it forms a 1 1 0
bi-concave lens.
The output Y is same as that come from NAND gate.
Case I When two equi-convex lens of focal length f1 and f2
respectively, are kept co-axially in contact, then the equivalent 18. (b) The excess pressure inside a soap bubble of radius r is
4T
focal length of combination is given by p=
r
Glass Glass where, T = surface tension.
If p0 be the outside pressure from the water, then total
pressure inside the bubble becomes
4T
Air p1 = p0 + … (i)
r
1 1 1 1 1 2
= + = + = [QHere, f1 = f2] The pressure at the depth Z0 below the water surface is
F1 f1 f2 f f f
p2 = p0 + Z0ρg … (ii)
f
⇒ F1 = … (i) As it is given that the pressure inside the bubble is same as
2 the pressure at depth Z0 , then equating
08 40 DAYS ~ NEET PHYSICS NEET Solved Paper 2019

Eqs. (i) and (ii), we get 21. (d) Given, speed of river, vR = 10 m/s
4T 4T
p0 + = p0 + Z0ρg ⇒ Z0 = … (iii) speed of swimmer in still water and vSN = 20 m/s.
r rρg
−2 vR
Here, T = 2.5 × 10 N/m, ρ = 10 kg/m ,
3 3
River flow
g = 10 ms−2 and r = 1 mm = 1 × 10−3 m
N
Substituting these values in Eq. (iii), we get vSN vS
4 × 2.5 × 10−2 W E
Z0 = = 10 × 10−3 m = 1 cm
1 × 10−3 × 103 × 10 θ
S
19. (a) The displacement of given particle is
For the shortest path to cross the river, he should swim at
y = A0 + Asinωt + B cosωt … (i) an angle (90° + θ) with the stream flow. From the figure
The general equation of SHM can be given as above, v SN = v R + v S
x = a sinωt + b cosωt … (ii) So, angle θ is given by
So, from Eqs. (i) and (ii), we can say that A0 be the value of v 10 1
sinθ = R = =
mean position, at which y = 0. v SN 20 2
∴Amplitude, R = A2 + B2 + 2 AB cosθ ⇒ θ = 30°
As two function sine and cosine have phase shift to 90°. As the river is flowing in East direction, so he should swim
towards West.
∴ R= A 2 + B2 [Q cos 90° = 0]
22. (b) Consider two infinite line charges with linear charge
20. (b) Key Idea Amount of work done in moving the given body densities + λ C/m and − λ C/m respectively, which are lying
from one point to another against the gravitational force is in y-direction as shown in the figure below.
equal to the change in potential energy of the body. +λ –λ
+ EB –
As we know, the potential energy of body of mass m of the + EA –
surface of earth is + –
+ –
GMm R
U1 = − … (i) + –
R + –
+ 2R –
where, G = gravitational constant,
A B
M = mass of earth and R = radius of earth.
Then, the electric field due to line A at the mid way between
When the mass is raised to a height h from the surface of the
the two line charges, i.e. at R is
earth, then the potential energy of the body becomes
λ
|E A| = N/C … (i)
2πε0 R
h=R which lies along +ve x-axis (outward), i.e. from A to B.
Similarly, the electric field due to line B at the mid-way
between the two line charges, i.e. at R is
λ
R |EB| = N/C … (ii)
2πε0 R
M Due to negative charge on B, EB also lies along
+ve x-axis (inward), i.e. from A to B.
So, the resultant electric field at R is given as
|ER| = |E A| + |EB|
GMm
U2 = − Substituting the values from Eqs. (i) and (ii), we get
(R + h) λ λ
|ER|= +
Here, h = R (given) GMm 2πε0 R 2πε0 R
⇒ U2 = − … (ii)
2R λ
= N/C
Thus, the change in potential energy, ∆U = U 2 − U1 πε0 R
Substituting the values from Eqs. (i) and (ii), we get Which also lies along the + ve x-axis, i.e. from A to B.
GMm GMm GMm gR2m 
Qg = 2 
GM
∆U = − + = = 23. (c) Among given devices fuse is used in electric circuit as a
2R R 2R 2R  R  protective device.
mgR
= It helps in preventing excessive amount of current to flow in
2 the circuit or from short circuiting.
Thus, the work done in raising the mass to a height R is It has low melting point and low resistivity, so when excess
mgR
equals to . amount of current flow in the circuit, it melts and break the
2 circuit.
NEET Solved Paper 2019 40 DAYS ~ NEET PHYSICS 09

24. (a) Key Idea The displacement current is precisely equals to ⇒ E (4 πr 2) =


Q
⇒ E=
Q
⇒E ∝
1
the conduction current, when the two are present in different ε0 4 πε0 r 2 r2
parts of the circuit.

Given, C = 20µF = 20 × 10−6 F and (ii) The electric field at the surface (r = R),
dV
= 3 V/s
dt Q
E=
The displacement current in a circuit is given by 4 πε0 R

I d = ε0 [from Maxwell’s equation) (iii) The electric field inside hollow sphere is
dt
Q
d E= =0 [Q Qinside = 0]
= ε0 (EA) [Q φ = EA] 4 πε0 r
dt
Thus, the electric field is zero inside the hollow sphere
= ε0 A  
d V
[Q V = Ed] maximum at the surface and decreases as r increases for
dt  d 
r > R. This can be shown graphically as
ε A dV
= 0 E
d dt
ε A Emax
As the capacitance, C = 0
d
dV E ∝ 12
∴ Id = C E=0 r
dt
Substituting the given values, we get r
O
I d = 20 × 10−6 × 3 = 60 × 10−6 A = 60 µ A R

As displacement current is in between the plates of capacitor 27. (b) Let the mass m which is attached to a thin wire and is
and conduction current is in the connecting wires which are whirled in a vertical circle is shown in the figure below.
equal to each other. So, I c = I d = 60 µA C
25. (b) In stretching a wire, the work done against internal
restoring force is stored as elastic potential energy in wire
and given by
D B
T
L L l m
P
θ mg cos θ
A
mg
l
The tension in the string at any point P be T.
M According to Newton’s law of motion, In equilibrium, net force
towards the centre = centripetal force
Mg mv2
⇒ T − mg cosθ =
1
U=W= × Force (F) × Elongation ()
l l
2 Here, l = length of wire and v = linear velocity of the particle
1 1 1 whirling in a circle.
= Fl = × Mg × l = Mgl
2 2 2 mv2
⇒ T = mg cosθ +
l
26. (a) As the hollow sphere is uniformly charged, so the net mv2
charge will appear on the surface of the sphere. At A, θ = 0° ⇒ TA = mg +
l
(i) The electric field at a point outside the hollow sphere is mv2
At B, θ = 90° ⇒ TB =
φ = ∫ E ⋅ dS = enclosed
Q
[from Gauss’ law] l
ε0 mv2
S
At C, θ = 180° ⇒ TC = − mg +
E l
mv2
At D, θ = 90° ⇒ TD = TB =
dS l
++ + So, from the above analysis, it can be concluded that the
+
+ + tension is maximum at A i.e. the lowest point of circle, So
+ chance of breaking is maximum.
+
R +
+ O
r
28. (d) Different colours of white light have different
+ + wavelengths. The descending order of the wavelength of the
+ Q component of white light is
+ + + λ Red > λ Green > λ Blue > λ Violet
10 40 DAYS ~ NEET PHYSICS NEET Solved Paper 2019

29. (b) As the three forces are represented by three sides of a Thus, substituting the given values in Eq. (ii), we get
triangle taken in order, then they will be in equilibrium. 800 × 5 × 10−5 × 0.05 × [cos 90° − cos 0° ]
e=−
P .
01
= 2000 × 10−5 = 0.02 V

33. (d) Given, radius R = 2 m, mass, m = 100 kg and


vCM (velocity centre of mass) = v = 20 cm/s = 20 × 10−2 m/s.
Then, according to work energy theorem, the work done in
Q stopping the disc is equal to the change in its kinetic energy,
R
i.e. W = KE f − KE i
⇒ Fnet = FPQ + FQR + FRP = 0
As, the disc stops at the end, so final velocity is zero. Thus,
dv
⇒ Fnet = m × a = m =0 KE f = 0
dt Since, the disc is rolling so, its initial kinetic energy would
dv
⇒ = 0 or v = constant have both rotational and translational kinetic energy
dt component.
So, the velocity of particle remain constant. 1 1
KE i = KE R + KE T = Iω2 + mv2
30. (d) The centripetal force required for circular motion is 2 2
provided by magnetic force 1 1 2 2 1 2 Q for disc, I = 1 mR2 
=  mR  ω + mv
mvp2 mvp 2 2  2  2 
⇒ = Bqvp ⇒ r = … (i)
1 1 1 v2 1
r qB = mR2ω2 + mv2 = mR2 2 + mv2 [Q v = rω]
where, vp = perpendicular velocity of particle and 4 2 4 R 2
q = charge on particle. 3
= mv2
As, momentum, p = mvp 4
3
∴ r=
p
[from Eq. (i)] ∴ W = |KE i| = mv2
qB 4
According to the question, moment of both particle is same. Substituting the given values, we get
3 3
⇒ r∝
1 W = × 100 × (20 × 10−2)2 = × 400 × 100 × 10−4 = 3 J
q 4 4
2 1/ 2
For ionised hydrogen atom, q H = e 34. (a) Given, X = A1 /B3 3
r q 2e 2 C D
and for α-particle, q α = 2e ⇒ H = α = = or 2 : 1
rα qH e 1 The percentage error in X is given by
∆X ∆A  1  ∆B 
31. (c) The rate of heat flow through a conductor of length L × 100 = 2  × 100 +   × 100
X  A 2 B 
and area of cross-section A is given by
1 ∆C   ∆D  × 100 …(i)
dQ
= KA
∆T
J/s or watt +   × 100 + 3  
dt L 3 C   D 
where, K = coefficient of thermal conductivity and ∆A ∆B
Here, × 100 = 1%, × 100 = 2%,
∆T = change in temperature A B
∆C ∆D
⇒ K =
L dQ × 100 = 3%, × 100 = 4%
A ∆T dt C D
metre Substituting these values in Eq. (i), we get
∴Unit of K = × watt = Wm −1 K −1 ∆X 1 1
(metre)2 × kelvin × 100 = 2(1%) + (2%) + (3%) + 3(4%)
X 2 3
32. (c) Given, area of coil, A = 0.05 m 2 = 2% + 1% + 1% + 12% = 16%
magnetic field, B = 5 × 10−5 T and Thus, maximum % error in X is 16%.
number of turns, N = 800 . 35. (a) Key Idea In head-on elastic collision, momentum and
The magnetic flux linked with the coil is kinetic energy before and after the collision is conserved.
φ = N(B ⋅ A) = N BA cosθ … (i) The given situation of collision can be drawn as
where, θ is the angle between B and A. 4m 2m 4m 2m
The emf induced when coil is rotated from θ1 = 0° to θ2 = 90°
is u u'= 0 v1 v2
∆φ ∆
e=− =− (NBA cosθ) [using Eq. (i)] A B A B
∆t ∆t 144442444443 144442444443
NBA Before collision After collision
=− (cosθ2 − cosθ1) … (ii)
∆t
Applying conservation of linear momentum,
Here, ∆t = 01. s
Initial momentum of system = Final momentum of system
NEET Solved Paper 2019 40 DAYS ~ NEET PHYSICS 11

⇒ (4m)u + (2m)u′ = (4m)v1 + (2m)v2 37. (c) The total internal reflection Normal
4mu = 4mv1 + 2mv2 or 2 u = 2v1 + v2 … (i) is the phenomenon of reflection
The kinetic energy of A before collision is back of light in the denser
1 medium when it travel from
KE A = (4m)u2 = 2mu2 ic
2 denser to rarer medium, when Denser
Kinetic energy of B before collision, KE B = 0 the angle of incidence is greater
The kinetic energy of A after collision is than the critical angle. While r=90°
the critical angle for a pair of Rarer
1
KE′A = (4m)v12 = 2 mv12 given media in contact is the
2
angle of incidence in denser
Kinetic energy of B after collision, medium for which the angle of
1
KE′B = (2m)v22 = mv22 refraction in rarer medium is 90°.
2
As, Initial kinetic energy of the system = Final kinetic energy 38. (d) Key Idea According to work-energy theorem, the
of the system change in kinetic energy of a particle is the amount
of work done on the particle to move, i.e.
⇒KE ′A + KE ′B = KE ′A + KE B′
W = − ∆KE = KE f − KE i
2mu2 + 0 = 2mv12 + mv22
2mu2 = 2mv12 + mv22 or 2u2 = 2v12 + v22 … (ii) Given, mass of cylinder, m = 2 kg
Solving Eqs. (i) and (ii), we get radius of cylinder, r = 4 cm = 4 × 10−2 m
1 4 2π π
v1 = u and v2 = u rotational velocity, ω = 3 rpm = 3 × = rad/s and θ = 2π
3 3 60 10
or the final velocity of A can be directly calculated by using revolution = 2π × 2π = 4 π rad.
2
the formula.
The velocity after collision is given by The work done in rotating an object by an angle θ from rest is
 m − m2  given by W = τθ
2m2u2
v1 =  1  u1 + As the cylinder is brought to rest, so the work done will be
 m1 + m2  m1 + m2 negative.
 4m − 2m  2(2m) × 0 According to work-energy theorem,
= u + [Q u2 = u′ = 0]
 4m + 2m  (4m + 2m) Work done = Change in rotational kinetic energy
2m 1 1 1 1 I(−ω2i )
= u= u − τθ = Iω2f − Iωi2 = I(ω2f − ωi2) ⇒ τ =
6m 3 2 2 2 2θ
∴Net decreases in kinetic energy of A [Qω f = 0]
∆KE = KE A − KE ′A = 2mu2 − 2mv12 = 2m(u2 − v12) ω2
=  mr 2  i  I = 1 MR2 (for cylinder)
1 1
Substituting the value of v1 , we get 22  θ  2 
 u2  16 mu2 1 ω 2
∆KE = 2m  u2 −  = = mr 2 [Qωi = ω]
 9 9 4 θ
∴The fractional decreases in kinetic energy is π
2
= × 2 × (4 × 10−2)2 ×   ×
1 1
∆KE 16 mu2 1 8 4  10  4π2
= × =
KE A 9 2mu2 9 1 π2 1
= × 2 × 16 × 10−4 × ×
36. (c) Let O be the centre of circle, then at t = 0, the 4 100 4 π 2
2
displacement y is maximum and have value 3 m. = × 10−4 = 2 × 10−6 N-m
y 100
t=0 39. (b) Given, mass of cylinder m = 10 kg,
t=t radius of cylinder, r = 1 m ω
ωt coefficient of friction, µ = 01 ..
The given situation can be as shown
O x fl
3m in the figure given alongside.
From the above figure, it can be mrω 2 N
concluded that the block will be
T=4s mg
stationary when the limiting friction
As, the general equation of displacement of a particle will be ( fl) is equal to or greater than the
in the form y = A cosωt downward force or weight of block,
Here, A = 3m i.e.
2π 2π π
Then, ω= = = [given, T = 4 s] fl ≥ mg …(i)
T 4 2
Also, the magnitude of limiting friction between two bodies
π
∴ y = 3cos  t  (in metre) is directly proportional to the normal reaction (N) between
2  them, i.e. fl ∝ N or fl = µ N …(ii)
12 40 DAYS ~ NEET PHYSICS NEET Solved Paper 2019

From Eqs. (i) and (ii), we get Similarly, for circuit 2, the addition of 10 Ω to voltmeter does
µN ≥ mg or µ( mrω2 ) ≥ mg [Q N = mrω2] not affect the current and hence
10 Ω i2
g
⇒ ω≥

10 Ω
Thus, the minimum angular velocity is V2
A2
g 10
ωmin = = = 10 rad/s
rµ 1 × 01.

40. (b) The cylinder can be considered to be made from Circuit 2


10 V
concentric circles of radius R. 10
10 i 2 = 10 ⇒ i 2 = = 1 A ⇒ V2 = i 2R = 1 × 10 = 10 V
R 10
∴ V1 = V2 and i1 = i 2

d 42. (a) As the temperature of gas in the container is increased,


the kinetic energy also increases. This is because the average
kinetic energy of a gas is given by
f
KE = nRT … (i)
(i) The magnetic field at point outside cylinder, i.e. d > R. 2
From Ampere’s circuital law, ∫ B ⋅ dl = µ 0 I where, f = degree of freedom,
n = number of moles of gas molecules,
µ 0I
⇒ B∫ dl = µ 0 I ⇒ B(2πd) = µ 0 I ⇒ B= R = universal gas constant, and T = absolute temperature of
2π d the gas.
where, µ 0 = permeability of free space. From Eq. (i), KE ∝ T
(ii) The magnitude field at surface, i.e. d = R option (b) is incorrect as increase in temperature will lead to
µ I increase in pressure as p ∝ T. Other options (c) and (d) are
B= 0
2πR also incorrect as molecular distance increases while mass
(iii) The magnetic field at inside point. The current for a remains the same for increase in the temperature.
point inside the cylinder is given by I′ = current per unit
cross-sectional area of cylinder × cross-section of loop
43. (c) Due to change in temperature, the thermal strain
produced in a rod of length L is given by
I Id 2 ∆L
= πd 2 = 2 = α ∆T ⇒ ∆L = L α ∆T
πR 2
R L
µ 0 I ′ µ 0 Id 2 µ I where l = original length of rod and α = coefficient of liner
∴ B= = = 0 d expansion of solid rod
2πd 2πR2d 2πR2
As the change in length (∆l) of the given two rods of copper
So, the variation of magnetic field can be plotted as
and aluminium are independent of temperature change, i.e.
B ∆T is same for both copper and aluminium.
LCuα Cu = LAlα Al … (i)
Here, α Cu = 1.7 × 10−5 K −1
B∝d
B∝ 1 . × 10−5 K −1 and
α Al = 22 LCu = 88 cm
d
Substituting the given values in Eq. (i), we get
L α 1.7 × 10−5 × 88 ~
LAl = Cu Cu = − 68 cm
O d =R
d α Al . × 10−5
22

41. (b) For an ideal voltmeter, the resistance is infinite and for an 44. (d) α-particles are doubly ionised helium nucleus (He 2+ )
ideal ammeter, the resistance is zero.
which are emitted in any radioactive process. So, they have
10 V i1 two protons, 2 neutrons in its nucleus and no electron.

45. (a) The angular width in YDSE is given by θ = β


D
V1 A1 where, β is the separation between two fringes. D is the
distance between the planed the slits and screen. If YDSE
apparatus is immersed in a liquid of refractive index µ, then
the wavelength of light and hence the angular width
Circuit 1
10 V decreases µ times.
So, the current in circuit 1 is β θ
i.e. θ′ = =
10 µD µ
R ×i = V or 10 i1 = 10 ⇒ i1 = =1A
10 0.2
Here, µ (for water) = 4 / 3 and θ = 0.2° ⇒θ′ = = 015
. °
∴ V1 = i1 × R = 1 × 10 = 10 V 4/ 3
NEET Solved Paper 2019 40 DAYS ~ NEET PHYSICS 13

NEET (Odisha)
SOLVED PAPER 2019
Held on : 20 May 2019
Duration : 3 Hours Max. Marks : 180

Important Instructions
Test Booklet contains 45 questions.
Each question carries 4 marks. For each correct response, the candidate will get 4 marks.
For each incorrect response, one mark will be deducted from the total scores.
The maximum marks are 180.

1. Two metal spheres, one of radius R and the other of The velocity of the larger piece will be
radius 2R respectively have the same surface charge (a) 4$i + 23$j − 8k$ (b) − 100$i − 35$j − 8k $
density σ. They are brought in contact and separated. (c) 20$i + 15$j − 80k$ (d) − 20 i − 15 j − 80k
$ $ $
What will be the new surface charge densities on them?
5 5 5 5 6. An object kept in a large room having air temperature
(a) σ1 = σ, σ 2 = σ (b) σ1 = σ, σ 2 = σ of 25°C takes 12 minutes to cool from 80°C to 70°C.
6 2 2 6
5 5 5 5 The time taken to cool for the same object from 70°C to
(c) σ1 = σ, σ 2 = σ (d) σ1 = σ, σ 2 = σ
2 3 3 6 60°C would be nearly
(a) 10 min (b) 12 min
2. The distance covered by a particle undergoing SHM in (c) 20 min (d) 15 min
one time period is (amplitude = A)
(a) zero (b) A 7. Two small spherical metal balls, having equal masses,
(c) 2A (d) 4A are made from materials of densities ρ1 and ρ2 (ρ1 = 8 ρ2)
and have radii of 1 mm and 2 mm, respectively. They
3. A mass falls from a height ‘h’ and its time of fall ‘t’ is are made to fall vertically (from rest) in viscous
recorded in terms of time period T of a simple medium whose coefficient of viscosity equals η and
pendulum. On the surface of earth it is found that t = 2 whose density is 0.1 ρ2 . The ratio of their terminal
T. The entire set up is taken on the surface of another velocities would be
planet whose mass is half of earth and radius the same. 79 19
Same experiment is repeated and corresponding times (a) (b)
72 36
noted as t′ and T ′.
39 79
(a) t ′ = 2 T ′ (b) t ′ > 2 T ′ (c) (d)
72 36
(c) t ′ < 2T ′ (d) t ′ = 2 T ′
8. A particle starting from rest, moves in a circle of radius
4. A tuning fork with frequency 800 Hz produces resonance ‘r’. It attains a velocity of v0 m/s in the nth round. Its
in a resonance column tube with upper end open and angular acceleration will be
lower end closed by water surface. Successive resonance v0 v 02
are observed at length 9.75 cm, 31.25 cm and 52.75 cm. (a) rad/s 2 (b) rad/s 2
n 2 πnr 2
The speed of sound in air is v 02 v2
(a) 500 m/s (b) 156 m/s (c) 344 m/s (d) 172 m/s (c) rad/s 2 (d) 0 rad/s 2
4πnr 2 4πnr
5. An object flying in air with velocity (20 $i + 25$j − 12k)
$
9. A person standing on the floor of an elevator drops a
suddenly breaks in two pieces whose masses are in the coin. The coin reaches the floor in time t1 if the elevator
ratio 1 : 5. The smaller mass flies off with a velocity is at rest and in time t2 if the elevator is moving
(100 $i + 35 $j + 8 k$ ). uniformly.
14 40 DAYS ~ NEET PHYSICS NEET Solved Paper 2019

The which of the following option is correct? (a) 10.4 × 10−26 m (b) 654 nm
(a) t 1 < t 2 or t 1 > t 2 depending upon whether the lift is going up (c) 654 Å (d) 654 × 10−11 m
or down
(b) t 1 < t 2 14. The circuit diagram shown here corresponds to the
logic gate,
(c) t 1 > t 2
(d) t 1 = t 2 +6V
0 R
A
10. A truck is stationary and has a bob suspended by a light
1
string, in a frame attached to the truck. The truck, 0
B
suddenly moves to the right with an acceleration of a. 1
The pendulum will tilt LED (Y)
(a) to the left and the angle of inclination of the pendulum with R
the vertical is sin−1  
g
 a
(b) to the left and angle of inclination of the pendulum with the (a) NOR (b) AND
 a (c) OR (d) NAND
vertical is tan−1  
g Cp 

(c) to the left and angle of inclination of the pendulum with the 15. The value of γ  =  , for hydrogen, helium and
 a
 CV 
vertical is sin1−  
g another ideal diatomic gas X (whose molecules are not
rigid but have an additional vibrational mode), are
(d) to the left and angle of inclination of the pendulum with the
respectively equal to
vertical is tan−1  
g
 a 7 5 9 5 7 9
(a) , , (b) , ,
5 3 7 3 5 7
11. In a u-tube as shown in a figure, water and oil are in the 5 7 7 7 5 7
(c) , (d) , ,
left side and right side of the tube respectively. The 3 5 5 5 3 5
heights from the bottom for water and oil columns are
15 cm and 20 cm respectively. The density of the oil is 16. An equi-convex lens has power P it is cut into two
[take ρwater = 1000 kg/m 3 ] symmetrical halves by a plane containing the principal
axis. The power of one part will be
P P
(a) 0 (b) (c) (d) P
2 4

17. In a Young’s double slit experiment, if there is no


20 cm initial phase-difference between the light from the two
15 cm slits, a point on the screen corresponding to the fifth
minimum has path difference.
λ λ λ λ
(a) 5 (b) 10 (c) 9 (d) 11
2 2 2 2

Water Oil 18. A double convex lens has focal length 25 cm. The
(a) 1200 kg/m 3 (b) 750 kg/m 3 radius of curvature of one of the surfaces is doubled of
(c) 1000 kg/m 3 (d) 1333 kg/m 3 the other. Find the radii, if the refractive index of the
material of the lens is 1.5.
12. A deep rectangular pond of surface area A, containing (a) 100 cm, 50 cm
water (density = ρ, specific heat capacity = s), is located (b) 25 cm, 50 cm
in a region where the outside air temperature is a (c) 18.75 cm, 37.5 cm
steady value at the − 26 °C. The thickness of the frozen (d) 50 cm, 100 cm
ice layer in this pond, at a certain instant is x.
Taking the thermal conductivity of ice as K, and its 19. Two bullets are fired horizontally and simultaneously
specific latent heat of fusion as L, the rate of increase of towards each other from roof tops of two buildings 100
the thickness of ice layer, at this instant would be given m apart and of same height of 200 m with the same
by velocity of 25 m/s. When and where will the two
(a) 26K/ρ r(L−4s) (b) 26K/(ρx2 − L) bullets collides. (g = 10 m/s 2 )
(c) 26K/(ρxL) (d) 26K/ρr(L + 4s) (a) After 2s at a height 180 m
(b) After 2s at a height of 20 m
13. An LED is constructed from a p-n junction diode using (c) After 4s at a height of 120 m
GaAsP. The energy gap is 1.9 eV. The wavelength of the (d) They will not collide
light emitted will be equal to
NEET Solved Paper 2019 40 DAYS ~ NEET PHYSICS 15

20. The stress-strain curves are drawn for two different 26. A straight conductor carrying current i splits into two
materials X and Y. It is observed that the ultimate parts as shown in the figure. The radius of the circular
strength point and the fracture point are close to each loop is R. The total magnetic field at the centre P at the
other for material X but are far apart for material Y. We loop is
can say that materials X and Y are likely to be
(respectively)
i1
(a) ductile and brittle (b) brittle and ductile R
(c) brittle and plastic (d) plastic and ductile
i P
21. A body of mass m is kept on a rough horizontal surface 90º
(coefficient of friction = µ). Horizontal force is applied on i2
the body, but it does not move. The resultant of normal
i
reaction and the frictional force acting on the object is
given F, where F is (a) Zero (b) 3 µ 0 i / 32 R, outward
(a) |F| = mg + µ mg (b) |F| = µmg µ i
(c) 3 µ 0 i /32R, inward (d) 0 , inward
(c) |F| ≤ mg 1 + µ 2 (d) |F| = mg 2R

22. A particle of mass 5 m at rest suddenly breaks on its 27. The variation of EMF with time for four types of
own into three fragments. Two fragments of mass m generators are shown in the figures. Which amongst
each move along mutually perpendicular direction with them can be called AC?
each speed v. The energy released during the process is E E
3 5
(a) mv 2 (b) mv 2
5 3
3 4 (a) t (b) t
(c) mv 2 (d) mv 2
2 3

23. An object of mass 500 g, initially at rest acted upon by a


E E
variable force whose X component varies with X in the
manner shown. The velocities of the object a pointX = 8
m and X = 12 m, would be the respective values of (c) t (d) t
(nearly)
F (N)
(a) (a) and (d) (b) (a), (b), (c) and (d)
20
(c) (a) and (b) (d) only (a)

28. The radius of the first permitted Bohr orbit for the
10
electron, in a hydrogen atom equals 0.51 Å and its
x (m) ground state energy equals − 13.6 eV. If the electron in
4 5 8 10 12 the hydrogen atom is replaced by muon (µ −1) [Charge
–10 same as electron and mass 207 me ], the first Bohr
radius and ground state energy will be
–20 (a) 0.53 × 10−13 m, − 3.6 eV (b) 25.6 × 10−13 m, − 2.8 eV
–25
(c) 2.56 × 10−13 m, − 2.8 keV (d) 2.56 × 10−13 m, − 13.6 eV
(a) 18 m/s and 24.4 m/s (b) 23 m/s and 24.4 m/s
(c) 23 m/s and 20.6 m/s (d) 18 m/s and 20.6 m/s 29. The reading of an ideal voltmeter in the circuit shown
is
24. A solid cylinder of mass 2 kg and radius 50 cm rolls up 20 Ω 30 Ω
an inclined plane of angle inclination 30°. The centre of
mass of cylinder has speed of 4 m/s. The distance
travelled by the cylinder on the inclined surface will be : V
(Take g = 10 m/s 2 )
(a) 2.2 m (b) 1.6 m (c) 1.2 m (d) 2.4 m 30 Ω 20 Ω

25. Two toroids 1 and 2 have total number of turns 200 and
100 respectively with average radii 40 cm and 20 cm 2V
respectively. If they carry same current i, the ratio of the
(a) 0.6 V (b) 0 V
magnetic fields along the two loops is (c) 0.5 V (d) 0.4 V
(a) 1 : 1 (b) 4 : 1 (c) 2 : 1 (d) 1 : 2
16 40 DAYS ~ NEET PHYSICS NEET Solved Paper 2019

30. The meter bridge shown in the balance position with 37. A sphere encloses an electric dipole with charge
P l1
= . If we now interchange the positions of ± 3 × 10 −6 C. What is the total electric flux across the
Q l2 sphere?
galvanometer and cell, will the bridge work? If yes, that (a) − 3 × 10−6 N-m 2 /C (b) zero
will be balanced condition? (c) 3 × 106 N-m 2 /C (d) 6 × 10−6 N-m 2 /C

38. Two identical capacitors C1 and C 2 of equal capacitance


P Q are connected as shown in the circuit.
k
G a c

b
l1 l2
V C1 C2
P l2 − l1
(a) yes, = (b) no, no null point
Q l2 + l1
P l P l1
(c) yes, = 2 (d) yes, =
Q l1 Q l2 Terminals a and b of the key k are connected to charge
capacitor C1 using battery of emf V volt. Now,
31. The relations amongst the three elements earth’s disconnecting a and b the terminals b and c are
magnetic field, namely horizontal component H, connected. Due to this, what will be the percentage
vertical component V and dip δ are, (BE = total magnetic loss of energy?
field) (a) 75% (b) 0% (c) 50% (d) 25%
(a) V = BE tan δ, H = BE
(b) V = BE sinδ, H = BE cos δ 39. The main scale of a vernier calliper has n divisions/cm.
(c) V = BE cos δ, H = BE sin δ n divisions of the vernier scale coincide with (n −1)
(d) V = BE , H = Be E tan δ divisions of main scale. The least count of the vernier
callipers is
32. The rate of radioactive disintegration at an instant for a 1 1
(a) cm (b) cm
radioactive sample of half life2 .2 × 10 9 s is 10 10 s −1 . The (n + 1) (n − 1) n
number of radioactive atoms in that sample at that 1
(c) 2 cm (d)
1
cm
instant is n n (n + 1)
. × 1020
(a) 317 . × 1017
(b) 317
. × 10
(c) 317 18
. × 1019
(d) 317 40. A person travelling in a straight line moves with a
constant velocity v1 for certain distance ‘x’ and with a
33. The time period of a geo-stationary satellite is 24 h, at a constant velocity v2 for next equal distance. The
height 6 RE (RE is the radius of earth) from surface of average velocity v is given by the relation
earth. The time period of another satellite whose height 1 1 1 2 1 1
(a) = + (b) = +
is 2.5 RE from surface will be v v1 v 2 v v1 v 2
24 12 v v + v2
(a) 6 2 h (b) 12 2 h (c) h (d) h (c) = 1 (d) v = v1v 2
2.5 2.5 2 2
34. A circuit when connected to an AC source of 12 V gives 41. Assuming that the gravitational potential energy of an
a current of 0.2 A. The same circuit when connected to a object at infinity is zero, the change in potential energy
DC source of 12 V, gives a current of 0.4 A. The circuit is (final – initial) of an object of mass m, when taken to a
(a) series LR (b) series RC height h from the surface of earth (of radius R) is given
(c) series LC (d) series LCR by,
GMm GMmh
35. A cycle wheel of radius 0.5 m is rotated with constant (a) − (b)
angular velocity of 10 rad/s in a region of magnetic field R+ h R (R + h)
of 0.1 T which is perpendicular to the plane of the wheel. GMm
(c) mgh (d)
The EMF generated between its centre and the rim is R+ h
(a) 0.25 V (b) 0.125 V
42. 1g of water, of volume 1 cm 3 at 100°C is converted into
(c) 0.5 V (d) zero
steam at same temperature under normal atmospheric
36. For a transparent medium relative permeability and − 1 × 10 5 Pa). The volume of steam formed
pressure= (~
permittivity µ r and ∈r are 1.0 and 1.44 respectively. The equals 1671 cm 3 . If the specific latent heat of
velocity of light in this medium would be vaporisation of water is 2256 J/g, the change in
(a) 2.5 × 108 m/s (b) 3 × 108 m/s internal energy is
(c) 2.08 × 10 m/s
8
(d) 4.32 × 108 m/s (a) 2423 J (b) 2089 J (c) 167 J (d) 2256 J
NEET Solved Paper 2019 40 DAYS ~ NEET PHYSICS 17

43. Angular width of the central maxima in the 44. The work function of a photosensitive material is 4.0
Fraunhoffer diffraction for λ = 6000 Å is θ 0 . When the eV. This longest wavelength of light that can cause
same slit is illuminated by another monochromatic photon emission from the substance is (approximately)
light, the angular width decreases by 30%. The (a) 3100 nm (b) 966 nm (c) 31 nm (d) 310 nm
wavelength of this light is
(a) 1800 Å 45. A proton and an α-particle are accelerated from rest to
(b) 4200 Å the same energy. The de-Broglie wavelengths λ p and
(c) 6000 Å λ α are in the ratio
(d) 420 Å (a) 2 : 1 (b) 1 : 1 (c) 2 :1 (d) 4 : 1

Answers
1. (d) 2. (d) 3. (d) 4. (c) 5. (a) 6. (d) 7. (d) 8. (c) 9. (d) 10. (b)
11. (b) 12. (c) 13. (b) 14. (a) 15. (a) 16. (d) 17. (c) 18. (c) 19. (a) 20. (b)
21. (c) 22. (d) 23. (c) 24. (d) 25. (a) 26. (a) 27. (b) 28. (c) 29. (d) 30. (d)
31. (b) 32. (d) 33. (a) 34. (a) 35. (b) 36. (a) 37. (b) 38. (c) 39. (c) 40. (b)
41. (b) 42. (b) 43. (b) 44. (d) 45. (a)

Answer with Explanations


1. (d) The surface charge density of a closed surface area having QP′ 1 Qt 1 20 πσR2 5
σP = = = = σ
charge Q is given by Area 3 Area 3 4 πR2 3
σ=
Charge Q
= or Q = σA QQ′ 2 Qt 2 20 πσR2 2 5 5
and σ Q = = = × = × σ= σ
Area A Area 3 Area 3 4 π(2R)2 3 4 6
Thus, the charges on sphere P and Q having same charge density
as shown in the figure below is given by 2. (d) In a simple harmonic motion (SHM) the particle
σ oscillates about its mean position on a straight line.
σ The particle moves from its mean position (O) to an extreme
position (P) and then return to its mean position covering same
distance of A.
R 2R Then by the conservative force, it is moved in opposite
direction to a point Q by distance A and then back to mean
P position covering a distance of A. This comprises of one time
Q period as shown below
QP = σ × 4 πR2 = 4 πσR2 …(i)
O
and QQ = σ × 4 π(2R) = 16 πσR
2 2
…(ii) P
Q
when they are brought in contact with each other, the total
charge will be
Q t = QP + QQ A A
= 4 πσR2 + 16 πσR2 [From Eq. (i) and (ii)] extreme mean extreme
position position position
= 20 πσR2 …(iii)
In one time period
In connection of two charged conducting bodies, the potential
Hence, in one time period it covers a distance of
will become same on both, i.e.
QP QQ x = OP + PO + OQ + QO = A + A + A + A = 4 A
=
4 π ∈0 R 4 π ∈0 2R 3. (d) The distance covered by the mass falling from height ‘h’
QP QQ QP 1 during its time of fall ‘t’ is given by
⇒ = ⇒ = 1
R 2R QQ 2 s = h = ut + gt 2
2
So, the charges on the sphere P and Q after separation will be 1 2 2h
distributed as As, u = 0 ⇒h = gt ⇒t = …(i)
1 2 2 g
⇒ QP′ = Qt and QQ′ = Qt
3 3 The time period of simple pendulum is
l
After separation, the new surface charge densities on P and Q T = 2π …(ii)
will be g
18 40 DAYS ~ NEET PHYSICS NEET Solved Paper 2019

where, l is the length of the pendulum. From Eq. (i) and (ii), As the object breaks in two pieces, so the momentum of the
since ‘h’ and ‘l’ are constant so, we can conclude that, system will remains conserved i.e. the total momentum
1 1 t (before breaking) = total momentum (after breaking)
t∝ and T ∝ ∴ =1
m 5m v 5v 2
g g T mv = v1 + v2 ⇒ v = 1 +
Thus, the ratio of time of fall and time period of pendulum is 6 6 6 6
independent of value of gravity (g) or any other parameter like Here, v = 20$i + 25$j − 12k
$
mass and radius of the planet. Thus, the relation between t′ and v1 = 100$i + 35$j + 8k $
T′ on another planet irrespective of its mass or radius will
$ = (100$i + 35$j + 8k) + 5v 2
$
remains same as it was on earth i.e. ⇒ 20$i + 25$j − 12k
t ′ = 2T ′ 6 6
⇒ (120$i + 150$j − 72k$ ) = (100$i + 35$j + 8k
$ ) + 5v
2
4. (c) For vibrating tunning fork over a resonance tube, the first
1
resonance is obtained at the length ⇒ v 2 = (20$i + 115$j − 80k) = 4$i + 23$j − 16k
$ $
A 5

6. (d) Key Idea From Newton’s law of cooling, the time taken (t )
l1 λ/4 by a body to cool from T1 to T2 when placed in a medium of
N temperature T0 can be calculated from relation
T1 − T2 1  T1 + T2 
=  − T0 
t K 2 
λ
l1 = …(i) When the object cool from 80°C to 70°C in 12 minutes, then
4
from Newton’s law of cooling,
and for second resonance,
80 − 70 1  80 + 70
A =  − 25 [QT0 = 25° C]
12 K 2 
λ/4 5 1
= 50 …(i)
6 K
N
Similarly, when object cool from 70°C to 60°C we get
70 − 60 1  70 + 60
− 25 ⇒
l2 10 1
=  = 40 …(ii)
λ/2 t K 2  t K
Divide Eq. (i) and (ii), we get
5 t 50 5
× = ⇒ t = × 12 = 15 minutes
6 10 40 4
λ λ 3λ 7. (d) The terminal velocity achieved by ball in a viscous fluid is
l2 = + = …(ii)
4 2 4 2(ρ − σ)r 2 g
vt =
From Eq. (i) and (ii), we get 9η
3λ λ λ
l 2 − l1 = − = where, ρ = density of metal of ball,
4 4 2
σ = density of viscous medium,
⇒ λ = 2(l 2 − l1)
r = radius of ball and
 v
⇒ v = 2 f (l 2 − l1) …(iii) Q λ =  η = coefficient of viscosity of medium
 f
Terminal velocity of first ball,
Here, f = 800 Hz, l1 = 9.75 cm, l 2 = 31.25 cm 2(ρ − σ)r12 g 2(8ρ2 − σ)r12 g
Substituting the given values in Eq. (iii), we get vt1 = 1 = …(i) [Qρ 1 = 8ρ2]
9η 9η
⇒ v = 2 × 800(31.25 − 9.75)
Similarly, for second ball
= 34400 cm/s = 344 m/s
2(ρ2 − σ)r22 g
vt2 = …(ii)
5. (a) Let m be the mass of an object flying with velocity v in air. 9η
When it split into two pieces of masses in ratio 1 : 5, the mass From Eq. (i) and (ii), we get
5m
of smaller piece is m/6 and of bigger piece is . vt1 2(8ρ2 − σ)r12 g 9η
6 = ×
vt2 2(ρ2 − σ)r22 g 9η
This situation can be interpreted diagrammatically as below
2
v1  8ρ − 01. ρ2   r1 
= 2   …(iii) [Qσ = 0.1ρ2]
m/6  ρ2 − 01
. ρ2   r2 
Here, r1 = 1 mm and r2 = 2 mm
m v
Substituting these values in Eq. (iii), we get
5m/6 vt1  7.9ρ2   1  2 79
⇒ =   =
v2 vt2
 0.9ρ2   2 36
NEET Solved Paper 2019 40 DAYS ~ NEET PHYSICS 19

8. (c) From third equation of motion for circular motion 12. (c) Key Idea If area of cross-section of a surface is not uniform
ω2 − ω20 = 2αθ …(i) or if the steady state condition is not reached, the heat flow
where, ω = final angular velocity of particle equation can be applied to a thin layer of material
ω0 = initial angular velocity perpendicular to direction of heat flow.
α = angular acceleration and The rate of heat flow by conduction for growth of ice is given
θ = angular displacement by,
v
Here, ω = 0 rad/s (where, r radius of the circle) dθ KA(θ 0 − θ 1)]
r =
dt x
ω0 = 0 (initially particle is at rest)
θ = 2πn (for n rounds) where, dθ = ρAdxL, θ 0 = 0 and θ 1 = − θ
Substituting these values in Eq. (i), we get
2
Given, θ0 = 0° C, θ1 = − 26° C
2
 v0  − 0 = 2α(2πn) ⇒ α = v0 rad/s 2 The rate of increase of thickness can be calculated from Eq.
 
 r 4 πnr 2 dθ KA(θ0 − θ1)
=
dt x
9. (d) Let h be the height through which the coin is dropped.
ρAdxL KA(θ0 − θ1)
Then, according to the equation of motion, it is given as ⇒ =
1 2h dt x
h = ut + gt 2 ⇒ t = [Qu = 0] dx KA(θ0 − θ1) K [0 − (−26)] 26K
2 g ⇒ = = =
1 dt ρAxL ρxL ρxL
⇒ t∝
g
13. (b) The energy of light of wavelength λ is given by
As the elevator is moving uniformly i.e. its velocity is constant,
hc hc
so the acceleration is zero. E = hν = ⇒ λ= …(i)
∴Relative acceleration of the lift when it is either moving λ E
upward or downward is given as, g′ = g ± a = g ± 0 = g Here, h = Planck’s constant = 6.63 × 10−34 J-s
Hence, the time for the coin to reach the floor will remains same
i.e. t1 = t2. c = speed of light = 3 × 108 m/s

10. (b) As the truck move to the right, so the bob will move to the E = energy gap = 1.9 eV = 1.9 × 1.6 × 10−19 J
left due to inertia of rest with acceleration a. Substituting the given values in Eq. (i), we get
Thus, the given situation can be drawn as
6.63 × 10−34 × 3 × 108
⇒ λ=
ma θ 1.9 × 1.6 × 10−19
a ⇒
θ = 6.54 × 10−7 m ≈ 654 nm
mg
Thus, the wavelength of light emitted from LED will be 654
(a) (b) nm.
From the above diagram (b) as the string moves by an angle of θ 14. (a) From the circuit diagram given below, it can be seen that
with the vertical then the tangent angle is the current will flow to ground if any of the switch is closed.
 a
⇒ θ = tan−1   Also, the LED will only glow when current flows through it.
ma
tanθ =
mg  g
+6V
11. (b) According to Pascal’s law ‘‘Pressure applied to an enclosed R
fluid is transmitted undiminished to every point of the fluid A 0
and the walls of the containing vessel.’’ 1
In the given situation as shown in the figure below B 0
1
LED (Y)

R
20 cm
15 cm
Thus, the truth table for the circuit diagram can be formed as
circuit diagram in given solution.
Water Oil A B Y
Pressure due to water column of height 15 cm = Pressure due to 0 0 1
oil column of height 20 cm
0 1 0
⇒ hwρw g = h0ρ0 g
15 1 0 0
15ρw = 20ρ0 ⇒ ρ0 = ρω
20
1 1 0
15
ρ0 = × 1000 (Qgiven, ρw = 1000kg m −3)
20 The output (Y) is equivalent to that of NOR gate.
= 750 kgm −3
20 40 DAYS ~ NEET PHYSICS NEET Solved Paper 2019

15. (a) The poisson’s ratio, 19. (a) Given, distance between the two buildings
γ= P
C
…(i) d = 100 m
CV height of each tower, h = 200 m
where, Cp = molar heat capacity constant pressure speed of each bullet, v = 25 ms−1
and CV = molar heat capacity at constant volume
The situation can be shown as below
Also, Cp = CV + R (from Mayer’s relation)
f
CV = R (where, f =degree of freedom) x 25 m/s
2
Cp =  + 1 R
f
⇒ 200 m 200 m
2 
So, Eq. (i) becomes,
2
⇒ γ =1 +
f 100 m
For hydrogen gas, which is diatomic, the degree of freedom is 5 where, x be the vertical distance travelled from the top of the
(3 translational, 2 rotational). building and t be the time at which they collide.
2 7 As two bullets are fired toward each other,
∴ γ =1 + =
5 5 So, their relative velocity will be
For helium gas, which is monoatomic, the degree of freedom is vrel = 25 − (−25) = 50 ms−1
3 (3 translational only). d 100
Then, time t = = = 2s
2 5
∴ γ =1 + = vrel 50
3 3
The distance or height at which they collide is calculated from
The diatomic gas X also have vibrational motion, so degree of equation of motion,
freedom is 7 (3 translational, 2 rotational and 2 vibrational). 1
2 9 x = ut + at 2
∴ γ =1 + = 2
7 7
The bullet is initially a rest i.e. u = 0 and as it is moving under
16. (d) When the lens is cut along its principal axis, the focal the effect of gravity a = − g, so
length of the two halves will remain same because the radius 1 1
x = − gt 2 ⇒ x = − × 10(2)2 = − 20 m
of curvature of both the surfaces are still same. So, the power 2 2
also remains same as
The negative sign shows that the bullets will collide 20 m
1 below the top of tower i.e. at a height of (200 − 20) = 180 m
P=
f from the ground after 2 s.
f
20. (b) The stress-strain curve for a material is shown

Stress Elastic D Ultimate


B limit strength point
A C
f f E Fracture point
Plastic behaviour
17. (c) In a YDSE, the path difference for nth minima is given by Elastic behaviour
λ
∆y = (2n − 1)
2 O
Strain
For 5th minima, n = 5
λ 9λ This curve specifies the behaviour of material.
∴ ∆y = [2(5) − 1] =
2 2 For the material, if distance between strength point and
fracture point is small, so it is brittle and will break easily on
18. (c) Using the lens maker’s formula the application of some extra stress after point D.
1  µ 2 − µ1   1 1  For material Y, if the distance between strength point and
=  − 
f  µ 1   R1 R2  fracture point is large, so it is a ductile material and can
withstand for some extra stress beyond point D.
Here, f = 25cm
µ 2 = 1.5, µ 1 = 1 (for air) 21. (c) The situation can be drawn as
let R1 = R and R2 = − 2R F N
1.5 − 1   1  1  
= 
1 1 1 1
⇒  − −  ⇒ = +
25  1   R  2R   25 × 0.5 R 2R
f FH
3
⇒ R = × 25 × 0.5 = 18.75cm
2
∴ R1 = 18.75 cm
mg
R2 = 2 × 18.75 = 37.5cm
NEET Solved Paper 2019 40 DAYS ~ NEET PHYSICS 21

The frictional force, f = µN = µ mg [QN = mg] 24. (d) When a body rolls i.e. have rotational motion, the total
From Free body diagram (FBD), the resultant force is kinetic energy of the system will be
|F| = N + f = (mg) + (µmg) = mg 1 + µ
2 2 2 2 2 1  k2 
KE = mv2 1 + 2 
This is the minimum force required to move the object. But as
2  R 
the body is not moving where, m = mass of body, v = velocity and
k = radius of gyration
∴ |F|≤ mg 1 − µ 2

22. (d) The particle of mass 5m breaks in three fragments of mass


m, m and 3m respectively. Two fragments of mass m each,
x h
move in perpendicular direction with velocity v and the left
–1
fragment will move in a direction with velocity v′ such that 4m
s
the total momentum of the system must remain conserved. v= 30°
v
Given, m = 2kg, θ = 30°, v = 4 ms−1
5m m v Let h be the height of the inclined plane, then from law of
m
conservation of energy,
v=0
3m KE = PE
v
1 2  k2 
By law of conservation of momentum, mv 1 + 2  = mgh
v v 2  R 
5m × 0 = mv$i + mv$j + 3mv ′ ⇒ v ′ = − $i − $j
3 3 Substituting the given values in the above equation, we get
2 2
k2 1
 × 2 × 161 +  = 2 × 10 × h [QFor cylinder 2 = ]
v  v v 2 1 1
∴ |v′| =  −  +  −  =
 3  3 3 2  2  R 2
2
1 2 1 2 1  v 2 3
∴ Energy released, E = mv + mv + × 3m  ⇒ 8 × = 10 h ⇒ h = 1.2 m
2 2 2  3  2
mv2 4 2 From the above diagram
= mv2 + = mv h
3 3 sinθ =
x
23. (c) The area under the force displacement curve give the h 1.2 Qsin 30° = 1 
amount of work done. ⇒x = = = 1.2 × 2 = 2 . 4 m
From work-energy theorem,
sinθ sin 30°  2
W = ∆KE …(i) 25. (a) The magnetic field within the turns of toroid is
∴At x = 8 m, µ 0 NI
B=
W = Area ABDO + Area CEFD = 20 × 5 + 10 × 3= 130 J 2πr
where, N = number of turns, I = current in loops and
20
A B r = radius of each turn
Here, N1 = 200, N 2 = 100 , r1 = 40 cm, r2 = 20 cm
C E K L
10 and current I is same, then
O M B1 µ 0 N1 I 2πr2
0 D F J x(m) = ×
4 5 8 10 12 B2 2πr1 µ 0 N 2I
–10
Substituting the given values in the above relation, we get
G B1  N1   r2   200   20  1
–20 I =    =    = 2 × =1
–25
H
B2  N 2   r1   100   40  2
∴ B1 : B2 = 1 : 1
Using Eq. (i)
1 1 500 2 26. (a) The magnetic field at the centre of an arc subtended at
⇒ 130 = mv2 = × v
2 2 1000 an angle θ is given by
µ i θ
⇒ v = 2 130 = 22.8 ms−1 ≈ 23 ms−1 B= 0 ×
2R 2π
At x = 12m
W = Area ABDO + Area CEFD + Area FGHIJ + Area KLMJ i1
W = 20 × 5 + 10 × 3 + (−20 × 2) +  × − 5 × 2 + 10 × 2
1 270°
R
2  P
i A 90°
[Q Area FGHIJ = Area FGIJ + Area GHI]
R
= 100 + 30 − 40 − 5 + 20 = 105 J i2
Using Eq. (i) B
1 1
∴ 105 = × × v2 ⇒ v = 2 105 ~ − 20.6 ms−1 i
2 2
22 40 DAYS ~ NEET PHYSICS NEET Solved Paper 2019

Then, the magnetic field due to larger arc AB is mass of muon, mµ = 207me and
µ i 270 mass of nucleus, M = 1836 me
B1 = 0 1 × …(i)
2R 2π When electron in hydrogen atom is replaced by muon, the
which acts in inward direction according to right hand thumb reduced mass of muon is
rule. mµ M
mµ ′ = …(i)
And magnetic field due to smaller arc AB is mµ + M
µ i 90 Substituting the given values in Eq. (i), we get
B2 = 0 2 × …(ii)
2R 2π 207me × 1836me
mµ′ = ≈ 186 me …(ii)
which acts in outward direction. 207me + 1836me
The resultant magnetic field The radius of first orbit in hydrogen atom for electron is given
BR = B1 + B2 by,
µ i × 270 µ 0 i 2 × 90 h2ε0
=− 01 + [From Eq. (i) and (ii)] …(iii) r1 = …(iii)
4 πR 4 πR πme e 2
which acts in inward direction as B1 > B2. The radius of first orbit for muon is
Two arcs can also be seen as the two resistances in parallel h2ε0
combination. r1 ′ = [Qcharge of µ = charge of e − ]
πmµ′ e 2
So, the potential across them will be same i.e.
h2ε0
V1 = V2 = [from Eq. (ii)]
π × 186me e 2
i1 R1 = i 2R2 …(iv)
 h2ε0  1 r
where, R1 and R2 = Resistance of respective segments =  = 1 [from Eq. (iii)]
 πme e  186 186
2
The wire is uniform so
R1 L1 R × 270 0.51
= = [Qlength of arc = radius × angle] = Å [Q r1 = 0.51Å]
R2 L2 R × 90 186
= 2.74 × 10−13 m [Q1 Å = 10−10 m]
From Eq. (iv), we get The total energy of electron is given by
i1 R2 90 1 − mZ 2e 4  1 
⇒ = = = En =
i 2 R1 270 3   ⇒ En ∝ m
8ε20 h2  n2 
or 3i1 = i 2 …(v) For electron in first orbit of hydrogen atom,
From Eq. (iii) and (v), we get E1 = kme …(iv)
µ µ e4
BR = 0 (−270i1 + 90i 2) = 0 [−270i1 + 90(3i1)] where, k = 2 2 = constant.
4 πR 4 πR 8ε0 h
µ
= 0 (−270i1 + 270i1) = 0 For muon in first orbit
4 πR
E1 ′ = kmµ ′ = k × 186me [from Eq. (i)]
27. (b) The emf generated due to a rotating conductor in a = 186kme = 186E1 [from Eq. (iv)]
generator is given by = 186(−136. ) eV (given)
dφB = − 2529.6 eV = − 2.5 keV
E=−
dt ∴ The values are closest to that of options (c).
where, φB = magnetic flux linked with conductor 29. (d) The given circuit diagram can be drawn as shown below
From the above equation we can conclude that the emf I1 20Ω B 30Ω
generated is a varying function of time with opposite polarity. A E
So, all the graph are correct for the given variations.
V
28. (c) Key Idea Hydrogen atom can be considered to be the system
30Ω 20Ω
of two charges, positive charged nucleus and negative D F
charged electron. A system of this kind is equivalent to a single I2 C
particle of mass m′ that revolves around the position of the I
heavier particle. Then, the reduced mass of electron is
mM
m′ =
m+ M
where, m = mass of electron and 2V
M = mass of nucleus The equivalent resistance of circuit is given by
Its values is less than m. 1 1 1 1 1
= + = +
Req RAE RDF (20 + 30) (30 + 20)
Given, radius of first orbit for electron r1 = 0.51 Å, 1 1 2
ground state energy of electron, E1 = − 136 . eV, = + =
50 50 50
mass of electron = me ⇒ Req = 25 Ω
NEET Solved Paper 2019 40 DAYS ~ NEET PHYSICS 23

V 2 R
The current in circuit, I = = A ⇒ R = λN or N= …(i)
R 25 λ
As the resistance of two branches is same i.e. 50Ω. Also, the half life is given by,
So, the current I1 = I 2 ⇒ I = I1 + I 2 0.693 0.693
T1 / 2 = ⇒ λ= …(ii)
2 1 λ T1 / 2
= 2I1 ⇒ I 2 = I1 = A
25 25 From Eq. (i) and (ii), we get
1 1010 × 2.2 × 109
∴The voltage across AB, V1 = I1 R1 = × 20 N=
R
× T1 / 2 = = 317
. × 1019
25 0.693 0.693
1
and voltage across CD, V2 = I 2R2 = × 30
25 33. (a) From Kepler’s third law, the time period of revolution of
30 20 10 satellite around earth is
∴Voltmeter reading = V2 − V1 = − = = 0.4 V
25 25 25 T 2 ∝ r 3 or T ∝ r 3/ 2 …(i)
where, r is the radius of satellite’s orbit.
30. (d) For balanced position in a meterbridge
Here, r1 = 6RE + RE , T1 = 24 h
r2 = 2.5RE + RE , T2 = ?
P Q where RE = radius of earth
So, from Eq. (i), we get
G 3/ 2 3/ 2
T1  r1  24  6RE + RE 
3/ 2
=  
7
=  ⇒ = 
l1 l2 T2  r2  T2  2.5RE + RE   35
. 
P l1
= 24 24 12
⇒ T2 = 3/ 2 = = = 6 2h
Q l2 (2) 2 2 2
Now, if position of G and cell is interchanged,
34. (a) When the circuit is connected to AC source,
G
Voltage, V = 12V
Current, I = 0.2 A
P Q
V 12
⇒ Impedance Z = = = 60 Ω
I 0.2
When it is connected to DC source,
l1 l2 Voltage, V = 12V
The balance condition still remains the same if the jockey points Current, I = 0.4 A
as the same point as given in the initial condition, for which V 12
⇒ Resistance, R = = = 30 Ω
there is no deflection in the galvanometer or no current will be I 0.4
drawn from the cell. Thus, the bridge will work as usual and
As in case of DC supply, the capacitor act as an open circuit and
balance condition is same, P / Q = l1 / l 2
no current flows through the circuit. So the given circuit will
31. (b) Let BE be the net magnetic field at same point. H and V be not have capacitor in series combination. Therefore the circuit
should be a series LR circuit.
the horizontal and vertical components of BE . Let δ be the
angle of dip, which is the angle between direction of earth’s 35. (b) When a conducting disc or wheel of radius r rotates with
magnetic field BE and horizontal line in the magnetic constant angular velocity of ω about its axis in a uniform
meridian. magnetic field perpendicular to its plane and parallel to its
H axis of rotation, then,
δ

ω=10 rad s–1


V

P
r=0.5m
BE
Thus, from figure, we can see that
H = BE cosδ and V = BE sinδ
Axis
32. (d) Given, half life T1 / 2 = 2.2 × 109 s 1
Induced emf is given by, e = Bωr 2 …(i)
Rate of disintegration, R = 1010 s−1 2
If N be the number of nuclei present, then the rate of Here, B = 01
. T, ω = 10 rad/s, r = 0.5 m
disintegration is Substituting these values in Eq. (i), we get
dN 1 1
= λN (λ = decay constant) ∴ e = × 01 . × 10 × (0.5)2 = V = 0125. V
dt 2 8
24 40 DAYS ~ NEET PHYSICS NEET Solved Paper 2019

36. (a) The speed of any electromagnetic wave (EM) like light in The loss of energy due to redistribution of charge is given by
C1 C2
any medium of permeability, µ and permittivity, ε is given by ∆U = (V1 − V2)2
1 2(C1 + C2)
v=
µε C×C 1
= (V − 0)2 = CV 2 [QC1 = C2]
2(C + C) 4
Also, µ = µ 0µ r and ε = ε0 εr
where, µ 0 and ε0 are permeability and permittivity in free space 1
CV 2
and µ r and εr are relative permeability and relative permittivity ∆U
∴Percentage loss = × 100 = 4 × 100 = 50%
of medium respectively. U 1 2
CV
Here, µ r = 1.0 2
εr = 1.44 39. (c) As it is given that n divisions of vernier scale coincide
1 with (n − 1) divisions of main scale i.e.
∴ v=
µ r εr µ 0 ε0 n(VSD) = (n − 1) MSD
(n − 1)
c  1  ⇒ 1VSD = MSD …(i)
= Q c =  n
µ r εr  µ 0 ε0  The least count is the difference between one main scale
3 × 108 division (MSD) and one vernier scale division (VSD).
=
1 × 1.44 ∴Least Count (LC) = 1 MSD − 1VSD
(n − 1)
= 2.5 × 108 ms −1 = 1 MSD − MSD [From Eq. (i)]
n
(n − 1) 
= 1 −
37. (b) When a sphere encloses a charged dipole, 1
 MSD = MSD
 n  n
1
Here,1 MSD = cm
n
+q –q
1 1 1
⇒ LC = × cm = 2 cm
n n n

40. (b) For distance x, the person moves with constant velocity
v1 and for another x distance, he moves with constant
velocity of v2, then
Total distance travelled, D = x + x = 2x
Here, q = ± 3 × 10−6 C
Total time-taken, T = t1 + t2
Thus, according to Gauss’s law, the net electric flux across the
x x  Distance 
closed surface is equal to the net charge enclosed by it divided = + Q t = Velocity 
by ε0 , i.e. v1 v2  
q +3 × 10−6 − 3 × 10−6 total distance D
φE = in = =0 The average velocity, Vav = =
ε0 ε0 total time T
Hence, electric flux across the sphere is zero. 2x 2
v= = [Qvav = v]
x x 1 1
38. (c) When C1 is connected to voltage source, it is charged to a + +
v1 v2 v1 v2
potential V and this will be stored as a potential energy in the
capacitor given by 1 1 2
⇒ + =
1
U = CV 2 v1 v2 v
2
a k c 41. (b) The gravitational potential energy of an object placed at
earth’s surface is
b
GMm
V C2 U1 = − …(i)
C1 R
where, G = gravitational constant,
When key is disconnected from battery and b and c are M = Mass of earth, m = mass of object and
connected, the charge will be transformed from the capacitor C1 R = radius of the earth
to capacitor C2, The negative sign in the above relation indicates that it is the
then work done in bringing the object from infinity to a distance R.
a k c
The gravitational potential energy of object at a height h above
b the surface of earth is
GMm
V
C1
C2 U2 = − …(ii)
(R + h)
NEET Solved Paper 2019 40 DAYS ~ NEET PHYSICS 25

So, the change in potential energy is As slit width is constant, so using Eq. (i) for these values, we
GMm  GMm  get
∆U = U 2 − U1 = − − − 
R+ h  R  θ0 λ
= 1 ⇒ λ 2 = λ1 × 0.7 = 6000 × 0.7 = 4200 Å
[From Eq. (i) and (ii)] 0.7θ0 λ 2
 1 1  h  44. (d) The work function of material is given by
= − GMm  −  = − GMm − 
 (R + h) R   R(R + h)  φ = hν
…(i) Q ν = 
hc c
=
GMmh φ=
R(R + h) λ  λ 
where, h = Planck’s constant = 6.63 × 10−34 J-s
42. (b) Given, mass of water, m = 1 g
c = speed of length = 3 × 108 ms −1 and
Volume of 1 g of water = 1 cm 3 = 10−6 m 3
Volume of 1 g of steam = 1671 cm3 = 1671 × 10−6 m3 λ = wavelength of light
Pressure, p = 1 × 10 Pa
5 Here, φ = 4 eV = 4 × 1.6 × 10−19 J
Latent heat of vaporization of water, L = 2256 J/g Substituting the given values in Eq. (i), we get
Change in volume, ∆V = (1671 − 1) × 10−6 m3 6.63 × 10−34 × 3 × 108
⇒ 4 × 1.6 × 10−19 =
= 1670 × 10−6 m3 …(i) λ
6.63 × 10−34 × 3 × 108
Heat supplied, ∆Q = mL = 1 × 2256 = 2256 J …(ii) or λ=
4 × 1.6 × 10−19
As the steam expands, so the work done in expansion is
∆W = p∆V = 3108
. × 10−7 m ~
− 310 nm
= 1 × 105 × 1670 × 10−6 [from Eq. (i)]
45. (a) The de-Broglie wavelength associated with a charged
= 167 J …(iii)
particle is given by
According to first law of thermodynamics, h
∆Q = ∆U + ∆W λ=
p
⇒ ∆U = ∆Q − ∆W where, h = planck’s constant and
= 2256 − 167 [from Eq. (ii) and (iii)] p = momentum = 2mKE (here, KE is the kinetic energy of the
= 2089 J charged particle)
h
⇒ λ=
43. (b) The angular width of central maxima is given by 2mKE
2λ For proton and α-particle, the wavelengths are respectively
2θ = …(i)
a given as,
h h
where, λ = wavelength of light used λp = and λ α =
2mpKE p 2mαKE α
a = width of the slit
λp 2mαKE α
For λ1 = 6000Å, 2θ = θ0 (given) ∴ = …(i)
λα 2mPKE P
For another light of wavelength λ 2 (says), the angular width
Here, KE α = KE P and mα = 4mp
decreases by 30% so,
Substituting these above mentioned relations in Eq. (i), we get
100 − 30 
2θ =  θ λP 4mP 2
 100  0 ⇒ = =
λα mp 1
70
= θ0 = 0.7θ0
100 or λP : λα = 2 :1
NEET
SOLVED PAPER 2020
1. The phase difference between displacement and 6. The average thermal energy for a monoatomic gas is
acceleration of a particle in a simple harmonic motion is (where, kB is Boltzmann constant and T is absolute
3π π temperature.)
(a) rad (b) rad
2 2 3 5
(a) kBT (b) kBT
(c) zero (d) π rad 2 2
7 1
(c) kBT (d) kBT
2. A long solenoid of 50 cm length having 100 turns 2 2
carries a current of 2.5 A. The magnetic field at the
centre of solenoid is (Take, µ 0 = 4 π × 10 −7 T m A −1 ) 7. Find the torque about the origin when a force of 3 $j N
. × 10−4 T
(a) 314 acts on the particle whose position vector is 2 k$ m.
(b) 6.28 × 10−5 T (a) 6 $j N-m (b) − 6 $i N-m
−5
. × 10
(c) 314 T (c) 6 k$ N-m (d) 6 $i N-m
−4
(d) 6.28 × 10 T
8. The mean free path λ for a gas, with molecular
3. Two bodies of mass 4 kg and 6 kg are tied to the ends diameter d and number density n can be expressed as
of a massless string. The string passes over a pulley 1 1
(a) (b)
which is frictionless (see figure). The acceleration of 2 nπd 2 2 n2 πd 2
the system in terms of acceleration due to gravity g is 1 1
(c) (d)
2 n2 π 2d 2 2 nπd

9. The energy equivalent of 0.5 g of a substance is


(a) 4.5 × 1013 J . × 1013 J
(b) 15
(c) 0.5 × 1013
J (d) 4.5 × 1016 J
4 kg
10. A screw gauge has least count of 0.01 mm and there
6 kg are 50 divisions in its circular scale.
The pitch of the screw gauge is
(a) g/2 (b) g/5
(a) 0.25 mm (b) 0.5 mm
(c) g/10 (d) g
(c) 1.0 mm (d) 0.01 mm
4. The ratio of contributions made by the electric field and
11. Two cylinders A and B of equal capacity are connected
magnetic field components, to the intensity of an
to each other via a stop cock. A contains an ideal gas at
electromagnetic wave is
standard temperature and pressure. B is completely
(where, c = speed of electromagnetic waves) evacuated. The entire system is thermally insulated.
(a) 1 : 1 (b) 1: c The stop cock is suddenly opened. The process is
(c) 1 : c 2 (d) c : 1 (a) adiabatic (b) isochoric
(c) isobaric (d) isothermal
5. In a certain region of space with volume 0.2 m 3, the
electric potential is found to be 5 V throughout. The 12. A cylinder contains hydrogen gas at pressure of 249 kPa
magnitude of electric field in this region is and temperature 27°C.
(a) 0.5 N/C (b) 1 N/C Its density is (R = 8 .3 J mol −1 K −1 )
(c) 5 N/C (d) zero
(a) 0.2 kg/m 3 (b) 0.1 kg/m 3
(c) 0.02 kg/m 3 (d) 0.5 kg/m 3
2 NEET Solved Paper 2020

235
13. When a uranium isotope 92 U is bombarded with a 20. In Young’s double slit experiment, if the separation
89 between coherent sources is halved and the distance of
neutron, it generates 36 Kr, three neutrons and
91
the screen from the coherent sources is doubled, then
(a) 40 Zr (b) 101
36 Kr the fringe width becomes
103
(c) 36 Kr (d) 144
56 Ba (a) half (b) four times
−4 −1 (c) one-fourth (d) double
14. A charged particle having drift velocity of 7.5 × 10 ms
−10 −1
in an electric field of 3 × 10 Vm , has a mobility 21. For the logic circuit shown, the truth table is
2 −1 −1
(in m V s ) of A
(a) 2.5 × 106 (b) 2.5 × 10−6 Y
(c) 2.25 × 10−15 (d) 2.25 × 1015 B

15. Taking into account of the significant figures, what is


the value of 9.99 m − 0.0099 m? (a) A B Y
(a) 9.98 m (b) 9.980 m 0 0 0
(c) 9.9 m (d) 9.9801 m 0 1 1
16. An iron rod of susceptibility 599 is subjected to a 1 0 1
magnetising field of 1200 A m −1 . The permeability of 1 1 1
the material of the rod is
(Take, µ 0 = 4 π × 10 −7 T m A −1 ) (b) A B Y
0 0 1
(a) 8.0 × 10−5 T m A −1
(b) 2.4 π × 10−5 T m A −1 0 1 1

(c) 2.4 π × 10−7 T m A −1 1 0 1


−4 −1 1 1 0
(d) 2.4 π × 10 TmA

17. A spherical conductor of radius 10 cm has a charge of (c) A B Y


3.2 × 10 −7 C distributed uniformly. What is the 0 0 1
magnitude of electric field at a point 15 cm from the 0 1 0
centre of the sphere? 1 0 0
 1 
 = 9 × 10 9 Nm 2 /C 2 1 1 0
 4 πε 0 
(d) A B Y
(a) 1.28 × 105 N/C
0 0 0
(b) 1.28 × 10 N/C6

(c) 1.28 × 107 N/C 0 1 0

(d) 1.28 × 10 N/C4 1 0 0


1 1 1
18. A series L-C-R circuit is connected to an AC voltage
source. When L is removed from the circuit, the phase 22. The color code of a resistance is given below
π
difference between current and voltage is . If instead
3
C is removed from the circuit, the phase difference is
π Yellow Violet Brown Gold
again between current and voltage. The power factor
3 The values of resistance and tolerance respectively, are
of the circuit is (a) 47 k Ω, 10% (b) 4.7 k Ω, 5%
(a) 0.5 (b) 1.0 (c) 470 Ω, 5% (d) 470 k Ω, 5%
(c) −10
. (d) zero
23. The capacitance of a parallel plate capacitor with air as
19. A capillary tube of radius r is immersed in water and medium is 6 µF. With the introduction of a dielectric
water rises in it to a height h. The mass of the water in medium, the capacitance becomes 30 µF. The
the capillary tube is 5 g. Another capillary tube of permittivity of the medium is
radius 2r is immersed in water. The mass of water that
(ε 0 = 8.85 × 10 −12 C 2N −1m −2)
will rise in this tube is
(a) 5.0 g (b) 10.0 g . × 10−12 C 2N−1m−2
(a) 177 (b) 0.44 × 10−10 C 2N−1m−2
2 −1 −2
(c) 20.0 g (d) 2.5 g (c) 5.00 C N m (d) 0.44 × 10−13 C 2N−1m−2
NEET Solved Paper 2020 3

24. A ball is thrown vertically downward with a velocity of 32. A ray is incident at an angle of incidence i on one
20 m/s from the top of a tower. It hits the ground after surface of a small angle prism (with angle of prism A)
some time with a velocity of 80 m/s. The height of the and emerges normally from the opposite surface. lf the
tower is (g =10 m/s 2 ) refractive index of the material of the prism is µ, then
(a) 340 m (b) 320 m
the angle of incidence is nearly equal to
2A
(c) 300 m (d) 360 m (a) (b) µA
µ
25. A body weighs 72 N on the surface of the earth. What µA A
is the gravitational force on it, at a height equal to half (c) (d)
2 2µ
of radius of the earth?
(a) 32 N (b) 30 N 33. A 40 µF capacitor is connected to a 200 V, 50 Hz AC
(c) 24 N (d) 48 N supply. The rms value of the current in the circuit is,
26. Two particles of mass 5 kg and 10 kg respectively are nearly
attached to the two ends of a rigid rod of length 1 m (a) 2.05 A (b) 2.5 A
with negligible mass. (c) 25.1 A (d) 1.7 A

The centre of mass of the system from the 5 kg particle 34. Dimensions of stress are
is nearly at a distance of (a) [ML2 T − 2 ] (b) [ML0 T − 2 ]
(a) 50 cm (b) 67 cm
(c) 80 cm (d) 33 cm (c) [ML−1T − 2 ] (d) [MLT − 2 ]

27. The increase in the width of the depletion region in a 35. The Brewster’s angle ib for an interface should be
p-n junction diode is due to (a) 30° < i b < 45°
(a) reverse bias only (b) 45° < i b < 90°
(b) both forward bias and reverse bias (c) i b = 90°
(c) increase in forward current (d) 0° < i b < 30°
(d) forward bias only
36. A wire of length L, area of cross-section A is hanging
28. Light of frequency 1.5 times the threshold frequency is
from a fixed support. The length of the wire changes to
incident on a photosensitive material. What will be the
L1 when mass M is suspended from its free end. The
photoelectric current if the frequency is halved and
expression for Young’s modulus is
intensity is doubled?
Mg (L1 − L) MgL
(a) Four times (b) One-fourth (a) (b)
AL AL1
(c) Zero (d) Doubled
Mg L MgL1
(c) (d)
29. Assume that, light of wavelength 600 nm is coming A(L1 − L) AL
from a star. The limit of resolution of telescope whose
objective has a diameter of 2 m is 37. A short electric dipole has a dipole moment of
. × 10
(a) 183 −7
rad (b) 7.32 × 10 −7
rad 16 × 10 −9 C-m. The electric potential due to the dipole
(c) 6.00 × 10−7 rad (d) 3.66 × 10−7 rad at a point at a distance of 0.6 m from the centre of the
dipole, situated on a line making an angle of 60° with
30. A resistance wire connected in the left gap of a metre  1 
bridge balances a 10 Ω resistance in the right gap at a the dipole axis is  = 9 × 10 9 Nm 2 /C 2
 4 πε 0 
point which divides the bridge wire in the ratio 3 : 2. If
the length of the resistance wire is 1.5 m, then the (a) 200 V (b) 400 V
length of 1 Ω of the resistance wire is (c) zero (d) 50 V
. × 10−1 m
(a) 10 . × 10−1 m
(b) 15 38. In a guitar, two strings A and B made of same material
. × 10−2 m
(c) 15 . × 10−2 m
(d) 10 are slightly out of tune and produce beats of frequency
2 6 Hz. When tension in B is slightly decreased, the beat
31. Light with an average flux of 20 W/cm falls on a
frequency increases to 7 Hz. If the frequency of A is
non-reflecting surface at normal incidence having
530 Hz, the original frequency of B will be
surface area 20 cm 2 . The energy received by the surface
(a) 524 Hz
during time span of 1 min is
(b) 536 Hz
(a) 12 × 10 J
3
(b) 24 × 10 J
3
(c) 537 Hz
(c) 48 × 10 J
3
(d) 10 × 103 J (d) 523 Hz
4 NEET Solved Paper 2020

39. An electron is accelerated from rest through a potential 43. Which of the following graph represents the variation
difference of V volt. If the de-Broglie wavelength of the of resistivity (ρ) with temperature (T) for copper?
electron is 1.227 × 10 −2 nm, the potential difference is
ρ ρ
(a) 102 V
(a) (b)
(b) 103 V
(c) 104 V
(d) 10 V T T
ρ ρ
40. The solids which have the negative temperature
coefficient of resistance are (c) (d)
(a) insulator only
(b) semiconductors only T T
(c) insulators and semiconductors 44. For transistor action, which of the following statements
(d) metals is correct?
−20 (a) Base, emitter and collector regions should have same size.
41. The energy required to break one bond in DNA is 10
(b) Both emitter junction as well as the collector junction are
J. This value (in eV) is nearly
forward biased.
(a) 0.6 (b) 0.06 (c) The base region must be very thin and lightly doped.
(c) 0.006 (d) 6 (d) Base, emitter and collector regions should have same
dopping concentrations.
42. The quantities of heat required to raise the temperature
of two solid copper spheres of radii r1 and r2 (r1 = 1.5 r2) 45. For which one of the following, Bohr model is not
through 1 K are in the ratio valid?
9 3 (a) Singly ionised helium atom (He + )
(a) (b)
4 2 (b) Deuteron atom
5 27 (c) Singly ionised neon atom (Ne + )
(c) (d) (d) Hydrogen atom
3 8

Answers
1. (d) 2. (d) 3. (b) 4. (d) 5. (d) 6. (a) 7. (b) 8. (a) 9. (a) 10. (b)
11. (a) 12. (a) 13. (d) 14. (a) 15. (a) 16. (d) 17. (a) 18. (b) 19. (d) 20. (b)
21. (d) 22. (c) 23. (b) 24. (c) 25. (a) 26. (b) 27. (a) 28. (c) 29. (d) 30. (a)
31. (b) 32. (b) 33. (b) 34. (c) 35. (b) 36. (c) 37. (a) 38. (a) 39. (c) 40. (c)
41. (b) 42. (d) 43. (b) 44. (c) 45. (c)

Answer with Explanations


100
1. (d) In SHM, equation of displacement of a particle is = 4 π × 10−7 × × 2.5
y = a sinωt 0.5
and equation of acceleration of a particle is = 6.28 × 10−4 T
A = aω2 sinωt = aω2 sin(ωt + π) Hence, correct option is (d).
∴Phase difference between displacement and acceleration of 3. (b) Given, m1 = 4 kg, m2 = 6 kg and a = ?
a particle is
= (ωt + π) − ωt = π rad
a
Hence, correct option is (d).

2. (d) Given, l = 50 cm = 0.5m, N = 100 turns 4 kg


and I = 2.5A
m 1g
∴Magnetic field at the centre of solenoid is 6 kg a

= µ 0 nI = µ 0   ⋅ I
N m 2g
 l
NEET Solved Paper 2020 5

From the above free body diagram, the relation for Pitch of the screw gauge = least count ´ number of division
acceleration of the given system can be given as on circular scale
æ m - m1 ö æ6 - 4ö g = 0.01 ´ 50 = 0.5 mm
a=ç 2 ÷× g= ç ÷ ´ g= Hence, correct option is (b).
è m1 + m2 ø è4 + 6ø 5
Hence, correct option is (b). 11. (a) Since, the entire system is thermally insulated. Thus, their
will be no transfer of heat between the system and the
4. (d) We know that, surrounding. Hence, when the stop cock is suddenly opened,
E0 a sudden process of expansion will takes place with no heat
=c transfer. Thus, the given process will be adiabatic, as it is a
B0
process that occurs without transferring heat or mass
where, E 0 and B0 are the peak values of electric field and between a thermodynamic system and its surroundings.
magnetic field, respectively.
Hence, correct option is (a).
\ E0 : B 0 = c : 1
Hence, correct option is (d). 12. (a) Given, pressure p = 249 kPa = 249 ´ 103 Pa
Temperature, T = 27° C = 273 + 27 K = 300 K
5. (d) Given, volume, V = 0.2 m3
Density, r = ?
Electric potential = 5V = constant As, from ideal gas equation,
Electric field = ? pV = nRT
We know that, for constant electric potential the value of m é as n = m ù
Þ pV = RT êë
electric field is zero. M M úû
- dV - d(5) Þ pVM = mRT
i.e., E= = =0
dr dr m é as m = r ù
Þ pM = RT = rRT êë V úû
Hence, correct option is (d). V
pM 249 ´ 103 ´ 2 ´ 10-3
6. (a) The average thermal energy of a system with degree of Þ r= =
freedom f is equals to its average energy, which is given as RT 8.3 ´ 300
f [Qfor hydrogen gas, M = 2 g = 2 ´ 10-3 kg]
= kB × T
2 Þ r = 0.2kg/m 3
For monoatomic gas, f = 3 Hence, correct option is (a).
3
\ Average thermal energy = kB × T 13. (d) Here,
2 235
92 U + 10 n ¾® 89 1
36 Kr + Z X + 3 ( 0 n)
A
Hence, correct option is (a).
According to the law of conservation,
7. (b) Given, position vector, r = 2k$ m (Atomic number) Reactant = (Atomic number) Product
Force, F = 3$j N 92 + 0 = 36 + Z Þ Z = 92 - 36 = 56
As, torque, t = r ´ F = 2k$ ´ 3$j Similarly,
= 6(- $i) = - 6$i N-m (Atomic masses) Reactant = (Atomic masses) Product
235 + 1 = 89 + A + 3 ´ 1 Þ A = 144
Hence, correct option is (b).
\ Other element is 144
56 Ba.
8. (a) The mean free path l for a gas, with molecular diameter d
and number density n is given by the relation Hence, correct option is (d).

l=
1 14. (a) Given, drift velocity, vd = 7. 5 ´ 10-4 m/s
2 npd 2 Electric field, E = 3 ´ 10-10 V-m
Hence, correct option is (a). Mobility, m = ?
-3 vd 7. 5 ´ 10-4
9. (a) Given, m = 0. 5 g = 0. 5 ´ 10 kg As, m= =
E 3 ´ 10-10
Relation for energy equivalent of mass is
= 2.5 ´ 106 m2 V -1s -1
E = mc 2
Hence, correct option is (a).
where, c is speed of light.
= 0. 5 ´ 10-3 ´ (3 ´ 108) 2 15. (a) The difference between 9.99 m and 0.0099 m is
13 = 9.99 - 0.0099 = 9.9801 m
= 4.5 ´ 10 J
Taking significant figures into account, as both the values has
Hence, correct option is (a). two significant figures after decimal.
10. (b) Given, least count = 0.01 mm So, their difference will also have two significant figures after
decimal, i.e. 9.98 m.
Number of divisions on circular scale = 50
Hence, correct option is (a).
6 NEET Solved Paper 2020

16. (d) Given, susceptibility, c = 599 20. (b) Fringe width, b = lD


d
Magnetic field, B = 1200 Am -1 l 2D2
Permeability of the material of the rod = ? b2 d2
\ =
As, m m = m 0 (1 + c) b1 l1 D1
= 4p ´ 10-7(1 + 599) d1
= 2.4p ´ 10-4 TmA -1 d
[As l1 = l 2, D2 = 2D1 and d2 = 1 ]
2
Hence, correct option is (d).
b2 2D1 d
\ = ´ 1 =4
17. (a) Given, radius, r = 10 cm = 10 ´ 10-2 m b1 æ d1 ö D1
ç ÷
Charge, q = 3.2 ´ 10-7 C è 2ø

Electric field, E = ? Þ b 2 = 4b1


Electric field at a point (x = 15cm) from the centre of the i.e., fringe width becomes four times.
sphere is Hence, correct option is (b).
1 q . ´ 10-7
32
E= × 2 = 9 ´ 109 ´ 21. (d) Given logic circuit is
4pe0 x (15 ´ 10-2)2
A
= 1.28 ´ 105 N/C A
Hence, correct option is (a). Y
B
18. (b) For L-C-R circuit, phase difference, B
X - XC ö
f = tan-1 æç L ÷ Y = A+ B
è R ø
= A×B
When L is removed, then X L = 0
-X [using de-Morgan’s theorem, x + y = x × y]
\ f = tan-1 æç C ö÷
è R ø = A× B [Q x = x]
½X C½
½ Truth table for above given logic circuit is
Þ tan f = ½ …(i)
½ R½ A B A B Y = A× B
When C is removed, then X C = 0 0 0 1 1 0
X
\ f = tan-1 æç L ö÷ 0 1 1 0 0
è R ø
1 0 0 1 0
X
Þ tan f = L …(ii) 1 1 0 0 1
R
From Eqs. (i) and (ii), we get Þ
XL = XC A B Y
\ Impedance, Z = 2 2
R + (X L - X C) = R 0 0 0
R R 0 1 0
Power factor, cos f = = =1
Z R 1 0 0
Hence, correct option is (b). 1 1 1

19. (d) Relation for height of water in capillary tube is Hence, correct option is (d).
2S cos q 22. (c) According to the carbon color code for resistors,
h=
rgr Code of yellow = 4
1 Code of violet = 7
Þ hµ
r Code of brown, i.e. multiplier = 101
h1 r2 2r
Þ = = =2 Code of gold, i.e. tolerance = ± 5%
h2 r1 r
Hence, resistance of resistor = 47 ´ 101 W , 5%
As m = A × h×r
m2 Ah2r = 470 W , 5%
\ =
m1 Ah1r Hence, correct option is (c).
h2 1 23. (b) Given,C0 = 6mF
= =
h1 2 Cm = 30mF
m 5 \ As, dielectric constant
Þ m2 = 1 = = 2.5g
2 2 C 30
K = er = m = =5
Hence, correct option is (d). C0 6
NEET Solved Paper 2020 7

Permittivity of the medium, 28. (c) Initially, frequency of light, n = 1.5n0


em = K ´ e0 = 5 ´ e0
But when the frequency of incident light is halved, then
= 5 ´ 8.85 ´ 10-12 n
n¢ = = 0.75n0 . Thus, it becomes less than the threshold
= 0.44 ´ 10-10 C2 N -1 m-2 2
frequency. In that case, no photoelectric effect takes place.
Hence, correct option is (b).
Hence, photoelectric current becomes zero.
24. (c) Given, u = 20 m/s, v = 80 m/s and h = ? Hence, correct option is (c).
From kinematic equation of motion,
v2 = u2 + 2gh
29. (d) Given, light of wavelength, l = 600 nm = 600 ´ 10-9 m
Diameter, d = 2m
v2 - u2 (80)2 - (20)2
Þ h= = (Q given, g = 10 m/s 2) As, limit of resolution of telescope, dq =
1. 22l
2g 2 ´ 10 d
= 300m 1.22 ´ 600 ´ 10-9
=
Hence, correct option is (c). 2
25. (a) Given, w = mg = 72N (on the surface of earth). . ´ 10-7 rad
= 366
R Hence, correct option is (d).
At height equal to half of radius of the earth, æç i.e. h = ö÷
è 2ø 30. (a) According to the question, the metre bridge is shown
Acceleration due to gravity, below,
2
æ R ö R 10 Ω
g¢ = gç ÷
è R + hø
2
æ R ö æ 4R2 ö
= gç ÷ = gç 2 ÷ G
è R + R/ 2ø è 9R ø
x1 x2
4 100 cm
Þ g¢ = g
9
4 4 x1 3
Þ mg¢ = mg = ´ 72= 32N Given, =
9 9 x2 2
w¢ = 32N At balance condition in metre bridge,
Hence, correct option is (a). R x
= 1
10 x 2
26. (b) Given, m1 = 5 kg, m2 = 10 kg and r = 1 m = 100 cm
x 3
Let the centre of mass lies at origin O. Þ R = 1 ´ 10 = ´ 10 = 15W
x2 2
m1 O m2 Now, length of given wire whose resistance 15W is 1.5 m.
r1 r2 Therefore, length of 1W resistance wire is
A B
r 1. 5 1
= = . = 1 ´ 10-1 m
= 01
15 10
m1 r1 - m2r2
\ =0 Hence, correct option is (a).
m1 + m2
31. (b) Given, average flux = 20 W/cm 2
Þ 5r1 - 10r2 = 0
r Surface area = 20 cm 2
Þ r2 = 1
2 Time = 1min = 60 s
Also, r1 + r2 = 100 For non-reflecting surface,
r energy received = average flux ´ surface area ´ time
Þ r1 + 1 = 100
2 = 20 ´ 20 ´ 60 = 24 ´ 103 J
Þ 3r1 = 200
32. (b) According to the question, the ray diagram can be shown
200 ~
Þ r1 = - 67 cm as
3 P
Hence, correct option is (b).
A
27. (a) Under reverse bias condition, the holes of p-side are
N α N′
attracted towards the negaive terminal of the battery and i
electrons of the n-side are attracted towards the positive r
terminal of the battery. This increases the width of the
depletion layer. Q R
However, in the case of forward biasing, the width of the
From figure in DPNN¢,
depletion layer decreases.
A + a + 90° = 180°
Hence, correct option is (a).
Þ a = 90° - A …(i)
8 NEET Solved Paper 2020

Also, r + a = 90° When tension in B is slightly decreased, then beats frequency


Þ r = 90° - a increases to 7 Hz. This is possible if we take original
= 90° - (90° - A) = A [Q from Eq. (i)] frequency of B as 524 Hz.
Þ r= A Hence, correct option is (a).
From Snell’s law, 39. (c) Given, l = 1.227 ´ 10-2 nm = 1.227 ´ 10-11 m
m1 sin i = m 2 sin r
Potential difference, V = ?
Þ sin i = m sin A (here, m1 = 1 and m 2 = m)
Relation for de-Broglie wavelength for moving electron is
For small angle, sin q ~
- q Þ i =mA
12.27
Hence, correct option is (b). l= Å
V
33. (b) Given, C = 40 mF = 40 ´ 10-6 F
12.27 12.27 ´ 10-10
Þ V = ´ 10-10 = = 102
Vrms = 200V l 1.227 ´ 10-11
n = 50Hz
1 1 Þ V = 104 V
As, XC = =
2pnC 2p ´ 50 ´ 40 ´ 10-6 Hence, correct option is (c).
106 250 40. (c) Insulators and semiconductors are those solids, which
= = W
100p ´ 40 p have negative temperature coefficient of resistance. As, when
Vrms 200
temperature increases number of free electrons increases in
\ I rms = = = 2.5A both insulator and semiconductor.
XC æ 250 ö
ç ÷ Hence, correct option is (c).
è p ø
Hence, correct option is (b). 41. (b) Given, E = 10-20 J
34. (c) Q Stress = Force In terms of eV, we get
Area 10-20
[MLT - 2] E= eV = 0.06 eV
\ Dimensions of stress = = [ML-1 T - 2] 1.6 ´ 10-19
[L2]
Hence, correct option is (b).
Hence, correct option is (c).
42. (d) Since, heat required, Q = mcDT
35. (b) The Brewster’s angle i b for an interface should lies
4
between 45° to 90°, i.e. 45° < i b < 90°. = æç pr 3 × r ö÷ cDT [Q m = Vsphere r]
è3 ø
Hence, correct option is (b).
Since, p , r , c and T are constant.
36. (c) Here, change in length, DL = (L1 - L)
Þ Q µ r3
Area = A 3 3
Force, F = Mg Q1 r13 æ r1 ö æ1.5r2 ö 27
or = =ç ÷ =ç ÷ =
Normal stress Q2 r23 è r2 ø è r2 ø 8
Young’s modulus, Y =
Longitudinal strain Hence, correct option is (d).
Mg
(F / A) MgL 43. (b) Resistivity of copper (a metal) as a function of
Þ Y = = A = temperature increases with the increase in temperature as
æ DL ö æ L1 - L ö A(L1 - L)
ç ÷ ç ÷ shown below,
è L ø è L ø
ρ
Hence, correct option is (c).

37. (a) Given, electric dipole moment, p = 16 ´ 10-9 C-m


Distance, r = 0.6 m
1 T(K)
Angle, q = 60° Þ cos 60° = O
2
For copper at 0K, value of resistivity is1.7 ´ 10-8 W-m.
Electric potential at a point which is at a distance r at some
angle q from electric dipole is Hence, correct option is (b).
1
9 ´ 109 ´ 16 ´ 10-9 ´ 44. (c) In case of transistor action, base region is thin and lightly
p cos q 2
V= = dopped. While, emitter region is heavily dopped and collector
4pe0 r 2 (0.6)2 region is moderately dopped. Also, the size of collector is
= 2 ´ 102 = 200 V slightly more than the size of emitter.
Hence, option (c) is correct.
Hence, correct option is (a).
38. (a) In case of beats formation unknown frequency 45. (c) Since, Bohr’s model is valid for hydrogen and hydrogen
(nB) = n A ± beats like atoms He+ , deuteron, etc. So, it is not valid for singly
where, n A = 530 Hz and beats = 6 Hz. ionised neon atom (Ne+ ).
Þ nB = 530 ± 6 = 536 or 524 Hz Hence, correct option is (c).
NEET SOLVED PAPER 2021
Instruction
l This question paper contains of 50 Multiple Choice Questions of Physics, divided into two Sections;
section A and section B.
l Section A contains 35 questions and all questions are compulsory.
l Section B contains 15 questions out of which only 10 questions are to be attempted.
l Each question carries 4 marks.

Section-A 4. A capacitor of capacitance C, is connected across an AC


source of voltage V, given by V = V0 sin ωt
1. An inductor of inductance L, a capacitor of capacitance The displacement current between the plates of the
C and a resistor of resistance R are connected in series
capacitor, would then be given by
to an AC source of potential difference V volts as shown V0
in figure. (a) Id = V0 ωC cos ωt (b) Id = cos ωt
ωC
V0
(c) Id = sinωt (d) Id = V0ωC sinωt
ωC

40 V 10 V 40 V 5. A thick current carrying cable of radius R carries


current I uniformly distributed across its cross-section.
The variation of magnetic field B due to the cable with
V the distance r from the axis of the cable is represented
Potential difference across L, C and R is 40 V, 10 V and by
40 V, respectively. The amplitude of current flowing
through L-C-R series circuit is 10 2 A. The impedance
of the circuit is
(a) B (b) B
(a) 4 2 Ω (b) 5 2 Ω
(c) 4 Ω (d) 5 Ω
r r
2. Find the value of the angle of emergence from the
prism. Refractive index of the glass is 3.

(c) B (d) B

r r
60º
6. A convex lens A of focal length 20 cm and a concave
(a) 60° (b) 30° (c) 45° (d) 90° lens B of focal length 5 cm are kept along the same axis
with a distance d between them. If a parallel beam of
3. A dipole is placed in an electric field as shown. In light falling on A leaves B as a parallel beam, then the
which direction will it move? distance d (in cm) will be
(a) 25 (b) 15
(c) 50 (d) 30

E 7. An electromagnetic wave of wavelength λ is incident


+q –q
on a photosensitive surface of negligible work function.
If m mass is of photoelectron emitted from the surface
has de-Broglie wavelength λ d , then
(a) λ = 
2 m 2
(b) λd = 
2 mc  2
(a) Towards the left as its potential energy will increase  λ λ
 hc  d  h 
(b) Towards the right as its potential energy will decrease
(c) λ = 
2 mc  2
(d) λ = 
2h  2
(c) Towards the left as its potential energy will decrease  λ  λ
(d) Towards the right as its potential energy will increase  h  d  mc  d
2 NEET Solved Paper 2021

8. Column I Gives certain physical terms associated with 13. A cup of coffee cools from 90°C to 80°C in t minutes,
flow of current through a metallic conductor. when the room temperature is 20°C. The time taken by
Column II Gives some mathematical relations a similar cup of coffee to cool from 80°C to 60°C at a
involving electrical quantities. Match Column I and room temperature same at 20°C, is
Column II with appropriate relations. 13 13
(a) t (b) t
10 5
Column I Column II 10 5
(c) t (d) t
A. Drift velocity 1. m 13 13
ne 2ρ
14. The number of photons per second on an average
B. Electrical resistivity 2. nevd emitted by the source of monochromatic light of
wavelength 600 nm, when it delivers the power of
C. Relaxation period 3. eE τ
m 3.3 × 10 −3 W will be (h = 6.6 × 10 − 34 J-s)
E
(a) 1018 (b) 1017
D. Current density 4.
(c) 1016 (d) 1015
J
15. A body is executing simple harmonic motion with
Codes frequency n, the frequency of its potential energy is
A B C D
(a) n (b) 2n
(a) 3 4 1 2
(c) 3n (d) 4n
(b) 3 4 2 1
(c) 3 1 4 2 16. An infinitely long straight conductor carries a current
(d) 3 2 4 1 of 5 A as shown. An electron is moving with a speed of
A 10 5 m/s parallel to the conductor. The perpendicular
9. A radioactive nucleus Z X undergoes spontaneous
distance between the electron and the conductor is
decay in the sequence 20 cm at an instant. Calculate the magnitude of the
Z X → Z − 1 B → Z − 3C →
A
Z − 2 D, where Z is the force experienced by the electron at that instant.
atomic number of element X . The possible decay Speed of Electron v=105 m/s
particles in the sequence are
(a) α, β − , β + (b) α, β + , β −
(c) β + , α, β − (d) β − , α, β + 20 cm

10. The effective resistance of a parallel connection that


consists of four wires of equal length, equal area of P 5A Q
cross-section and same material is 0.25 Ω. What will be − 20
(a) 4 × 10 N (b) 8 π × 10− 20 N
the effective resistance if they are connected in series?
(c) 4 π × 10− 20 N (d) 8 × 10− 20 N
(a) 0.25 Ω (b) 0.5 Ω
(c) 1 Ω (d) 4 Ω 17. If force [ F] , acceleration [ a] and time [T ] are chosen as
11. A particle is released from height S from the surface of the fundamental physical quantities. Find the
the Earth. At a certain height its kinetic energy is three dimensions of energy.
times its potential energy. The height from the surface (a) [F ] [a] [T ] (b) [F ] [a] [T 2 ]
−1
of earth and the speed of the particle at that instant are (c) [F ] [a] [T ] (d) [F ] [a− 1 ] [T ]
respectively
18. Match Column l with Column ll and choose the correct
S 3gS S 3gS
(a) , (b) , match from the given choices.
4 2 4 2
S 3gS S 3gS Column I Column II
(c) , (d) ,
2 2 4 2 A. Root mean square of gas 1. 1 nmv 2
molecules 3
12. The half-life of a radioactive nuclide is 100 h. The B. Pressure exerted by ideal gas 2. 3RT
fraction of original activity that will remain after 150 h M
would be
1 1 C. Average kinetic energy of a 3. 5 RT
(a) (b) molecule 2
2 2 2
2 2 D. Total internal energy of 1 mole of 4. 3 k T
(c) (d) a diatomic gas 2
B
3 3 2
NEET Solved Paper 2021 3

Codes 24. Two charged spherical conductors of radii R1 and R2


A B C D
are connected by a wire. Then, the ratio of surface
(a) 3 1 4 2
(b) 2 3 4 1
charge densities of the spheres (σ1 / σ 2) is
(c) 2 1 4 3 R1 R2
(a) (b)
(d) 3 2 1 4 R2 R1

19. A small block slides down on a smooth inclined plane, R  R12


(c)  1  (d)
starting from rest at time t = 0. Let s n be the distance  R2  R 22
travelled by the block in the interval t = n − 1 to t = n.
s 25. A spring is stretched by 5 cm by a force 10 N. The time
Then, the ratio n is
sn + 1 period of the oscillations when a mass of 2 kg is
suspended by it is
2n − 1 2n − 1
(a) (b) (a) 0.0628 s (b) 6.28 s
2n 2n + 1
(c) 3.14 s (d) 0.628 s
2n + 1 2n
(c) (d)
2n − 1 2n − 1 26. For a plane electromagnetic wave propagating in
x-direction, which one of the following combination
20. A nucleus with mass number 240 breaks into two gives the correct possible directions for electric field (E)
fragments each of mass number 120, the binding and magnetic field (B) respectively?
energy per nucleon of unfragmented nuclei is 7.6 MeV
(a) $j + k$ , $j + k$ (b) − $j + k,
$ − $j − k$
while that of fragments is 8.5 MeV. The total gain in
the binding energy in the process is (c) $j + k$ , − $j − k$ (d) − $j + k,
$ − $j + k$
(a) 0.9 MeV (b) 9.4 MeV
(c) 804 MeV (d) 216 MeV 27. The escape velocity from the Earth’s surface is v. The
escape velocity from the surface of another planet
21. A screw gauge gives the following readings when used having a radius, four times that of Earth and same
to measure the diameter of a wire mass density is
Main scale reading : 0 mm (a) v (b) 2 v
Circular scale reading : 52 divisions (c) 3 v (d) 4 v
Given that, 1 mm on main scale corresponds to 100 28. In a potentiometer circuit, a cell of emf 1.5 V gives
divisions on the circular scale. The diameter of the wire balance point at 36 cm length of wire. If another cell of
from the above data is emf 2.5 V replaces the first cell, then at what length of
(a) 0.52 cm (b) 0.026 cm the wire, the balance point occurs?
(c) 0.26 cm (d) 0.052 cm (a) 60 cm (b) 21.6 cm
22. The equivalent capacitance of the combination shown (c) 64 cm (d) 62 cm
in the figure is
29. The velocity of a small ball of mass M and density d
C when dropped in a container filled with glycerine
becomes constant after some time. If the density of
glycerine is d / 2 , then the viscous force acting on the
C ball will be
Mg
(a) (b) Mg
2
C 3
(c) Mg (d) 2 Mg
(a) 3C (b) 2C 2
C 3C
(c) (d)
2 2 30. A parallel plate capacitor has a uniform electric field E
in the space between the plates. If the distance
23. A lens of large focal length and large aperture is best between the plates is d and the area of each plate is A ,
suited as an objective of an astronomical telescope, the energy stored in the capacitor is (ε 0 = permittivity
since of free space).
(a) a large aperture contributes to the quality and visibility of 1
(a) ε0 E 2 (b) ε0 EAd
the images 2
(b) a large area of the objective ensures better light gathering
1 E 2 Ad
power (c) ε0 E 2 Ad (d)
(c) a large aperture provides a better resolution 2 ε0
(d) All of the above
4 NEET Solved Paper 2021

31. The electron concentration in an n-type semiconductor 37. For the given circuit, the input digital signals are
is the same as hole concentration in a p-type applied at the terminals A, B and C. What would be the
semiconductor. An external field (electric) is applied output at the terminal?
across each of them. Compare the currents in them. t1 t2 t3 t4 t5 t6
(a) Current in n-type = Current in p-type 5
(b) Current in p-type > Current in n-type A
0
(c) Current in n-type> Current in p-type
(d) No current will flow in p-type. current will only flow in n-type
5
B
32. Consider the following statements (A) and (B) and 0
identify the correct answer.
(A) A Zener diode is connected in reverse bias, when
used as a voltage regulator. 5
C
(B) The potential barrier of p-n junction lies between 0.1 0
V to 0.3 V.
A
(a) (A) and (B) both are correct.
(b) (A) and (B) both are incorrect.
(c) (A) is correct and (B) is incorrect.
(d) (A) is incorrect but (B) is correct. B Y

33. Polar molecules are the molecules


(a) having zero dipole moment
(b) acquire a dipole moment only in the presence of electric C
field due to displacement of charges. t1 t2 t3 t4 t5 t6
(c) acquire a dipole moment only when magnetic field is Y
absent (a)
0V
(d) having a permanent electric dipole moment
5V
34. If E and G respectively denote energy and gravitational (b)
E 0V
constant. then has the dimensions of
G
(a) [M 2 ] [L − 1] [T 0 ] (c) 5V
(b) [M] [L − 1] [T − 1]
(c) [M] [L 0 ] [T 0 ]
(d) [M 2 ] [L − 2 ] [T − 1] 5V
(d)
35. Water falls from a height of 60 m at the rate of 15 kg/s 0V
to operate a turbine. The losses due to frictional force
38. A ball of mass 0.15 kg is dropped from a height 10 m,
are 10% of the input energy. How much power is
strikes the ground and rebounds to the same height.
generated by the turbine? (g = 10 m/s 2 ) The magnitude of impulse imparted to the ball is nearly
(a) 10.2 kW (b) 8.1 kW (g = 10 m/s 2 )
(c) 12.3 kW (d) 7.0 kW
(a) 0 (b) 4.2 kg-m/s
Section-B (c) 2.1 kg-m/s (d) 1.4 kg-m/s

36. A car starts from rest and accelerates at 39. A uniform rod of length 200 cm and mass
5 m/s 2 . At t = 4s, a ball is dropped out of a window by a 500 g is balanced on a wedge placed at 40 cm mark. A
person sitting in the car. What is the velocity and mass of 2 kg is suspended from the rod at 20 cm and
acceleration of the ball at t = 6 s? (Take, g = 10 m/s 2 ) another unknown mass m is suspended from the rod at
160 cm mark as shown in the figure. Find the value of m
(a) 20 m/s, 5 m/s 2 (b) 20 m/s, 0
such that the rod is in equilibrium. (g = 10 m/s 2 )
(c) 20 2 m/s, 0 (d) 20 2 m/s,10 m/s 2
NEET Solved Paper 2021 5

0 20 cm 40 cm 160 cm 43. In the product


F = q(v × B) = qv × (B$i + B$j + B0 k$ )
For q = 1 and v = 2 $i + 4 $j + 6 k$ and F = 4 $i − 20 $j + 12 k$
What will be the complete expression for B?
2 kg m
(a) − 8$i − 8$j − 6 k$
1 1 (b) − 6$i − 6$j − 8 k$
(a) kg (b) kg
2 3 (c) 8$i + 8$j − 6 k$
1 1
(c) kg (d) kg (d) 6$i + 6$j − 8 k$
6 12

40. A point object is placed at a distance of 44. A particle of mass m is projected with a velocity
60 cm from a convex lens of focal length v = kve (k < 1) from the surface of the Earth.
30 cm. If a plane mirror were put perpendicular to the (Here, ve = escape velocity)
principal axis of the lens and at a distance of 40 cm The maximum height above the surface reached by the
from it, the final image would be formed at a distance particle is
2 2
of  k   k 
(a) R   (b) R  
1 − k 1 + k
R2k Rk 2
(c) (d)
1+ k 1 − k2

45. Twenty seven drops of same size are charged at 220 V


each. They combine to form a bigger drop. Calculate the
60 cm 40 cm
potential of the bigger drop.
(a) 20 cm from the lens, it would be a real image (a) 660 V (b) 1320 V
(b) 30 cm from the lens, it would be a real image (c) 1520 V (d) 1980 V
(c) 30 cm from the plane mirror, it would be a virtual image
(d) 20 cm from the plane mirror, it would be a virtual image 46. A series L-C-R circuit containing 5.0 H inductor, 80 µF
capacitor and 40 Ω resistor is connected to 230 V
41. A step down transformer connected to an AC mains variable frequency AC source. The angular frequencies
supply of 220 V is made to operate at 11 V, 44 W lamp. of the source at which power transferred to the circuit
Ignoring power losses in the transformer, what is the is half the power at the resonant angular frequency are
current in the primary circuit ? likely to be
(a) 0.2 A (b) 0.4 A (a) 25 rad/s and 75 rad/s
(c) 2 A (d) 4 A (b) 50 rad/s and 25 rad/s
(c) 46 rad/s and 54 rad/s
42. Three resistors having resistances r1, r2 and r3 are
(d) 42 rad/s and 58 rad/s
connected as shown in the given circuit. The ratio i3 / i1
of currents in terms of resistances used in the circuit is 47. A uniform conducting wire of length 12a and resistance
r2
R is wound up as a current (I) carrying coil in the shape
i2
of
i1 r1 1. an equilateral triangle of side a.
A B 2. a square of side a.
The magnetic dipole moments of the coil in each case
i3 r3 respectively are
r1 r2 (a) 3 Ia2 and 3 Ia2
(a) (b)
r2 + r3 r2 + r3 (b) 3 Ia2 and Ia2
r r (c) 3 Ia2 and 4 Ia2
(c) 1 (d) 2
r1 + r2 r1 + r3 (d) 4 Ia2 and 3 Ia2
6 NEET Solved Paper 2021

48. From a circular ring of mass M and radius R, an arc 50. A particle moving in a circle of radius R with a uniform
corresponding to a 90° sector is removed. The moment speed takes a time T to complete one revolution.
of inertia of the remaining part of the ring about an If this particle were projected with the same speed at
axis passing through the centre of the ring and an angle θ to the horizontal, the maximum height
perpendicular to the plane of the ring is K times MR 2 . attained by it equals 4R. The angle of projection θ is
Then, the value of K is then given by
3 7 1/ 2
(a) (b)  gT 2 
4 8 (a) θ = cos − 1  2 
1 1  π R
(c) (d)
4 8 1/ 2
 π2R 
(b) θ = cos − 1  2 
49. Two conducting circular loops of radii R1 and R2 are  gT 
placed in the same plane with their centres coinciding.
1/ 2
If R1 >> R2 , the mutual inductance M between them  π2R 
(c) θ = sin− 1  2 
will be directly proportional to  gT 
R1 R2
(a) (b) 1/ 2
R2 R1  2gT 2 
(d) θ = sin− 1  2 
R12 R 22  π R
(c) (d)
R2 R1

Answers
1. (d) 2. (a) 3. (b) 4. (a) 5. (c) 6. (b) 7. (c) 8. (a) 9. (c) 10. (d)
11. (d) 12. (b) 13. (b) 14. (c) 15. (b) 16. (d) 17. (b) 18. (c) 19. (b) 20. (d)
21. (d) 22. (b) 23. (d) 24. (b) 25. (d) 26. (b) 27. (d) 28. (a) 29. (a) 30. (c)
31. (c) 32. (c) 33. (d) 34. (a) 35. (b) 36. (d) 37. (c) 38. (b) 39. (d) 40. (d)
41. (a) 42. (b) 43. (b) 44. (d) 45. (d) 46. (c) 47. (a) 48. (a) 49. (d) 50. (d)
Answer with Explanations
1. (d) The given circuit diagram as shown below 3. (b) We know that, the direction of electric dipole moment is
from negative charge to positive charge.

L C R

40 V 10 V 40 V • •
p
+q –q E
E1 E2
V
Given,
VL = 40 V, VC = 10 V The strength of the electric field at + q is greater than that of - q
and VR = 40 V charge, i.e.| E1 | > | E 2 |.
The amplitude of the current flowing in the L-C-R series circuit, So, the net force on the dipole act towards the right side.
We know that, a system always moves to decrease in its
I 0 = 10 2 A …(i) potential energy.
We know that, rms current in the L-C-R series circuit, So, the given dipole move towards right side as its potential
energy will decreases.
I
Irms = 0
2 4. (a) Given, AC source voltage,
10 2 V = V0 sin wt …(i)
Þ Irms = [from Eq. (i)]
2 We know that,
Þ Irms = 10 A Q = CV …(ii)
\ Vrms = VR2 + (VL - VC) 2 Here, Q is the charge on the capacitor,
C is the capacitance of the capacitor,
Þ Vrms = (40)2 + (40 - 10)2
V is the AC source voltage.
Þ Vrms = 50 V On differentiate Eq. (ii) w.r.t. time, we get
The impedance of the L-C-R series circuit, dQ d(CV)
=
V 50 V dt dt
Z = rms Þ Z = or Z = 5W
Irms 10 A dQ Cd(V0 sin wt)
Þ = [from Eq. (i)]
dt dt
2. (a) Given, the refractive index of the glass, n2 = 3
dQ
The refractive index of the air, n1 = 1 Þ = CwV0 cos wt
dt
The path of ray incident on the prism is as shown below As we know, the displacement current,
dQ
Id = Þ I d = V0 w C cos wt
30º dt
r1
r2 5. (c) Magnetic field inside the conducting cylindrical cable,
e m0 I
B= r
60º 2pR2
Here, R is the radius of the cylindrical cable,
From the given figure, r1 = 0°
r is the distance from the axis of the cylinder,
As, A = r1 + r2
I is the current carrying in the cylindrical cable.
where, A is the angle of prism .
Þ Bµr
30° = 0° + r2 Þ r2 = 30°
\ The graph of magnetic field B with r is a straight line passing
Using the Snell's law, in glass and air interference, through origin.
n2 sin q2 = n1 sin q1 For a point outside the cylinder,
Þ n2 sin r2 = n1 sin e m I 1
B= 0 Þ Bµ
3 2pR R
Þ 3 sin 30° = 1 ´ sin e Þ sin e =
2 The graph of magnetic field (B) with r is a rectangular hyperbola
Þ sin e = sin 60° Þ e = 60° passing through the outer surface of the cylinder.
8 NEET Solved Paper 2021

\ The variation of the magnetic field B due to the cable with the Substituting in the Eq. (i), we get
distance r from the axis of the cylindrical cable is as shown in h
the figure. ld =
hc
2m æç ö÷
èlø
B Squaring both sides of the above equation, we get
h2 2mc ö
l2d = Þ l = æç 2
÷ ´ ld
hc è h ø
r=R r 2m æç ö÷
èlø
6. (b) Given, the focal length of convex lens A,
f1 = + 20 cm 8. (a) As we know that, the expression of the drift velocity,
The focal length of concave lens B, eE
vd = t
m
f2 = - 5 cm
Here, e is the electric charge,
The convex lens is the converging lens and the concave lens is
the diverging lens. E is the electric field,
d m is the mass of an electron,
t is the relaxation time.
Consider the conductor having length l, area of the
F cross-section A and the charge density n.
5 cm m
Electrical resistivity, r = 2
A ne t
20 cm B
Rearranging the above expression,
When the parallel beam of light passes through the convex m
lens, it converges at the focal point F and the distance between Relaxation period, t= 2
ne r
lens A to F point is 20 cm.
As we know that, the expression of current density,
Now, after emerging from the lens B, the beam is still parallel in
nature. So, the net focal length of the lens A and B will be I neAvd
J= = = nevd
infinite. A A
As we know, E E 1
Again, = = =r
1 1 1 d J sE s
= + - E
f f1 f f1 f2 \ Electrical resistivity, r =
J
1 1 1 d
Þ = - + The correct match is A® 3, B® 4, C® 1, D® 2.
¥ 20 5 20 ´ 5
3 d 9. (c) As we know that,
Þ =
20 20 ´ 5 In a-decay, the atomic number is decreased by 2 units.
or d = 3 ´ 5 = 15 cm X ZA ¾® A-4
+ 42He
Z - 2Y
7. (c) We know that, de-Broglie wavelength, In b + -decay, the atomic number is decreased by 1 unit.
h
ld = X ZA ¾® A 0
mv Z - 1Y + +1 e + n

Kinetic energy of the photoelectrons, In b - -decay, the atomic number is increased by 1 unit.
1 2 ´ KE 0
KE = mv2 Þ v = X ZA ¾® A
Z + 1Y + -1 e + n
2 m
From the above relations, the spontaneous decay given in
Here, m is the mass of photoelectrons. question can be written as
Substituting the value of v in de-Broglie
b
+
a b-
wavelength equation, we get X ZA ¾® Z - 1B ¾® Z - 3C ¾® Z - 2 D
h
ld = …. (i) The possible decay particles in the sequence are b + , a, b - .
2mKE
Since, work-function of photosensitive surface is negligible, 10. (d) The four resistances have equal length, equal
hence the kinetic energy of photoelectron is equal to the energy cross-sectional area and same material.
of the photon of electromagnetic wave, So, R1 = R2 = R3 = R4 = R (say)
1 hc
KE = mv2 = Given, the effective resistance,
2 l
Req = 0.25 W
NEET Solved Paper 2021 9

For parallel arrangement, The temperature of the surrounding, T0 = 20°C


1 1 1 1 1 Using the Newton's law of cooling,
= + + +
Req R1 R2 R3 R4 dT é T + Tf ù
rate of cooling = =K ê i - T0 ú
1 1 1 1 1 dt ë 2 û
Þ = + + +
Req R R R R
Substituting the values in the above equation, we get
1 4 90 - 80 90 + 80
Þ = Þ R =1W = K éê - 20ùú
0.25W R t ë 2 û
For series combination, 10 2
Þ = K [65] Þ K =
Req = R1 + R2 + R3 + R4 t 13t
Þ Req = R + R + R + R In second conditions;
Þ Req = 4 R Þ Req = 41
() = 4W The initial temperature of the cup of coffee,Ti¢ = 80° C
The final temperature of the cup of coffee, Tf ¢ = 60° C.
11. (d) Let the particle at height S from the surface
of the Earth is as shown. Using the Newton's law of cooling,
Given, at height x, kinetic energy dT é T ¢ + Tf¢ ù
rate of cooling = =K ê i - T0 ú
= 3 (potential energy) S dt ë 2 û
Þ KE = 3mgx x Substituting the values in the above equation, we get
At height S, total energy, TE = PE + KE 80 - 60 2 é 60 + 80
= - 20ùú
TE = mgS+ 0= mgS t1 13t êë 2 û
At height x, total energy, 20 2 13
= [50] Þ t1 = t
TE = PE + KE t1 13t 5
TE = mgx + 3 mgx
14. (c) Given, the monochromatic light of the wavelength,
mgS = 4 mgx l = 600 nm
S The power of the source,
Þ x= …(i)
4
P = 3.3 ´ 10-3 W
Now, we shall determine the speed of the particle at this height.
nhc
As, KE = 3 ´ mgx We know that, P =
l
S
KE = 3 ´ mg [from Eq. (i)] Here, P is the power of the source,
4
n is the number of photons per second,
1 3
mv2 = mgS h is the Planck’s constant,
2 4
3 c is the speed of the light in vacuum,
Þ v= gS
2 l is the wavelength of the monochromatic light.
Substituting the values in the above equation, we get
12. (b) Given, the half-life of a radioactive nuclide, t1 / 2 = 100 h
n ´ 6.6 ´ 10-34 ´ 3 ´ 108
3.3 ´ 10-3 =
As we know, 600 ´ 10-9
A0
A= t / t1 / 2 Þ n = 1016
2
Here, A0 is the original activity of the nuclide, 15. (b) In simple harmonic motion, both kinetic energy and
potential energy attains their maximum value two times in
A is the activity of the nuclide after time t.
one complete oscillation. Hence, frequency of kinetic energy
150
A t/ t
- 1 and potential energy is 2 for one complete oscillation. So, the
Þ = 2 1 / 2 = 2 100 = 2-3/ 2 =
A0 2 2 frequency of the potential energy of a body executing SHM
with frequency n is 2n.
So, the fraction of original activity that will remain after 150 h
1 16. (d) Given, the current carrying in infinitely long straight
will be . conductor, I = 5A
2 2
The electron is moving with speed, v = 105 m/s
13. (b) In first conditions;
Given, the initial temperature of the cup of coffee ,Ti = 90° C The perpendicular distance between the electron and the
conductor, r = 20 cm = 0.20 m
The final temperature of the cup of coffee, Tf = 80° C
Now, the magnetic field experience by electron due to infinitely
The time taken to drop the temperature 90°C to 80°C is t.
long straight conductor,
10 NEET Solved Paper 2021

m0 I 4p ´ 10-7 ´ 5 19. (b) Distance covered nth seconds is s n .


B= Þ B=
2pr 2p (0.20) Distance covers in (n + 1) th seconds is s n + 1 .
Þ B = 5 ´ 10-6 T Initial velocity of small block, u = 0
The magnetic force experience by an electron, Distance cover in nth seconds,
F = evB a
sn = u + (2n - 1)
Here, e is the charge of an electron, 2
v is the velocity of an electron, a
B is the magnetic field experience by an electron. Þ s n = 0 + (2n - 1)
2
Substituting the values in the above equation, we get a
Þ s n = (2n - 1) …(i)
F = 1.6 ´ 10-19 ´ 105 ´ 5 ´ 10-6 2
F = 8 ´ 10-20 N Distance cover in (n + 1)th seconds,
a
17. (b) Given, fundamental physical quantities are force [F], s n + 1 = u + [2(n + 1) - 1]
2
acceleration [a] and time [T].
a
Now, we shall determine the dimensions of the energy. Þ s n + 1 = 0 + (2n + 2 - 1)
2
Energy depends on force, acceleration and time as, a
Þ s n + 1 = (2n + 1) …(ii)
[E] = [F]a[a]b[T]c 2
Þ [ML2T–2] =[MLT–2]a[LT-2]b[T]c On dividing Eq. (i) by Eq. (ii), we get
a
Þ [ML2T-2] = [M]a[L]a + b [T]-2a - 2b + c (2n - 1)
sn
= 2
Comparing the powers of M, L and T on both sides, we get sn + 1 a
(2n + 1)
a = 1, a + b = 2 and -2a - 2b + c = - 2 2
sn (2n - 1)
Þ 1+b=2 Þ =
s n + 1 (2n + 1)
Þ b = 1,
Þ -21
() - 21
() + c = - 2 Þ c = 2 20. (d) A nucleus with mass number 240 breaks into two
The dimensions of the energy are [F1 ][a]1 [T]2. fragment each of mass number 120.
X 240 ¾® Y 120 + Z120
18. (c) We know that, the rms speed of the gas molecules,
Given, the binding energy per nucleon of unfragmented nuclei,
vrms = 3RT / M
X = 7.6 MeV
Here, T is the temperature of the gas, The binding energy per nucleon of fragmented nuclei, Y = Z
R is the universal gas constant, = 8.5 MeV
M is the molar mass of the gas. Now, we shall determine the total gain in binding energy.
Pressure exerted by an ideal gas, Gain in binding energy = Binding energy of products –
1 Binding energy of reactants
p = nmv 2
3 Gain in binding energy = 2 ´ 120 ´ 8.5 - 240 ´ 7.6 = 216 MeV
Here, n is the number of moles, 21. (d) Given, the main scale reading, MSR = 0
m is the mass of the gas, The circular scale reading, CSR = 52 divisions
v is the average speed of the gas molecules. Now, we shall determine the least count of the screw gauge,
The average kinetic energy of a molecule, p
LC =
3 n
KE av = kB T
2 Here, p is the pitch of the screw,
Here, kB is the Boltzmann constant, n is the number of circular divisions in one complete revolution.
T is the temperature of the gas. 1
LC = mm
For diatomic gas, degree of freedom, f = 5 100
As, total internal energy of 1 mole of diatomic gas, Þ LC = 0.01 mm Þ LC = 0.001 cm
n f RT
DU = Thus, the least count of the screw gauge is 0.001 cm.
2
Therefore, diameter of the wire of screw gauge,
1 (5)RT 5RT
Þ DU = Þ DU = D = MSR + (CSR ´ LC)
2 2
Þ D = 0 + (52 ´ 0.001)
The correct match is A ® 2, B ® 1, C ® 4 and D ® 3.
Þ D = 0.052 cm
NEET Solved Paper 2021 11

22. (b) Consider the nodes be A, B, E and F as shown in the s1 q1 R22


Þ =
figure s2 q 2 R12
C s1 R1 R22
F Þ = [from Eq. (i)]
s2 R2 R12
s1 R2
C Þ =
A B s2 R1

E 25. (d) Given, the mass of suspended, m = 2 kg


C
The spring is stretched, x = 5 cm = 0.05 m
The potential at node A is equal to the potential at node B. The constant force applied on the spring, F = 10 N
Similarly, the potential at F is equal to the potential at node E. As we know that, spring force,
So, there is no potential difference between the EF arms, thus, F = kx Þ 10 N = k (0.05 m)
no current (hence, charge) will flow in the circuit.
Þ k= 200 N/m
So, EF behave as open circuit.
Now, time period of the oscillation,
C
m 2
T = 2p Þ T = 2p
k 200
Time period, T = 0.628 s
A B
26. (b) We know that, in electromagnetic wave, the electric field
(E) and magnetic field (B) are perpendicular to each other,
C
E× B = 0
Now, the two capacitors are arranged in parallel. Consider the option (a);
The net equivalent capacitors, $ ) × ($j + k
($j + k $) = 1 + 1 = 2 ¹ 0
Ceq = C1 + C2
So, it is incorrect option.
Þ Ceq = C + C
Consider the option (b);
Þ Ceq = 2C $ ) × (- $j - k
$) = 1 - 1 = 0
(- $j + k
23. (d) The objective of an astronomical telescope has large focal Hence, it satisfies the condition E × B = 0
length and large aperture. Because the lens used is convex
lens and the magnification of an astronomical telescope is Similarly, options (c) and (d) are incorrect.
directly proportional to the focal length of the objective lens. So, the correct option is (b).
This large aperture also contributes to the better quality and
27. (d) We know that,
visibility of the images. Thus, it provides better resolution.
2GM e
Due to large focal length of the objective, it covers large area escape velocity from Earth’s surface, ve =
which ensures better light gathering power. Re
where, G is the gravitational constant,
Thus, all options (a), (b) and (c) are correct. Re is the radius of the Earth,
M e is the mass of the Earth.
24. (b) When two charged spherical conductors are connected by mass
a conducting wire, then the potential becomes same in both Also, density =
volume
the spherical conductor, i.e.
V1 = V2 Þ Mass =Density ´ Volume
or Me = r ´ V
Kq1 Kq 2 q R
Þ = Þ 1 = 1 …(i) 4 4 3ö
R1 R2 q 2 R2 or M e = r ´ pRæçQ3
e V = pRe ÷
3 è 3 ø
As we know that, the surface charge density of the charged Substituting the values in the escape velocity expression, we
spherical conductor, get
q
s= 4
4pR2 2Gr ´ pRe3
ve = 3
where, q is the charge on the spherical conductor, Re
r is the radius of the spherical conductor. 4
q1 ve = pRe2
2Gr ´ …(i)
3
s1 4pR12 Now, we shall determine the escape velocity for another planet.
\ =
s2 q2
Given, radius of another planet is 4 times the radius of the
4pR22 Earth,
12 NEET Solved Paper 2021

4 2 1 æ e0 A ö 2
Þ vp = 2Gr ´ pRp U= ç ÷ (Ed) [Q from Eqs. (i) and (ii)]
3 2è d ø
1 1
4 U = (e0 A)(E)2 d = e0 E 2 Ad
vp = 2Gr ´ p(4Re)2 (Q Rp = 4Re) 2 2
3
4
31. (c) Given, the electron concentration in n- type
vp = 4 2Gr ´ p(Re)2 …(ii) semiconductor is equal to the hole's concentration in p-type
3 semiconductor.
On dividing Eq. (ii) by Eq. (i), we get When electric field is applied across the semiconductor, the
4 electrons and holes will move opposite to each other. The
4 2Gr ´ p(Re)2 mobility of n-type semiconductor is greater than mobility of
vp 3
= p-type semiconductor.
ve 4
2Gr ´ pR2e So, the current in the n-type semiconductor is greater than
3 current in the p-type semiconductor.
Þ vp = 4ve = 4v
32. (c) Zener diode is a type of special p-n junction diode that
28. (a) Given, length of balancing point, L1 = 36 cm operates in reverse bias. Thus, it is used as a voltage regulator.
So, the statement (A) is correct.
E1 =1.5V The potential barrier of p-n junction for silicon is 0.6-0.7 V and
E 2 = 2.5V for germanium is 0.2-0.35 V.
We know that, So, the statement (B) is incorrect.
E
Potential gradient = 33. (d) In polar molecules, the positive charge and negative
L charge do not overlap each other. So, the total charge on the
E1 E 2 polar molecule is zero. Thus, these types of molecules have
Q = permanent electric dipole moment. e.g. Water is polar
L1 L2
molecule.
where, L2 is the length of the balancing point for cell of emf E 2.
34. (a) The dimensions of energy [E] = [F]× [d]
Substituting the values in the above equation, we get
1.5 2.5 Þ [E] = [MLT-2] [L] Þ [E] = [ML2T-2]
= Þ L2 = 60 cm
36 L2 As we know that, the expression of gravitational force,

29. (a) Given, the density of the small ball is d GM1 M 2 Fr 2


F= Þ G=
The mass of the small ball is M r2 M1 M 2
The density of the glycerine is d/2. [F] [r 2] [MLT-2][L]2
\ [G] = Þ [G] =
As we know that, [M1 ] [M 2] [M] [M]
viscous force = weight – buoyant force
Viscous force = Vd1 g - Vd2 g Þ [G] = [M -1L3T-2]
Here, V is the volume of submerged bodies, The dimensions of
g is the acceleration due to gravity, E [ML2T-2] E
d1 is the density of the small ball, = Þ éê ùú = [M 2L-1 T0 ]
G [M -1L3T-2] ë Gû
d2 is the density of the glycerine,
d1 = d and d2 = d / 2 35. (b) Given, the flow rate of the water, m =15 kg/s
t
Substituting the given values in the viscous force expression, The height of the water fall, h = 60 m
we get Loss due to frictional force = 10%
d The power used in the turbine
Viscous force = Vdg - V g
2 = (100 – 10) % = 90%
Vdg Mg
Viscous force = = (Q M = d ´ V) The acceleration due to gravity, g =10 m/s 2
2 2
We know that, power generated by the turbine
30. (c) We know that, capacitance of a parallel plate capacitor, mgh
= change in potential energy = 0.90
Ae0 t
C= …(i)
d = 0.90 × 15 × 10 × 60 = 8100 W = 81
. kW
The relation between the potential difference (V) and electric
36. (d) Given, the initial velocity of a car, u = 0
field (E),
The acceleration of a car, a = 5 m/s2
V = Ed …(ii)
The energy stored in the capacitor, At t = 4 s, v = u + at
1 Þ v = 0 + (5)4 Þ v = 20 m/s
U = CV 2
2 Thus, the final velocity of car at t = 4 s is 20 m/s.
NEET Solved Paper 2021 13

At t = 4 s, the ball is dropped out of a window by a person Þ v1 = - 210


( )(10)
sitting in the car.
Þ v1 = - 10 2 m/s
The velocity of the ball in the x-direction,
vx = 20 m/s (due to the car) For case (2), ball rebounds to the same height.

Therefore, in the y-direction, the acceleration is equal to the The velocity with which the ball just reaches to the same
acceleration due to gravity, height,

a y = g = 10 m/s2 v2 = 2gh
Þ v2 = 210
( )(10)
The velocity of the ball in the y-direction,
vy = u + a yt Þ v y = 0 + 10 ´ 2 Þ v2 = 10 2 m/s

Þ v y = 20 m/s Now, magnitude of the impulse imparted to the ball,


Thus, the velocity of the ball in y-direction is 20 m/s. |Impulse|= m | Dv |
The net velocity at t = 6s, = 0.15 |10 2 - (-10 2) |

v= vx2 + v2y Þ v = (20)2 + (20)2 = 0.15|2 ´ 10 2|


= 4.2 kg - m/s
Þ v = 20 2 m/s
Thus, the velocity of the ball at t = 6 s is 20 2 m/s. 39. (d) Given, the length of a uniform rod,
L = 200 cm
and there is no acceleration in the x-direction, a x = 0 ms-2
The wedge is placed at the mark = 40 cm
In y-direction, a y = 10 ms-2 The balanced mass placed at 100 cm mark on wedge = 500 g =
Now, we shall determine the net acceleration 0.5 kg
at t = 6 s, The mass suspended from the rod at 20 cm distance from the
end, M = 2 kg
a= a x2 + a 2y Þ a = (0) + (10)2 Þ a = 10 ms-2
Another unknown mass suspended from the rod at 160 cm
37. (c) Output of the given logic circuit is given as distance from the end = m
Let's draw the diagram of the uniform rod suspended with
Y = AB + BC
mass.
Truth table for the given input waveforms A, B and C.
160 cm
20 cm 20 cm 60 cm 80 cm
Time interval A B C AB BC AB BC Y = AB + BC
0 - t1 0 0 1 0 0 1 1 1 O
t1 - t 2 1 0 1 0 0 1 1 1 40 cm
t2 -t3 0 1 0 0 0 1 1 1
2 kg 0.5 g mg
t3 -t4 1 1 0 1 0 0 1 1
t4 -t5 0 0 1 0 0 1 1 1 As we know in equilibrium net moment of force is equals to
zero.
t5 -t6 1 0 1 0 0 1 1 1
Þ 0.5 g (0.60) + mg (1.20) - 2g (0.20) = 0
Hence, output waveform will be 5V. Þ 0.3+1.20 m - 0.4 = 0
Thus, option (c) is correct. 1
Þ 1.20 m = 0.1 Þ m = kg
38. (b) Given, the mass of the ball dropped from the height, m = 12
0.15 kg 40. (d) Given, the focal length of the convex lens, f =+30 cm
The height from the ball dropped, h = 10 m The object distance from the lens, u = - 60 cm
We know that, The image formed by the lens be at distance v, then by using
|Impulse | = m | Dv | lens formula,
where, Dv = v2 - v1 1 1 1
= -
Here, v2 is velocity reaches to the same height, f v u
v1 is velocity just before striking to the ground. 1 1 1
= -
For case (1), ball dropped from the 10m height and strikes to +30 v (-60)
the ground. 1 60 + v
Þ =
Now, the velocity of the ball just before striking to the ground is +30 60 v
v1 = - 2gh Þ 2v = 60 + v Þ v = 60 cm
14 NEET Solved Paper 2021

Let's draw the ray diagram of the convex lens and mirror. æ $i $j k$ ö
ç ÷
$ $ $
4 i - 20 j + 12k = ç 2 4 6÷
çç ÷
èB B B0 ÷ø
O1 I2 I1 $ = $i(4B - 6B) - $j(2B - 6B) + k
$ (2B - 4B)
Þ 4$i - 20$j + 12k 0 0

Comparing the LHS and RHS of the above equation, we get


60 cm 40 cm 20 cm
$i terms:
This real image will now act as a virtual object for the mirror.
4B0 - 6B = 4 …(i)
Thus, a real image from the mirror is formed 20 cm in front of $j terms:
the mirror. Hence, at 20 cm distance from the lens. Also, this
real image acts as an object for lens for final image. 2B0 - 6B = 20 …(ii)
Again by using lens formula, $ terms:
k
1 1 1 2B - 4B = 12
+ =
v 20 30
Þ B= -6
or v = -60 cm from the lens
Substituting the value of B in the Eq. (ii), we get
i.e., 20 cm from the plane mirror and virtual in nature.
2B0 - 6 (-6) = 20 B0 = - 8
Þ
41. (a) Given, the main supply line voltage in transformer, $
Thus, the magnetic field vector, B = - 6 i - 6$j - 8k
$
V = 220 V
The rating of the lamp = 11 V, 44 W 44. (d) Given, the mass of the particle = m
We know that, The velocity of the projected particle, v = kve
P = VI P …(i) Initial energy at the time of projection
1 GMm
Here, P is the power that operates the lamp, = mv2 -
2 R
V is the supply line voltage,
Final energy at the maximum height h from surface of the
I P is the primary current in the transformer. 1 GMm
Earth = mv2 -
From Eq. (i), we get 2 R+ h
Þ 44 = I P ´ 220 1 GMm
= m(0)2 -
Þ I P = 0.2 A 2 R+ h
(Q final velocity is zero at height h from the surface
42. (b) According to the given circuit diagram in question, of the Earth)
i1 = i 2 + i 3 …(i) GMm
=-
In parallel arrangement of the electrical circuit, R+ h
the voltage remains same.
Using the law of conservation of energy,
\ V = ir (using Ohm’s law)
initial energy at the time of projection = final energy at the
Þ i 2r2 = i 3r3 maximum height h from the surface of the Earth
ir 1 GMm GMm
Þ i2 = 3 3 mv2 - =-
r2 2 R R+ h
Substituting the values in the Eq. (i), we get Substituting the values in the above equation, we get
ir i r 1 GMm GMm
i1 = 3 3 + i 3 Þ 1 = 3 + 1 m(kve)2 - =-
r2 i 3 r2 2 R R+ h
i1 r3 + r2 i3 r2 1 æ 2GM ö
Þ = Þ = mk2 æ 2 GM ö - GMm = - GMm
i3 r2 i1 r3 + r2 Þ ç ÷ çQ ve = ÷
2 è R ø R R+ h è R ø
43. (b) Given, velocity, v = 2$i + 4$j + 6k$ k2 1 1
Þ - =-
$
Force, F = 4$i - 20$j + 12k R R R+ h

As we know, 1 1 k2
Þ = -
R+ h R R
F = q (v ´ B)
Here, q = 1 (given) R
Þ =R+ h
1 - k2
Substituting the values in the above equation, we get
$ = ()[( $ ) ´ (B$i + B$j + B k
$ )] R
4$i - 20$j + 12k 1 2$i + 4$j + 6k 0 Þ h= -R
1 - k2
NEET Solved Paper 2021 15

æ 1 ö Using the Eq. (i), we get


Þ h= Rç 2
- 1÷
è1 - k ø w = w r ± Dw
æ1 - 1 + k2 ö w = (50 ± 4) rad/s
Þ h= Rç ÷
è 1-k ø
2 So, the angular frequency is likely to be 46 rad/s
and 54 rad/s.
æ k2 ö
Þ h= Rç 2
÷ 47. (a) Given, the length of a uniform conducting wire, L = 12a
è1 - k ø
The resistance of a uniform conducting wire = R
45. (d) Given, the number of small drops, n = 27 We know that, magnetic dipole moment, M = nIA
Potential on each charged drop, V = 220 V Here, I is the current in the loop,
Consider r be the radius of the small drops and R be the radius A is the area of the coil,
of the bigger drop.
n is the number of loops.
According to the question, 27 small drops of same size combine
to form a bigger drop. So, the volume remains same, i.e. For case (1):

Vbigger = 27 ´ Vsmall An equilateral triangle of side a.


4 4 Total wire length = n ´ Perimeter of the triangle
Þ pR3 = 27 ´ pr 3
3 3 12a = n ´ 3a Þ n = 4
3 3
Þ R = 27 ´ r Þ R = 3r
Now, the potential of the small drop, a
Kq
V= …(i)
r
n=4
Here, q is the charge on the small drop.
The potential of the bigger drop, 3 2
Area of the triangle, A = a
KQ 4
V¢ =
R Magnetic dipole moment, M = nIA
Here, Q is the charge on the bigger drop. 3 2
Þ M = 4I ´ a
The charge on the bigger drop is 27 times the charge on the 4
smaller drop, i.e. Q = 27q
Þ M= 3Ia 2
So, the potential of the bigger drop,
For case (2) :
K ´ 27q
V¢ = A square of side a.
3r
Total wire length = n ´ Perimeter of the square
Kq
Þ V¢ = 9 12a = n ´ 4a Þ n = 3
r
V ¢ = 9V [from Eq. (i)]
= 9 ´ 220 = 1980 V a

46. (c) Given, in the L-C-R series circuit, L = 5H ,


n=3
C = 80 mF, R = 40 W
Supply voltage, V = 230 V Area of the square loop, A = a 2
1 Magnetic dipole moment, M = nIA Þ M = 3Ia 2
We know that, resonance frequency, wr =
LC
48. (a) Given, the mass of the circular ring = M
Substituting the values in the above equation, we get
The radius of the circular ring = R
1
wr = = 50 rad/s We know that, the moment of inertia of the circular ring,
5 ´ 80 ´ 10-6
I = MR2
Therefore, the angular frequencies of the source at which An arc corresponding to a 90° sector in the circular ring is
power transferred to the circuit is half the power at the removed, means one-fourth part of circular ring is removed.
resonant angular frequency are Then, the remaining mass of the circular ring,
w = w r ± Dw …(i) M
M¢ =
We know that, 4
R 40 MR2
Dw = Þ Dw = Þ Dw = 4 The new moment of inertia, I ¢ = M ¢ R2 =
2L 2(5) 4
16 NEET Solved Paper 2021

Now, the moment of inertia of the remaining part, Thus, the mutual inductance is directly proportional to the
MR 2 R22
I ¢ ¢ = I - I ¢ = MR2 - .
4 R1
3MR2 50. (d) Given, the radius of the circular path = R
Þ I¢ ¢ =
4 The time taken by the particle to complete one revolution = T
Comparing with I ¢ ¢ = KMR2 When the particle is projected with the same speed (by which it
The value of K = 3/ 4 × is moving in circular orbit) at angle q to the horizontal, the
maximum height attained it is given as
49. (d) Since, both conducting circular loops are in same plane
u2 sin2 q
and their centres are coinciding. Hmax = …(i)
2g
Hence, magnetic flux,f = Mi …(i)
Here, M is the mutual inductance. Hmax = 4R (given)
Also, the magnetic flux, f = BA Also, we know that,
Here, B is the magnetic field and 2pR
speed of the particle in circular path, u =
T
A (= pR22) is the area of the inner circular loop.
Substituting the values in the Eq. (i), we get
If the current passing through the outer loop is i, then the 2
magnetic field, æ 2pR ö sin2 q
ç ÷
è ø
m i 4R = T
B= 0 2g
2R1
1/ 2
m0 i æ 2gT 2 ö
Magnetic flux, f = BA Þ f = pR22 …(ii) Þ sin q = ç 2 ÷
2R1 èp Rø
Comparing Eqs. (i) and (ii), we get 1/ 2
æ 2gT 2 ö
m Þ q = sin-1 ç 2 ÷
M = 0 pR22 èp Rø
2R1
EXAM BITES

This Pdf Is
Downloaded From
www.exambites.in

Visit www.exambites.in for


More Premium Stuffs,Latest
Books,Test Papers,Lectures etc.
jeeneetadda
jeeneetadda_official
jeeneetadda

VISIT NOW !!

You might also like